You are on page 1of 480

SK Origional – Golden 11 1

SK – Pearls
ORIGINAL

Golden – 11
100% Errata Free
By
Dr. Atif Afzal
&

500 Important Points

By: Dr. Salahuddin Kamal


SK Origional – Golden 11 2

If you want to buy hard copy of this book


SK Origional – Golden 11 3

If you want to buy hard copy of this book

with 25% Discount


Call/Whatsapp: 0321-2066562
SK Origional – Golden 11 4

Table of Contents
S/No. SUBJECTS Page No.

SURGERY & ALLIED


1 October, 1st – Morning 06
2 October, 1st – Afternoon 22
3 November 25th – Morning 37
4 November 25th – Afternoon 51
5 November 25th – Night 66

MEDICINE & ALLIED


6 September, 29th – Afternoon 83
7 September, 29th – Night 92
8 September, 30th – Afternoon 108
9 September, 30th – Night 126
10 October, 1st – Afternoon 143
11 October 1st – Night 156
12 November, 24th – Afternoon 170
13 November, 24th – Night 182
14 November, 25th – Afternoon 197
15 November, 25th – Night 202
16 November 26th - Morning 215
17 November 26th – Afternoon 228

GYNAECOLOGY
18 September, 30th – Morning 242
19 September, 30th – Afternoon 258
20 November, 25th – Morning 274
21 November, 25th – Afternoon 287

RADIOLOGY
22 September, 29th – Morning 307
23 September, 29th – Afternoon 316
24 November, 24th – Morning 330
25 November, 24th – Afternoon 342

ANAESTHESIA
26 September, 29th – Morning 357
27 November, 24th – Morning 370

28 500 – SK Golden Points 383


SK Origional – Golden 11 5

Dedication
This boos is dedicated to the efforts of all
people who are involved in
Khatm e Nabuwat movement.

Khatm e Nabuwat Zindabad


SK Origional – Golden 11 6

Special Thanks To

Justice Ali Nawaz Chohan (Rtd. Chief Justice)


Mr. Bashart Mahmood Shahzad (Additional Director General FIA)
Mr. Naimatullah Abid (Rtd. Director General NIM)
Mr. Suhail Khalid (District Police Officer - Buner)
Lt. Col. Majid.
SK Origional – Golden 11 7

Helping Stars
Dr. Zoshia Ali Zaidi
Dr. Choudhary Abrar
Dr. Aniqa Rahman
Dr. Saher Rafique

Dr. Saima Khan

Dr. Anum Amjad Qureshi

Dr. Noor ul Ain Lali

Dr. Gulalai Burki

Dr. Zonehra Majid

Dr. Noor-e-Hira Imtiaz

Dr. Salar Ahmad

Dr. Sara Aslam

Dr. Yousra Naz Rajput

Dr. Ahsan Ali Bohio

Dr. Ali Maqbool

Dr. Aeman Shafique

Dr. Rubab Fatima

Dr. Maria Eram

Dr. Kalsoom Rasheed

Dr. Maham Kayani

Dr. Ezatullah Viqar


SK Origional – Golden 11 8

ALL SURGERY
September – November – 2020 Papers
SK Origional – Golden 11 9

SURGERY & ALLIED a. Syphilis


st
October 1 , 2020 – Morning b. AIDS
c. Tb
d. Grauloma Inguinale
1) Lateral Fornix of Vagina is related to:
a. Ureter Ans: D
6) Fenestrations are more numerous &
b. Pudendal Artery
large in:
c. Pudendal Nerve
a. Ora Serrata
d. Broad Ligament
b. Retina
Ans: A
c. Submacular Area
Explanation:
d. Choroid
 Lateral to Upper part: Ureter
Ans: C
 Lateral to Middle Part: Anterior Fibers of
Explanation: The part of choriocappillary that
Levator Ani
Faces the RPE has large fenestration, larger &
 Lateral to Lower Part: Urogenital
more numerous in the submacular Area.
Diaphragm Ref: ADLER‘S Physiology of Eye.
2) Which of the following is more 7) Submucosal Fibrosis (SMF) is:
appropriate regarding Esophagus? a. Premalignant
a. Upper 2/3rd Muscular Layer is Skeletal + b. Benign
Smooth c. Malignant
b. Lower Third has Simple Columnar
d. Dysplasia
Epithelium Ans: A
c. Stratified Squamous Epithelium through Explanation:Submucosal fibrosis is a
out
premalignant condition of oral cavity.
d. Squamous Cell Carcinoma common in 8) Terminal branch of anterior tibial
lower 1/3rd artery:
Ans: C a. Lateral Plantar Artery
Explanation: b. Fibular Artery
 Middle 1/3rd of Esophagus is Mixed c. Dorsalis Pedis Artery
that‘s both Muscular & Smooth Muscle d. Perforating Branch of Fibular Artery
 Esophagus is having Stratified Squamous Ans: C
Epithelium throughout its whole length Explanation: Dorsalis pedis artery is the
 Adenocarcinoma of Esophagus is continuation of anterior tibial artery at the level of
Common in lower 1/3rd of Esophagus. ankle joint.
3) Glans of Penis & Corpora Lymphatic 9) Anterior Interosseus Nerve is the
Drainage: branch of:
a. Para Aortic a. Axillary Nerve
b. Superficial Inguinal b. Musculocutaneous Nerve
c. Deep Inguinal c. Median Nerve
d. Internal Iliac d. Radial Nerve
Ans: C e. Ulnar Nerve
Explanation:Glans of Penis & Corpora is Ans: C
drained into Deep Inguinal Lymphatic Nodes Explanation:
while Skin of Glans Penis is drained into  Anterior Interosseus Nerve is the branch
superficial inguinal lymphatic nodes. of Median Nerve
4) Glans of Penis Lymphatic drainage:  Posterior Interosseus Nerve is the branch
a. Para Aortic of Radial Nerve
b. Superficial Inguinal 10) Headache after LP:
c. Deep Inguinal a. Compression of Dura
d. Internal Iliac b. Compression of Falx Cerebri
Ans: C c. Stretch of Large Venous Sinuses
5) Donovan Bodies are present in: d. Stretch of Falx
SK Origional – Golden 11 10

Ans: B supination of arm at elbow joint due


Explanation:Headache after lumber punctureis to damage to:
mainly due to dural sinuses stretch but if that‘s not a. Upper Trunk of Brachial Plexus (C5-C7)
present in options then go with compression of b. Lower Trunk of Brachial Plexus
falx cerebri followed by compression of tentorium c. Radial Nerve
cerebelli. d. Ulnar Nerve
11) Radial Pulse is palpated between Ans: A
which tendons? Explanation: In upper trunk of brachial. Plexus
a. Between Abductor Pollicis brevis & there is mostly damage to musculocutaneous,
Longus axillary and suprascapular nerve that leads to loss
b. Between Abductor Pollicis Longus & of abduction, flexion & supination of forearm.
Extensor Pollicis Longus 15) Common Organ involved in SLE:
c. Between Adductor longus & Abductor a. Skin
Brevis b. Eyes
d. Between Abductor brevis & Flexor c. Joints
Tendon d. Lung
Ans: B e. Liver
Explanation: Radial Pulse is mainly Ans: C
palpatedpalpated between the tendons of Explanation: The most common site involved
Brachioradialis & Flexor Carpi Radialis, but this SLE is joints followed by kidney.
option wasn‘t in the board exam so the next close 16) Autoradiography uses which
one is the site of snuff box where Radial Artery technique:
can be palpated between the tendons of Abductor a. Fluorescent Dye
Pollicis Longus & Extensor Pollicis Longus. b. Isotopes bound with Antibodies
12) Lateral Palmar Sensation lost & c. Radioactive ions
flexion of Index Finger lost: d. Iron bound material
a. Radial Nerve Ans: B
b. Median Nerve 17) 10th Intercostal Nerve pierces Rectus
c. Ulnar Nerve Sheath at which Level?
d. Axillary Nerve a. Xiphisternum
Ans: B b. Costal Groove
Explanation:Lateral side of Palm sensation is c. Umbilicus
Carried by median nerve along with flexion of d. Pyramidalis
index finger mainly by Median nerve through its Ans: C
supply to FDP & FDS. Explanation: T10 nerve pierces rectus
13) Nerves involved in fracture of sheathumbilical level that‘s initial site for
Humerus: appendicitis pain.
a. Radial, Ulnar, Median 18) Tubular structure with absorptive
b. Axillary, Ulnar, Median surface area & numerous lymphoid
c. Axillary, Radial, Ulnar tissues:
d. Axillary, Radial, Musculocutaneous a. Appendix
Ans: C b. Colon
Explanation: All the above options are very c. Cecum
close but we have to select the best among these d. Rectum
and the most common nerve damaged during Ans: A
Humerus Fracture are Axillary at surgical neck of Explanation: On the board there was no option
humerus, Radial in Spiral groove and Ulnar behind of Ileum prefer that if present, if not then the next
the medial epicondyle of Humerus. closest answer is Appendix among the given
14) A baby after an obstructed delivery options.
presented with inability to abduct the 19) Breast mass with irregular shape
arm, loss of flexion & loss of that‟s non mobile & 3cm in size with
skin involvement will look on gross:
SK Origional – Golden 11 11

a. Metaplasia Ans: A
b. Dysplasia Explanation: Lung compliance is
c. Desmoplasia increasedmainly in old age due to loss of elastic
d. Anaplasia tissue while vessel compliance is decreased.
e. Hperplasia 24) Feedback Control of Renal system is
Ans: C by:
Explanation: The Scenario is related tomalignant a. Glomerulofeedback
condition of breast that‘s why it will look as b. Tubuloglomerular Feedback
anaplastic condition on gross examination. c. PCT
20) 60 Years Old man presented with d. CCT
hematuria & right flank pain with Hb Ans: B
of 18, Normal BUN & Creatinine: 25) The most important mechanism for
a. Renal Cell Carcinoma Glomerulo-Feedback System is:
b. Renal Lipoma a. Increase Peritubular Colloid Pressure
c. Renal Angiolipoma b. Decrease Peritubular Sodium
d. Renal Artery Stenosis Concentration
Ans: A c. Decrease Peritubular Colloid Pressure
21) Which of the following is d. Increase Oncotic pressure
Paraneoplastic Syndrome of Kidney? Ans: B
a. Polycythemia Explanation: Concentration & dilution of urine
b. Carcinoid Syndrome is mainly related to Sodium & Water.
c. PrRP 26) Which of the following muscle is
d. ADH supplied by ansa cervicalis?
Ans: A a. Geniohyoid
Explanation: Renal system mainly b. Omohyoid
secreteserythropoietin that leads to Polycythemia c. Stylohyoid
& is considered as paraneoplastic syndrome. d. Mylohyoid
22) Erythropoietin is released by which Ans: B
Organ? Explanation: The following mentioned muscles
a. Lung are supplied by Ansa Cervicalis ―Sternohyoid,
b. Liver Sternothyroid & Omohyoid‖
c. Bone Ref: R.Snell (Page # 589, Edition: 09)
d. Kidney 27) Hyponatremia with Increased ECF
e. Skin Volume:
Ans: D a. Cardiac Failure
23) A person with 2L of blood loss the b. Excessive Water Intake
blood volume will be replaced back c. Excessive Dehydration
by: d. Burn
a. DCT Ans: A
b. Thick Ascending LOH Explanation: CCF leads to Sodium retention.
c. Thin Ascending LOH That in turn leads to passive water reabsorption
d. CCT that leads to dilutional hyponatremia with increase
Ans: A ECF.
Explanation: DCT is the site for Macula densa 28) A man presented with pitting edema,
& that‘s involved in bringing back the lost volume urinary albumin 2+ & bilateral pleural
of blood into normal by the RAAS feedback effusion:
system. a. Hypoalbuminemia
24) Lung compliance is increased in: b. Increase Hydrostatic Pressure
a. Old Age c. Increase Oncotic Pressure
b. Bronchoconstriction d. Hemorrhage
c. Fibrosis Ans: A
d. Negative Pressure 29) Atrial myxoma located in:
SK Origional – Golden 11 12

a. Left Atrium 35) Rocky Mountain Spotted Fever is


b. Right Atrium caused by:
c. Left Ventricle a. Rickettsia Rickettsii
d. Right Ventricle b. Rickettsia Prowazaki
Ans: A c. Rickettsia Typhus
30) Upper Esophageal Blood Supply: d. Coxiella Burnetii
a. Superior thyroid artery Ans: A
b. Inferior thyroid artery 36) Growth not arrested at G1 phase due
c. Internal thoracic to loss of which gene?
d. Celiac artery a. K-Ras
Ans: B b. P53
Explanation: c. BCR-ABL
 Upper 1/3rd: Inferior thyroid Artery. d. BCL2
 Middle 1/3rd: Thoracic Aorta. Ans: B
 Lower 1/3rd: Left Gastric Artery. Explanation: Mutation in P53 which is tumour
31) Thyroid Gland moves during Suppressor gene leads to unregulated growth
swallowing due to: Ref: Robbins ―Page # 181, Edition 09‖:
a. Prevertebral Fascia 37) Tumor Suppressor gene:
b. Pretracheal Fascia a. P53
c. Deep Cervical Fascia b. K-Ras
d. Superficial Fascia c. BCL-2
Ans: B d. ABL
32) Imaginary line drawn at Upper level of Ans: A
Iliac Crest: 38) Ovarian tumor Marker is raised in
a. L1 which other organ?
b. L2 a. Breast
c. L3 b. Pancreas
d. L4 c. Liver
e. L5 d. Rectum
Ans: D Ans: A
33) Regarding parotid gland which of the Explanation:BRCA gene is involved in both
following is correct: ovarian & breast carcinoma.
a. Duct Pierces Masseter Muscle 39) Muscle that Opens mouth:
b. Duct Opens behind the 2nd lower molar a. Medial Pterygoid
teeth b. Lateral Pterygoid
c. Pure Serous Acini c. Temporalis
d. Derived from Mesoderm d. Masseter
Ans: C Ans: B
Explanation: Parotid gland is purely serous while 40) Muscle attached with
mandibular gland is mixed & sublingual gland is temporomandibular Capsule that
purely mucinous. opens mouth:
34) 60 years old patient presented with a. Medial Pterygoid
meningitis caused by streptococcus b. Lateral Pterygoid
pneumonia gram staining will show: c. Masseter
a. Beaded Appearance d. Buccinators
b. Diplococcus (Lancet Shaped) Ans: B
c. Diplococcus (Kidney Shaped) 41) Patient with chronic headache on one
d. Appearance like bunches of grapes side, on examination temporal artery
Ans: B stiffness diagnosis:
Explanation: Streptococcus pneumonia is gram a. Rheumatoid Arthritis
positive diplococcus that is lancet shaped. b. Giant Cell Arteritis
c. Kawsaki disease
SK Origional – Golden 11 13

d. Takayasu Arteritis 48) Fat Digestion is done by:


Ans: B a. Emulsification, Bile Salt, HCO3 & Lipase
42) Bone Remodeling is done via: b. Emulsification & Bile Salt
a. Osteoblast c. Amylase & Lipase
b. Osteoclast d. Lipase
c. Osteocyte + Osteoblast Ans: A
d. Osteoblast +Osteoclast 49) Calculate alveolar ventilation, where
Ans: D vital capacity is 350 ml, dead space is
43) Diabetic patient presented with 100 ml & rr is 18/min:
morning hyperglycemia management: a. 3.5 L/Min
a. Regular Insulin b. 4.5 L/Min
b. Short Acting Insulin c. 5.5 L/Min
c. Long Acting Insulin d. 6.0 L/Min
d. Regular + Intermediate Insulin Twice Ans: B
Ans: D Explanation:
44) Head of femur blood supply in  Alveolar Ventilation is equal to (VC –
children: Dead space) * RR
a. Retinacular Artery  (350 – 100 ) * 18
b. Circumflex Artery  (250 ml) * 18
c. Obtruator Artery  4500 ml/Min
d. Arcuate Artery  4.5 L/ Min
Ans: C 50) Blood Pressure dropped what will
Explanation: decrease:
A. Head of femur blood supply in children: a. Firing of Hering Sinus nerve
Obtruator artery b. Cardiac Contractility
B. Head of femurblood supply in adults: c. Venous Tone
Retinacular Artery d. TPR
45) Head of Femur Blood supply in Ans: A
Adults: Explanation: On dropping the blood pressure.
a. Retinacular Artery There will be decrease firing of baroreceptor that
b. Circumflex Artery will lead to increase sympathetic response &
c. Obtruator Artery decrease parasympathetic response.
d. Arcuate Artery 51) Trypsin Act on which Amino Acid:
Ans: A a. Carboxyl terminal Amino Acids
46) Which of the following is activated in b. Lysine
Lung? c. Tryptophan
a. AG1 d. Tyrosine
b. AG2 Ans: B
c. Bradykinin Explanation: Trypsin mainly act on the carboxyl
d. Renin end of the amino acid arginine & lysine.
Ans: B 52) Trypsinogen is activated by:
Explanation: AG1 is converted into AG2 in a. Food
Lung while AG2 is activated in lung. b. Acid
Ref: Ganong‘s review of medical physiology. c. Enterokinase
47) Structure from outer to inner villous d. Protein
core formed by: Ans: C
a. Synchito, Cyto, Connective tissue & 53) Anterior relation of hilum of right
Endothelium kidney:
b. Endothelium, Synchito, Cyto a. Pancreas
c. Cyto, Synchito, Endothelium b. Gallbladder
d. Connective Tissue, Cyto, Synchito c. Hepatic flexure
Ans: A d. 2nd part of Duodenum
SK Origional – Golden 11 14

Ans: D Ans: B
54) A man presented with fever & Explanation: Ligament that‘s damaged in
generalized cervical lymphadenopathy excessive eversion of foot is deltoid ligament.
with TLC of 7230 and biopsy of 60) Foot Drop Nerve damage is:
cervical lymph nodes done: a. Superficial Perioneal Nerve
a. Infectious Mononucelosis b. Tibial Nerve
b. CLL c. Common Perioneal Nerve
c. CML d. Obtruator Nerve
d. ALL Ans: C
Ans: A 61) Esophageal carcinoma dietary
55) A man presented with non-tender causative agent:
cervical lymphadenopathy with small a. Aryl Amines
lymphocytes: b. Nitrosamines
a. HL-Lymphocyte Predominant c. Hydrocarbons
b. CLL d. Alkali
c. CML Ans: B
d. AML 62) Girl presented with palpitation &
Ans: A tachycardia with normal Sodium &
56) Chromosomal Translocation of CML: K+ values:
a. 9:22 a. Hypotension
b. 8:21 b. Hyperaldosteronism
c. 7:22 c. Hyperthyroidism
d. 9:20 d. Pheochromocytoma
Ans: A Ans: C
57) 60 years old male presented with Explanation: As the lab values are normal for
massive spleenomegaly with TLC sodium & potassium so rest of sign & symptoms
count of 50 * 103& immature cells: points toward hyperthyroidism.
a. 9:22 63) Findings in CHF:
b. 8:21 a. Increase Hydrostatic Pressure
c. 8:22 b. Decrease Hydrostatic Pressure
d. 14:18 c. Increase Oncotic Pressure
Ans: A d. Decrease Oncotic Pressure
Explanation: The scenario is related to CML. Ans: A
That‘s with massive Spleenomegaly, Increase TLC 64) Decrease Lymphatic flow due to:
count & blast stage can be found in CML too. a. Decrease Oncotic Pressure
58) Child with 60% weight loss, not b. Increase Permeability
responding to vocal stimulus & c. Hemorrhage
bruises on leg: d. Increase Hydrostatic Pressure
a. Marasmus Ans: C
b. Physical Abuse 65) Decrease Lymphatic flow due to:
c. Malnutrition a. Increase Plasma Oncotic Pressure
d. Hypothyroidism b. Decrease Plasma Oncotic Pressure
Ans: B c. Increase Hydrostatic Pressure
Explanation: Loss of weight with no reponseto d. Increase Permeability
vocal stimulus and bruises on legs points toward Ans: A
physical abuse. 66) Major Blood Vessels Location:
59) Maximum Eversion of foot which a. Subarachnoid Space
ligament is damaged: b. Epidural Space
a. Collateral c. Extradural Space
b. Deltoid d. Between Calvaria & Skin
c. Calcanofibular Ans: A
d. Anterior Talofibular 67) Posterior Cranial Fossa relation:
SK Origional – Golden 11 15

a. Separate Cerebrum from Cerebellum d. Cryoprecipitate


b. Separate Pons from Medulla Ans: D
c. Basilar Artery Anterior to Pons Explanation: Hemophilia A is the classical
d. Basilar Artery Posterior to Pons hemophilia that‘s with Factor VIII deficiency &
Ans: C Cryoprecipitate is concentrated with Factor VIII.
68) A girl presented with epistaxis with 72) Thalassemia trait findings:
platelets count of 30,000 what is the a. Increase HbA2
most probable mechanism behind? b. Increase fetal Hb
a. Anti-Platelets Antibodies c. Increase HbA
b. Anti-IgG Antibodies d. Increase Gower Hb
c. Complement Mediated Ans: A
d. Factor 8 Deficiency 73) Which of the following is X-Linked
Ans: A disease?
Explanation: The aforementioned scenario is a. Huntington Disease
related to ITP & the pathophysiology behind ITP b. Adult Polycystic kidney disease
is antibodies against platelets that leads to platelets c. Achondroplasia
destruction. d. G6PD
69) Patient presented with epistaxis, Ans: D
bluish discoloration of skin which test 74) Reflex vision stimulates which of the
to perform initially: following?
a. BT a. Superior Colliculus
b. CT b. Inferior Colliculus
c. Aptt c. Superior Olivary Nucleus
d. Platelet Count d. Inferior Olivary Nucleus
Ans: D Ans: A
Explanation:The scenario is related to ITP & the Explanation: Superior colliculus is related with
initial test to perform is platelet count to see if its vision while inferior colliculus is related with
level is low. auditory.
70) 28 Years old Girl (Same in CPSP 75) A young boy sitting in a working room
Paper) with circumcision history of with sweating & BP of 150/80 which
bleeding which test to perform to of the following is the most likely
confirm the diagnosis: cause:
a. Factor VIII Assay a. Mental Stress
b. Factor VII Assay b. Exercise
c. Aptt c. Heat Stroke
d. PT d. Dehydration
Ans: A Ans: A
Explanation: 76) Atropine inhibits which of the
 This is a scenario related to hemophilia, following?
now two doubts arises either they a. Constriction of GIT Sphincters
mistyped girl instead of boy or if not then b. Bronchodilation
in some countries Circumcision is carried c. Pupillary Constriction
in girls too. Hemophilia is X-Linked that‘s d. Sweat Gland Secretion
very rare in girls but still it can express Ans: D
itself in female gender too. Explanation: Sweat Gland secretion is
 Initial Test for Hemophilia is aPTT mainlyunder the control of Post ganglionic
 Confirmatory Test is Factor VIII Assay. cholinergic sympathetic system & Atropine will
71) Which of the following is best therapy inhibit it by blocking the cholinergic response.
for Hemophilia A? 77) Structure passing through esophageal
a. FFP opening of Diaphragm:
b. Blood a. Azygous Vein
c. Plasma b. Thoracic Duct
SK Origional – Golden 11 16

c. Phrenic nerve b. Celiac plexus


d. Right Vagus nerve c. IVC
Ans: D d. IJV
78) Which of the following nerve damaged Ans: B
bilaterally leads to death? 85) Structure damaged at T10 level during
a. Trigeminal trauma:
b. Facial a. Spleen
c. Vagus b. Liver
d. Sympathetic Trunk c. Gallbladder
Ans: C d. Aorta
79) Vagus Nerve cut & central part Ans: A
stimulated will lead to: 86) Most radiosensitive tumor is:
a. Tachycardia a. Craniopharyngioma
b. Tachypnea b. Astrocytoma
c. Increase Respiration c. Lymph node
d. Apnea d. Glioma
Ans: D Ans: C
80) Phrenic Nerve Main Root Value: Explanation: Lymphoma > Seminoma >
a. C2 Glioma > Craniopharyngioma.
b. C3 87) Fullness of bladder & rectum
c. C4 sensation is carried by:
d. C5 a. DCML
Ans: C b. Lateral Spinothalamic Tract
81) Immediate relation of IJV after its exit c. Ventral Spinothalamic Tact
from jugular foramen: d. Corticospinal Tract
a. ICA Ans: A
b. Spinal part of Accessory Nerve 88) CT brain done & that showed necrosis
c. Vagus Nerve which type is that?
d. Facial Nerve a. Coagulative
Ans: A b. Caseous
82) On Radiograph Transverse cut section c. Liquefactive
structure not visible at T11 but Visible d. Fat Necrosis
at T12 behind the stomach: Ans: C
a. Tail of Pancreas 89) Hyperextension of neck will lead to:
b. Spleen a. Lower motor neuron lesion of Upper
c. Aorta limb
d. IVC b. Upper motor Neuron lesion of Upper
Ans: A Limb
Explanation: Tail of Pancreas lies behind the c. Lower motor neuron lesion of Lower
stomach while spleen lies behind the stomach too limb
but that‘s at T9-11 Level while rests of the other d. Loss of Tactile stimulus lower limb
don‘t fit. Ans: A
83) Patient present with pain after taking 90) Epitheloid cells is seen in which of the
fatty meal & pain is referred to tip of following?
right shoulder, this is due to: a. Sarcoidosis
a. Pancreas b. Leprosy
b. Gallbladder c. Amyloidosis
c. Kidney d. Fat necrosis
d. Appendix Ans: B
Ans: B 91) Regarding monocytes:
84) Structure lying anterior to aorta: a. Forms Multinucleated macrophages in
a. Azygous Vein certain chronic Inflammation
SK Origional – Golden 11 17

b. Goes in & out of Circulation d. Respiratory System


c. Increases in Acute Inflammation Ans: A
d. Increases in Fat necrosis 99) Permanent cells are:
Ans: A a. Hepatocytes
92) Cells increased in worm infestation: b. Glial Cells
a. Basophils c. Myocardial Cells
b. Eosinophils d. Skin
c. Neutrophils Ans: C
d. Lymphocytes Explanation: Among the given options
Ans: B myocardial cells are permanent cells & can‘t
93) IgE receptors are found on which reproduce or regenerate after injury.
cells? 100) Breast Lateral Lymphatic drainage:
a. Basophils a. Pectoral (Axillary) Lymph Nodes
b. Eosinophils b. Posterior Axillary Lymph Nodes
c. Neutrophils c. Internal Thoracic Lymph Nodes
d. Lymphoctyes d. Supraclavicular Lymph Nodes
Ans: A e. Infraclavicular Lymph Nodes
Explanation: IgE receptors are mainly formedat Ans: A
the surface of Mast cells & Basophils. Explanation: Lateral lymphatic drainage of reast
94) Coagulation pathway & kinin pathway is mainly to pectoral lymph nodes & medical to
activated via: internal thoracic lymph nodes.
a. Factor X 101) Wheat contains which vitamin?
b. Hagman Factor a. Niacin
c. Factor V b. Biotin
d. Prothrombin c. Thiamine
Ans: B d. Panthothenic Acid
95) Basophilia is given to cell by: Ans: C
a. Ribosomes 102) Thiamine deficiency findings:
b. Nucleus a. Low RBC transketolase Activity
c. Cytoplasm b. Low RBC folate
d. SER c. Subacute combined degeneration
Ans: A d. Scurvy
96) Neonate presented with history of not e. Pellegra
passing meconium, O/E meconium Ans: A
present in vagina: 103) Thyroid hormone at physiological
a. Imperforate Anus with rectovaginal condition causes:
fistula a. Fatty Acid Oxidation
b. Imperforate Anus with rectoperineal b. Protein Synthesis
fistula c. Gluconeogensis
c. Bladder extrophy d. Entry of Potassium into cells
d. Patent urachus Ans: B
Ans: A Explanation:
97) Hypospadias is due to abnormality of: The general effect of thyroid hormone is
a. Urogenital Sinus to activate nuclear transcription of large numbers
b. Urogenital Folds of gene therefore, in virtually all cells of body,
c. Urachus great number of protein enzymes, structural
d. Bladder proteins are synthesized. (Guyton).
Ans: B 104) Cortisol Main function is:
98) Ectoderm give rise to: a. Lipolysis
a. CNS b. Gluconeogensis
b. Urogenital System c. Ketogenesis
c. GIT d. Glycolysis
SK Origional – Golden 11 18

Ans: B d. Captopril
Explanation: Cortisol causes gluconeogenesis, Ans: B
extra hepatic protein catabolism & intrahepatic Explanation:Drug of choice for T.Solium is
protein anabolism. Prizaquental, but that wasn‘t in options the next
105) Volume of plasma filtered per unit best choice in Levinson is Albendazole while in
time is called: Katzung Pharmacology book it‘s Niclosamide so
a. Filtered Load we prefer Katzung.
b. Renal Clearance 111) Acetaminophen metabolism reaction:
c. Renal Blood Flow a. Acetylation
d. GFR b. Glucoronidation
Ans: D c. Oxidation
Explanation: d. Reduction
 Volume of plasma filtered per unit time is Ans: B
called GFR. Explanation: Acetaminophen is extensively
 Volume of substance filtered per unit Metabolized by the liver mainly through
time is called filtered load. glucoronidation.
106) Normal GFR value is: 112) Newborn presented with respiratory
a. 125 ml/min tract infection which physiological
b. 135 ml/min antibody is missing:
c. 110 ml/min a. IgA
d. 130 ml/min b. IgG
Ans: A c. IgM
107) Gunshot injury to cervical region & d. IgE
difficulty in breathing, muscle Ans: B
damaged: Explanation:
a. Serratus Anterior  Newborn the antibody missing is IgG.
b. Diaphragm  Child the antibody missing is IgA.
c. External Intercostal Muscle 113) Diabetic Nephropathy is diagnosed
d. Internal Intercostal Muscle by:
Ans: B a. Increase urinary Albumin
108) A British woman went to Thailand b. Increase Serum Albumin
and presented with fever, rigors & c. Microalbuminuria
chills is most likely suffering from: d. Increase Urinary Nitrogen
a. Malaria Ans: C
b. Infectious Mononucleosis Explanation: Microalbuminuria is only in case of
c. Enteric Fever DM, but if increase urinary albumin this could be
d. AIDS either nephritic or Nephrotic syndrome.
Ans: A 114) Venous drainage of heart is mainly by:
Explanation:Thailand is geographically more a. Great Cardiac Vein
related to malaria & the findings are suggestive for b. Anterior Cardiac Vein
malaria too those are Fever, rigors & specially c. Middle Cardiac Vein
chills. d. Coronary Sinus
109) Scenario of typhoid confirmation test: Ans: D
a. Western blot test Explanation: The coronary sinus receives mostof
b. CD4 count the heart‘s blood. Ref: (RJ Last).
c. Widal test 115) Apex of Heart blood supply:
d. Immunoassay a. Marginal Artery
Ans: C b. Anterior interventricular Artery
110) Drug of choice for T.Solium: c. Circumflex Artery
a. Albendazole d. PAD
b. Niclosamide Ans: B
c. Metronidazole 116) Highest systolic pressure seen in:
SK Origional – Golden 11 19

a. Aorta a. Decrease PH
b. Renal Artery b. Increase PH
c. Pulmonary Artery c. Increase osmolarity
d. Radial Artery d. Increase protein
Ans: B Ans: A
117) Familial medullary carcinoma & 124) Which of the following is decreased in
history of hypertension: CSF?
a. Pheochromocytoma a. Glucose
b. Addison Disease b. Protein
c. Adrenal Crisis c. Sodium
d. Essential HTN d. Potassium
Ans: A Ans: B
118) Genetic Analysis done with: Explanation: Both Glucose & protein is in
a. CVS decrease amount in CSF but protein is decreased
b. Amniotic Fluid Analysis more than glucose.
c. USG 125) Patient with history of Meningitis the
d. Doppler drug of choice is:
Ans: B a. Ceftazidime
119) Muscle injury assessment done with: b. Ceftrixone
a. CK-MM c. Penicillin G
b. CK-MB d. Macrolides
c. Muscle Biopsy Ans: B
d. Myoglobin 126) Transmitted by mosquito:
Ans: A a. Yellow Fever
120) A pregnant lady from remote village b. AIDs
died on the way to hospital, the c. IM
probable cause is: d. Hep. B
a. Amniotic Fluid Embolism Ans: A
b. Fat Embolism 127) Ovaries lymphatic drainage:
c. Pulmonary Embolism a. Para-Aortic Nodes
d. Hemorrhage b. Internal Iliac Nodes
Ans: A c. External Iliac Nodes
Explanation:Pregnant lady on way to hospital d. Celiac Nodes
having no history of RTA so the most likely cause Ans: A
is amniotic fluid embolism. 128) Ureter damaged during hysterectomy:
121) During contraction the length of a. Pelvic Brim
following decreases: b. Bifurcation of Common Iliac Artery
a. I band c. Base of Broad Ligament
b. Actin d. Suspensory Ligament
c. Myosin Ans: C
d. Sarcomere Explanation:Ureter damaged during
Ans: A >D(Both Shorten) hysterectomy follows the following order:
122) Analgesic Effect of TCA: Cardinal Ligament > Broad Ligament >
a. 1 Week Pelvic Brim.
b. 1 Month 129) Parasympathetic Action:
c. 1 day a. Opens GIT Sphincters
d. 2 months b. Closes GIT Sphincters
Ans: A c. Bronchodilation
Explanation:Analgesic effect of TCA takes 1 – 2 d. Increase HR
Weeks while antidepressant effect of TCA takes 3 Ans: A
– 4 weeks. 130) Nucleus of tractus solitarius contains:
123) CSF differ from plasma by: a. 1st Order Neuron
SK Origional – Golden 11 20

b. 2nd Order Neuron 137) Anti-hyperlipedemiac drug side effect:


c. 3rd Order Neuron a. GIT disturbances
d. None b. Constipation
Ans: B c. Seizure
131) A patient presented with paralysis of d. Hypertension
left limb & right lower face along with Ans: A
homonymous Hemianopia lesion is Explanation: The commonest side effects for
present in: most of the drugs are GIT disturbances.
a. Basilar Pons 138) T-Test done for:
b. Forebrain a. Two groups
c. Midbrain b. Three Groups
d. Medulla c. Variable
e. Thalamus d. None
Ans: B Ans: A
Explanation:Homonymous Hemianopia is 139) Good doctor & patient relationship is:
related only to forebrain lesion among the a. Update Skills
mentioned. b. Good Bedside manners
132) 03 weeks of IV antibiotics given & c. Having sound knowledge
now presented with bleeding what d. Active Listening to every question of the
could be the causes? patient.
a. Vitamin K Deficiency Ans: D
b. Vitamin C Deficiency Explanation: B & D both are part of good
c. B12 Deficiency doctor-patient relationship but active listening is
d. Low Platelets count foremost important.
Ans: A 140) Thiazide diuretics leads to:
133) Child prone to systemic candidiasis: a. Hyperglycemia
a. Antibiotic Use b. Hypokalemia
b. Hospitalization c. Hyperlipidemia
c. Immunosuppressant d. Hyperurecemia
d. Viral infection e. Hypercalcemia
Ans: C Ans: B
134) Virus related cancer: Explanation:Follow the following
a. Renal Cell Carcinoma sequence:Hypokalemia > Hyperglycemia >
b. Small Cell Carcinoma Hyperlipidemia > Hyperurecemia >
c. Diffuse Cell Carcinoma Hypercalcemia.
d. T Cell Leukemia 141) Cajal Stain cells within CNS with
Ans: D multiple cytoplasmic processes:
135) Which abnormality is associated with a. Protoplasmic Astrocytes
bicuspid aortic valve? b. Oligodendrocytes
a. ASD c. Schwann Cells
b. VSD d. Neuronal Cells
c. PDA Ans: A
d. Coaractation of Aorta 142) Duchene – MuscularDystrophy
Ans: D defective gene makes the cell
136) Crescents are mainly formed by: impermeable to which of the
a. Endothelial Cells following?
b. Mesengial Cells a. Amino Acid
c. Parietal Cells b. Water
d. Blood Cells c. Electrolytes
Ans: C d. Protein
Explanation: Crescents are mainly formed by Ans: A
macrophages & Parietal cells.
SK Origional – Golden 11 21

143) Most reliable sign of irreversible cell Explanation:


injury:  Down Syndrome: Trisomy 21
a. Massive Calcium influx  Klienfilter Syndrome: Trisomy 18
b. Lysosomal Enzyme Release  Patau Syndrome: Trisomy 13
c. Cell Membrane Damage 150) Skin cancer most commonly
d. Mitochondrial Damge associated with HIV:
Ans: C a. Kaposi Sarcoma
Explanation: According to Pathoma the most b. Burkitt Lymphoma
reliable sign of irreversible cell injury is Cell c. B-Cell Lymphoma
membrane damage, while in Robbins massive d. Hodgkin‘s Lymphoma
calcium influx is given but I am in favor of Ans: A
Pathoma here. 151) Loss of thirst due to:
144) Active transport requires: a. Low Plasma osmolarity
a. Pump b. High Plasma osmoalrity
b. Enzyme c. Vasopressin
c. Carrier d. Angiotensin
d. Protein Ans: A
Ans: A 152. Aldosterone decreases secretion of:
Explanation: Active transport mainly a. Potassium
requiresenergy & that‘s provided via Na-K b. Chloride
ATPase Pump. c. Sodium
145) Camp involves: d. Magnesium
a. Receptor Ans: C
b. Enzyme 153. Tetanus prophylaxis is done with:
c. Carrier a. Tetanus Toxoid
d. Pump b. Tetanus Immunoglobulin
Ans: A c. Washing hands
146) Sepsis is confirmed with: d. Chlorhexidine
a. Heart Rate > 100 Ans: A
b. Positive Blood Culture 154) Injury to kidney with intact gerota
c. Negative Blood Culture fascia, the blood can go into:
d. Low BP a. Pelvis
Ans: B b. Opposite Kidney
147) 65 years old diabetic with dyspepsia c. Both
will require: d. None
a. PPI Ans: A
b. Prokinetics Explanation:Injury to kidney with intact fascia
c. H2 – Blockers the blood will go inferiorly into pelvis but if the
d. None fascia was damaged too then it can go to opposite
Ans: B kidney too.
148) Mode is: 155) Colon removed will lead into:
a. Least repeated Value a. Electrolytes & Water Imbalance
b. Average of Values b. Water Imbalance
c. Most repeated Value c. Electrolyte Imbalance
d. Middle Value d. Fatty Stool
Ans: C Ans: A
149) Down syndrome is trisomy: 156) Characteristic feature of mandible in
a. 21 old age:
b. 18 a. Obtuse Angle
c. 13 b. Mental foramen lies in middle of body of
d. 14 mandible
Ans: A
SK Origional – Golden 11 22

c. Mental foramen lies in upper of body of 164) Directly proportional for oxygen
mandible diffusion:
d. Mental foramen lies in lower of body of a. Alveolar Capillary surface Area
mandible b. Thickness
Ans: C (Ref: Parikh Page # 80) c. Fibrosis
157) Reticular formation location: d. Hemoglobin
a. Floor of Sulvyian Aqueduct Ans: A
b. Roof of Sulvyian Aqueduct Explanation: Diffusion is directly proportional
c. Lateral Ventricle with Area, differences in partial pressures &
d. Fourth Ventricle indirectly proportional with thickness.
Ans: A 165) Staging of tumor:
158) Skeletal muscle attached to bone via: a. Extent of spread
a. Tendon b. Degree of differentiation
b. Ligament c. Both
c. Muscle d. None
d. Membrane Ans: A
Ans: A 166) Grading of tumour:
159) Internal oblique muscle forms: a. Extent of Spread
a. Deep Inguinal Ring b. Degree of Differentiation
b. Superficial Inguinal Ring c. Both
c. Conjoint Tendon d. None
d. None Ans: B
Ans: C 167) After knee trauma patient presented
160) Woman presented with bitemporal with bone in muscle due to:
hemianopsia & increase Prolactin a. Dysplasia
level which organ is damaged: b. Metaplasia
a. Hypothalamus c. Anaplasia
b. Thyroid Gland d. Desmoplasia
c. Ovary Ans: B
d. Pituitary Gland 168) Action of SCM Muscle:
Ans: D a. Turning of Head to Same Side
161) Estrogen & Progesterone at 07 b. Turning of Head to Opposite Side
months of pregnancy are produced by: c. Elevating Neck
a. Placenta d. Depressing Scapula
b. Corpus Luteum Ans: B
c. Corpus Hemorrhagicum 169) Sequence for vitamin-D activation:
d. Ovary a. 25 Hydroxylation in Liver & 1
Ans: A Hydroxylation in Kidney
162) Carcinoma of head of pancreas will b. 1 Hydroxylation in Liver & 25
compress: Hydroxylation in Kidney
a. Cystic Duct c. Both 1 & 25 Hydroxylation in Kidney
b. IVC d. Both 1 & 25 Hydroxylation in Liver
c. CBD Ans: A
d. None 170) Vitamin D formation decreases in:
Ans: C a. Acute Renal Failure
163) T-Configuration of hemoglobin has b. Chronic Renal Failure
decrease affinity for: c. Acute Liver Failure
a. Oxygen d. Acute metabolic disturbances
b. CO2 Ans: B
c. Helium 171) Neck Swelling with dropping of
d. Nitrogen eyelids:
Ans: A
SK Origional – Golden 11 23

a. Antibodies against Acetylcholine d. Antibodies


receptors Ans: A
b. Antibodies against calcium receptors 179) A+ blood group transfusion reaction
c. Antibodies against both receptors occurs if transfused with:
d. None a. O+Ve
Ans: A b. O-Ve
Explanation: The mentioned scenario is of c. AB-ve
myasthenia gravis & that‘s positive for d. A+Ve
acetylcholine receptors antibodies. Ans: C
172) Cardiogenic shock is caused by: Explanation: O+ is universal donor & transfusion
a. Myocardial Infarction reaction will not occur, however AB- is universal
b. Hemorrhage acceptor and not donor so reaction would occur.
c. Sepsis 180) Anemia during pregnancy is due to:
d. Hypovolemia a. Dilutional Anemia
Ans: A b. Iron deficiency Anemia
173) FSH is inhibited by: c. Anemia of Chronic disease
a. LH d. Sideroblastic Anemia
b. Testosterone Ans: A
c. Inhibin 181) Individual muscle fiber is covered
d. Dopamine with:
Ans: C a. Endomysium
174) Right Ventricle characteristic feature b. Perimysium
is: c. Epimysium
a. Pulmonary Trunk arises d. All
b. Aorta Arises Ans: A
c. Partial pressure of oxygen increases 182) During exercise, if heart rate is
d. Partial Pressure of CO2 decreases doubled then cardiac cycle time will
Ans: A be:
175) Increase cerebral blood flow: a. 0.2 Seconds
a. Isoflurane b. 0.4 Seconds
b. Sevoflurane c. 0.8 Seconds
c. NO d. 1.2 Seconds
d. Halothane e. 1.6 Seconds
Ans: D Ans: B
176) Juxtaglomerual nephrons most Explanation: Normal Cardiac cycle is 0.8 Secso if
important function is: heart rate is doubled then cardiac cycle time will
a. Counter current Multiplier decrease to half that‘s 0.4 Sec.
b. Counter Current Exchanger 183) PTU inhibits oxidation of:
c. Increase Renin Secretion a. Iodide
d. Decrease Renin Secretion b. T3
Ans: A c. T4
177) During general anesthesia decrease d. TSH
heat production is due to: Ans: A
a. Channels inhibition 184) Platelets aggregation is done by:
b. Decrease Muscle tone a. Prostacylin
c. Decrease venous tone b. TXA2
d. Decrease compliance c. Angiotensin
Ans: B d. Renin
178) Most important cells in immunity: Ans: B
a. Helper T-Cells 185) Hydrogen Per oxide is found in:
b. Cytotoxic T-Cells a. Peroxisome
c. NK Cells b. Lysosome
SK Origional – Golden 11 24

c. Nucleus 193) Oral fecal route of which of the


d. Ribosome following?
Ans: A a. Hep. E
186) Spore forming Bacteria: b. Hep. B
a. Clostridium c. Hep. C
b. Corynebacterium d. CMV
c. Candida Albicans Ans: A
d. T.Pallidum Explanation: Hep. A & Hep. E is having oral
Ans: A fecal route, prefer Hep. A if present in option.
187) Which of the following is an Opsonin? 194) Moderate Exercise increases:
a. C3a a. Stroke volume
b. C3b b. Venous Compliance
c. C4 c. Stress
d. IgM d. Cholesterol
Ans: B Ans: A
188) Regression of tumour after 195) DVT is due to:
chemotherapy is done by: a. Factor C deficiency
a. Apoptosis b. Platelet Deficiency
b. Lysosome c. Increase WBC
c. Necrosis d. Prolong Immobilization
d. Nucleus Ans: D
Ans: A 196) A boy on bicycle had an accident and
189) AJK earthquake leading to death of his presented with unable to void is due to
uncle & now in depression: injury to:
a. Social Crisis a. Urethra
b. Situational Crisis b. Urinary Bladder
c. Emotional Crisis c. Prostate
d. Developmental Crisis d. Vas deferens
e. None Ans: A
Ans: B 197) Most potent stimulus for
190) Dehydration in a child is checked by erythropoietin release is:
palpating: a. Hypokalemia
a. Bregma b. Hyperkalemia
b. Lambda c. Hypoxia
c. Nasion d. Hyperoxia
d. Frontal bone Ans: C
Ans: A 198) S2 splitting occurs due to:
191) Skin disinfection: a. Aortic closure before pulmonary
a. Phenol b. Mitral closure before pulmonary
b. Ethanol c. Tricuspid closure
c. Methanol d. ASD
d. Chlorhexidine Ans: A
Ans: D 199) Hematocrit rises in carcinoma of:
192) A man working in factory exposed to a. Lungs
nitrates develops headache every b. Kidney
Monday is due to: c. Liver
a. Tachyphylaxis d. Bone
b. Tolerance Ans: B
c. Desensitization
d. Dependence
Ans: B ===============
SK Origional – Golden 11 25

SURGERY & ALLIED c. External Laryngeal Nerve


st
October 1 , 2020 - Afternoon d. Internal Laryneal Nerve
Ans: B
1) The medial wall of ischiorectal fossa is Explanation: Vagus nerve is Posterolateral to
formed by: trachea. While Recurrent laryngeal nerve is medial
a. Levator Ani to it.Ref: (Snell, Page # 657)
b. Perineal Membrane 6) Drug with U=100, V=2 & P=10 find C:
c. Obtruator Internus a. 20
d. External Anal Sphincter b. 30
e. Gluteus Maximus c. 10
Ans: A d. 50
Explanation: Ans: A
 Medial wall: Sloping fibers of Levator Ani Explanation:
& Anal Sphincter  C=UV/P
 Lateral Wall: Lower part of Obtruator  C=100 * 2/10
Internus with covering Fascia  C=200/10
2) Melanoma invade orbital cavity  C=20
through: 7) Bilateral Ptosis with muscle wasting
a. Sclera on mastication:
b. Optic Nerve a. Myasthenia Gravis
c. Emissary Vein b. Eaton Lambort Syndrome
d. Retina c. GBS
e. Lymphatic d. Polio
Ans: C Ans: A
Explanation: Uveal malignant melanoma is the Explanation: Ptosis & muscle weakness is more
most Common primary intraocular tumour of typicalFor MG from the above mentioned
adulthood & orbital extension probably occurs choices.
through the emissary vein. 8) Best measure of renal plasma flow:
3) Fracture of Pterion which artery a. Creatinine Clearance
damaged? b. Inulin Clearance
a. Middle Meningeal Artery c. PAH
b. Middle Cerebral Artery d. Glucose
c. Posterior Cerebral Artery Ans: C
d. Anterior Cerebral Artery Explanation:
Ans: A A. Best Measure of RPF = PAH
Explanation: Pterion is related with Middle B. Best Measure of GFR = Inulin Clearance
Meningeal Artery & that‘s what will be damaged at C. Best Estimate of GFR = Creatinine
this site trauma Clearance.
4) Structure from medial to lateral in 9) Normally present in Amniotic Fluid &
femoral triangle: raised in HCC:
a. Nerve, Artery, Vein a. AFP
b. Vein, Artery, Nerve b. LDH
c. Nerve, Vein, Artery c. Esterase
d. None d. Elastase
Ans: B Ans: A
Explanation: Remember the word ―VAN‖ for Explanation: AFP is normally found in Amniotic
Structures from medial to lateral in femoral fluid & it‘s the tumour marker for hepatocellular
triangle that‘s vein, artery & nerve. carcinoma (HCC).
5) Nerve lying lateral to trachea: 10) Fetal Period beings at which week:
a. Recurrent Laryngeal Nerve a. 10th Week
b. Vagus Nerve b. 3rd Week
c. 2nd Week
SK Origional – Golden 11 26

d. 9th Week 16) Which vein drains into superior vena


Ans: D cava?
Explanation: a. Azygous Vein
A. Embryonic Period: Upto 8th Week b. Hemiazygous Vein
B. Fetal Period: 9th Week onward c. Lumber Vein
11) Blood supply of Posterior 1/3rd of IV d. Sacral Vein
cardiac Septum: Ans: A
a. RCA 17) A patient presented to you with chest
b. LCA pain what question will give you
c. Circumflex maximum information?
d. Marginal a. Tell me about pain
Ans: A b. Where‘s the pain
Explanation: Posterior 1/3rd of IV septum is c. How‘s the pain
suppliedby RCA while Anterior 2/3rd is supplied d. Point to the location of Pain
by LCA. Ans: A
12) Location of SA Node: Explanation: Always try to ask open ended
a. Lower part of Atrium question. In order to gain more information.
b. Upper Right Atrium 18) Estimation of GFR is done via:
c. Upper Left Atrium a. Inulin Clearance
d. Close to Ventricles b. Creatinine Clearance
Ans: B c. PAH
Explanation: SA node is located in upper part of d. Glucose
right atrium in the upper part of crista terminalis. Ans: B
13) Schaumann bodies are found in: Explanation:
a. Sarcoidosis  Best Measure of RPF = PAH
b. Parkinson‘s Disease  Best Measure of GFR = Inulin Clearance
c. Rabies  Best Estimate of GFR = Creatinine
d. Asbestosis Clearance.
Ans: A 19) HLA B-27 is associated with:
Explanation: a. Ankylosing Spondylitis
 Schaumann Bodies: Sarcoidosis b. Gout
 Negri Bodies: Rabies c. Sepsis
 Lewy Bodies: Parkinson‘s Disease d. IBS
 Ferruginous Bodies: Asbestosis Ans: A
14) Nerve passing through foramen Explanation:
magnum:  HLA B-27 is associated with ―PAIR‖
a. Spinal Part of Accessory nerve  P= Psoraitic Arthritis
b. Occulomotor Nerve  A= Ankylosing Spondylitis
c. Trochlear Nerve  I= IBD associated Arthritis
d. Trigeminal Nerve  R= Reactive Arthritis
Ans: A 20. Actin is bound to Z line by:
15) Obese lady presented with myxedema a. Myosinin
is at risk of: b. Laminin
a. Fracture c. Capsinin
b. Osteoporosis d. Actinin
c. Carpal tunnel syndrome Ans: D
d. Tarsal Tunnel Syndrome Explanation: Actinin is a microfilament protein
Ans: C that‘s Necessary for the attachment of Actin
Explanation: Carpal tunnel syndrome is filament to the Z-Lines in Skeletal muscle cells &
commonly seen in pregnancy followed by to the dense bodies in smooth muscle cells.
myxedema. 21) Ulnar nerve injured at medial
epicondyle will cause:
SK Origional – Golden 11 27

a. Hypothenar Wasting b. Venous Compliance


b. Claw hand c. TPR
c. Cubitus Varus d. Cardiac Index
d. Cubitus Valgus Ans: A
Ans: A 28) Structures passing through posterior
Explanation: diaphragmatic opening:
 Ulnar nerve damaged at elbow that‘s a. IVC
medial epicondyle level will lead to b. Aorta, IVC
hypothenar wasting. c. Aorta, Azygous Vein & Thoracic Duct
 Ulnar damaged at wrist level will lead to d. None
claw hand. Ans: C
22) Phrenico-Colic Ligament support: Explanation: Remember the pneumonic for
a. Spleen structures passing through posterior opening
b. Liver that‘s ATA (Aorta, Thoracic Duct, Azygous Vein).
c. Kidney 29) Urine specific gravity indicates:
d. Intestine a. Concentration
Ans: A b. Filtration
23) Behind the neck of Pancreas: c. Secretion
a. SMA d. Absorption
b. IVC Ans: A
c. Portal Vein 30) Kaplan-Meier Curve indicates:
d. CBD a. Incidence of Disease
Ans: C b. Demography
24) Which of the following has profound c. Severity of Disease
effect on the heart? d. Survival
a. Potassium Ans: D
b. Calcium 31) Hyperextension of Neck will lead to:
c. Sodium a. Lower motor neuron lesion of upper limb
d. Glucose b. Upper motor neuron lesion of upper limb
Ans: A c. Lower motor neuron lesion of lower limb
25) Major buffer of the body: d. Loss of tactile stimulus of lower limb
a. Ammonia Ans: A
b. Protein 32) Median nerve injury leads to:
c. Phosphate a. Thenar wasting
d. HCO3 b. Hypothenar wasting
Ans: D c. Claw Hand
Explanation: d. Carpal tunnel syndrome
 Most Important buffer of body: HCO3 Ans: A
 Intracellular Buffer: Protein 33) Submandibular gland surgery which
 Kidney Buffer: Phosphate of the following nerve is damaged?
 Only in Kidney: Ammonia a. Occulomotor
26) Majority part of blood CO2 in the form b. Trochlear
of: c. Trigeminal
a. Carbaminohemoglobin d. Marginal madibular branch of facial nerve
b. Bicarbonate Ans: D
c. Dissolved form 34) HSV most commonly affects which of
d. None the following nerve?
Ans: B a. Facial nerve
Explanation:More than 70% of blood CO2 is in b. Abducent nerve
the form of bicarbonate. c. Trigeminal nerve
27) Athletes are normally having high: d. Trochlear nerve
a. Stroke volume Ans: C
SK Origional – Golden 11 28

Explanation: Herpes simplex virus (HSV) most Ans: D


commonly effect Trigeminal Nerve& leads to Explanation: Among the mentioned ATT drugs
Trigeminal Neuralgia pyrazinamide is associated with gout.
35) Erythropoietin is secreted by: 42) A child can‟t close his lips due to
a. Peritubular capillaries damage to:
b. Mesengial Cells a. Orbicularis Oculi
c. Glomerulus b. Orbicularis Oris
d. Epithelial Cells c. Levator Palpebrae
Ans: A d. Masseter
Explanation: Erythropoietin is secreted by Ans: B
interstitial fibroblasts of peritubular capillaries & 43) Patient planned for angiography
PCT. developed bleeding due to:
36) Erythropoietin is secreted by which a. Corticosteroids
organ? b. ACEI
a. Lung c. Beta Blockers
b. Intestine d. Aspirin
c. Liver Ans: D
d. Kidney 44) Irreversible Cox Inhibitor:
Ans: D a. Corticosteriods
Explanation: 90% of Erythropoietin is secreted b. Rifampicin
via kidney & only limited via liver. c. Celecoxib
37) Energy used during respiration: d. Aspirin
a. 5% Ans: D
b. 10% 45) Basophilia of Cell is due to:
c. 15% a. Ribosomes
d. 20% b. Lysosomes
Ans: A c. Peroxisome
38) Palpable lymph nodes in pectoral part d. Mitochondria
of axillary nodes are most likely: Ans: A
a. Upper outer quadrant breast 46) Prominent feature of reversible cell
b. Medial quadrant breast injury:
c. Whole breast a. Cellular swelling
d. None b. Cell Membrane Damage
Ans: A c. Lysosomal Damage
39) Testosterone leads to: d. Mitrochondrial Damage
a. Breast development Ans: A
b. Laryngeal thickening 47) Patient with chest pain, O2& NG
c. Esophagus growth given next best step is:
d. None a. Coronary Angiography
Ans: B b. Medications
40) Post MI a patient died on 9th day: c. Trade mill exercise test
a. Cardiac Tamponade d. Enzyme Level
b. Arrhythmia Ans: D
c. Shock 48) Nerve that passes through superficial
d. Blood Occlusion inguinal ring:
Ans: A a. Sciatic Nerve
41) Patient on ATT with big toe swelling b. Femoral Nerve
most likely due to: c. Ilioinguinal Neve
a. Rifampicin d. Obtruator Nerve
b. Streptomycin Ans: C
c. Ethambutol 49) Right boarder of heart is formed by:
d. Pyrazinamide a. Right Atrium
SK Origional – Golden 11 29

b. Right Ventricle 57) Drug of choice for T.Solium:


c. Left Atrium a. Albendazole
d. Left Ventricle b. Prizaquental
Ans: A c. Metronidazole
Explanation:Right boarder of heart is formed by d. Vancomycin
right atrium but if asked on CXR then RA+SVC. Ans: B
50) Numbness of the thumb which Explanation: DOC for T.Solium is Prizaquental,
dermatome is involved? but when not in option then prefer Niclosamide
a. C3 followed by Albendazole.
b. C4 58) In hypothyroidism what will increase?
c. C5 a. T3
d. C6 b. T4
e. T1 c. Both T3,T4
Ans: D d. TSH
51) Barret esophagus is: Ans: D
a. Anaplasia Explanation:
b. Dysplasia  In Hypothyroidism TSH will increase &
c. Metaplasia T3,T4 will decrease.
d. Desmoplasia  In Hyperthyroidism TSH will decrease &
Ans: C T3,T4 will increase.
52) Erosive protruded area on endoscopy: 59) Unmyelinated slow pain fibers:
a. Ulcer a. C-Fibers
b. Lesion b. A-Fibers
c. Condition c. B-Fibers
d. Malignancy d. A-Delta
Ans: A Ans: A
53) PT is done for: 60) After tubal ligation bleeding source:
a. HCC a. Uterine Artery
b. Jaundice b. Ovarian Artery
c. Cholestasis c. Internal Thoracic Artery
d. Antibiotic over Use d. None
Ans: B Ans: A
54) Poor Prognostic factor for burn Explanation:
patient is:  During Hysterectomy & Tubal Ligation
a. DIC the artery damaged is Uterine Artery.
b. Shock  During oophorectomy the artery damaged
c. Hypoxia is internal thoracic artery.
d. Hyponatremia 61) After hysterectomy patient complains
Ans: A of pain in lumber region:
55) Diapedesis refers to: a. Ureter Ligation
a. Margination b. Uterine Artery Ligation
b. Migration c. Ovarin Artery Ligation
c. Chemotaxis d. Bladder Trauma
d. Apoptosis Ans: A
Ans: B 62) Which of the following cells are not
56) Bladder on cystoscopy with 1cm normally present in pancreas?
growth & basement membrane intact: a. Alpha Cells
a. Ca in Situ b. Beta Cells
b. UTI c. D Cells
c. BPH d. Mucous Cells
d. Bladder Ca e. Serous Cells
Ans: A Ans: D
SK Origional – Golden 11 30

63) Winging of scapula due to damage to d. None


which nerve? Ans: A
a. Long thoracic nerve 70) Slow growing tumour of Parotid:
b. Accessory Nerve a. Warthin‘s tumour
c. Hypoglossal Nerve b. BCC
d. Suprascapular Nerve c. Squamous Cell Carcinoma
Ans: A d. Pleomorphic Adenoma
64) Small intestine epithelial projections Ans: D
called: 71) Old male with weight loss & opacity
a. Microvilli in right lower lob with increase
b. Paneth Cells calcium & decrease phosphate:
c. M-Cells a. Hyperparathyroidism
d. Cilia b. Hypoparathyroidism
Ans: A c. Thyroid disorder
65) Enzyme responsible for DNA size: d. Pituitary disorder
a. Isomerase Ans: A
b. Telomerase 72) Cause of fat necrosis in breast:
c. Synthase a. Peritonitis
d. Gyrase b. Breast Trauma
Ans: B c. RTA
66) Regarding Neutrophils correct d. Fat Embolism
statement is: Ans: B
a. Migrate into & out of blood 73) Enlarged jugulodigastric lymph
b. More phagocytic in blood stream nodes:
c. Decrease in infective condition a. Palatine tonsil
d. Decrease when corticosteroid given b. Phyarngeal tonsil
Ans: B c. Tubal tonsil
67) Slow component of nystagmus is d. All
controlled by: Ans: A
a. Vestibular Apparatus 74) Head trauma breathing pattern:
b. Brainstem a. Kussmaul Breathing
c. Cortex b. Cheyne-Stokes Breathing
d. Medulla c. Biot‘s Respiration
Ans: A d. Periodic Breathing
Explanation: Endolymph flow stimulates hair Ans: B
cells that‘s vestibular ganglion & vestibular nuclei Explanation:
while fast component by cortex.  Cheyne-Stokes breathing is commonly
68) Radiation sensitive tumour: seen after stroke & head trauma
a. Craniopharyngioma  Kussmaul Breathing pattern is seen DKA
b. Brainstem Glioma & Uremia
c. Medullary Ca 75) Hemorrhagic infarct is seen in:
d. Esophageal Ca a. Spleen
Ans: B b. Retina
Explanation: c. Intestine
 Tumour sensitive to radiation follow the d. Ovary
following order: Ans: C
 Lymphoma > Seminoma > Glioma > 76) Patient was diagnosed to have a pale
Craniopharyngioma infarct, in which condition pale infarct
69) Tachyphylaxis: occurs?
a. Rapid unresponsiveness a. Hypertensive Stroke
b. Deficiency of Receptors b. Embolic/Ischemic Stroke
c. Gradual decrease in drug action c. Rupture of Berry Aneurysm
SK Origional – Golden 11 31

d. Atherosclerosis Explanation:3rd Degree burn would present with


Ans: B wound contraction but in black people with
Explanation: keloid.
 Pale (Ischemic) types of infarctions: 82) Vasculitis features with purpura on
 Increased density of tissue (Heart, the buttocks in a child:
kidney,spleen) prevents RBCs released a. HSP
from damaged vessels from diffusing b. Arthritis
through the necrotic tissue therefore the c. SLE
tissue has a pale appearance. d. Sjogren Syndrome
77) Male with urethral discharge, gram Ans: A
staining showed gram negative 83) Negative Free water Clearance:
diplococci: a. Nephrogenic DI
a. N.Gonorrhea b. Central DI
b. N.Meningitidies c. SIADH
c. Pseudomonas d. Carcinoma
d. Staph Aureus Ans: C
Ans: A 84) Head Injury/Trauma leads to:
78) A mechanic presented with dysuria & a. Central DI
yellow discharge: b. Nephrogenic DI
a. Gonorrhea c. Both
b. Chlamydia d. None
c. T.Pallidum Ans: A
d. E-Coli 85) A man ingested some medications &
Ans: B then presented with Hyperventilation
79) Pus Contains: and PH of 7.48, the drug is:
a. Plasma Cells a. Dopamine
b. Neutrophils b. Adrenaline
c. Lymphocytes c. Acetyl Salicylic Acid
d. Antibodies d. Steroids
Ans: B Ans: C
80) Stab injury wound healing with 86) Alcoholic patient presented with
abundant collagen & raised lesion jaundice & AFP of 988:
projecting beyond original wound: a. Alcoholic Hepatitis
a. Keloid b. RHF
b. Hypertrophic Scar c. Iron overdose
c. None d. Cholestasis
d. Both Ans: A
Ans: A 87) Male presented with limb weakness &
Explanation: respiratory symptoms after consuming
 Keloid extends beyond boarders of canned pack foods:
original wound with claw-like projections a. Botulism
typically on earlobes, face & upper b. GBS
extremities. c. MG
 Hypertrophic Scar is confined to boarders d. Staph Aureus
of original wound. Ans: A
81) 3rd degree burn: 88) Common perioneal nerve damaged
a. Wound Contraction which of the following will be lost?
b. Keloid a. Inversion of foot
c. Both b. Plantar flexion of Foot
d. None c. Eversion of foot
Ans: A d. All of the above
Ans: C
SK Origional – Golden 11 32

Explanation:Common perioneal nerve causes a. Bifurcation of Common Iliac Artery


eversion & dorsiflexion of foot and if damaged b. Pelvic Brim
these action will be lost. c. Ischial Spine
89) Nasopharyngeal carcinoma is caused d. Infront of Sacrum
by: Ans: B
a. EBV 95) Gummatous Necrosis is a feature of:
b. CMV a. Syphilis
c. HPV b. AIDS
d. HIV c. IM
Ans: A d. Hepatitis
Explanation: Ans: A
 Nasopharyngeal carcinoma is caused by 96) 10 years old child presented with lip
EBV cyanosis:
 Oropharyngeal carcinoma is caused by a. TGA
HPV b. TOF
 Bronchial carcinoma is caused by CMV c. ASD
90) Patient presented with d. Bicuspid Aortic Valve
nasopharyngitis & lymphadenopathy Ans: B
these features are also carried by the 97) Which of the following unlock the
organism that causes: knee joint?
a. Burkitt Lymphoma a. Sarotrius
b. AIDS b. Gluetus minimus
c. Bronchogenic Carcinoma c. Rectus Femoris
d. BCC d. Popliteus
Ans: A Ans: D
Explanation:EBV causes nasopharngeal 98) Specific gravity of CSF:
carcinoma & that‘s linked with burkitt lymphoma a. 1.005 – 1.009
too. b. 1.004 – 1.008
91) CSF & Serum differs from each other c. 1.009 – 2.009
by: d. 1.003 – 1.007
a. Low Glucose Ans: A
b. Sodium 99) Turner syndrome Genetic:
c. Potassium a. 45 XO
d. Chloride b. 45 XXO
Ans: A c. 42 XXY
Explanation: CSF has low glucose & protein d. 42 XX
compare with serum. Ans: A
92) Young female presented with atypical 100) Gynacomastia is caused by:
lymphoctyosis: a. Klienfilter syndrome
a. Infectious Mononucleosis b. Down syndrome
b. AIDS c. Edward Syndrome
c. HBV d. Patau Syndrome
d. CMV Ans: A
Ans: A 101) A girl with primary amenorrhea,
93) Which of the following is higher in height 5‟4”, well developed breasts,
tears as compare to plasma? small vagina, no uterus on U/S &
a. Amino Acids genotype XY:
b. Sodium a. Adrenogenital Syndrome
c. Potassium b. Testicular Feminization Syndrome
d. Bicarbonate c. CAH
Ans: A d. Drug Induced
94) Ureter obstruction most common site: Ans: B
SK Origional – Golden 11 33

Explanation: Androgen insensitivity syndrome 109) Sarcoma diagnosis:


also known as testicular feminization syndrome a. Increase Vascularity
with female genitalia & female appearance in male b. Pleomorphism
genotype. c. Metaplasia
102) Femoral sheath is formed by: d. Dysplasia
a. Fascia Ilica & Fascia Transversalis Ans: A
b. Fascia Lata & Fascia Iliaca 110) Regarding Cartilage:
c. Fascia Posas & Iliaca a. Epiglottis is hyaline
d. None b. Synovial Joint has fibrous cartilage
Ans: A c. Appositional growth
103) Thyroid swelling moves due to: d. Transitional growth
a. Pretracheal Fascia Ans: C
b. Vertebral Fascia 111) Malignant feature:
c. Carotid Fascia a. Pleomorphism
d. Muscles b. Metaplasia
Ans: A c. Desmoplasia
104) Broca‟s Aphasia due to blockage of: d. Increase N/C ratio
a. MCA Ans: A
b. ACA Explanation:
c. PCA  Malignancy is characterized by:
d. Basilar Artery  Metastasis > Invasion > Pleomorphism >
Ans: A Increase N/C Ratio
105) A boxer hit on chin will lead to: 112) True regarding osteogenesis:
a. Loss of sensation of upper teeth a. Density increases after deposition
b. Loss of sensation of lower teeth b. Osteoid formation
c. Loss of sensation of neck dermatome c. Osteoblasts are multinucleated
d. Loss of function of platysma d. Osteoblasts derived from osteoclast
Ans: B Ans: B
106) Respiratory centre is located in: 113) Autoradiography uses which
a. Medulla technique?
b. Pons a. Fluorescent Dye
c. Midbrain b. Isotopes bound with Antibodies
d. Cortex c. Radioactive Ions
Ans: A d. Iron bound Material
107) Child with decreased MCV & Ans: B
Spleenomegaly what investigation to 114) Growth hormone inhibitor:
be done: a. Somatostatin
a. TIBC b. Somatomedian
b. Ferritin c. Lactotrophs
c. Serum Iron d. CRH
d. Hb Electrophoresis Ans: A
Ans: D 115) Stimulus for aldosterone secretion:
108) 40 years old male drink water from a a. Hypokalemia
stream developed mild diarrhea & b. Hyperkalemia
other symptoms along with two c. Hypercalcemia
children who developed same d. Hypocalcemia
symptoms too: Ans: B
a. E. Coli 116) Gastrectomy performed 1.2 years back
b. Giardia now presented with deficiency of:
c. Vibrio Cholera a. Vitamin B12
d. Staph Aureus b. Vitamin B6
Ans: C c. Vitamin B3
SK Origional – Golden 11 34

d. Vitamin B5 123) Subcapsular afferent lymph vessel is a


Ans: A feature of:
117) Mitral valve transplanted presented a. Spleen
with decrease hemoglobin: b. Thymus
a. Macroangiopathic Hemolytic Anemia c. Lymph Node
(MAHA) d. None
b. Microangiopathic Hemolytic Anemia Ans: C
c. IDA 124) Increase serum bilirubin & total
d. Sideroblastic Anemia bilirubin with anemic features:
Ans: A a. Sideroblastic Anemia
118) Ilectomy done will lead to: b. Hemolytic Anemia
a. Increase fat content of stool c. Thalassemia
b. Increase Water content of stool d. IDA
c. Increase Electrolytes in blood Ans: B
d. None 125) Patient presented with shortness of
Ans: B breath & pulmonary edema what to
119) Excessive talking that makes no give?
sense: a. Beta Blocker
a. Wernickes Aphasia b. CCB
b. Brocas‘s Aphasia c. Nitrates
c. Transcortical Aphasia d. Furosemide
d. None Ans: D
Ans: A 126) Appendices epiploic are present in:
Explanation: a. Rectum
 Excessive talking that makes no sense & b. Appendix
defective comprehensive ability in c. Small Intestine
Wernicke‘s Aphasia. d. Sigmoid Colon
 Unable to produce words in Broca‘s Ans: D
Aphasia. 127) CT scan done at L1 level will show:
120) Child with rigors, chills & headache, a. SMA
organism detected on giemsa: b. Portal Vein
a. P.Falciparum c. Hepatic Vein
b. P.Ovale d. Juglar Vein
c. Dengue Fever Ans: A
d. Enteric Fever 128) Old patient with weight loss, patches
Ans: A in lungs, hemoptysis, epitheloid cells
121) Surgeon wants to know the anatomy next step is:
of chest what‟s located in anterior a. AFB
mediastinum? b. CXR
a. Aorta c. HIV Serology
b. Azygous Vein d. CD4 Count
c. Esophagus Ans: A
d. Thymus Explanation: The scenario is related to TB and
Ans: D next best step is AFB to confirm the diagnosis.
122) Patient with Diabetes & Hypertension 129) Lump in hand after mastectomy due
which drug to be prescribed? to:
a. Beta Blockers a. Vascular Invasion
b. Nitrates b. Lymphatic Obstruction
c. CCB c. Viral Infection
d. ACEI d. Pyogenic Condition
Ans: D Ans: B
130) Nerve metabolism is blocked by:
SK Origional – Golden 11 35

a. Cyanide a. Vasoconstriction of Skin vessels


b. Transferrin b. Vasodilation of Skin vessels
c. Free Radicals c. Anterior Hypothalamus
d. Virus d. Preoptic area
Ans: A Ans: D(Ganong +Guyton + FA)
131) Potent analgesic but weak Anesthetic: 139) Supraoptic nucleus damaged what will
a. Halothane be the result?
b. Sevoflurane a. Increase & Dilute Urine
c. Isoflurane b. Decrease & Concentrated Urine
d. NO c. Decrease urine Formation
Ans: D d. None
132) Glucose absorbed by which segment Ans: A
of nephron: Explanation: Damaged to Supraoptic Nucleus
a. PCT will lead to decrease ADH secretion & that in turn
b. DCT will lead to less water absorption so increasing
c. LOH urine amount & making it dilute.
d. CCT 140) O-ve blood group characteristic:
Ans: A a. No A & B Antigen
133) Regarding DCT which of the b. No A & B Antibodies
following is correct: c. Universal Acceptor
a. Glucose absorption occurs here d. Can receive blood from A+Ve
b. ADH act on DCT Ans: A
c. Maximum water reabosroption Explanation: O blood group has no Antigen &
d. Site of action of Thiazide Diuretics having antibodies A & B, Universal donor & can
Ans: B only receive blood from O blood groups.
134) Meckel‟s diverticulum related to: 141) Clear cells on histology organ:
a. Ileum a. Kidney
b. Jejunum b. Lung
c. Colon c. Liver
d. Rectum d. Brain
Ans: A Ans: A
135) Huntington‟s disease: Explanation: Clear cell carcinoma is seen in
a. 50% children affected kidney.
b. Only female affected 142) Cushing triad consists of bradycardia,
c. Only Male affected increase icp and one of the following:
d. 1/3rd of children affected a. Hypotension
Ans: A b. Hypertension
136) Radioisotope scan of thyroid indicated c. Malignancy
in: d. Regular Respiration
a. Cold Nodule Ans: B
b. Hyperthyroidism Explanation: Cushing triad consists of
c. Grave‘s Disease bradycardia, hypertension, increase ICP &
d. Hodgkins Lymphoma irregular respiration.
Ans: A 143) Intrinsic factor is required for
137) Maximum permeability of Cell activation of which vitamin?
membrane to: a. Vitamin B6
a. Electrolytes b. Vitamin B3
b. Water c. Vitamin B12
c. Amino Acids d. Folate
d. Proteins Ans: C
Ans: B
138) Heat loss is controlled by:
SK Origional – Golden 11 36

144) Blood is warmed at 37 degree Celsius c. Niacin Deficiency


& transfused after that on ECG tall d. Folate Deficiency
peaked T wave seen due to: Ans: A
a. Hypokalemia Explanation: Here most likely the scenario was
b. Hyperkalemia related to Beri beri & that‘s due to Vitamin B1
c. Hypocalcemia (Thiamine) Deficiency.
d. Hpercalcemia 151) Lipid contains:
Ans: B a. Phosphorus
Explanation: Stored blood & then ECG findings b. Carbon
those are tall peaked T wave is suggestive of c. Potassium
hyperkalemia. d. Sodium
145) Radiation induced CNS tumour: Ans: B
a. Meningoma 152) Lipoprotein with highest
b. Leukemia phospholipids:
c. Astrocytoma a. HDL
d. GBM b. LDL
Ans: A c. VLDL
Explanation: Leukemia is commonly seen with d. Chylomicrons
radiation but CNS induced tumour after radiation Ans: A
is meningoma. Explanation:
146) Primary CNS tumour:  Lipoprotein with highest
a. Glial Cell Tumour Phospholipids:HDL > LDL > VLDL >
b. Mets Chylomicrons
c. Meningoma  Lipoprotein with highest Protein:HDL >
d. Schwanoma LDL > VLDL > Chylomicrons
Ans: A  Lipoprotein with highest cholesterol:
147) Bitemporal Hemianopsia is due to LDL > VLDL > HDL > Chylomicrons
damage to:  Lipoprotein with highest
a. Optic Nerve TG:Chlylomicrons > VLDL > LDL >
b. Optic Radiation HDL
c. Optic Chiasma 153) Genital branch of genitofemoral nerve:
d. Occipital Cortex a. Gives branches to anterior scrotal wall
Ans: C b. Gives branches to posterior scrotal wall
148) TPR is related to: c. Doesn‘t pass through deep inguinal ring
a. Vascular compliance d. Both A+C
b. Vascular Tone Ans: A
c. Heart Rate Explanation: Genital branch of genitofemoral
d. Stroke Volume nerve supplies anterior part of scrotum & passes
Ans: B through deep inguinal ring.
149) Most potent antioxidant: 154) ACL rupture position of tibia on
a. Vitamin A Femur:
b. Vitamin E a. Anterior
c. Vitamin C b. Posterior
d. Vitamin K c. Lateral
Ans: B d. Medial
Explanation: Follow the following sequence: Ans: A
Glutathione > Vitamin E > Vitamin C > Vitamin 155) S2 heart sound produced by:
A. a. Closure of Mitral valve
150) Scenario related to enlarged heart due b. Close of Tricuspid Valve
to: c. Closure of Aortic + Pulmonary Valve
a. Vitamin B1 deficiency d. None
b. Vitamin B2 Deficiency Ans: C
SK Origional – Golden 11 37

c. Whole Blood
d. Plasma
Explanation: Ans: B
 S1 heart sound is produced on closure of Explanation: The best treatment for Hemophilia
Mitral & Tricuspid Valve. A that‘s classical Hemophilia is factor VIII when
 S2 heart sound is produced on closure of that‘s not available the next best option is
Aortic & Pulmonary Valve. cryoprecipitate?
156) Development of Adenohypophysis: 163) When a test is performed in a disease
a. Rathke‘s Pouch from Dienchephalon person and person is positive for that
b. Outpouching from stomodium then the test is?
c. From telencephalon a. Sensitive
d. None b. Specific
Ans: B c. Positive Predictive Value
157) Mesenphric duct remnant: d. True Negative
a. Ductus Deferns e. True Positive
b. Seminal Vesicle Ans: E
c. Seminiferous tubules 164) 2/3rd of Body fluid:
d. Uterus a. ICF
Ans: A b. ECF
158) Extraembryonic coleom derived from: c. Plasma
a. Epiblast d. Interstitial Fluid
b. Hypoblast Ans: A
c. Lateral Plate Mesoderm Explanation:
d. Intermediate Mesoderm  2/3rd of total body fluid is ICF
Ans: B  1/3rd of total body fluid is ECF
Explanation: 165) Most common cause of pulmonary
 Intraembryonic Coelom: Lateral Plate embolism:
Mesoderm. a. Left Ventricular Failure
 Extraembryonic coelom: Hypoblast. b. Right Heart Failure
 Etraembryonic Mesoderm: Epiblast. c. Thromboembolism
159) Hydrocephalus most common cause: d. Atherosclerosis
a. Aqueductal Stenosis Ans: C
b. 4th Ventricle Obstruction 166) Vital capacity less than 80% in:
c. Lateral Ventricle Obstruction a. Restrictive Lung disease
d. None b. Normal Person
Ans: A c. Tension Pnumothorax
160) Athlete maximum heat loss through: d. Heart Failure
a. Vasoconstriction Ans: A
b. Sweating 167) Platelets adhesion:
c. Shivering a. Normal Endothelium
d. Posterior Hypothalamus b. Rough Endothelium
Ans: B c. Prostacyclins
161) Bee sting which immunoglobulin d. PGs
found? Ans: B
a. IgA 168) Atypical pneumonia feature is seen
b. IgE with:
c. IgM a. Staph Aureus
d. IgG b. S.Pnumoniae
Ans: B c. Mycoplasma
162) Hemophilia A is best treated with: d. TB
a. FFP Ans: C
b. Cryoprecipitate
SK Origional – Golden 11 38

169) Aspiration in supine position the d. CBC


content will go into: Ans: B
a. Left posterior zone 164) Edema/Ascites in liver patient is due
b. Left Lower zone to:
c. Right Posterior Zone Upper lobe a. Decrease Oncotic Pressure
d. Superior Segment of Right lower lobe b. Increase Oncotic Pressure
Ans: D c. Increase Permeability
Explanation: d. Increase interstitial hydrostatic pressure
 Aspiration of FB in supine position will Ans: A
go into superior bronchopulmonary Explanation:In liver failure there‘s decrease in
segment of right Lower Lobe Albumin level & Albumin maintains oncotic
 Aspiration of FB in standing position will pressure that‘s why oncotic pressure will decrease?
go into posterobasal segment of right 165) Pinworm is caused by:
lower lobe. a. Ingestion of larva
 Aspiration of FB in right sided position b. Ingestion of egg of larva
will lodge in posterior bronchopulmonary c. Ingestion of worm
segment of right upper lobe. d. None
170) Squash Player lesser trochanter of Ans: B
femur injured which of the following 166) Vitamin D resistant rickets:
muscle damaged? a. Hyperparathyroidism
a. Psoas b. Fanconi‘s syndrome
b. Sarotrius c. RTA
c. Gluteus medius d. Hypoparathyroidism
d. Gluteus Maximus Ans: B
Ans: A 167) HIV patient floor disinfection after
Explanation: Psoas muscle is attached to lesser surgery:
trochanter of femur and will be damaged if lesser a. 1% hypochlorite
trochanter is damaged. b. 4% hypochlorite
171) BP 160/110 mmHg & increase renin is c. Gluteraldehyde
due to: d. Soap & Water
a. Sympathetic Renal nerve e. Water
b. Parasympathetic Renal Nerve Ans: A
c. Hypokalemia Explanation:
d. Hypocalcemia  Floor: 1% Hypochlorite
Ans: A  Instruments: 2% Gluteraldehyde
172) A patient with abdominal pain, 168) BCG Vaccine:
bloating and foul smelling most useful a. T cells Delayed Response
investigation would be: b. B Cells
a. Stool DR c. Plasma Cells
b. Culture d. Immunoglobulin‘s
c. Gram staining Ans: A
d. Colonoscopy 169) Referral System:
Ans: A a. Two way communication between health
Explanation: Stool D/R (Detailed Report) gives facilities
information about all stool contents foul smelling b. Higher Hospitals are better
stools are due to fat malabsorption i.e. Steatorrhea c. To decrease burden
& culture and sensitive has nothing with it. d. For higher procedure
163) Child which Jaundice which Ans: A
investigations need to be performed? 170) In hemorrhage patient aldosterone
a. Bilirubin level increased by:
b. ALT + Bilirubin a. ADH
c. Serum Ferritin b. Oxytocin
SK Origional – Golden 11 39

c. AG2 problem what other factor you will see


d. Vasopressin in this condition:
Ans: C a. Low urinary sodium
171) Central chemoreceptors respond to: b. Increase plasma osmolarity
a. Increase PCO2 c. Decrease plasma osmolarity
b. Decrease PCO2 d. Fixed specific gravity
c. Increase PO2 Ans: A
d. Decrease PO2 178) Complete ventricular depolarization is
Ans: A seen in:
Explanation: a. QRS
 Central Chemoreceptors responds to: b. ST Segment
Increase H+ ions in CSF > Increase c. PR Segment
PCO2> Decrease PO2. d. PR Interval
 Peripheral Chemoreceptors responds to: Ans: B
Decrease PO2>Increase PCO2. 179) Increase in heart rate what will be
172) Defense against tumour: decreased?
a. T cells a. Slow ventricular filling phase
b. B Cells b. Slow ejection phase
c. Plasma Cells c. Rapid ejection phase
d. NK Cells d. Atrial systole
Ans: D Ans: A
173) Epicardium: 180) Childhood tumour with gene
a. Visceral Layer of Serous Pericardium amplification:
b. Parietal Layer of Serous Pericardium a. Retinoblastoma
c. Parietal Layer of Myocardium b. Neuroblastoma
d. None c. Wilms Tumour
Ans: A d. RCC
174) Minor calyx receives urine from: Ans: B
a. Major Calyx 181) Transitional epithelium of bladder is
b. Papilla derived from:
c. Ureter a. Endoderm
d. DCT b. Mesoderm
e. Pelvis c. Endoderm +Mesoderm
Ans: B d. Ectoderm
175) Half-life of drug depends on: Ans: A
a. Dose Explanation:
b. Rate of Distribution  Transitional epithelium of bladder is
c. Rate of Clearance derived from endoderm.
d. Volume of Distribution  Trigone of bladder is derived from
Ans: C mesoderm.
176) Axoneme of cilia contains how many  Epithelium of bladder is derived from
microtubules: both endoderm and mesoderm.
a. 10 182) Regarding phospholipids:
b. 20 a. Insoluble in Water
c. 30 b. Soluble in Water
d. 22 c. Backbone is made by Lipids
Ans: B d. Backbone is made by Chloride
Explanation: 09 doubles and 02 single so total Ans: A
20. 183) Main stimulus for aldosterone release:
177) Patient passing dilute urine, there‟s a. Hyperkalemia
tubuloglomerular concentration b. Hypokalemia
c. Increase Renin
SK Origional – Golden 11 40

d. Decrease Renin
Ans: A
184) Pancreatic fluid accumulates in: SURGERY & ALLIED
a. Paracolic gutter November 25th, 2020 – Morning
b. Subphrenic Space
c. Lesser Sac
1) School Children presented with
d. Subdiaphramatic Space
bloody diarrhea containing mucus is
Ans: C
most likely caused by which of the
185) Highest energy compound:
following:
a. Starch
A) Shigella
b. ATP
B) Vibrio Cholera
c. ADP
C) Salmonella
d. Camp D) Streptococcus
Ans: B
Ans: A
Explanation:
Explanation: Shigella is the leading cause of
 Highest energy compound/molecule: childhood diarrhea, the manifestation can be both
ATP in form of watery diarrhea or dysentery that‘s
 Highest Energy content: Starch diarrhea with fresh blood & mucus.
186) Fat content in adult male is: 2) Infection of neck in front of
a. 15% pretracheal fascia will spread to:
b. 17% A) Anterior mediastinum
c. 40% B) Superior mediastinum
d. 45% C) Inferior Mediastinum
Ans: A D) Posterior Mediastinum
Explanation: Non Aethlete Male has 15% and Ans: A
Non Aethlete Female has 27%(Guyton) Explanation:
187) Most common Leukemia in children:  Infection infront of pretracheal fascia
a. ALL spreads to: Anterior Mediastinum
b. CLL
 Infection posterior prevertebral fascia
c. AML
spreads to: Superior Mediastinum.
d. CML 3) Bronchogenic carcinoma metastasize
Ans: A
most commonly to which of the
following endocrine gland:
A) Pituitary Gland
===============
B) Adrenal Gland
C) Thyroid Gland
D) Parathyroid Gland
Ans: B
Explanation: Adrenal gland is the commonest
endocrine gland site for metastasis of the
bronchogenic carcinoma that‘s 22% followed by
Pituitary gland.
4) Female hand cut with knife, doesn‟t
heal properly since last two weeks,
presented with fever, Hb 8gm/dl, RBS
130 mg/dl what‟s the probable cause:
A) Anemia
B) Protein Deficiency
C) Diabetes Mellitus
D) Infection
Ans: D
SK Origional – Golden 11 41

Explanation: Ans: B
 Infection is one of the commonest cause Explanation: Anthrax is a bacterial disease of
of delayed wound healing & here fever is animals most commonly seen in cattle, sheep and
a clue toward infection. goats.
 DM is the systemic cause of 9) Shigella causes dysentery by
delayedwound healing. mechanism of:
5) Which of the following is the A) Mucosal Invasion
lymphatic drainage of external nose? B) Cell Wall
A) Upper Deep cervical Lymph Nodes C) Proteoglycan
B) Retropharyngeal Lymph Nodes D) Phospholipid
C) Submandibular Lymph Nodes Ans: A
D) Parotid Lymph Nodes Explanation: Shiga toxin causes GI mucosal
Ans: C invasion & damage leading to dysentery.
Explanation: 10) Knee infection in a young person is
 External Nose & Anterior nasal cavity: most likely caused by:
Submandibular Lymph Nodes A) Streptococcus
 Posterior Nasal Lymph Nodes: B) Hemophilus
Retropharyngeal Lymph nodes & JD C) Staph Aureus
Nodes D) N.Gonorrhea
 Remaining Nasal cavity: Upper Deep Ans: C
cervical lymph nodes Explanation: Staph Aureus is the leading cause
6) Most common tumor of Salivary of septic arthritis that commonly involves knee
Glands: joints.
A) Parotid Gland 11) Which of the following is
B) Sublingual characteristic of bacteroids infection?
C) Submandibular A) Can‘t cause abscess
D) Mixed B) Foul smelling discharge
Ans: A C) Not part of normal flora
Explanation: D) Fruity smell
 Most common Salivary gland tumor is Ans B
pleomorphic adenoma of Parotid gland Explanation: Bacteroids infections can cause
 Most common malignant tumor of abscess, the discharge is foul smelling and is part
salivary gland is mucoepidermoid of GIT normal flora.
carcinoma. 12) A patient had trauma to femur now
7) Stenson duct is present in: presented with bony growth in soft
A) Mandibular Space tissue due to:
B) Sublingual Space a. Metaplasia
C) Mandibular Angle b. Neoplasia
D) Buccal Space c. Dysplasia
Ans: D d. Abscess
Explanation: The main duct of the parotid gland Ans: A
(Stenson‘s duct) courses in transverse fashion Explanation: This is the case of“myositis
through the buccal fat pad and it pierces the ossificans” is essentially metaplasia of the
buccinators muscle opposite the second maxillary intramuscular connective tissue resulting in extra
molar. osseous bone formation without inflammation.
8) A farmer contacted a disease from 13) Breast mass with irregular shape
sheep & the skin lesion is ulcerated that‟s non mobile & 3cm in size with
black most likely is: skin involvement will look on gross:
A) Plaque a. Metaplasia
B) Anthrax b. Dysplasia
C) Pneumoconiosis c. Desmoplasia
D) Leishmania d. Anaplasia
SK Origional – Golden 11 42

e. Hyperplasia B) Rarely affect male


Ans: C C) Protects against P.Falciparum
14 Which of the following is the D) Genetic disorder
landmark muscle of the neck? Ans: B
A) SCM Explanation: G6PD is a genetic disorder that‘s
B) Scalenus Anterior X-Linked & commonly affects male & is
C) Anterior belly of digastric muscle protective against P. Falciparum.
D) Posterior belly of digastric muscle 20) Little girl developed purpura after
Ans: A Vaccination:
Explanation: The land mark muscles of neck: a. TTP
SCM > Scalenus Anterior. b. ITP
15) Medial canthal ligament attachments: c. VWD
A) Ethmoid & Frontal bones d. Hemophilia
B) Lacrimal Bone & Frontal process of Ans: B
Maxilla 21) Patient presented with generalized
C) Sphenoid Bone weakness, easy bruisibility, fever, Hb
D) Mandible 6gm/dl, Platelets 50,000, TLC 3000 is
Ans: B most likely suffering from:
16) Which of the following is Anti-Cancer A) ITP
& Nephrotoxic drug? B) Dengue
A) Cyclosporin C) Aplastic Anemia
B) Cyclophosphamide D) CML
C) Cisplatin E) IDA
D) Azithromycin Ans: C
Ans: C Explanation: As the patient has decreased
Explanation: Cisplatin is both anti-cancer Hemoglobin, Platelets count & TLC that‘s
& Nephrotoxic drug. pancytopenia and is commonly seen in aplastic
17) A pregnant woman used some anemia.
medications for Acne & developed 22) Which of the following is part of
congenital anomaly in baby is most respiratory conducting portion?
likely due to: A) Trachea
A) Vitamin A B) Alveoli
B) Vitamin B C) Alveolar Duct
C) Vitamin C D) Alveolar Sac
D) Folate Ans: A
Ans: A Explanation: Trachea is part of conducting
Explanation: Commonly prescribed drug for portion of respiratory system while Alveolar Duct,
acnes is Isotretenoin that‘s derivative of Vitamin Alveoli & Alveolar Sac are part of Respiratory
A and is teratogenic in nature. portion.
18) Which of the following causes 23) Alveoli doesn‟t easily collapse due to
thrombus formation? the presence of:
A) Decrease Blood flow A) Surfactant
B) Decrease Viscosity B) Lipids
C) Endothelial damage C) Steriods
D) Increase blood flow D) Collagen Fibers
Ans: C Ans: A
Explanation: The Virchow‘s triad of thrombosis 24) Terminal Bronchioles have which of
consist of stasis of blood flow, hypercoagulability the following cells:
& Endothelial Injury. A) Goblet Cells
19) Which of the following isn‟t true B) Clara Cells
regarding G6PD deficiency? C) Hair Cells
A) X-Linked Disease D) Sweat Gland
SK Origional – Golden 11 43

Ans: B  Acute Hyperkalemia & Hypokalemia both


25) Which of the following structure leads to flaccid paralysis of heart
passes through foramen rotundum: 30) Initial, most common side effect of
A) Opthalmic Nerve Lidocaine:
B) Maxillary Nerve a. Treated with hydrocortisone
C) Mandibular Nerve b. Given as 2% solution to avoid adverse
D) Vagus Nerve effect
Ans: B c. Causes convulsions before cardiac toxicity
Explanation: for several hours
 The maxillary branch of trigeminal d. Perioral parasthesia
nerve passes through foramen Ans: D
rotundum. Explanation:
 The mandibular branch of trigeminal  Initial : Perioral Parasthesia
Nerve passes through foramen Ovale.  Later on: Convulsion
26) Apex of Heart blood supply: 31) Mixed blood obtained from:
A) Marginal Artery A) Right atrium
B) Anterior Interventricular Artery B) Right ventricular
C) Circumflex Artery C) Pulmonary vein
D) PAD D) Pulmonary artery
Ans: B Ans: D
27) Major supply of Interventricular 32) True regarding action potential:
septum: A) Subsequent action potential can summate
A) Anterior Interventricular Artery B) With more stimulation action potential
B) Posterior Interventricular Artery can occur above threshold
C) Circumflex Artery C) More amplitude of stimulus can increase
D) RCA amplitude of action potential
Ans: A D) None of the above
Explanation: Major portion of interventricular Ans: A
septum that‘s anterior 2/3rd is supplied by anterior 33) Malignant hyperthermia occurs due to
interventricular septum while posterior 1/3rd is change in which of the following?
supplied by posterior interventricular artery. A) Sodium
28) Extrasystole means: B) Potassium
A) Heart is filled with more blood C) Calcium
B) Heart is filled with less blood D) Magnesium
C) Extra Beat Ans: C
D) Cardiac dilatation 34) Hyperkalemia is caused by:
Ans: C A) Exercise
Explanation: A premature contraction of the B) Beta Agnoist
heart that is independent of the normal rhythm of C) Aldosterone
the heart and that araises in response to an D) Renin
impulse in some parts of the heart other than the Ans: A
normal impulse that‘s from SA node. Explanation: During exercise more potassium
29) Hyperkalemia leads to which of the comes out from intracellular to extracellular fluid.
following: 35) Growth hormone needs which of the
A) Decrease Action Potential following to increase maximum effect:
B) Flaccidity A) Insulin like growth factor
C) Tachycardia B) Cortisol
D) Powerful contraction C) Aldosterone
Ans: B D) Steroids
Explanation: Ans: A
 Hyperkalemia leads to suppression of SA
node & thus causes bradycardia
SK Origional – Golden 11 44

Explanation: Growth Hormone needs Insulin supination of arm at elbow joint due
like growth factor (Somatomedian-C) to exert to damage to:
maximum effect. A) Upper Trunk of Brachial Plexus (C5-C7)
36) Slow wave is characterized by: B) Lower Trunk of Brachial Plexus
A) Low frequency waves C) Radial Nerve
B) Teeth grinding D) Ulnar Nerve
C) Active dreaming Ans: A
D) NREM Explanation: In upper trunk of brachial plexus
Ans: A there is mostly damage to musculocutaneous,
Explanation: axillary and suprascapular nerve that leads to loss
 Slow wave: Delta waves low frequency of abduction, flexion & supination of forearm.
and high amplitude. 42) Lateral cord of brachial plexus
 Teeth grinding in stage 2 of NREM. supplies which of the following
37) Aldosterone exerts its maximum effect muscle?
on which of the following site? A) Biceps Brachii
A) PCT B) Deltoid
B) LOH C) Serratus Anterior
C) DCT D) Diaphragm
D) CCT Ans: A
Ans: D Explanation: The lateral cord of brachial plexus
Explanation: is made by confluences of anterior division of
 Aldosterone acts on: CCT > DCT > CT upper & middle trunk of brachial plexus and
 ADH acts on : CT > DCT > CCT musculocutaneous nerve one of its branch
38) Stability of knee joint is by: supplies bicep brachii.
A) Vastus Medialis 43) Gastrin secretion is decreased by:
B) Vastus Lateralis A) Somatostatin
C) Sarotrius B) Insulin
D) Adductor C) Glucagon
Ans: B D) Acetylcholine
Explanation: Ans: A
 Stability of knee joint is by Vastus 44) Gastric acid secretion is maximally
Lateralis stimulated by:
 Stability of Patella is by Vastus Medialis A) Protein
39) Which of the following nerve supply B) Carbohydrates
adductor longus: C) Vitamins
A) Posterior division of Obturator nerve D) Lipids
B) Anterior division of obturator nerve Ans: A
C) Femoral nerve Explanation: BRS physiology states that the most
D) Long thoracic nerve potent stimulus for gastrin secretion is
Ans: B phenylalanine & tryptophan which are amino
40) Supply of lateral skin of knee: acids and peptides (protein).
A) Obturator nerve 45) Dermatome supply of lateral hand:
B) Femoral nerve A) T1
C) Sciatic Nerve B) C8
D) Common perioneal nerve C) C6
Ans: B D) C7
Explanation: The innervation to the lateral knee Ans: C
skin is variable from either the lateral femoral 46) Hand muscles are supplied by:
cutaneous nerve or branches of the femoral nerve. A) Radial nerve
41) A baby after an obstructed delivery B) Ulnar Nerve
presented with inability to abduct the C) Median Nerve
arm, loss of flexion & loss of D) Axillary Nerve
SK Origional – Golden 11 45

Ans: B 51) Posterior arcuate fibers are called:


Explanation: Small muscles of hand is supplied A) Spinotectal tract
by both Ulnar & Median Nerve but if one to B). Ceunocerebellar tract
choose then Ulnar > Median. C) Lateral Tract
47) A patient can‟t abduct & Adduct his D) Medial Leminiscus
fingers is due to damage of which of Ans: B
the following nerve: 52) After pelvic fracture patient went into
A) Ulnar Nerve shock due to:
B) Median nerve A) Hemorrhage
C) Radial Nerve B) Air Embolism
D) Axillary Nerve C) Pulmonary Embolism
Ans: A D) Cardiogenic Shock
Explanation: Both DAB: Dorsal interosseous Ans: A
causing abdution & PAD: Palmar interoseous Explanation:
causing adduction are supplied by deep branch of  Hemorrhage due to femoral artery or iliac
the ulnar nerve. artery rupture.
48) Radial Pulse is palpated between  For Air embolism there will be history
which tendons: procedure like passing CVP then
A) Between abductor pollicis brevis & longus developed SOB.
B) Between abductor pollicis longus &  For pulmonary embolism there will be
extensor pollicis longus history of stasis like patient with long
C) Between adductor longus & abductor bones fracture was operated and on 3rd
brevis day developed sudden SOB that‘s more
D) Between abductor brevis & flexor tendon like pulmonary embolism.
Ans: B 53) Olfactory nerve cells repair or change
Explanation: Radial Pulse is mainly in every 2 weeks:
palpated.Palpated between the tendons of A) Basal cells
brachioradialis & flexor carpi radialis, but this B) Supporting cells
option wasn‘t in the board exam so the next close C) Bipolar cells
one is the site of snuff box where radial artery can D) None of the above
be palpated between the tendons of abductor Ans: A
pollicis longus & extensor pollicis longus. 54) Commonest brain tumor:
49) Pain receptor in which tract: A) Astrocytoma
A) Lateral spinothalamic tract B) Meningoma
B) Anterior spinothalamic tract C) Medulloblastoma
C) Lateral spinocerebellar tract D) Pineal gland tumor
D) DCML Ans: A
Ans: A Explanation:
Explanation:  Commonest brain tumor is: Astrocytoma
 Lateral spinothalamic tract transmits pain  Commonest brain tumor in children is:
& temperature. Medulloblastoma
 Anterior spinothalamic tract transmits 55) Niacin deficiency can lead to:
crude touch & firm pressure. A) Lesion of CNS
 DCML transmits fine touch, vibration & B) Lesion in PNS
proprioception. C) Lesion in vascular system
50) Which of the following is slow D) Defect in some pathway coenzymes
adapting receptor: Ans: A> D
A) Hair Follicle 56) Young boy with epistaxis, mother told
B) Pacinian other female and male both in family
C) Messiner had these signs PT, APTT and BT is
D) Pain receptor raised:
Ans: D A) VWD
SK Origional – Golden 11 46

B) Hemophillia A  135 + 4 – (15 + 100)


C) Christmas disease  139 – 115
D) Platelet dysfunction  24
Ans: A 62) PH = 7.49, PCO = 60, HCO3 =30
Explanation: As APTT & BT is raised and both which of the following Acid Base
male & female members of the family are affected abnormality does it indicates?
so this is most likely VWD that‘s Autosomal A) Respiratory acidosis
dominant. B) Partially compensated respiratory acidosis
57) Which of the following deficiency C) Partially compensated metabolic alkalosis
leads to bleeding? D) Mixed acidosis
A) Vitamin A Ans: C
B) Vitamin B Explanation: As the PH is in alkalosis range now
C) Vitamin C we look toward the HCO3 that‘s in alkalosis range
D) Vitamin D too so we keep metabolic alkalosis in mind, after
Ans: C looking to PCO2 that‘s in Acidotic range shows
58) Fibrinogen is converted to Fibrin the struggle toward compensation but not fully
which of the following product is compensated yet hence, this is case of partially
released? compensated metabolic alkalosis.
A) Fibrinopeptide A 63) An 18 Years old girl with bicornuate
B) FibrinoPeptide B uterus is most likely due to which of
C) Laminin the following abnormality:
D) Cadherin A) Paramesonephric duct
Ans: A B) Mesonephric duct
Explanation: Fibrionpeptide A is a 16-Amino C) Wolffian duct
acid cleavage product of thrombin induced D) Tubule
proteolytic cleavage of fibrinogen. Ans: A
59) A young boy came after injury; BP Explanation: Paramesonephric duct (Mullerian
Test shows tetany other test isnormal: Duct) give raise to female genital structures like
A) Vit. D deficiency Fallopian tube, Uterus & upper 1/3rd of Vagina.
B) Osteoporosis 64) Difference between sarcoidosis & Tb:
C) Primary hyper parathyroid A) Symmetrical B/L hilar lymphadenopathy
D) Hyperthyroidism B) Fever
Ans: A C) Constitutional symptoms
60) A middle aged woman with increased D) Night Sweats
TSH, Low T3 & T4 and having Ans: A
overweight most likely due to: 65) Drug of choice for T.Solium:
A) Over eating A) Albendazole
B) Hypothyroidism B) Niclosamide
C) Hyperthyroidism C) Metronidazole
D) Hypopituitarism D) Captopril
Ans: B Ans: B
Explanation: Increase TSH, Low T3 and T4 with Explanation:Drug of choice for T.Solium is
overweight is most likely due to hypothyroidism. prizaquental, but that wasn‘t in options the next
61) Calculate the anion gap for HCO3 15 best choice in Levinson is Albendazole while in
meq, Na 135, Cl 100 & K+4: Katzung Pharmacology book it‘s Niclosamide so
A) 20 we prefer Katzung.
B) 24 66) Which of the following is defective in
C) 30 Marfan syndrome:
D) 45 A) Collagen
Ans: B B) Fibrillin
Explanation: C) Elastin
 Anion Gap = Na + K – (HCO3 + Cl) D) Hydroxyproline
SK Origional – Golden 11 47

Ans: B 73) Mononuclear phagocytes cells derived


67) Which of the following collagen is from:
found at the basement membrane? A) Bone marrow
A) Type 1 B) Spleen
B) Type 2 C) Lymph node
C) Type 3 D) Thymus
D) Type 4 Ans: A
Ans: D 74) Most important factor associated with
68) Transitional epithelium characteristic: malignancy:
A) Dome Cells A) Grading
B) Flat Cells B) Staging
C) No change in cellular layer C) Invasion
D) Keratinized covering D) None of the above
Ans: A Ans: B
69) Nissel bodies are present in: 75) Which of the following method is used
A) RER for circulating tumor cells:
B) SER A) Peripheral Blood smear
C) Mitochondria B) Tumor markers
D) Axon Hillock C) Biopsy
Ans: A D) None of the above
Explanation: Nissel bodies are maximally found Ans: A
in RER & absent in axon hillock. Explanation:
70) Which of the following  PBS is used for tumor cells.
volume/capacity in the lungs after  Tumor markers the commonest way to
normal tidal volume expiration? detect any tumor.
A) Residual volume 76) Nucleus of tractus solitarius contains:
B) FRC A) 1st Order Neuron
C) ERV B) 2nd Order Neuron
D) IRV C) 3rd Order Neuron
Ans: B D) None
71) Variance is related to: Ans: B
A) Variability 77) Pt. came with dysphagia, polyuria &
B) SD sleeping disorders lesion in:
C) Efficacy A) Hypothalamus
D) Potency B) Subthallamus
Ans: A C) Pituitary
Explanation: D) Adrenal
 Variance is related to variability. Ans: A
 Standard deviation is the square root of 78) Retired officer came with c/o tremors
variance. while doing small work! Gait
72) Confidence interval is: abnormality and past pointing, when
A) Mean & its standard error asked to touch nose, Lesion in:
B) Standard error of mean & T-value A) Basal ganglia
C) Mean & Mode B) Cerebellum
D) Median & Standard Error C) Cerebrum
Ans: A D) Pons
Explanation: Ans: B
 Confidence interval: Mean & its standard 79) Structure preventing damage from
error. rotatory movement of head:
 Confidence limit: Standard error of mean A) Falx cerebri
& T-value. B) Tentorium cerebelli
C) Vessels
SK Origional – Golden 11 48

D) Sinus Explanation: Melatonin controls pigmentation


Ans: A changes by aggregation of melanin into the
80) A man with presented stroke, ACA is melanocytes within the skin causing the skin to
affected; it will affect which area of change colour.
brain: 86) Which of the following hormone acts
A) Somatosensory Area on NA-K ATPase:
B) Brocas Area A) Insulin
C) Occipital Area B) Glucagon
D) Primary Motor area C) Cortisol
Ans: A D) GH
81) Child blood transfusion after 10 days Ans: A
develops rash jaundice, diarrhea& 87) In DCML lesion which of the
increased ALT cause: following modality will be lost:
A) Graft verses host disease A) Pain
B) CMV infection B) Temperature
C) Acute hemolytic disease C) Proprioception
D) HSR Type 1 D) Crude touch
Ans: A Ans: C
82) A patient came with newly developed Explanation: DCML carries proprioception, Fine
murmur he gives history of having touch & Vibration.
rheumatic fever in childhood, what is 88) Which of the following is the effect of
the cause: catecholamines after injury:
A) Mitral stenosis A) Vasodilation
B) Aschoff bodies B) Vasoconstriction
C) Papillary muscle damage C) Increase Permeability
D) Aortic Regurgitation D) None of the above
Ans: A Ans: B
83) Drainage of ischiorectal fossa, 89) Bioavailability of a drug increases if it
structure damage is: is:
A) Pudendal artery A) Hydrophobic & Lipophilic
B) Inferior rectal nerve B) Lipophobic
C) Perineal nerve C) Hydrophilic
D) Internal Iliac artery D) None of the above
Ans: B Ans: A
84) Patient with chronic hypertension 90) First pass effect can be avoided
with decreased visual acuity changes through which route?
in fundoscopy is most likely in which A) Sublingual
of the following: B) IM
A) Retinal nerves C) PO
B) Vein D) PR
C) Arteries Ans: A
D) Arterioles Explanation: IV > SL > IM
E) Pigment epithelium 91) Patient with chest pain, O2& NG
Ans: D given next best step is:
85) Melatonin regulated which of the A) Coronary Angiography
following: B) Medications
A) Pigmentation C) Trade mill exercise test
B) Taste D) Enzyme Level
C) Vision Ans: D
D) Visual Acuity 92) Largest zone of the prostate:
Ans: A A) Peripheral
B) Central
SK Origional – Golden 11 49

C) Median Ans: A
D) Lateral Explanation: The transverse cervical artery is the
Ans: A next branch off the thryocervical trunk.
Explanation: 99) Deep inguinal canal relation with
 Largest zone is peripheral. inferior epigastric artery:
 Structurally largest lobe is median. A) Lateral
 Anatomically largest lobe is lateral. B) Medial
93) Vertebral column is held in its place C) Posterior
by which ligament: D) Anterior
A) Ligamentum Flavum Ans: A
B) Anterior ligament 100) Terminal part of common bile duct
C) Alar Ligament relation with pancreas:
D) Anterior & Posterior Longitudinal A) Anterior to it
ligament B) Embedded in it
Ans: D C) Lateral
94) Rectus abdominis nerve supply: D) Media
A) Last 5 intercostal and subcostal nerves Ans: B
B) Iliohypogastric nerve 101) A child aspirated peanut (position
C) Ilioinguinal nerve wasn‟t mentioned in 2nd stem) it will
D) Obtruator nerve lodge in:
Ans: A A) Right upper lobe
95) Artery arising from posterior aspect of B) Right lower lobe
bifurcation of aortic arch: C) Right middle lobe
A) Median sacral D) Left upper lobe
B) Middle aortic Ans: B
C) Common ilac 102) Perforation of posterior wall massive
D) External iliac bleeding due to:
E) Internal iliac A) Gastro duodenal artery
Ans: A B) Right Gastric Artery
Explanation: The Median sacral artery or middle C) Left Gastric Artery
sacral artery is a small vessel that arises from D) Right Colic Artery
posterior to the abdominal aorta & superior to its Ans: A
bifurcation. Explanation:
96) Maximum pressure in aorta during:  Perforation of anterior wall of duodenum
A) Rapid ejection phase leakage of content into greater sac, right
B) Reduced ejection phase paracolic gutter & right iliac fossa.
C) Isovolumetric contraction  Perforation of posterior wall massive
D) Isovolumetric relaxation bleeding due to gastro duodenal artery.
Ans: B 103) Magna Artery is a branch of:
97) Highest systolic pressure seen in: A) Splenic Artery
A) Aorta B) Gastro duodenal Artery
B) Renal Artery C) Celiac Trunk
C) Pulmonary Artery D) Aorta
D) Radial Artery Ans: A
Ans: B Explanation: The greater pancreatic artery also
98) During surgery profuse bleeding from known as pancreatic magna artery is a branch of
transverse cervical artery which of the the splenic artery that supplies the pancreatic tail
following to be ligated: and body.
A) Thyrocervical Trunk 104) Collateral branch of intercostals nerve:
B) Aorta A) Gives lateral cutaneous branch
C) Vertebral Artery B) Runs in subcostal space
D) Carotid Artery C) Run in intervertebral space
SK Origional – Golden 11 50

D) Supply muscle B) Vertical group of superficial inguinal


Ans: A nodes
Explanation: Each intercostal nerve gives off a C) Horizontal group of lymph nodes
collateral branch to the inferior part of the D) Common iliac nodes
intercostal space and a lateral cutaneous branch to Ans: B
the side of the chest. In addition to being 110) How many days do a full sperm
distributed to muscle and skin, branches are given regeneration cycle takes:
to the parietal pleura, mammary gland and A) 20 days
periostum of the ribs. B) 25 days
105) Which of the following Hepatitis has C) 64 days
most lethal effect during pregnancy: D) 80 days
A) Hep. A Ans: C
B) Hep. B 111) Breast Lateral Lymphatic drainage:
C) Hep. C A) Pectoral (Axillary) Lymph Nodes
D) Hep. D B) Posterior Axillary Lymph Nodes
E) Hep. E C) Internal Thoracic Lymph Nodes
Ans: E D) Supraclavicular Lymph Nodes
Explanation: E) Infraclavicular Lymph Nodes
 Hepatitis A is more common than Ans: A
Hepatitis E even in pregnancy. Explanation: Lateral lymphatic drainage of breast
 But, the most lethal hepatitis in pregnancy is mainly to pectoral lymph nodes & medical to
is Hep. E. internal thoracic lymph nodes.
106) Which of the following is true 112) Lateral cord of brachial plexus
regarding virus: supplies which of the following
A) They contain both RNA & DNA muscle:
B) HIV virus is a DNA virus A) Pectoralis major
C) CMV is a DNA virus B) Deltoid
D) All herpes viruses are sensitive to C) Serratus Anterior
acyclovir D) Intercostal muscle
Ans: C Ans: A
Explanation: 113) A 60 years old known diabetic patient
 Viruses contain only DNA or RNA not for last 20 years presented with rapidly
both of them progressive renal failure, most likely
 HIV virus is an RNA virus cause is:
 CMV is a DNA virus A) Amyloidosis
107) Which of the following is the B) Diffuse glomeruloscelrosis
karyotype of turner syndrome: C) Kimmel stein wilson syndrome
A) 45 XO D) UTI
Ans: C
B) 46 XX
114) Manibro-sternal joint type:
C) 46 XY A) Primary cartilaginous
D) XXY B) Secondary cartilaginous
Ans: A C) Symphysis
108) Lymphatic drainage of Appendix: D) Syndesmosis
A) Superior mesenteric lymph nodes Ans: B
B) Inferior mesenteric lymph nodes 115) Most common fungal opportunistic
C) Iliac nodes infection in HIV:
D) Celiac nodes A) Pneumonia
Ans: A B) Candida
109) Lymphatic drainage of big toe: C) Pseudomonas
A) Popliteal nodes C) Cryptococcus
Ans: B
SK Origional – Golden 11 51

Explanation: 121) In Factor 8 deficiency when factor 8


 Most common opportunistic infection isn‟t available, which of the following
in HIV: TB. is administered:
 Most common fungal opportunistic A) Cryoprecipitate
infection in HIV: Candida. B) Platelets
116) Drug that causes nephrotoxicity but C) Packed red Blood Cells
does not cause bone marrow D) None of the above
suppression in liver transplant patient: Ans: A
A) Cyclophosphamide 122) Reed – Sternberg cells is found in
B) Azathioprine which of the following:
C) Prednisone A) HIV
D) Cyclosporine B) Non – Hodgkins
Ans: D C) Hodgkins lymphoma
117) Dystrophic calcification commonly D) IM
occurs in: Ans: C
A) Pancreas 123) Liver is derived from:
B) Renal Pelvis A) Ventral foregut
C) Renal tubules B) Dorsal foregut
D) Necrotic Tissue C) Midgut
Ans: D D) Hindgut
118) Which of the following anemia is seen Ans: A
in ESRD: 124) Drainage of ischiorectal fossa,
A) IDA structure damage is
B) Anemia due to erythropoietin deficiency A) Pudendal artery
C) Megaloblastic anemia B) Inferior rectal nerve
D) Sideroblastic anemia C) Perennial nerve
Ans: B D) Internal Iliac artery
Explanation: Erythropoietin is secreted by Ans: B
peritubular renal capillaries & that‘s decreased in 125) Epinephrine decreases which of the
ESRD thus leading to anemia. following:
119) Decrease serum calcium, increase A) Heart rate
serum phosphate and decrease urinary B) Cardiac contractility
calcium seen in: C) Cutaneous blood flow
A) Hypoparathyroidism D) Bronchi diameter
B) Hyperparathyroidism Ans: C
C) Hypothyroidism Explanation: Epinephrine increases HR, Cardiac
D) Hyperthyroidism contractility, bronchodilation & decreases
Ans: A cutaneous blood flow.
Explanation: PTH increases serum calcium and 126) Successful assessment of stable
decrease serum phosphate level and when there‘s hemodynamic state after resuscitation:
hypoparathyroidism serum calcium will decrease A) Increase Urinary Output
& serum phosphate will increase. B) Increase Blood pressure
120) A patient with increase APTT & BT is C) Increase pulse
seen in which of the following D) Decrease Blood pressure
condition: Ans: A
A) DIC 127) Type III HSR is seen in which of the
B) VwD following condition:
C) Hemophilia A) TB
D) ITP B) SLE
Ans: B C) RA
D) Atopic Asthma
Ans: B
SK Origional – Golden 11 52

128) Which of the following is D) Aspirin


paraneoplastic syndrome of kidney: Ans: A
A) Polycythemia Explanation: CCL4 produces free radicals and
B) Carcinoid Syndrome leads to hepatocytes damage.
C) PrRP 134) Which of the following is muscle of
D) ADH inspiration in quiet respiration:
Ans: A A) External Intercostal
Explanation: Renal system mainly secretes. B) Internal Intercostal
Erythropoietin that leads to Polycythemia & is C) Diaphragm
considered as paraneoplastic syndrome. D) Serratus Anterior
129) A patient presented with low grade Ans: C
fever, night sweats & productive 135) Diplotene stage in which phase:
cough most likely due to: A) Prophase
A) TB B) Metaphase
B) AIDS C) Interphase
C) Sarcoidosis D) Telophase
D) Histoplasmosis Ans: A
Ans: A Explanation: All primary oocytes are formed by
130) Hippocratic Oath clearly describes: the fifth month of fetal life and remain dormant in
A) Sexual boundaries prophase (Diplotene stage) of meiosis I until
B) Advertisement puberty.
C) Confidentiality 136) Which of the following act on Na-K
D) Doctors right ATPase:
Ans: C A) Insulin
131) 80 years old developed difficulty in B) ADH
walking with increased MCV is most C) Adrenaline
likely suffering from: D) Aldosterone
A) Megaloblastic Anemia Ans: A
B) Iron deficiency Anemia 137) Philadelphia chromosome
C) Sidereoblastic Anemia translocation 9:22 is present in which
D) Thalassemia of the following:
Ans: A A) CML
Explanation: B) ALL
 Megaloblastic anemia is associated with C) AML
increased MCV & subacute combined D) CLL
degeneration of spinal cord that leads to Ans: A
difficulty in walking. 138) Anti DsDNA positive along with
132) Which of the following has shortest hematuria & foamy urine is most
pre erythrocytic phase: likely:
A) Malariae A) UTI
B) Ovale B) Bladder carcinoma
C) Vivax C) Lupus nephritis
D) Falciparum D) Acute pyelonephritis
Ans: D Ans: C
Explanation: Explanation: Lupus Nephritis is inflammation
 Shortest: Falciparum of the kidney that is caused by SLE &
 Longest: Malariae presents with hematuria & foamy urine.
133) Free radicals are produced by which 139) If radial artery is chosen to monitor
of the following poisoning: intra-arterial pressure in adult, the
A) CCL4 appropriate size Cannula in order to
B) Ethanol avoid artifacts is:
C) Opioids A) 14 Gauge
SK Origional – Golden 11 53

B) 16 Gauge 144) Lipoprotein with highest


C) 18 Gauge phospholipids:
D) 20 Gauge A) HDL
E) 22 Gauge B) LDL
Ans: D C) VLDL
140) Lips are closed with the help of which D) Chylomicrons
muscle: Ans: A
A) Orbicularis oris Explanation:
B) Orbicularis oculi  Lipoprotein with highest Phospholipids:
C) Levatorlabi Superioris HDL > LDL > VLDL > Chylomicrons
D) Buccinator  Lipoprotein with highest Protein:
Ans: A HDL > LDL > VLDL > Chylomicrons
Explanation:  Lipoprotein with highest cholesterol:
 The orbicularis oris muscle, the lip muscle LDL > VLDL > HDL > Chylomicrons
is a broad elliptical muscle around the  Lipoprotein with highest TG:
mouth that when tonically contracted Chylomicrons > VLDL > LDL > HDL
closes the mouth. 145) Regarding Cartilage:
 Lateral pterygoid muscle is the only A) Epiglottis is hyaline
muscle of mastication that actively opens B) Synovial Joint has fibrous cartilage
the jaw. C) Appositional growth
141) Which of the following vein directly D) Transitional growth
drains into SVC: Ans: C
A) Azygous Vein 146) Fetal period starts after which week:
B) Hepatic Vein A) 8 week
C) Hemiazygous Vein B) 9 week
D) IVC C) 7 week
Ans: A D) 10 week
142) A man with presented stroke, ACA is Ans: A
affected, it will affect which area of Explanation:
brain:  Embryonic Period: Upto 8 Weeks
A) Somatosensory area  Fetal Period: After 8 Weeks
B) Brocas area 147) Gas with highest diffusion capacity:
C) Occipital area A) Oxygen
D) Primary motor area B) CO2
Ans: A C) CO
143) A man went from sea level to 1200 D) Nitrogen
height in 2 days. After 12 hours Ans: B
reaching the destination, he developed Explanation: Carbon dioxide (CO2) has the
dyspnoea. What the cause: maximum diffusion coefficient/capacity that‘s
A) Pulmonary edema 20.3, Oxygen 1, CO 0.81, Nitrogen 0.53 and
B) Secondary polycythaemia helium 0.95.
C) Primary polycythaemia 148) Slowest nerve fiber:
D) Heart Failure A) C fiber
Ans: A B) A fiber
Explanation: C) B fiber
 High Altitude Pulmonary Edema (HAPE) D) A delta fiber
is a no cardiogenic pulmonary edema which Ans: A
typically occurs in lowlanders who ascend rapidly Explanation: Among the mentioned fibers ―C-
to higher altitudes. Early symptoms includes non- Fibers‖ are the slowest nerve fibers that transmit
productive cough, dyspnea on exertion & reduced slow pain.
exercise performance & later on dyspnea occurs 149) Most of the optic fibers ends at:
eve at rest. A) Lateral Geniculate body
SK Origional – Golden 11 54

B) Medial Geniculate body SURGERY & ALLIED


C) Optic chiasma November 25th, 2020 - Afternoon
D) Pretectum
Ans: A
01) All interrosei of hand are supplied by:
Explanation: A) Median nerve
 Optic fibers that‘s visual pathway ends at B) Ulnar nerve
LGB C) Radial nerve
 Auditory fibers ends at MGB D) Musculocutaenous nerve
 L for Light that‘s vision E) Cuboidal nerve
 M for Music that‘s hearing Ans: B
150) A man with high BP, and decreased 02) 52-years-old man presented in opd
heart rate, what is the cause: with unusual facial expressions, half
A) Raised Intracranial Pressure side of the face was red and absence
B) Decreased Intracranial Pressure of sweating what could be the possible
C) Normal Intracranial Pressure cause for this condition:
D) None of the above A) Sympathetic Chain damage
Ans: A B) Facial nerve damage
Explanation: C) Parasympathetic damage
 Because the skull is rigid after infancy, D) None of the above
Intracranial masses or swelling may Ans: A
increase intracranial pressure. When Explanation: This is case of Horner‘s syndrome
intracranial pressure is increased that presents due to damage of Cervical
sufficiently, regardless of the cause Sympathetic damage with Ptosis, anhydrosis &
cushing‘s reflex & other autonomic constricted pupil (Miosis).
abnormalities can occur. 03) A pregnant woman presented with
 Cushing‘s reflex includes systolic HT, thecomplain of severe diarrhea, on
increase pulse pressure & bradycardia. microscopy there is tumbling
movement of the organism, what
could be the possible cause of
=============== diarrhea in this woman:
A) Enterococus
B) Listeria
C) Staph Aureus
D) Shigella
E) Salmonella
Ans: B
Explanation: Listeria monocytogenes is having
characteristic tumbling motility when viewed with
light microscope.
04) A patient presented in semi-conscious
condition with subclavian steal
syndrome, blood supply to which
artery is causing this condition:
A) Internal carotid artery
B) Vertebral artery
C) Maxillary artery
D) Facial artery
E) External carotid artery
Ans: B
05) A patient presented with infection in
his big toe, the lymphatic‟s from the
SK Origional – Golden 11 55

big toe goes to which group of lymph 10) Which of the following ions is main
nodes: determinant of plasma osmolarity:
A) Horizontal group of superficial inguinal A) Potassium
lymph nodes B) Sodium
B) Vertical group of superficial inguinal C) Magnesium
lymph nodes D) Chloride
C) Deep inguinal lymph nodes E) Calcium
D) Para-aortic lymph nodes Ans: B
E) Retroperitoneal lymph nodes 11) A 50 years old man started on oral
Ans: B anti-coagulant therapy, which of the
06) An old man presented with urinary followings used to monitor oral anti-
symptoms, after examination, doctor coagulant:
made a diagnosis of benign prostate A) APTT
hypertrophy, which zone of prostate is B) PT
involved in this condition: C) INR
A) Central zone D) D-diamers
B) Peripheral zone Ans: C
C) Transitional zone Explanation: INR > PT
D) Anterior zone 12) A sample of blood was drawn from the
Ans: C vein of a patient of renal failure, which
07) Long head of bicep femoris muscle is of the followings is estimated from
supplied by: this sample:
A) Common peroneal nerve A) Creatinine
B) Sciatic nerve B) PAH
C) Tibial nerve C) Nitrogen
D) Sural nerve D) Inulin
E) Common fibular nerve Ans: A
Ans: C 13) A lactating female presented with boil
Explanation: on labia, which of followings is safe to
 Short head of Bicep femoris is supplied prescribe in this female for the pain:
by common perioneal branch of sciatic A) Naproxen
nerve. B) Ibuprofen
 Long head of Bicep femoris is supplied by C) Diclofenac
Tibial branch of sciatic nerve. D) Celecoxib
08) A female with sign and symptoms of Ans: B
hyperprolactinoma was diagnosed 14) A 58 years old male presented with
with pituitary adenoma, if this tumor lethargy, malaise and cervical lymph
grows anteriorly; it will compress adenopathy, diagnosed as case of
which of the following structures: CML, which of the followings is
A) Cavernous sinus associated with the pathology in this
B) Optic nerve patient:
C) Optic chiasma A) 9:22
D) Optic Tract B) 9:21
Ans: B C) 8:14
09) Which of the following muscles D) 14:18
transverses the shoulder joint: Ans: A
A) Short head of bicep 15) A male presented with fever and
B) Long head of bicep cervical lymphadenopathy, he was
C) Deltoid diagnosed as a case of Burkett‟s
D) Trapezious lymphoma,which of the followings is
Ans: B associated with his condition:
A) 9:22
SK Origional – Golden 11 56

B) 9:21  Main pillar of medial longitudinal arch of


C) 8:14 the foot is: Talus
D) 14:18  Main support of medial longitudinal arch
Ans: C of the foot is: Calcaneum
16) Which of the following childhood  Main pillar of lateral longitudinal arch of
tumors associated with gene the foot is: Cuboid
amplifications? 21) Atlas has no:
A) Retinoblastoma A) Spinous process
B) Meningioma B) Articulation with Skull
C) Nephroblastoma C) Lumen
D) Neuroblastoma D) Arch
Ans: D Ans: A
17) Septic shock differs from hypovolumic Explanation: Atlas is made of a thick anterior
shock in: arch and a think posterior arch with lumen & skull
A) Cardiac output articulation but has no spinous process.
B) Urinary output 22) Which of the followings is most
C) Blood pressure important in doctor-patient
D) Pulse relationship:
Ans: A A) Confidence
Explanation: Septic shock has increase cardiac B) Mutual respect
output while hypovolemic shock has decrease C) Mutual trust
cardiac output. D) Clarity
18) A patient presented with headache, Ans: C
nausea vomiting, he was diagnosed as 23) A 26 years old female diagnosed case
a case of brain eating amoeba, which of myxedema presented with loss of
of the followings is causative sensations in thumb, wasting of
organism: thinner eminence, which of the
A) Proteus followings is associated with his
B) Naegleria floweri condition:
C) Aglilis A) Cubital tunnel syndrome
D) Vespertilio B) Carpel tunnel syndrome
Ans: B C) Ulnar nerve compression
19) Which of the followings is largest of D) Cervical rib
the cells in the blood: Ans: B
A) Monocyte 24) At which of the following parts of
B) Megakaryocyte stomach, abundant circular muscles
C) Basophills are found:
D) Neutrophils A) Antrum
Ans: A B) Body
Explanation: C) Cardiac end
 Largest cell in bone marrow or overall is: D) Pylorus
Megakaryocyte. Ans: A
 Largest cell in blood is: Monocyte. 25) Right gastric artery is branch of:
20) Which of the following is the main A) Gastro duodenal artery
pillar of medial longitudinal arch of B) Proper Hepatic artery
the foot: C) Left Gastric Artery
A) Talus D) Celiac artery
B) Calcaneum Ans: B
C) Cuboid Explanation:
D) Cuneiform  Right gastric artery in most cases arises
Ans: A from proper hepatic artery.
Explanation:  Left gastric artery arises from celiac trunk
SK Origional – Golden 11 57

26) Blow at anterior inferior iliac spine, out of his nose, what could be the
muscle damaged in pelvic avulsion: causative organism?
A) Sartorious A) Pin-wom
B) Retus femoris B) Taenia solium
C) Obturator internus C) Echinococcus granlosus
D) Hamstrings D) Ascaris
Ans: B Ans: D
Explanation: 33) Scenario asking for the remnant of
 Avulsion at inferior iliac spine muscle Notochord
damaged: Rectus Femoris A) Spinal meninges
 Avulsion at superior iliac spine muscle B) Annulus fibrosis
damage is: Sarotrius C) Spinal cord
27) A highly infectious respiratory virus D) Nucleus pulposus
discovered recently as covid-19, it is Ans: D
now named as: 34) A 30 years old patient presented with
A) Novel-corona pain in abdomen, on investigations it
B) Sars-cov-1 was found that a stone is blocking the
C) Sars-cov-2 proximal part of CBD, this blockage
D) Sars-Cov-9 results in:
Ans: C A) Increased amount of fats in stool
28) Which of the following muscles help B) Decreased bile salts absorption
in un-locking of knee: C) Decreased bile salts synthesis
A) Vastus laterals D) Blood in stool
B) Vastus medialis Ans: A
C) Quadriceps Explanation: Increase stool water content >
D) Popliteus Increase fat in stools.
Ans: D 35) An 11 years old girl referred to the
29) Anti-hyperlipedemiac drug side effect: radiologist for the x-ray of his elbow
A) GIT disturbances for the accurate measure of her age
B) Constipation the ossification centre will be present
C) Seizure at:
D) Hypertension A) Lateral epicondyl
Ans: A B) Medial epicondyl
Explanation: The commonest side effect formost C) Trochlea
of the drugs is GIT disturbances. D) Capitulum
30) Mucus secreting glands are found in Ans: A
which of the following parts of GIT: 36) A 19 Years old girl living with her aunt
A) Sigmoid colon is under treatment for her depression,
B) Duodenum her aunt called her doctor and
C) Ileum inquired about the present condition
D) Cecum of the patient, but,the doctor is
Ans: B reluctant to give any information
31) Which of the following test is best to about the present condition of his
check the function of thyroid gland: patient, this action of the doctor is
A) TSH under heading of which of the
B) T3 following aspects of the doctor‟s
C) T4 profession:
D) rT3 A) Justice
Ans: A B) Non-maleficence
32) An Eight years old boy looking pale C) Autonomy
with history of eating mud, he D) Confidentiality
vomited and a 6 inch long worm came Ans: D
SK Origional – Golden 11 58

37) Which of the following movements A) Botulism


take place at Atlanto - Axial joint: B) GBS
A) Flexion C) MG
B) Extension D) Staph Aureus
C) Abduction Ans: A
D) Adduction 43) An old age man presented with some
E) Rotation neurological symptoms, on
Ans: E examination past pointing, Romberg
Explanation: sign came out positive, which of the
 Rotation that‘s ―No‖ takes place at following parts of brain responsible for
atlanto-axial joint. his condition?
 Flexion that‘s ―Yes‖ takes place at A) Cerebrum
atlanto-occipital joint. B) Cerebellum
38) A male patient known case of renal C) Pons
failure, presented with sign and D) Medulla
symptoms of upper respiratory tract Ans: B
infection, which of the following 44) A 60 years old male presented to OPD
antibiotics, safe for this patient: with complain of dementia, doctor
A) Azithromycin noticed the tremors in his hand, mask
B) Ciprofloxacin like face, stooped posture, lesion in
C) Gentamycin which part of brain responsible for his
D) Norfloxacin condition:
Ans: A A) Midbrain
39) Scenario on inverse stretch reflex, B) Cerebullum
organ was asked: C) Medulla
A) GTO D) Thalamus
B) Muscle spindle Ans: A
C) Myosin Explanation: This is scenario of parkinson‘s
D) Actin disease that‘s caused by damage to substantia nigra
Ans: A that‘s present in midbrain.
40) Scenario on the patient with the 45) Which of the followings present in
absent knee jerk, root value of knee axilla:
jerk was asked: A) Cords
A) L1-L2 B) Trunks
B) L3-L4 C) Roots
C) S1-S2 D) Divisions
D) L4-L5 Ans: A
Ans: B 46) A lesion in the basal ganglia results in
41) Child with 60% weight loss, not which of the followings:
responding to vocal stimulus & A) Dysdydokinesia
bruises on leg: B) Intention termers
A) Marasmus C) Cog-wheel rigidity
B) Physical Abuse D) Involuntary movements
C) Malnutrition Ans: D
D) Hypothyroidism 47) Which of the followings is the most
Ans: B toxic anesthetic agent:
Explanation: Loss of weight with no response to A) Lidocaine
vocal stimulus and bruises on legs points toward B) Mepivacaine
physical abuse. C) Bupivacaine
42) Male presented with limb weakness & D) Procaine
respiratory symptoms after consuming Ans: C
canned pack foods: 48) Fascial spaces are filled with:
SK Origional – Golden 11 59

A) Connective tissues B) Free nerve endings


B) Vessels and nerves C) Beta Fibers
C) Adipose tissues D) Largest diameter fibers
D) Capillaries Ans: B
Ans: A 54) Lymphatic‟s from the breast mostly
Explanation: The spaces filled with loose areolar goes to the axilla, involves which
connective tissue may also be called clefts that groups of axillary nodes:
contains nerves and blood vessels. A) Medial+lateral
49) Commonest type of bronchogenic B) Apical+medial
carcinoma: C) Superior+inferior
A) Sqamous cell carcinoma D) Anterior+inferior
B) Adenocarcinoma Ans: A
C) Small cell carcinoma 55) A team of professionalists, for the
D) Large cell carcinoma research purpose, selected smoker
Ans: B people and divides as group A and
Explanation: Adenocarcinoma of the lung is the group B, after some time, group A
most common form of lung cancer accounting for people diagnosed with bronchogenic
30% of all cases overall and about 40% of all non- carcinoma, there are chances of
small cell lung cancer occurrences. development of carcinoma in the other
50) Operating on the inguinal hernia, group also, this type of study is called:
surgeon found that the hernia is A) Cohort study
immediately medial to inferior B) Case control study
epigastric artery, what type of hernia C) Observational study
was the surgeon operating: D) Cross sectional study
A) Indirect inguinal hernia Ans: A
B) Direct inguinal hernia Explanation:
C) Femoral hernia  From disease to cause is case control
D) Umbilical hernia study.
Ans: B  From cause to disease is cohort study.
Explanation:  Here It‘s from cause to disease and is a
 Direct inguinal hernia lies medial to kind of prospective cohort study.
epigastric vessels. 56) Effect of hypernatremia on the body
 Indirect inguinal hernia lies lateral to is:
epigastric vessels. A) Cell swelling
51) A 25 years old male encountered RTA B) Cell shrinkage
diagnosed with fracture of ribs, which C) Apoptosis
of the following is the site of a rib that D) Difficulty in the generation of action
could be fractured commonly: potential
A) Angle of the rib Ans: B
B) Body of the rib Explanation:
C) Neck of the rib  Hypernatremia means ECF has most
D) Tubercle of the rib sodium that will lead to extravasation of
Ans: A fluid from cells into ECF.
52) Which of the following are the slowest  Action potential will easily generate with
and unmylinated fibres: hypernatremia.
A) A-delta 57) Which of the following disease is
B) A-alpha caused by thiamine deficiency:
C) Beta fibers A) Hyperthyroidism
D) C-fibers B) Scaly dermatitis
Ans: D C) Beri-beri
53) Withdrawal reflex involves: D) Jaundice
A) Mylinated fibers Ans: C
SK Origional – Golden 11 60

58) A child aspirated a peanut while he D) 2kcal


was lying in supine position, the Ans: B
peanut will most probably lodge in: Explanation:
A) Apical segment of right lower lobe  1 Gram of Carbohydrate & protein yields:
B) Posterior segment of right lower lobe 4 Kcal
C) Anterior segment of right lower lobe  1 Gram of Fats yields: 9 Kcal
D) Posterior segment of right upper lobe 64) Which of the followings spread by
Ans: A mosquito bite:
Explanation: A) Leishmeniasis
 Aspiration of FB in supine position will B) Filariasis
go in superior/Apical bronchopulmonary C) Ascariasis
segment of right upper lobe. D) Schistosomiasis
 Aspiration of FB in standing position will Ans: B
go into posterobasal segment of right 65) A child presented with defect in
lower lobe. carbohydrate metabolism, there are
 Aspiration of FB in right sided position increased glucagon levels but glucose
will lodge in posterior bronchopulmonary levels not increasing, he was
segment of right upper lobe. diagnosed as a case of von Gierke‟s
59) Most common location of carcinoma disease, which of the following
of tongue is: enzyme is deficient in this patient:
A) Dorsum of tongue A) Glucose-6-phosphatase
B) Ventral surface of tongue B) Hexosaminidase
C) Tip of tongue C) Galactose
D) Lateral surface of tongue D) Phospho – glucokinase
Ans: D Ans: A
60) Which of the following is used to Explanation: Von Gierke disease is an autosomal
calculate total body water: recessive disorder caused by a deficiency of the
A) Evans blue dye enzyme glucose-6-phosphatase into the
B) Antipyreine endoplasmic reticulum for further metabolism.
C) Inulin 66) Rh negative female gave birth to Rh
D) Mannitol positive baby, the baby develops
Ans: B severe hemolytic anemia for which the
61) Normal value of FEV1/FVC is: baby required urgent blood
A) 1 transfusion, which antibody crossed
B) 0.8 the placenta and responsible for the
C) 5 hemolytic disease of this baby:
D) 3 A) IgM
Ans: B B) IgG
Explanation: Normal Value of FEV1/FVC is 0.8 C) IgA
or 80%. D) IgE
62) Highest systolic pressure is present in: Ans: B
A) Renal artery Explanation:
B) Aorta  IgG is the only antibody that crosses the
C) Pulmonary Artery placenta and take place in erythroblastosis
D) Carotid artery fetalis reaction.
Ans: A 67) Kidney is a vital organ and performs
Explanation: Highest systolic pressure is present so many functions, what is the most
in large arteries distal to heart like renal artery. basic function of the kidney?
63) 1 gram of fat gives how much energy: A) Concentration of urine
A) 1 kcal B) Control of blood pressure
B) 9kcal C) Removal of nitrogen
C) 4 kcal D) Removal of wastes
SK Origional – Golden 11 61

Ans: D 73) Goblet cells are present in which of


68) A 30 years old male presented with the following parts of respiratory
history of fever, significant weight system:
loss, painful micturition, hematuria A) Conducting zone
and flank mass, his Hb is 18 what B) Alveolar ducts
could be the possible pathology C) Terminal bronchioles
associated with his condition: D) Alveolar sacs
A) Renal cell carcinoma Ans: A
B) Rhabdomyosarcoma 74) A 48 years old male presented with
C) Angiomyolipoma complain of severe headache and
D) Polycystic kidney disease blurring of vision, doctor performed
Ans: A the LP, he found that the CSF is blood
Explanation: As the patient presented with stained, what could be the possible
history of weight loss and symptoms related to cause of this blood stained CSF:
renal system along with increase hemoglobin A) Sub-dural hemorrhage
that‘s due to increase erythropoietin level secreted B) Epi-dural hemorrhage
by kidneys and the most suitable among the C) Sub-archinoid hemorrhage
mentioned options is RCC. D) Extra-dural hemorrhage
69) A boy presented in ER with history of Ans: C
blow to the orbit, on examination the 75) The fastest and the most reliable route
duty doctor felt pulsations at the floor of administrating an antidote is:
of the orbital cavity, which of the A) Par-entral
following could be possible the cause B) Per oral
of these pulsations: C) Intramuscular
A) Cavernous sinus+ internal carotid artery D) Intravenous
B) Cavernous sinus+ external carotid artery Ans: D
C) Maxillary artery 76) A 55 years old female known case of
D) Facial artery some hematological pathology,
Ans: A admitted to the hospital with the
70) Thyroid hormone receptors found at history of sudden onset of severe
which of the following sites: dyspnea, on echocardiography it was
A) Nucleus found that the lady has severe
B) On cell membrane pulmonary hypertension what could
C) Cytosol be the best possible cause associated
D) Mitochondria with the condition:
Ans: A A) Repeated pulmonary emboli
71) Plasma cells are produced by which of B) Massive pulmonary embolism
the following: C) DVT
A) B-cell D) MI
B) T-cell Ans: A
C) M-cells 77) Melanoma is spread to the orbital
D) CD-4 cells cavity via:
Ans: A A) Dural venous sinuses
72) Pain of gastric ulcer is commonly B) Emissary veins
referred to which of the following C) Cavernous sinus
sites: D) Optic nerve
A) Back Ans: B
B) Shoulder 78) A child known case of cystic fibrosis
C) Epigastrium presented with acute respiratory
D) Right upper quadrant of abdomen infection, on investigation the
Ans: C organism diagnosed as pseudomonas
SK Origional – Golden 11 62

aruginosa, what are the other findings D) Ligamentum venosum


associated with this patient: Ans: C
A) Bronchial asthma 83) A farmer while spraying pesticides to
B) Bronchiectasis the fields developed the signs and
C) Emphysema symptoms of toxicity, what is the
D) Pleural effusion specific antidote for the
Ans: B organophosphate poisoning:
79) Sub-dural hemorrhage is caused by A) Atropine
the rupture of which of the followings: B) Pralidoxime
A) Berry aneurysm C) Naloxone
B) Middle cerebral vein D) Lidocaine
C) Superior cerebral vein Ans: B
D) Middle meningeal artey Explanation: Specific antidote for OP is
Ans: C pralidoxime while antidote for symptoms reversal
Explanation: is Atropine.
 Bridging veins > Superior cerebral vein in 84) Which of the following structures
case of Subdural hematoma. contains no lymph nodes:
 Middle meningeal artery in case of Epi A) Palatine tonsils
dural hematoma. B) Pharyngeal tonsils
80) Anterior surface of the heart is formed C) Thymus
by: D) Spleen
A) Right ventricle Ans: C
B) Right atrium 85) A 50 years old male presented in ER
C) Left ventricle with acute chest pain, doctor
D) RV + SVC performed his ECG, he found that
Ans: A there is ST elevation in the lead-II,
Explanation: which of the following arteries
 Anterior surface (Sternocostal surface) of blocked that was responsible to supply
heart is formed mainly by right ventricle this area:
 Posterior surface (Base) of heart is mainly A) Right coronary artery
formed by Left atrium B) Left coronary artery
 Right surface of heart is formed by right C) Anterior interventricular artery
atrium D) Left circumflex artery
81) A female gave birth to a baby, after the Ans: A
birth she developed severe 86) A young male living in a populated
hemorrhage for which she was town, he died in an RTA on the
transfused with blood, after few autopsy the lymph nodes of lungs are
minutes she developed signs and found black in color, which of the
symptoms of transfusion reaction, followings associated with the
what type of hypersensitivity response condition of this patient:
associated with transfusion related A) Melatonin
reaction: B) Anthracosis
A) Type-I hypersensitivity C) Bronchogenic carcinoma
B) Type-II hypersensitivity D) Bronchial asthma
C) Type-III hypersensitivity Ans: B
D) Type-IV hypersensitivity 87) Which of the following is the most
Ans: B common cause of delayed wound
82) Bare area of the liver is limited by healing:
which of the following structures: A) Mobility
A) Ligamentum arteriosum B) Foreign body
B) Falciform ligament C) Infection
C) Coronary ligament D) Old age
SK Origional – Golden 11 63

Ans: C A) Glycogenesis
Explanation: B) Glycolysis
 The commonest local cause for delayed C) Gluconeogenisis
wound healing is infection. D) Lactogenisis
 The commonest systemic cause is DM. Ans: C
88) Malignant feature: Explanation: Between Meals Glycogenolysis
A) Pleomorphism >Gluconeogenesis(FA)
B) Metastasis 93) Ewings sarcoma involves which of the
C) Invasion followings:
D) Increase N/C ratio A) Diaphysis
Ans: B B) Metaphysis
Explanation: C) Epiphysis
 Malignancy is characterized by: D) Epiphysial line
 Metastasis > Invasion > Pleomorphism > Ans: A
Increase N/C Ratio Explanation:
89) A 50 years old male known case of  Giant cell tumor most common site:
renal failure GFR less than 50%, Epiphysis
which of the following de-  Osteosarcoma: Metaphysis
arrangement is most alarming for the  Ewing Sarcoma: Diaphysis
doctor in this patient:  Most common malignant tumor in young
A) Hyponatremia age: Osteosarcoma
B) Hyperkalemia  Most common primary tumor of bone:
C) Hypokalemia Osteosarcoma
D) Hypermagnesemia  Most common benign tumor:
Ans: B Osteochondroma
90) Which of the following causes  Soap bubble appearance: Giant cell tumor
thrombus formation: 94) A person got a stab wound 3cm
A) Decrease Blood flow vertical on lateral right side of linea
B) Decrease Viscosity alba, initially he was fine but then
developed hypotension & shock.
C) Endothelial damage
Which vessel is most likely damaged:
D) Increase blood flow A) IVC
Ans: C B) Superior mesenteric Artery
Explanation: The Virchow‘s triad of thrombosis C) Inferior Mesenteric Artery
consists of stasis of blood flow, hypercoagulability D) Abdominal Aorta
& Endothelial Injury. E) Ileal branch of superior mesenteric artery
Ans: A
91) Which of the following investigations Explanation: The surface marking of the inferior
most sensitive for the SLE: vena cava is a vertical line 2.5 cm – 3 cm to the
A) ANA right of the midline from the intertubercular plane
B) AMA to the sixth costal cartilage.
C) Anti Ds DNA 95) Which of the following decreases
D) Anti Ro Antibodies osmolarity:
Ans: A A) Vasopressin (ADH)
Explanation: B) Epinephrine
 Most sensitive test for SLE is: ANA C) Aldosterone
 Most Specific test for SLE is: Anti
D) Renin
Smith > Anti DsDNA
92) Liver maintains serum glucose levels Ans: A
between meals by which of the Explanation: Vasopressin (ADH) absorbs water
followings: & decreases plasma osmolarity.
SK Origional – Golden 11 64

96) Which of the following is the 102) Which of the following is the direct
characteristic of Botulinism: branch of subclavian artery that take
A) Paralysis of Abdominal muscles part in the shoulder joint anastomosis:
B) Paralysis of Respiratory muscles A) Dorsal scapular artery
C) Death B) Suprascapular artery
D) Cardiac arrest C) Thyrocervical trunk
Ans: B D) Thoraco-acromial artery
97) ESR more than 100 is seen in: Ans: A
A) RA Explanation:
B) OA  Dorsal scapular artery is the direct branch
C) Multiple Myeloma of subclavian artery that takes part in
D) SLE shoulder joint anastomosis.
Ans: C  Thyrocervical trunk is the main branch
98) In fasting during summer urine but it doesn not take part in shoulder
osmolarity is increased due to: joint anastomosis, but through its branch
A) ADH called suprascapular artery.
B) Oxytocin 103) Muscles have different classifications
C) Epinephrine according to their actions and features
D) NE they are divided in different groups
Ans: A and sub groups, according to one
Explanation: During fasting when ECF volume classification, which of the following
is decreased ADH is secreted that leads to muscle is white muscle:
retention of water and thus increasing urine A) Masseter
osmolarity. B) Temporalis
99) A 50 years old male, diagnosed case of C) Gastrocnemius
chronic pancreatitis, presented with D) Buccinators
steatorrhea, this abnormality is caused Ans: C
by the deficiency which lipid digestive 104) Anterior fontanelle closes at the age
enzyme: of:
A) Amylase A) 12 months
B) Lipase B) 3 year
C) Lactase C) 2 years
D) Enterokinase D) 24 months
Ans: B Ans: D
100) 1 degree rise in temperature increases Explanation: Anterior fontanelle closes at 18-24
CRMO2 by: months of age (Klm+BD+Google)
A) 8% 105) Drug receptors are:
B) 7% A) Protein in nature
C) 6% B) Fixed in number
D) 5% C) Decreased in old age
Ans: D D) Variable
101) Which of the following is the Ans: A
parasympathetic effect: 106) A female presented with burning
A) Increases HR micturition, no any history of fever or
B) Decreases HR flank pain, what could be the
C) Increase Contractility organism responsible for her
D) Increases TPR condition:
Ans: B A) S.aurues
Explanation: Parasympathetic system decreases B) Pseudomonas arugenosa
Heart rate, TPR & contractility. C) Proteus mirabilus
D) E.coli
Ans: D
SK Origional – Golden 11 65

Explanation: The commonest cause for UTI is C) Baroreceptors


E. Coli D) RAAS
107) 0.5 gram/dl protein is present in Ans: C
which fluid: Explanation: Rapid & fasted control of blood
A) Lymph pressure is brought by baroreceptor while most
B) CSF potent is CNS ischemiac response and most
C) Saliva prolong is RAAS.
D) Tears 114) Which of the following occurs in
Ans: A Uterus during pregnancy:
108) Nasopharyngeal carcinoma is caused A) Hypertrophy
by: B) Hyperplasia
A) EBV C) Hypertrophy + Hyperplasia
B) CMV D) Atrophy
C) HPV Ans: C
D) HIV 115) Which of the following is commonest
Ans: A bladder carcinoma:
Explanation: A) Transitional Carcioma
 Nasopharyngeal carcinoma is caused by B) BCC
EBV C) SCC
 Oropharyngeal carcinoma is caused by D) Melanoma
HPV Ans: A
 Bronchial carcinoma is caused by CMV 116) Patient after burn presented with
109) Diapedesis refers to: edema due to decrease level of which
A) Margination of the following:
B) Migration A) Globulin
C) Chemotaxis B) Albumin
D) Apoptosis C) Antibodies
Ans: B D) Interstitial Pressure
110) CO2 passes through which of the Ans: B
following: 117) A child died of some renal problem
A) Passive diffusion that showed multiple cysts on autopsy
B) Active Transport what is the mood of inheritance:
C) Secondary Transport A) Autosomal Recessive
D) Facilitated B) Autosomal Dominant
Ans: A C) X Linked Recessive
111) Oxygen requirement for anesthesia: D) X Linked Dominant
A) 100 Ans: A
B) 250 Explanation: Polycystic kidney disease is
C) 350 autosomal recessive in children & autosomal
D) 500 dominant in adults.
Ans: B 118) AG2 increase which of the following:
112) A patient presented after a bee sting A) Thirst
with dyspnea most likely due to: B) Urine
A) Type I HSR C) Sweating
B) Type II HSR D) Osmolarity
C) Type III HSR Ans: A
D) Type IV HSR 119) Zona Glomerulosa is linked with:
Ans: A A) Sodium reabsorption
113) Fastest control of blood pressure is B) Potassium reabsorption
achieved by: C) Sex hormone maintenance
A) CNS Ischemiac response D) None of the above
B) Chemoreceptors Ans: A
SK Origional – Golden 11 66

Explanation: Zona glomerulosa secretes C) Fetus with raised middle cerebral artery
aldosterone that‘s linked with sodium absorption D) Doppler has low probability of anemia
and Potassium secretion. E) Can detect renal changes in fetus during
120) Cimetidine interaction with warfarin: last months
A) Increase hepatic clearance of warfarin Ans: B
B) Decrease hepatic clearance of warfarin 126) Heart can‟t be tetanized due to:
C) Increase half-life of warfarin A) Short refractory period
D) Decrease half-life of warfarin B) Long refractory period
Ans: B C) Gap junctions
Explanation: Cimetidine is potent CYP3A4
enzyme inhibitor & thus decreases the hepatic D) Desmosome
clearance of warfarin. Ans: B
121) Most common leukemia in children: 127) 28 Years old Girl (Same in CPSP
A) ALL Paper) with circumcision history of
B) AML bleeding which test to perform to
C) CML confirm the diagnosis:
D) CLL A) Factor VIII Assay
Ans: A B) Factor VII Assay
122) Increase level of calcium will lead to C) Aptt
increase in which of the following: D) PT
A) 1,25 cholecalciferol Ans: A
B) 24,25 cholecalciferol Explanation:
C) 1 cholecalicferol  This is a scenario related to hemophilia,
D) 25 cholecalciferol now two doubts arises either they
Ans: B mistyped girl instead of boy or if not then
123) Prostate carcinoma involves which of in some countries Circumcision is carried
the following lobe: in girls too. Hemophilia is X-Linked that‘s
A) Anterior very rare in girls but still it can express
B) Posterior itself in female gender too.
C) Lateral  Initial Test for Hemophilia is aPTT
D) Median  Confirmatory Test is Factor VIII Assay
Ans: B 128) A patient receives radiotherapy for
Explanation: Prostate carcinoma involves cancer, after a few days he develops
posterior lobe or peripheral zone. scaring & ulceration of the skin it‟s
124) Pregnant lady with delayed reach to due to:
hospital presented with dyspnea most A) Endarteritis Obliterans
likely due to: B) Desquamation
A) Fat Embolism C) Venous thrombosis
D) Infection
B) Air Embolism Ans: B
C) Amniotic fluid embolism Explanation: Within few days desquamation
D) Hemorrhage develops while endarteritis obliterans takes time to
Ans: C develop.
Explanation: Pregnant lady with dyspnea without 129) Parturition is initiated by:
any other factors like trauma is most likely A) Oxytocin
amniotic fluid embolism. B) Prostaglandins
125) Doppler studies during pregnancy: C) Fetal Cortisol
A) Can detect fetal cardiac activity before 8 D) Fetal Adrenals
weeks Ans: C
B) Pulstility in uterine areries increase in
growth restricted fetus
SK Origional – Golden 11 67

130) A British woman went to Thailand D) Atonic paralysis


and presented with fever, rigors & Ans: B
chills is most likely suffering from: 136) Alveoli doesn‟t easily collapse due to
A) Malaria the presence of:
B) Infectious Mononucleosis A) Surfactant
C) Enteric Fever
B) Lipids
D) AIDS
Ans: A C) Steroids
Explanation:Thailand is geographically more D) Collagen Fibers
related to malaria & the findings are suggestive for Ans: A
malaria too those are Fever, rigors & specially 137) Little girl developed purpura after
chills. vaccination:
131) Which of the following is the origin of A) TTP
external oblique muscle:
B) ITP
A) Ribs 3 – 4
B) Ribs 4 – 5 C) VWD
C) Ribs 5 – 12 D) Hemophilia
D) Ribs 12 Ans: B
Ans: C 138) Hyperkalemia is caused by:
132) Basic mechanism involved in aqueous A) Exercise
humor production: B) Beta Agnoist
A) Active Sodium Secretion C) Aldosterone
B) Active Sodium Secretion & Ultrafiltration D) Renin
C) Potassium Secretion Ans: A
D) Vessels permeability Explanation: During exercise more potassium
Ans: B comes out from intracellular to extracellular fluid.
133) Regarding cardiac cycle: 139) Common features of skeletal muscles
A) S2 at start of T wave & cardiac muscles:
B) P wave is followed by T wave A) Transverse Striation
C) 1st heart sound is produced in B) Transverse Tubules
C) Gap Junction
isovolumetric ventricular contraction
D) Action Potential
D) 1st heart sound is produced in Ans: A
isovolumetric ventricular relaxation Explanation:
Ans: C  Skeletal and cardiac muscles : Transverse
Explanation: 1st heart sound is produced in striation
isolvolumetric ventricular contraction due to  Skeletal and smooth muscles: increase
closure of AV valves that‘s mitral & tricuspid Calcium before contraction
 Smooth and cardiac muscles: Gap
valves.
Junctions
134) Mechanism of reduction of size of 140) Submucosal Fibrosis (SMF) is:
organ in patient undergoing A) Premalignant
chemotherapy: B) Benign
A) Apoptosis C) Malignant
B) Direct action of Chemotherapy D) Dysplasia
C) Necrosis Ans: A
D) Cellular Damage Explanation:Submucosal fibrosis is
Ans: A apremalignant condition of oral cavity.
135) Atrophy of calf muscles due to: 141) Acute diarrhea causes which of the
A) Upper motor neurons lesion following:
B) Lower motor neurons lesion A) Metabolic Alkalosis
C) Spastic paralysis B) Metabolic Acidosis
SK Origional – Golden 11 68

C) Mixed Acidosis Explanation:


D) Respiratory Acidosis  The Orbicularis Oris muscle, the lip
Ans: B muscle is a broad elliptical muscle around
Explanation: the mouth that when tonically contracted
 Acute/Severe diarrhea leads to Metabolic closes the mouth.
Acidosis  Lateral Pterygoid muscle is the only
 Chronic diarrhea leads to Metabolic muscle of mastication that actively opens
Alkalosis the jaw.
142) During thyroidectomy superior 146) Muscle of hands are supplied by:
thyroid artery is ligated. The nerve A) Ulnar Nerve
likely to be injured: B) Median Nerve
A) External Laryngeal nerve C) Radial Nerve
B) Internal Laryngeal nerve D) C8 & T1
C) Recurrent Laryngeal nerve Ans: D
D) Superior Laryngeal nerve Explanation: Small muscles of hands are
D) Vagus nerve supplied by both Ulnar & Median nerve so here
Ans: A root value in common is C8 and T1.
Explanation: 147) Early diagnosis & prompt treatment
 If superior artery is ligated the nerve most is:
likely to be damaged is external laryngeal A) Counseling
nerve. B) Primary prevention
 If inferior artery is ligated the nerve most C) Secondary prevention
likely to be damaged is recurrent laryngeal D) Simple prevention
nerve. E) Tertiary prevention
 During thyroidectomy the nerve most Ans: C
commonly damaged is external laryngeal Explanation:
nerve.  Primary prevention: health promotion &
143) Stratum functionalis of uterus is specific protection
formed by:  Secondary prevention: early diagnosis &
A) Stratum compactum & Spongiosum prompt treatment
B) Stratum basale & Spongiosum  Tertiary prevention: Disability limitation
C) Stratum compactum & basale & rehabilitation
D) Stratum basale & Spongiosum 148) Albumin & RBCs in urine due to
Ans: A damage of:
Explanation: Stratum functionalis of uterus is A) PCT
composed of stratum compactum & stratum B) DCT
spongiosum while the regenerating layer is stratum C) Glomerulus
basale. D) Tubules
144) Testosterone is released by: Ans: C
A) Sertoli Cells 149) Increase BP 220/130 due to defect in
B) Lydig Cells which of the following structure:
C) Both A) JG Cells
D) None of the above B) PCT
Ans: B C) LOH
145) Lips are closed with the help of which D) Efferent Vessels
muscle: Ans: A
A) Orbicularis oris 150) Internal spermatic fascia formed by:
B) Orbicularis oculi A) External Oblique muscle
B) Internal Oblique muscle
C) Levator labi superiors
C) Fascia Transversalis
D) Buccinators D) Fascia Iliaca
Ans: A Ans: C
SK Origional – Golden 11 69

Explanation: 04) Tumor of pituitary gland spreads to


 External Oblique muscle forms External which bony structure?
spermatic fasica a. Frontal
 Internal Oblique muscle forms b. Maxillary
Cremasteric fascia c. Orbital
 Fascia transversalis forms Interanal d. Sphenoid
spermatic fascia Ans: D
Explanation: The bone to which pituitary gland
tumor spread is Sphenoid while pituitary tumor
=============== compresses optic nerve.
05) RBC is kept in hypertonic solution
containing Urea what will be the
SURGERY & ALLIED result:
November 25th, 2020 – Night a. Swell & lyse
b. Shrink, Swell & lyse
c. Shrink
01) Which of the following is d. Lyse & Swell
characteristic lesion in cerebellar Ans: B
disease? Explanation: Keeping RBC in hypertonic Urea
a. Dysdiadoochokinesia solution, initially the fluid will come out of the
b. Hypotonia RBC and it will shrink and then Urea freely moves
c. Ataxia into cell and that will swell and ultimately lyse.
d. Dysarthria 06) Bitemporal Hemianopsia is due to
Ans: A damage to:
Explanation: All of the above are findings of a. Optic Nerve
cerebellar disease but dysdiadochokinsea is b. Optic Radiation
c. Optic Chiasma
the characteristic feature. d. Occipital Cortex
02) Nasopharyngeal carcinoma is caused
Ans: C
by:
07) Lesion to Right optic radiation lateral
a. EBV fibers in temporal lobe will lead to:
b. CMV a. Right Superior quadranopia
c. HPV b. Right Inferior quadranopia
d. HIV
c. Left Superior quadranopia
Ans: A
d. Left inferior quadranopia
Explanation: Ans: C
 Nasopharyngeal carcinoma is caused by Explanation:
EBV
 Lesion to Right temporal optic fibers
 Oropharyngeal carcinoma is caused by leads to left superior qaudranopia.
HPV  Lesion to Right parietal optic fibers leads
 Bronchial carcinoma is caused by CMV to left inferior quadranopia.
03) Butchers are at highest risk of getting 08) Incisive foramen contains which of
infected with which of the following the following structure?
disease? a. Maxillary Nerve
a. Brucellosis b. Mandibular Nerve
b. HIV c. Nasopalatine Nerve
c. IM d. Cilliary Nerve
d. Dengue Fever Ans: C
Ans: A Explanation:
Explanation: Butchers & slaughterhouse workers  Foramen rotundum contains maxillary
are at highest risk of contracting zoonotic nerve.
diseases.
SK Origional – Golden 11 70

 Foramen ovale contains mandibular  U wave is seen in Hypokalemia.


nerve. 14) Which of the following is prominent
 Foramen incisive contains nasopalatine feature of RTA?
nerve. a. Hyperkalemia
09) Foramen Cecum contains which of b. Hyponatremia
the following? c. Nitrogen Imbalance
a. Emissary Veins d. Urea Imbalance
b. Optic Nerve Ans: A
c. Lacrimal Nerve 15) Which of the following is found more
d. Frontal Nerve in ICF?
Ans: A a. Potassium
Explanation: b. Sodium
 Foramen Cecum contains Emissary Veins c. Magnesium
 Optic canal contains Optic nerve d. Carbon
 Superior orbital fissure contains Lacrimal Ans: A
& frontal nerves. Explanation: Cells are bags of potassium &
10) Which of the following has highest found in highest quantity in ICF.
frequency vibration? 16) Visceral efferent fibers to which of the
a. Messiners following structures:
b. Pacinian a. Skeletal Muscles
c. Pain Receptors b. Auditory system
d. Chemoreceptors c. Taste fibers
Ans: B d. Smooth muscle & heart
11) SA node acts as a pacemaker because And: D
of which of the following quality? 17) Hepatoduodenal ligament contains
a. Automaticity which of the following structure?
b. Highest frequency of firing rate than a. Celiac Trunk
others b. IVC
c. More gap junctions c. Aorta
d. Due to its size d. Portal Vein
Ans: B Ans: D
Explanation: AV node is having automaticity Explanation: The contents of Hepatoduodenal
property too but SA node acts as a pacemaker due ligament contain Portal Vein, Hepatic Artery &
to highest frequency firing rate than other Bile duct.
structures. 18) Muscles of back are supplied by:
12) Which of the following represent a. Dorsal rami
complete systole on ECG? b. Dorsal Primary rami
a. QRS c. Intercostal nerve
b. ST Segment d. Long thoracic nerve
c. PR Segment Ans: B
d. P Wave 19) Muscles of back are supplied by:
Ans: B a. Dorsal rami
13) Tall T wave is seen in which of the b. Dorsal Primary rami
following abnormality? c. Intercostal nerve
a. Hypokalemia d. Long thoracic nerve
b. Hyperkalemia Ans: B
c. Hypocalcemia Explanation: The question was repeated twice.
d. Hypercalcemia 20) Which of the following test to be done
Ans: B for typhoid fever during the first week?
Explanation: a. Urine culture
 Peaked Tall T waves are seen in b. Blood Culture
Hyperkalemia. c. Antibody Test
SK Origional – Golden 11 71

d. Stool Culture Explanation:


Ans: B  Increase level of Vitamin D leads to
Explanation: increased Calcium, Phosphate, BUN &
 Remember BASU Urea level.
 Blood Culture = 1st week  Milk Alkali syndrome has somewhat same
 Agglutination Test (Widal Test) = 2nd findings too but that needs history of
week ingestion of some antacids.
 Stool culture = 3rd week 26) Which of the following has stratified
 Urine culture = 4th week cuboidal epithelium?
21) Gonorrhea can be easily diagnosed a. Ducts of Salivary Glands
with the help of: b. Alveoli
a. Gram Staining c. Linings of Lungs
b. ZN Stain d. Esophagus
c. Biopsy Ans: A
d. Culture Explanation:
Ans: A  Ducts of Salivary glands contain stratified
22) Which of the following are flexors of columnar epithelium but stratified
metacarpophalangeal joints? cuboidal epithelium can be found too.
a. Interosseous  Stratified cuboidal epithelium specially
b. Lumbricals found in the ducts of sweat glands.
c. Digitorium Superficial is 27) A 56 years old executive officer after
d. Biceps eating burger with fries, gets his lipid
Ans: B profile done. After centrifuge his
23) A child presented with Sickle cell plasma appears milky white, this is
anemia with cordman triangle due to:
Osteomyelitis most likely due to a. LDL
which of the following: b. VLDL
a. Salmonella c. HDL
b. E. Coli d. IDL
c. Staph Aureus e. Chylomicrons
d. Streptococcus Ans: E
Ans: A 28) 23 weeks pregnancy which of the
Explanation: following drug is used in
 The commonest cause of Osteomyelitis is hyperthyroidism:
Staph Aureus a. PTU
 While, in Sickle cell anemia it‘s Salmonella b. Thyroxin
24) Which of the following organism c. T3
causes Pseudomembranous colitis? d. Iodine
a. Gonorrhea Ans: A
b. Staph Aureus Explanation: During first trimester we give ptu
c. Difficile in hyperthyroidism & in 2nd and 3rd trimester we
d. E. Coli give methimazole, but there was no option of
Ans: C methimazole that‘s why we pick ptu, even
25) A patient presented with increase in according to some latest researches it says ptu is
calcium, phosphate & urea level due safe in all trimesters in case of hyperthyroidism
to which of the following: during pregnancy.
a. Milk Alkali Syndrome 29) In third trimester Estrogen &
b. Vitamin D toxicity progesterone are produced by:
c. Vitamin A toxicity a. Placenta
d. RTA b. Corpus Luteum
Ans: B c. Adrenal Glands
d. Fetus
SK Origional – Golden 11 72

Ans: A c. Both supra renal veins drain into renal


30) Oxygen requirement in anesthetized vein
70Kg man: d. Right supra renal vein drains into
a. 150 common iliac vein
b. 250 Ans: B
c. 450 Explanation:
d. 500  Right Supra renal vein drains into IVC.
Ans: B  Left supra renal vein drain into renal vein.
31) Heparin primarily acts on: 35) Thyroglobulin is produced by:
a. Factor 10 a. Follicular Cells
b. Thrombin b. Parafollicular Cells
c. Anti-thrombin 3 c. Papillary Cells
d. Factor 12 d. Chief Cells
Ans: C Ans: A
Explanation: 36) Snow Storm appearance is seen in:
 Heparin binds & Act on: Anti Thrombin a. Incomplete Mole
3. b. Complete Mole
 Heparin degrades factor 2 (Thrombin), c. Lymphoma
factor 10 and 11. d. Miscarriage
32) Which of the following arches over Ans: B
root of right lung? 37) Excessive talking that makes no
a. Aorta sense:
b. IVC a. Wernickes Aphasia
c. Azygous Vein b. Brocas‘s Aphasia
d. Vagus Nerve c. Transcortical Aphasia
Ans: C d. None
Explanation: Ans: A
 Structure infront of root of lungs: Phrenic Explanation:
Nerve  Excessive talking that makes no sense &
 Structure behind the root of lungs: Vagus defective comprehensive ability in
Nerve Wernicke‘s aphasia.
 Structure which arches over root of right  Unable to produce words in broca‘s
lung: Azygous Vein aphasia.
 Structure which arches over root of left 38) Which of the following is the product
lung: Arch of Aorta of carbohydrate digestion?
33) S1 heart sound is produced during: a. Galactose
a. Iso Volumetric Contraction b. Glucose
b. Iso Volumetric Relaxation c. Sucrose
c. Ejection Phase d. Maltose
d. Relaxation phase Ans: B
Ans: A 39) XXY is seen in:
Explanation: a. Down Syndrome
 S1 heart sound is produced during iso b. Klinfelter syndrome
volumetric contraction c. Edward Syndrome
 S2 heart sound is produced during iso d. Patau Syndrome
volumetric relaxation Ans: B
34) Regarding supra renal vein which of 40) Fibroblasts secrete which of the
the following is correct? following:
a. Right Supra renal vein drains into Renal a. Collagen Fibers
vein b. Elastin Fibers
b. Left supra renal vein drains into renal c. Amorphous Ground Substance
vein d. Globulin
SK Origional – Golden 11 73

Ans: C Ans: A
Explanation: Explanation: Lateral lymphatic drainage of breast
 Fibroblasts secrete amorphous ground is mainly to pectoral lymph nodes & medical to
substance. internal thoracic lymph nodes.
 Fibrocartilage has collagen fibers. 48) Which factor is determined by TPR
41) Which of the following supply supra Index?
renal gland? a. Pulse Pressure
a. Greater Splanchnic Nerve b. Means Arterial pressure
b. B) Lesser Splanchnic Nerve c. Diastolic Pressure
c. Least Splanchnic Nerve d. Systolic Pressure
d. Intercostal nerves Ans: C
Ans: A 49) Shigella causes dysentery by
42) Which of the following muscle rotates mechanism of:
scapula? a. Mucosal Invasion
a. Serratus Anterior b. Cell Wall
b. Pectoralis Major
c. Proteoglycan
c. Pectorlis Minor
d. Deltoid d. Phospholipid
Ans: A Ans: A
43) Antigen appears on RBC at which of Explanation: Shiga toxin causes GI mucosal
the following week? invasion & damage leading to dysentery.
a. 10th Week 50) Sympathetic stimulation causes which
b. 15th Week of the following?
c. 20th Week a. Increase Heart rate
d. 30th Week b. Decrease Cardiac contractility
Ans: C c. Decrease Cutaneous blood flow
44) Hyperplasia means which of the d. Decrease Bronchi diameter
following? Ans: A
a. Increase in size of cells Explanation: Epinephrine increases HR, cardiac
b. Increase in number of cells contractility, bronchodilator & decreases
c. Increase in stromal tissue cutaneous blood flow.
d. Decrease in number of cells 51) Sympathetic beta adrenergic leads to
Ans B which of the following?
45) Gut bacteria helps in synthesis of a. Increase Airway resistance
which of the following vitamin? b. Decrease Airway resistance
a. Vitamin A c. Increase Bronchoconstriction
b. Vitamin B d. Decrease Heart rate
c. Vitamin C Ans: B
d. Vitamin K 52) 10 Years old presented with cyanosed
Ans: D lips & fingers most likely suffering
46) Morphological & functional change is: from:
a. Dysplasia a. TGA
b. Atrophy b. TOF
c. Metaplasia c. ASD
d. Anaplasia d. Bicuspid aortic valve
Ans: C Ans: B
47) Breast Lateral Lymphatic drainage: Explanation:
a. Pectoral (Axillary) Lymph Nodes  Most common cyanotic heart disease
b. Posterior Axillary Lymph Nodes immediately after birth and also in infant
c. Internal Thoracic Lymph Nodes of diabetic mother is: TGA
d. Supraclavicular Lymph Nodes
e. Infraclavicular Lymph Nodes
SK Origional – Golden 11 74

 Most common cyanotic heart disease is: 58) Which of the following is last to
TOF appear in blood after hemorrhage?
 Most common acyanotic heart disease is: a. Hematocrit
VSD b. Hemoglobin
 Most common congenital heart disease is: c. RBCs
VSD d. Reticulocytes
 Most common acyanotic heart disease in Ans: C
adult is: ASD 59) Old aged man with testicular
 The only cyanotic heart disease where swelling & raised LDH is most
shunt is must for survival of baby is : likely suffering from:
TGA a. Seminoma
53) Which of the following accurately b. Lymphoma
measure GFR? c. Embryonal Carcinoma
a. Inulin d. Yolk Sac tumor
b. Creatinine Clearance
c. Serum Creatinine
Ans: B
d. PAH Explanation:
Ans: A  Old aged male with raised LDH is
Explanation: Remember measure by inulin & suffering from Lymphoma
estimated by creatinine clearance.  Young aged male with Raised LDH is
54) Which of the following hyperglycemic Seminoma
factor is released by pancreas?  As general seminoma > Lymphoma
a. Insulin 60) Type III HSR is seen in which of the
b. Somatostatin following condition:
c. Glucagon a. Tb
d. Cortisol b. SLE
Ans: C c. RA
Explanation: Glucagon is secreted via pancreas d. Atopic Asthma
that increases blood sugar level & is hyperglycemic Ans: B
in nature. 61) Microscopic feature of chronic
55) Which of the following is therapeutic inflammation of liver:
approach to MRSA: a. Lymphocytes infiltration
a. Resistant to Vancomycin b. Fatty Accumulation
b. Sensitive to Vancomycin c. Portal & periportal fibrosis
c. Sensitive to Metronidazole d. Dense lymphocytic infiltrates
d. Sensitive to penicillin Ans: C
Ans: B 62) Epitheloid cells found on liver
56) Which of the following is the histology is most likely:
functional unit of liver? a. Gangrenous Necrosis
a. Lobule b. Liquefective Necrosis
b. Hepatocyte c. Caseous Necrosis
c. Segment d. Fat Necrosis
d. Hepatic Acinus Ans: C
e. Sinusoids 63) Inflammation of all three zones of liver
Ans: D and wide spread necrosis, what is the
57) Histology of lymph node with cause?
squamous epithelium: a. Acute hepatitis
a. Pharyngeal tonsil b. Fulminant hepatitis
b. Peyer patches c. Chronic hepatitis
c. Palatine tonsil d. Chronic active hepatitis
d. None of the above Ans: D
Ans: C
SK Origional – Golden 11 75

64) Attachment of epithelium to basal a. Isoflurane


membrane of gingiva by: b. Sevoflurane
a. Desmosome c. Ketamine
b. Hemidesmosome d. No
c. Fibrillin Ans: C
d.. Myosin 71) If ICF is increased what will decrease?
Ans: B a. Heart rate
65) Which of the following is the tumor of b. Fluid
Cementum? c. Blood pressure
a. Cemantoblastoma d. Tissue Oxygenation
b. Cemantochorisotoma Ans: A
c. Blastomcytoma 72) A person presented who complaints of
d. Cemantoma can‟t see in dark due to deficiency of:
Ans: A a. Retinol
66) A female presented with pain in hands b. Niacin
and joints with morning stiffness most c. Pyridoxine
likely suffering from: d. Ascorbic Acid
a. Osteoarthritis Ans: A
b. Rheumatoid arthritis Explanation: Can‘t see in dark is night blindness
c. SLE due to deficiency of Vitamin A (Retinol).
d. Septic arthritis 73) Perforation of posterior wall massive
Ans: B bleeding due to:
Explanation: a. Gastroduodenal artery
 RA: Morning stiffness present b. Right Gastric Artery
 OA: Morning stiffness if present less than c. Left Gastric Artery
30 minutes d. Right Colic Artery
67) Which of the following increase Ans: A
potassium in ECF? Explanation:
a. Strenuous exercise  Perforation of anterior wall of duodenum
b. AG2 leakage of content into greater sac, right
c. Renin paracolic gutter & right iliac fossa.
d. Aldosterone  Perforation of posterior wall massive
Ans: A bleeding due to gastroduodenal artery.
68) Grading of Tumour: 74) Small intestine pain is felt at:
a. Extent of Spread a. Umbilicus
b. Degree of Differentiation b. Epigastrium
c. Both c. Hypogastrium
d. None d. Pelvis
Ans: B Ans: A
Explanation: 75) Notching on CXR is seen in:
 Grading of tumor: Degree of a. Aortic arch aneurysm
differentiation b. Pre ductal coarctation of Aorta
 Staging of tumor: Extent of Spread c. Post ductal coarctation of Aorta
69) Which of the following drug is actively d. Bicuspid Aortic Valve
secreted by kidneys? Ans: C
a. Penicillin Explanation: Notching on CXR due to dilated
b. Streptomycin intercostals arteries eroding the lower boarders of
c. Rifampin ribs.
d. Metronidazole 76) Hypophyseal portal system to:
Ans: A a. Neurohypophysis
70) Which of the following anesthetic is b. Adenohypophysis
contraindicated with increase ICP? c. Adrenal Glands
SK Origional – Golden 11 76

d. Posterior pituitary gland c. The End of 4th Week


Ans: B d. Start of 4nd Week
77) Artery arising from posterior aspect of Ans: B
bifurcation of aortic arch: 83) Where does ectoderm and endoderm
a. Median sacral meets?
b. Middle aortic a. Clocacal membrane
c. Common iliac b. Small Intestine
d. External iliac c. Vitelline duct
e. Internal iliac d. Urachus
Ans: A Ans: A
Explanation: The Median sacral artery or middle 84) RBB is supplied by:
sacral artery is a small vessel that arises from a. RCA
posterior to the abdominal aorta & superior to its b. LCA
bifurcation. c. Marginal Artery
78) A shoulder separation that involves d. Apical Artery
the lateral end of the clavicle sliding Ans: B
onto the superior aspect of acromion 85) Population divided into subgroups
would likely result from damage to: and then given random treatment:
a. Coaracoacromial ligament a. Sample sampling
b. Coracoclavicular ligament b. Stratified Random sampling
c. Costoclavicular ligament c. Complex sampling
d. Manubrosternal ligament d. Frequent Sampling
Ans: B Ans: B
Explanation: Snell BCQ 86) Which of the following does show
79) Which of the following helps in mean?
rotation of radio-ulnar joint? a. Sum of all and then divided by its number
a. Bicep b. Product of all
b. Tricep c. Division of all
c. Pronator teres & Supinator d. Sum of all minus Product of all
d. Brachioradialis Ans: A
Ans: C 87) After RTA with multiple ribs fracture
80) Supinator muscle is damaged now which of the following must be the
supination will be done with the help first treatment?
of: a. Fluid Replacement
a. Tricep b. Blood Replacement
b. Biceps brachii c. Platelets
c. Coarcobrachilis d. FFPs
d. Pronator Teres Ans: A
Ans: B 88) A female presented with history of
81) Post HIV surgery floor is washed fever since last 10 days now with
with: continuous chest pain that radiates to
a. 1% Hypochlorite back and left parasternal rustling
b. 3% Hypochlorite sound most likely suffers:
c. 2% Gluteraldehyde a. Myocarditis
d. 3% Gluteraldehyde b. Pericarditis
Ans: A c. Endocarditis
Explanation: d. Pleurisy
 Floor: 1% Hypochlorite Ans: B
 Instruments: 2% Gluteraldehyde 89) A female with sore throat & diagnosed
82) Primordial germ cells are formed at: with water house friedrichsen
a. The beginning of 3rd Week syndrome most likely caused by:
b. The end of 3rd Week a. Neisseria Meningitides
SK Origional – Golden 11 77

b. Gonorrhea d. Celiac nodes


c. Staph Aureus Ans: A
d. Streptococcus Pneumonia 96) Which of the following is present in
Ans: A posterior most opening of diaphragm?
Explanation: The commonest cause of Water a. IVC
house friedrichsen syndrome is N. Meningitides b. Azygous vein
and symptoms of sore throat is seen at initial c. Aorta, thoracic duct & azygous vein
stages. d. Hemiazygous Vein
90) Pinworm is caused by: Ans: C
a. Ingestion of larva Explanation: The posterior most opening of
b. Ingestion of egg of larva diaphragm contains ATA (Aorta, Thoracic Duct
c. Ingestion of worm & Azygous Vein).
d. None 97) During ACL tear what will be the
Ans: B direction of tibia over the femur?
91) Patient with constipation with mouth a. Anterior
taste change and blood picture shows b. Posterior
microcytic hypochromic anemia: c. Anterolateral
a. Ankylostoma duodenale d. Anteromedial
b. E. coli Ans: A
c. Ameboma Explanation:
d. Ascaris  In ACL tear the tibia will move anterior
Ans: A compare with femur and the femur will
92) Maximum pressure in aorta during: be posterior to tibia.
a. Rapid Ejection phase  In PCL tear the tibia will move posterior
b. Reduced Ejection phase compare with femur and the femur will
c. Isovolumetric Contraction be anterior to tibia.
d. Isovolumetric Relaxation 98) A pregnant lady from remote village
Ans: B died on the way to hospital, the
93) A person lost 2 ml of water by probable cause is:
sweating, later this was replaced by a. Amniotic Fluid Embolism
drinking of 2 ml of pure water, which b. Fat Embolism
of the following is correct regarding c. Pulmonary Embolism
this condition? d. Hemorrhage
a. Increase ECF Ans: A
b. Decrease ICF Osmolarity Explanation:Pregnant lady on way to hospital
c. Decrease ICF Volume having no history of RTA so the most likely cause
d. Increase ICF Volume is amniotic fluid embolism.
Ans: D 99) A pregnant lady after delivery in
Explanation: Increase ICF Volume > Decrease village by Dai presented with increase
ICF Osmolarity > Increase ECF Volume > PT, APTT and BT most likely
Decrease ECF Osmolarity. suffering from:
94) Which of the following is air borne a. ITP
preventable disease? b. TTP
a. HIV c. DIC
b. IM d. VwD
c. Tuberculosis Ans: C
d. Sarcoidosis 100) A child with fever and sore throat for
Ans: C one week now presented with
95) Lymphatic drainage of appendix: epistaxis and platelets count is 30,000
a. Superior mesenteric lymph nodes which of the following test need to be
b. Inferior mesenteric lymph nodes done?
c. Iliac nodes a. Anti-Platelet antibodies
SK Origional – Golden 11 78

b. PT d. Suxamethonium
c. BT Ans: A
d. CT Explanation: According to applied therapeutics
Ans: A there‘s no release of histamine by cisatracurium
Explanation: This is most likely case of ITP and that‘s why it‘s considered safe in asthmatic
to confirm we go with Anti platelet antibodies. patients.
101) A child presented with failure to thrive 105) Ulcer on posterior wall of Gastric
and multiple blood transfusion history antrum will lead to acute
to his brother which of the following abscess/Peritonitis in which of the
test need to be done? following space?
a. Ferritin a. Greater Sac
b. Hb Electrophoresis b. Lesser Sac
c. Serum Iron c. Right Sub hepatic Space
d. TIBC d. Right Posterior sub phrenic space
Ans: B Ans: B
102) Six months transplant rejected & Explanation:
reversed with immunosuppressive  Ulcer on posterior wall of duodenum or
drugs is most likely: Gastric antrum will lead to fluid
a. Acute Cellular Rejection accumulation in: Lesser Sac
b. Chronic Rejection  Anterior duodenal perforation the
c. Preformed Antibodies contents will accumulates in: Posterior
d. None of the above subphrenic space
Ans: A  In erect posture in case of Duodenal
Explanation: For Acute & chronic transplant perforation the contents will collect in:
rejection 6 months is the boundary line, here the Right Iliac Fossa > Right Paracolic gutter
reversal is in favor of acute as chronic is 106) Which of the following is locally
irreversible. malignant tumor?
103) Lateral cord of brachial plexus a. Melanoma
supplies which of the following b. Squamous cell carcinoma
muscle? c. BCC
d. Adenoid cystic carcinoma
a. Biceps Brachii Ans: C
Explanation:
b. Deltoid  Locally Malignant tumor is Basal cell
carcinoma
c. Serratus Anterior  Locally spreading tumor is
Ameloblastoma
d. Diaphragm  More Aggressive tumor is Melanoma
107) Which of the following is most
Ans: A common benign tumor?
a. Pleomorphic Adenoma
Explanation: The lateral cord of brachial plexus b. Warthin Tumor
is made by confluences of anterior division of c. Mucoepidermoid Ca
upper & middle trunk of brachial plexus and d. SCC
musculocutaneous nerve one of its branch Ans: A
supplies bicep brachii. Explanation:
104) Muscle relaxant that can be given to  Most common malignant:
asthmatic patient: Mucoepidermoid
a. Cisatracurium  Most common Benign: Pleomorphic
b. Atracurium Adenoma
 Most common in smokers: Warthins
c. Succinylcholine
SK Origional – Golden 11 79

108) Lateral side of foot and from the back 114) A British woman went to Thailand
and lateral side of calf lymphatic and presented with fever, rigors &
drains into: chills is most likely suffering from:
a. Vertical group Lymph nodes a. Malaria
b. Horizontal group lymph nodes b. Infectious Mononucleosis
c. Inguinal lymph nodes c. Enteric Fever
d. Popliteal lymph nodes d. AIDS
Ans: D Ans: A
109) Atrophy of calf muscles due to: Explanation:Thailand is geographically more
a. Upper motor neurons lesion related to malaria & the findings are suggestive for
b. Lower motor neurons lesion malaria too those are Fever, rigors & specially
c. Spastic paralysis chills.
d. Atonic paralysis 115) Patient who is known case of
Ans: B Hypothyroidism which of the
110) Which of the following muscle following will increase in the blood:
produce yawing & increase cardiac a. TSH
output? b. T3
a. Lateral pterygoid muscle c. T4
b. Medial pterygoid muscle d. Cholesterol
c. Masseter Ans: D
d. Buccinators Explanation: TSH can increase only in primary
Ans: A hypothyroidism but not in other cases while
111) Which of the following paraneoplastic cholesterol is increase in all hypothyroidism cases
syndrome is associated with Oat cell so we prefer Cholesterol > TSH.
carcinoma of lungs? 116) A tall man on oxygen suddenly started
a. ACTH being dyspnic due to:
b. Insulin a. Spontaneous pneumothorax
c. Glucagon b. Tension pneumothorax
d. Androgen c. Cardiac tamponade
Ans: A d. COPD
Explanation: Oat cell carcinoma of lungs that‘s Ans: A
small cell carcinoma of lungs produces ACTH & 117) Which of the following is lymphatic
ADH. draining of fundus of stomach?
112) Which of the following nerve supply a. Celiac Nodes
obtruator externus muscle: b. Inferior mesenteric node
a. Anterior division of obtuator nerve c. Superior mesenteric node
b. Posteriro division of obtruator nerve d. Inguinal node
c. Femoral nerve Ans: A
d. Sciatic nerve 118) CO2 during first minute of apnea:
Ans: B a. 6mmhg
Explanation: Obtruator externus muscle is b. 10mmhg
supplied by the posterior division of the obtruator c. 8mmhg
nerve (l3 and l4) originating from the lumber d. 12mmhg
plexus. Ans: A
113) S2 – S4 at the level of: Explanation:
a. L1 – 2  During first min of apnea in conscious
b. S1 – 2 person CO2 : 6mmhg
c. S3 – 4  If Anesthetized then : 10mmhg
d. L4 – 5 119) Patient presented with decrease
Ans: A peripheral pulsation with abdominal
pain and creatine kinase normal and
pulsating abdominal mass with
SK Origional – Golden 11 80

fasting glucose of 130 – 150 from last 125) During surgery profuse bleeding from
20 years is most likely suffering from: transverse cervical artery which of the
a. Thoracic aortic Aneurysm following to be ligated?
b. Abdominal Aortic Aneurysm a. Thyrocervical Trunk
c. Rupture of IVC b. Aorta
d. Common Iliac artery Aneurysm c. Vertebral Artery
Ans: B d. Carotid Artery
120) Lymphatic drainage of external nose Ans: A
& anterior part of nasal cavity: Explanation: The transverse cervical artery is the
a. Submandibular Lymph Nodes next branch off the thryocervical trunk.
b. Deep cervical lymph nodes 126) Surfactant deficiency will lead to:
c. Retropharyngeal Lymph nodes a. ARDS
d. Occipital Lymph Nodes b. Abdominal muscle paralysis
Ans: A c. IBS
Explanation: d. RA
 External nose & Anterior nasal cavity Ans: A
drained by submandibular lymph nodes 127) In fasting during summer urine
 Posterior Nasal cavity drained by osmolarity is increased due to:
retropharyngeal lymph nodes & JD nodes a. ADH
121) Male presented with Limb weakness b. Oxytocin
& respiratory symptoms after c. Epinephrine
consuming canned pack foods: d. NE
a. Botulism Ans: A
b. GBS Explanation: During fasting when ECF volume
c. MG is decreased ADH is secreted that leads to
d. Staph Aureus retention of water and thus increasing urine
Ans: A osmolarity.
122) Which of the following is the 128) ESR more than 100 is seen in:
characteristic of Botulinism? a. RA
a. Paralysis of Abdominal muscles b. OA
b. Paralysis of Respiratory muscles c. Multiple Myeloma
c. Death d. SLE
d. Cardiac arrest Ans: C
Ans: B 129) Renin is persistently increased in:
123) A child presented with a. Malignant phase of Essential HTN
thrombocytopenia, immunodeficiency b. Hypotension
& eczema is most likely suffering c. CCF
from: d. MI
a. Wiskott Aldrich Syndrome Ans: A
b. IGA Deficiency 130) Urologist doing Transurethral surgery,
c. Allergic Rhinitis 2 arteries at 5 & 7 clock position
d. SLE started bleeding and his way is
Ans: A blocked, name the arteries:
124) Candidiasis in child is most likely due a. Floch‘s Artery
to: b. Badenoch Arteries
a. IGA Deficiency c. Branches of Vesicle Artery
b. Immunosuppressive d. Capsular Artery
c. Immunocompetent Ans: B
d. None of the above 131) Supernumerary rib will compress:
Ans: B a. Brachial plexus & subclavian vessels
b. Brachial branches
c. Roots
SK Origional – Golden 11 81

d. Aorta 138) Internal spermatic fascia formed by:


Ans: A a. External Oblique muscle
132) Regarding weber syndrome: b. Internal Oblique muscle
a. Contralateral paralysis with ipsilateral c. Fascia Transversals
ptosis and lateral deviation of eye d. Fascia Iliac
b. Ipsilateral paralysis with ipsilateral ptosis Ans: C
c. Ipsilateral Paralysis & contralateral ptosis Explanation:
d. Contralateral ptosis & ipsilateral weakness  External oblique muscle forms external
Ans: A spermatic fasica.
Explanation: Weber‘s Syndrome, also known as  Internal oblique muscle forms cremasteric
superior alternating hemiplegia, a form of stroke fascia.
that affects the medial portion of midbrain. It‘s  Fascia transversalis forms internal
characterized by the presence of ipsilateral spermatic fascia.
occulomotor nerve palsy & contralateral 139) What are chondrocytes?
hemiparesis or hemiplegia. a. Part of Bone
133) Submucosal Fibrosis (SMF) is: b. Mature cartilage
a. Premalignant c. Mature bone
b. Benign d. Immature cartilage
c. Malignant Ans: B
d. Dysplasia Explanation: Chondrocytes are cells of mature
Ans: A cartilages.
Explanation:Sub mucosal fibrosis is a 140) Middle meningeal artery is a branch
premalignant condition of oral cavity. of:
134) Malignant feature: a. Ophthalmic artery
a. Pleomorphism b. Maxillary Artery
b. Metastasis c. Mandibular Artery
c. Desmoplasia d. Facial artery
d. Increase N/C ratio Ans: B
Ans: B 141) Which of the following nerve damage
Explanation: Malignancy is characterized by: leads to winging scapula?
Metastasis > Invasion > Pleomorphism > a. Accessory Nerve
Increase N/C Ratio. b. Long thoracic nerve
135) Metastasis due to: c. Axillary Nerve
a. Decrease in E Cadherin d. Phrenic Nerve
b. Increase in E Cadherin Ans: B
c. Invasion of Epithelium Explanation: Long thoracic nerve supplies
d. Loss of Actin serratus anterior muscle & damage to long
Ans: A thoracic nerve will lead to winging scapula.
136) Which of the following is the role of 142) After sweating which of the following
cytokeratin? electrolyte maintain homeostasis:
a. Detect carcinoma a. Potassium
b. Detect Dyspasia b. Calcium
c. Metastasis c. Sodium
d. Invasion d. Cadmium
Ans: A Ans: C
137) Which of the following nerve damage 143) A 70 years old with recurrent UTI
leads to foot drop? most likely due to:
a. Femoral Nerve a. Outflow obstruction
b. Sciatic Nerve b. Carcinoma
c. Common Peroneal nerve c. Pyelonephritis
d. Obtruator nerve d. Renal Failure
Ans: C Ans: A
SK Origional – Golden 11 82

Explanation: This is most likely due to BPH b. Partially compensated respiratory acidosis
leading to outflow obstruction. c. Partially compensated metabolic alkalosis
144) An Old man with recurrent UTI: d. Mixed Acidosis
a. BPH Ans: C
b. Carcinoma Explanation: As the PH is in alkalosis range now
c. Pyelonephritis we look toward the HCO3 that‘s in alkalosis range
d. Renal Failure too so we keep metabolic alkalosis in mind, after
Ans: A looking to PCO2 that‘s in Acidotic range shows
145) Hb 4.5 with indirect jaundice 4 and the struggle toward compensation but not fully
reticulocytes 10 most likely: compensated yet hence, this is case of partially
a. IDA compensated metabolic alkalosis.
b. Hemolytic anemia 150) A patient presented with Post OP
c. Thalassemia hypotension & abdominal swelling
d. Siderblastic anemia most likely due to:
Ans: B a. Hypovolemic shock
146) A pregnant lady presented with b. DIC
obstructive Jaundice features which of c. Cardiogenic shock
the following investigation will d. Cardiac arrest
confirm the diagnosis? Ans: A
a. GGT 151) Droplet infection spread by:
b. Bilirubin a. 1 – 3 feet
c. ALP b. 1 – 3 Meter
d. AST c. 3 – 6 feet
Ans: A d. 1 – 6 feet
Explanation: ALP is used for diagnosis of Ans: A
Obstructive jaundice but in pregnancy ALP is Explanation:
increased so we go for GGT.  Droplet infection : 1 – 3 feet
147) A 30 years old male suddenly died  Airborne infection: 3 – 6 feet
after a sudden rise in blood pressure, 152) Thrombocytopenia is caused by:
at autopsy the ruptured vessel a. Quinine
completely lack the tunica media at b. Thiazide
the point of aneurysm most likely c. Insulin
aneurysm is: d. ACEI
a. Berry Aneurysm Ans: A
b. Dissecting Aneurysm Explanation: Heparin > Quinine > Thiazide
c. Marfan Aneurysm 153) 5 years after transplant surgery
d. Mycotic Aneurysm malignancy is:
Ans: A a. Skin
Explanation: Dissecting Aneurysm has thin b. Lymphoproliferative
tunica media while berry aneurysm has absent c. Rectal
tunica media. d. Lungs
148) Which of the following delays gastric Ans: B
emptying? Explanation:
a. CCK  The commonest malignancy after
b. Gastrin transplant is: Skin
c. Secretin  Before 5 years: Skin
d. Insulin  After 5 years: Lymphoproliferative
Ans: A 154) Anti sera agglutinates with A and D
149) PH = 7.49, PCO = 60, HCO3 = 30 and plasma has Anti B antibodies
which of the following Acid Base what‟s blood group?
abnormality does it indicates? a. A –ve
a. Respiratory Acidosis b. A + Ve
SK Origional – Golden 11 83

c. O –ve b. Loss of sensation on lateral aspect of


d. B +Ve hand
Ans: B c. Loss of sensation on medial aspect of
Explanation: As anti-sera agglutinates A and D hand
that means the group has A & D (Rh) Antigen d. Loss of sensation on dorsal aspect of
and as well plasma has Anti B antibodies that‘s hand
why it‘s A +ve that has Antigen A & Rh with Ans: B
antibody B. Explanation:
155) Half-life of dopamine is 2 minutes,  Do not confuse yourself with spared
after how much time will it reach sensation on thenar eminence because
steady state concentration: lateral side of hand has palm and fingers
a. 2 minutes too, not only thenear eminence.
b. 4 minutes  Wasting of thenar eminence can occur
c. 6 minutes but that takes time.
d. 9 minutes 161) Vascular resistance in response to
e. 2 hour surgery is due to:
Ans: D a. Cortisol
Explanation: A drug reaches after 4-5 half-lives b. ACTH
to its steady state concentration so here half-life is c. Catecholamine‘s
2 minutes for dopamine and will reach to steady d. Aldosterone
state in 4-5 half-lives that‘s 8 – 10 minutes so we Ans: C
pick 9 minutes here. Explanation:
156) Fisherman with gum swelling and  Vascular resistance in response to surgery
bleeding most likely due to deficiency is due to adrenaline.
of:  Stress hormone is cortisol.
a. Vitamin A 162) Thyroid hormone at physiological
b. Vitamin B condition causes:
c. Vitamin C a. Fatty Acid Oxidation
d. Vitamin D b. Protein Synthesis
Ans: C c. Gluconeogenesis
157) Acidosis and hypokalemia fluid loss d. Entry of potassium into cells
from: Ans: B
a. Small intestine Explanation: The general effect of thyroid
b. Colon hormone is to activate nuclear transcription of
c. Jejunum large numbers of gene therefore, in virtually all
d. Ileum cells of body, great number of protein enzymes,
Ans: B structural proteins are synthesized. (Guyton)
158) Breaking bad news: 163) Erythroblastosis fetalis have which of
a. Attending doctor should tell in exclusive the following HSR:
formal session a. Type I HSR
b. Should tell on bed side b. Type II HSR
c. No need to tell c. Type III HSR
d. Tell them directly d. Type IV HSR
Ans: A Ans: B
159) Erythropoietin is secreted during: 164) Muscle which elevates palate derived
a. Hypoxia from?
b. Hyperoxia a. 1st Arch
c. Hypercapnia b. 6th Arch
d. Low Altitude c. 4th& 6th Arch
Ans: A d. 4th& 5th Arch
160) Carpal tunnel syndrome causes: Ans: C
a. Wasting of thenar eminence
SK Origional – Golden 11 84

165) Which of the following is sensitive test 171) A pregnant lady presented with Hb 9.6
for diabetic nephropathy? gm/dl, MCV 58, Low MCH and
a. Microabluminiuria normal serum ferritin and TIBC which
b. Creatinine clearance of the following next text will confirm
c. Inulin diagnosis:
d. BUN a. Serum Iron
Ans: A b. HbA2
166) Child with periorbital edema and c. HbA1
generalized edema due to: d. Fetal Hb
a. Hypoalbuminemia and increase salt water Ans: B
retention Explanation: As MCV is low along with MCH
b. Hyperalbuminemia but Serum Ferritin and TIBC level is normal so
c. Increase Salt retention the next best test is HbA2 to look for thalassemia.
d. Decrease Salt retention 172) HCO3 is absorbed at which of the
Ans: A following site?
167) Which of the following is Hemophilia a. PCT
A factor? b. PCT + DCT
a. Factor 8 Major c. PCT + CT
b. Factor 8 Minor d. CT
c. Factor 7 Ans: C
d. Von Willibrand factor 173) A baby after an obstructed delivery
Ans: B presented with inability to abduct the
168) Which of the following isn‟t true arm, loss of flexion & loss of
regarding G6PD deficiency? supination of arm at elbow joint due
a. X – linked disease to damage to:
a. Upper Trunk of Brachial Plexus (C5-C7)
b. Rarely affect male
b. Lower Trunk of Brachial Plexus
c. Protects against P. falciparum c. Radial Nerve
d. Genetic disorder d. Ulnar Nerve
Ans: B Ans: A
Explanation: G6PD is a genetic disorder that‘s X Explanation: In Upper Trunk of brachial Plexus
– Linked & commonly affects male & is protective there is mostly damage to musculocutaneous,
against P. falciparum. Axillary and suprascapular nerve that leads to loss
of Abduction, flexion & Supination of forearm
169) Best measure of renal plasma flow:
a. Creatinine Clearance
b. Inulin Clearance
c. PAH
===============
d. Glucose
Ans: C
Explanation:
 Best Measure of RPF = PAH
 Best Measure of GFR = Inulin Clearance
 Best Estimate of GFR = Creatinine
Clearance
170) Local anesthetic affects which of the
following fibers?
a. A fibers
b. B fibers
c. C fibers
d. A delta
Ans: C
SK Origional – Golden 11 85

ALL MEDICINE & ALLIED


September – November, 2020 (Papers)
SK Origional – Golden 11 86

MEDICINE & ALLIED Explanation:As this is the case of re-infection so


th
September 29 , 2020 - Afternoon it‘s like acute infection after the first one was
treated & cured hence in acute inflammation the
level of Neutrophils will raise again.
1) Diplotene stage in which phase:
A. Prophase 5) Philadelphia chromosome
translocation 9:22 is present in which
B. Metaphase
of the following:
C. Interphase
A. CML
D. Telophase
B. ALL
Ans: A
C. AML
Explanation:All primary oocytes are formed by
D. CLL
the fifth month of fetal life and remains dormant
Ans: A
in prophase (Diplotene stage) of meiosis I until
puberty. 6) Megaloblastic anemia is caused due to
deficiency of which of the following?
2) Warfarin Mechanism of Action:
A. Vitamin B1 Deficiency
A. Competitively inhibits the vitamin K
B. Vitamin B12 Deficiency
epoxide reductase complex 1 (VKORC1)
C. Folic Acid Deficiency
B. Non-competitively inhibits Vitamin D
D. Niacin Deficiency
reductase
Ans: C
C. Directly inhibit Factor 12
Explanation:Megaloblastic Anemia is caused due
D. Indirectly inhibit factor X
Ans: A to deficiency of Either Vitamin B12 or Folic Acid
but as here no hint is given so we choose Folic
Explanation:Warfarin competitively inhibits the
acid as that‘s most commonly depleted than
Vitamin K epoxide reductase complex 1
Vitamin B12 which takes years to deplete, if
(VKORC1), which is an essential enzyme
history of gastrectomy was given then we can
for activating the vitamin K available in
prefer Vitamin B12.
the body. Through this mechanism
7) Most common mode of inheritance of
warfarin can deplete functional vitamin K
congenital heart disease is:
reserves and therefore reduce the
synthesis of active clotting factors. A. Autosomal Dominant
B. Multifactorial
3) Cimetidine interaction with warfarin:
C. Autosomal Recessive
A. Increase hepatic clearance of warfarin
D. X-Linked Recessive
B. Decrease hepatic clearance of warfarin
Ans: B
C. Increase half-life of warfarin
8) Which of the following is the karotype
D. Decrease half-life of warfarin
for down syndrome?
Ans: B
A. Trisomy 13
Explanation:
B. Trisomy 18
 Cimetidine is potent CYP3A4 enzyme
C. Trisomy 21
inhibitor & thus decrease the hepatic
D. Trisomy 24
clearance of warfarin
Ans: C
4) A patient giving history of some
Explanation:
infection 1 month back, now presents
 Patau Syndrome: Trisomy 13
with same symptoms and same nature
of pain in same area. What type of  Edward Syndrome: Trisomy 18
inflammatory cells will be found at  Down Syndrome: Trisomy 21
this time of the disease: 9) Left Gastric Artery is a branch of:
A. Neutrophils A. Celiac Trunk
B. Lymphocytes B. Common Hepatic Artery
C. Eosinophils C. Gastroduodenal Artery
D. Basophils D. Superior Mesenteric Artery
E. Macrophages Ans: A
Ans: A
SK Origional – Golden 11 87

Explanation:The Left Gastric Artery is the first Ans: A


and smallest branch of the celiac trunk which Explanation:
descends along the superior half of the lesser  Short term memory loss is due to lesion
curvature of the stomach and anastomosing with of Hippocampus while long term memory
the right gastric artery. changes are due to protein synaptic
10) Direct inguinal hernia is located: changes.
A. Medial to Inferior epigastric vessels 14) Typhoid Diagnostic test:
B. Lateral to Inferior epigastric vessels A. Widal Test
C. Medial to Superior epigastric vessels B. Immunoassay
D. Lateral to Superior epigastric vessels C. Westeron blot Test
Ans: A D. CD4 Count
Explanation: Ans: A
 Direct inguinal hernia lies medial to 15) Heart failure causes:
inferior epigastric vessels A. Increase contractility
 Indirect inguinal hernia lies lateral to B. Increase SV
inferior epigastric vessels C. Increase HR
11) Bronchogenic carcinoma is due to D. Edema
exposure to: Ans: D
A. Asbestosis Explanation:Heart failure leads to back pressure
B. Nickel & leading to dependent edema.
C. Cadmium 16) Women with Hb 9 gm/dl, after 2
D. Carbon months of iron therapy Hb 10 gm/dl.
Ans: A Which of the following investigations
Explanation: will you do for persistent Anemia:
 Bronchogenic carcinoma & mesothelioma A. Ferritin
is caused by exposure to asbestosis B. Hb Electrophoresis
 Bronchogenic carcinoma > Mesothelioma C. Iron Profile
on exposure to Asbestosis D. Folate level
12) A 40 years old hypertensive man had Ans: B
thalamic hemorrhage, the most Explanation:As the patient is already on iron
prominent finding on the examination supplements since last two months and still not
of his sensory system will be: improving so we have to look for other causes
A. Abnormal thermal sensation that‘s thalassemia and we opt Hb electrophoresis.
B. Altered sexual sensations 17) Sarcoidosis is diagnosed
C. Hyperaesthesia microscopically by:
D. Numbness A. Granuloma with Asteroid bodies
E. Reduced kinaesthesia B. Non-Caseating Granuloma
Ans: C C. Caseating Granuloma
Explanation:Thalamic Pain Syndrome affects the D. Epitheloid Cells
ventral nuclear group. Patients present with Ans: B
burning, aching pain in contralateral limbs or body Explanation:Both Microscopic & Histological
involvement of the DCML part of VPL increases features of Sarcoidosis is Non-Caseating
the sensitivity to pain and presents as contralateral granuloma, Don‘t confuse Microscopic feature
loss of vibratory sense and gait ataxia. Thalamic with grauloma with Asteroid bodies.
pain syndrome is resistant to Analgesic 18) Virus causes cancer by alteration of:
medications. A. Protein synthesis
13) Short term memory loss occurs due to B. Oncogenes
to lesion of: C. DNA
A. Hippocampus D. Genome
B. Parahippocampal Ans: A
C. Neocortex Explanation:
D. Frontal Lobe  Virus has Oncogenes
SK Origional – Golden 11 88

 Virus cause cancer by Proto-Oncogenes C. Atrophy of Mucosal Glands


 Virus causes cancer by alteration of D. Enlargement of Air spaces
protein synthesis Ans: A
19) Virus Cause cancer by: Explanation:Histological findings of chronic
A. Oncogenes bronchitis is characterized by hyperplasia of
B. Proto-Oncogenes mucus secreting glands in bronchi that‘s Reid
C. DNA index (thickness of mucosal gland layer to
D. RNA thickness of wall between epithelium and
Ans: B cartilage) > 50%.
20) Which intermediate filament present 24) Sternocostal surface of heart is formed
in mesenchme: by:
A. Desmin A. Right Ventricle
B. Lamin B. Left Ventricle
C. Vimentin C. Right Atrium
D. Vesmin D. Left Atrium
Ans: C Ans: A
Explanation: Explanation:
 In mesenchyme: Vimentin  Right boarder of heart is formed by: Right
 In Muscles: Desmin Atrium
 In Epithelial Cells: Keratin  Left boarder of heart is formed by: Left
21) Calculate Alveolar Ventilation when Ventricle
Tidal Volume is 500 ml & Respiratory  Inferior boarder of heart is formed by:
Rate is 10: right Ventricle
A. 2500 ml  Sternocostal (anterior) surface of heart is
B. 3000 ml formed by: right ventricle
C. 3500 ml  Diaphragmatic (inferior ) surface of heart
D. 4500 ml is formed by: left ventricle
Ans: C  Base (posterior) surface of heart is
Explanation: formed by: Left Atrium
 Alveolar Ventilation = (Tidal Volume – 25) In an RTA inferior pelvis Avulsion
Dead Space) RR muscle involved is:
As per normal calculation Dead space is A. Rectus Femoris
150 ml B. Quadricep Femoris
 Alveolar Ventilation = (500-150) * 10 C. Adductor Longus
 350 * 10 D. Adductor Brevis
 3500 ml Ans: A
22) Which of the following is laboratory Explanation:
findings in case of hyperthyroidism:  Inferior pelvic Avulsion muscle: Rectus
A. Increase TSH Femoris
B. Increase T3  Superior Pelvic Avulsion muscle:
C. Increase rT3 Sartorius
D. Decrease T4 26) The main difference between
Ans: B Anaphylactic shock & hypovolumic
Explanation: shock is:
 In hyperthyroidism there will be Increase A. Decrease CO
T3, T4 and Decrease TSH B. Increase CO
 In hypothyroidism there will be decrease C. Increase TPR
T3,T4 and increase TSH D. Vasoconstriction
23) Histological findings of chronic Ans: B
bronchitis: Explanation:
A. Reid Index > 50 %  In Hypovolumic shock there‘s decrease
B. Reid Index < 50% blood volume leading to decrease venous
SK Origional – Golden 11 89

return & decrease EDV, Decrease SV and A. Berry Aneurysm


hence Decrease Cardiac Output. B. Dissecting Aneurysm
 In Anaphylactic Shock there is peripheral C. Marfan Aneurysm
vasodilation leading to decrease Blood D. Mycotic Aneurysm
Pressure that in turn through Ans: A
baroreceptors increase HR & CO. Explanation:Dissecting Aneurysm has thin
27) DIC is related to all except: tunica media while berry aneurysm has absent
A. Cancer tunica media.
B. Amniotic fluid embolism 33) ECG findings of Hypokalemia:
C. Sepsis A. U wave
D. OCPs B. J wave
E. Bacteremia C. Peaked tall T wave
Ans: D D. Inverted QRS complex
Explanation: Ans: A
 As all others causes mentioned in Explanation:
Robbins and Goljan hence I prefer OCPs  Hypokalemia most prominent feature is
best among others U wave
 OCPs might indirectly somehow causing  Tall peaked T wave is seen in
it still not mentioned as direct cause. Hyperkalemia
28) ASO-Titer is used as marker for:  J wave also called Osborn wave is seen in
A. PSGN Hypothermia
B. Diabetic nephropathy 34) Which of the following hormone act
C. Minimal change disease on Na-K Pump:
D. Proliferative diseases A. Thyroxin
Ans: A B. GH
Explanation:ASO-Titer is used in PSGN (Post C. Cortisol
streptococcal Glomerulonephritis). D. Dopamine
29) Which of the following is used as Ans: A
marker for Autoimmune disease: Explanation:Both Insulin & thyroxine acts on
A. Anti DsDNA Na-K pump but prefer Insulin if present.
B. Anti RNA 35) Growth Hormone needs which of the
C. ANA following hormone for sufficient and
D. Anti-Rho effective function:
Ans: C A. Cortisol
30) Sjogren syndrome marker is: B. Aldosterone
A. Anti SSA C. Somatomedin
B. Anti DsDNA D. Glucagon
C. Anti RNA Ans: C
D. ANA Explanation:
Ans: A  GH needs Somatomedin C ( IGF-I) for
31) Venous thrombosis results in: effective and efficient work
A. Pulmonary Embolism 36) A patient presented with episodic
B. Thrombophlebitis hypertension, headache, diaphoresis
C. Peripheral gangrene & palpitations what could be the
D. Neuropathy possible cause:
Ans: A A. Pheochromocytoma
32) A 30 years old male suddenly died B. CRF
after a sudden rise in blood pressure, C. Adrenal Tumour
at autopsy the ruptured vessel D. Renal Cell Carcinoma
completely lack the tunica media at Ans: A
the point of aneurysm most likely Explanation:The classic triad of
aneurysm is: Pheochromocytoma is headache, diaphoresis and
SK Origional – Golden 11 90

palpitations though only 40% individuals present  Aplasia is failure of cell production during
with these symptoms. embryogenesis
37) Which GI Hormone stimulates HCO3  Hypoplasia is decrease cell production
and water secretion from liver & bile during embryogenesis
production: 42) Which of the following is functional
A. CCK remanent of mesonephric duct in
B. Somatostatin males:
C. Secretin A. Seminal Vesicle
D. Histamine B. Epididymis
E. None C. Ductus Deferens
Ans: C D. Sertoli Cells
38) Atrial fibrillation ECG findings: Ans: C
A. Absent P-Waves 43) External auditory meatus develops
B. Irregular R-R Interval from:
C. Regular R-R Interval A. 1st branchial arch
D. Prominent QRS B. 2nd branchial arch
Ans: B C. 1st branchial cleft
Explanation:A-Fib is characterized by both D. 2nd branchial cleft
absent P wave & Irregular R-R Interval but Ans: C
irregular R-R interval more than Absent P wave. 44) Biopsy in neck region followed by ear
39) In 2nd degree heart block which lobe numbness which nerve is
statement is correct: effected:
A. Stroke volume is decreased A. Greater Auricular nerve
B. Stroke Volume is increased B. Auriculotemporal nerve
C. Ventricular rate is greater than Atrial rate C. Lessesr auricular nerve
D. Atrial rate is greater than ventricular rate D. Temporal nerve
Ans: D Ans: A
40) Theta Wave appears in which stage of Explanation:
sleep:  During leprosy there is hanging of ear
A. Alert Wakefulness lobe why: greater auricular nerve
B. Quiet Wakefulness  Ear lobe is supplied by which nerve:
C. Stage 2 greater auricular nerve
D. Stage 4  Ear lobe numbness: greater auricular
Ans: C nerve
Explanation: 45) Vocal cord paralysis is due to
 According to Guyton: Alert Wakefulness involvement of which of the following
(Beta Wave) nerve:
 Quiet Wakefulness (Alpha Wave) A. External Laryngeal nerve
 Stage 1: Low voltage & Spindles B. Internal Laryngeal nerve
 Stage 2 & 3: Theta Wave C. Recurrent Laryngeal nerve
 Stage 4: Delta Wave D. Superior Laryngeal nerve
 While in FA theta wave in stage 1 NREM Ans: C
 Follow Guyton 46) 60 years old man chewing pan for the
41) Dysplasia refers to: last 20 years presented with an ulcer
A. Disordered Cellular growth over the tongue is most likely:
B. Failure of cell production A. Submucosal fibrosis
C. Decrease in cell production B. Leukoplakia
D. Increase in cell production C. Erythroplakia
Ans: A D. Oral cancer
Explanation: Ans: A
 Dysplasia refers to disordered cellular Explanation:
growth
SK Origional – Golden 11 91

 Pan Chewing has an association with pituitary gland & known as neurosecretory
Submucosal fibrosis hormones.
47) A person having blood group A+ve 52) CO2 transport is an example of:
will develop transfusion reaction if A. Positive feedback
transfused with: B. Negative feedback
A. O+ve C. Feedback forward mechanism
B. A+ve D. Baroreceptor
C. B+Ve Ans: B
D. A-Ve 53) During moderate exercise which of
Ans: C the following organ is not effected:
Explanation:A person having blood group A can A. Kidneys
only take blood from A or O which is universal B. Liver
donor & will develop transfusion reaction if C. Brain
transfused with blood group B+ve. D. Heart
48) Most common risk of giving systemic Ans: C
steroids to diabetic patients: Explanation:During moderate exercise blood
A. Deranged diabetic control flow to brain remains unchanged.
B. Cataract 54) The body will lose temperature via:
C. Neuropathy A. Anterior Hypothalamus
D. Nephropathy B. Posterior Hypothalamus
Ans: A C. Anterior Pituitary
49) A patient presented with lesion in the D. Cortex
nose, granulomas & proteinuria with Ans: A
vasculitis, histology shows necrosis Explanation:The body loses heat via activation
most likely lesion is: of Anterior hypothalamus that‘s AC for the body.
A. Leprosy 55) Cleft palate due to malformation of:
B. TB A. Palatine shelves
C. Sarcoidosis B. Maxillary prominence
D. Wegner granulomatosis C. Medial nasal prominence
Ans: D D. Lateral nasal prominence
Explanation:Wegner Granulomatosis involves Ans: A
Nasopharynx & renal system. Explanation:
50) Which of the following is the action of  Cleft palate due to malformation of
Oxytocin: palatine shelves
A. Ejection of Milk  Cleft lip due to failure of fusion of medial
B. Production of Milk nasal prominence and maxillary
C. Maintenance of pregnancy prominence
D. Growth of fetus 56) Commonest side effect of any drug is:
Ans: A A. Seizure
Explanation:The two main action of Oxytocin in B. Nausea & Vomiting
the body are contraction of the uterus during C. Hypotension
childbirth & ejection of milk. D. Hypertension
51) Which of the following is Ans: B
neurosecretory hormone? 57) 30 years old male treated for
A. ADH meningitis 1 month ago came with
B. Cortisol morning headache having B/L
C. GH papilledema cause is:
D. Insulin A. Blockage of dural sinus
Ans: A B. Hydrocephalus
Explanation:Oxytoxin & ADH are produced in C. Involvement of cerebra l vessels
the hypothalamus & transported to posterior D. Involvement of meninges
E. Subdural collection
SK Origional – Golden 11 92

Ans: B D. Increase tension of lens ligament


Explanation:Hydrocephalus is one of the Ans: C
complications of Meningitis & presents with B/L 64) If HR is increased what will be its
papilledema & morning headaches. effect on cardiac cycle:
58) Patient suffering from anemia has A. Decreases
presented with gall stones & increase B. Increases
unconjugated bilirubin, anemia is: C. No effect
A. Hemolytic Anemia D. Increase & then Decrease
B. Vitamin B12 deficiency Ans: A
C. Folate deficiency 65) 70 years old male has TBW of 42 Liter
D. Megaloblastic Anemia how much fluid will be present in
Ans: A interstitial space:
59) Which of the following vitamin is used A. 28 L
as prophylaxis in ileum resection? B. 11 L
A. Vitamin B1 C. 15 L
B. Vitamin B12 D. 20 L
C. Vitamin B2 Ans: B
D. Folate Explanation:TBW of 42 L, ICF is 2/3rd that‘s 28
Ans: B L, ECF is 1/3rd that‘s 14 L, Interstitial fluid is
Explanation:Vitamin B12 is reabsorbed in 3/4th of ECF that‘s 10.5L.
terminal ileum & after ileum resection Vitamin 66) Clark Nucleus level:
B12 maybe need as prophylaxis. A. C8-S3
60) Three layers of muscles in which part B. T1-L2
of GIT: C. T1-T4
A. Fundus of Stomach D. C1-C4
B. Part of Esophagus Ans: B
C. Rectum Explanation:
D. Appendix  Clark Nucleus level: T1-L2
Ans: A  Sacral parasympathetic nucleus level: S2-
61) Infection of danger area of face spread S4
to:  Phrenic Nucleus: C3-C5
A. Pons  Accessory Nucleus: Medulla-C5
B. Midbrain 67) Staging of tumor:
C. Caverneous Sinus A. Extent of spread
D. Cortex B. Degree of differentiation
Ans: C C. Both
62) Lesion in which area of the brain D. None
causes resting tremors and Ans: A
expressionless face: 68) Grading of tumor:
A. Basal Ganglia A. Extent of spread
B. Substantia Nigra B. Degree of differentiation
C. Midbrain C. Both
D. Cerebellum D. None
E. Cerebrum Ans: B
Ans: B 69) Patient is dehydrated maximum water
Explanation:The signs are typical for Parkinsons is absorbed at which segment:
disease and that occurs due to lesion of substantia A. PCT
nigra. B. DCT
63) Visual accommodation involves: C. CCT
A. Increase IOP D. LOH
B. Relaxation of sphincter muscle of iris Ans: A
C. Contraction of ciliary muscle Explanation:PCT absorbs 60-65% of water.
SK Origional – Golden 11 93

70) Fastest conducting tissue in human 74) Two groups of patients given
heart: hemorrhoidal treatment and their
A. AV node effect is seen with time, it‟s:
B. SA node A. Prospective Cohort Study
C. Bundle of HIS B. Case Control Study
D. Purkinje fibers C. Randomized Control Study
Ans: D D. None of the Above
Explanation: Ans: C
Follow the following order for speed of Explanation:Random division & treatment given
conduction: to see response is randomized controlled trial.
Purkinje > Atria > Ventricle > SA Node > AV 75) 500 patients exposed to smoking Vs
Node. other subjects are not which type of
71) Child with 60% weight loss, not study is this to see the effect:
responding to vocal stimulus & A. Case Control
bruises on leg: B. Cohort
A. Marasmus C. RCT
B. Physical abuse D. Cross sectional
C. Malnutrition Ans: B
D. Hypothyroidism Explanation:
Ans: B  Cohort: Exposed Vs Non Exposed
Explanation:Loss of weight with no response to  Case Control: Disease Vs Non Diseased
vocal stimulus and bruises on legs points toward 76) Woman after delivery by Dai
Physical Abuse. presented with Increase PT, APTT &
72) A 30 years old female patient Low platelet count is most likely
developed proximal symmetrical suffering from:
muscle weakness, deltoid muscle A. DIC
biopsy shows necrosis with B. VWD
neutrophils, ANA was negative and C. Hemophilia
Anti-Jo antibodies were positive D. Liver Disease
what‟s the patient suffering from: Ans: A
A. SLE Explanation:Delivery by Dai and then lab picture
B. Dermatomyositis that‘s increase PT, APTT and low platelets point
C. RA toward DIC.
D. Sjogren syndrome 77) 12 years old girl suffering from beta
Ans: B hemolytic streptococcal pharyngitis
73) A sprained ankle resulting from treatment:
excessive eversion most likely A. Benzathine Penicillin
demonstrates which structure is B. Benzyl Penicillin
damaged: C. Benzamine
A. Talofibular Ligament D. Penicillin
B. Deltoid Ligament Ans: A
C. Tendo Calcaneus 78) Characteristic of actinomyces:
D. Interosseous Ligament A. Abscess
Ans: B B. Granuloma
Explanation: C. Cancer
 Excessive eversion damages deltoid D. Abscess with Granuloma
ligament Ans: D
 Excessive inversion damages talofibular 79) A patient with HR of 75 has
ligament ventricular systole duration of 0.3
 MC sprain Inversion > Eversion seconds. At HR of 225, the systole
timing will be:
A. 1 sec
SK Origional – Golden 11 94

B. 0.9 S 86) Low testosterone level is present in:


C. 0.05 S A. Preschool
D. 0.1 S B. Old age
Ans: D C. Infant
Explanation:At 75 it was 0.3S now with D. Neonatal
increasing HR to 225 that‘s thrice the systole time Ans: A
will decrease by thrice that‘s 0.1 Seconds. 87) Superior rectal artery is branch of:
80) Drug of choice for A. Superior mesenteric artery
Pseudomembranous Colitis: B. Inferior mesenteric artery
A. Ceftrixone C. Pudendal artery
B. Vancomycin D. Right internal thoracic artery
C. Cefotaxime Ans: B
D. Clindamycin 88) Type of lymphoma associated with
E. Metronidazole EBV is:
Ans: E A. Burkett lymphoma
Explanation:Pseudomonas Colitis is caused by B. Follicular lymphoma
Clindamycin and treated with Metronidazole that‘s C. Mantle cell lymphoma
initial & DOC for it but if severe then go for D. Germinal lymphoma
Vancomycin. Ans: A
81) Nucleus Ambigous Contains: 89) Which of the following relay in
A. CN 8,9,10 substantia gelatinosa?
B. CN 3,4,5 A. Fine touch
C. CN 9,10,11 B. Proprioception
D. CN 12 C. Pain
Ans: C D. Vibration
82) Bony landmark for pudendal nerve Ans: C
block is: 90) Which of the following causes food
A. Ischial tuberosity poisoning by release of exotoxin:
B. Ischial Spine A. Shigella
C. Pubic tubercle B. E coli
D. Acetabulum C. P.Jirovecii
Ans: B D. Staph Aureus
83) Rectum in pelvic region is identified Ans: D
by:
A. Appendices epiplociae
B. Complete muscular coat ANS ===============
C. Posterior to prostate
D. Forms medial wall of ischiorectal fossa
Ans: B
84) Patent lumen of allantois forms:
A. Urachal sinus
B. Urachal cyst
C. Umbilical vein
D. Urachal fistula
Ans: D
85) Which of the following is the feature
of UMN lesion:
A. Babinski negative
B. Hyperreflexia
C. Hyporeflexia
D. Flaccidity
Ans: B
SK Origional – Golden 11 95

MEDICINE & ALLIED Explanation: Severe thunder headache, that‘s


th
September 29 , 2020 – Night worse headache of my life accompanied by
vomiting is most likely SAH.
6. A patient presented with keratitis,
1) Which of the following is most
important for autoimmune disease? xerostomia & joint pain is most likely
suffering from:
a. ANA
a. Rheumatoid Arthritis
b. Anti DNA
b. Sjogren Syndrome
c. Anti RNA
c. SLE
d. TLC
d. Wegner Granulomatosis
Ans: A
Ans: B
Explanation: Anti-Nuclear Antibodies also
Explanation: Sjogren syndrome is characterized
known as anti-nuclear factor are autoantibodies
that bind to content of the cell nucleus & most by Keratoconjunctivitis Sicca, Xerostomia &
Rheumatoid Arthritis.
commonly used in detecting autoimmune
7. Which of the following is an important
conditions.
hormone for parturition?
2) Most important test for diagnosing
a. Fetal Cortisol
lymphoma:
b. Fetal ACTH
a. Immunohistochemistry
c. Fetal Adrenals
b. ELISA
d. Prostaglandins
c. Immunofluorescence
d. Radioimmunoassay Ans: A
Explanation: For parturition initiation prefer
e. Leukocyte count
Fetal Cortisol > Fetal ACTH.
Ans: A
8. A 40 years old alcoholic patient
3) Vessels involved in subdural
presented with consolidation in right
hematoma:
lung apex, he died & autopsy shows
a. Superior cerebral Veins
enlarged liver and hepatocytes filled
b. Middle Cerebral Veins
with lipids, cause of death is:
c. Inferior Cerebral Veins
d. None a. Alpha 1 Antitrypsin Deficiency
b. Chronic Alcoholism
Ans: A
c. Smoking
Explanation: Prefer bridging veins if present in
d. Liver disease
options if not then pick superior cerebral veins.
Ans: B
4. Extradural Hematoma is caused due
Explanation: As the scenario mentions history of
to rupture of which of the following
drinking Alcohol so we pick Chronic Alcoholism
vessel?
as our option or else Alpha 1 anti tryspin
a. Posterior Cerebral artery
b. Anterior cerebral artery deficiency may involve liver and lungs too but
here chronic alcoholism is preferred.
c. Middle Meningeal artery
9. 4 years old child with history of RTA
d. Middle cerebral artery
11 days back, now presented with
Ans: C
fever, epistaxis with Hb of 5.2, raised
Explanation: Extra dural or Epi Dural hematoma
TLC & low Platelets count what
is caused by anterior branch of Middle meningeal
investigation will be performed next:
artery.
a. PT
5. Severe headache with vomiting & CSF
is blood tinged: b. APTT
c. BT
a. Subarchnoid hemorrhage
d. CT
b. Subdural Hemorrhage
Ans: C
c. Epidural Hemorrhage
Explanation: The scenario looks like ITP and as
d. Raised ICP
CBC is already done the next test toward
e. None
confirmation of our diagnosis would be Bleeding
Ans: A
Time (BT).
SK Origional – Golden 11 96

10) Patient is suffering from dysphagia, c. Tidal Volume


dysarthria, analgesia and d. Expiratory Reserve Volume
thermoanesthesia on ipsilateral side of Ans: B
the face and ipsilateral horner‟s Explanation: Inspiratory capacity indicates
syndrome. Occlusion of which of the TV+IRV that‘s what the patient just breathed in?
following artery will produce these 14) Which of the following lung
sign & symptoms: volume/capacity will remain in the
a. Posterior inferior cerebellar artery lungs after maximum full expiration?
b. Superior cerebellar artery a. Residual volume
c. Anterior inferior cerebellar artery b. Functional residual capacity
d. Anterior spinal artery c. Inspiratory reserve volume
e. Posterior spinal artery d. Tidal volume
Ans: A Ans: A
Explanation: This is the case of Lateral Explanation: The volume of air remaining in the
Medullary (Wallenberg) Syndrome that‘s caused by lungs after maximum full expiration is known as
occlusion of PICA (Posterior inferior Cerebellar RV (Residual Volume) that‘s 1200 ml.
Artery) & leads to Vomiting, vertigo, nystagmus, 15) PH = 7.49, PCO = 60, HCO3 =30
Decrease Pain and temperature sensation from which of the following Acid Base
ipsilateral face and contralateral body, dysphagia, abnormality does it indicates?
hoarseness, decrease gag reflex, ipsilateral horner a. Respiratory Acidosis
syndrome, ataxia and dysmetria. b. Partially compensated Respiratory
11) Half-life of carboxyhemoglobin at Acidosis
100% Oxygen: c. Partially compensated metabolic alkalosis
a. 20 Minutes d. Mixed Acidosis
b. 40 – 80 Minutes Ans: C
c. 6 Hours Explanation: As the PH is in alkalosis range now
d. 8 Hours we look toward the HCO3 that‘s in alkalosis range
Ans: B too so we keep metabolic alkalosis in mind, after
Explanation: looking to PCO2 that‘s in Acidotic range shows
 Room Air: 4 – 6 hours the struggle toward compensation but not fully
 100% Oxygen: 40 – 80 Minutes compensated yet hence, this is case of partially
 Hyperbaric Oxygen: 15 – 30 Minutes compensated Metabolic Alkalosis.
12) Half-life of dopamine is 2 minutes, 16) The outer most layer of retina consist
after how much time will it reach of:
steady state concentration: a. Rods & Cones bodies
a. 2 minutes b. Rods & Cones Process
b. 4 minutes c. Pigmented Epithelial Cells
c. 6 minutes d. Choroid
d. 9 minutes Ans: C
e. 2 hour Explanation: The outer most layer of retina
Ans: D consists of Pigmented epithelium followed by
Explanation: A drug reaches after 4-5 half-lives photoreceptors layers consisting of Rods &
to its steady state concentration so here half-life is Cones.
2 minutes for dopamine and will reach to steady 17) Which chlamydial species cause
state in 4-5 half-lives that‘s 8 – 10 minutes so we blindness?
pick 9 minutes here. a. Chlamydia Pneumonia
13) Patient expired normally & then b. Chlamydia Psittaci
inspired maximally which of the c. Chlamydia Trachomatis D – K
following lung volume or capacity d. Chlamydia Trachomatis A – C
does it points toward: e. Chlamydia Trachomatis L1 – L3
a. Inspiratory Volume Ans: D
b. Inspiratory Capacity Explanation:
SK Origional – Golden 11 97

 Chlamydia Trachomatis A – C : Blindness  Sub-acute endocarditis is caused by Strep


 Chlamydia Trachomatis D – K : STDs Viridans.
 Chlamydia Trachomatis L1 – L3:  If prosthetic valve mentioned then prefer
Lymphogranuloma Venerum Staph epidermidis.
18) Brain eating parasite: 23) A patient was diagnosed with PSGN
a. Naegleria Fowleri which type of HSR does this involves?
b. Chlamydia a. Type 1 HSR
c. Nocardia b. Type 2 HSR
d. Toxoplasmosis c. Type 3 HSR
Ans: A d. Type 4 HSR
Explanation: Naegleria fowleri also known as Ans: C
brain eating amoba, is a species of the genus 24) PPD Tb Skin test shows which of the
Naegleria belonging to phylum percoloza which is following type of HSR?
technically not classified as true amoeba? a. Type 1 HSR
19) A patient presented with diarrhea, b. Type 2 HSR
donut shaped cysts are seen in: c. Type 3 HSR
a. Cryptosporidium d. Type 4 HSR
b. Cystocercosis Ans: D
c. Ameoba Explanation: Skin PPD test that‘s used for
d. Staph Aureus diagnosis of Tb is basically Type 4 HSR & caused
Ans: A by helper T cells & macrophages.
20) Yellow vaginal discharge with motile 25. Which of the following is known as
trophozite: anti-tumour cells?
a. Trichomonas Vaginalis a. NK Cells
b. Gonorrhea b. Antibodies
c. Candida c. Plasma Cells
d. T.Pallidum d. TNF
Ans: A Ans: A
21) After secretion of trypsinogen into 26) Corona virus was recently named as:
duodenum, the enzyme is converted a. Navel Corona
into its active form, trypsin by: b. SARS-CoV-2
a. Procarboxypeptidase c. SARS-CoV-1
b. Pancreatic lipase d. SARS -3
c. Previously secreted trypsin Ans: B
d. Enteropeptidase Explanation: The international committee on
e. Alkaline PH taxonomy of viruses (ICTV) announced severe
Ans: D acute respiratory coronavirus 2 (SARS-CoV-2) as
Explanation: Enterokinase, that‘s the most the name of the new virus on February, 2020.
suitable answer but that wasn‘t there in options so 27) Which hormone is responsible for
the next best to choose among these are delayed gastric emptying?
enteropeptidase. a. CCK
22) Subacute endocarditis is caused by b. Gastrin
which of the following organism? c. Secretin
a. Staph Aureus d. Somatostatin
b. Step Viridans Ans: A
c. Staph Epidermidis Explanation: CCK increases intestinal motility &
d. E. coli decreases gastric emptying.
Ans: B 28) Saw tooth appearance on ECG is seen
Explanation: in:
 Acute endocarditis is caused by Staph a. Atrial Fibrillation
Aureus. b. Atrial Flutter
c. Heart Blocks
SK Origional – Golden 11 98

d. Paroxysmal Nodal tachycardia Explanation: On right side IVC is located &


Ans: B that‘s why consultant didn‘t let the resident to
29) Old age women with history of IHD operate on right side due to risk of damage to
presented with irregular heart beat & IVC.
abnormal QRS complexes which of 34) Which one of the following causes
the following drug to be administered: bladder carcinoma?
a. Digoxin a. Cyclophosphamide
b. Lignocaine b. Aniline dye
c. Amiodarone c. Smoking
d. Beta blocker d. Alcohol
Ans: B Ans: C
Explanation: VTs secondary to IHD then always Explanation:
go for Lignocaine, if no history of IHD then  Bladder carcinoma risk factors in order of
DOC would be Amiodarone. frequency.
30) Drug of choice for traveler‟s diarrhea:  Squamous Cell carcinoma:
a. Diphenoxylate Schistosmoiasis > Calmatte Guerin
b. Norfloxacin treatment > Smoking.
c. Ciprofloxacin  Transitional Carcinoma: Smoking >
d. Metronidazole Exposure to Aniline Dye in printing
Ans: A industry > Rubber Manifacture >
Explanation: The drug of choice for traveler‘s Cyclophosphamide.
diarrhea is Diphenoxylate but when resistant to 35) Angiosarcoma of liver is caused by:
anti motility drug then go for Norfloxacin. a. Vinyl Chloride
31) Traveler‟s diarrhea not responding to b. Arsenic
anti motility drugs which of the c. Aflatoxin
following to be administered? d. Aniline Dye
a. Diphenoxylate Ans: A
b. Norfloxacin Explanation:
c. Ciprofloxacin  If in history farmer is given then go for:
d. Metronidazole Arsenic.
Ans: B  If in history factory worker is given then
Explanation: Here it‘s resistant case of Traveler‘s go for: Vinyl chloride.
diarrhea so we go for Norfloxacin.  In a vague stem like this prefer: Vinyl
32) Which of the following vessel is chloride > arsenic.
anterior to IVC? 36) Obstructive lung disease is diagnosed
a. Left Renal Vein in patient which of the following is the
b. Right Renal Vein causative agent:
c. Right Testicular Artery a. Nicotine
d. Left Testicular artery b. Arsenic
Ans: C c. Nickle
Explanation: d. Cadmium
 Anterior to IVC is right testicular artery. Ans: A
 Anterior to aorta is left renal vein. 37) Which of the following doesn‟t occur
33) A resident successfully performed left in edema?
side adrenalectomy, but right side a. Decrease Oncotic pressure
wasn‟t allowed by consultant due to b. Increase Capillary Oncotic pressure
risk of rupture of: c. Increase Capillary Hydrostatic Pressure
a. IVC d. Increase Permeability
b. Aorta Ans: B
c. Right Gland Explanation: Edema occurs due to decrease
d. Kidney capillary oncotic pressure, increase capillary
Ans: A hydrostatic pressure & increase capillary
SK Origional – Golden 11 99

permeability so here increase capillary oncotic we have to go for APTT and if confirmatory test
pressure doesn‘t occur in edema. is asked toward the diagnosis then we will pick
38) A patient with 3rd degree extensive Factor 8 Assay.
burn, cause of edema is: 43) Thyroid hormone receptors are
a. Acidosis present:
b. Alkalosis a. On cell membrane
c. Hypoalbuminemia b. Inside the Nucleus
d. Hypokalemia c. Outside the cell
Ans: C d. In cytoplasm
39) Stratum corneum, spinosum & Ans: B
lucidum acute loss will lead to: 44) In dilated cardiomyopathy which
a. Acute water loss functions is disturbed:
b. Pressure sensation effect a. Systolic Dysfunction
c. Sebum b. Diastolic dysfunction
d. Hair follicle c. Both
Ans: A d. None
40) A patient presented with weight loss, Ans: A
productive cough & night sweats what Explanation:
will be seen on histology?  In dilated cardiomyopathy there‘s systolic
a. Non caseous necrosis dysfunction.
b. Caseous necrosis  In hypertrophic & constrictive
c. Granuloma with Asteroid cardiomyopathy there‘s diastolic
d. Non Asteroid dysfunction.
Ans: B 45) Hormone responsible for
Explanation: The scenario is related to Tb and glycogenolysis, Gluconeogensis &
on histology there will be caseous necrosis. ketogenesis:
41) A child presented with episodes of a. Cortisol
epistaxis after vaccination which of b. Growth hormone
the following investigation is required c. Glucagon
to be performed initially: d. Insulin
a. BT Ans: C
b. CT 46) A 30 years old female presented to
c. Platelet Count OPD with excess hair growth,
d. PT hypertension, hyperglycemia,
Ans: C hyperpigmentation, increased
Explanation: The scenario looks like ITP & depression, edematous face, increased
that‘s why Platelet count is foremost important to fat on chest & trunk, it‟s likely due to
reach the diagnosis. excess of:
42) 2 years old boy brought for a. Cortisol
circumcision there‟s history of b. ACTH
excessive bleeding from umbilical c. GnRH
stump at time of birth, 2 cousins bled d. LH
profusely after circumcision in the Ans: B
past. The test likely to give clue to Explanation: This is scenario of Cushing disease
diagnosis is: as there‘s pigmentation history and that‘s due to
a. APTT increase ACTH level.
b. PT 47) A patient presented with hypertension
c. BT & raised Sodium, low Potassium
d. Factor 8 &raised PH what could the possible
Ans: A underlying reason:
Explanation: The case looks like Hemophilia, but a. Hyper aldosteronism
as we have three types of Hemophilia so initially b. Pheochromocytoma
SK Origional – Golden 11 100

c. Increase Cortisol b. Streptomycin


d. Increases Glucagon c. Isoniazid
Ans: A d. Ethambutol
Explanation: This is most likely due to Increase Ans: C
Aldosterone level that will lead to increase Sodium Explanation: INH (Isoniazid) needs to be
level, decrease Potassium level & will secrete H+ stopped as it leads to vitamin B6 deficiency &
ions leading toward Metabolic Alkalosis with leading to pin & needle sensations and must be
increase PH. administered with Pyridoxine (B6).
48) Diagnostic test for Leprosy: 53) Which of the following is the drug of
a. AFB choice in a 5 months old pregnant
b. Montoux Test lady of thyrotoxicosis?
c. Nasal Scrapping a. PTU
d. None b. Methimazole
Ans: C c. Thyroxin
49) Calcium & phosphate balance is done d. T3
by: Ans: B
a. PTH Explanation:
b. Vitamin D  PTU is used in the first trimester.
c. Cortisol  Methimazole is used in 2nd& 3rd
d. Insulin Trimesters.
Ans: A 54) Dorsomedial nucleus of thalamus
50) Majority of seminal fluid: related with:
a. Seminal Vesicle a. Sweet taste
b. Prostate b. Loud Noise
c. Adrenals c. Itching
d. Ejaculatory duct d. Memory
Ans: A Ans: D
Explanation: Maximum amount of seminal fluid Explanation:
is formed by seminal vesicles round about 70%.  Insula is associated with Tactile sensation.
51) Heat loss in humid environment is  Insula is present in Temporal lobe.
through which of the following?  Sense of disgust is by left inusla.
a. Evaporation  Olfactory cortex associated with Uncus.
b. Convection  Memory related with Dorsomedial
c. Conduction nucleus of thalamus.
d. Radiation 55) Ventricles are completely depolarized
Ans: B at which part?
Explanation: a. P wave
 If patient is naked heat loss is through: b. ST Segment
Radiation. c. TP Segment
 If patient is naked & lying (Contact with d. QRS
table) then: Conduction. Ans: B
 If patient is naked & temperature 56) Which of the following is increased in
mentioned: Radiation. athletes at rest?
 If humidity mentioned: Convection a. HR
(Convection requires medium & here b. SV
Humidity is medium) c. TPR
 If trachestomy ETT: Evaporation d. VR
52) A patient on Anti Tb drugs presented Ans: B
with pins and needle sensations. Explanation: Athletes are having increase Stroke
Which of the following drug must be Volume & Decrease HR during rest.
stopped? 57) A/A ratio is more than:
a. Pyrazinamide a. 25
SK Origional – Golden 11 101

b. 45 a. Pernicious Anemia
c. 65 b. Sideroblastic Anemia
d. 75 c. IDA
Ans: D d. Folate Deficiency
58) Index of total peripheral resistance: Ans: A
a. Mean arterial pressure 64) A 45 years old man with long history
b. Diastolic pressure of alcoholism presented with severe
c. Systolic pressure epigastric pain, nausea, vomiting,
d. Venous Return fever & increase in serum amylase
Ans: B diagnosis of acute pancreatitis
59) A 16 years old female presented with superimposed on chronic pancreatitis
hypertension & increasing weight. was made in this condition which of
Which of the following features would the following type of necrosis is most
be most suggestive of cushing‟s likely?
syndrome rather than simple obesity: a. Coagulative necrosis
a. Abdominal Striae b. Enzymatic necrosis
b. Acanthosis Nigricans c. Fibrinoid
c. Buffalo Hump d. Caseous
d. Moon Face e. Liquefective necrosis
Ans: A Ans: B
60) Nutmug liver occurs in: 65) Which of the following type of
a. Periporatal Congestion necrosis occurs in mesentery?
b. Centrilobular Congestion a. Fat Necrosis
c. Chronic Passive Congestion b. Coagulative necrosis
d. Alcoholic Cirrhosis c. Liquefective necrosis
Ans: C d. Fibrinoid
61) Industry worker cause of Ans: A
Angiosarcoma: 66) Enzymatic necrosis affecting
a. Arsenic pancreases:
b. Vinyl chloride a. Liquefective
c. Cadmium b. Coagulative
d. Asbestosis c. Fat
Ans: B d. Traumatic
Explanation: Ans: C
 Factory Worker: Vinyl Chloride. Explanation: Enzymatic fat necrosis occurs in
 Farmer: Arsenic. acute pancreatitis & omental necrosis.
62) Patient known HIV suffering from 67) Preferential thorough fare channel:
severe headache, photophobia & sign a. Small caliber
of meningeal irritation. On b. Having smooth muscles
microscopy organism has halo around c. Open on demand
it. The organism is: d. Precapillary sphincter
a. Toxoplasmosis Ans: B
b. Histoplasmosis Explanation: Preferential thorough fare channels
c. Cryptococcus caliber is larger than true capillaries, resembles
d. Candida continuous capillaries but few smooth muscle cells
e. CMV are present around endothelium.
Ans: C 68) CD means:
Explanation: a. Cluster of Differentiation
 Brain Abscess: Toxoplasma Gondii b. Classes of Differentiation
 Halo: Cryptococcus (Silver Stain) c. Conduct of division
63) Which of the following Anemia occurs d. None of the above
due to Intrinsic factor deficiency? Ans: A
SK Origional – Golden 11 102

69) Aldosterone acts on: b. Corneal Reflex


a. PCT c. Accomodation
b. DCT d. Cardiac Reflex
c. CCT Ans: B
d. LOH 76) Which of the following is correct
Ans: C relation of phrenic nerve?
Explanation: Aldosterone acts on cortical A. Left phrenic nerve descends anterior
collecting tubule (CCT) > DCT > CD. to the left lung root
70) Lady after hemorrhage goes into B. Right phrenic nerve descends
Acute Kidney injury which of the posterior to the left lung root
following part of nephron will be most C. Phrenic nerve passes posterior to
likely affected: lung
a. PCT D. None of the above
b. DCT Ans: A
c. LOH 77) Which of the following vitamin is
d. CT necessary with protein intake?
Ans: A a. Vitamin B12
71) Boy after hemorrhage, lost 1.5L of b. Vitamin B1
blood will present with: c. B2 (Riboflavin)
a. Normocytic Normochromic Anemia d. Folate
b. Megaloblastic Anemia Ans: C
c. Pernicous Anemiaa Explanation:
d. Microcytic Hypochromic Anemia  Thiamine is necessary for carbohydrate
Ans: A intake.
72) Excessive Glucocorticoids causes  Biotin is necessary for lipids intake.
which of the following?  B2 (Riboflavin) is necessary for protein
a. Neutropenia intake.
b. Basopenia 78) Protein can‟t cross the glomerular
c. Lymphocytopenia basement membrane due to:
d. Thrombocytopenia a. Positive charge
Ans: C b. Negative charge
Explanation: Excessive glucocorticoids leads to c. Neutral
lymphoctyopenia & neutrophili. d. None
73) Selective CoX – 2 inhibitor: Ans: B
a. Meloxicam 79) Von Gierke‟s disease is due to
b. Ibuprofen deficiency of:
c. Indomethacin a. Glucose-6 Phosphatase
d. Aspirin c. Acid maltase
Ans: A c. Muscle Phosphorylase
Explanation: d. Liver Phosphorylase
 Selective CoX-2 inhibitor: Celecoxib > Ans: A
Moloxicam Explanation:
 Potent CoX-2 Inhibitor: Moloxicam  Von Gierke‘s Disease is due to deficiency
74) Diagnostic criteria of aplastic anemia: of Glucose-6 Phosphatase.
a. Fatty bone marrow  Pompe‘s Disease is due to deficiency of
b. Pancytopenia Acid maltase.
c. Bicytopenia  McArdle‘s disease is due to deficiency of
d. Thrombocytopenia Muscle phosphorylase.
Ans: A  Her‘s disease is due to deficiency of Liver
75) Which of the following is trigeminal Phosphorylase.
reflex? 80) Largest Molecular weight protein:
a. Pupillary Reflex a. Albumin
SK Origional – Golden 11 103

b. Globulin Ans: A
c. Titin Explanation:
d. Myosin  In anterior Cruciate ligament injury Tibia
e. Actin will move anterior in relation with the
Ans: C femur & Femur will move posterior.
Explanation: Titin, is definitely the largest  In Posterior Cruciate ligament injury
protein in the body, with a molecular weight of 3 Tibia will move posterior in relation with
million Dalton and composed of 27000 amino the femur & femur will move anterior.
acids. 85) If cimetidine and sucralfate are given
81) Right gastric artery is branch of: together, the effect will be:
a. Gastroduodenal artery a. Decrease Absorption
b. Proper Hepatic artery b. Increase Absorption
c. Left Gastric Artery c. Inhibit function
d. Celiac artery d. Delayed function
Ans: B Ans: A
Explanation: 86) Kaplan-Meier curve indicates:
 Right gastric artery in most cases arises a. Incidence of disease
from proper hepatic artery. b. Demography
 Left gastric artery arises from celiac trunk. c. Severity of disease
82) Skin of Glans penis lymphatic d. Survival of patient
drainage: Ans: D
a. Deep inguinal nodes 87) Regarding Cell cycle G2 Phase is
b. Superficial inguinal nodes between:
c. Internal iliac node a. G1 – S
d. Para Aortic lymph node b. S–M
Ans: B c. G1 – G2
Explanation: d. S –G2
 Skin of glans penis is drained by Ans: B
superficial inguinal lymph nodes. Explanation:
 Glans penis is drained by deep inguinal  G2 phase lies between S – M Phase
lymph nodes.  G1 Phase lies between M – S phase
83) Arterial blood supply of Upper 1/3rd of 88) Sarcomere lies between:
esophagus is: a. Two Z-Lines
a. Inferior thyroid artery b. Two A-Lines
b. Descending aorta c. Two I-Lines
c. Left gastric artery d. Two Y-Lines
d. Right gastric artery Ans: A
Ans: A 89) Continuous capillaries are present in:
Explanation: a. Muscle
 Upper 1/3rd of esophagus is supplied by: b. Kidney
Inferior thyroid artery. c. Brain
 Middle 1/3rd of esophagus is supplied by: d. Liver
Descending aorta. Ans: C
 Lower 1/3rd of esophagus is supplied by: 90) Prosopagnosia means:
Left gastric artery. a. Inability to read
84) After anterior cruciate ligament injury b. Inability to walk
tibia will moves in which direction in c. Inability to recognize faces
relation with the femur: d. Inability to move
a. Anterior Ans: C
b. Anterolateral 91) Virus act on the cells by following
c. Anteromedial mechanism:
d. Posterior a. Damage to cell membrane
SK Origional – Golden 11 104

b. Damage to nuclear membrane thrice so the cardiac cycle will decrease by thrice
c. Involving protein synthesis that‘s 0.1 Seconds.
d. Changing protein structure 97) Fetal component of placenta if formed
Ans: C by:
92) Unlocking of knee joint muscle: a. Chorionic Frundosum
a. Sarotrius b. Functionalis Layer
b. Popliteus c. Basalis Layer
c. Gluteus d. Endometrium
d. Adductors Ans: A
Ans: B 98) Hemiballismus due to lesion of:
93) 55 years old with testicular swelling a. Substantia Nigra
only LDH raised without any marker b. Subthalamic Nuclei
which of the following does he suffer c. Globus Pallidus
from: d. Hypothalamus
a. Seminoma Ans: B
b. Lymphoma Explanation:
c. Teratoma  Hemiballismus is due to lesion of
d. Yolk Sac tumor subthalamic nuclei.
Ans: B  Parkinson‘s is due to lesion of Substantia
Explanation: In young age Seminoma is Nigra.
commonest testicular tumor while in old age 99) In dialysis patient how to check GFR:
lymphoma. a. Creatinine Clearance
94) Medial part of frontal & parietal lobe b. Inulin Clearance
blood supply: c. Urea
a. MCA d. BUN
b. ACA Ans: A
c. PCA Explanation:
d. Spinal Artery  GFR is best measured by Inulin.
Ans: B  GFR is best estimated by creatinine.
95) Damage to glossopharyngeal nerve  GFR is checked clinically by Creatinine
leads to loss of which of the following Clearance.
taste sensation? 100) Right Optic tract lesion effect:
a. Sweet a. Left Homonymous Hemianopia
b. Salt b. Right Homonymous Hemianopia
c. Bitter c. Bitemporal vision loss
d. Sour d. Binasal vision loss
Ans: C Ans: A
Explanation: Glossopharyngeal nerve takes taste Explanation:
& general sensation from posterior 1/3rd of the  Right optic tract lesion leads to Left
tongue that‘s related with transmission of bitter homonymous hemianopia.
taste sensation.  Left optic tract lesion leads to right
96) A patient whose heart is 75 beats/min homonymous hemianopia.
with cardiac cycle of 0.3 seconds, 101) Which of the following drug blocks H-
calculate his cardiac cycle if his heart K pump?
rate increase to 225: a. Histamine
a. 0.1 S b. Omperazole
b. 0.2 S c. Cimetidine
c. 3 Seconds d. Protamine
d. 4 Minutes Ans: B
Ans: A 102) Which of the following structure lies
Explanation: As with heart rate of 75, cardiac deep to carotid sheath?
cycle was 0.3 Seconds & now heart rate increased a. Vagus
SK Origional – Golden 11 105

b. Internal Jugular a. Decrease discharge rate of carotid sinus


c. Trachea b. Increase parasympathetic activity
d. Sympathetic Trunk c. Increase hering nerve activity
Ans: D d. Decrease hering reflex
103) Massetric fascia develops from: Ans: A
a. Pretracheal Explanation: If blood pressure is decreased that‘s
b. Prevertebral in hypotension & hypovolemia then the discharge
c. Carotid of carotid sinus will decrease in order to increase
d. Superficial layer of Deep cervical fascia sympathetic & decrease parasympathetic response
Ans: D to increase the blood pressure.
104) Atlanto-Axial Joint movement: 109) On initial patient interview least likely
a. Rotation thing to be done is:
b. Yes a. Open ended questions
c. Flexion b. Project an image of authority
d. Extension c. Politeness
Ans: A d. Well behavior
Explanation: Ans: B
 Rotational movement is produced at 110) Renin release is increased by:
Atlanto-Axial joint that‘s no movement. a. Increase sympathetic activity by renal
 Flexion movement that‘s yes movement is nerve
produced at atlanto-occipital joint. b. Hypervolemia
105) What should be the pharmaceutical & c. Hypo-osmolarity
doctor interaction purpose? d. Increase parasympathetic activity
a. Hospital welfare Ans: A
b. Staff welfare 111) Related to lateral corticospinal tract:
c. Should be entertained for patient‘s benefit a. Rubro
d. Earning purpose b. Reticulo
Ans: C c. Vestibular
Explanation: Doctor & pharmaceutical d. Ventral
companies‘ interaction should always be for the Ans: A
benefit & well-being of the patients followed by 112) Which of the following structure does
Hospital welfare. aorta compresses at Aortic hiatus?
106) Iodine is excreted through: a. Accessory vagus
a. Urine b. Esophagus
b. Feces c. IVC
c. Saliva d. Azygous & thoracic duct
d. Sweat Ans: D
Ans: A Explanation: Aorta Passes at T12 level along
107) Small muscles of hand & medial with Azygous & thoracic duct, and these can be
palmar surface of hand affected & compressed in case of aortic dilatation &
Lateral part of thenar is spared which aneurysm. ATA= Aorta, Thoracic Duct,
of the following nerve is damaged: Azygous Vein.
a. Median Nerve 113) Congenital diaphragmatic hernia
b. Radian nerve occurs due to:
c. Ulnar Nerve a. Absence of pleuorperitoneal membrane
d. C8-T1 b. Incomplete pleuroperitoneal membrane
Ans: C c. Defective pleuroperitoneal membrane
Explanation: As Medial side of the hand is d. Defective pleuropericardial membrane
affected & lateral side isn‘t effected the most likely Ans: B
nerve damaged is Ulnar nerve. 114) Blood CSF is found in:
108) During decrease in blood pressure, a. Epidural space
which compensatory response occur? b. Subarachnoid space
SK Origional – Golden 11 106

c. Extradural Space 120) Tragus of external ear developed from:


d. None of the above a. 1st arch
Ans: B b. 1st cleft
115) Nucleus pulposus derived from: c. 2nd arch
a. Dermatome d. 2nd cleft
b. Sclerotome Ans: B
c. Notochord 121) Loss of pain & temperature sensation
d. Dytome but intact touch sensation in which of
Ans: C the following:
Explanation: The only adult anatomy derived a. Tabes Dorsalis
from the notochord is the nucleus pulposus of the b. DM
intervertebral disc. c. Syringomyelia
116) Grading of carcinoma shows: d. SCD of Spinal cord
a. Type of Epithelium Ans: C
b. Type of Muscle 122) Middle diastolic murmur, pulmonary
c. Type of Cell HTN and pulmonary edema, which of
d. Type of Nuclei the following the patient will most
Ans: C likely develop:
Explanation: Grading shows type of Cells or a. Right ventricular Hypertrophy + LVH
differentiation. b. Right Ventricular hypertrophy
117) Which of the following organism is c. Right atrial hypertrophy
responsible for rheumatic fever? d. None of the above
a. Group A Streptococcus Ans: B
b. Group B Streptococcus Explanation: This is the case of Mitral stensosis
c. Group C Streptococcus & due to back pressure the patient is at risk of
d. Statph Aureus developing RVH (Right ventricular Hypertrophy).
Ans: A 123) Which of the following is most
Explanation: Group A streptococcus commonly associated with smoking?
(Streptococcus Pyogenes) is responsible for a. PAN
Rhumatic Fever. b. Thromboangitis Obliterns
118) Cat is denervated at mid collicular c. Kawasaki Disease
level, what will happen? d. Giant cell arteritis
a. Decerebrate posture Ans: B
b. Decorticate posture 124) Superficial thrombophlebitis involves
c. Increase flexor tone of muscle spindle which of the following?
d. None of the above a. Saphenous Vein
Ans: A b. Deep veins
Explanation: c. Asymptomatic
 Decerebrate rigidity at midcollicular level. d. Perforators
 Decorticate rigidity if lesion is just below Ans: A
superior colliculus. 125) Most specific findings in acute MI:
 Decerebrate rigidity: Increase extensors a. ST segment Elevation
tone of all four limbs. b. Inversion of Lead I
 Decorticate Rigidity: Increase flexor tone c. Q wave measuring 50% of QRS height
In upper limbs. d. Delta waves
119) Primary cartilaginous Joint: Ans: A
a. Costochondral Joint 126) Right bundle branch is present in:
b. Teeth & Jaw a. Sulcus terminalis
c. Inferior tibiofibular joint b. Crista terminalis
d. Symphysis pubis c. Septomerginal
e. Sutures of skull d. Apex
Ans: A Ans: C
SK Origional – Golden 11 107

127) Which of the following is important d. Both on ciliary


function of progesterone? Ans: B
a. Lactation inhibition 133) Desmoplasia means:
b. Fetal well being a. Growth of fibrous tissue
c. Endometrial thickness maintenance b. Growth of muscle
d. Ripening of cervix c. Growth of epithelium
Ans: C d. None of the above
Explanation: Prefer Endometrial thickness Ans: A
maintenance > Secretory phase of Endometrium. Explanation: Desmoplasia is the growth of
128) First pass metabolism of drug can be fibrous or connective tissue.
best avoided via administration at 134) Naloxone given to a patient as
which route? antidote & has no effect, poisoning is
a. IV of:
b. Sublingual a. Opoids
c. IM b. Phenobarbitone
d. P/O c. Morphine
Ans: A d. None
Explanation: IV > S/L > IM > P/O Ans: B
129) Vasculitis that involves same vessel or Explanation: Naloxone act as antagonist for
different vessels with lesion of Opoids & phenobarbitone is barbiturates that‘s
different ages: why Naloxone won‘t reverse the symptoms.
a. Kawsaki disease 135) Leiomyoma is:
b. PAN a. Benign Epithelial tumour
c. Giant cell arteritis b. Benign mesenchymal tumour
d. Temporal Cell arteritis c. Malignant tumour
Ans: B d. None of the above
130) Emphysema due to smoking: Ans: B
a. Panaciniar 136) Lipids after metabolism absorbed in
b. Centriacinar the form of:
c. Both a. LDL
d. None of the above b. VLDL
Ans: B c. Chylomicrons
Explanation: d. HDL
 Centriacinar begins in the respiratory Ans: C
bronchioles and spreads peripherally 137) Which of the following vitamin is
mainly in the upper half of the lungs and deficient post pacreatecomtomy?
is usually associated with long standing a. Thiamine
cigarette smoking. b. Niacin
 Panacinar associated with Alpha-1 c. Vitamin D
antitrypsin deficiency. d. Vitamin A
131) Norepinephrine will cause Ans: C
constriction of: 138) Which of the following carcinoma is
a. Arterioles related to smoking?
b. Bronchioles a. SCC
c. Ciliary body b. BCC
d. Intestine c. Melanoma
Ans: A d. None of the above
132) When looking to near object what will Ans: A
increase? 139) MOA of benzodiazepine:
a. Size of pupil a. Inhibition of GABA
b. Parasympathetic on ciliary b. Potentiation of GABA
c. Sympathetic on ciliary c. Inhibition of Na channels
SK Origional – Golden 11 108

d. Stimulation of Na channels  Right boarder of heart is formed by: Right


Ans: B Atrium
140) Cause of death in Extensive PE is:  Left boarder of heart is formed by: Left
a. Thromboembolism Ventricle
b. Air  Inferior boarder of heart is formed by:
c. Fat right Ventricle
d. Gas  Sternocostal (anterior) surface of heart is
Ans: A formed by: right ventricle
141) Pregnant lady on hilly area which of  Diaphragmatic (inferior ) surface of heart
the following cardiac defect can be is formed by: left ventricle
seen in the baby:  Base (posterior) surface of heart is
a. ASD formed by: Left Atrium
b. VSD 146) Patient is dehydrated, maximum
c. PDA water is re absorbed from:
d. TGA a. PCT
Ans: C b. DCT
Explanation: As ductus arteriosus needs high c. LOH
tension oxygen post birth, to close and there‘s low d. CT
partial pressure of Oxygen in hilly areas thus that Ans: A
will lead to Patent ductus arteriosus (PDA). Explanation: Maximum water is re absorbed
142) Increasing Stroke volume by keeping from PCT that‘s 60 – 65%.
HR constant will lead to: 147) What is the purpose of giving
a. Increase in both Pulse pressure & MAP combined ATT?
b. Decrease in both pulse pressure & MAP a. To prevent development of resistance
c. Increase only in MAP b. To enhance each other action
d. Increase only in pulse pressure c. To prevent side effect of individual drug
Ans: A d. None of the above
143) Which segment of the lung is first Ans: A
seen on bronchoscopy? 148) The most characteristic feature of
a. Right middle lobe sarcoidosis:
b. Right upper lobe a. Hypercalcemia
c. Right lower lobe b. Hepatic granuloma
d. Left upper lobe c. Erythema Nodosum
Ans: B d. Pleural effusion
144) Which of the following blood group is Ans: C
considered universal donor: 149) Farmer‟s lung is caused by:
a. A+Ve a. Grain dust
b. B+Ve b. Silica
c. AB+Ve c. Asbestosis
d. O-Ve d. Iron
Ans: D Ans: A
Explanation: 150) Which of the following is the action of
 O-ve is universal donor Aldosterone:
 AB+ve is universal acceptor a. Sodium reabsorption
145) Sternocostal surface of heart is formed b. Potassium reabsorption
by: c. Chloride secretion
a. Right ventricle d. Chloride reabsorption
b. Left ventricle Ans: A
c. Right atrium Explanation: Aldosterone leads to Sodium
d. Left atrium reabsorption & potassium secretion.
Ans: A 151) Severe diarrhea leads to which acid
Explanation: base imbalance:
SK Origional – Golden 11 109

a. Metabolic alkalosis  CN 9,10,11 & 12: Medulla


b. Metabolic acidosis 156) Cranial neuropore closure defect with
c. Mixed acidosis absent brain calvarium:
d. None of the above a. Hydrocephalus
Ans: B b. Anencephaly
Explanation: c. Microcephaly
 Acute or severe diarrhea leads to d. Myeloschiasis
Metabolic Acidosis. Ans: B
 Chronic diarrhea leads to Metabolic 157) Patient came from abroad with
Alkalosis. lymphadenopathy, infection and fever:
152) Prevalence is: a. HIV seroconversion
a. New cases b. CMV
b. All new cases Plus Old c. EBV
c. Old cases d. HPV
d. None of the above Ans: A
Ans: B 158) Immediate reversal of Warfarin
Explanation: toxicity:
 Incidence indicates new cases over a a. FFPs
period of time b. Vitmain K
 Prevalence shows both new & old cases c. Whole blood
over a period of time d. Platelets
153) After pregnancy and parturition uterus Ans: A
size is reduced by: Explanation: Immediate reversal of Warfarin
a. Lysosome best choice would be both FFP & vitamin K but
b. Peroxisome when one to choose it‘s FFP > Vitamin K.
c. Golgi apparatus 159) A very young age patient presented
d. Ribosome with fever & his WBC were increased
Ans: A 50 folds is most likely suffering from:
154) Virus associated with nasopharyngeal a. ALL
carcinoma: b. AML
a. EBV c. CML
b. HPV d. CLL
c. CMV Ans: B
d. H. Pylori Explanation: AML is commonly seen in young
Ans: A age patient.
Explanation: 160) Which of the following has shortest
 Oropharyngeal carcinoma is associated pre erythrocytic phase?
with HPV. a. P. Malaria
 Nasopharyngeal carcinoma is associated b. P. Ovale
with EBV. c. P. Vivax
 Bronchogenic carcinoma is associated d. P. Falciparum
with CMV. Ans: D
155) 3rd cranial nerve origin: Explanation:
a. Medulla  P.Malariae has longest pre erythrocytic
b. Pons phase
c. Midbrain  P.Falciparum has shortest pre erythrocytic
d. Cortex phase
Ans: C 161) MELAS syndrome histological
Explanation: findings are:
 CN 1 & 2 : Cortex a. Loflar bodies
 CN 3 & 4 : Midbrain b. Inclusion bodies
 CN 5,6,7 & 8: Pons c. Granuloma
SK Origional – Golden 11 110

d. Red ragged fibers c. 41% of blood is RBC+WBC+Platelets


e. Cyst d. None of the above
Ans: D Ans: B
Explanation: MELAS (Mitochondrial 168) Dead space isn‟t affected by:
Encephalopathy Lactic acidosis & Stroke like a. Shallow breathing
episodes) syndrome is a rare disorder that begins b. Deep breathing
in the childhood, usually between two and fifteen c. Tracheostomy
years of age and mostly affect the nervous system. d. Atelectasis
162) Post-transplant patient prone to Ans: A
infection: Explanation: Breathing pattern doesn‘t change
a. CMV dead space but if we have to choose it‘s Shallow
b. HBV Breathing > Deep Breathing.
c. HIV 169) Motor aphasia due to:
d. HAV a. Brocas area
Ans: A b. Wernickes area
Explanation: c. Angular gyrus
 Post-transplant: CMV d. None of the above
 Post transfusion commonest: HBV Ans: A
 Post transfusion lethal: CMV Explanation:
163) Which of the following prevent muscle  Motor aphasia: Brocas
rupture?  Sensor aphasia: Wernickes
a. GTO  Anomic aphasia: Angular gyrus
b. Nuclear bag 170) Ureter epithelium is:
c. Nuclear chain a. Transitional epithelium
d. Spindle b. Squamous epithelium
Ans: A c. Simple Cuboidal Epithelium
164) Man presented with gum bleeding is d. Stratified squamous
due to deficiency of which of the Ans: A
following?
a. Vitamin A
b. Vitamin B ==============
c. Vitamin C
d. Vitamin E
Ans: C
165) Crescents are formed by:
a. Parietal cells
b. Mesengium
c. Epithelial cells
d. Nephron
Ans: A
Explanation: Crescents are formed by
macrophages & parietal cells.
166) Which of the following indicates
ventricular depolarization on ECG?
a. P wave
b. QRS
c. ST Segment
d. TP segment
Ans: B
167) 41% hematocrit:
a. 41% of blood is wbc
b. 41% of blood is RBC
SK Origional – Golden 11 111

MEDICINE & ALLIED C) Mitral Regurgitation


th
September 30 , 2020 – Afternoon D) Mitral Stenosis
Ans: A
Explanation: Aortic regurgitation is associated
1) Clara cells are present in:
A) Respiratory Bronchioles with increased EDV, early diastolic murmur & left
ventricular hypertrophy due to back flow of the
B) Terminal Bronchioles
blood into left ventricle.
C) Alveoli
5) Voluntary muscle‟s movements are
D) Bronchi
coordinated by:
Ans: B
A) CST
Explanation: Clara Cells also called Bronchiolar
B) Cerebellum
Secretory Cells are nonciliated & secret a serous
C) Spinocerebellum
solution sililar to surfactant. Clara Cells are
abundant in ther terminal bronchioles, where they D) Basal Ganglia
Ans: A
make up about 80% of the epithelial cell lining.
Explanation:
2) Which of the following is the main
pillar of medial longitudinal arch of  Voluntary muscles are
the foot? coordinated/Executed by: Corticospinal
A) Talus Tract (CST).
B) Calcaneum  Controlled/Balanced by Cerebellum.
C) Cuboid 6) Ulcer on posterior wall of Gastric
D) Cuneiform antrum will lead to acute
Ans: A abscess/Peritonitis in which of the
Explanation: following space:
 Main pillar of medial longitudinal arch of A) Greater Sac
the foot is: Talus. B) Lesser Sac
 Pillar of Medial longitudinal arch C) Right Subhepatic Space
D) Right Posterior subphrenic space
posteriorly is Calcaenum
Ans: B
 Main support of medial longitudinal arch
Explanation:
of the foot is: Calcaneum.
 Ulcer on posterior wall of duodenum or
 Main pillar of lateral longitudinal arch of
Gastric antrum will lead to fluid
the foot is: Cuboid.
accumulation in: Lesser Sac.
3) Which of the following Hernia is
 Anterior duodenal perforation the
associated with herniation through
contents will accumulate in: Posterior sub
Hesselbach‟s triangle?
phrenic space.
A) Indirect Inguinal hernia
B) Direct Inguinal hernia  In erect posture in case of Duodenal
C) Femoral hernia perforation the contents will collect in:
D) Lumber Hernia Right Iliac Fossa > Right Paracolic gutter.
Ans: B 7) Patient with upper motor neuron
Explanation: lesion exhibits signs of right side
 Direct inguinal hernia occurs through hypertonia with exaggerated reflexes
& positive babinski‟s sign which of
hesselbach‘s triangle & lies medial to
the following are is affected:
Inferior epigastric vessels.
A) Left Broadman Area 4
 Indirect Inguinal hernia lies lateral to
B) Left Internal Capsule
inferior epigastric vessels.
C) Left Pyramids
4) Which of the following abnormality is
D) RubroSpinal
associated with Left Ventricular
Ans: A
Hypertrophy, early diastolic murmur
8) Under normal physiological condition:
& increase EDV?
A) Venous Return is equal with pulmonary
A) Aortic Regurgitation
flow
B) Aortic Stenosis
SK Origional – Golden 11 112

B) Venous Return is more than pulmonary D) Inhibits Hormone release


flow E) Decrease TSH
C) Venous Return is less than pulmonary Ans: B
flow Explanation:PTU inhibits the thyroid peroxidase
D) Venous Return is greater than cardiac enzyme, and thus inhibits the synthesis of thyroid
output hormones.
Ans: A 13) A patient on protein diet would also
9) A patient with history of resection of need to take:
terminal Ileum should be offered A) Thiamine
which of the following vitamin? B) Biotin
A) Vitamin B1 C) Riboflavin
B) Vitamin B12 D) Folate
C) Niacin Ans: C
D) Folate Explanation:
Ans: B  Thiamine is necessary for carbohydrate
Explanation: Vitamin B12 is absorbed in intake.
terminal ileum & after the resection of Terminal  Biotin is necessary for Lipids intake.
Ileum patient will need to take Vitamin B12  B2 (Riboflavin) is necessary for protein
prophylactically. intake.
10) Following an RTA, a patient 14) Highest presentation on the motor
developed right side homonymous homunculus:
hemianopia which of the following A) Hand
site is damaged: B) Lips
A) Right Optic Tract C) Face
B) Left Optic Tract D) Thigh
C) Optic Chiasma Ans: A
D) Optic Nerve Explanation:
Ans: B  Highest sensory area: Lips.
Explanation:  Highest Motor area: Thumb > Hand.
 A patient with lesion of right optic tract  Smallest motor area: Thigh.
will develop Left Homonymous 15) Damage to supraoptic nucleus will
Hemianopia. produce which of the following:
 A patient with lesion of left optic tract A) Increase Volume of Diluted Urine
will develop right Homonymous B) Decrease Volume of Urine
Hemianopia. C) Concentrated Urine
11) A female known case of thyrotoxicosis D) None of the Above
presented to ER with heart rate of 180 Ans: A
beats/mins and ECG shows Explanation: Supraoptic nucleus is responsible
irregularly irregular pulse, drug of for secreting ADH & in case of damage to this
choice will be: nucleus there will be no production of ADH that
A) Amiodarone will lead to dilute & increase amount of urine.
B) Propronolol 16) Which of the following paraneoplastic
C) Digoxin syndrome is associated with Oat cell
D) Verapamil carcinoma of lungs?
Ans: B A) ACTH
Explanation: Propronolol that‘s Beta blocker is B) Insulin
used as rate control drug. C) Glucagon
12) Mechanism of action of D) Androgen
prophylthiouracil: Ans: A
A) Inhibition of Iodine Pump Explanation: Oat Cell Carcinoma of lungs that‘s
B) Blocks the synthesis of thyroid hormone small cell carcinoma of lungs produces ACTH &
C) Blocks the synthesis of thyroglobulins ADH.
SK Origional – Golden 11 113

17) Most important lipoprotein with Ans: B


highest cholesterol content used by Explanation:
the cell:  Primary Syphilis: Formation of non-itchy
A) LDL and painless chancre
B) VLDL  Secondary Syphilis: Rashes
C) HDL  Tertiary Syphilis: Asymptomatic
D) Chylomicrons  Late Syphilis: Neurological disorders
Ans: A Cardiac Disorder
Explanation: 20) Patient who is known case of
 Lipoprotein with highest Phospholipids: hypothyroidism which of the following
HDL > LDL > VLDL > Chylomicrons will increase in the blood?
 Lipoprotein with highest Protein: HDL A) TSH
> LDL > VLDL > Chylomicrons B) T3
 Lipoprotein with highest cholesterol: C) T4
LDL > VLDL > HDL > Chylomicrons D) Cholesterol
 Lipoprotein with highest TG: Ans: D
Chlylomicrons > VLDL > LDL > HDL Explanation: TSH can increase only in primary
18) Patient presented with history of hypothyroidism but not in other cases while
Arthritis & Urinary problem that was cholesterol is increase in all hypothyroidism cases
found to be HLA-B27 +Ve & so we prefer Cholesterol > TSH.
associated with ocular problems: 21) Patient having history of weight gain,
A) Reiter‘s Syndrome hoarseness of voice, lab shows high
B) Ankylosing Spondylitis TSH & cholesterol, TRH was
C) UTI administered that leads to an
D) Rheumatoid Arthritis exaggerated increase in TSH level
Ans: A where does the problem lies:
Explanation: A) Thyroid
 HLA B27 is associated with: B) Hypothalamus
 Ankylosing Spondylitis C) Anterior pituitary gland
 Acute Anterior Uveitis D) Posterior pituitary gland
 Reactive Arthritis Ans: A
Explanation: This is case of primary
 Enteric Arthropathy
hypothyroidism that means problem lies in thyroid
 Psoriatic Arthritis gland as the patient already have symptoms of
 Reactive Arthritis formely known as hypothyroidism so the main problem lies in
Reiter‘s syndrome is a form of thyroid gland.
inflammatory arthritis that develops in 22) Medical lab technician presented with
response to an infection in another part history of fatigue for 4 months, LFTs
of the body (Cross-Reactivity). Reiter‘s deranged with ballooning &
syndrome consist of Arthritis, uveitis & inflammation of hepatocytes on
urethritis. biopsy, which test to perform?
19) A male with history of travel to A) HbsAg
Bangkok, presented with painless B) HAV
penile ulcer with localized C) HIV
lymphadenopathy that were found to D) EBV
be rubbery & painless upon Ans: A
examination, additionally he had rash 23) Esophageal Varices are
on his palms & soles: morphologically found in which layer
A) Primary Syphilis of GIT:
B) Secondary Syphilis A) Muscosa
C) Tertiary Syphilis B) Submucosa
D) Latent Syphilis C) Muscularis Externa
SK Origional – Golden 11 114

D) Serosa B) Vitamin C
Ans: B C) Vitamin E
Explanation: Esophageal varices are extremely D) Folate
dilated sub-mucosal veins in the lower third of the Ans: C
esophagus. Explanation: The potent antioxidant is
24) Which of the following organism Glutathione but as that‘s not mentioned so we
causes meningitis in age group 06 follow the following sequence: Gluthathione >
months to 02 years old? Vitamin E > Vitamin C > Vitamin A.
A) Streptococcus pneumonia 28) What does the word tachyphylaxis
B) Staph Aureus means?
C) E. coli A) Rapid decrease in drug responsiveness
D) Group B streptococcus B) Rapid increase in drug responsiveness
Ans: A C) Gradual increase in drug responsiveness
Explanation: The organisms causing meningitis D) Gradual decrease in drug responsiveness
in various age groups are different in various Ans: A
books but as per FA that‘s most favorite CPSP 29) Regarding weber syndrome:
book as under: A) Contralateral paralysis with ipsilateral
 0 – 6 Months: GBS, E coli ptosis and lateral deviation of eye
 6 Months – 6 Years: S.Pneumoniae, B) Ipsilateral paralysis with ipsilateral ptosis
N.meningitis, H-Influenzae C) Ipsilateral Paralysis & contralateral ptosis
 6 – 60 years: S.Pneumonia, N.meninitis D) Contralateral ptosis & ipsilateral weakness
25) A patient presented with upper GIT Ans: A
bleeding who is a known case of CLD Explanation: Weber‘s Syndrome, also known as
which of the following is the drug of superior alternating hemiplegia, a form of stroke
choice? that affects the medial portion of midbrain. It‘s
A) Inj. Octerotide characterized by the presence of ipsilateral
B) Inj. Terlipressin occulomotor nerve palsy & contalateral
C) Inj. Propornaolol hemiparesis or hemiplegia.
D) Inj. Histadine 30) Which one of the following is the
Ans: B MOA of TCA drug?
Explanation: The Drug of choice in upper GIT A) Block reuptake of serotonin &
bleeding in case of CLD is Injection terlipressin norepinephrine in post synaptic terminal
but in case of CLD with IHD then we prefer B) Block reuptake of Serotonin &
Injection octerotide, as terlipressin is Norepinephrine in Pre synaptic terminal
contraindicated in IHD. C) Block reuptake of Insulin
26) A newborn presented with repeated D) Increase reuptake of Serotonin &
respiratory tract infection, which of Norepinephrine.
the following physiological antibody is Ans: B
missing: 31) Segment of Lung with bronchus &
A) IgA arterial supply is called:
B) IgG A) Brochopulmonary Segment
C) IgM B) Bronchoalevolar Segment
D) IgD C) Bronchiole
Ans: B D) Bronchi
Explanation: Ans: A
 In Newborn (NG) the antibody missing is 32) Which of the following nerve is
IgG. damaged during ligation of inferior
 In case of child the antibody missing is thyroid artery?
IgA. A) External Laryngeal nerve
27) Which of the following is a potent B) Internal Laryngeal nerve
antioxidant? C) Recurrent Laryngeal nerve
A) Vitamin A D) Glossopharyngeal nerve
SK Origional – Golden 11 115

Ans: C A) Excessive blood loss


Explanation: B) Cardiogenic shock
 External laryngeal nerve is at risk while C) Anaphylactic Shock
ligating superior thyroid artery. D) Toxemic Shock
 Recurrent laryngeal nerve is at risk while Ans: A
ligating inferior thyroid artery. 38) A patient with history of injury to
33) Which of the following is the thigh presented after a month with
treatment of acute ulcerative colitis? swelling in the area with bony tissue
A) IV Corticosteriods found upon evaluation is most likely:
B) Sulfasalazine A) Metaplasia
C) Salicylates B) Dysplasia
D) Aminophyline C) Desmoplasia
Ans: A D) Anaplasia
Explanation: Ans: A
 In Acute/Severe Ulcerative Colitis: IV Explanation: This is the case of Myositis
corticosteroids. Ossificans, which is a condition in which bony
 In other cases: 5 Amino Salicylates, tissue forms inside muscle or other soft tissue
sulfasalazine being a prototype. after injury. It tends to develop in young adults &
34) Damage at sacral spinal level will lead athletes who are more likely to experience
to: traumatic injuries. Most of the time, Myositis
A) Automatic Bladder Ossifcans occurs in large muscles of the arm or
B) Spastic Bladder the legs.
C) Neurogenic Bladder 39) Which of the following is the
D) Autonomous Bladder difference between anaphylactic &
Ans: D hypovolemic shock?
Explanation: A) Decrease Cardiac Output
 Damage to sacral segment of spinal cord B) Increase Cardiac Output
leads to Atonic or Autonomous Bladder. C) Increase TPR
 Damage above sacral segment of spinal D) Increase Peripheral Vasoconstriction
cord leads to Spastic, Neurogenic or Ans: B
Automatic bladder. 40) 50 years man with back pain, apple
35) End arteries are present in: green birefringence & congo stain
A) Spleen positive on damage to kidney is most
B) Brain likely suffering from:
C) Kidney A) SLE
D) Retina B) Multiple Myeloma
E) Heart C) RA
Ans: D D) Scleroderma
Explanation: End arteries are present with Ans: B
following sequence to follow: Retina > Spleen > Explanation: Amloidosis & multiple myeloma
Functional End artery heart. shows green birefringence when stained with
36) History of fracture after a while congo red stain.
developed crepitus, foul smelling 41) Which of the following has highest
discharge with low BP: systolic pressure?
A) Hypovolemic Shock A) Aorta
B) Cardiogenic Shock B) Renal Artery
C) Anaphylactic Shock C) Renal Vein
D) Toxemic Shock D) Pulmonary Artery
Ans: D Ans: B
37) History of RTA with pelvic fracture Explanation: Systolic pressure increases in large
which of the following could be the peripheral arteries than aorta.
possible cause for low blood pressure?
SK Origional – Golden 11 116

42) Which of the following Anti-Tb drug Ans: C


causes inability to distinguish Explanation: Follow the following order for
between red & green colour? blood group antigens: RBC Surface >
A) Rifampicin Glycoprotein > Immunogenic > Dominant
B) Ethambutol Mendalian.
C) Streptomycin 48) Mask face with Tremors in loss of
D) INH neurons of which area?
Ans: B A) Substantia Nigra
Explanation: Among the following Anti-Tb B) Caudate nucleus
drugs ethambutol leads to Red-Green colour C) Cerebellum
blindness. D) Cortex
43) Old man died of MI five days back, on Ans: A
autopsy which of the following cells Explanation: The scenario looks like parkinson‘s
will be found? disease & that‘s due to loss of neuron in
A) Macrophages substantia nigra.
B) Neutrophils 49) Baby following vaccination developed
C) Eosinophils petichae with low platelets count is
D) Basophils most likely suffering from:
Ans: A A) TTP
44) Which of the following does B) ITP
phagocytosis in blood? C) VwD
A) Neutrophils D) Hemophilia
B) Macrophages Ans: B
C) Lymphocytes 50) A pregnant lady presented with low
D) Basophils MCV, Low MCHC, normal iron study
Ans: A profile which of the following need to
Explanation: Neutrophils does phagocytosis in be done in order to diagnose the case:
blood while macrophages in tissues. A) HbA2
45) Median nerve is damaged which of B) Ferritin
the following function of thumb will C) Iron profile
be lost? D) Transferrin Saturation
A) Extension Ans: A
B) Opposition 51) Which of the following muscle
C) Rotation protrude the mandible?
D) Adduction A) Medial Pterygoid
Ans: B B) Lateral Pterygoid
Explanation: Opponens pollicis is a small muscle C) Masseter
of hand that‘s act in opposition of the thumb & is D) Buccinator
supplied by recurrent branch of median nerve. Ans: B
46) In Scabies, itching is due to: 52) A patient with no associated risk
A) Crawling of mites over the skin factors presented with features of SCC
B) Hypersensitivity to Mite protein Lungs, factor most commonly
C) Type III HSR involved in this case:
D) None of the above A) Smoking
Ans: B B) Asbestosis
Explanation: The itching in scabies is type 4 C) Radon
HSR to mite protein. D) Thalium
47) Blood group antigens are: Ans: C
A) Equally immunogenic 53) Which of the following tumor marker
B) Present on Hb surface is raised in HCC?
C) Present on the cell membrane of RBC A) Alpha Fetoprotein ( AFP)
D) Autosomal dominant B) LDH
SK Origional – Golden 11 117

C) Serum Bilirubin permanently activating Gs that leads to increase


D) Hb chloride ion secretion in gut & H2O efflux which
Ans: A is the final pathway.
54) Which of the following measure Renal 58) Which of the following is the correct
plasma flow? sequence of medical ethics?
A) Hippurate A) Autonomy, Justice & Consent
B) Cretinine B) Autonomy, Non-maleficience & Consent
C) Inulin C) Autonomy,beneficence,Non-Maleficience
D) Insulin & Justice
Ans: A D) Autonomy & Justice
Explanation: Ans: C
 Renal plasma flow is measured by: Explanation: Remember ABNJ (Autonomy,
Hippurate (HPA). Beneficence, Non-Maleficience & Justice)
 GFR is best measured by: Inulin 59) Confidence interval is:
clearance. a. Mean & its standard error
 GFR is best Estimated by: Creatinine b. Standard error of mean & T-value
clearance. c. Mean & Mode
 GFR is best Measured clinically: d. Median & Standard Error
Creatinine clearance. Ans: A
55) Injury at foramen ovale will lead to: Explanation:
A) Loss of sensation of at lower face  Confidence interval: Mean & its standard
B) Loss of sensation at upper face Error.
C) Loss of sensation of forehead  Confidence limit: Standard error of mean
D) Loss of bitter taste & T-value.
Ans: A 60) Which of the following volume
Explanation: Mandibular nerve is passing combine makes FRC?
through formaen ovale & due to damage to this A) RV + ERV
foramen and nerve sensation of lower face will be B) RV – ERV
affected. C) IRV + RV
56) Blood Transfusion reaction is likely to D) VC + RV
occur when a person having blood Ans: A
group A+ is transfused with blood 61) When light strikes the eye what
group of: happens?
A) A+ A) Increase cGMP
B) A- B) Increase cAMP
C) AB- C) Increase neurotransmitter Release
D) O- D) Increase conversion of cis retinal to all
Ans: C trans retinal
Explanation: The rule is antibodies of the Ans: D
recipient will reach with antigens of the donor. Explanation: In presence of light Rhodopsin (11
Blood group A+ve will have antibodies B & no cis retinal + Opsin) dissociate to all trans retinal +
Rh antibodies so it will react with the antigens on Opsin.
blood group AB-ve. 62) CEA is normally found in embryonic
57) Vibrio causes due to which of the period, 60% pancreatic carcinoma and
following final pathway? is also found in which of the following
A) Increase in Chloride ion secretion carcinoma:
B) cAMP Activation A) Lungs
C) cGMP Activation B) Colon
D) Increase in Sodium ion secretion C) Liver
Ans: A D) Kidneys
Explanation: According to FA: Cholera toxin Ans: B
overactivates adneylate cyclase (cAMP) by
SK Origional – Golden 11 118

63) Which of the following 67) Which of the following is inactivated


hypersensitivity is associated with in the lungs?
erythroblastosis fetalis? A) Histamine
A) Type I HSR B) Serotonin
B) Type II HSR C) AG1
C) Type III HSR D) Renin
D) Type IV HSR Ans: B
Ans: B 68) Which of the following tumor marker
64) Patient presented with biliary colic is positive in meigs syndrome?
morphine was already given & which A) CA-123
of the following is needed to be added B) CA-125
now: C) AFP
A) Hyosine Butylbromide D) LDH
B) Nitrates Ans: B
C) BB Explanation: Meigs syndrome is defined as the
D) CCB triad of benign ovarian tumor with ascites and
Ans: A pleural effusion that resolves after resection of the
Explanation: As the patient presented with tumor.
biliary colic & there‘s spasm that causes pain so 69) Stroke volume equals to which of the
with morphine we add Hyosine Butylbromide. following:
65) A patient with temperature of 21.6 C & A) CO * HR
lying uncovered over the OT table, B) CO/HR
heat loss will occur through: C) HR/CO
A) Conduction & Radiation D) CO * TPR
B) Convection Ans: B
C) Radiation & Evaporation Explanation:
D) Convection & Radiation CO = SV * HR
Ans: A SV = CO/HR
Explanation: 70) A patient presented with history of low
 If patient is naked heat loss is through: grade fever, weight loss, productive
Radiation. cough & Granulomatous nasal lesion
 If patient is naked & lying (Contact with is most likely suffering form:
table) then: Conduction A) M-TB
 If patient is naked & temperature B) M-Leprae
mentioned: Radiation C) Staph Aureus
 If humidity mentioned: Convection D) Hemophilus
(Convection requires medium & here Ans: B
Humidity is medium). 71) Streptococal toxic shock syndrome is
 If trachestomy ETT: Evaporation due to:
66) Which of the following is correct A) Pyrogenic Exotoxin
sequence in Infective endocarditis? B) Streptolysin S
A) Damaged valve, thrombosis, Bacteremia, C) Streptolysin O
Perforation. D) Pyogenic Exotoxin
B) Damaged Valve, Bacteremia, thrombus & Ans: A
perforation Explanation:
C) Bacteremia, Perforation & thrombus  Pyrogenic exotoxin due to Streptococcus
D) Perforation & Bacteremia  Pyogenic exotoxin due to Staph Aureus.
Ans: A 72) Which of the following is the
Explanation: Remember DTBP for the sequence parasympathetic effect?
(Damaged Valve, Thrombosis, Bacteremia & A) Increases HR
Perforation). B) Decreases HR
C) Increase Contractility
SK Origional – Golden 11 119

D) Increases TPR Explanation: Crescents are formed by parietal


Ans: B cells & macrophages.
Explanation: Parasympathetic system decreases 78) Important finding at angle of Louis:
Heart rate, TPR & contractility. A) Opening of Azygous vein in SVC
73) A patient with abdominal pain, B) Opening of Azygous vein in IVC
bloating and foul smelling most useful C) Joining of SVC with Hemiazygous Vein
investigation would be: D) Joining of IVC with Hemiazgous vein
A) Stool D/R Ans: A
B) Culture 79) At which of the following pressure
C) Gram Staining does pulmonary valve opens?
D) Colonscopy A) 8 mmhg
Ans: A B) 25 mmhg
Explanation: Stool D/R (Detailed Report) gives C) 80 mmhg
information about all stool contents foul smelling D) 120 mmhg
stools are due to fat malabsorption i.e. Steatorrhea Ans: A
& culture and sensitive has nothing with it. Explanation:
74) Long scenario with features of  Pulmonary valve opens at 8 mmhg
hyperthyroidism & hypersensitivity  Aortic valve opens at 80 mmhg
reaction of grave‟s disease was asked: 80) Pinworm is caused by:
A) Type I HSR A) Ingestion of Larva
B) Type II HSR B) Ingestion of Egg of larva
C) Type III HSR C) Ingestion of worm
D) Type IV HSR D) None of the above
Ans: B Ans: B
Explanation: 81) Long scenario of with swelling of jaw
 Grave‘s Disease: Type II HSR & EBV positive is most likely
 Hashimoto‘s thyroiditis: Type IV HSR suffering from:
75) Which of the following is urine output A) Burkitt Lymphoma
in a 70kg person in 24 hours? B) HIV
A) 550 ml C) Wegner Granulomatosis
B) 1000 ml D) Churg strauss syndrome
C) 1500 ml Ans: A
D) 3000 ml 82) Which of the following will decrease
Ans: C in case of decreased blood pressure?
Explanation: Normal Urine output in a 70 kg A) Neuronal activity of chemoreceptor
person is 1500 – 2200 ml. B) Neuronal activity of Baroreceptor
76) Which of the following Anti Tb drug C) Activity of cardiac contractility
inhibits transcription of DNA? D) None of the above
A) Rifampin Ans: B
B) Ethambutol Explanation: In case of hypotension & low
C) INH blood pressure the firing rate of baroreceptor
D) Streptomycin decreases in order to increase sympathetic
Ans: A response & decrease parasympathetic response.
Explanation: Rifampin inhibits bacterial DNA 83) After long fasting gluconeogensis
dependent RNA polymerase and thus prevents depends on:
transcription of DNA into mRNA. A) Insulin: Glucagon ratio
77) RPGN, Crescents are formed by: B) Availability of amino acid alanine
A) Parietal Cells C) Availability of Fatty acids
B) Epithelial Cells D) Availability of Glucose
C) PCT Ans: B
D) LOH 84) Cytoskeleton connected to ECM by:
Ans: A A) Intermediate filament
SK Origional – Golden 11 120

B) Integrin C) Flexor Digitorium


C) Proteoglycan D) Abductor pollicis
D) Cadehrin Ans: A
Ans: B 90) Which of the following is part of triple
Explanation: screening?
 Cytoskeleton connected to ECM by A) Estriol
Integrin. B) Estrone
 ECM connected to ICM by Intermediate C) Estradiol
filament. D) Progesterone
85) ESR is increased with which of the Ans: A
following: Explanation:
A) Increase Albumin  Estriol in triple screening & after marriage
B) Decreased Albumin  Estrone in post-menopausal
C) Decreased Fibrinogen  Estradiol in premenopausal just engaged
D) Decreased Globulin 91) Which of the following function is
Ans: B unique to fallopian tube?
Explanation: Decreased Albumin leads to A) Conduit between Uterus & Cervix
increase ESR & increase albumin leads to B) Conduit between Peritoneum & Uterus
decreased ESR. C) Lined with Stratified columnar epithelium
86) Albumin (Protein) 0.5 Gram/dl is D) Length of 20cm in adults
found in: Ans: B
A) Lymph 92) Chronic diarrhea with crescent shaped
B) CSF organism:
C) Blood A) Giardia
D) Serum B) Staph
Ans: A C) E. Coli
87) Oxyhemoglobin curve shifts to left: D) Cryptococcus
A) Haldane effect Ans: A
B) Bohr effect 93) Anterior wall MI, Left ventricle
C) Ficks effect damaged which of the following artery
D) None of the above is involved:
Ans: A A) RCA
Explanation: B) LAD
 HaLdane effect: Shift to Left C) PAD
 BohR effect: Shift to right D) Right marginal artery
88) Which of the following antacid Ans: B
decreases gastric empyting? 94) At Mc Burney‟s point during
A) Magnisum hydroxide appendectomy, the structure likely to
B) Aluminum hydroxide be damaged is:
C) Chloride hydroxide A) Femoral Nerve
D) Cadmium B) Iliohypogastric Nerve
Ans: B C) Deep Circumflex Artery
Explanation: D) Superficial Circumflex Artery
 Aluminum hydroxide is used in upset Ans: B
stomach, indigestion & chronic diarrhea. Explanation:
 Magnisum hydroxide is used in chronic  If structure or nerve mentioned then:
constipation. Iliohypogastric nerve.
89) Supinator muscle damaged which of  If artery mentioned then: Deep Cirumflex
the following muscle is alternate Artery.
muscle of supination: 95) Afferent Renal arterioles form:
A) Bicep A) Glomerulus
B) Tricep B) Efferent arteriole
SK Origional – Golden 11 121

C) Peritubular capillaries Ans: C


D) None of the above 101) Difference between dwarfism &
Ans: A cretinism, as the later has:
Explanation: Afferent Renal Arterioles leads to A) Less mental backwardness
Glomerulus that in turn leads to efferent arterioles B) Severe mental backwardness
followed by peritubular capillaries. C) More short stature
96) Which of the following is tumor D) Less short stature
marker of both pancreatic & colonic Ans: B
carcinoma? 102) Decrease in oxygen saturation without
A) CEA change in oxygen tension is seen in:
B) AFP A) CO poisoning
C) LDH B) High Altitude
D) CA-125 C) Cyanide Poisoning
Ans: A D) Hypoventilation
97) Cushing reflex is initiated by: Ans: A
A) Raised ICP 103) PO 100mmHg is seen in which of the
B) Decreased ICP following?
C) Increased baroreceptor response A) Right to Left shunt
D) Increase fluid overload B) Left to Right shunt
Ans: A C) PDA
Explanation: Because the skull is rigid after D) VSD
infancy, intracranial masses or swelling may Ans: B
increase intracranial pressure when ICP is 104) Surfactant deficiency leads to:
increased sufficiently, regardless of the causes A) Cardiac impairment
cushing reflex and other autonomic abnormalities B) Renal Impairment
can occur. C) RDS
98) Central Chemoreceptors responds to: D) GIT Abnormality
A) Increase PCO2 Ans: C
B) Decrease PCO2 105) Commencement of trachea:
C) Increase PO2 A) C3
D) Decrease PO2 B) C4
Ans: A C) C5
 Central Chemoreceptors responds to: D) C6
Increase H+ ions in CSF > Increase Ans: D
PCO2> Decrease PO2. 106) Fibrinogen is converted to Fibrin
 Peripheral Chemoreceptors responds to: which of the following product is
Decrease PO2>Increase PCO2. released?
99) Which of the following drug causes A) Fibrinopeptide A
thrombocytopenia? B) FibrinoPeptide B
A) Quinine C) Laminin
B) Thiazide D) Cadherin
C) Captopril Ans: A
D) Propranolol Explanation: Fibrionpeptide A is a 16-Amino
Ans: A acid cleavage product of thrombin induced
Explanation: proteolytic cleavage of fibrinogen.
 Quinine > Thiazide 107) Maturation of Seminiferous tubules is
100) Dura of middle cranial fossa is done via:
supplied by: A) Androgen
A) Glossopharyngeal nerve B) Androgen & FSH
B) Vagus nerve C) Androrgen & LH
C) Trigeminal nerve D) Inhibin
D) Ophthalmic Nerve Ans: B
SK Origional – Golden 11 122

108) Endotoxin causes: 114) Peripheral nervous system consists of


A) IL1 activation which of the following?
B) IL6 activation A) Cranial nerves
C) Complement activation B) Spinal nerves
D) TNF activation C) Cranial nerves + Spinal Nerves
Ans: C D) Autonomic Ganglia
109) Which of the following decreases Ans: C
osmolarity? 115) Asbestosis is related to which of the
A) Vasopressin (ADH) following:
B) Epinephrine A) Benign mesothelioma
C) Aldosterone B) Malignant Mesothelioma
D) Renin C) Endothelial cancer
Ans: A D) Bladder Cancer
Explanation: Vasopressin (ADH) absorbs water Ans: B
& decreases plasma osmolarity. Explanation: Asbestosis is related with
110) Diazepam relaxes skeletal muscle by bronchogenic carcinoma > Malignant
acting on: Mesothelioma.
A) Cerebral cortex 116) Which of the following is seen in
B) Cerebellum cardiac muscles after irreversible
C) Interneuron muscle injury?
D) Spinal cord A) Mitochondrial rupture
Ans: C B) Nucleus rupture
Explanation: Diazepam acts via GABA B C) Contraction band necrosis
receptor at interneurons. D) None of the above
111) Short term increase in blood pressure Ans: C
is by which of the following? 117) Meiosis is a type of cell division in
A) RAAS which?
B) Baroreceptor A) DNA synthesis takes place
C) CNS Ischemic Response B) Exchange of genetic material takes place
D) Chemoreceptor in autosomes.
Ans: B C) One cell give rise to eight spermatozoa
Explanation: D) Take place in somatic cells
 Short term & rapid increase in blood Ans: B
pressure is by baroreceptor. 118) Ptosis & mydriasis is seen in which of
 Long term changes in blood pressure are the following nerve damage:
by RAAS. A) 4th CN
 Most potent increase in blood pressure is B) 3rd CN
by CNS Ischemic Response. C) 6th CN
112) Which of the following is responsible D) 10th CN
for increasing blood flow to exercising Ans: B
muscles? 119) Mother Rh +Ve & father is Rh –ve
A) Local Metabolites what‟s the risk?
B) Parasympathetic effect A) All babies will be effected
C) Sympathetic effect B) Needs serious counseling
D) Adrenergic Agonist C) Rh antigen will leak into mother
Ans: A circulation
113) Primary Ooctyes enter meiosis during: D) Rh antibodies will be formed
A) Intra uterine life Ans: C
B) Puberty Explanation: If mother is Rh+ then the babies
C) Before fertilization won‘t be affected & no need of worry, but still
D) After Ovulation antigen can leak into mother circulation & that‘s
Ans: A the best option among the mentioned.
SK Origional – Golden 11 123

120) In exercise, venous return is due to:  The sixth arch persists as proximal
A) Contraction of leg muscles portion of right pulmonary artery.
B) Relaxation of leg muscles 125) A patient on beta blocker presented
C) Increase TPR with prolong PR interval which of the
D) Decrease Stroke Volume following could be the possible cause?
Ans: A A) Drug induced 1st degree heart block
121) Which of the following receptors are B) 2nd Degree heart block
with no adaptation? C) Complete heart block
A) Pain receptors D) Sinus Tachycardia
B) Baroreceptors Ans: A
C) Chemoreceptors 126) Which of the following ion has
D) Pacinin Receptors profound effect in ECF?
Ans: A A) Calcium
122) Man working in ship industry is at risk B) Sodium
of developing: C) Potassium
A) Urothelial carcinoma D) Chloride
B) Gastrick Carcinoma Ans:. B
C) Bronchogenic Carcinoma C127) Which of the following is caused by
D) GIT Carcinoma sympathetic over activity?
Ans: C A) Bronchodilation
123) Posterior 1/3rd of interventricular B) Bronchoconstriction
septum artery accompanies which C) Decreases HR
vein? D) Decrease Cardiac contractility
A) Great Cardiac Vein Ans: A
B) Middle Cardiac Vein Explanation: Sympathetic activity leads to
C) Lesser Cardiac Vein bronchodilation, increase HR & cardiac
D) Coronary Sinus contractility.
Ans: B 128) Which of the following is post
Explanation: sympathetic cholinergic?
 Posterior interventricular artery A) Intestine
accompanies Middle cardiac vein. B) Sweat Glands
 Anterior interventricular artery C) Muscles
accompanies Great cardiac vein. D) Cardiac tissues
124) Maxillary artery supplies which of the Ans: B
following aortic arch? 129) Which of the following is normal adult
A) 1st hemoglobin?
B) 2nd A) HbA
C) 3rd B) HbF
D) 4th C) Gower Hb
Ans: A D) HbA2
Explanation: Ans: A
 Maxillary artery is the branch of 1st arch Explanation: Both HbA & HbA2 is found in
(Mandibular Arch). normal adult but HbA > HbA2.
 The ventral end of the second develops 130) Which of the following muscle leads
into the ascending pharyngeal artery & its to Knee flexion & hip extension?
dorsal end gives origin to stapedial artery. A) Sarotrius
 The third arch gives raise to the proximal B) Semitendinous
end of internal carotid artery. C) Adductor magnus
 The fourth arch forms the right D) Rectus Femoris
subclavian artery. Ans: B
 The fifth arch regresses. Explanation:
 Hip & Knee flexion: Sartortius.
SK Origional – Golden 11 124

 Hip flexion & Knee extension: Rectus 137) Pregnant lady with delayed reach to
Femoris. hospital presented with dyspnea most
 Hip Extension & Knee Flexion: likely due to:
Semitendinosus. A) Fat Embolism
 Hip Extension without knee flexion: B) Air Embolism
Adductor Magnus. C) Amniotic Fluid Embolism
131) Which of the following is side effect of D) Hemorrhage
Levodopa? Ans: C
A) Seizure Explanation: Pregnant lady with dyspnea without
B) Orthostatic Hypotension any other factors like trauma is most likely
C) HTN amniotic fluid embolism.
D) Provoking Angina 138) Which of the following is nerve supply
Ans: B of SCM?
132) AJK earthquake leading to death of his A) Accessory nerve
uncle & now in depression: B) Vagus nerve
A) Social crisis C) Trigeminal nerve
B) Emotional crisis D) Abducent nerve
C) Situational crisis Ans: A
D) Developmental crisis 139) Which artery leads to posterior wall
Ans: C MI?
133) Bell‟s phenomenon is: A) RCA
A) Bell‘s Palsy B) LCA
B) Eye globe downwards and drooping of lid C) Cirumflex
C) Eye globe upwards, drooping of lid D) Marginal artery
D) Eye globe moves upward when he tries to Ans: A
close his eyes 140) Outer sheath of the nerve is called:
Ans: D A) Endoneurium
134) Triple pattern if microtubules are B) Perineurium
present in: C) Epineurium
A) Kinetochore D) None of the above
B) Centrioles Ans: C
C) Centrosomes Explanation:
D) Centromere  Individual nerve fiber covered with:
E) Mitotic apparatus Endoneurium.
Ans: B  Nerve fascicle is covered with:
135) Which of the following is normal Perinuerum.
GFR?  Whole bundle of neurons that‘s complete
A) 110 ml/min nerve outer covering is: Epineurium.
B) 125 ml/min 141) Xerostomia, dry eyes & otitis media,
C) 135 ml/min where is the nuclei located?
D) 150 ml/min A) Cerebral cortex
Ans: B B) Pons
136) An Old diabetic patient presented C) Midbrain
with gastroparesis which of the D) Medulla
following drug would be helpful? Ans: B
A) Prokinetics 142) Patient had difficulty in breathing for
B) PPI some days, O/E breath sounds are
C) H2 Blockers decreased on both sides and dullness
D) Antacids on percussion, CXR shows bilateral
Ans: A fluid & leg edema what could be the
Explanation: In diabetic gastroparesis the drug of possible cause?
choice is prokinetics i.e Metoclopromide. A) Hypoalbuminemia
SK Origional – Golden 11 125

B) Hypoglobulinemia 148) Which of the following is Virus related


C) Hypernatremia cancer?
D) Deranged LFTs A) T Cell Leukemia
Ans: A B) Renal cell carcinoma
143) Splenic immunological function: C) Squamous cell carcinoma
A) Red Pulp D) Diffuse cell carcinoma
B) White Pulp Ans: A
C) Marginal Zone 149) Which of the following is feature of
D) Arterioles megaloblastic anemia?
Ans: B A) Hypersegmented Neutrophils
Explanation: B) Hypersegmented Basophils
 Splenic immunological function: White C) Hypersegmented Lymphoctes
pulp. D) None of the above
 Splenic immunological filtration: Red Ans: A
pulp. 150) Which of the following is most potent
 APC portion of spleen: Marginal zone. stimulus for Aldosterone release?
144) Which of the following hormone A) Hypokalemia
decreases contraction of stomach & B) Hyperkalemia
gallbladder? C) Hypernatremia
A) Insulin D) Hyponatremia
B) VIP Ans: B
C) CCK Explanation: Aldosterone that‘s released most
D) Secretin potently by increase Potassium level and leads to
Ans: B hypokalemia & hypernatremia.
145) During hysterectomy Ureter is 151) Cheliosis is due to deficiency of which
commonly damaged at which of the of the following vitamin?
following site: A) Thiamine
A) Pelvic brim B) Biotin
B) Broad Ligament C) Riboflavin
C) Bifurcation of Common Iliac artery D) Folate
D) Behind IVC Ans: C
Ans: B 152) Woman married now presented with
Explanation: During hysterectomy Ureter is RIF pain & suprapubic pain, which of
damaged with the following order: Cardinal the following organism is involved?
Ligament > Broad Ligament > Pelvic Brim A) Chlamydia
146) A body cut his finger the first response B) Gonorrhea
will be: C) E. coli
A) Vasoconstriction D) Bacteriods
B) Vasodilation Ans: A
C) Increase Permeability Explanation: This is a case of PID & the
D) Decrease Permeability commonest organism causing PID is Chlamydia
Ans: A followed by Gonorrhea.
147) Oral fecal route of which of the 153) Decrease GFR due to blood loss is
following? due to:
A) Hep. A A) Decrease Arterial blood pressure
B) Hep. B B) Decrease Arterial blood flow
C) Hep. C C) Increase Arterial blood pressure
D) CMV D) Decrease Albumin
Ans: A Ans: B
Explanation: For feco-oral route Hep A > Hep 154) Patient having hypokalemia,
E. hypertension, with normal cortisol
SK Origional – Golden 11 126

&VMA & raised aldosterone is most 160) Graft will be successful in which of the
likely suffering from: following condition?
A) Cushing disease A) Increase Corticosteriods
B) Cushing Syndrome B) Agammaglobuinemia
C) Conn‘s Syndrome C) Aspirin Overdose
D) Pheochromoctyoma D) In old age
Ans: C Ans: B
Explanation: As VMA is normal so 161) Obese female pre surgery had taken
Pheochromocytoma is excluded and then with gentamicin BD is likely to have
normal cortisol cushing disease & syndrome is impaired:
excluded. This is case of Conn‘s syndrome due to A) Filtration
increase aldosterone level that has led to decrease B) Excretion
potassium level, hypertension. C) Absorption
155) Which of the following is potent D) Hepatic Metabolism
anabolic hormone? Ans: B
A) DHA 162) Which of the following is major cause
B) Testosterone of non-healing ulcer?
C) Estrogen A) Infection
D) Inhibin B) Old age
Ans: B C) Protein deficiency
156) Which of the following is low velocity D) Suture
fibers: Ans: A
A) A fibers Explanation:
B) B fibers  Local cause for non-healing wound ulcer
C) C fibers is: Infection
D) A delta fibers  Systemic cause for non-healing wound
Ans: C ulcer is: DM
157) A patient presented with history of 163) Which of the following is secreted by
joint pain & gastric ulcer which of the fibroblasts?
following is drug of choice? A) Collagen
A) Aspirin B) Hyaline
B) Celecoxib C) Amorphus Ground substance
C) Corticosterids D) None of the above
D) PGs Ans: C
Ans: B Explanation:
Explanation: The patient suffers from peptic  Fibroblasts secrete amorphous ground
ulcer that‘s why we can selectively give CoX-2 substance.
inhibitors for pain that‘s celecoxib.  Fibrocartilage secretes collagen.
158) Thoracic aorta passes posterior to: 164) The ability of the kidney to excrete
A) Root of left lung concentrated urine will increase if:
B) Root of right lung a. The permeability of the proximal tubules
C) Behind IVC to water decreases
D) Behind Azygous Vein b. The rate of blood flow through the
Ans: A medulla decreases
159) Loss of parietal cells due to damage to c. The rate of flow through the LOH
which of the following area? increases
A) Fundus d. The activity of Na-K pump in the LOH
B) Antrum decreases
C) Duodenum e. The permeability of the collecting duct to
D) Lesser curvature water decreases
Ans: A Ans: B
SK Origional – Golden 11 127

Explanation: The rate of blood flow decreases in C) Subarchnoid Space


hypovolemia through medulla & hence D) None of the above
concentrated urine is produced. Ans: C
165) Which of the following is investigation 171) Quiet breathing with increase AP
of choice for infective endocarditis? diameter due to which of the following
A) ASO titer muscle?
B) Blood Culture A) Diaphragm
C) Urine Culture B) External Intercostal muscle
D) Sputum Culture C) Internal Intercostal muscle
Ans: B D) Rectus Abdominis
Explanation: Most preferable test in infective Ans: B
endocarditis is blood culture followed by ECHO ExplanationThe muscle in quiet respiration is
to confirm the diagnosis. Diaphragm but here increase AP diameter that‘s
166) A patient with primary not possible with diaphragm so here external
hyperthyroidism disease would most interocostal muscle.
likely have decrease of which of the 172) Eventration of diaphragm occurs due
following? to improper function of:
A) TSH A) Dorsal mesentery of esophagus and
B) T3 pleuroperitoneal membrane
C) T4 B) Dorsal mesentery of esophagus &
D) rT3 pleuropericardial membrane
Ans: A C) Lateral muscular wall with
Explanation: pleuroperitoneal membrane
 Primary Hyperthyroidism will have D) Ventral mesentery of esophagus & lateral
increase T3, T4 & decrease TSH. body wall.
 Primary Hypothyroidism will have Ans: C
decrease T3, T4 & Increase TSH. 173) Which of the following is study of
167) Which part of lung only have two (02) disease and non-diseased?
bronchopulmonary segments? A) Case Cohort
A) Right Upper Lobe B) Cohort
B) Right Middle Lobe C) Cross sectional
C) Right Lower Lobe D) RCT
D) Left Upper lobe Ans: A
Ans: B Explanation:
168) Which of the following HSR in  Disease and non-diseased: Case control
myasthenia gravis?  Presence and absence of Risk factor to
A) Type I HSR see outcome: Cohort
B) Type II HSR 174) Most common cause of negative
C) Type III HSR montoux test:
D) Type IV HSR A) Immunocompetent
Ans: B B) Immunosuppression
169) Which of the following occurs in C) Increase Steroids Intake
pregnant uterus? D) Old age
A) Hyperplasia Ans: B
B) Hypertrophy 175) Which of the following hormone
C) Hyperplasia + Hypertrophy increased during sleep?
D) Atrophy A) GH
Ans: C B) Cortisol
170) Which of the following is site for LP C) Insulin
for CSF collection? D) Glucagon
A) Epidural Space Ans: A
B) Extradural Space 176) Severe chest with diabetic ACS is due
SK Origional – Golden 11 128

to: Ans: B
A) Embolism Explanation: Atlas is made of a thick anterior
B) Thrombus arch and a think posterior arch with lumen & skull
C) Damaged valves articulation but has no spinous process.
D) None of the above 182) Circle of willis is formed by:
Ans: A A) One anterior cerebral arteries
177) A man presented with fever, rigors & B) Three posterior cerebral arteries
renal parenchymal damage is most C) Two Posterior Communicating arteries
likely suffering from: D) Two anterior communicating arteries
A) UTI Ans: C
B) Chronic Pyelonephritis Explanation: Circle of willis is formed by two
C) Acute Pyelonephritis anterior cerebral arteries, two posterior cerebral
D) Appendicitis arteries, two posterior communicating arteries &
Ans: C one anterior communicating artery.
178) A child died with having multiple 183) Lung compliance is increased in:
cysts on autopsy of kidneys, what is A) Old Age
the mode of inheritance of the B) Bronchoconstriction
disease? C) Fibrosis
A) Autosomal dominant D) Negative Pressure
B) Autosomal Recessive Ans: A
C) X-Linked Dominant Explanation: Lung compliance is increased
D) X-Linked Recessive mainly in old age due to loss of elastic tissue while
Ans: B vessel compliance is decreased.
Explanation: Polycystic kidney disease is
autosomal recessive in children & autosomal
dominant in elders. ===============
179) Patient falls to the left with tremors at
the end of movement & abnormal gait
is most likely having lesion of:
A) Right Cerebellum
B) Left Cerebellum
C) Right Cerebrum
D) Left Cerebrum
Ans: B
Explanation: The signs & symptoms are related
to cerebellar lesion and in cerebellar lesion the
patient falls toward the side of the lesion, like here
the patient falls to left side so the lesion is in left
cerebellum.
180) Which of the following is having
highest vitamin A?
A) Carrots
B) Lemons
C) Oranges
D) Apples
Ans: A
181) Which of the following is not found in
atlas?
A) Articulation with skull
B) Spinous process
C) Lumen
D) Arch
SK Origional – Golden 11 129

MEDICINE & ALLIED Explanation: The greater auricular nerve supplies


th
September 30 , 2020 – Night the fascia superficial and deep to parotid gland
and transmits the pain caused by stretching of the
fascial envelope when acute enlargement of the
1) Which of the following is locally
malignant tumor? gland occurs as in mumps.
6) Which of the following nerve damage
A) Melanoma
leads to winging scapula?
B) Squamous Cell Carcinoma
A) Accessory Nerve
C) BCC
B) Long thoracic nerve
D) Adenoid cystic carcinoma
C) Axillary Nerve
Ans: C
D) Phrenic Nerve
Explanation:
Ans: B
 Locally Malignant tumor is Basal cell
Explanation: Long thoracic nerve supplies
carcinoma.
Serratus Anterior muscle & damage to long
 Locally spreading tumor is
thoracic nerve will lead to winging scapula.
Ameloblastoma.
7) Osteons are present in:
 More Aggressive tumor is melanoma. A) Spongy bone
2) What are chondrocytes? B) Compact bone
A) Part of Bone C) Cartilages
B) Mature cartilage D) Muscle Fibers
C) Mature bone
Ans: B
D) Immature cartilage Explanation:Compact bone consists of closely
Ans: B packed osteons or harversian systems. The osteon
Explanation: Chondrocytes are cells of mature
consists of a central canal called the ostenic
cartilages. (Haversain) canal, which is surrouned by
3) An Old diabetic patient presented concentric rings (Lamellae) of matrix.
with gastroparesis which of the 8) Distant metastasis is used for:
following drug would be helpful: A) Grading
A) Metoclopromide
B) Invasion
B) PPI
C) Staging
C) H2 Blockers D) Cadherin
D) Antacids Ans: C
Ans: A Explanation:
Explanation: In diabetic gastroparesis the drug of
 Staging determines extent of tumor.
choice is prokinetics i.e metoclopramide.
 Grading determines degree of
4) Oral fecal route of which of the
differentiation.
following:
9) Repetitive division/mitosis is seen in:
A) Hep A
A) Grading
B) Hep B
B) Staging
C) Hep C
C) Permanent Cells
D) CMV
D) Cardiac Cells
Ans: A
Ans: A
Explanation: For feco-oral route Hep A > Hep
10) Diabetic patient, presented with
E.
abdominal pain & visible peristalsis is
5) Which of the following nerve supply
most likely suffering from:
skin over the parotid region?
A) Pyloric Stenosis
A) Greater Auricular Nerve
B) Pyloric Perforation
B) Auriculotemporal Nerve
C) Gastric perforation
C) Greater Occipital Nerve
D) Intestinal Obstruction
D) Lesser Occipital Nerve
Ans: A
Ans: A
11) Ataxic gait & ipsilateral face sensation
lost where is the site of lesion:
SK Origional – Golden 11 130

A) Cerebro Pontine Angle B) CN III


B) Cerebellopontine Angle C) CN IV
C) Midbrain D) CN VI
D) Pons Ans: B
Ans: B Explanation: Ptosis is due to lesion of CN III
12) ACA supplies which of the following that supplies Levator palpebrae superioris.
area? 17) Interruption to cervical sympathetic
A) Somatosensory association area chain resulted in Ptosis, Miosis &
B) Brocas Area anhydrosis is due to:
C) Motor Cortex A) Horner Syndrome
D) Visual Cortex B) CN III damage
Ans: A C) CN VI damage
13) Lower limb fine sensation is carried D) Pons Lesion
by: Ans: A
A) Fasciculus Gacilis Explanation: Horner syndrome is characterized
B) Fasciculus Cuneatus by interruption of cervical sympathetic chain that
C) Spinothalamic Tract results in partial ptosis, miosis and anhydrosis.
D) Ventral thalamic tract 18) A newborn after few hours of birth
Ans: A presented with active bleeding, which
Explanation; of the following could be the possible
 Lower Limbs fine sensation is carried out management:
by Fasciculus Gracilis. A) Vitamin K Administration
 Upper Limb fine sensation is carried out B) Whole Blood
by Fasciculus Cuneatus. C) Platelets
14) Which of the following tract carries D) Cryopreciptate
sensation of crude touch, Pain & Ans: A
temperature? Explanation: In a newborn the gut is sterilized &
A) DCML not properly developed that‘s why there is not
B) Anterolateral Tract enough bacteria to synthesis vitamin K.
C) Olivocerebellum tract 19) Which of the following is the drug of
D) Corticospinal Tract choice for Pseudomembranous
Ans: B Colitis?
Explanation: A) Vancomycin
 The posterior or dorsal column medial B) Metronidazole
leminscal pathway carries proprioception, C) Tinidazole
vibration sense and fine discriminative D) Clindamycin
touch. Ans: B
 The Anterolateral pathways include the Explanation: Pseudomembranous Colitis is
spinothalamic tract and other associated commonly associated with antibiotic overuse
tracts that convey pain, temperature sense specially Clindamycin and the drug of choice for
& crude touch. Pseudomembranous Colitis is Metronidaozle
15) Face sensation lost along with hearing while in severe cases we go for Vancomycin if not
loss, which of the following is the site managed with metronidazole.
of lesion? 20) A mother has given folic acid, which
A) Pons of the following other condition can
B) Medulla lead to Neural tube defect:
C) Internal Auditory Meatus A) Vitamin A toxicity
D) Cortex B) Vitamin B12 Overdose
Ans: C C) Niacin Deficiency
16) Diplopia along with ptosis is most D) Biotin
likely due to lesion of: Ans: A
A) CN II
SK Origional – Golden 11 131

21) Spinal nerve comprises of which of the A) Increase Albumin


following? B) Decrease Albumin
A) Motor component C) Increase Fibrinogen
B) Sensory Component D) Decrease Hydrostatic Pressure
C) Motor & Sensory Component Ans: B
D) None of the above Explanation: Edema could be caused by decrease
Ans: C oncotic pressure that‘s due to decrease albumin
22) Regarding Anti Parkinson‟s drugs use level or can occur due to increase hydrostatic
which of the following is correct? pressure.
A) Reduce the risk of tardive dyskinesia if 27) Patient presented with vomiting
given with phenothiazine blood, upper GI endoscopy shows
B) Increase the risk of tardive dyskinesia if columnar epithelium, 3cm above
given with pehnothiazine the GE junction, most likely
C) No effect if used with phenothiazine morphological change is:
D) None of the above
Ans: B
A) Dysplasia
23) Single muscle fiber is covered with: B) Metaplasia
A) Endomysium C) Anaplasia
B) Perimysium D) Atrophy
C) Epimysium Ans: B
D) Epineurum Explanation: This is case of barret
Ans: A esophagus & that‘s metaplastic change.
Explanation: 28) Which of the following is related to
 Single muscle fiber is covered with: worst prognosis in burn patient?
Endomysium A) Excessive burn
 Bundle of muscle fiber is covered with: B) DIC
Perimysium
C) Hypovolemia
 The entire muscle tissue is covered with:
D) Shock
Epimysium
24) Dysphagia to liquids is due to: Ans: B
A) Decrease peristalsis 29) A patient presented with
B) Loss of neuromuscular coordination hemisection of spinal cord, what
C) Loss of Motor tone changes could be seen in the
D) Carcinoma patient:
Ans: B A) Contralateral loss of pain &
Explanation: There is most likely loss of temperature at the site below the
intramural neurons loss in the wall of esophagus lesion
leading to loss of neuromuscular coordination. B) Ipsilateral loss of pain & Temperature
25) Hodgkins disease spread to lymph C) Contralateral loss of Proprioception
nodes through:
D) Spastic paralysis on both sides
A) Contagious spread
B) Non contagious Spread Ans: A
C) Invasion Explanation: Hemisection of spinal cord
D) Distant spread leads to contralateral loss of pain &
Ans: A temperature, Ipsilateral loss of proprioception
Explanation: & Ipsilateral spastic paralysis.
 Hodgkin‘s disease spread through 30) Plastic factory worker, exposed to
contagious spread hydrocarbons develop fever and
 Non-Hodgkin‘s disease spread through significant weight loss, suspected
non-contagious spread case of:
26) Child presented with edema is most A) Bladder Carcinoma
likely due to:
SK Origional – Golden 11 132

B) Gastric Carcinoma 36) Which cervical ganglion is located


C) UTI opposite to 2nd cervical vertebrae?
D) Mesothelial Carcinoma A) Superior Cervical Ganglion
Ans: A B) Inferior Cervical Ganglion
31) Actin at rest has tropomysin what‟s C) Middle Cervical Ganglion
D) None of the above
the function of it: Ans: A
A) Blocks mysoin binding sites on actin Explanation:
B) Activates mysoin binding sites on  The Superior Cervical Ganglion is located
actin opposite the second and third cervical
C) Causes movement of troponin vertebrae.
D) Binds with Calcium  The inferior Cervical ganglion is located
Ans: A opposite to fourth and sixth cervical
Explanation: At rest, tropomysin blocks myosin vertebrae.
binding sites on actin secured by troponin,  The middle cervical ganglion is the
calcium binds to troponin that cause movement of smallest of the three cervical ganglia and
troponin and release tropomyosin thus exposes is occasionally absent, it‘s placed opposite
myosin binding sites on actin. to sixth cervical vertebrae.
32) Cell is arrested at G1 by: 37) A 20 years pregnant lady who is Rh
A) TP53 negative gives birth to Rh negative
B) Rb Gene baby, shee has history of allergic
C) K-Ras reactions. Due to bleeding she was
D) Bcl2 advised transfusion but she was
Ans: A transfused with Rh positive blood
33) A boy after circumcision has history of accidently. Which of the following is
profuse bleeding, which of the true:
following factor will confirm the A) Anti D antibodies must be given
diagnosis: immediately to lady
A) Aptt B) Next baby may develop hemolytic
B) PT Anemia
C) Factor 8 (Minor) C) No reaction will occur
D) Factor 8 (Major) D) No need to give Anti D antibody
Ans: C Ans: A
Explanation: Explanation: According to NICE guidelines any
 For hemophilia initial diagnosis is: APTT contact of Rh Positive blood with Rh negative
 Confirmatory test for hemophilia: Factor mother or in case of any sort of placental
8 ( Minor). abruption or delivery complication the mother
34) In Factor 8 deficiency which of the must be transfused with Anti D on immediate
following is administered: basis.
A) Cryopreciptate 38) Anti-hyperlipedemiac drug side effect:
B) Platelets A) GIT disturbances
C) Packed red Blood Cells B) Constipation
D) None of the above C) Seizure
Ans: A D) Hypertension
35) In active bleeding during factor IX Ans: A
deficiency which of the following is Explanation: The commonest side effect for
administered? most of the drugs is GIT disturbances.
A) Platelets 39) On percussion of posterior aspect of
B) Cryoprecipitate chest along the scapular line in quiet
C) FFP respiration, the resonance extends
D) Whole Blood down the level of:
Ans: C A) 6th Rib
SK Origional – Golden 11 133

B) 8th Rib 44) Nutmeg liver scenerio, alcoholic


C) 10th Rib man with raised JVP and hepatic
D) 12th Rib congestion, likely cause is:
Ans: C A) Chronic Alcoholism
40) Which of the following is the level of B) RHF
Xiphoid process?
A) T7
C) Passive Congestion
B) T8 D) Periportal congestion
C) T9 Ans: A
D) T10 Explanation: Cause is Chronic Alcoholism.
Ans: C While condition is RHF due to distended
Explanation: T9 is the level of Xiphoid Process neck veins.
& T7 is dermatome. 45) Mallory bodies & AST is increased
41) Which of the following is the initial in which of the following:
step in atherosclerosis? A) Alcoholic Hepatitis
A) Endothelial Injury B) Drug Induced hepatitis
B) Fatty Streaks formation C) Viral Hepatitis
C) Release of PDGF
D) Plaque formation
D) Liver Cirrhosis
Ans: A Ans: A
Explanation: Endothelial injury increases 46) Inflammation of portal triads that
Permeability and LDL gets inside tunica intima, spills into adjacent lobules, bands
the LDL and lipid eaten by macrophages will form of inflammation extending
foam cells, that will lead to fatty streaks formation between portal areas and terminal
and platelet adhesion, which will release PDGF hepatic (Central) Veins with foci of
and smooth muscle proliferation and will form liver necrosis, diagnosis:
fibrous cap, Plaque will be formed and after A) Chronic Passive congestion
plaque rupture thrombogenic foam cells will be B) Chronic Active hepatitis
exposed. C) Fulminant hepatitis
42) An elderly obese man with severe
atherosclerosis developed acute
D) Cirrhosis
abdominal pain. On laprotomy small Ans: B
intestine was dark purple in colour. 47) Which of the following nucleus is
Superior mesenteric artery was found in upper 1/3rd of Medulla?
atherosclerotic and thrombosed, A) Inferior Olivary Nucleus
mesenteric vein is patent, this small B) Superior Olivary Nucleus
intestine pathology will be called: C) Nucleus Ambigous
A) Dry gangrene D) Vagus Nucleus
B) Red Infarction Ans: A
C) Volvulus formation Explanation: Upper 1/3rd of Medulla has
D) Wet Gangrene inferior olivary Nucleus > Vestibulocochlear
E) GIT bleeding
Ans: D
nucleus.
43) A 60 years old ate street food 48) Which of the following nucleus
developed lung abscess due to: give raise to CN 9,10,11?
A) Indirect spread from liver A) Nucleus soliterius
B) Direct Spread from liver B) Nucleus ambigous
C) Release of exotoxin C) Central nucleus
D) Release of endotoxin D) Spinal nucleus
Ans: B Ans: B
SK Origional – Golden 11 134

49) Which of the following is the D) Glomerulus


prominent step in irreversible cell Ans: A
injury? 54) Which of the following is the
A) Cell membrane damage function of Erythropoietin?
B) Massive Calcium influx A) Increase Hemoglobin
C) Cellular Swelling B) Decrease hemoglobin
D) Mitochondrial swelling C) Increase Platelet count
Ans: A D) Decrease Platelet count
Explanation: Ans: A
 The prominent step of Irreversible cell 55) Pulmonary hypoxic
injury according to pathoma is Cell vasoconstriction occurs due to:
membrane damage while in Robbins A) Increase PO2
mechanism explains massive calcium B) Decrease PO2
influx too but that‘s not clearly C) Increase PCO2
mentioned. D) Decrease PCO2
 The prominent step of Reversible cell Ans: B
injury is cellular swelling. Explanation: In pulmonary Vasculature
50) First posterior intercostal vein Decrease PO2 leads to vasoconstriction while
drains into: in other vessels of the body decrease PO2
A) Superior vena cava leads to vasodilation.
B) Azygous Vein 56) Pain in left flank, Hx of weight
C) Hemiazygous Vein loss, hematuria, BUN and
D) Brachiocephalic Vein creatinine normal, calcium 11 and
E) Subclavian Vein HGB 18, most likely:
Ans: D A) Liver carcinoma
51) Which of the following is the B) Renal cell carcinoma
mechanism of action of shigella C) UTI
toxin? D) Prostatitis
A) Exotoxin Ans: B
B) Endotoxin Explanation: The scenario leads toward
C) Mucosal Invasion Problem in kidney and most likely RCC due
D) Release of TNF to weight loss and increase hemoglobin.
Ans: C 57) Renin is inhibited by:
Explanation: Invasion of M Cells is the A) Angiotensin
keyto pathogenicity. B) Increase sodium
52) RMP is maintained by: C) Increase Adlosterone
A) Potassium influx D) Bradykinin
B) Potassium Efflux Ans: A
C) Na-K pump 58) Angiotensin causes:
D) Sodium influx A) Increase thirst
Ans: C B) Decrease thirst
Explanation: RMP is produced by potassium C) Decrease Plasma Osmolarity
efflux & maintained by Na-K pump. D) Decrease Blood pressure
53) Erythropoietin is secreted by: Ans: A
A) Peritubular Capillaries 59) Which of the following leads to
B) Mesangial Cells fetal excitation when administered
C) PCT along with MAOI?
A) Fentanyl
SK Origional – Golden 11 135

B) Pethidine C) External & Internal Jugular Vein


C) Halothane D) Right & Left Subclavian Vein
D) H2 Blocker Ans: A
Ans: B Explanation: The brachiocephalic vein is
60) In which part of Nephron HCO3 formed by the confluence of the subclavian
absorption occurs? vein & internal jugular vein.
A) PCT 66) Preload depends on:
B) DCT A) CVP
C) PCT + CT B) End diastolic pressure
D) LOH C) End diastolic Volume
Ans: C D) Venous Return
61) Which of the following is Ans: C
difference between PCT & DCT? Explanation: Preload depends on End
A) PCT has brush boarder diastolic volume & Venous Return, but
B) DCT has brush boarder according to books lines it‘s EDV.
C) Maximum Absorption occurs in DCT 67) Which of the following is muscle of
D) Maximum water is absorbed in PCT mastication & attached to mid
Ans: A mandible?
62) Which of the following segment A) Masseter
has most dilute urine? B) Mentalis
A) LOH C) Temporalis
B) DCT C) Buccinator
C) CT Ans: A
D) PCT Explanation: The masseter muscle
Ans: B providespowerful elevation & protrusion of
Explanation: Most dilute urine is found in the mandible by originating from the
Early DCT > Thick Ascending LOH. zygomatic arch & inserting along the angle
63) Diabetic, smoker, increased and lateral surface of the mandible.
cholesterol with increased LDL is 68) Which of the following protrudes
at a risk of: the Jaw?
A) Atheroma A) Medial Pterygoid muscle
B) Atherosclerosis B) Lateral Pterygoid muscle
C) Kidney Failure C) Buccinator
D) Liver Failure D) Platysma
Ans: B Ans: B
64) Which of the following is the 69) Mother has infection of
st
virulence of Tb? toxoplasmosis in 1 trimester, &
A) Exotoxin was treated. Now her baby is born,
B) Endotoxin to check acute infection in baby on
C) Cell Wall composition ELISA method which antibody will
D) Nucleus composition be there:
Ans: C A) IgA
65) Brachiocephalic Vein is formed by B) IgM
which of the following? C) IgG1
A) Subclavian Vein & Internal jugular D) IgE
vein E) IgG2
B) Sublcavian Vein & External Jugular Ans: B
vein
SK Origional – Golden 11 136

70) Hemoglobin Saturation increases Ans: B


in which of the following Explanation:
condition?  Right Supra renal vein drains into
A) Increase PO2 IVC.
B) Decrease PO2  Left Supra renal vein drains into left
C) Increase PCO2 renal vein.
D) Decrease PCO2 74) Which of the following is the
Ans: A feature of long bone?
71) Bulbar Palsy is related to which of A) Contact surfaces are covered with
the following: elastic cartilages
A) Upper Motor Neuron disease B) Has a central marrow cavity lined with
B) Lower Motor Neuron disease periosteum
C) Cerebellum lesion C) Has a central marrow cavity lined with
D) Basal Ganglia lesion endosteum.
Ans: B D) None of the above
Explanation: Ans: C
 Bulbar Palsy is a lower motor neuron Explanation: In adults the diaphysis is
Palsy that affects the nuclei of the 9th, cylindric with walls of compact bone and a
10th, 11th& 12th CN. central marrow cavity line with endosteum,
 Pseuodobulbar Palsy is an upper the epiphysis contains mostly spongy bone &
motor. contact surfaces are covered with hyaline
 Neuron palsy that affects the cartilage.
corticobulbar tracts of the 5th, 7th, 9th, 75) Which of the following is the cell of
10th, 11th& 12th. atopic asthma?
72) A patient climbed a peak with a A) Mast Cells
heavy bag on his shoulder after B) Eosinophils
that he developed loss of adduction C) Basophils
of arm with loss of flexion at elbow D) Lymphocytes
& loss of extension at wrist. Most Ans: A
likely damage occurs to which of Explanation: For Atopic Asthma prefer the
the following: cells in the following order:Mast Cells >
A) Radial Nerve Basophils > Eosinophils
B) Musculocutaneous nerve 76) Which of the following cells are
C) Axillary Nerve found in parasitic infection?
D) Upper Part of brachial plexus A) Basophils
Ans: D B) Eosinophils
Explanation: This is the case of Erb‘s Palsy C) Neutrophils
(waiter‘s tip) resulted from traction or tear of D) Mast Cells
upper trunk of brachial plexus (C5,C6) Ans: B
resulted in loss of abduction, lateral rotation, 77) A known smoker, presents with
flexion & supination of arm. abdominal pain, bloody vomit and
73) Left Supra Renal vein drains into a 3cm lesion with no mass at the
which of the following? gastric antrum:
A) IVC A) Peptic Ulcer Disease (PUD)
B) Left Renal Vein B) Gastric Carcinoma
C) Right Renal Vein C) Esophgeal Carcinoma
D) Common Iliac Vein D) Mallory weis syndrome
SK Origional – Golden 11 137

Ans: A B) Lipooxygenase pathway


78) Nucleus Ambigous contains which C) TXA1 Pathway
of the following Cranial nerves: D) ADP Pathway
A) CN 9, 10, 11 Ans: A
B) CN 9, 11, 12 Explanation: Aspirin belongs to the group of
C) CN 3, 4, 5 the drugs that inhibits platelets activation by
D) CN 1, 3, 5 blocking TXA2 pathway.
Ans: A 84) Lymphoid tissue covered with
79) Which of the following is the simple cuboidal epithelium:
nucleus of central medulla? A) Tubal Tonsil
A) Vagus Nucleus B) Palatine Tonsil
B) Nucleus Ambigous C) Peyer‘s Patches
C) Dorsal Nucleus D) Appendix
D) Inferior Olivary Nucleus Ans: C
Ans: B 85) Drug receptors are which of the
Explanation: following in nature?
 Nucleus Ambigous is the central A) Protein
nucleus of the medulla. B) Lipids
 Inferior Olivary Nucleus > C) Carbohydrates
Vestibulocochlear nucleus are the D) Carbohydrates+Lipids
nucleus of upper 1/3rd of the medulla. Ans: A
80) Which of the following necrosis is 86) In epithelial malignancy which of
seen in acute pancreatitis? the following is found?
A) Fat necrosis A) Cytokeratin
B) Caseous Necrosis B) Vimentin
C) Liquefective Necrosis C) Desmin
D) Gangrenous Necrosis D) Vesmin
Ans: A Ans: A
81) Which of the following necrosis is Explanation:
seen in brain?  Intermediate filament: Keratin
A) Coagulative Necrosis  Connective Tissue: Vimentin
B) Liquefective necrosis  Muscles: Desmin.
C) Fat Necrosis 87) Plasma half-life depends on:
D) Caseous Necrosis A) Dose
Ans: B B) Route of Administration
82) Which of the following are the C) Rate of Clearance
findings of viral hepatitis? D) Volume of distribution
A) Increased Bilirubin E) Rate of Absorption
B) Increased AST Ans: C
C) Increased ALT & Bilirubin 88) Continuous contraction without
D) Increased Albumin relaxation is:
Ans: C A) Tetany
83) How is Aspirin beneficial in B) Tetanus
inhibiting platelets aggregation by C) Tetanic Contraction
blocking which of the following D) Atony
pathway? Ans: C
A) TXA2 Pathway
SK Origional – Golden 11 138

89) Which of the following is A) Dysdiadoochokinesia


importance of Fusion fascia? B) Hypotonia
A) Position C) Ataxia
B) Lymphatic Drainage D) Dysarthria
C) Vascular Structures Ans: A
D) Innervation Explanation: All of the above are findings of
Ans: A cerebellar disease but dysdiadochokinsea is
Explanation: The avascular plane referred to the characteristic feature.
as fusion fascia of Toldt is well known among 95) A person drink 600 ml of water,
laparoscopic colorectal surgeons, as it how much water of it will be
facilitates the dissection of the right present in plasma:
mesocolon and peritoneum. A) 200 ml
90) Bone demineralization is caused B) 300 ml
by which of the following? C) 50 ml
A) PTH D) 100 ml
B) Calcitonin Ans: C
C) Cortisol Explanation:
D) Testosterone  Of 600 ml of water, 1/3rd that‘s 200
Ans: A ml will be found in ECF.
91) Waddling gait is caused by  Among 200 ml of ECF 1/4th will be
damaged to which of the following found in plasma that‘s 50ml.
nerve: 96) Heart works as a syncytium due to
A) Inferior gluteal nerve which of the following?
B) Superior gluteal nerve A) Intercalated discs
C) Sciatic Nerve B) Gap Junctions
D) Femoral Nerve C) Thickness
Ans: B D) Rapid AP
92) Lesion in which of the following Ans: B
area is related with rigidity & 97) Prominent prepotential is found in
resting tremor: which of the following?
A) Caudate nucleus A) SA Node
B) Substantia Nigra B) AV Node
C) Globus Pallidus C) Bundle of HIS
D) Thalamus D) Purkinjee Fibers
Ans: B Ans: A
Explanation: Lesion in substantia nigra leads 98) Hypotonic urine is found in which
toparkinson‘s disease that leads to rigidity & part of the nephron?
resting tremors. A) PCT
93) Which of the following syndrome B) DCT
has short stature & webbed neck? C) LOH
A) Down Syndrome D) CT
B) Patau Syndrome Ans: B
C) Turner Syndrome Explanation:
D) Edward syndrome  Dilute urine is found in early DCT >
Ans: C thick ascending LOH.
94) Which of the following is  If not with above sequence then
characteristic lesion in cerebellar prefer DCT.
disease?
SK Origional – Golden 11 139

99) A Person sweats 2L and drinks 2L 104) Common peroneal nerve relation to
of pure water, what will happen? bicep femoris tendon in popliteal
A) Increase ICF Volume fossa:
B) Increase ECF Volume A) Medial
C) Increase ICF Osmolarity B) Lateral
D) Increase ECF Osmolarity C) Anterior
Ans: A D) Posterior
Explanation: This will lead to Increase ICF Ans: A
volume Greater than ICF osmolarity. 105) Which of the following is correct
100) A woman runs on a treadmill for a route of CSF route?
few minutes, HR doubles, cardiac A) Fourth ventricles to Third ventricles
cycle length will be: B) Lateral ventricles to fourth ventricles
A) 0.8 Seconds C) Foramen Magendi to subarachnoid
B) 0.4 Seconds space
C) 0.1 Seconds D) Foramen Lushka to magendi
D) 1 Minutes Ans: C
Ans: B 106) Female only eats animal based diet
Explanation: As cardiac cycle takes 0.8 and lacks fruits and vegetables,
seconds now that the heart rate is doubled the likely cause of poor wound healing
cardiac cycle will decrease by half that‘s 0.4 would be:
Seconds. A) Defective Collagen
101) A soldier presented with 3 months B) Vitamin C deficiency
history of fever, bilateral C) Vitamin K deficiency
lymphadenopathy, D) Decrease tensile strength
hepatospleenomegaly, anemia, Ans: B
fatigue & H/O of sleeping on floor 107) A doctor was offered a chance of
and bitten by some fly, diagnosis: travel in exchange for taking part
A) Kala Azar in upcoming discussion of a drug
B) Malaria in the hospital, what should be the
C) CML response of the doctor?
D) Filariasis A) Accept it
Ans: A B) Refuse Politely
102) A patient having ecchymosis & C) Inform hospital administration
reduced platelet count is suffering D) None of the above
from: Ans: B
A) Hemophilia 108) Hypertrophic cardiomyopathy:
B) ITP A) Myocyte Necrosis
C) TTP B) Myoctye disarray
D) VwD C) Systolic Dysfunction
Ans: B D) Decrease TPR
103) A student fall down while watching Ans: B
a surgery due to: 109) A wound healing that goes beyond
A) Increase TPR its margins is most likely
B) Decrease TPR A) Keloid
C) Increase CO B) Hypertrophic Scar
D) Decrease VR C) Contracture
Ans: B D) Hyperplastic
SK Origional – Golden 11 140

Ans: A 114) Normal distribution curve will


Explanation: A keloid scar crosses its have which of the following
margins & goes out of its original area. characteristics?
110) Which of the following is defective A) Mean, Mode & Median are equal
in Marfan syndrome? B) Mode > Median
A) Collagen C) Median > Mean
B) Elastin D) Mean > Mode
C) Fibrillin Ans: A
D) Hyaline 115) Most common site of ectopic
Ans: C pregnancy:
111) Successful PR prolongation A) Uterine Tube
followed by dropping of beat is B) Cervix
which type of heart block? C) Ovary
A) First Degree heart block D) Uterus
B) Mobitz type 1 Ans: A
C) Mobitz type 2 116) Fat malabsorption could lead to:
D) Complete Heart block A) Bleeding
Ans: B B) Steatorrhea
Explanation: C) Vitamin A deficiency
 First Degree heart block is D) Carbohydrate Imbalance
characterized by prolongation of PR Ans: B
interval. 117) Excessive talking that makes no
 Second degree heart block has two sense:
types: Mobitz type 1: Characterized by A) Wernickes Aphasia
prolongation of PR interval followed B) Brocas‘s Aphasia
by a beat drop. Mobitz type 2: C) Transcortical Aphasia
D) None
characterized by Constant PR interval
Ans: A
with beats drops. Explanation:
 Third degree heart block or complete  Excessive talking that makes no sense &
heart block characterized by no defective comprehensive ability in
association of SA node with AV node. Wernickes Aphasia.
112) In which of the following MI  Unable to produce words in Broca‘s
there‟s ST elevation in lead II, III Aphasia.
& AvF? 118) In Tuberculosis which of the
A) Inferior wall MI following necrosis is found?
B) Lateral Wall MI A) Caseous
C) Anterior wall MI B) Coagulative
C) Enzymatic
D) Posterior wall MI
D) Fat
Ans: A Ans: A
113) Chi square test is used for: 119) Heart with Rustling sound is most
A) Statistical analysis likely:
B) Categorical Analysis A) Myocarditis
C) Numerical analysis B) Pericarditis
D) Analytical Analysis C) Endocarditis
Ans: B D) Cardiac Tamponade
Ans: B
SK Origional – Golden 11 141

120) Pregnant lady, 20th week of 127) Angiotensin II mainly causes:


gestation, DOC for A) Decrease Thirst
hyperthryoidism: B) Secretion of Cortisol
A) PTU C) Secretion of Catecholamines
B) Methiamzole D) Vasoconstriction of blood vessels
Ans: D
C) Thyroxine
128) Patient having signs of hypoxia
D) Iodine
with normal breathing, PO2 95,
Ans: B
PCO2 24 and HCO3 18 with normal
Explanation:
PH:
 In First trimester: PTU
 In Second – Third trimester: Methimazole
A) Partially Compensated Metabolic
121) Child with rigors, chills & headache, Acidosis
organism detected on Giemsa: B) Fully Compensated Metabolic
A) P.Falciparum Acidosis
B) P.Ovale C) Respiratory Acidosis
C) Dengue Fever D) Mixed Acidosis
D) Enteric Fever Ans: B
Ans: A 129) Pnumatic bones mainly found in:
122) S1 Heart sound is produced during: A) Face
A) Iso Volumetric Contraction B) Skull
B) Iso Volumetric Relaxation
C) Neck
C) Ejection Phase
D) Relaxation phase
D) Upper Limb
Ans: A Ans: A
123) S2 heart sound is produced during: 130) Old aged man with testicular
A) Iso Volumetric Contraction swelling & raised LDH is most
B) Iso Volumetric Relaxation likely suffering from:
C) Ejection Phase A) Seminoma
D) Relaxation phase B) Lymphoma
Ans: B C) Embryonal Carcinoma
124) Nissel bodies are found in: D) Yolk Sac tumor
A) SER Ans: B
B) RER
Explanation:
C) Axon hillock
 Old aged male with raised LDH is
D) Mitochondrion
Ans: B suffering from Lymphoma.
125) On H & E staining a student sees the  Young aged male with Raised LDH is
hallow structure around the nucleus. Seminoma.
What it could be?  As general seminoma > Lymphoma.
A) Golgi Apparatus 131) Supraoptic nucleus secretes:
B) Lysosome A) ADH
C) Ribosome B) Oxytocin
D) Endoplasmic Reticulum C) AG2
Ans: B D) Renin
126) BCG vaccines produces which of the
Ans: A
following immunity?
A) Type I HSR
Explanation:
B) Type II HSR  ADH is mainly secreted by Supraoptic
C) Type III HSR nucleus.
D) Delayed Cell Mediated  Oxytocin is mainly secreted by
Ans: D Paraventricular nucleus.
SK Origional – Golden 11 142

132) Which of the following decreases C) Maltose


Stomach motility and increases D) Sucrose
intestinal motility? Ans: B
A) Gastrin 137) Which of the following is tumor
B) Secretin marker of hepatocellular carcinoma?
C) CCK A) AFP
D) Glucagon B) LDH
Ans: C C) Bilirubin
Explanation: D) CEA
 CCK decreases stomach motility & Ans: A
increases intestinal motility. 138) A farmer with OP Poisoning feature:
 Gastrin increases both gastric & intestinal A) Dilated pupils
motility. B) Bilateral pin pointed pupils
 Secretin decreases both Gastric & C) Decrease Salivation
intestinal motility. D) Decrease Lacrimation
133) A patient presented with acute left Ans: B
ventricular failure & dynpnea which of Explanation: Organophosphorus (OP) Poisoning
the following is the drug of choice? is characterized by bilateral pin pointed pupil,
A) IV Furosemide increase salivation & lacrimation.
B) ACEI 139) Which of the following most
C) ARBs common bleeding disorder from
D) Beta blockers female carrier to male born?
Ans: A A) VwD
Explanation: As this is case of Pulmonary B) Hemophilia
Edema due to left ventricular failure & in acute C) Bernard soulier syndrome
condition of having dypnea we give IV Lasix
D) ITP
(Furosemide).
134) Chlamydia causes which of the Ans: B
following: Explanation: Hemophilia is X-Linked disorder
A) Syphilis that‘s transferred from mother to male babies.
B) Granuloma Inguinlae 140) Bleeding into lesser omentum by
C) Lymphogranuloma Venereum damage to:
D) AIDS A) Left Gastric Artery
Ans: C B) Gastroduodenal artery
Explanation: C) Right Gastroepiploic artery
 Syphilis is caused by: T-Pallidum D) Celiac Trunk
 Granuloma Inguinale is caused by: Ans: A
Klebsiella granulomatis 141) Maximum normal systolic pressure in
right ventricle:
 Lymphogranuloma Venererum is caused
A) 8 mmhg
by: Chlamdia
B) 25 mmhg
 AIDS is caused by: HIV
C) 80 mmhg
135) CSF differs from plasma:
D) 120 mmhg
A) Increase PH
Ans: B
B) Decrease PH
Explanation:
C) Increase Protein
 Right ventricular systolic pressure: 25
D) Increase Glucose
mmhg.
Ans: B
 Left ventricular systolic pressure: 120
Explanation: CSF has decrease PH, Decrease
protein & decrease glucose compare with plasma. mmhg.
136) Semen contains: 142) Which of the following has highest
A) Glucose systolic pressure?
B) Fructose A) Aorta
SK Origional – Golden 11 143

B) Renal Artery 148) In acute infection of toxoplasmosis


C) Renal Vein which of the following antibody will
D) Pulmonary Artery be raised?
Ans: B A) IgM
Explanation: Systolic Pressure increases in large B) IgE
peripheral arteries than aorta. C) IgG
143) Which of the following has same D) IgD
osmolarity to plasma? Ans: A
A) 25% DW 149) Growth hormone increases maximally
B) 0.9% N/S during:
C) 3% NS A) Exercise
D) 10% DW B) Hypoglycemia
Ans: B C) Sleep
144) Which of the following is Epicardium D) Study
blood supply? Ans: B
A) Pericardiophrenic Artery Explanation: GH increase with following order:
B) Coronary Artery Hypoglycemia > Exercise > Sleep.
C) Carotid Artery 150) A person living at a high altitude is
D) Coronary Sinus most likely to have:
Ans: B A) Increase Hemoglobin
Explanation: B) Decrease Hemoglobin
 Epicardium is supplied by Coronary C) Pedal edema
artery > Pericardiophrenic artery.
D) Renal failure
 Pericardium is supplied by
pericardiophrenic artery.
Ans: A
145) In essential hypertension which of the Explanation: People at high altitude have
following is increased: secondary polycythemia due to decrease PO2.
A) Increase Oxygen demand 151) Child presents with active bleed
B) Increase size from umbilical stump, which of the
C) Increase Work load following vitamin deficiency?
D) Increase cardiac output A) Vitamin A
Ans: C B) Vitamin K
146) Which of the following is autsomal C) Vitamin C
dominant condition? D) Vitamin D
A) Familial adenomatous polyposis Ans: B
B) Hemophilia
Explanation: Newborn have deficient GIT flora
C) DMD
that‘s why there‘s mostly vitamin K deficiency.
D) None of the above
152) Following an RTA, a patient
Ans: A
developed right side homonymous
Explanation:
hemianopia which of the following
 FAP is autosomal dominant site is damaged:
 Hemophilia is X-Linked recessive A) Right Optic Tract
 DMD is X-Linked recessive B) Left Optic Tract
147) Which of the following antibody cross C) Optic Chiasma
the placenta? D) Optic Nerve
A) IgM Ans: B
B) IgE Explanation:
C) IgG  A patient with lesion of right optic tract
D) IgD will develop Left Homonymous
Ans: C Hemianopia.
SK Origional – Golden 11 144

 A patient with lesion of left optic tract 158) Pregnant lady delivered at home
will develop right Homonymous yesterday, presents now with active
Hemianopia. bleeding and low platelet count,
153) A known case of CRF, likely gland likely diagnosis:
to enlarge: A) TTP
A) Thyroid B) Hemophilia
B) Parathyroid C) DIC
C) Adrenal gland D) VwD
D) Pituitary Ans: C
Ans: B 159) A 20 year old lady has amenorrhea
154) Drug to be used with caution in for the past 6 months but otherwise
patient on warfarin: reportedly normal, cause:
A) Cimetidine A) Anovulation
B) PPI B) PCOS
C) ARBs C) Precocious Puberty
D) Vitamin K D) Granulosa Cell Tumor
Ans: A Ans: A
Explanation: Cimetidine is a potent inhibitor of 160) Pregnant ladies are always advised
P450 enzyme that acts on warfarin & will increase to lie on the left lateral position to
chance of bleeding. avoid compression of:
155) Successful resuscitation after shock A) Aorta
treatment: B) IVC
A) Increase Pulse C) Common Iliac Artery
B) Increase BP D) Femoral Artery
C) Increase Urinary Output Ans: B
D) Increase Cardiac Output 161) Which of the following organ is
Ans: C resistant to hyper acute graft
156) Which of the following is rejection?
immediate mechanism for heat A) Kidney
loss? B) Heart
A) Cutaneous Vasodilation C) Liver
B) Cutaneous Vasoconstriction D) Lung
C) Activation of Anterior hypothalamus Ans: C
D) Activation of Preoptic nucleus 162) Which of the following is the drug
Ans: A of choice of Schizophrenia?
157) Lyme disease is caused by which A) Risperidone
of the following? B) Fluoxetine
A) Chlamydia C) SSRI
B) Gonorrhea D) Alprazolam
C) Spirochete Ans: A
D) Viral Explanation: According to the American
Ans: C Psychiatric Association, second generation
Explanation: (Atypical) antipsychotics with the exception of
 Lyme disease is caused by Borrelia Cloazapine are the agents of choice for the
Burgdorferi is a bacterial species of first line treatment of Schizophrenia.
the spirochetes class of the genus 163) Which of the following are lungs
borrelia. main fighter cells?
A) Dendritic macrophages
SK Origional – Golden 11 145

B) Alevolar Macrophages Ans: A


C) Masts Cells Explanation:
D) Neutrophils  Atonic bladder is due to tabes dorsalis
Ans: B or lesion of sacral segment of spinal
164) Double ureter is due to: cord.
A) Ureteric bud fails to reach  Above sacral segment of Spinal cord
B) Ureteric bud fails to induce lesion leads to Spastic/Neurogenic
C) Early division of Ureteric bud bladder or automatic bladder.
D) Loss of mesoderm 168) HLA B-27 is associated with:
Ans: C a. Ankylosing Spondylitis
Explanation: b. Gout
 When ureteric bud fails to reach c. Sepsis
mesonephric cap: Renal Agensis d. IBS
 When ureteric bud fails to induce Ans: A
Explanation:
maturation: Polycystic Kidney
 HLA B-27 is associated with ―PAIR‖
 When Ureteric bud division occurs  P= Psoraitic Arthritis
early: double Ureter  A= Ankylosing Spondylitis
165) Blood stem cells maturation is  I= IBD associated Arthritis
stimulated by:  R= Reactive Arthritis
A) Erythropoetin 169) Left gastric is a branch of:
B) Growth Factors A) Celiac Trunk
C) Cortisol B) Proper hepatic artery
D) Testosterone C) Gastroduodenal artery
Ans: B D) Superior mesenteric artery
Explanation: Ans: A
 RBCs production is stimulated by Explanation:
 Left Gastric artery is a branch of Celiac
Erytropoetin.
trunk.
 Other cells are stimulated by Growth  Right gastric artery is a branch of proper
Factors. hepatic artery.
166) Mylination of fetus starts at: 170) Low MCV is seen in which of the
A) 3rd Month following?
B) 4th Month A) IDA
C) 6th Month B) Thalassemia
D) 2 Years C) Anemia Of chronic disease
Ans: B D) Megaloblastic anemia
Explanation: Ans: A
 Mylination starts during 4th month of 171) Which of the following act as 1 carbon
carrier?
intrauterine life A) Biotin
 Mylination of Cranial nerves 6th B) Folate
month C) Phosphorus
 Mylination is completed at 2-3 Years D) tRNA
167) Tabes dorsalis leads to: Ans: A
A) Atonic Bladder Explanation:
B) Neurogenic bladder  Transfer: Folate
C) Spastic Bladder  Carrier: Biotin
D) Automatic Bladder 172) Pellegra occurs due to deficiency of
which of the following?
A) Niacin
SK Origional – Golden 11 146

B) Thiamine 178) Cardiac tissue is most vulnerable to


C) Folate ventricular fibrillation at which point?
D) Riboflavin A) At the start of action potential
Ans: A B) Just at the end of action potential
Explanation: Niacin deficiency leads to Pellegra C) At peak of action potential
(Diarrhea, Dementia & Death). D) At mid of Action potential
173) Structure which lies anterior to trachea Ans: B
in thoracic cavity: 179) True about IBS:
A) Arch of Aorta A) Psychosomatic disturbance
B) Thoracic Duct B) Continues Constipation
C) Jugular Vein C) Continues Diarrhea
D) Esophagus D) None of the above
Ans: A Ans: A
174) Rectum in pelvic region is identified
by:
Explanation: IBS is related to psychosomatic
A) Complete muscular coat disturbances that are characterized by
B) Incomplete muscular coat alternating constipation followed by diarrhea.
C) Anterior to prostate
D) None of the above
Ans: A ===============
175) Which of the following is sulfur
containing amino acids?
A) Cysteine MEDICINE & ALLIED
B) Phenylalanine October 1st, 2020 - Afternoon
C) Lysine
D) Arginine 1) A young boy presented with jaundice
Ans: A and fever. What test will you do to
Explanation: Methionine, Cysteine, confirm the diagnosis?
Homocysteine and Taurine are the four common A) ALT & Bilirubin
sulfur containing amino acids. B) Hep A & Hep B
176) Premalignant lesion which leads to C) Hep C
SCC and need excision is: D) ALP
A) Actinic Keratosis Ans: A
B) Bowen‘s Disease Explanation: In young patients, jaundice is most
C) Nevus likely hemolytic jaundice & we go for ALT &
D) Leucoplakia bilirubin to confirm the diagnosis.
Ans: A 2) A patient was operated for abscesses
Explanation: in posterior triangle of neck, now
 Premalignant lesion which leads to SCC is cannot raise her arm overhead, which
bowen‘s disease. nerve is damaged:
 Premalignant lesion which leads to SCC A) Vagus Nerve
and need excision is Actinic Keratosis. B) Spinal part of Accessory nerve
 Premalignant lesion on cheeks which C) Cervical plexus
leads to SCC and need excision is Actinic D) Axillary Nerve
Keratosis. Ans: B
177) Primary active transport requires: Explanation: Overhead abduction is caused by
A) Carrier Trapezius & Serratus anterior muscles. Here as
B) Pump abscess is drained in posterior triangle of neck
C) Protein that‘s where spinal part of accessory nerve is at
D) Lipids risk of damage.
Ans: B 3) Phospholipid is an important
component of cell membrane due to:
SK Origional – Golden 11 147

A) It is insoluble in water Explanation: The symptoms & the abroad


B) It has both polar & non polar ends history with findings are suggestive of AIDs.
C) It is covalently bond to protein 8) A man with presented stroke, ACA is
D) It only has polar ends
affected, it will affect which area of
Ans: B
4) Which of the following vein directly brain:
drains into SVC? A) Somatosensory Area
A) Azygous Vein B) Brocas Area
B) Hepatic Vein C) Occipital Area
C) Hemiazygous Vein D) Primary Motor area
D) IVC Ans: A
Ans: A
9) In aircraft, during ascent the pilot
5) Which of the following vessel supply
posterior interventricular septum? have his blood pooled in veins of
A) Left Coronary Artery lower limbs. What will happen?
B) Right Coronary Artery A) Arterial pressure will decrease
C) Right Marginal Artery B) No change on arterial pressure
D) Left Marginal Artery C) Venous return increased
Ans: B D) Increased cardiac output
6) Lips are closed with the help of which
Ans: D
muscle?
10) A man went from sea level to 1200
A) Orbicularis oris
height in 2 days. After 12 hours
B) Orbicularis oculi
reaching the destination, he developed
C) LevatorlabiSuperioris
dyspnea. What the cause?
D) Buccinator
A) Pulmonary edema
Ans: A
B) Secondary polycythemia
Explanation:
C) Primary Polycythemia
 The Orbicularis Oris muscle, the lip
D) Heart Failure
muscle is a broad elliptical muscle around
the mouth that when tonically contracted Ans: A
closes the mouth Explanation: High Altitude Pulmonary Edema
 Lateral Pterygoid muscle is the only (HAPE). Is a noncardiogenic pulmonary edema
muscle of mastication that actively opens which typically occurs in lowlanders who ascend
the jaw. rapidly to higher altitudes. Early symptoms
7) A man who is taxi driver in UAE for 3 includes non-productive cough, dyspnea on
years, has now returned to Pakistan exertion & reduced exercise performance & later
working and presented with C/O on dyspnea occurs eve at rest.
fever, weight loss, anorexia, diarrhoea, 11) A 60 year old businessman is
spleenomegaly. He is emaciated, has evaluated by physician for his high BP
jaundice which was 185/130mm Hg.
andGeneralizedLymphadenopathy.Wh Labrevealedincreased plasma renin
at is the diagnosis: activity.His plasma aldosterone level
A) AIDS is high.Left renal Vein Renin level is
B) Leishmaniasis high and right Renal Vein Renin Level
C) Yellow fever is decreased.Most Likely diagnosis is:
D) Dengue Fever A) Left renal Artery Stenosis
Ans: A B) Right renal Artery Stenosis
C) Bilateral Renal Artery Stenosis
SK Origional – Golden 11 148

D) Adrenal Mass Ans: A


Ans: A Explanation:
Explanation: As the patient has hypertension  Remember the pneumonic PED:
with increased aldosterone & renin level , and o P = Perioneal
after investigation it was found that renin was o E = Eversion
increased in left renal vein so this is most probably o D = Dorsiflexion
16) A patient had an abdominal surgery.
case of Left renal artery stenosis.
After few months, the scar became
12) Platelet adhesion to endothelium is
enlarged and nodular, biopsy shows
caused by:
high collagen and fibroblasts:
A) Smooth endothelial surface
A) Foreign body due to sutures
B) Rough endothelial surface
B) Keloid scar
C) Increased Hydrostatic pressure
C) Infection
D) Thrombocytosis
D) Contracture
Ans: B
Ans: B
13) Mesangial cells have contractile
17) Tachyphylaxis:
property which helps them to A) Rapid unresponsiveness
maintain: B) Deficiency of Receptors
A) Renal blood flow C) Gradual decrease in drug action
B) Blood pressure D) None
C) Erythropoiesis Ans: A
D) Aldosterone secretion 18) Hyperextension of Neck will lead to:
A) Lower motor neuron lesion of Upper
Ans: A
limb
14) A 14yr old boy with fluctuating type of B) Upper motor Neuron lesion of Upper
deafness. On examination, Tympanic Limb
membrane retracted inward. Rinne C) Lower motor neuron lesion of Lower
test is negative, weber lateralized to limb
affected ear. What is the probable D) Loss of Tactile stimulus lower limb
cause? Ans: A
19) Sudden relief of respiratory distress
A) Otitis media
with oxygen. Patient has apnoea now.
B) Eustachian tube blockage
What is the cause?
C) Adenoids
A) Decreased arterial O2
D) SNHL
B) Increased arterial O2
Ans: B
C) Venous blood CO2 fall to 45mmg
Explanation:As the Rinne test is negative it
D) Venous blood CO2 fall less than
means it is conductive hearing loss that‘s
25mmHgA
confirmed with the weber test that‘s lateralized
Ans: B
toward the affected side, now the probable cause
20) Extraembryonic mesoderm is formed
for fluctuating hearing loss at this age is
by:
Eustachian tube blockage. A) Hypoblast
15) CommonPeronealnerve damaged. B) Epiblast
What will be affected the most? C) Lateral plate mesoderm
A) Eversion D) Intermediate Mesoderm
B) Inversion Ans: B
C) Plantar flexion Explanation:
 Extraembryonic mesoderm: Epiblast >
D) Plantar flexion of big toe
Hypoblast
SK Origional – Golden 11 149

 Extraembryonic coleom: Hypoblast A) Beginning of menstruation


 Intraembryonic coelom: Lateral Plate B) Ovulation
mesoderm C) Degeneration of corpus leuteum
21) A 45 years old lady with D) Secretory Phase
hyperpigmentation, loss of sensation Ans: B
of lower limbs, lethargy, with Explanation:As the estrogen level is high &
decreased Hb, Raised MCV, serum Progesterone isn‘t detectable this is most likely
ferritin and iron raised Diagnosis: ovulation phase that leads to LH surge.
A) Megaloblastic anemia 25) Fanconi syndrome, decrease ATP at
B) Sideroblasticanemia the PCT linked with:
C) IDA A) Hyperphosphatemia
D) Thalassemia B) Glycosuria
Ans: A C) Metabolic alkalosis
Explanation:As the MCV is increased & patient D) Hpermagnesemia
complains of loss of lower limbs sensations that‘s Ans: B
commonly seen in cases of megaloblastic anemia. Explanation:Fanconi Syndrome is a rare disorder
22) A man presented in E/R with of kidney tubule function that result in excess
shortness of breath. CXR shows amounts of glucose, bicarbonate, phosphates, Uric
enlarged heart and basal lung acid & certain Amino acids.
pulmonary edema. TLC normal which 26) Osteomyelitis of jaw is commonly
vitamin deficiency will beassociated caused by:
with this condition? A) Staph Aureus
A) Vit. K B) Streptococcus
B) Vit. C C) Pseudomonas
C) Vit. B1 D) Nocardia
D) Vit. B2 Ans: A
Ans: C 27) Most potent stimulus for release of
Explanation:This is most likely case of Beriberi aldosterone is:
that‘s caused by deficiency of Thiamine (Vitamin A) Renin
B1). B) Hyperkalaemia
23) Drug given for Pseudomonas C) Inc. serum glucose level
Infection: D) Increase serum sodium level
A) Vancomycin Ans: B
B) Ceftrioxone Explanation:Most potent stimulus for releaseof
C) Pencillin aldosterone is hyperkalemia, as aldosterone will
D) Ceftazidime lead to hypokalemia & hypernatremia.
Ans: D 28) Lipoprotein with highest
Explanation: phospholipids:
 DOC for Pseudomonas is: Ceftazidime A. HDL
 UTI by Pseudomonas DOC: B. LDL
Ciprofloxacin. C. VLDL
24) A lady who has Normal Menstrual D. Chylomicrons
cycle of 28 days and now since 12 hrs Ans: A
her estradiol level is on peak and Explanation:
progesterone is not detectable. What  Lipoprotein with highest Phospholipids:
HDL > LDL > VLDL > Chylomicrons
can be the possible cause?
SK Origional – Golden 11 150

 Lipoprotein with highest Protein:HDL > 35) Complete depolarisation of ventricles


LDL > VLDL > Chylomicrons on ECG is shown by:
 Lipoprotein with highest cholesterol: A) QT interval
LDL > VLDL > HDL > Chylomicrons B) ST segment
 Lipoprotein with highest TG :
C) PR interval
Chlylomicrons > VLDL > LDL > HDL
29) Vagus nerve cut & central part D) QTc complex
stimulated will lead to: E) QRS complex
A. Tachycardia Ans: B
B. Tachypnea 36) The DOC for HTN in
C. Increase Respiration pregnancy/pregnancy Induced
D. Apnea hypertension is:
Ans: D A) Methyldopa
30) Which of the following nerve damage
B) Hydralazine
leads to winging scapula?
A) Accessory Nerve C) Labetalol
B) Long thoracic nerve D) ACEI
C) Axillary Nerve Ans: C
D) Phrenic Nerve Explanation:
Ans: B  As Per JNC # 8 guidelines for HTN,
Explanation:Long thoracic nerve supplies labetaolol is first line drug for pregnancy
Serratus Anterior muscle & damage to long induced hypertension followed by
thoracic nerve will lead to winging scapula. Methyldopa.
31) A female presented with a complaint  In Emergency cases the DOC is
of itching, dryness of eyes and mouth, Hydralazine.
dysphagia, angular fibrosis. Her X-ray 37) Fetal period starts after which week:
shows interstitial fibrosis.Cause: A) 8 week
A) Sjorgensyndrome B) 9 week
B) Progressive systemic sclerosis C) 7 week
C) SLE D) 10 week
D) RA Ans: A
Ans: B (Kaplan ) Explanation:
32) Epicardium:  Embryonic Period: Upto 8 Weeks
A. Visceral Layer of Serous Pericardium  Fetal Period: After 8 Weeks
B. Parietal Layer of Serous Pericardium 38) A farmer presented in ER with
C. Parietal Layer of Myocardium frothing but no salivation and no
D. None excessive lacrimation, he is suspected
Ans: A case of OP poisoning, what other
33) Regarding Cartilage: clinical sign he will be present in this
A. Epiglottis is hyaline
B. Synovial Joint has fibrous cartilage case to reach the diagnosis:
C. Appositional growth A) Pin point pupils
D. Transitional growth B) Bradycardia
Ans: C C) Bronchoconstriction
34) Most Important component of Lipid: D) Dilated pupils
A. Phosphorus Ans: A
B. Carbon 39) Autoimmune disease against single
C. Potassium
organ:
D. Sodium
Ans: B A) Rheumatoid Arthritis
B) Hashimoto's Thyroiditis
SK Origional – Golden 11 151

C) SLE 44) A patient came with poisoning with


D) Sjogren Syndrome signs of opioid like poisoning, when
Ans: B given nalaxone, the symptoms don't
40) True statement about upper half of improve. What can be the probable
anal canal is: cause?
A) Sensitive to pain A) Opium
B) Drained by superficial inguinal lymph B) Phenothiazine
node C) Phenobarbitone
C) Lined by squamous epithelium D) Alcohol
D) Supplied by inferior rectal artery Ans: C
E) Sensitive to stretch 45) True about ependymal cells:
Ans: E A) Lines the ventricles and spinal canal
Explanation:The upper 2/3rd of Anal canal is B) Covers neurons
lined bysimple columnar Epithelium, supplied by C) Function in phagocytosis
Superior rectal artery & is sensitive to stretch. D) Help in defense
41) Site of red infarct is: Ans: A
A) Spleen Explanation:Ependymal cell, type of neuronal
B) Liver support cell (Neuroglia) that forms the epithelial
C) Kidney lining of ventricles in the brain & the central canal
D) Brain of spinal cord.
Ans: B 46) Which carcinoma is autosomal
Explanation: dominant?
 Red infarct occurs in organ with dual A) Hepatocellular carcinoma
blood supply like: Liver & Lung
B) Colon carcinoma
 Pale infarct occurs in solid organs like:
Heart, Spleen & Kidney C) Childhood retinoblastoma
42) Diapedesis refers to: D) DMD
A) Migration of WBCs from intravascular Ans: C
compartment to interstitial space 47) Most of the optic fibers ends at:
B) Margination of WBCs A) Lateral geniculate body
C) Chemotaxis B) Medial geniculate body
D) Deaths of Cells C) Optic chiasma
Ans: A D) Pretectum
43) Fracture involving radial groove of Ans: A
humerus will lead to: Explanation:
A) Loss of sensation at the dorsum of root  Optic fibers that‘s visual pathway ends at
of thumb LGB
B) Claw deformity  Auditory fibers ends at MGB
C) Extension of fingers  L for Light that‘s vision
D) Loss of Opposition of thumb  M for Music that‘s hearing
Ans: A 48) A female patient with achalasia would
Explanation:The Nerve running in the spiral or have following:
radialgroove of humerus is Radial nerve, if that‘s A) Absence of myenteric plexus in lower
damage extension of fingers & arm will be lost esophagus
along with loss of sensation of dorsum of arm & B) Loss of lower esophageal sphincter tone
hand. C) Increase Stomach peristalsis
D) Increase Intestinal motility
Ans: A
SK Origional – Golden 11 152

Explanation:Achalasia Cardia is one of the Explanation:


common cause of motor dysphagia that‘s caused  Mood: Serotonin
by loss of inhibitory ganglion in the myenteric  Emotion: Dopamine
plexus in lower esophagus. 53) Drug given for motion sickness:
49) Abdominal angina is commonly due A) Promethazine
to: B) Corticosteriods
A) Superior mesenteric artery C) PPI
B) Inferior mesenteric artery D) H2 Blocker
C) Aorta Ans: A
D) Common Iliac Artery Explanation:Scopolamine & Hyoscyamine were
Ans: A not in options.
Explanation:Abdominal Angina is defined asthe 54) A patient with striae, weight gain,
post prandial pain that occurs in individuals who acne, and excessive facial hair has
have mostly superior mesenteric artery occlusion. high ACTH.What is the cause?
50) A man with high BP, and decreased A) Primary increased ACTH production
heart rate, what is the cause: B) Exogenous steroids
A) Raised Intracranial Pressure C) Thyroid problem
B) Decreased Intracranial Pressure D) Decreased ACTH
C) Normal Intracranial Pressure Ans: A
D) None of the above 55) Poor prognostic factor in sepsis:
Ans: A A) DIC
Explanation: B) Vasodilatation
 Because the skull is rigid after infancy, C) Fever
intracranial masses or swelling may D) Inc. WBC count
increase intracranial pressure. When Ans: A
intracranial pressure is increased 56) Fisherman only eating fish and rice,
sufficiently, regardless of the cause developed anaemia. CBC result
cushing‘s reflex & other autonomic revealed low HB, MCV 106ft.Signs of
abnormalities can occur. neuropathy present. Stool examination
 Cushing‘s reflex includes systolic HT, shows ova. Cause:
increase pulse pressure & bradycardia. A) T solium
51) Subdural haemorrhage is caused by
B) Tapeworm
damage to: C) Echinococcus
A) Superior cerebral veins
D) Diphylobothriumlatum
B) Bridging veins
Ans: D
C) Diploic veins Explanation:As the worm absorbs approximately
D) Middle cerebral veins
80% of dietary vitamin B12 that‘s why prolong
Ans: B
D.Latum infection leads to vitamin B12 deficiency
Explanation:Subdural haemorrhage is caused by
bridging veins followed by superior cerebral veins. that‘s manifested with above findings.
52) Neurotransmitters for mood: 57) Taste sensation from posterior one
A) Dopamine third of tongue is carried via:
B) Serotonin A) Hypoglossal nerve (CN XII)
C) Acetycholine B) Glossopharyngeal nerve (CN IX)
D) Epinephrine C) Facial nerve (CN VII)
Ans: B D) Vagus nerve (CN X)
SK Origional – Golden 11 153

Ans: B 63) In ECG you will find 2 P waves


Explanation: followed by one QRS complex, where
 Taste sensation from Anterior 2/3rd is the pacemaker lies?
carried via facial nerve A) SA node
 Taste sensation from posterior 1/3rd is B) AV node
carried via Glossopharyngeal nerve. C) Purjinkee fibres
58) Following sudden relief in ARDS,
D) Bundle of His
patient develops:
Ans: A
A) Apnea
64) Patient diagnosed to have pale infarct.
B) Hypertension
Inwhich of the following conditions
C) Postural Hypotension
pale infarct occur:
D) GIT disturbances
A) Hypertensive stroke
Ans: A
B) Embolic/ischemic stroke
59) Autoradiography uses which
C) Atherosclerosis
technique?
A. Fluoresent Dye D) Rupture of Berry Aneurysm
B. Isotopes bound with Antibodies Ans: B
C. Radioactive ions Explanation:
D. Iron bound material  Pale (Ischemic) types of infarctions:
Ans: B  Increased density of tissue (Heart, kidney,
60) A 30 year old male died suddenly after spleen) prevents RBCs released from
a sudden rise in blood pressure . On damaged vessels from diffusing through
biopsy, on biopsy the ruptured vessel the necrotic tissue therefore the tissue has
a pale appearance.
lacked tunica media at the point of
65) Patient presented with Right sided
aneurysm. The likely cause of
weakness of upper and lower limbs
aneurysm is:
and left sided deviation of angle of
A) Mycotic Aneurysm
mouth. Where is the lesion?
B) Marfan Aneurysm A) Internal capsule
C) Berry Aneurysm
B) Mid brain
D) Dissecting Aneurysm C) Pons
Ans: C
D) Medulla
Explanation:The aneurysm that lacks the tunica Ans: A
media IS most likely berry aneurysm. Explanation:As this is uncrossed lesion that‘s
61) Vitamin D resistant Rickets causes: why itmost probably lies in internal capsule, in
A) Hyperparathyroidism crossed lesion then we will prefer pons?
B) Malabsorption syndrome 66) Scenerio of hemolytic anemia. Test to
C) Fanconi syndrome reach the diagnosis:
D) Megaloblastic Anemia A) Coombs Test
Ans: C B) Osmotic Fragility test
62) In Duodenal ulcer which artery is C) Electrophoresis
involved? D) HbA2
Ans: A
A) Gastroduodenal artery Explanation:The initial test for haemolytic
B) Splenic artery anemia &small RBCs is Coombs test followed by
C) Gastroepoploic artery osmotic fragility test to exclude hereditary
D) Left gastric artery spherocytosis.
Ans: A 67) A female presented with complaint of
itching and dryness of eyes, and
SK Origional – Golden 11 154

mouth, dysphagia, angular fibrosis, Ans: E


xray shows interstitial fibrosis. Cause: 73) A patient presented with rigors and
A) Sjogren syndrome chills. After taking treatment,
B) Progressive systemic sclerosis developed dark coloured urine. What
C) SLE is the reason?
D) RA A) G6PD
Ans: B B) Sickle cell disease
68) Serum marker for hepatocellular C) Thalassemia
carcinoma is: D) Malaria falciparum
A) CEA E) PNH
B) CA-125 Ans: A
C) AFP 74) Patient who is smoker and
D) CA-19 alcoholic.His lab report revealed
Ans: C increase bilirubinand increase alpha
Explanation: feto protein.Cause of raised alpha feto
 Hepatocellular Ca: AFP protein:
 Colorectal Ca: CEA A) Alcohol
 Ovarian Ca: CA-125 B) Smoking
 Pancreatic Ca: CA-19
C) Hep. C
69) Female presented with history of fall
D) Drug use
from height, developed urinary and
Ans: A
fecal incontinence. Lesion is at:
Explanation:AFP is raised in HCC & the
A) S1, S2, S3
commonest cause is Alcohol.
B) S2, S3, S4
75) Most common cause of fat necrosis:
C) Conus medullaris syndrome
A) Trauma
D) L3, L4
B) Enzymes
Ans: B
C) Infection
70) Jugulodigastric lymph node drain:
D) None of the above
A) Palatine tonsil
Ans: A
B) Pharyngeal tonsil
76) Vaccination for TB(BCG)type of
C) Tubal tonsil
immunity:
D) None of the above
A) CD4 cell hypersensitivity
Ans: A
B) CD8 cell mediated toxicity
71) Chromosome translocation 9:22occurs
C) Neutrophils
in:
D) Basophils
A) AML
Ans: A
B) CLL
Explanation: In BCG vaccination HSR CD4 &
C) CML
macrophages are found.
D) ALL
77) In bitemporal hemianopia, lesion is
Ans: C
seen at:
72) Nerve lateral to trachea:
A) Upper part of optic chiasma
A) Spinal Accessory Nerve
B) Middle part of optic chiasma
B) Reccurent Nerve
C) Lower part Of optic chiasma
C) PhrenicNerve
D) Optic Tract
D) Long ThoracicNerve
Ans: B
E) Vagus Nerve
SK Origional – Golden 11 155

78) Posterior 1/3rd of tongue senses C) Urea


which taste modality? D) Inulin
A) Bitter Ans: B
B) Sour 83) Injury to posterior head resulted in
C) Sweet vision loss. Artery involved:
D) Salty A) Posterior Cerebral Artery
E) Umami B) MCA
Ans: A C) ACA
79) Structures of femoral canal from D) Spinal Artery
medial to lateral: Ans: A
A) Vein, Artery, Nerve 84) Proprioception is carried by:
B) Nerve, Artey , Vein A) Dorsal column
C) Vein , Nerve , Artery B) Ventral column
D) Vein, Artery, Nerve, lymphatics C) Lateral column
Ans: A D) Spinothalamic Tract
Explanation:Remember Van (vein, artery & Ans: A
nerve). Explanation:
80) A 30-year-old patient presented with  Fine touch, proprioception & vibration is
3rddegree burn.He is most likely to carried away by DCML
develop:  Crude touch, temperature & pain is
A) Keloid carried away by Anterolateral tract
85) Anterior cerebral artery supplies:
B) Scar
A) Paracentral Lobule
C) Contracture
B) Frontal eye field
D) Wound dehiscence
C) Broca‘s area
Ans: C
D) Primary Auditory cortex
Explanation:
Ans: A
 In 3rd degree burns: contracture.
86) Structure forming the most important
 If Black skinned person mentioned then:
Keloid. part in anterior surface of heart:
81) AP Diameter of chest during quiet A) Right atrium
respiration is increased by B) Right ventricle
A) Diaphragm C) Left ventricle
B) External intercostal D) Left auricle
C) Internal intercoastal E) Aorta
D) Serratus Anterior Ans: B
Ans: B 87) Most potent antioxidant:
Explanation:The muscle used during quiet A) Vitamin E
respiration is Diaphragm, while if the AP diameter B) Vitamin A
isn‘t increased by diaphragm & is increased by C) Vitamin C
external intercostal muscle. D) Glutathione
82) Diffusion depends on size of particle, Ans: D
which one has high relative diffusion Explanation:Glutathione > Vitamin E > Vitamin
capacity: C > Vitamin A.
A) Water 88) DOC for T.solium Infection:
B) Glucose A) Praziquantel
B) Niclosamide
SK Origional – Golden 11 156

C) Metronidazole D) Posterior Mediastinum


D) Miconazole Ans: A
Ans: A Explanation:
Explanation:The drug of choice for T.Solium  Infection infront of pretracheal fascia
isPraziquantel followed by Niclosamide & spreads to: Anterior Mediastinum
Albendazole.  Infection posterior prevertebral fascia
89) Drug of choice in bacterial meningitis: spreads to: Superior Mediastinum
A) Ceftriaxone 94) Bicarbonates are maximally absorbed
B) Cefotaxime from:
C) Amoxcillin A) Loop of henle and collecting tubule
D) Ampicillin B) DCT and Collecting tubule
Ans: A C) PCT and collecting tubule
Explanation: D) PCT
 The drug of choice in bacterial meningitis Ans: C
in adults is: ceftriaxone. 95) Why infections are common in HIV
 The drug of choice in bacterial meningitis patients, postoperatively?
in children is: Cefotaxime A) Low CD8
 If Rash is found & N.Meningitides B) Low CD4
suspected then: Penicillin
90) A pateint labs revealed only Anti HBc C) Low B cell count
positive all other are negative, Most D) Neutrophenia
likely is: Ans: B
A) Past exposure period 96) In viral disease what will increase:
B) Acute resolving period A) Neutrophils
C) Window period B) Esinophils
D) Immunized C) Lymphocytes
Ans: A D) Macrophages
91) Most common complication after Ans: C
modified radical mastectomy is: Explanation:
A) Hematoma  In viral diseases Lymphocytes increases
 In parasitic infecion eosinophils increases
B) Anaphylaxis
 In bacterial infection neutrophils inreases
C) Lymph edema 97) Specific Gravity of CSF:
D) Wound dehiscence A. 1.005 – 1.009
Ans: C B. 1.004 – 1.008
92) Rh Blood group antigen present on: C. 1.009 – 2.009
A. Not present in tissues other than blood D. 1.003 – 1.007
B. Autosomal recessive Ans: A
C. Present on Hemoglobin 98) Condyloma latum is present in which
D. Present on RBC membrane stage of syphilis?
Ans: D A) Primary
Explanation:Rh blood group antigen is found on
B) Secondary
RBC surface > Glycoprotein > Immunogenic >
Dominant Mendalian Inheritance. C) Tertiary
93) Infection of neck in front of D) None of the above
pretracheal fascia will spread to: Ans: B
A) Anterior mediastinum Explanaiotn:Condyloma latum is a cutaneous
B) Superior mediastinum condition characterized by wart like lesions on the
C) Inferior Mediastinum genitals, in secondary stages of syphilis.
SK Origional – Golden 11 157

99) Scenerio of hutchinson teeth and 105) Complete ventricular depolarization is


gummas, cause: seen in:
A) Primary Syphilis A. QRS
B. ST Segment
B) Secondary Syphilis
C. PR Segment
C) Tertiary syphilis D. PR Interval
D) Congenital syphilis Ans: B
Ans: D 106) Which of the following must be given
100) About osteogenesis: in immediate warfarin toxicity
A) Newly laid uncalcified bone matrix is management?
called Osteoid A) FFP
B) Vitmain K
B) Whole cartilage is replaced by bone
C) Whole Blood
C) Articular surfaces covered by elastic D) Platelets
cartilage Ans: A
D) None of the above Explanation:
Ans: A  The Antidote for warfarin reversal is
101) Regarding histology of normal kidney: Vitamin K
A) Cuboidal epithelium in parietal layer of  But in immediate reversal we give FFPs +
bowman capsule Vitamin K > FFPs
B) Podocyte on visceral layer of bowman 107) Unable to produce words:
capsule A. Wernickes Aphasia
C) Dct longer then pct B. Brocas‘s Aphasia
D) Glomerulus has single arteriole C. Transcortical Aphasia
Ans: B D. None
Ans: B
102) A patient came to you with
Explanation:
primaryamenorrhoea,webbed  Excessive talking that makes no sense &
neck,short stature diagnosed as defective comprehensive ability in
Turner syndrome, what is her Wernickes Aphasia
karyotype:  Unable to produce words in Broca‘s
A) 45XO Aphasia
B) 46XY 108) Which of the following is more
appropriate regarding esophagus?
C) 47XXY
A. Upper 2/3rd Muscular Layer is Skeletal +
D) XX Smooth
Ans: A B. Lower Third has Simple Columnar
103) Karotype of Turner syndrome: Epithelium
A) 45XO C. Stratified Squamous Epithelium through
B) 46XY out
C) 47XXY D. Squamous Cell Carcinoma common in
lower 1/3rd
D) XX
Ans: C
Ans: A Explanation:
Explanation: This was asked twice.  Middle 1/3rd of Esophagus is mixed that‘s
104) Vasculitis features with purpura on both muscular &smooth muscle.
the buttocks in a child:  Esophagus is having stratified squamous
A. HSP epithelium throughout its whole length.
B. Arthritis  Adenocarcinoma of esophagus is
C. SLE common in lower 1/3rd of esophagus.
D. Sjogren Syndrome 109) Calculate CO when HR is 72 BPM &
Ans: A SV is 70 ml:
SK Origional – Golden 11 158

A) 5000 ml A. Nifedepine
B) 4000 ml B. Nimodipine
C) 3000 ml C. Captopril
D) 2000 ml D. Acetazolamide
Ans: A Ans: B
Explanation: 115) Circle of willis is formed by:
 CO = HR * SV A. One anterior cerebral arteries
 CO = 72 * 70 B. Three posterior cerebral arteries
 CO = 5040 ml C. Two Posterior Communicating arteries
110) Muscle relaxant that can be given to D. Two anterior communicating arteries
asthmatic patient: Ans: C
A. Cisatracurium Explanation:Circle of willis is formed by two
anterior cerebral arteries, two posterior cerebral
B. Atracurium
arteries, two posterior communicating arteries &
C. Succinylcholine one anterior communicating artery.
D. Suxamethonium 116) Nerve supply of anterior abdominal
Ans: A wall:
Explanation:According to applied therapeutics A. T7 – L1
there‘s no release of histamine by cisatracurium B. T6 – T9
that‘s why it‘s considered safe in Asthmatic C. T4 – T5
Patients? D. T5 – T6
111) MOA of cabergoline: Ans: A
A. D1 Agonist Explanation:Lower six intercostal nerves & first
B. D2 Agonist lumber nerve supply anterior abdominal wall.
C. D1 Antagonist 117) Initial, most common side effect of
D. D2 Antagonist Lidocaine:
Ans: B A. Treated with hydrocortisone
112) Post-operative patient taking B. Given as 2% solution to avoid adverse
gentamicin having problem in which effect
of the following? C. Causes convulsions before cardiac toxicity
A. Excretion for several hours
B. Distribution D. Perioral parasthesia
C. Absorption Ans: D
D. Potency Explanation:
Ans: A  Initial : Perioral Parasthesia
Explanation:Gentamicin is eliminated through  Later on: Convulsion
renal system and in post-operative patient there‘s 118) Maximum lymphatic drainage of
chances of dehydration that‘s why it will lead to breast is parallel with which of the
difficulty with excretion of the drug? following artery?
113) Patient having diabetic retinopathy & A. Axillary Artery
IHD developed chest pain due to: B. Inernal carotid artery
A. Thrombosis C. Internal thoracic artery
B. Embolism D. Subclavian artery
C. Coronary artery disease Ans: A
D. DVT
Explanation:Maximum of breast is drained via
Ans: C
Explanation:In DVT & prolong immobilization Anterior (Pectoral) Axillary lymph nodes & that‘s
history the patient presents with chest pain & that parallel with Axillary artery.
most likely indicates CAD which is due to 119) Submucosal Fibrosis (SMF) is:
Thrombosis A. Premalignant
114) Drug for SAH (Subarachnoid B. Benign
hemorrhage): C. Malignant
SK Origional – Golden 11 159

D. Dysplasia D) Fat Necrosis


Ans: A Ans: B
Explanation:Submucosal fibrosis is a 5) Which of the following area is
premalignant condition of oral cavity. involved in motor aphasia?
120) 10th Intercostal Nerve pierces Rectus A) Brocas
Sheath at which Level: B) Wernickes
A. Xiphisternum C) Arcuate
B. Costal Groove D) Occipital lobe
C. Umbilicus Ans: A
D. Pyramidalis Explanation:
Ans: C  Motor Aphasia: Broacas Area
Explanation:T10 nerve pierces rectus  Sensory Aphasia: Wernicke‘s Area
sheathumbilical level that‘s initial site for  Anomic Aphasia: Arcuate fasciculus
appendicitis pain. 6) Which of the following has negative
inotrophic effect?
A) Acetycholine
=============== B) Epinephrine
C) Adrenaline
MEDICINE & ALLIED D) Glucagon
Ans: A
October 1st, 2020 - Night Explanation:Acetycholine has negative
Inotrophic effect while Adrenaline & glucagon
1) Esophageal opening is located at have positive inotrophic effect.
which of the following: 7) Hearing improves in noisy
A) T8 environment in which of the following
B) T10 condition?
C) T12 A) Meniere‘s Disease
D) L1 B) Otosclerosis
Ans: B C) Tympanic membrane damage
Explanation: D) Ossicular damage
 Inferior Vena Caval Opening: T8 Ans: B
 Esophageal Opening: T10 8) Which of the following is correct
 Aorta Opening: T12 regarding plasma protein binding?
2) Which of the following is root value of A) Acidic drugs binds with Acid glycoprotein
phrenic nerve? B) Acidic drugs bind with Albumin
A) C3,4,5 C) Basic drugs bind with Albumin
B) C2,3,4 D) Basic drugs bind with globulin
C) C3,4 Ans: B
D) C1,2 Explanation:
Ans: A  Acidic drugs bind with Albumin
3) Which of the following necrosis occurs  Basic drugs bind with Acid glycoprotein
in kidney? 9) Conductive system of heart lies in:
A) Liquefective Necrosis A) Subendocardium
B) Coagulative Necrosis B) Subepicardium
C) Caseous Necrosis C) Endocardium
D) Fat Necrosis D) Epicardium
Ans: B Ans: A
4) Which of the following necrosis occurs Explanation:
in heart?  Conductive system of heart lies in
A) Liquefective Necrosis Subendocaridum
B) Coagulative Necrosis  SA node lies in subepicardium
C) Caseous Necrosis
SK Origional – Golden 11 160

 AV node lies in interatrial septum Ans: B


endocardium 16) Case of malaria with complication,
10) Renin is secreted by: black discoloration of urine, organism
A) Peritubular Capillaries involved:
B) JG Cells A) Plasmodium Ovale
C) PCT B) Kala azar
D) LOH C) Plasmodium Falciparum
Ans: B D) Malaria
Explanation: Ans: C
 Renin is secreted by JG cells 17) Heparin is secreted from which of the
 Erythropoetin is secreted by peritubular following cells?
capillaries A) Eosinophils
11) Halo seen around the organism B) Mast Cells
involving the brain tissue: C) Neutrophils
A) Cryptococcus Neoformans D) Macrophages
B) Candida Albicans Ans: B
C) Staph Aureus Explanation:Heparin is secreted by basophils &
D) Chlamydia mast cells.
Ans: A 18) Neural crest cells migrates to hind
brain to form:
12) Cholangiocarcinoma is caused by A) Papillary muscle
which of the following? B) Foramen Ovale
A) Clonorchis Sinensis C) Right Atrium
B) E. Coli D) Endocardial Cushion
C) Hematobium Ans: D
D) HIV 19) Case of liver cirrhosis, Portal HTN,
Ans: A active variceal bleeding, which of the
Explanation:Clonorchis Sinensis is a following vessel is involved?
carcinogenic human liver fluke that can lead to A) Left Gastric Artery
periductal fibrosis and even cholangiocarcinoma. B) Left Gastric Vein
13) Bradykinin involves which of the C) Right Gastric Vein
following system? D) Splenic Vein
A) Kallikrin System Ans: B
B) Coagulation pathway 20) Which of the following
C) Fibrinolytic System benzodiazapine has shortest half life?
D) None of the above A) Midazolam
Ans: A B) Diazepam
14) Cochlear action potential is generated C) Temazepam
by: D) Clonazepam
A) K+ efflux from the cells Ans: A
B) K+ Influx into cells Explanation:Alprazolam, Midazolam and
C) Na+ efflux from the cells triazolam have half life less than 5 hours.
D) Na+ influx into cells 21) Cimetidine interaction with warfarin:
Ans: B A) Increase hepatic clearance of warfarin
15) A 14 years old boy gets his finger cut B) Decrease hepatic clearance of warfarin
by a knife. Which one of the following C) Increase half life of warfarin
is primary/early mediator of acute D) Decrease half life of warfarin
inflammatory response? Ans: B
A) IL1 Explanation:Cimetidine is potent CYP3A4
B) Histamine enzyme inhibitor & thus decreases the hepatic
C) Bradykinin clearance of warfarin.
D) Platelet activating factor
SK Origional – Golden 11 161

22) Which of the following is related to 29) In pregnancy Ovulation is inhibited


anterior surface of SCM? by which of the following:
A) Internal Jugular Vein A) Increase Prolactin
B) External Jugular Vein B) Decrease Prolactin
C) Brachiocephalic Vein C) Increase GnRH
D) SVC D) Decrease GnRH
Ans: B Ans: A
23) A case of UTI, which of the following 30) Pericytes are present in:
organism is commonly involved? A) Arteries
A) E. coli B) Veins
B) Staph Aureus C) Capillaries
C) Streptococcus D) Aorta
D) Chlamydia Ans: C
Ans: A Explanation:Pericytes are cells present at
24) Psychiatric patient, information intervals along the wall of capillaries and post
should be: capillary venules.
A) Kept Confidential 31) Which of the following is the main
B) Shared with Family factor for metastasis?
C) Shared with Police A) Overexpression of E-Cadherins
D) Shared with Staff B) Loss of E-Cadherins
Ans: A C) Damage to membrane
25) In a cardiac patient which of the D) Damage to Phospholipid bilayer
following is the cause of edema? Ans: B
A) Increase Plasma Oncotic pressure 32) Mandibular bridge is:
B) Decrease Hydrostatic pressure A) Angle of Mandible
C) Increase Hydrostatic pressure B) Root of Mandible
D) Decrease vascular permeability C) Condyle of mandible
Ans: C D) Coronoid process of mandible
Explanation:Edema is caused by increased Ans: A
hydrostatic pressure, decrease plasma oncotic 33) Which of the following tumor marker
pressure & increase vascular permeability. is raised in neural tube defect?
26) Hemoglobin binds to which of the A) AFP
following? B) LDH
A) Hemopexin C) CEA
B) Haptoglobin D) CA-125
C) Albumin Ans: A
D) Globulin Explanation:AFP is raised in neural tube defect
Ans: B & helps in diagnosis while the confirmatory test is
Explanation:Heme bind with Hemopexin Increase AChE in amniotic fluid.
27) Cryptorchidism increases the risk of: 34) Grading of Tumour:
A) Malignancy A) Extent of Spread
B) Infection B) Degree of Differentiation
C) UTI C) Both
D) Renal problem D) None
Ans: A Ans: B
28) Hydrogen Per oxide & Oxidases are Explanation:
found in:  Grading of tumor: Degree of
A. Peroxisome differentiation
B. Lysosome  Staging of tumor: Extent of tumor
C. Nucleus 35) Renal plasma flow is measured by:
D. Ribosome A) PAH
Ans: A B) Creatinine
SK Origional – Golden 11 162

C) Inulin C) Below L5
D) BUN D) Above L5
Ans: A Ans: A
Explanation: Explanation: Between L4-5 and Below L4 and
 Renal plasma flow is measured by : PAH above L5
 GFR is Estimated by: Creatinine 42) A 20 years old boy with rheumatic
clearance valvular heart disease has been
 GFR is Measured by: Inulin running a low grade fever for the last
36) In IDA which of the following is two weeks. In addition to a
investigation of choice: pansysotlic murmur, he is found to
A) Serum Iron have splenomegaly and clubbing, the
B) Serum Ferritin Level treatment should be:
C) HbA2 A) Inj. Cefotaxime + Metronidazole
D) MCV B) Inj. Ceftriaxone
Ans: B C) Inj. Penicillin
37) Propanolol acts on which of the D) Inj. Penicillin + Gentamycin
following receptors? E) Inj. Ciprofloxacin
A) Beta 1 Ans: D
B) Beta 2 43) Dermatitis & diarrhea is caused by
C) Both Beta 1 & Beta 2 deficiency of which of the following
D) Alpha 2 vitamin?
Ans: C A) Thiamine
38) Protein diet, which of the following B) Riboflavin
vitamin must be taken along with it? C) Niacin
A) Thiamine D) Folate
B) Riboflavin Ans: C
C) Biotin Explanation:Niacin (B3) deficiency leads to
D) Niacin dementia, dermatitis & diarrhea.
Ans: B 44) Mesothelioma is caused by which of
Explanation: the following?
 Along with protein diet: Riboflavin (B2) A) Asbestosis
 Along with Carbohydrate diet: Thiamine B) Coal Worker
(B1) C) Silicosis
 Along with Fatty diet: Biotin (B7) D) Berylosis
39) Barret esophagus increases the risk of Ans: A
which of the following? Explanation:Asbestosis causes bronchogenic
A) Squamous cell carcinoma carcinoma > Mesothelioma.
B) Adenocarcinoma esophagus 45) Which of the following is the karotype
C) Gastric carcinoma of turner syndrome?
D) PUD A) 45 XO
Ans: B B) 46 XX
40) Which of the following murmur is C) 46 XY
heard in rheumatic fever? D) XXY
A) Pansystolic Ans: A
B) Early diastolic 46) Regarding cartilage:
C) Early systolic A. Epiglottis is hyaline
D) Systolic clicks B. Synovial Joint has fibrous cartilage
Ans: A C. Appositional growth
41) Which of the following is best site for D. Transitional growth
LP? Ans: C
A) Below L4 47) Webbed neck, short stature, primary
B) Above L4 amenorrhea, diagnosis:
SK Origional – Golden 11 163

A) Patau syndrome B) Ovarian Artery


B) Edward syndrome C) Cervix
C) Turner syndrome D) Fallopian tube
D) Down syndrome Ans: A
Ans: C 55) Anti DsDNA positive along with
48) Ectopic pregnancy diagnosis is done hematuria & foamy urine is most
by which of the following? likely:
A) Transabdominal USG A) UTI
B) TVS (TransVaginal USG) B) Bladder Carcinoma
C) CT Scan Abdomen C) Lupus Nephritis
D) X-Ray Abdomen D) Acute Pyelonepritis
Ans: B Ans: C
49) After injury which of the following is Explanation:Lupus Nephritis is inflammation of
first response? the kidney that is caused by SLE & presents with
A) Vasodilation hematuria & foamy urine.
B) Vasoconstriction 56) Which of the following causes
C) Increase Permeability pseudomembranous colitis?
D) Decrease Permeability A) E. coli
Ans: B B) Streptococcus
50) Posterior dislocation of elbow, which C) Giardia
of the following ligament is damaged? D) C.Difficile
A) Ulnar Collateral ligament Ans: D
B) Radial ligament 57) Tongue ulcer, CXR shows B/L hilar
C) Transverse Ligament lymphadenopathy, caseous necrosis,
D) Lateral ligament cause is:
Ans: A A) TB
51) Aplastic anemia diagosis is done by B) Histoplasmosis
which of the following? C) Wegener granulomatosis
A) Bone marrow Biopsy D) Sarcoidosis
B) Blood Smear Ans: A
C) CBC Explanation:Caseous necrosis is mainly seen in
D) USG Tb & other points are in favor of Tb too like hilar
Ans: A lymphadenopathy & mouth ulcer can also be seen
52) Which of the following is the karotype in Tb.
of true hermaphrodite? 58) A patient is already on Warfarin, PT
A) XX measures which of the following
B) XY factors?
C) XXY A) 2, 5, 7, 10
D) XYY B) 8, 9, 10, 11, 12
Ans: A C) 1, 3, 5
Explanation:XX > XX/XY > XXY D) 2, 9,11
53) Smoker with hydrocarbon exposure Ans: A
presents with hematuria and weight Explanation:
loss is likely suffering from:  PT measures factor 2, 5, 7 & 10
A) Bronchogenic carcinoma  APTT measures factor 8, 9, 10, 11, 12
B) Bladder Carcinoma 59) Kartagener‟s Syndrome involves:
C) UTI A) Collagen
D) Acute pyelonephritis B) Fibrillin
Ans: B C) Dynein Arms
54) Uterine artery crosses anterior to D) Centrioles
which of the following structure? Ans: C
A) Ureter 60) Renin is released by:
SK Origional – Golden 11 164

A) Sympathetic response Explanation:In diabetic gastroparesis the drug of


B) Parasympathetic response choice is prokinetics: Metoclopromide.
C) Increase Sodium 66) Sensory supply of Temporal/parietal
D) AG2 area of scalp:
Ans: A A) Supraorbital
61) Red blood cells Cytoskeleton is B) Supratrochlear
associated with which of the following C) Auriculotemporal
protein: D) Greater Auricular
A) Actin Ans: C
B) Myosin 67) Penetration of Corona radiata is done
C) Spectrin by which of the following component
D) Protein 3 of Sperm:
Ans: C A) Hyaluronidase
Explanation:The red cell membrane skeleton is a B) Fibroelastase
pseduohexagonal meshwork of spectrin, actin, C) Nucleases
protein 4.1R, Ankyrin and actin associated D) Lysosome
proteins that laminates the inner membrane Ans: A
surface and attaches to the overlying lipid bilayer Explanation:Hyaluronidase is secreted by sperm
via band 3 containing mutliprotein complexes at acrosome and penetrates the corona radiate.
the ankyrin and actin binding ends of spectrin. 68) Complement activation is mainly done
62) Which of the following is the cheapest by:
and cost effective diagnosis of cervical A) IgM
cancer screening? B) IgE
A) Colposcopy C) IgD
B) Cone biopsy D) IgA
C) Cervical Smear Ans: A
D) PAP Smear 69) Which of the following antibody is
Ans: D present in mother‟s milk?
63) Hyperthyroidism in pregnancy, which A) IgA
of the following is the drug of choice? B) IgE
A) Methimazole C) IgD
B) Thyroxine D) IgG
C) Iodide Ans: A
D) Corticosteroids Explanation:
Ans: A  Mother‘s milk contains IgA
64) Which of the following is the common  IgG can cross the placenta
complication of Ultraviolet light for 70) A Patient with HTN, died and on
eye? autopsy an enlarged heart was found,
A) Glaucoma cause is:
B) Hemorrhage A) Hypertrophy
C) Cotton Wool spots B) Hyperplasia
D) Cataract C) Hypertrophy + Hyperplasia
Ans: D D) Atrophy
65) A known diabetic presented with Ans: A
gastroparesis which of the following is Explanation:Cardiac Cells are non dividing cells
the drug of choice: so only hypertrophy will occur in these cells after
A) Prokinetics hypertension work overload.
B) PPI 71) Stroke patient with irregular
C) Antacids breathing:
D) H2 Blocker A) Kussmual Breathing
Ans: A B) Periodic Breathing
C) Normal Breathing
SK Origional – Golden 11 165

D) Cheyne stokes breathing A) Atrial Systole


Ans: D B) Ventricular Relaxation
72) Micro-organism resistant to C) Ventricular Contraction
antibiotics and disinfectant: D) Atrial relaxation
A) E. coli Ans: A
B) Streptococcus Explanation:P wave is related with atrial systole
C) Candida and in absence of P wave the atrial contraction
D) Pseudomonas will be affected.
Ans: D 78) Saw tooth pattern on ECG is seen in:
73) In appendicitis pain radiates to which A) Atrial fibrillation
of the following area? B) Atrial Flutter
A) Umbilicus C) Ventricular Fibrillation
B) Pelvic region D) Ventricular Tachycardia
C) Epigastric Ans: B
D) Right Shoulder 79) Which of the following is the relation
Ans: A of left renal vein?
Explanation:Due to dermatomal supply pain of A) Anterior to IVC
appendix is referred to umbilical area. B) Anterior to Aorta
74) Which of the following is the largest C) Posterior to IVC
cell in the blood? D) Posterior to Aorta
A) Megakarocyte Ans: B
B) Monocyte Explanation:
C) Basophil  Left Renal vein is anterior to aorta
D) Mast Cell  Right Testicular artery is anterior to IVC
Ans: B 80) Which of the following causes
Explanation: vasoconstriction?
 Largest Cell in the blood is: Monocyte A) Adrenaline
 Largest Cell in the bone marrow is: B) ADH
Megakarocyte C) AG2
75) A mother taking which of the D) AG1
following anti hypertensive drug leads Ans: B
to renal agenesis: Explanation:ADH (Vasopressin) is the most
A) Metoprolol potent vasoconstrictor among the mentioned
B) Captopril options.
C) Furosemide 81) Which of the following is correct
D) Hydrochlorothiazide regarding Mobitz type 1?
Ans: B A) Progressive PR interval without loss of
76) Anomic aphasia is caused due to beat
lesion of: B) Progressive PR interval with more than 4
A) Dominant Brocas area beat loss after each P wave
B) Wernickes area C) Progressive PR interval with loss of beat
C) Non dominant hemisphere brocas area D) Static PR interval with loss of beat
D) Occipital region Ans: C
Ans: C Explanation:
Explanation:  First Degree heart block is characterized
 Lesion of non dominant hemisphere by prolongation of PR interval
brocas area leads to Anomic Aphasia  Second degree heart block has two
 Motor Aphasia: Brocas area types:Mobitz type 1: Characterized by
 Sensory Aphasia: Wernkicke‘s Area prolongation of PR interval followed by a
77) A patient had arrhythmia and P waves beat dropMobitz type 2: characterized by
were absent, which part of the cardiac Constant PR interval with beats drops
cycle is affected:
SK Origional – Golden 11 166

 Third degree heart block or complete C) Terlipressin


heart block characterized by no D) Octerotide
association of SA node with AV node. Ans: B
82) Paramesonephric duct gives raise to 88) A wound healing that goes beyond its
which of the following? margins is most likely
A) Vas deferens A) Keloid
B) Uterus + Fallopian tube B) Hypertrophic Scar
C) Whole Vagina C) Contracture
D) Only Uterus D) Hyperplastic
Ans: B Ans: A
Explanation:Paramesonephric duct gives raise to Explanation:A keloid scar crosses its margins &
fallopain tube, uterus & upper part of vagina. goes out of its original area.
83) A case of meningitis, decrease 89) Head trauma breathing pattern:
glucose, Increase Protein and A. Kussmaul Breathing
lymphocytes 88%: B. Cheyne-Stokes Breathing
A) Bacterial C. Biot‘s Respiration
B) Viral D. Periodic Breathing
C) TB Ans: B
D) HSV Explanation:
Ans: C  Cheyne-Stokes breathing is commonly
Explanation:Increase protein, decrease glucose & seen after stroke & head trauma
lymphocytosis points toward tb meningitis.  Kussmaul Breathing pattern is seen DKA
84) Patient undergoes renal transplant, & Uremia
now on immunosuppressive therapy, 90) Right to left shift of oxygen
developed angular cheilitis, which of dissociation curve in which of the
the following is the causative agent? following?
A) CMV A) Increase Temperature
B) EBV B) CO poisoning
C) Herpes C) Increase H+
D) Candida D) Decrease PH
E) Aspergillus Ans: B
Ans: A Explanation:
85) Drug of choice for T.Solium:  Left to right shift is caused by
A. Albendazole hyperthermia, Increase H+ , Decrease PH
B. Prizaquental  Right to left shift is caused by
C. Metronidazole Hypothermia, Decrease H+, Increase PH,
D. Vancomycin fetal Hb & CO poisoning
Ans: B
Explanation:DOC for T.Solium is Prizaquental, 91) DLCO decreases in:
but when not in option then prefers Niclosamide A) Pulmonary Fibrosis
followed by Albendazole. B) Polycythemia
86) Which of the following is the drug of C) Pulmonary hemorrhage
choice in SAH? D) Asthma
A) Manitol Ans: A
B) Nimodipine 92) Which of the following muscle flexes
C) Captopril the hip joint?
D) Metoprolol A) Gluteus Maximus
Ans: B B) Adductor magnus
87) Which of the following is the drug of C) Semimembraneous
choice in atonic uterus after PPH? D) Psoas
A) Ergometrine Ans: D
B) Oxytocin
SK Origional – Golden 11 167

93) Anti Depressant effect of TCA takes the hospital in semiconscious state is
how much long: most likely due to:
A) 1 Week A) Heat Stroke
B) 1 – 2 Weeks B) Heat Exhaustion
C) 3 – 4 Weeks C) Excessive Sweating
D) 2 Months D) Normal Phenomenon
Ans: C Ans: B
Explanation:Analgesic effect of TCA takes 1-2 Explanation:
Weeks while antidepressant effect of TCA takes 3-  Heat exhaustion is characterized by
4 weeks. excessive sweating and cold, clammy skin
94) Percentage of Fats in adult male:  Heat Stroke is characterized by
A) 10% Temperature above 100 & absence of
B) 15% sweating with loss of consciousness
C) 30% 100) Which of the following is most
D) 40% powerful stimulus for gastric
Ans: B secretion?
Explanation: A) Protein
 Percentage of Fat in male: 15% B) Carbohydrates
 Percentage of fat in female: 30 – 35% C) Lipds
95) Ataxic gait & Ipsilateral face sensation D) Acids
lost where is the site of lesion: Ans: A
A) Cerebro Pontine Angle Explanation:According to BRS physiology the
B) Cerebellopontine Angle most potent stimulus for gastrin secretion is
C) Midbrain phenyalanine & tryptophan which are amino acids
D) Pons and peptides (Protein).
Ans: B 101) Erythropoietin is secreted by which
96) Cellular vault is maintained by: organ?
A) Apoptosis A. Lung
B) Necrosis B. Intestine
C) Lysosome C. Liver
D) Enzymatic Destruction D. Kidney
Ans: A Ans: D
97) Gastrocnemius size is reduced after Explanation:
cast applied for few weeks of fracture  90% of Erythropoietin is secreted via
is most likely due to: Kidney & only limited via liver.
A) Decrease Blood supply 102) Urine examination of a patient with
B) Decrease muscle caliber diabetes demonstrates ketone bodies,
C) Denervation atrophy what is the mechanism of formation of
D) Decrease Actin & Myosin these ketone bodies?
Ans: D A) Insulin deficiency
98) Eight years old girl with well B) Hyperglycemia
developed breast and pubic hair is due C) Defective fat metabolism
to: D) Hyopalbuminemia
A) Granulosa cell tumor E) Defective glucose metabolism
B) CAH Ans: A
C) Theca cell tumor 103) Which of the following is renal
D) Normal compensation for hyperkalemia?
Ans: A A) Acidosis
99) A patient having temperature of 99F, B) Alkalosis
cold clammy skin, sweating profusely, C) Increase Glutamine secretion
and working on a hot day, brought to D) Decrease Glutamine secretion
Ans: A
SK Origional – Golden 11 168

104) Which of the following B. Streptomycin


volume/Capacity is left in the lungs C. Ethambutol
after forceful expiration? D. Pyrazinamide
A) FRC Ans: D
B) Residual volume Explanation:Among the mentioned ATT drugs
C) Inspiratory reserve volume pyrazinamide is associated with gout.
D) VC 111) 50 years old hypertensive patient
Ans: B presented with severe pain radiating
Explanation:After forceful expiration the volume towards back suddenly collapsed. On
of Air remained in the lungs is Residual volume Autopsy what could be seen:
(RV) that‘s 1200 ml. A) Ischemic Necrosis
105) Major blood vessels are found in B) Medial Necrosis
which of the following space? C) Atherosclerosis
A) Epidural space D) Arterioscleorsis
B) Extradural space Ans: B
C) Subarachnoid Space Explanation:Aortic dissection commonly results
D) None of the above from hypertension; chronic hypertension causes
Ans: C hyaline arteriosclerosis of vasa vasorum that
106) Odontoid process is present in which supply media and leads to necrosis of media and
of the following vertebrae? weakness in vessel wall leading to tear &
A) 1st Cervical Vertebrae dissection.
B) 2nd Cervical Vertebrae 112) Osmosis depends on:
C) 3rd Cervical Vertebrae A) Number of solute particles
D) 7th Cervical Vertebrae B) Osmoreceptors
Ans: B C) Concentration gradient
107) Digitalis toxicity is increased by which D) Electrical gradient
of the following? Ans: A
A) Hydrochlorothiazide 113) Pseudostratified columnar epithelium
B) Propanolol is present in which of the following?
C) Aspirin A) Lungs
D) Heparin B) Trachea
Ans: A C) Palatine Tonsils
Explanation:Hydrochlorothiazide leads to D) Uterus
hypokalemia & hypercalcemia both of them Ans: B
worsens digitalis toxicity. 114) Baroreceptor responds to which of the
108) Which of the following is the nerve following?
supply of suprarenal gland? A) Rapidly increase blood pressure
A) Lesser splanchnic nerve B) Rapidly decrease blood pressure
B) Greater splanchnic nerve C) Increase in PO2
C) Obturator nerve D) Decrease in PO2
D) Renal Nerve Ans: B
Ans: B 115) Stony dull percussion note in found in
109) Which of the following is present in which of the following?
hypertrophic cardiomyopathy? A) Pneumoniae
A) Myoctyes necrosis B) Pleural effusion
B) Myoctyes atrophy C) Normal Lungs
C) Myoctyes Hyperplasia D) Pnuemothorax
D) Myocytes disarray Ans: B
Ans: D 116) Hemorrhage leads to decrease arterial
110) Patient on ATT with big toe swelling pressure that in turn leads to:
most likely due to: A) Increase Venous capacitance
A. Rifampicin B) Decrease Venous Capacitance
SK Origional – Golden 11 169

C) Decrease TPR B) Fat embolism


D) Decrease CO C) Gas embolism
Ans: B D) Cardiac arrest
117) Growth hormone increases in which of Ans: B
the following condition? 123) Which of the following increases in
A) Exercise pregnancy?
B) REM A) Serum transferring
C) NREM B) Red cell mass
D) Stress C) MCH
Ans: A D) Iron absorption from gut
118) Axillary nerve supplies which of the Ans: B
following? 124) Which of the following drug decreases
A) Deltoid TGA by decreasing FFA?
B) Serratus Anterior A) Niacin
C) Pectoralis Major B) Statin
D) Bicep Brachii C) Gemfibrozil
Ans: A D) Fenofibrates
Explanation:Axillary nerve supply teres minor & Ans: C
deltoid muscles. Explanation:According to Chaterjii: Fibrates
119) A boy with tender knee & bleeding such as gemfiborozil are derivates of fibric acid,
history which of the following test will they are used to reduce TGA level in the blood.
confirm the diagnosis? 125) Rickets is caused by deficiency of
A) Aptt which of the following vitamin?
B) PT A) Vitamin A
C) Factor 8 Assays B) Vitamin B
D) Platelet count C) Vitamin C
Ans: C D) Vitamin D
Explanation:This is case of hemophilia & the Ans: D
confirmatory/diagnostic test is Factor 8 Assay 126) Which of the following is given in
while initial test is APTT. anaphylactic & hypersensitivity
120) A patient with IDA which of the reaction?
following will confirm the diagnosis: A) Adrenaline
A) Serum Iron B) NE
B) HbA2 C) Anti Histamine
C) Serum Ferritin D) Dopamine
D) Iron Saturation Ans: A
Ans: C 127) Which of the following is
121) Test to exclude non affected premalignant lesion of Vulva?
completely: A) Paget disease
A) Sensitivity B) Lichen scleorosis
B) Specificity C) Lichen planus
C) Predictive Value D) Krasu Vulvae
D) None of the above Ans: A
Ans: B Explanation:Paget Disease > Krause Vulvae >
Explanation: Lichen Sclerosis.
 Rule out disease: Sensitivity 128) Which of the following antiemetic is
 Rule out non disease: Specificity given before surgery?
122) A patient having history of RTA A) Metocloperamide
presented after 3 days with respiratory B) Odensetrone
distress which of the following is the C) Pyrodixine
possible reason? D) Meclizine
A) Air embolism Ans: A
SK Origional – Golden 11 170

Explanation: D) Glossopharyngeal
 Metocloperamide is preffered in pre OP Ans: B
cases while Odenstrone is preferred in 135) Vibration is carried by which of the
post OP, pre chemo and radiation following tract?
induced vomiting cases A) Anterolateral system
129) Nucleus of tractus solitarius contains: B) DCML
A. 1st Order Neuron C) Spinocerebellum
B. 2nd Order Neuron D) Spinotectum
C. 3rd Order Neuron Ans: B
D. None Explanation:
Ans: B  Fine touch, vibration & proprioception is
130) Investigation for Hashimotos carried out by DCML.
thyroiditis:  Crude touch, pain & temperature is
A) Anti thyroglobulin & anti TPO carried out by anterolateral system.
B) Anti T3 136) Which of the following is cause of
C) TFTs global blindness?
D) Anti Thyroglobin A) Bacterial infection
Ans: A B) Viral infection
Explanation:Anti microsomal and anti C) Chlamydial infection
thyroglobulin antibodies destroy parenchyma and D) Measles
are present in hashimotos. Ans: C
131) Asthmatic Patient having respiratory 137) Paroxysmal hypertension & episodic
rate of 30/min and dyspneic, findings: headache is seen in which of the
A) Increase Lung compliance following condition?
B) Decrease Lung compliance A) Pheochromocytoma
C) No change B) Adrenal tumor
D) None of the above C) Myxoma
Ans: A D) Renal Cell carcinoma
132) Phrenic nerve avulsion at its origin Ans: A
will lead to: 138) An Old diabetic patient presented
A) Loss of sensation of central tendon of with gastroparesis which of the
Diaphragm following drug would be helpful?
B) Hemi diaphragm paralysis A) Prokinetics
C) Loss of respiratory reflex B) PPI
D) Loss of respiration C) H2 Blockers
Ans: B D) Antacids
Explanation:Phrenic nerve avulsed at its origin Ans: A
will lead to:Hemi diaphragm sensory loss > Hemi Explanation:In diabetic gastroparesis the drug of
diaphragm paralysis > Central tendon sensation choice is prokinetics i.e metoclopramide.
lost. 139) Anti H antibodies are present in
133) Intervertebral discs have which of the which blood group?
following cartilage? A) A +ve
A) Fibrocartilage B) B +Ve
B) Hyaline C) Oh
C) Elastic D) AB +Ve
D) Articular Ans: C
Ans: A 140) After a marathon race which of the
134) SCM muscle is supplied by which of following is found:
the following nerve? A) Increase Insulin
A) Vagus B) Decrease glucagon
B) Accessory C) Decrease Insulin & Increase Glucagon
C) Hypoglossal D) Decrease Glucagon & Insulin
SK Origional – Golden 11 171

Ans: C 147) A 12 years old boy presented with


Explanation:In Marathon race the racer needs diarrhea not responding to routine
glucose that‘s why insulin will be decreased & anti diarrheal treatment. Jejunal
glucagon will increase? biopsy showed partial villous atrophy
141) Cola color urine is seen in which of with mansy PAS positive
the following: macrophages. Most likely diagnosis is:
A) P.Vivax A) Carcinoid syndrome
C) P.Malaria B) Topical sprue
D) P.Faciparum C) Giardiasis
Ans: D D) Immunoproliferative disease
142) Pregnant lady in 1st trimester having E) Whipple disease
Hb of 10 gm/dl, on 3rd trimester Ans: E
having Hb of 9 gm/dl most likely due Explanation:
to:  Improved with gluten free diet: Celiac
A) IDA disease
B) Thalassemia  Not responding to Gluten free diet:
C) Physiological change Giardiasis
D) Sideroblastic Anemia  PAS positive: Whipples
Ans: C 148) Accommodation reflex intact but light
143) Which of the following is Post reflex is lost due to lesion in:
sympathetic cholinergic? A) Cerebral cortex
A) Sweat Glands B) Midbrain
B) Adrenal Glands C) Pretectum
C) Pituitary Gland D) Pons
D) Muscles Ans: C
Ans: A Explanation:
144) A patient after RTA presented with  If accommodation or accommodation
low Blood pressure due to: reflex lost without 3rd cranial nerve
A) Hypovolemic shock involvement: Cerebral cortex
B) Septic shock  With 3rd Cranial nerve involvement:
C) Cardiogenic shock Midbrain
D) Cardiac Tamponade  Accommodation reflex intact but light
Ans: A reflex lost in lesion of: Pretectum
145) Pregnant lady with delayed reach to 149) Reticular fibers are present in:
hospital presented with dyspnea most A) Dermis
likely due to: B) Epidermis
A) Fat Embolism C) Tonsils
B) Air Embolism D) Salivary Glands
C) Amniotic Fluid Embolism Ans: C
D) Hemorrhage Explanation:Tonsils > Dermis
Ans: C 150) CSF is absorbed by:
Explanation:Pregnant lady with dyspnea without A) Arachnoid granulation
any other factors like trauma is most likely B) Pial Granulation
amniotic fluid embolism. C) Oligodendrocytes
146) Which of the following is most D) Sinuses
common findings in Autoimmune Ans: A
conditions? Explanation:CSF is absorbed by arachnoid
A) RA granulation and then drains into dural venous
B) Hematological sinus.
C) Malignancy 151) S2 differs from S1 by:
D) Skin Rash A) S2 has lower Frequency
Ans: B B) S2 has higher frequency
SK Origional – Golden 11 172

C) S1 is high pitched Explanation:


D) S2 is low pitched  A patient with lesion of right optic tract
Ans: B will develop Left Homonymous
Explanation: Hemianopia
 S1: low pitched, low frequency, duration  A patient with lesion of left optic tract
of about 0.15 seconds will develop right Homonymous
 S2: High pitched, high frequency, Hemianopia
duration of about 0.12 seconds 157) A young boy sitting in a working room
152) Dialysis is monitored by which of the with sweating & BP of 150/80 which
following? of the following is the most likely
A) Creatinine clearance cause:
B) Inulin clearance A. Mental Stress
C) BUN B. Exercise
D) Albumin C. Heat Stroke
Ans: A D. Dehydration
153) Oral fecal route of which of the Ans: A
following? 158) In IV drug abuser, type of
A) Hep. A endocarditis:
B) Hep. B A) Infective
C) Hep. C B) Libman Sac
D) CMV C) SLE
Ans: A D) Marantic
Explanation:For feco-oral route Hep A > Hep Ans: A
E. 159) 10 mins after transplant rejection is
154) Malignant Melanoma risk factor: due to:
A) Prolong Sunlight exposure A) Cell Mediated
B) Xeroderma Pigmentosum B) Preformed Antibodies
C) Actinic Keratosis C) CD4
D) SCC D) CD8
Ans: A Ans: B
Explanation: 160) Six months transplant rejected &
 Exposure to excessive sunlight (UVA an reversed with immunosuppressive
UVB) at an early age is the single most drugs is most likely:
important risk factor A) Acute Cellular Rejection
 Prolong sunlight exposure> Xeroderma B) Chronic Rejection
Pigmentosum C) Preformed Antibodies
155) A tall man on oxygen suddenly started D) None of the above
being dyspnic due to: Ans: A
A) Spontaenous Pneumothorax Explanation:For Acute & chronic transplant
B) Tension Pneumothorax rejection 6 months is the boundary line, here the
C) Cardiac Tamponade reversal is in favor of acute as chronic is
D) COPD irreversible.
Ans: A 161) Droplet infection spread by:
156) Following an RTA, a patient A) 1 – 3 feet
developed right side homonymous B) 1 – 3 Meter
hemianopia which of the following C) 3 – 6 feet
site is damaged: D) 1 – 6 feet
A) Right Optic Tract Ans: A
B) Left Optic Tract Explanation:
C) Optic Chiasma  Droplet infection : 1 – 3 feet
D) Optic Nerve  Airborne infection: 3 – 6 feet
Ans: B
SK Origional – Golden 11 173

162) CRF patient presented with anemia C) Adductor Longus


must be given: D) Masseter
A) Iron E) Soleus
B) Erythropoietin Ans: B
C) Nothing 2) Draining sinus with yellow granules
D) Calcium supplements (granuloma):
Ans: B A) Nocardia
163) Which of the following nerve damage B) Syphilis
leads to winging scapula? C) Actinomycosis
A) Accessory Nerve D) E. Coli
B) Long thoracic nerve E) H. Pylori
C) Axillary Nerve Ans: C
D) Phrenic Nerve 3) The site for Pleural tap is:
Ans: B A) Upper border of the 5th rib at mid
Explanation:Long thoracic nerve supplies axillary line
Serratus Anterior muscle & damage to long B) 5th intercostal place
thoracic nerve will lead to winging scapula. C) Lower border of 7th rib at mid clavicular
164) Femoral artery pulse is palpated at: line
A) Middle of Inguinal ligament D) Lower border of 9th ICS at midaxillary
B) Mid Inguinal point line
C) At start of inguinal point E) Lower border of 9th rib at mid-clavicular
D) Near Pubic symphisis line
Ans: B Ans: C
165) Atypical Lymphocytosis is seen in Explanation: Lower Border 9th ICS and Upper
which of the following? of 9th Rib at Mid axillary line
A) SLE 4) A patient with ear discharge
B) AIDS alongside tympanic membrane
C) Infectious Mononucleosis perforation:
D) RA A) ASOM
Ans: C B) CSOM tubotympanic
166) A married patient in the ward C) Acquired Cholesteatoma
diagnosed as HIV and request you not D) Congenital Cholesteatoma
to tell this news to his wife. What will E) Barotrauma
the doctor about confidentiality Ans: B
ethics? 5) The highest systolic BP occurs in:
A) Persuade him to tell his wife, if refuses A) Renal arteries
then inform the wife B) Pulmonary arteries
B) Don‘t inform the wife C) Brachial artery
C) Inform the Police D) Pulmonary vein
D) Do nothing Ans: A
Ans: A 6) The renal estimation is done
through:
A) Inulin
=============== B) Creatinine
MEDICINE & ALLIED C) PAH
November 24th, 2020 – Afternoon D) Evan‘s blue
Ans: B
7) Before starting oral anticoagulant
1) Which of the following is white
heparin therapy, what test should be
muscle?
performed in order to analyze
A) Platysma
heparin activity?
B) Gastrocnemius
A) PT/APTT
SK Origional – Golden 11 174

B) INR B) Rectus femoris


C) APTT C) Abdominus Rectus
D) APTT/INR D) Fascia lata
Ans: D Ans: B
8) NSAIDs considered safe in lactating 14) Mallory bodies are made up of:
mother? A) Actin
A) Toradol B) Myosin
B) Ibubrofen C) Intermediate filaments
C) Naproxen D) Microtubule
D) Ketorolac Ans: C
Ans: B Reference: Robbins (Intermediate Cytokeratins)
9) Which of the following translocation 15) Peripheral vasodilation along with
is seen in Burkitt‟s Lymphoma? hypotension occurs in which type of
A) t (8:14) shock:
B) t (8:21) A) Cardiogenic shock
C) t (9: 22) B) Hypovolemic shock
D) t (11:14) C) Toxic shock
E) t (X:18) D) Septic shock
Ans: A Ans: D
10) Which of the following translocation 16) The interossei muscle of hands are
is seen in AML? supplied by:
A) t (8:14) A) Radial
B) t (8:21) B) Ulnar
C) t (9:22) C) Medial
D) t (11:14) D) Brachial
E) t (X:18) Ans: B
Ans: B 17) A man with severe diarrhea and a
10) Which of the following translocation is binucleated organism with flagella
seen in CML? extracted, what is the drug of choice?
A) t (8:14) A) Norfloxacin
B) t (8:21) B) Levosulpride
C) t (9:22) C) Loperamide
D) t (11:14) D) Metronidazole
E) t (X:18) Ans: D
Ans: C 18) Hyaline cartilage is present in:
11) Which of the following is brain eating A) Interphalyngeal joints
amoeba? B) Intercostal joints
A) Naegleria Fowleri C) Sacral joints
B) Entamoeba Histolytica D) Verterbral joints
C) Amoeba Proteus Ans: A >B
D) Chaos Carolinense 19) Effect of parasympathetic is:
Ans: A A) Increase HR
12) The amplification of nmyC gene B) Decrease HR
causes which child tumor? C) Decrease secretions
A) Retinoblastoma D) Increase heart contractions
B) Nephroblastoma Ans: B
C) Hodgkin lymphoma 20) The lymphatic drainage of the big toe
D) Neuroblastoma is to:
Ans: D A) Vertical group of superficial inguinal
13) Pelvic avulsion in inferior ischial lymph nodes
spine: B) Horizontal group of superficial inguinal
A) Rectus sheath lymph nodes
SK Origional – Golden 11 175

C) Deep inguinal lymph nodes B) Subdural hemorrhage


D) Popliteal nodes C) Subarachnoid hemorrhage
Ans: A D) Normal condition
21) Inflammation marker that is related to Ans: C
Ischemic heart disease: 28) The fracture of ischial spine and
A) PGE1 sacrum would result in injury to:
B) CRP A) Bladder
C) C3b B) Prostate
D) Integrins C) Rectum
Ans: B D) Urethra
22) Carcinoma of prostate involves which Ans: C
zone of prostate? 29) Largest cells amongst these in blood:
A) Lateral A) Neutrophils
B) Medial B) Basophils
C) Anterior C) Macrophages
D) Peripheral Zone D) Megakaryocytes
E) Posterior E) Monocytes
Ans: D Ans: E
Explanation: Prostate CA in peripheral zone and Remember: If asked largest cells among these,
posterior Lobe and BPH in Transitional Zone and without mentioning blood then choose
median Lobe Megakaryocytes.
23) Most common bladder carcinoma is: 30) What is the name of virus causing
A) Choreocarcinoma sudden acute respiratory syndrome
B) Transitional Carcinoma (SARS) of new corona infection?
C) BPH A) Novel Corona Virus
D) Seminoma B) Covid-19
E) Teratoma C) SARS-CoV-2
Ans: B D) Corona Virus
24) A damage to the right optic chiasm Ans: C
would result in which kind of vision 31) The unlocking of the knee is done by:
loss? A) Tibialis anterior
A) Bitemporal Hemianopia B) Tibialis Posterior
B) Right Homonymous Hemianopia C) Peroneus longus
C) Left Homonymous Hemianopia D) Popliteus
D) No vision impairment Ans: D
Ans: A 32) Long head of the bicep femoris is
25) What is common in an obese supplied by:
Myxedema lady? A) Tibial part of sciatic nerve
A) Tarsal tunnel syndrome B) Popliteal nerve
B) Carpal tunnel Syndrome C) Superior gluteal nerve
C) Horner‘s Syndrome D) Inferior gluteal nerve
D) Follicular carcinoma Ans: A
Ans: B 33) What acid-base abnormality occurs in
26) Right gastric artery is a branch of: diarrhea?
A) Splenic artery A) Metabolic alkalosis
B) Celiac artery B) Metabolic acidosis
C) Left gastric artery C) Respiratory acidosis
D) Hepatic artery D) Respiratory alkalosis
Ans: D Ans: B
27) A patient with blood tinged CSF is a 34) Dorsal Column Medial Leminiscus
case of: ends in:
A) Epidural hemorrhage A) Fasciculus Gracilis
SK Origional – Golden 11 176

B) Fasciculus Cuneatus C) Vertical


C) Nucleus Gracilis and Nucleus Cuneatus D) Rotation
D) Substantia Nigra Ans: D
Ans: C 42) The sac of the direct inguinal hernia
35) In adrenogenital syndrome urine is:
contains which substance? A) Medial to inferior epigastric
A) 17-alpha ketosteroids B) Lateral to inferior epigastric
B) 18- alpha ketosteroids C) Medial to superior epigastric
C) 24,25 progesterone D) Lateral to superior epigastric
D) 24 pregnolone Ans: A
Ans: A 43) Rib fracture most commonly occurs
36) A patient who was admitted to a at:
hospital suddenly developed dyspnea? A) Inferior portion
A) Pulmonary edema B) Superior portion
B) Pulmonary embolism C) Angle
C) Polycythemia D) Inner portion
D) Septic shock Ans: C
Ans: B 44) Which of the following is
37) A human hearts stops for a while what unmyelinated fibre?
will be the pressure in arteries and A) A alpha fiber
veins? B) A beta fiber
A) Mean Systemic Pressure C) A delta fiber
B) Mean arterial pressure D) C fiber
C) Mean venous pressure Ans: D
D) Systolic pressure 45) Withdrawal effect is generated by:
Ans: A A) Golgi tendons
38) B-Lymphocytes produce which type B) Nuclear bag
of cells? C) Free nerve endings
A) Monocytes D) Ruffini corpuscles
B) T-cells Ans: C
C) Neutrophils 46) Path from axillary lymph flow from
D) Plasma Cells breast has which two parts:
Ans: D A) Medial and lateral
39) Which of the following muscles do not B) Superior and anterior
tetanize? C) Anterior and Posterior
A) Skeletal muscles D) Lateral and Superior
B) Smooth muscles Ans: C
C) Both skeletal and smooth muscles Reference: RJ Last and G-rays anatomy.
D) Cardiac muscles 47) Inverse stretch reflex is elicited by:
Ans: D A) Golgi tendons
40) Blood transfusion reaction is which B) Nuclear bag
type of hypersensitivity reaction? C) Free nerve endings
A) Type-I D) Nuclear ends
B) Type-II Ans: A
C) Type-III 48) Single nerve supplies:
D) Type-IV A) Tibialis anterior
E) Cell-mediated B) Tibialis Posterior
Ans: B C) Peroneus longus
41) The type of rotation that occurs at D) Popliteus
atlantoaxial joint: Ans: A
A) Linear 49) Tin and canned foods contain:
B) Horizontal A) E.Coli
SK Origional – Golden 11 177

B) Bordetella E) T3 & T4
C) Shagella Ans: D
D) Botilinum 57) An anemic kid was seen with worm
Ans: D crawling out of his mouth:
50) The sign of past pointing occurs in: A) Ascaris Lumbricoid
A) Cerebrum B) Diphyllobotrum Latum
B) Cerebellum C) Tinea Solium
C) Mid Brain D) Entamoeba Histolytica
D) Medulla Ans: A
Ans: B 58) Which of the following ossifies last?
51) Main determinant of plasma A) Medial epicondyle
osmolality: B) Lateral Epicondyle
A) Glucose C) Olecrenon
B) Sodium D) Trochlea
C) Albumin Ans: B
D) Globulin Remember: Critol
Ans: B
Bone Age
52) Stomach circular muscles get more
thicker in: Capetulum 1 year
A) Fundus Radial Head 3 years
B) Antrum Internal Epicondyle 5 years
C) Pylorus Trochlea 7 years
D) Cardiac Olecrenon 9 years
E) Gastroduodenal orifice Lateral Epicondyle 11 years
Ans: E 59) Tubular organ with characteristic
53) Prevention of excessive medial and aggregate of lymphoid tissue:
lateral movements at TMJ: A) Palatine tonsil
A) Sphenomandibular ligament B) Jejunum
B) Stylomandibular ligament C) Appendix
C) Collateral ligament D) Ileum
D) Anterior ligament Ans: C>D
Ans: C 60) A patient has depressive state and her
54) Which of the following side effects is aunt was interested in knowing her
related to the use of statins? disease but the doctor refused; what is
A) GI Disturbance this termed as?
B) Vomiting A) Maleficence
C) Nausea B) Autonomy
D) Headache C) Confidentiality
Ans: A D) Respect
55) Bulbar urethra lies in: Ans: C
A) Deep perineal pouch 61) Which of the following is an adult
B) Superficial perineal pouch derivate of notochord?
C) Scrotum A) Nucleus Pulposus
D) Bladder B) Vertebrae
Ans: B C) Vertebral canal
56) Which of the following hormones is of D) Annulus fibrosis
immense importance to assess thyroid Ans: A
function? 62) Receptors for thyroid hormones are
A) T3 present in which part of the target cell:
B) Calcitonin A) Inside the nucleus
C) T4 B) On the nuclear membrane
D) TSH C) Inside the cell membrane
SK Origional – Golden 11 178

D) Cytoplasm A) Enterococcus
E) On the cell membrane B) Streptococcus
Ans: A C) Hemophilus
63) Testosterone is synthesized by: D) Klebsiella
A) Seminiferous tubules Ans: A
B) Sertoli cells 70) A 53 year old lady is concerned about
C) Leydig cells her strong family history of Breast
D) Parenchymal cells Cancer and IHD. She is also
Ans: C concerned about osteoporosis, which
Explanation: Leutenizing Hormone (LH) causes prophylaxis drug will you give?
testosterone synthesis from Leydig cells. A) Bisphosphonates
64) The fluidity of the cell membrane is B) Tamoxifen
caused by: C) Raloxifen
A) Integrins D) cHRT
B) Lipopolysacharride E) Calcium and Vitamin D
C) Glucose Ans: C>A
D) Cholesterol 71) A disease caused by the bite of a
Ans: D mosquito:
65) Anterior fontanelle closes at which the A) Ochronosis
age of how many months? B) Whipple disease
A) 8 months C) Ascariasis
B) 12 months D) Filariasis
C) 22 months Ans: D
D) 26 months 72) Metastasis most common to the brain
Ans: B is from:
Range: 9-18 months. A) Liver
66) Regarding tracheoesophageal fistula, B) Kidney
which structure separates them? C) Lungs
A) Bronchial bud D) Breast
B) Laryngeal fold Ans: C
C) Laryngotracheal groove 73) Which of the following ligaments is
D) Thymus involved in maintaining the medial
Ans: C longitudinal arch?
Explanation: At the age of 4 weeks A) Calcaneonavicular ligament
Laryngeotracheal groove is formed which divides B) Posterior talofabular ligament
into respiratory and digestive tracts. C) Anterior talofibular ligament
67) Which part of the brachial plexus is D) Calcaneofibular ligament
present in axilla? Ans: A
A) Trunks 74) A man with moon face and skin
B) Cord pigmentation alongside a buffalo
C) Division hump includes:
D) Root A) Increase FSH
Ans: B B) Increase MSH
68) C1 (Atlas) Vertebra lacks which one of C) Increase LH
the following? D) Increase ACTH
A) Spinous process Ans: D
B) Articular facet 75) Aqueous humor is produced by:
C) Lateral process A) Non pigmented ciliary epithelium
D) Tranverse process B) Pigmented ciliary epithelium
E) Body C) Iris and lens
Ans: A D) Ganglion cell layer of the retina
69) Gram +ve Diplococi bacteria is: Ans: A
SK Origional – Golden 11 179

76) Carcinoma of the tongue most 82) A patient presents with BP of 180/110.
commonly occur at: Which of the following kidney
A) Lateral border of anterior 2/3rd structure is involved?
B) Lateral border of posterior 1/3rd A) JG Cells
C) Tip B) Glomerulus
D) Dorsum C) Bowmen‘s capsule
Ans: A D) Epithelial cells
77) Absence of ganglion cells occurs in Ans: A
which of the following conditions? 83) A patient presents with burns, which
A) Hirschprung‘s disease test should be done in him?
B) Toxoplasmosis A) Keratin
C) Ulcerative Colitis B) Globulin
D) IBD C) Albumin
E) Crohn‘s disease D) Transferrin
Ans: A E) Electrolytes
78) A 6 year old child with history of joint Ans: C
swelling and bleeding during a 84) An pulsating eye involves:
circumcision. Raised aPTT. Which A) External jagular vein
test should be done to confirm the B) Internal carotid artery and cavernous
diagnosis? sinus fistula
A) Factor V laden C) External carotid artery and cavernous
B) Factor 9 assay sinus fistula
C) Factor 8 assay D) Cavernous sinus fistula
D) PT Ans: B
Ans: C 85) A patient with right carotid
79) A kid with distended abdomen along endartectomy presents with dizziness
with hepato-splenomegaly and streaks on vigorous right arm movements and
in hair and skin? diagnosed as emboli in subclavian,
A) Kwashiorkor which artery is involved?
B) Marasmus A) Vertebral
C) Beri-Beri B) Carotid
D) Hypothyroidism C) Jagular
Ans: A D) Middle meningeal
80) Von Gierke disease is caused due to E) Anterior communicating
deficiency of: Ans: A
A) Branching enzymes 86) A patient with hematuria and RBC
B) Acid maltase cast in urine. Structure involved:
C) Glucose 6 Phospate A) JG Cells
D) Debranching enzymes B) Glomerulus
Ans: C C) Bowmen‘s capsule
81) A female patient with Hb 9.6 g/dl and D) Epithelial cells
doctor prescribed iron, after 2 months Ans: B
she presents with Hb 10.2 g/dl. What 87) A young patient presents with
should be further considered for jaundice. The investigations revealed
diagnosis? high serum ALP and decreased Hb
A) Ferritin and increased retic count. Most likely
B) Bone Marrow Biopsy cause of jaundice is:
C) Hb electrophoresis A) Hepatitis
D) Transferrin B) Drug induced cholestasis
Ans: C C) Cholingiocarcinoma
D) Hematological malignancy
E) Pigment stones
SK Origional – Golden 11 180

Ans: E A) Respiratory disease


88) 1 gram of fat contains: B) Carcinoma
A) 3 calories C) DVT
B) 6 calories D) Aortic aneurysm
C) 9 calories Ans: C
D) 12 calories 94) A patient who was a known case of
Ans: C cystic fibrosis now presents with
Remember: recurrent chest infections, Diagnosis
is of?
1 gram of proteins 4 calories
A) Bronchiectasis
1 gram of carbohydrates 4 calories B) Lung carcinoma
1 gram of alcohol 7 calories C) Hepatic adenoma
D) Cystic hygroma
89) Which of the following statements Ans: A
regarding sympathetic and 95) Gall bladder has a stone in the upper
parasympathetic preganglions is true? part of the bile duct, what will
A) They are both cholinergic and adrenergic increase?
B) They are neither cholinergic nor A) Glucose in blood
adrenergic B) Sodium in urine
C) They are cholinergic C) Fat in stool
D) They are adrenergic D) Glucose in urine
Ans: C Ans: C
90) A patient presents with red face and 96) A patient‟s intestine does not gets
loss of sweating function. Which nerve back to abdomen. Diagnosis?
supply is damaged? A) Gastroschisis
A) Cervical sympathetic B) Omphalocele
B) Lesser splanchnic C) Meckel‘s diverticulum
C) Greater splanchnic D) Intussusception
D) Least splanchnic E) Volvulus
Ans: A Ans: B
91) The bare area of liver is limited by: 97) Oral anticoagulant administration by
A) Right and left triangular ligament a patient is monitored by:
B) Falciform ligament A) PT
C) Ligamentum teres B) aPTT
D) Ligamentum venosum C) INR
E) Coronary ligament D) Hb
Ans: E Ans: C
92) A 64 years old female with fatigue, 98) Diagnosis of transudate is made upon:
easy bruisibility after small cuts and A) High sodium
recurrent infection alongside fever B) High potassium
from 2 weeks. After investigations the C) Hypercellular
report shows blast cells and Auer D) Hypocellular
bodies in smear. What is the likely Ans: D
diagnosis? 99) Staging for any malignancy is done
A) Acute promyeloblastic leukemia by:
B) Acute lymphoblastic leukemia A) Degree of differentiation
C) Acute myeloid leukemia B) Extent of spread
D) Non Hodgkin lymphoma C) Benign tumor
E) Aplastic anemia D) Recovery
Ans: C Ans: B
93) A pregnant woman is at risk of Remember:
developing: G > D : Grading for Differentiation
SK Origional – Golden 11 181

S >S : Staging for Spread Ans: D


100) A 70 years old lady admitted into a 106) A child with cysts in the kidney, the
hospital for long time is at high risk of pattern of inheritance in him is:
developing: A) Autosomal recessive
A) Respiratory disease B) Autosomal dominant
B) Carcinoma C) X-linked dominant
C) DVT D) X-linked recessive
D) Aortic aneurysm Ans: A
Ans: C 107) An adult with cystic kidney disease
101) Hemolytic disease of a new born will exhibit the pattern of inheritance
caused by RH Blood group of:
incompatibility requires maternal A) Autosomal recessive
products to enter the fetal blood B) Autosomal dominant
stream; the mediator of this disease is C) X-linked dominant
most likely: D) X-linked Recessive
A) IgG antibody Ans: B
B) IgM antibody 108) Which of the following of nerve is at
C) IgA antibody risk of damage with superior thyroid
D) RH antigen artery?
E) IgM and RH antigen A) Internal laryngeal nerve
Ans: A B) External laryngeal nerve
102) Goblet cells in a respiratory mucosa C) Recurrent laryngeal nerve
are present till: D) No risk to any nerve
A) Respiratory bronchioles Ans: B
B) Carina Remember:
C) Terminal bronchioles Superior Thyroid artery  External Laryngeal
D) Bronchi nerve
Ans: D Superior Laryngeal Artery  Internal Laryngeal
Explanation : Goblet cells upto Bronchi and nerve
Clara cells in Terminal Bronchioles MC Nerve damage  External Laryngeal Nerve
103) A patient with left ventricular Upper pole nerve damage  External Laryngeal
hypertrophy and pulmonary Lower pole nerve damage  Recurrent Laryngeal
hypertension would also have: 109) Drug of choice in heart failure:
A) Aortic aneurysm A) Dopamine
B) Right ventricular hypertrophy B) Adrenaline
C) Aortic stenosis C) Furosemide
D) Aortic regurgitation D) Atropine
Ans: B Ans: C
104) A soldier who was posted at Siachen 110) Development of breast lobules and
presents to the emergency with alveoli:
numbness of the fingers and A) Estrogen
headache. Most probable diagnosis: B) Progesterone
A) Primary polycythemia C) FSH
B) Pulmonary Embolism D) LH
C) Secondary Polycythemia Ans: B
D) Polycythemia Vera 111) A man with mind alert with eyes
Ans: C closed?
105) Largest difference of O2 lies in: A) Alpha waves
A) Aorta and pulmonary vein B) Beta waves
B) Aorta and vena cava C) Delta waves
C) Vena cava and pulmonary artery D) Theta waves
D) Vena cava and pulmonary vein Ans: A
SK Origional – Golden 11 182

112) A cotton factory worker will develop Ans: A


which of the following diseases? 120) What is the relation of left renal vein
A) Silicosis anteriorly?
B) Anthracosis A) IVC
C) Asbestosis B) SMA
D) Byssinosis C) Phrenic nerve
Ans: D D) Aorta
113) A halo shape organism: Ans: B
A) Enterococcus 121) Why are TB drugs administered in
B) E.coli combination?
C) Cryptococcus A) To increase half-life
D) Amoeba B) To decrease side effects
Ans: C C) To increase potency
114) What is more in dialysis fluid? D) To decrease resistance
A) Potassium Ans: D
B) Magnesium 122) Heart muscles act as syncytium due
C) Sodium to:
D) Bicarbonate A) Gap junctions
Ans: D B) High contractibility
115) Most common side effect of IUCD: C) SA node
A) PID D) AV node
B) Purulent discharge Ans: A
C) Irregular vaginal bleeding 123) In a diabetic patient with
D) Infections administration of insulin what will
Ans: C happen?
116) Deep sleep is characterized by: A) Increase PH
A) Bruxism B) Decrease PH
B) Decrease Vascular tone C) No change in PH
C) Increase Vascular tone D) Increase Sodium
D) Night terrors Ans: A
Ans: B 124) Thyroglobulin is produced from:
117) What will be shown on A) Chief cells
electroencephalograph of an B) Follicular cells
anesthetic patient? C) Parafollicular cells
A) High waves D) Isthmus
B) Low waves Ans: B
C) Irregular waves 125) Post ganglionic sympathetic nerve
D) Flat line supply is to:
Ans: B A) Detrusor muscle
118) A patient with anemia and PaO2 is B) Sweat gland
decreased: C) Adrenal medulla
A) Anemic hypoxia D) Adrenal cortex
B) Hypoxic hypoxia Ans: B
C) Secondary polycythemia 126) GEV of vagus nerve is:
D) Pulmonary embolism A) Nucleus ambiguous
Ans: B B) DCML
119) An patient turns out to be HIV C) Globus pallidus
positive, what will you do? D) Tractus solitaries
A) Order some more tests E) Dorsal vagus
B) Counsil Ans: E
C) Show sympathy 127) Scurvy is due to deficiency of:
D) Avoid telling patient A) Calcium
SK Origional – Golden 11 183

B) Niacin raised relics. Diagnostic investigation


C) Ascorbic acid will be:
D) Vitamin D A) Coomb‘s Test
Ans: C B) Blood Culture
128) A 35 years old hypertensive patient C) Urine Culture
comes to the emergency, you gave D) Osmotic fragility test
him anti-hypertensive that resulted in Ans: A
the increase of bilirubin level. Which 135) What is the most prominent feature in
antihypertensive is it? ovulation?
A) Hydralazine A) Increase temperature
B) Beta Blocker B) Increase PH
C) Methyl Dopa C) Ferning of cervix
D) Nitroglycerine D) Increase movements
E) Hydrochlorothiazide Ans: C
Ans: C 136) The normal range of FEV1/FEV is:
129) Pudendal nerve block is done at: A) 0.2
A) Ischial tuberosity B) 0.4
B) Ischial spine C) 0.3
C) Inguinal ligament D) 0.8
D) Coccyx Ans: D
Ans: B 137) Tumbling motility occurs in
130) Pancreatic enzyme deficiency would Coccobacillus bacteria are:
result in: A) Enterococcus
A) Decreased Micelle formation B) Streptococcus
B) Lactose intolerance C) Hemophilus
C) Decrease absorption of triglycerides D) Klebsiella
D) Increase absorption of protein E) Listeria
Ans: C Ans: E
131) Nerve supply to the skin below the 138) Annular pancreas represents:
orbit of the eyes and above mouth: A) Pancreatic head regression
A) Mandibular nerve B) Pancreatic neck suppression
B) Facial nerve C) Ventral bud fails to rotate
C) Maxillary nerve D) Beta cells of pancreas
D) Zygomatic nerve Ans: C
Ans: C 139) Regarding fat embolism:
132) Short term effect of angiotensin 2 is: A) Fracture of long bones
A) Increase thirst B) 72 to 96 hours after trauma
B) Increase aldosterone C) Reversible with treatment
C) Arteriolar vasoconstriction D) Long lasting
D) Increase PH Ans: A
Ans: C 140) Germ cells tumor of testis:
133) Thoracic duct drains into: A) Lymphoma
A) Confluence of left internal jagular vein B) Teratoma
and subclavian vein C) Cystic hygroma
B) Brachiocephalic vein D) Seminoma
C) Left Subclavian vein Ans: B
D) Internal jagular vein 141) Main feature pertaining to malignancy
Ans: A is:
134) A young patient with Hb level: 4.5 A) Pleomorphism
g/dl , WBC: 7000cells/cmm. Raised B) Metastasis
platelets and bilirubin. On blood C) Invasion of surrounding tissues
smear there was polychromasia and D) Infarction
SK Origional – Golden 11 184

Ans: B 147) A 65 years old Type-II DM with


Remember: Metastasis > Invasion > chronic renal failure and is on dialysis,
Pleomorphism. has developed chest infection. Which
142) Microscopic feature of malignancy us: of the following antibiotics can be
A) Pleomorphism used in him?
B) Metastasis A) Amoxicillin
C) Invasion of surrounding tissues B) Ceftazidime
D) Infarction C) Ciprofloxacin
Ans: C D) Co-Trimoxazole
Remember: Hall mark of malignancy  E) Tetracycline
Metastasis. Ans: C
143) Melanoma reaches to the orbital cavity Remember: If Azithromycin or Doxycycline in
via: option - clicks that:
A) Optic nerve 148) The pain from the gastric ulcers is
B) Sclera referred to:
C) Retina A) Epigastrium
D) Lymphatics B) Hypochondrium
E) Emissary veins C) Shoulder
Ans: E D) Right iliac fossa
144) A patient with pain in left flank and E) Umbilicus
hematuria. His labs reveal a normal Ans: A
level of BUN and creatinine. Calcium 149) Which of the following structures
level of 11 mg/dL and Hb 18 g/dl and shows concomitant hyperplasia and
mass is felt at left flanks. What is the hypertrophy?
diagnosis? A) Breast growth in puberty
A) Renal cell carcinoma B) Exercising muscle
B) HCC C) Uterus during pregnancy
C) Adrenal carcinoma D) Liver
D) Heaptoma Ans: C
Ans: A Remember: Uterus during pregnancy > Breast
145) The delay to wound healing is most growth in puberty.
commonly caused by: 150) Chemotherapy involves the
A) Infection mechanism of:
B) Malnutrition A) Apoptosis
C) Foreign body B) Necrosis
D) Immobility C) Infarction
Ans: A D) Metastasis
Remember: Ans: A
151) A patient is suffering from
Systemic factor causing Diabetes mellitus Pseudomonas aeruginosa infection
delayed wound healing (DM) with fever. What factor can worsen
Local factor causing Infection this kind of infection?
delayed wound healing A) IL-I
B) PG-E
146) Best route for anti-dote administration C) TNF
is: D) CD4
A) IM Ans: C
B) IV 152) A patient with osteoporosis and
C) SC osteoclastic activity, hormone involved
D) Oral would be:
Ans: B A) Calcitonin
B) Chalciferol
SK Origional – Golden 11 185

C) PTH ‗PAH‘, rather they were asking in pertinence to


D) TSH plasma volume measure so Evan‘s blue is the best
Ans: C option.
153) Decrease aldosterone would result in Source: The Evans blue dye (EBD) dilution
decrease reabsorption of: method, including a dye extraction step, is a
A) Potassium standard way of measuring plasma volume.
B) Sodium (NCBI).
C) Water http://pubmed.ncbi.nlm.nih.gov/9007461/.
D) Glucose Remember that NCBI is the only trusted source
Ans: B on web. Don‘t blindly follow Google.
154) Ewing sarcoma involves: 3) Exogenous pyrogens stimulate
A) Diaphysis production of cytokines. What is the
B) Metaphysis role of cytokines in the eventual
C) Epiphysis production of fever?
D) Metaphysis and Epiphysis A) Recruit neutrophils to kill bacteria
Ans: A B) Stimulates prostaglandins (PG) synthesis
Remember: GEOMED C) Directly acts on the hypothalamus
D) Cause increased production of fibrinogen.
Giant cell tumor Epiphysis
E) Stimulates production of C-reactive
Osteomyelitis Metaphysis proteins
Ewing sarcoma Diaphysis Ans: B
4) What is the first step in hemostasis
after a vascular injury?
=============== A) Fibrin formation
B) Vascular vasodilation
C) Vascular vasoconstriction
MEDICINE & ALLIED D) Platelet aggregation
November 24th, 2020 – Night E) Complement system activation
Ans: C
1) Which of the following properties best 5) Supernumerary cervical rib may cause
define Vitamin – E (Tocopherol) in the compression of:
executing its function? A) Brachial plexus and subclavian blood
A) Anticoagulant vessels.
B) Antioxidant B) Thoracic duct
C) Antilipidemic C) Inferior vena cava (IVC)
D) Co-enzyme D) Thoracic aorta
E) Co-factor E) Abdominal aorta
Ans: B Ans: A
Remember:Glutathione > Transferrin > Vitamin 6) Which of the following structures do
E > Vitamin C > Vitamin A. the cervical vertebrae lack?
2) Which of the following substances can A) Posterior horn
be used to measure the volume of B) Anterior horn
plasma? C) Lateral horn
A) Inulin D) White matter
B) Para-aminohippurate (PAH) E) Grey matter
C) Antipyrene Ans: C
D) Radiolabeled iodinated globin 7) In certain pathologies crescents in
E) Evans blue dye. glomeruli are formed due to the
Ans: E proliferation of:
Explanation: CPSP wasn‘t asking regarding renal A) Mesangial cells
blood flow measurement; had that been the case, B) Mesangial and endothelial cells
the correct option to opt for would have been C) Parietal cells
SK Origional – Golden 11 186

D) Visceral cells A) Osmolality increases in plasma


E) Endothelial cells B) Na increases in urine
Ans: C C) Na increases in plasma
References: D) Osmolality decreases in urine
 Essential of Nephrology [2nd Edition] by Ans: D
R.Kasi Visweswaran. 13) An athlete presents with a nodule type
 Robbins and Cotran Pathologic Basis of swelling on his upper arm for no
Diseased [8th Edition]. apparent reason. After three months it
8) Largest cells amongst these in blood: gradually decreased in size. The
A) Neutrophils changed occurred is called?
B) Basophils A) Dysplasia
C) Macrophages B) Hypertrophy
D) Megakaryocytes C) Hyperplasia
E) Monocytes D) Metaplasia
Ans: E E) Hypertrophy followed by hyperplasia
Rationale: If asked largest cells among Ans: D
these,without mentioning blood then choose Explanation: This condition is termed as
Megakaryocytes. ‗Myositis Ossificans‘. Yes! You can Google it now.
9) A 55-year-old patient present with 14) Deep Vein Thrombosis (DVT) from
coughing. He is a diagnosed oncology the lower limbs commonly lodges
case and has arrived for follow-up into:
treatment. The surgeon wants to A) Lungs
operate upon him; during the surgery B) Kidneys
he notices black lesions in hilar lymph C) Heart
nodes, what is the cause? D) Brain
A) Metastatic lesions E) Peripheries
B) Anthracosis Ans: A
C) Asbestosis 15) Which is the type of endocytosis that
D) Ferruginous bodies occurs in Lysosomes?
E) Silicosis A) Pseudopods mediated
Ans: B B) Phagocytosis
10) Which of the following embolisms C) Pinocytosis
causes the most extensive necrosis? D) Cell lysis
A) Air embolism Ans: B
B) Fat embolism 16) A slide of a tissue taken from the oral
C) Thromboembolism cavity is viewed under the electron
D) Nitrogen embolism microscope and the substance shows
Ans: B acini and ducts, which structure is it?
11) Half-life of the Dopamine is 2 A) Salivary gland
minutes; so after how many minutes B) Pharyngeal tonsils
the steady state will reach? C) Palatine tonsils
A) 12 minutes D) Enamel
B) 24 minutes Ans: A
C) 30 minutes 17) Which of the following suits the
D) 9 minutes Eucromatins?
E) 1 hour A) Contains densely packed chromatins
Ans: D B) Long chained
Explanation: The steady state is attained after C) Silent DNA
four to five half-lives so the correct answer that D) Stains densely with basophilic dye
suits this question is 9 minutes. Ans: D
12) A person has been given 1L of normal Explanation: Eucromatins are active in protein
saline infusion; what will happen? synthesis, light stained and have prominent
SK Origional – Golden 11 187

nucleolus. They stain densely with basophilic dye D) Type – IV


since they have abundant RNA (active in protein E) Cell – Mediated
synthesis) and RNA is basophilic. Ans: B
18) A female presented with intense Explanation: It is a cytotoxic (antibody mediated)
feeling of heat (hyperthermia) and her Type – II hypersensitivity disorder.
eyes were bulging outside 22) A CT Scan of an old man showed
(exophthalmos) and in investigations necrotic changes in his hypothalamic
her TSH level was low. Which type of area. Which type of necrosis has
hypersensitivity is this case? occurred in him?
A) Type-I A) Coagulative necrosis
B) Type-II B) Fibrinoid necrosis
C) Type-III C) Liquefactive necrosis
D) Type-IV D) Gummatous necrosis
E) Cell-mediated Ans: C
Ans: B Explanation: The type of necrosis that occurs in
Explanation: It is the condition of Graves‘ brain tissue is liquefactive.
disease (Hyperthyroidism) which is cytotoxic 23) A doctor in his clinical or hospital can
(antibody mediated) Type-II hypersensitivity. practice an effective management by:
19) Which of the following is true A) Evidence based management
pertaining to glomerulus of the B) Good and effective communication skills
kidney? C) Providing guided information
A) Podocytes make visceral layer D) Showing empathy and sympathy towards
B) It is located in DCT patient.
C) It is a triple layered structure Ans: B
D) Lymph nodes surround it all around 23) Accessory cervical rib is causing
Ans: A numbness and pain in the hand of a
20) An RH negative female who gave patient. Which part of the brachial
birth to an RH negative baby. She had plexus is associated with it?
a history of hemolytic reactions. Due A) C6
to puerperium blood loss, she was B) C7
transfused an RH positive blood. C) C5
Which option suits her best? D) C8
A) Transfusion reaction E) T1
B) Hemolytic reaction might ensue in future Ans: E
pregnancies 24) The accessory cervical rib is the
C) No need to give Anti-D antibody outgrowth of which cervical vertebra:
D) Give anti-D antibody Immedietly A) C6
Ans: D B) C7
Explanation: C) C5
 According to NICE guidelines any D) C8
pregnant woman who is RH negative & E) T1
exposed to RH postivive blood or any Ans: B
trauma during pregnancy which led to 25) Negative free water clearance occurs
blood mixing must be immedietly given in:
Anti-D A) SIADH
B) Diabetic Insipidus (DI)
21) The type of hypersensitivity that exists C) Hyperaldosteronism
in the Neuromuscular junction D) Conn syndrome
disorder of Myasthenia Gravis is: E) Addison Disease
A) Type – I Ans: A
B) Type – II 26) An elderly female patient presents
C) Type – III with a memory issue. The lesion
SK Origional – Golden 11 188

pertaining to this condition exists in Ans: C


which lobe? 32) Which of the following process is most
A) Frontal lobe important in the pathogenesis of clot
B) Parietal lobe formation?
C) Temporal lobe A) Endothelial injury
D) Occipital lobe B) Platelet aggregation
E) Occipito-parietal lobe C) Complement system activation
Ans: C D) Thromboxane A2
27) Lesion involving the parietal lobe of Ans: A
the brain will result in: 33) The substance released from
A) Athetosis fibroblast:
B) Asteriognosis A) Heparin
C) Prosopagnosia B) Histamine
D) Astigmatism C) Amorphous substance
Ans: B D) Serotonin
28) A patient presents with dysphagia and Ans: C
dysarthria. He also had ipsilateral 34) Which of the following are
Horner‟s Syndrome. Which of the predominantly the anti-tumor cells?
following arteries is involved in the A) CD – 4
lesion of the associated area? B) CD – 8
A) PICA C) NK Cells
B) AICA D) IL – I
C) ASA Ans: C
D) MCA 35) Important antigen that is involved in
E) PCA graft rejection:
Ans: A A) MHC
29) Which of the following is an index for B) HLA
the total peripheral resistance (TPR)? C) Rh factor
A) Mean Arterial Pressure (MAP) D) AB Blood antigen
B) Diastolic BP Ans: B
C) Systolic BP 36) A female arrived to the hospital for her
D) Pulse Pressure follow up; a pap smear of her cervix
E) Stroke Volume was obtained which showed cells with
Ans: B large nuclei and koilocytic changes in
30) In an acute hemolytic blood disease, the epithelium. The change is termed
the hemoglobin from the lysed cells as?
bind with: A) Metaplasia
A) Haptoglobin B) Dysplasia
B) Hemopexin C) Hypertrophy
C) Hemosiderin D) Neoplasia
D) Hemocysteine E) Hyperplasia
Ans: A Ans: B
Remember: In hemolytic blood diseases the level 37) A 48-year-old woman presents with
of haptoglobin decreases; having being bound to fever from 3 days. She also complains
hemoglobin. about urinary frequency beside loin
31) A protein that is important for the (flank) pain. The white blood cells
contraction of skeletal muscle but not count is 35,000/cmm. What is the
smooth muscle: next best step?
A) Myosin A) Urine culture
B) Actin B) Blood culture
C) Troponin C) CBC
D) Ca-ATPase D) ESR
SK Origional – Golden 11 189

E) Urine and blood culture 44) Dietary carcinogen active for


Ans: E esophageal carcinoma:
38) The mechanism of gene involvement A) Alcohol
for self – destruction is termed as: B) Nicotine
A) Necrosis C) Nitrosamine
B) Ischemia D) Aflatoxin
C) Apoptosis Ans: C
D) Cell – lysis 45) Dysphagia is most commonly present
Ans: C in which of the following conditions?
39) Which of the following structures A) SLE
shows concomitant hyperplasia and B) Dermatomyositis
hypertrophy? C) Scleroderma
A) Breast growth in puberty D) Lesch-Nyhan Syndrome
B) Exercising muscle Ans: C
C) Uterus during pregnancy 46) A patient presents with stroke, which
D) Liver has resulted in Left sided hemiplegia
Ans: C and he is unable to talk (aphasia), this
Remember: Uterus during pregnancy > Breast lesion is as a result of which affected
growth in puberty. artery?
40) Side effect following Total Parenteral A) PICA
Nutrition (TPN): B) AICA
A) Hypocalcaemia C) MCA
B) Hyperglycemia D) ACA
C) Hypernatremia E) PCA
D) No change Ans: C
Ans: B Remember: (Just for fun)To talk you need to
41) A young male presents with increased close and open your mouth; so when MOUTH
levels of renin and Blood Pressure of CLOSING ARTERY (MCA) becomes affected it
160/110 mmHg. What is the post results in aphasia.
probable cause? Note: MCA is not a mouth closing artery, it‘s
A) Afferent dilation written just to remember.
B) Sympathetic Stimulation Also note:
C) Essential Hypertension
Presentation Artery
D) Conn Syndrome
involved
E) Cushing Disease
Ans: B Lower limb weakness > Upper limbs CA
42) What causes decrease intact oxygen weakness
saturation without affecting oxygen Upper limbs weakness + unable to MCA
tension? talk (aphasia)
A) Carbon monoxide (CO) Lower limbs weakness + unable to MCA + ACA
B) V/Q talk (aphasia)
C) FEV1/FVC
D) Hypoxia 47) Transfer of heat from the core of the
Ans: A body to the surface depends on:
43) A kid with distended abdomen along A) Core body temperature gradient
with hepato-splenomegaly and streaks B) Remains constant
in hair and skin? C) Depends on external environment
A) Kwashiorkor D) Controlled by Sub-thalamus
B) Marasmus Ans: A
C) Beri-Beri Also remember:
D) Hypothyroidism Loss of water by evaporation and Core body
Ans: A insensible loss from the body temperature
SK Origional – Golden 11 190

depends on B) Trapezius
Loss of water by evaporation and Hypothalamus C) Serrates posterior inferior
insensible loss from the body is D) Subscapularis
controlled by Ans: B
54) Trachea is descending in superior
48) Malignant tertian malaria‟s causative mediastinum, which structure runs to
agent is? its right side?
A) Ovale A) Vagus nerve
B) Malariae B) Common Carotid Artery
C) Vivax C) Arch of aorta
D) Knowlesi D) Descending aorta
E) Plasmodium Falciparum Ans: A
Ans: E 55) A female presents with the problem in
49) Prominent pre-potential can be seen maintaining her memory. She was also
in which tissue? complaining of diarrhea from some
A) AV node days and also had rashes over the skin.
B) SA node Deficiency of what causes this
C) Bundle of His condition?
D) Purkinje fibers A) Vitamin B1
E) Myocardial tissue B) Vitamin B2
Ans: B C) Vitamin B6
50) Conducting system of the heart D) Vitamin D
comprises of: E) Vitamin B3
A) Nicotinic fibers Ans: E
B) Cholinergic fibers Explanation:Dementia, Dermatitis, Diarrhea.
C) Sympathetic fibers This triad represents ‗Pellagra‘ caused by the
D) Modified cardiac tissue deficiency of Niacin (Vitamin B3).
E) Adrenergic fibers 56) Anterior relations of suprarenal gland:
Ans: D A) Hepatorenal ligament
51) A boy is presented with loss of B) Aorta
consciousness by his mother. His C) Liver
investigation showed high RBS level D) Lesser sac
and his mouth gave a fruity odor, what Ans: D
is the most probable cause? 57) What is the most important factor for
A) Stroke exudate formation?
B) DKA A) Chemo taxis
C) Head injury B) Increased permeability of blood vessels
D) Pancreatitis C) Increased oncotic pressure
E) Uremia D) Decrease hydrostatic pressure
Ans: B Ans: B
52) A person exhibiting an extra 58) Hypornatremia is due to:
chromosome in his genes instead of A) Increased level on water
normal two: B) Increased level of sodium
A) Trisomy C) Increase hydrostatic pressure
B) Triploids D) Decreased hydrostatic pressure
C) Klinefelter Syndrome E) Increased oncotic pressure
D) Turner Syndrome Ans: A
Ans: A 59) A fisherman presents with low Hb and
53) Stab wound to the posterior triangle of macrocytic anemia, which organism is
the neck most likely causes damage to the causative agent?
which muscle? A) Diphyllobothrium latum
A) Scalene B) Ancylostoma dueodenale
SK Origional – Golden 11 191

C) Ascariasis 65) An 85-year-old patient presents to the


D) Trichuris clinic with pneumonia. He had done
E) Clonorchis Sinesis splenectomy 20 years back. The
Ans: A causative agent of pneumonia in him
Remember: is:
A) Klebsiella
Macrocytic anemia Diphyllobothrium latum
B) Mycoplasma
Microcytic anemia Ancylostoma dueodenale C) Pseudomonas
60) Which of the following parameters D) Staph Aureus
decreases during strenuous exercise? E) Strep Pneumonia
A) Cardiac output Ans: E
B) TPR 66) A young patient underwent surgery of
C) SV metallic valve replacement. He was
D) Diastolic BP sent home on regular drugs but he
E) Heart Rate presented with nose bleed (epistaxis)
Ans: B just 2 hours later. His platelets and
61) Cardiac muscle cannot be tetanized Hb counts are low and PT is
easily owing to: prolonged. What is the cause?
A) Short refractory period A) Idiopathic
B) Long refractory period B) Traumatic
C) Low threshold C) Drug-induced
D) Pre-potential D) Failed surgery
Ans: B Ans: C
62) An ECG with three „P‟ waves prior to 67) A male who is a homosexual and is a
„QRS‟ complex is due to: diagnosed case of HIV positive,
A) Decreased conduction via AV node presented with diarrhea. What will be
B) Increase firing of SA node the next best test to help him in
C) High conduction via AV node further management?
D) Mobitz-II heart block A) HIV PCR
Ans: D B) CD4 cells count
63) A child suffering from infection sent C) ELISA
his blood for histopathology. There D) CD8 cells count
were Warthin Finkeldey giant cells Ans: B
present in slide. The condition is: 68) A patient of ischemic heart disease
A) Rubella presents with the loss of
B) Chicken Pox accommodation reflex. The CT scan
C) Toxoplasmosis of his brain showed a clot in the brain.
D) Small Pox Where is the lesion?
E) Measles A) Cerebellum
Ans: E B) Midbrain
64) Alcoholic patient presents with an C) Medulla
episode of flu, the color of sputum is D) Pons
yellow and there are atypical E) Cerebrum
pneumonia symptoms. X-ray showed Ans: B
bilateral diffuse infiltrates. What is the 69) In treatment the cimetidine and
causative agent? sucralfate are given two hours apart.
A) Klebsiella What‟s the logic behind?
B) Mycoplasma A) Sucralfate lowers the half-life of
C) Pseudomonas cimetidine
D) Staph Aureus B) Cimetidine clearance increases
E) Strep Pneumonia C) Cimetidine is inactivated
Ans: B
SK Origional – Golden 11 192

D) Sucralfate halts the absorption of D) N-myc


cimetidine Ans: C
Ans: D 76) Virus causes cancer by undergoing
70) A patient who is on Anti-Tubercular alteration in:
Therapy (ATT) presents with tingling A) Nuclear integration
sensations in his extremities. The B) Cell membrane proliferation
drug causing it is: C) Protein synthesis
A) Isoniazid D) Lysis of Nucleic Acids
B) Pyrazinamide Ans: C
C) Ethambutol 77) Exclusive buffer found in the renal
D) Streptomycin system (Renal tubules) is:
Ans: A A) Phosphate
71) A patient who is on ATT presents with B) Ammonia
swelling and pain in his big toe. The C) Bicarbonate
causative drug is: D) Hemoglobin
A) Isoniazid Ans: B
B) Pyrazinamide 78) An ECG pattern of inverted T wave
C) Ethambutol with U wave present represents:
D) Streptomycin A) Hypercalcemia
Ans: B B) Hypokalemia
72) A patient presents with nerve deafness C) Hyperkalemia
in one ear (unilateral). The test will D) Hypernatremia
show: E) Hyponatremia
A) Normal sound in normal ear Ans: B
B) High sound in normal ear 79) The delay to wound healing might be
C) High sound in affected ear caused by:
D) No sound in affected ear A) Infection
Ans: A B) Malnutrition
73) A jet-engineer presents to the ENT C) Foreign Body
physicians and complains that he D) Immobility
works in an environment of loud Ans: A
noises. What kind of hearing defect he 80) The part of nephron that amounts for
might be suffering from on an 60-80% water absorption:
audiogram: A) DCT
A) Deafness to low frequency sound B) PCT
B) Deafness to every sound C) Ascending loop of Henle
C) Deafness to high frequency sound D) Collecting tubules
D) No deafness E) Descending loop of Henle
E) Audiogram would show dip at 2000 Hz Ans: B
Ans: A 81) A damage to the right optic tract
74) The oncological marker for ovarian would result in which kind of vision
cancer is: loss?
A) CEA A) Bitemporal Hemianopia
B) AFP B) Right Homonymous Hemianopia
C) CA – 125 C) Left Homonymous Hemianopia
D) Calcitonin D) No vision impairment
Ans: C Ans: C
75) Which of the following is the tumor 82) A patient presents with prolactinoma
suppressor gene? from some time which has impaired
A) Ras his vision in a way that he can‟t see
B) C-myc the lateral sides on his both eyes. This
C) p53 is:
SK Origional – Golden 11 193

A) Right Homonymous Hemianopia 88) An alcoholic man presents with


B) Left Homonymous Hemianopia exhibition of prominent neck veins
C) Astigmatism and limb swelling. He also had
D) Bitemporal Hemianopia centrilobar congestion and early
Ans: D fibrosis of liver. The cause is:
83) Which factor sees an increase in A) Alpha 1 antitrypsin deficiency
restrictive lung diseases? B) Right Heart Failure
A) FEV1/FVC C) Left Heart Failure
B) Compliance D) Chronic Alcoholism
C) FRC Ans: D
D) TV 89) Hypercalcemia causes increased
E) ERV production of:
Ans: A A) 24,25 dihydroxycalciferol
84) A diabetic patient with poor control of B) Vitamin – D3
blood pressure (Hypertensive) C) 25 Hydroxycalciferol
presents with bilateral pedal edema. D) Vitamin – D4
Drug that alters glycemic control: Ans: A
A) Amiloride 90) What is benedict test used for?
B) Furosemide A) To detect sugar in urine
C) Hydrochlorothiazide B) To detect reducing agents in urine
D) Triamterine C) For dyslipidemias
E) Spironolactone D) Hydrogen bonding
Ans: C Ans: B
85) A cardiac patient presents to the 91) The neurotransmitter responsible for
emergency; on examination he had a slow pain:
collapsing radial plus alongside A) GABA
diastolic murmur. What is the B) Glutamate
probable diagnosis? C) Ach
A) Aortic stenosis D) Substance P
B) Mitral stenosis Ans: D
C) Mitral Regurgitation 92) Most common side effect of
D) Aortic Regurgitation protamine sulphate:
E) Mitral Valve Prolapsed A) Hypertension
Ans: D B) Hypotension
86) A patient with Left Ventricular C) Bleeding
Hypertrophy (LVH) and increased D) Headache
EDV along with pulmonary edema. Ans: B
On auscultation he had diastolic Remember:
murmurs. His diagnosis is:
Most common and immediate Hypotension
A) Aortic Regurgitation
side effect of Protamine
B) Mitral stenosis
Sulphate.
C) Mitral Regurgitation
D) Aortic Stenosis Side effect that occurs after Bleeding
E) Mitral Valve Prolapsed prolonged use.
Ans: A
87) An injury to the foramen ovale in skull 93) The most common type of duodenal
would result in: ulcer and artery rupture occurs in:
A) Paralysis of muscle of mastication A) Anterior wall of stomach and gastric
B) Loss of accommodation reflex artery.
C) Loss of sensation over zygoma B) Posterior wall of duodenum and gastro
D) Loss of forehead sensations duodenal artery.
Ans: A
SK Origional – Golden 11 194

C) Anterior wall of duodenum and splenic 99) HLA-B27 is associated with:


artery. A) Reiter‘s Syndrome
D) Lesser curvature of stomach with gastric B) Type – I DM
artery. C) Rheumatoid Arthritis
Ans: B D) Type – II DM
94) A patient is suffering from Ans: A
Pseudomonas aeruginosa infection. 100) Bones strength depends on which of
What factor can worsen this kind of the following?
infection? A) Arrangement of collagens
A) IL – I B) Holes and gaps
B) PG – E C) Decrease density
C) TNF D) Increase density
D) CD4 E) Bone matrix and mineral ratio
Ans: C Ans: E
95) The mechanism of child birth is an 101) A patient appears to be having a
example of: butterfly rash on his face. What is the
A) Negative feedback best initial test for this patient?
B) Positive cycle A) ANA
C) Vicious cycle B) Anti – ds – DNA
D) Coagulation Cascade C) Anti – Ro
Ans: B D) Anti – SSA
96) C5 and C6 roots of brachial plexus are Ans: A
damaged, the weakness would result Remember: For SLE
in which muscle?
Best initial test ANA
A) Infraspinatus
B) Extensor ulnaris Confirmatory/Diagnostic Anti-ds-DNA
C) Flexor carpi ulnaris test
D) Flexor Pollicis longus
E) Flexor Pollicis Brevis 102) A patient present with Rheumatoid
Ans: A arthritis with CCP being raised and
97) The portion of brain lesion that would ANA negative. There is a precursor
result in fluent aphasia: protein which stains red on Congo
A) Wernicke‘s Area stain. What is the precursor protein?
B) Broca‘s Area A) B2 micro-globulin chain
C) Premotor cortex B) Serum amyloid associated protein
D) Somatosensory association area C) Lambda chains
Ans: A D) Complement cascade
Remember: Ans: B
103) A foreigner was on a visit to Pakistan
Non-fluent aphasia Broca‘s area (Broken for his first time. While on his stay, he
Words) developed fever and light cough. On
Fluent aphasia Wernicke‘s area lab investigation his RFTs were
deranged. What‟s the cause?
98) A patient has been seen to have A) Leptospirosis
keratitis and dry eyes alongside B) Lyme disease
xerostomia (dry mouth). The C) Falciparum Malaria
condition is known as: D) Hemolytic crisis
A) Reiter‘s Syndrome Ans: C
B) Sjogren Syndrome 104) A patient presents to clinic with his
C) Rheumatoid Arthritis Hb level being 4 g/dl and Coomb‟s
D) Multiple Myeloma Test positive. What is the cause?
Ans: B A) G6PD Deficiency
SK Origional – Golden 11 195

B) Autoimmune Hemolytic Anemia 109) When does the secondary oocyte


C) Megaloblastic anemia completes its maturation?
D) Pernicious anemia A) At fetal life
Ans: B B) At puberty
105) A young patient with HB level: 4.5 C) At ovulation
g/dl, WBC: 7000 cells/cmm. Raised D) At fertilization
platelets and bilirubin. On blood Ans: D
smear there was polychromasia and 110) In ITP which derangement is most
raised relics. Diagnostic investigation common?
will be: A) Bleeding Time
A) Coomb‘s Test B) PT
B) Blood Culture C) A-PTT
C) Urine Culture D) Hb level
D) Osmotic fragility test Ans: A
Ans: A 111) Blockage of Azygous vein in aortic
106) A 45-year-old female presents with opening on diaphragm would result in
bone pain and paresthesia in the lower blood pooling in:
limbs. Her laboratory investigations A) Right ascending lumbar and subcostal
were done and it showed Serum B) Left ascending lumbar and subcostal
Calcium level: 2.60 mmol/L (2.10-2.65 C) Subclavian vein
mmol/L Normal), inorganic D) Inferior Vena Cava
phosphates: 0.9mmol/L (0.8-1.9 Ans: A
mmol/L is Normal). ALP 385 IU/L 112) A patient was administered drug by a
(Normal: 120-365 IU/L) and the quack, he presents with jaundice.
serum PTH levels were border-line Which of the following enzymes will
raised; 6.1 mmol/L (Normal: 1.5-6.0). be raised?
What is the most likely diagnosis? A) ALP
A) Primary Hyperparathyroidism B) AST
B) Pseudo Hyperparathyroidism C) ALP & AST
C) Secondary Hyperparathyroidism D) ALT
D) Tertiary Hyperparathyroidism Ans: D
E) Normal clinical picture 113) A patient with anemia of chronic
Ans: A disease and raised ALP with increased
107) A female patient with primary unconjugated bilirubin and decreased
amenorrhea, webbed neck and short Hemoglobin. What‟s the cause?
stature. The chromosomal A) Hemolytic anemia
presentation will be: B) Cholelithiasis
A) 46XO C) Pigment stones
B) 46XXY D) Alcoholism
C) 45XXY Ans: C
D) XX/XY 114) The help for the diagnosis of
E) 45XO malignant sarcoma would be taken
Ans: E from?
108) During the third trimester of A) Pleomorphism
pregnancy the uterus is most sensitive B) Necrosis
to which hormone? C) Increase Mitotic activity
A) Oxytocin D) Increase vascularity
B) Calcitonin Ans: D
C) Progesterone 115) The cancer which is caused as a result
D) Estrogen of Strontium-90 which is found in
E) HCG milk, is:
Ans: A A) Osteosarcoma
SK Origional – Golden 11 196

B) Renal Cell Carcinoma C) Deep inspiration


C) Hepatocellular Carcinoma D) Shallow breathing
D) Ewing Sarcoma Ans: D
Ans: A 123) A drug with a half-life of 36 hours and
116) A pregnant lady presents at 20 weeks it‟s concentration in blood is 4
of gestation. What advice would you moles/L. Safe concentration is blood
give her to take a diet high in? is 1 mole/L. How long should the
A) Vitamin E stoppage of this drug be done before
B) Vitamin A starting the therapy?
C) Calcium A) 72 hrs
D) Fat B) 108 hrs
Ans: C C) 32 hrs
117) An old patient present with depression 4) 9 hrs
and lack of motivation after a an injury Ans: A
to his brain. The lesion would be 124) Concentration of a substance in urine
present at: is 2 moles/L. Urine flow rate is 100
A) Temporal lobe m/s and plasma flow rate is 10 m/s.
B) Frontal lobe How much substance will be cleared?
C) Parietal lobe A) 10
D) Occipito-parietal lobe B) 20
Ans: B C) 25
118) Derivative of 4th and 6th brachial D) 30
arches: E) 40
A) Cricoid cartilage Ans: B
B) Thyroid Remember: Clearance = UV/P
C) Mylohyoid 125) The sympathetic system maintains the
D) Maxillary artery auto-regulation of:
Ans: B A) Skin
119) An experiment was carried out on an B) Exercising skeletal muscle
animal. Its ventromedial nucleus of C) Heart tissue
the hypothalamus was destroyed. The D) Kidney
experiment will cause? Ans: A
A) Overeating above limits 126) A patient presents with unique
B) Temporary inhibition of feeding choreiform movements due to the
C) Permanent inhibition of feeding lesion to:
D) No change A) Substantia Nigra
Ans: A B) Caudate
120) Ratheke‟s pouch is the derivation of? C) Mid brain
A) Posterior pituitary D) Amygdala
B) Stomodaeum E) Hippocampus
C) Ventral Gut Ans: B
D) Yolk Sac 127) A child was presented with constricted
Ans: B pupil after taking a drug. Naloxone
121) Medial necrosis on autopsy: was administered but it did not work.
A) Berry aneurysm Possible toxic agent is?
B) Dissecting aneurysm A) Phenobarbitone
C) Polyarthritis nodosa B) Ethusoxamide
D) Fibrinoid necrosis C) Benzodiazepines
Ans: B D) Sodium Valproate
122) Dead space doesn‟t change with: Ans: A
A) ETT 128) Maximum arterial pressure dissipation
B) Tracheostomy at:
SK Origional – Golden 11 197

A) Left ventricle to aorta junction D) Heparin


B) Arteriolar capillary junction Ans: A
C) Pulmonary artery junction 135) A 40-year-old teacher presents with
D) Anastomosis tremors and poor handwriting. There
Ans: B was no rigidity and the movement was
129) Mechanism of action of intact. What is your drug of choice?
Acetazolamide: A) Bromocriptine
A) Increase H+ secretions B) Propranolol
B) Decrease Na/K activity C) Fluoxetine
C) Decrease H+ secretions and increase D) Imipramine
Na/K activity. Ans: B
D) Increase Na secretions 136) Enterokinase acts for conversion on:
Ans: C A) Trypsin
130) Low level of cholesterol in B) Trypsinogen
premenopausal women and protection C) Amylase
from thromboembolism is done by: D) Chymotrypsinogen
A) Progesterone Ans: B
B) HCG 137) Progressive lengthening of PR interval
C) FSH until a drop beat occurs on ECG is the
D) LH representation of which block?
E) Estrogen A) Mobitz – II
Ans: E B) 1st– Degree Heart Block
131) A patient presented with severe fluid C) 3rd Degree Heart Block
loss due to diarrhea that resulted in D) Mobitz – I
metabolic alkalosis. The site of fluid E) Wolff – Parkinson – White syndrome
loss is: Ans: D
A) Jejunum 138) A prokinetic used for diabetic gastro
B) Ileum paresis:
C) Stomach A) Ondansetron
D) Colon B) Dimenhydranate
Ans: D C) Metoclopramide
132) A road crusher presents to the clinic D) Levosulpride
with chest problems and interstitial Ans: C
fibrosis. The condition is: 139) An obese patient with raised RBC has
A) Silicosis been diagnosed as a case of Diabetes
B) Asbestosis Mellitus. Drug of choice?
C) Pneumoconiosis A) Insulin
D) Anthracosis B) Biguanides
Ans: A C) Sulfonylureas
133) A slide was study for infection, an D) Alpha Glucosidase inhibitors
oxidative burst was seen. It Ans: B
represents: 140) Most radiosensitive tumor:
A) Neutrophil marginalization A) Lymph node
B) Lytic activity of nucleus B) Osteosarcoma
C) Generation of microbicidal activity C) Ewing Sarcoma
D) Normal cell activity D) RCC
Ans: C Ans: A
134) Treatment of choice in the upper GI 141) Nerve carrying parasympathetic
bleeding: components:
A) Octerotide A) Pudendal nerve
B) Vitamin – K B) Greater splanchnic nerves
C) FFP C) Lesser splanchnic nerves
SK Origional – Golden 11 198

D) Pelvic splanchnic nerves A) Lower limbs are stronger than upper


Ans: D limbs
142) Functional residual capacity (FRC) is B) Can hold neck
the measure of: C) C – Shaped vertebral column
A) ERV+RV D) All the four limbs are equal in size.
B) ERV+TV Ans: C
C) RV+TV 149) The process of long term memory
D) ERV+TV+RV involves?
Ans: A A) Protein synthesis
143) Vessel involved in abdominal angina: B) Long chain fatty acids
A) SMA C) Carbohydrates breakdown
B) IMA D) Protein and carbohydrates lysis
C) Gastrodueodenal artery Ans: A
D) Gastric artery 150) Total Peripheral Resistance is related
E) External Iliac artery to the measurement of:
Ans: A A) Cardiac output
144) The initial structure of the lungs seen B) HR
on bronchoscopy: C) MAP
A) Anterior division of the Left upper lobe D) Diastolic BP
B) Anterior division of the Right upper lobe E) Pulse Pressure
C) Posterior division of the right lower lobe Ans: D
D) Posterior division of the right middle lobe 151) A patient was started on therapy of
Ans: B clindamycin and now develops
145) A patient presents with anemia post- diarrhea. The causative agent is:
gastrectomy. Which cells of the A) Clostridium difficile
stomach are liked involved? B) Staph Aeureus
A) G cells C) Staph Epidermidis
B) S cells D) E.Coli
C) Parietal cells Ans: A
D) Mucous cells 152) Most common face lesion in HIV:
Ans: C A) Dysplastic Nevus
146) A soldier who was posted at Siachen B) Kaposi Sarcoma
presents to the emergency with C) Discoid rash
numbness of the fingers and D) Melanoma
headache. Most probable diagnosis: Ans: B
A) Primary polycythemia 153) A little boy who took anti-malarial
B) Pulmonary Embolism now presents with dark colored urine,
C) Secondary Polycythemia what is the cause?
D) Polycythemia Vera A) Autoimmune hemolytic anemia
Ans: C B) G6PD deficiency
147) A 40-year-old patient presented to the C) Sickle cell anemia
emergency department with acute gall D) Good pasture syndrome
bladder colic pain. She was prescribed Ans: B
morphine. What else should be added 154) Gastro-esophageal pyloric and other
to further relieve her pain? sphincters are relaxed by:
A) Hyoscine A) CCK
B) Ketorolac B) Gastrin
C) Pethidine C) VIP
D) Paracetamol D) Trypsin
Ans: A Ans: C
148) What is true about a new born? 155) A patient presents with rigidity, the
affected area has a lesion in:
SK Origional – Golden 11 199

A) Caudate Ans: B
B) Amygdala 163) Common remnant of allantois is termed
C) Sub thalamus as:
D) Substantia Nigra A) Urachal cyst
Ans: D B) Urachal sinus
156) Injury to the neck of the fibula would C) Urachal fistula
cause an injury to which artery? D) Medial ligament
A) Anterior tibial Ans: A
B) Posterior tibial Remember:
C) Peroneal
Total failure of urachus to Urachal fistula
D) Lateral Genicular
undergo obliteration
Ans: A
157) An autosomal dominant: Partial failure of urachus to Urachal cyst
A) Both male and female are involved obliterate
B) Only male is involved Patent lumen that remains Urachal sinus
C) Only female is involved either superior or inferior
D) Affects only male children
Ans: A 164) A person sitting quietly in a chair with
158) Definition of an axon: closed eyes and resting. What type of
A) Brings information towards the cell body sleep waves will be present?
B) Insulated by oligodendrocites A) Beta
C) Takes away information from the cell B) Alpha
body. C) Delta
D) Has abundant nuclei D) Gamma
Ans: C Ans: B
159) Fracture of the lesser trochanter 165) A patient has low Glomerular
causes injury to which artery? Filtration Rate (GFR). A Physician
A) Psoas muscle must be concerned mostly about
B) Gluteus Maximus which parameter?
C) Gluteus Minimus A) High creatinine
D) Sartorius B) Hypernatremia
Ans: A C) Potassium
160) 5 year post-op transplant which will be D) Calcium
the most likely cancer? Ans: C
A) RCC 166) Lymphatic drainage of the skin of
B) HCC glans penis:
C) Melanoma A) External iliac nodes
D) Lymph proliferative disorder B) Internal iliac nodes
Ans: D C) Superficial inguinal lymph nodes
161) Drug that is inactivated highly in D) Celiac lymph nodes
lever: Ans: C
A) Streptomycin 167) Normal daily requirement of iron in
B) Regular Insulin woman?
C) Nitroglycerine A) 2g
D) Paracetamol B) 4g
Ans: C C) 1g
162) Concept of mother transmitting D) 6g
genetical disease via DNA of: Ans: A
A) Nucleus 168) Where are the smooth muscle
B) Mitochondria pericytes found in the body?
C) Golgi body A) Neurons
D) Nucleolus B) Nephrons
SK Origional – Golden 11 200

C) Pars Nervosa MEDICINE & ALLIED


D) Blood vessels November 25th, 2020 - Afternoon
Ans: D
Remember: Pars Nervosa has pituicytes not
1) 1 gram of fat gives how much calories?
pericytes. a. 6
169) Mechanism of action in fever is:
b. 7
A) Cox-II inhibition
c. 9
B) Thromboxane A2 inhibition
d. 12
C) Cox-I and Cox-II inhibition
e. 13
D) Increase prostaglandins synthesis
Ans: C
Ans: D
2) Which among the following nerve is
170) What increases insulin secretion?
damaged in a patient with ptosis,
A) Beta Blockers mydriasis, diplopia and crossed
B) Somatostatin
hemiplegia?
C) Secretin
a. Occulumotor
D) Hypokalemia
b. Anducent
Ans: C
c. Trochlear
171) Patient presented to the emergency
d. Trigeminal
room after multiple fractures. What
Ans: A
will be the first intervention?
3) Most common opportunistic infection
A) Fluid resuscitation in AIDS?
B) Take X-ray
a. Pseudomonas
C) Referral on the spot
b. Histoplasma
D) CT Scan
c. Pneumocystis carni
Ans: A
d. M.Bovis
172) A patient presented with signs of
d. S. aureus
hypovolemic shock and received
Ans: C
treatment for this very cause. What
4) Schistosoma hematobium cause
indicator should be assessed for initial which cancer of bladder?
recovery in him?
a. Transitional cell ca
A) CO
b. Squamous cell ca
B) HR
c. Adenocarcinoma
C) Urine output
d. None
D) Edema
Ans: B
Ans: C
5) This AIDS opportunist mostly causes
diarrhea:
a. Coccidioides immitis
===============
b. Cryptococcus
c. Cryptosporidia
d. Histolasma
e. Toxoplasma
Ans: C
6) In a 60 yr old female with breast
cancer and osteoporosis hx of
ischemic heart disease which should
be given?
a. Raloxifen
b. Bisphosphonates
c. Vit. D
d. Calcium
Ans: A
SK Origional – Golden 11 201

7) Which of following vertebrae has no Ans: A


spine? 13) Which of the following occurs in
a. C5 severe diarrhea?
b. C2 a. Metabolic alkalosis
c. C1 b. Hyperkalemia
d. C4 c. Hyponatremia
e. C7 d. Metabolic acidosis
Ans: C Ans: D
8) Girl of 11 year was referred to X-ray 14) Goblet cells in respiratory epithelium
department for X ray of elbow joint to are not present beyond:
know agw so x ray will of? a. Bronchi
a. Capitulum b. Respiratory bronchioles
b. Trochlea c. Terminal bronchioles
c. Medial epicondyle d. Alveoli
d. Lateral epicondyle Ans: A
Ans: D 15) Best markers of MI in first hour:
9) In thyroidectomies, the superior a. Trop I
thyroid artery is ligated right at the b. AST
upper pole of thyroid gland. The nerve c. Myoglobin
liable to be injured if the artery is d. Trop T
ligated some distance from the gland Ans: C
will be: 16) Organism of sub-acute endocarditis
a. External laryngeal nerve is:
b. Internal laryngeal nerve a. Staph.Aureus
c. Phrenic nerve b. Strep. Viridans
d. Recurrent laryngeal nerve c. Enterococcus fecalis
e. Superior laryngeal nerve d. Pseudomonas
Ans: A Ans: B
10) Injury in the posterior triangle of neck 17) Which immunoglobulin is present in
damages which structure? RH incompatibility?
a. Spinal part of accessory nerve a. IgG
b. CNS part of accessory nerve b. IgM
c. Vagus nerve c. IgD
d. 2nd part of subclavian artery d. IgE
Ans: A e. IgA
11) 7th month pregnant lady with chronic Ans: A
ITP having ecchymosis and epistaxis. 18) Graves ophthalmopathy true is:
Her platelets count 5*10 and on full a. Optic nerve compress
steroid dose what will be next suitable b. Unilaterally involves
step? c. Medial & lateral rectus muscle involve
a. IV immunoglobulin Ans: A
b. Immunosuppressant‘s 19) Prolong use of ocp will increase risk
c. Splenectomy of:
d. Steroids a. Thromboembolism
Ans: A b. Breast cancer
12) 5 year old child episodes of epistaxis c. Endometrial carcinoma
and gum bleed, plt count normal. BT d. Ovarian carcinoma
20 MIN. APTT 28 SEC. Diagnosis: Ans: A
a. VWD 20) Set point in temperature regulation
b. Hemophilia control by the response to change in
c. ITP temperature, Receptors are located in:
d. TTP a. Anterior hypothalamus
SK Origional – Golden 11 202

b. Great veins c. B6
c. Posterior hypothalamus d. B7
d. Skin Ans: B
e. Spinal card 27) Cervical dysplasia is caused by which
Ans: A of following?
21) From sitting to standing position a. CMV
there is compensation by which b. Herpes
mechanism? c. HPV
a. Increase HR d. Pseudomonas
b. Increase Venous return Ans: C
c. Decrease HR 28) Down syndrome is caused by:
d. Increase TPR a. Trisomy 23
e. Increase CO b. Trisomy 21
Ans: A c. Trisomy 24
22) Staging for any malignancy done to d. Trisomy 25
see: Ans: B
a. Pleomorphism 29) Most common cardiac anamoly is:
b. Extend of spread a. ASD
c. Metastasis b. VSD
d. Invasion c. PDA
Ans: B d. TOF
23) Which of the following has highest Ans: B
chances of developing endometrial 30) Woman with mitral stenosis at 28
cancer and indication of week of gestation dyspnea worsen due
Hysterectomy? to:
a. Simple hyperplasia a. Pulmonary hypertension
b. Complex hyperplasia with atypia b. IVC compression
c. Complex hyperplasia without atypia c. Systolic HTN
d. Squamous metaplasia d. Diastolic HTN
e. Chronic endometritis Ans: A
Ans: B 31) Posterior 1/3rd blood supply of heart:
24) A female of 35 yr present with increase a. LCA
central obesity hyperpigmentation b. RCA
over the body which one is responsible c. LAD
for her changes? d. Marginal Artery
a. Fsh Ans: B
b. Acth 32) Increase stroke volume keeping TPR
c. Lh constant:
d. TRH a. Increase pulse pressure and MAP
Ans: B b. Increase stroke volume and map
25) Anterior pituitary tumor will c. Increase cardiac output and map
compress: d. Decrease pulse pressure and map
a. Optic nerve Ans: A
b. Occulomotor nerve 33) A chronic pressure overload in the left
c. Abducent nerve ventricle due to aortic stenosis results
d. Trochlear nerve in:
e. Trigeminal nerve a. Altered left ventricle distensibility
Ans: A b. Decreased contractility
26) Beri Beri is caused by which of c. Left ventricular dilatation
following vitamin deficiency? d. Left ventricular hypertrophy
a. B2 e. No change
b. B1 Ans: D
SK Origional – Golden 11 203

34) Person work in digging deep earth d. A women told you she is depressed and
crust is prone to: occasionally she has suicidal thoughts
a. Silicosis e. None of the above
b. Anthacosis Ans: E(FA Step 2 )
c. Asbestosis 41) At apex of lungs:
Ans: A a. Increase ventilation
35) A patient had non-productive cough b. Increase V/Q
for 2 months non-smoker. Now c. Increase perfusion
presented with blood streaked d. Decrease V/Q
sputum. Left Lung has 5 cm hilar Ans: B
mass. Sputum shows hyper chromatic, 42) 30 yr old male who was treated for
pleomorphic cells with scanty meningitis one month back has
cytoplasm. Cause of this condition? complaint of morning headache. O/e
a. Silicosis there is bilateral papilledema. Likely
b. Asbestosis cause:
c. Smoking a. Blockage of dural sinus
d. Passive smoking b. Hydrocephalus
e. Radon exposure c. Involvement of meninges
Ans: E d. Involvement of cerebral vessels
36) Similarity between cardiac and e. Subdural collection
skeletal muscle is: Ans: B
a. Transverse Striations 43) An old man falls from the bed,
b. Transverse Tubules fractured the neck of femur. Cause of
c. Gap junctions osteoporosis:
Ans: A a. Poor diet
37) Most serious side effects of estrogen b. Decrease blood supply
in postmenopausal: c. Old age
a. Thromboembolism d. Hormonal insuffienccy
b. Endometrial Ca Ans: D
c. Breast Ca 44) The most commonest hospital
d. Ovarian carcinoma acquired organism for UTI:
Ans: B a. E. Coli
38) Male presented with head injury b. P-auriginosa
present with decrease thirst and c. S-aureus
decrease plasma osmolarity: d. Citrobacter species
a. Stimulation of lateral hypothalamus e. Klebsella species
b. SIADH Ans: A
c. Cushing Reflex 45) Von Gierke‟s disease is due to
Ans: B deficiency of enzyme:
39) Increase sodium causes: a. Glucose-6-phosphatase
a. Cell expansion b. Glucose-1-phosphatase
b. Cell shrinkage c. Branching enzyme
c. Cell burst d. Myophosphorylase
d. None Ans: A
Ans: B 46) Type of hypersensitivity in pt who has
40) Confidentiality shouldn‟t be breached been given diphtheria toxoid?
in situation: a. Type1
a. An HIV b. Type2
b. An adult male sexually abusing daughter c. Type3
c. A girl told you she has hid 100 tablets to d. Type4
commit suicide Ans: C
SK Origional – Golden 11 204

47) Unconscious patient with pin point Ans: B


pupil not responding to naloxone 54) Babinski sign is positive in lesion in
most probably due to: which of following?
a. Herion a. Pyramidal lesion
b. Hashesh b. Hippocampus
c. Morphine c. Amygdala
d. Phenobarbitone d. Cerebellum
Ans: D Ans: A
48) Which of the following drugs is 55) Right gastric artery is branch of:
recommended for the treatment of a. Hepatic artery
severe, multidrug resistant falciparum b. Left hepatic artery
malaria? c. Splenic artery
a. Artemisinin d. Gastroepiploic artery
b. Chloroquine Ans: A
c. Quinine 56) Costal cartilage is:
d. Sodium stibogluconate a. Fibrocartilage
e. Primaquine b. Hyaline
Ans: A c. Elastic cartilage
49) A 60 years old patient having d. Chondro cartilage
hypertension, diabetes which drug will Ans: B
be safe for him? 57) Most lethal organism after transfusion
a. Ca blockers is:
b. ACEI a. Hep. B
c. B. Blockers b. Hep. C
d. Nitroglycerin c. Herpes
Ans: B d. CMV
50) Which hormone responsible for Ans: D
delayed gastric emptying? 58) Good pasture syndrome:
a. CCK a. Sub endothelial deposition
b. Gastrin b. Sub epithelial deposition
c. Secretin c. Linear deposition
d. Somatostatin d. IgG deposit
Ans: A Ans: C
51) Structure posterior to carotid sheath: 59) The differentiating feature of
a. Vagus kwashiorkor from marasmus:
b. Internal jugular a. Fat loss
c. Trachea b. Pitting edema
d. Sympathetic trunk c. Muscle wasting is not present
Ans: D d. None of them
52) Parasympathetic supply to Ans: B
submandibular salivary gland is from: 60) Child engulf peanut suddenly become
a. Sup salivary nucleus cyanosed common region where
b. Inf. salivary nucleus peanut lodge:
c. Auriculotemporal nerve a. Rt. superior
d. Lesser petrosal nerve b. Rt. middle
e. Greater petrosal nerve c. Rt. inferior
Ans: A d. Left middle
53) Neck of femur is supplied by which Ans: C
arteries? 61) Best pulmonary function test for the
a. Obturator and medial circumflex Asthma is:
b. Medial and lateral circumflex a. FEV-1
c. Posterior and medial circumflex b. Residual volume
SK Origional – Golden 11 205

c. Tidal volume e. Nasopalatine Nerve


d. Vital capacity Ans: E
e. Inspiratory capacity 4) Muscle of extensors of back is
Ans: A supplied by which of the following
62) Production of saliva mainly increased nerves?
by: a. Intercostal nerves
a. Anti-cholinergic drug b. Dorsal primary rami
b. Parasympathetic simulation c. Ventral primary rami
c. Sympathetic stimulation d. Greater splanchnic nerves
d. Somatostatin Ans: B
Ans: B 5) A person losses 2 Liters of water and
now he replaces it with pure water.
What will happen?
=============== a. Decrease plasma osmolality
b. Increase plasma osmolality
MEDICINE & ALLIED c. Increase urine osmolality
d. No change expected
November 25th, 2020 - Night Ans: A
6) A woman losses 2 Liters of water and
1) The association of IgA nephropathy is now he replaces it with distilled water.
which kind of deposits? What will happen?
a. Sub epithelial deposits a. Increase in ECF Volume
b. Sub endothelial deposits b. Increase plasma osmolality
c. Linear sub endothelial deposits c. Increase urine osmolality
d. Dome and spike sub epithelial deposits d. No change expected
e. Meningeal deposits Ans: A
Ans: E 7) What is the genetical inheritance
Remember: pattern of Hemophilia?
Sub epithelial deposits PSGN + MPGN a. Autosomal dominant
(Tram track) b. Autosomal recessive
Sub endothelial SLE c. Chromosomal aberration
deposits d. X-linked recessive
Linear Sub endothelial Goodpasture Ans: D
deposits Syndrome 8) A damaged occurred to the foramen
Dome and Spike Sub MGN ovale and as a result the structures
endothelial passing through it have suffered lesion
Kimmestiel Wilson DM Nephropathy too, what will be the presentation of
lesions the patient?
a. Upper face affected
2) Which structure is transmitted via b. Lower third face affected
foramen cecum? c. Face and upper limbs affected
a. Ophthalmic vein d. Loss of sensation over orbital cavity
b. Ophthalmic artery Ans: B
c. Emissary Vein 9) The salivary secretions from parotid
d. Olfactory nerve gland are controlled by the following
Ans: C nerve:
3) The transmission of the following a. Facial nerve
structure occurs via incisive foramen: b. Auriculo-temporal nerve
a. Ophthalmic vein c. Hypoglossal nerve
b. Emissary vein d. Glossopharyngeal nerve
c. Olfactory nerve Ans: D
d. Mandibular branch of Trigeminal Nerve
SK Origional – Golden 11 206

10) The blood group antigens are not a. Warfarin


present on RBCs until: b. Heparin
a. 5 weeks c. FFP
b. 10 weeks d. Warfarin and FFP
c. 15 weeks Ans: B
d. 20 weeks 17) The ESR above 100 mm/hr. is in
e. 25 weeks which of the following diseases?
Ans: D a. Congestive heart failure
11) The nerve supply to the sides of the b. Polycythemia
scalp at temporal and parietal region c. Leukocytosis
is from: d. Multiple Myeloma
a. Maxillary division of the trigeminal nerve Ans: D
b. Mandibular division of the trigeminal 18) The lesion occurred at the caudate
nerve and globus pallidus of the brain.
c. Auriculo-Temporal Nerve There was loss of GABA in Substantia
d. Chorda tympani Nigra. The condition is of:
Ans: C a. Alzheimer‘s Disease
12) A person has visual difficulty while b. Parkinsonism
coming down the stairs, which nerve c. Horner Syndrome
seems affected? d. Retrograde Amnesia
a. Abducens nerve Ans: B
b. Occulomotor nerve 19) A section of caudal pons consists of
c. Facial nerve horizontal fibers. Theses fibers most
d. Optic nerve likely consist of:
Ans: B a. Superior cerebellar peduncle
13) A renal disease with continuous b. Middle cerebellar peduncle
elevated levels of renin: c. Inferior cerebellar peduncle
a. Child polycystic kidney disease d. Sub thalamus
b. Adult polycystic kidney disease Ans: B
c. Focal segmental glomerulonephritis High Yield:
d. Goodpasture syndrome
Cerebellum connected Superior cerebellar
Ans: B
to mid brain via peduncle
14) An apparently male presents with
gynaecomastia, tall stature and small Cerebellum connected Inferior cerebellar
testis. What is the most probable to medulla via peduncle
diagnosis? A section of horizontal Middle cerebellar
a. Turner Syndrome fibers in caudal pons peduncle
b. Klinefelter Syndrome 20) The part of the brain responsible for
c. Down syndrome anxiety, emotions and rage:
d. Pseudo hermaphrodite a. Amygdala
Ans: B b. Lateral Thalamic nuclei
15) After an injury to the shoulder a c. Hippocampus
person has loss of sensation over the d. Medial Thalamic nuclei
lateral side of the arm and he couldn‟t Ans: A
abduct his arm too, lesion is to: 21) Lesion at the left optic radiations of
a. Axillary nerve the optic track would result in:
b. Musculocutaneous nerve a. Right Homonymous Hemianopia
c. Radial nerve b. Left Homonymous Hemianopia
d. Ulnar nerve c. Bitemporal Hemianopia
Ans: A d. Right Temporal Vision loss
16) Natural antithrombotic agent Ans: A
amongst the following:
SK Origional – Golden 11 207

22) Lesion of Right lateral geniculate d. Constipation


body would result in: Ans: D
a. Right Homonymous Hemianopia 28) An 8 year old patient presents to the
b. Left Homonymous Hemianopia clinic with periorbital edema. His labs
c. Bitemporal Hemianopia showed protein greater than 3.5g/dl.
d. Right Temporal Vision loss What is the probable cause?
Ans: B a. Potassium retention
23) The vitamins which are produced by b. Sodium retention
intestinal flora and absorbed into the c. Naturesis
blood: d. Albuminuria and sodium retention
a. Vitamin D Ans: D
b. Vitamin C 29) The PH of urine can‟t be lowered
c. Vitamin K below 5.4 even after giving
d. Vitamin B12 Bicarbonates (HCO3). What is the
e. Vitamin B6 condition termed as?
Ans: C a. RTA1
24) Loss of two points discrimination, fine b. RTA2
touch and vibrations in the lower c. RTA4
limbs. The region at brain affected d. Both RTA1 & RTA4
would be: Ans: A
a. Fasciculus Gracilis 30) Renal tubular necrosis would result in:
b. Fasciculus Cuneatus a. Hyperkalemia
c. Globus Pallidus b. Hypernatremia
d. Substantia Nigra c. Hypokalemia
Ans: A d. Hyponatremia
Remember: Ans: A
31) Diagnosis of gonococcus is made by:
Signals from the upper Fasciculus
a. Eosin
limbs received at Cuneatus
b. Basophilic dye
Signals from the lower Fasciculus Gracilis c. Gram stain
limbs received at d. Fermentation
25) The nerve supply to the gracillis Ans: C
muscle is from: 32) Which of the following increases
a. Pudendal nerve absorption of iron in the gut?
b. Iliohypogastric nerve a. Green vegetables
c. Obturator nerve b. Citrus fruit
d. Both obturator and pudendal nerves c. Chocolates
Ans: C d. Apple
26) A lady is in labor, she has been Ans: B
administered oxytocin but she is not 33) The lesion in left Pre-Central Gyrus
responding to it, the reason lies in the would result in:
fact of: a. Right sided hemiplegia
a. Decrease oxytocin receptors on b. Left sided hemiplegia
myometrium c. Paraplegia
b. Resistance to oxytocin d. Facial paralysis
c. No oxytocin receptors present at all Ans: A
d. Drug interaction 34) The deep vein thrombosis (DVT)
Ans: A would initially lodged into:
27) What is the side-effect of Morphine? a. Periphery
a. Hypotension b. Coronary artery
b. Rashes c. Cerebral artery
c. Diarrhea d. Branches of pulmonary artery
SK Origional – Golden 11 208

Ans: D 41) The primordial germs cells are formed


35) Which is the primary site of DVT? at which week?
a. Pulmonary artery a. End of 3rd week
b. Pulmonary vein b. Start of 1st week
c. Abdominal veins c. End of 6th week
d. Deep veins of calf d. At birth
Ans: D Ans: A
36) Breaking a bad news to the patient: 42) The ectoderm and endoderm meet at
a. In a formal session caudal ends at:
b. In a hurried manner a. Vitelline duct
c. Through written form b. Cloacal membrane
d. Shouldn‘t be told c. Yolk sac
Ans: A d. Extraepiploic membrane
Remember: If ‗formal session‘ is not in the Ans: B
option and instead ‗in a crispy and acceptable way‘ 43) Parasympathetic act on SA node via
is present, choose the later then. which neurotransmitters?
37) What type of necrosis will be seen in a. Nor-epinephrine
brain on CT scan? b. Adrenaline
a. Coagulative c. Ach
b. Fibrinoid d. Epinephrine
c. Medial Ans: C
d. Gummatus 44) The low voltage QRS complexes
e. Liquefactive commonly represent:
Ans: E a. Acute MI
38) A liver specimen for microscopy was b. LVH
presented and there was less material c. RVH
surrounding epithelial cells, likely d. Old MI
cause would be: Ans: D
a. Caseous necrosis 45) Ventricular systole on the ECG is
b. Non Caseous Necrosis mainly represented by:
c. High vascularity a. P waves
d. Apoptosis b. ST segment
Ans: A c. QRS Complex
39) Immediate reversal of warfarin toxicity d. PR segment
is done via: Ans: C
a) Vitamin – K 46) The cause of the Left Axis deviation
b. FFP on then ECG is:
c. Whole Blood a. Left ventricular muscle damage
d. Cryoprecipitate b. AV node damage
Ans: B c. Purkinje fibers damage
Remember: d. Right ventricular damage
 Immediate FFP Ans: D
 Slow Vitamin – K 47) A patient with angular stomatitis and
40) Diphtheria toxoids resulted in Arthus cheilosis, the deficiency is in:
formation. Type of Hypersensitivity a. Vitamin K
reaction is: b. Riboflavin
a. Type – I c. Thiamine
b. Type – II d. Cholecalciferol
c. Type – III Ans: B
d. Type – IV 48) Which of the following nerve lesion
Ans: C due to DM results in vision problems?
a. Occulomotor
SK Origional – Golden 11 209

b. Abducens a. Increase CO
c. Trochlear b. Decrease HR
d. Optic c. Decrease SV
Ans: A d. Decrease MAP
49) A patient who was on Anti-TB (ATT) Ans: B
drugs developed swelling of the big 56) A soldier who was posted at Siachen
toe. The drug responsible is: presents to the emergency with
a. INH numbness of the fingers and
b. Ethambutol headache. Most probable diagnosis:
c. Pyrazinamide a. Primary polycythemia
d. Streptomycin b. Pulmonary Embolism
Ans: C c. Secondary Polycythemia
50) A patient had a car accident; which d. Polycythemia Vera
mechanism will be responsible for Ans: C
restoring his MAP? 57) A soldier went on a high altitude and
a. Brain-Bridge Reflex developed dyspnea. What is the
b. RAAS cause?
c. Baro-receptor reflex a. Primary polycythemia
d. Knee jerk reflex b. Pulmonary Embolism
Ans: C c. Pulmonary Edema
51) The antibodies used in diagnosis of d. Polycythemia Vera
hashimoto thyroiditis: Ans: C
a. Ant-Ro 58) Difficulty in differentiating red from
b. Anti-mitochondrial green has developed in a patient on
c. Anti-thyroglobulin and anti-mitochondrial ATT. The drug responsible is:
d. Anti-parietal a. INH
e. Calcitonin b. Ethambutol
Ans: C c. Pyrazinamide
52) Type of necrosis in dissecting aortic d. Streptomycin
aneurysm: Ans: B
a. Coagulative 59) Gastro-esophageal pyloric and other
b. Fibrinoid sphincters are relaxed by which
c. Liquefactive hormone?
d. Medial a. CC
Ans: D b. Gastrin
53) A patient died and after his autopsy c. VIP
was done, the tunica media was d. Trypsin
lacking in blood vessel, The condition Ans: C
is: 60) The splitting of S2 occurs due to:
a. Berry aneurysm a. Early closure of aortic valve
b. Caseous necrosis b. Late closure of aortic valve
c. Sarcoidosis c. Early closure of pulmonary valve
d. Fibrinoid necrosis d. Late closure of pulmonary valve
Ans: A Ans: D>A
54) The local anesthetic will affect which 61) Which of the following acts via c-
fibers first? GMP?
a. Type A delta a. LH
b. Type A beta b. ACTH
c. Type C c. ANP
d. Type B d. FSH
Ans: C Ans: C
55) The increase in CVP is owing to:
SK Origional – Golden 11 210

62) The diarrhea which does not respond Ans: A


to anti-diarrheal is treated by: 68) Type of change in metaplasia:
a. Diphenoxylate a. Polarity change
b. Norfloxacin b. Irreversible change
c. Metronidazole c. Necrotic change
d. Levosulpride d. Functional and morphological change
Ans: B Ans: D
Remember: Norfloxacin > Diphenoxylate 69) Best drug to lower the Triglyceride
63) A patient presented with severe fluid (TG) level:
loss due to diarrhea that resulted in a. Gemifibrozil
metabolic acidosis and hypokalemia. b. Niacin
The site involved is: c. Omega – 3
a. Jejunum d. Nicotinic acid
b. Ileum Ans: A
c. Stomach 70) Nerve supply of the adrenal medulla
d. Colon is:
Ans: D a. Lesser splanchnic nerve
64) A mother brings her child with a b. Greater splanchnic nerve
complaint that her child has visual c. Pudendal nerve
problems at night, what is d. Iliohypogastric nerve
responsible? Ans: B
a. Riboflavin 71) Gastric lymphoma in a patient occurs
b. Niacin owing to:
c. Retinol a. DVT
d. Vitamin – K b. CMV Infection
Ans: C c. H.pylori infection
65) Terminal ileum was resected, what d. Fungal infection
should be injected to the patient? Ans: C
a. Vitamin – B12 72) In an injury to the pyramidal tract,
b. Vitamin C what kind of bodily functions are lost?
c. Vitamin D a. Sensory
d. Riboflavin b. Motor
e. Niacin c. Mixed
Ans: A d. Pain and temperature
66) The pain related to peptic ulcer is Ans: B
carried by: 73) What is the change that occurs at the
a. Phrenic nerve 36 weeks of pregnancy?
b. Greater splanchnic nerve a. Decrease in FRC
c. Lesser splanchnic nerve b. Increase in FRC
d. Vagus c. Increase in Hb
Ans: B d. Decrease blood volume
67) A young patient who had eaten junk Ans: A
food half an hour ago, presents to the 74) The renal problem associated with
doctor with abdominal pain. His SLE:
blood sample was taken for LFTs, a. IgA Nephropathy
after centrifugation his plasma is b. FSGN
milky white, what is responsible? c. PSGN
a. Chylomicrons d. Lupus Nephritis
b. LDL Ans: D
c. VLDL 75) What is the rationale behind
d. Cholesterols concentrated urine in summers?
a. Increase secretion of ADH
SK Origional – Golden 11 211

b. Decrease in ADH 82) Stratified cuboidal epithelium is found


c. Increase Aldosterone in:
d. Increase insulin a. Ducts of sweat gland
Ans: A b. Ducts of salivary gland
76) The most easy and the cheapest c. Vagina
investigation of choice to diagnose d. Urethra
cervical cancer: Ans: A
a. Colposcopy Remember:
b. Cervical biopsy Stratified columnarepithelium Salivary gland
c. Shave biopsy Stratified cuboidal epithelium Sweat gland
d. Pap-Smear
Ans: D 83) Submucosal fibrosis is a:
77) Risk factor for the development of a. Malignant condition
malignant melanoma: b. Premalignant condition
a. Nitrosamine c. Premalignant lesion
b. Xeroderma pigmentosa d. Prebenign condition
c. Congenital Navus Ans: B
d. Lichen Planus 84) A patient who underwent develop
Ans: B laparotomy and after this suddenly
78) The exposure to ultraviolet light can developed hypotension, abdominal
result in: distention and tachycardia. The cause
a. Cataract is?
b. Conjunctivitis a. Cardiogenic shock
c. Glaucoma b. Hypovolemic shock
d. Trachoma c. Toxic shock
Ans: A d. Normal after surgery
79) Vessel narrowing involved in Ans: B
abdominal angina: 85) What takes place in neurogenic
a. SMA shock?
b. IMA a. Decrease peripheral vascular resistance
c. Gastrodueodenal artery b. Increase peripheral vascular resistance
d. Gastric artery c. Increase Heart Rate
e. External Iliac artery d. Increase Cardiac Output
Ans: A Ans: A
80) Atrophy of the calf muscles and lost of 86) After 10 days of transfusion a child
stretch reflex with weakness of legs, developed fever, diarrhea and skin
the lesion in: rashes. What is the cause?
a. UMN lesion a. Anaphylactic reaction
b. LMN lesion b. Type – II Hypersensitivity
c. Sensory lesion c. Graft Vs Host rejection
d. Mixed lesion d. Type – III HSR
Ans: B Ans: C
81) A histological slide of a tissue taken 87) Whit is the mechanism of action
from the oral cavity is viewed under (MoA) of captopril?
the electron microscope and the a. Decrease Angiotensin – II
substance shows connective tissue b. Decrease Angiotensin – I
acini and ducts, which structure is it? c. Vasoconstriction
a. Salivary gland d. Diuresis
b. Pharyngeal tonsils Ans: A
c. Palatine tonsils 88) Clostridium botolinum causes:
d. Enamel a. Flaccid paralysis of respiratory muscles
Ans: A b. Constipation
SK Origional – Golden 11 212

c. Skin rashes 96) The muscle involved in unlocking of


d. Angiosarcoma the knee:
Ans: A a. Sartorius
89) A patient post-accident has blood loss b. Vastus medialis
and low GFR, the cause: c. Vastus lateralis
a. Low BP d. Popliteus
b. Low blood cells Ans: D
c. Low hematocrit 97) The deficiency seen in plummer
d. Low renal blood flow vinson syndrome:
Ans: D a. Calcium
90) A patient was started on therapy of b. Magnesium
clindamycin and now develops c. Sodium
diarrhea. The causative agent is: d. Potassium
a. Clostridium difficile e. Iron
b. Staph aeureus Ans: E
c. Staph epidermidis 98) Most common cause of hepatic
d. E.Coli cirrhosis:
Ans: A a. Hepatitis B
91) The drug of choice for gas gangrene b. Hepatitis A
is: c. Hepatitis E
a. Clindamycin d. Hepatitis C
b. Ciprofloxacin Ans: D
c. Norfloxacin Remember: Alcohol > Hepatitis C
d. Penicillin 99) cAMP second messenger system acts
Ans: D through:
92) About adrenal glands: a. Protein change
a. Left adrenal vein drains into renal vein b. Receptors
b. Right adrenal vein drains into renal vein c. Signals
c. Left adrenal vein drains into IVC d. Clearance
d. Right adrenal vein drains left adrenal Ans: B
gland 100) First line of defence against microbial
Ans: A agents:
93) Sympathetic beta adrenergic effect on a. Lungs
heart is: b. Liver
a. Decrease HR c. Skin
b. Decrease AV Nodal conduction d. Neutrophils
c. Increase contractility of heart Ans: C
d. Decrease contractility of heart 101) A person on exogenous steroids would
Ans: C have:
94) The sign of cerebellar damage: a. Increase blood levels of ACTH
a. Resting tremor b. Decrease blood levels of ACTH
b. Athetosis c. Increased cortisol
c. Chorea d. Increased insulin
d. Dysdiadokinesis Ans: B
Ans: D 102) A group of people divided in a group
95) The main determinant of diastolic and were randomly given a drug.
blood pressure is: What kind of a sampling it is?
a. TPR a. Non-randomized sampling
b. HR b. Stratified random sampling
c. SV c. Non-startified random sampling
d. CO d. Cohort study
Ans: A Ans: B
SK Origional – Golden 11 213

103) A patient presented to the clinic with a. Increases


pain, there was rustling sound on b. Decreases
auscultation of the heart, which part c. Remains normal
pertaining to heart is affected: d. Becomes zero
a. LV e. Infinity
b. RV Ans: A
c. Myocardium Remember:
d. Endocardium Increase! Before you make unnecessary errata.
e. Pericardium 109) Main feature of malignancy:
Ans: E a. Invasion
104) A diabetic patient presented with b. Metastasis
pneumonia. He has thick mucoid c. No change
sputum. Causative organism: d. Increases in size
a. Staph Aureus Ans: B
b. S.Pneumoniae
c. Klebsiella Pneumoniae
d. Staph agalactae
Ans: C 110) The increase in number of cells is:
105) Investigation of choice for typhoid a. Hypertrophy
fever in first week: b. Metaplasia
a. Urine culture c. Dysplasia
b. Blood culture d. Hyperplasia
c. Antibody test Ans: D
d. Stool Antigen 111) Thyroglobulin is synthesized by which
Ans: B thyroid cells?
Remember: BASU (Pneumonic). a. Parafollicular
b. Follicular
1st Week Blood culture
c. Colloid cells
2nd Week Antibody (Widal)
d. Thyroid lymphatics
3rd Week Stool antigen
Ans: B
4th Week Urine culture Remember:
Parafollicular  Produces Calcitonin.
Note: It doesn‘t necessarily mean this chart is
112) An emphysema patient presents with
standardized. Use at your own discretion.
decrease air entry on left side and
106) Grading is done for:
trachea is shifted to the left. The
a. Degree of differentiation
investigation of choice in her will be:
b. Extent of spread a. CT Scan
c. Benign tumor b. MRI
d. Recovery
c. CXR
Ans: A
d. Lung biopsy
Remember:
Ans: C
G->D: Grading for Differentiation
113) Function of thyroid at cellular level:
S->S : Staging for Spread
a. Increase fats synthesis
107) What is the predisposing condition to b. Decrease protein synthesis
Adenocarcinoma of the esophagus? c. Increase protein synthesis
a. Lichen Planus
d. Decrease glucose synthesis
b. Leukoplakia
Ans: C
c. Erythroplakia
114) Potent and major stimulus resulting in
d. Barret Esophagus
release of erythropoietin:
Ans: D
a. Hypoxia
108) V/Q Ratio in physiological dead
b. Hyperglycemia
space: c. Hypoglycemia
SK Origional – Golden 11 214

d. High Na 122) On ECG there is absent P wave but


e. Low K normal QRS complex; where is the
Ans: A pacemaker located?
115) Renin secretion is increased by: a. SA node
a. Increased potassium concentration b. AV node
b. Decrease potassium concentration c. Parkine fibers
c. Increase calcium d. Bundle of His
d. Hypoxia Ans: B
Ans: B 123) A patient having paralysis of right side
116) Morphological changes in chronic of body with deviation of angle of
hepatitis: mouth to the left and dribbling of
a. Apoptosis saliva but the patient can close both
b. Complement cascade eyes. Injury is at the level of:
c. Peripheral necrosis and fibrosis a. Supranuclear
d. No major changes b. Pontine nucleus
Ans: C c. Petrous part of temporal bone
117) What are the inflammatory cells d. Stylomastoid foramen
responsible for sarcoidosis? Ans: A
a. B – Lymphocytes 124) The best quality of a doctor to reach at
b. T – Lymphocytes the diagnosis of a disease:
c. Memory cells a. Sympathetic attitude
d. Neutrophils b. Active listening
Ans: B c. Helping nature
118) The root value for ansa cervicalis: d. Quick response
a. S1,S2 and S3 Ans: B
b. C1,C2 and C3 125) Most common cause of liver cirrhosis
c. C1,C2, C3 & Hypoglossal nerve in a young man:
d. T1 and T2 a. Alcoholism
Ans: C b. Hepatitis B
119) In fight and flight what response c. Hepatitis C
occurs? d. Fungal infection
a. Decrease CO Ans: A
b. Decrease HR 126) The elevation of renin level is due to:
c. Decrease Airway resistance a. High parasympathetic activity
d. Decrease cardiac contractility b. Hypoxia
Ans: C c. Decrease stretch over atrial baroreceptors
120) Drug acting on H+/K+ pump and d. High brain bridge reflex
decreases acidity: Ans: C
a. Metronidazole 127) Down syndrome is a trisomy:
b. Metoclopramide a. 13
c. Omeprazole b. 18
d. Ketoconazole c. 21
Ans: C d. 23
121) The cause of increase ketone bodies is Ans: C
due to: 128) Fibroblasts secrets:
a. Insulin deficiency a. Heparin
b. TSH deficiency b. Histamines
c. High T3 and T4 c. Amorphous substances
d. Glucose deficiency d. Osteoclasts
Ans: A Ans: C
129) Accessory cervical nerve causing
numbness and pain the hand, the
SK Origional – Golden 11 215

compression is at which root of a. Pigmented epithelium


brachial plexus? b. Rods
a. C5 c. Bipolar cells
b. C6 d. Amacrine cells
c. C7 Ans: A
d. C8 136) The metastasis involves:
e. T1 a. Integrin attachment
Ans: E b. ICAM-I
130) A young boy was presented by mother c. Loss of E-Cadherin
with a complaint of rash starting from d. Gain of E-Cadherin
the side of the face (ears) and Ans: C
spreading down to trunk. Diagnosis: 137) The structure damaged above carpal
a. ITP tunnel:
b. Measles a. Median nerve
c. Toxoplasmosis b. Ulnar nerve
d. Rubella c. Radial nerve
Ans: B d. Radial artery
131) Tube passed into the interstitial space Ans: B
for pleurocentasis via the lower border 138) The virus involved in spread of
of rib. The structure that would Nasopharyngeal carcinoma:
damage first: a. CMV
a. Intercostal nerve b. EBV
b. Intercostal artery c. HSV
c. Intercostal vein d. HPV
d. Both nerve and vein Ans: B
Ans: A 139) The Hemophilia trait lacks:
Remember:From upper border to lower border a. Factor 8 major components
(van) – vein then artery then nerve. b. Factor 8 small components
132) A diabetic patient with a draining c. Factor 9
sinus/pus. What will histology show? d. Factor 12
a. Fibrosis Ans: B
b. Granuloma with abscess 140) What is the investigation of choice for
c. Serosis a diabetic patient with nephropathy?
d. Increase Vascularity a. Microalbuminemia
Ans: B b. Inulin
133) V/Q ratio in Pulmonary Embolism c. Creatinine
would be: d. Urea
a. Zero Ans: A
b. Increased 141) A 5-year-old boy comes to the clinic
c. Decreased with recurrent cervical
d. Infinity lymphadenopathy. On each occasion
Ans: D klabseilla pneumonia is isolated. He is
134) A patient with mid diastolic murmur most likely suffering from which
and Increased Pulmonary Wedge immunodificient state:
Pressure alongside pulmonary a. Bruton‘s agamaglobinemia
hypertensive, he could have: b. Severe combined agamaglobinemia
a. LVH c. Chronic granulomatous disease
b. RVH d. Common variable agamaglobinemia
c. Septal perforation Ans: C
d. Aortic dissection 142) A point that distinguishes partial
Ans: B transection of the spinal complete
135) The outermost retinal layer is: transection:
SK Origional – Golden 11 216

a. Spinal shock c. Pudendal nerve


b. Bladder atonia d. Obturator nerve
c. Increase reflexes Ans: A
d. Diminish reflexes 150) A NID – Diabeticlady after 8th Post-op
Ans: A day presented with dyspnea. What
143) The possible side effect of levodopa is: could be the cause?
a. Constipation a. Pulmonary embolism
b. Dry mouth b. Polycythemia
c. Orthostatic hypotension c. Pneumonia
d. Hypertension d. TB
Ans: C Ans: A
144) Patient presents with dry eyes and dry 151) Pituitary adenoma invades which
mouth, what investigation you would bone?
prefer to reach the diagnosis? a. Occipital
a. Anti-Ds-DNA b. Parietal
b. ANA c. Temporal
c. Anti-SSA d. Sphenoid
d. Anti-mitochondrial antibodies Ans: D
Ans: C 152) A fine touch and skilled movements
145) A patient took antimalarial from a executed by an artist is all due to:
quack in his village after sometime he a. Rubrospinal tract
presents to the doctor with cola- b. DCML
coloured urine, the diagnosis is: c. Anterior spinothalmic tract
a. Sickle – cellanemia d. Corticospinal tract
b. G6PD deficiency Ans: D
c. Spherocytosis 153) A patient is suffering from ecchymosis
d. Normal presentation and gum bleeding, there is deficiency
Ans: B of:
146) Drugs involved in causing a. Vitamin A
thrombocytopenia: b. Vitamin C
a. Thiazides c. Vitamin K
b. Furosemide d. Niacin
c. Metformin Ans: B
d. Penicillin 154) The deficiency of surfactant will
Ans: A cause:
147) Auto-immune disease is manifested a. Respiratory distress syndrome (RDS)
by: b. Emphysema
a. Hematological studies c. Pneumonia
b. Arthritis d. Pulmonary congestion
c. Rashes Ans: A
d. Diarrhea 155) What is the feature of smooth
Ans: A muscles?
148) During osteogenesis: a. Tetanizes early
a. Osteoblasts secretes ALP b. High action potential
b. Osteocytes secrets ALP c. Sustained contraction
c. Increase of serum calcium d. Striated
d. Osteoclasts are bone-forming cells Ans: C
Ans: A 156) A patient is suffering from End Stage
149) Waddling gait in a child is owing to Renal Disease (ESRD), he has had
the damage of: repeated transfusions, what treatment
a. Superior gluteal nerve should be given to reduce the rate of
b. Inferior gluteal nerve transfusions:
SK Origional – Golden 11 217

a. Subcutaneous renin b. Shivering


b. Subcutaneous erythropoietin c. Nothing
c. Subcutaneous Vitamin K d. Increase HR
d. FFP Ans: B
Ans: B 163) Vibrations are lost, the receptors
157) Blood supply of the upper 1/3rd of the responsible are:
esophagus: a. Merkel‘s discs
a. Inferior thyroid artery b. Free nerve endings
b. Branches from descending thoracic aorta c. Pacinian
c. Left gastric artery d. Baroreceptors
d. Superior thyroid artery e. Krause
Ans: A Ans: C
Remember: 164) Hypophyseal structure receiving
signals from hypothalamus via
Upper 1/3rd Inferior thyroid artery
hypophyseal portal system:
Middle 1/3rd Thoracic descending aorta a. Neurohypophysis
branches b. Adenohypophysis
Lower 1/3rd Left gastric artery c. Pericytes
d. Infundibulum
158) RBCs are kept in a solution with Ans: B
hyperosmolar state, containing urea 165) Renin is secreted by:
which is permeable to RBC. What will a. Endothelial cells of afferent arterioles
happen? b. JG cells
a. Cell will shrink c. PCT
b. Cell will swell and lyse d. DCT
c. Cell will shrink and lyse e. Collecting ducts
d. Cell will shrink then swell and then lyse Ans: B
Ans: D 166) Which of the following drugs
159) A patient was having epigastric pain increases concentration of other
for months for which he was taking drugs?
antacids and now presents with a. Ethosuximide
weight loss. What is the cause? b. Phenobarbital
a. H.Pylori c. Benzodiazepines
b. EBV d. Sodium valproate
c. Hepatitis C Ans: D
d. CMV 167) A patient presented with multiple
Ans: A bone fractures, first measure to be
160) What is the function of dead space? taken is:
a. Exchange of gases a. IV fluids resuscitation
b. Blockage of gases b. BP monitoring
c. Conditioning of air c. Surgical procedure
d. Increases air entry d. Pulse monitoring
Ans: C Ans: A
161) The function of pacinian corpuscle is: 168) Aspirin toxicity causes:
a. High frequency vibration a. Respiratory alkalosis
b. Low frequency vibration b. Respiratory acidosis
c. Two points discrimination c. Metabolic alkalosis
d. Extreme pain d. Doesn‘t affect acid-base level
Ans: A Ans: A
162) The core body temperature is Remember:Aspirin toxicity initially causes
decreased what will happen? Respiratory alkalosis then metabolic acidosis.
a. Sweating
SK Origional – Golden 11 218

169) The predisposing factor of


mesothelioma is: 5) Loss of sympathetic supply to the eyes
a. Silicosis would result in:
b. Carbon A) Tears
c. Asbestosis B) Meiosis
d. Anthracnose‘s C) Drooping eyelids
Ans: C D) Squint
Ans: B
6) Jejunum nerve supply is via:
=============== A) Greater splanchnic nerve
B) Lesser splanchnic nerve
C) Greater and lesser splanchnic nerves
MEDICINE & ALLIED D) Least splanchnic nerve
November 26th, 2020 – Morning Ans: C
7) Sensory supply to the eyes is lost and
1) Mitochondria are abundant in the the jaw movements are intact. Which
apical end of: of the following has undergone lesion?
A) Cardiac cells A) Sensory nucleus of trigeminal nerve
B) Non-ciliated cells B) Optic nerve nucleus
C) Ciliated cells C) Occulomotor nerve
D) Nephrons D) Trochlear nerve
Ans: C Ans: A
2) What is the amount of energy released 8) An old patient presents with a mask
by 1 molecule of glucose? like face and resting tremors, which of
A) 36 ATPs the following brain area is involved?
B) 38 ATPs A) Caudate
C) 37 ATPs B) Globus pallidus
D) 2 ATPs C) Thalamus
Ans: B D) Nucleus basalis
3) A 10-year-old boy gradually developed Ans: B
a disturbed behavior. His temperature 9) Parkinsonism affects:
rose to 100 degree centigrade and has A) Nucleus Basalis
developed convergent squint. What is B) Substantia Nigra
the diagnosis? C) Subthalamus
A) Subsclerosing panencephalitis D) Globus Pallidus
B) Tuberos sclerosis Ans: B
C) TBM 10) A patient presented to the emergency
D) Frontal lobe tumor room with injury to scalene anterior
Ans: C muscle and 1st cervical rib. The radial
4) pH of 7.2 , HCO3 levels of 19 and pulse is feeble on examination. The
PCO2 of 35 is the state of: damage is to which blood vessel:
A) Metabolic alkalosis A) Aorta
B) Respiratory acidosis B) IVC
C) Metabolic alkalosis C) SVC
D) Metabolic acidosis D) Internal Jagular Vein
Ans: D E) Subclavian vein
Remember: Ans: E
Remember:Damage to first rib; always bring
subclavian vein into your mind.
11) Neck‟s muscle of demarcation:
A) Scalene posterior
B) Serratus Anterior
SK Origional – Golden 11 219

C) Trapezius Sour + salty + Facial nerve CN-vii


D) Sternocleidomastoid sweet
Ans: D Bitter Glossopharyngeal nerve
12) A young patient with condylar fracture CN-ix
and as a result lost the abduction and
extension of thumb and index finger. 18) The nerve that causes dorsiflexion of
Damage is to which nerve? the foot:
A) Radial nerve A) Popliteal nerve
B) Ulnar nerve B) Deep peroneal nerve
C) Median nerve C) Tibial nerve
D) Musculocutaneous nerve D) Saphenous nerve
Ans: A E) Superficial peroneal nerve
13) Pulmonary edema at the base of the Ans: B
lung is due to: 19) A patient suffering from pneumonia,
A) Increase interstitial pressure had green sputum on laboratory examination.
B) Increase air pressure What do you think is the causative agent?
C) Decrease interstitial pressure A) Legionella
D) Increase intravascular pressure B) Klebsiella
Ans: D C) Pseudomonas
14) A patient feels sensation of numbness D) Psittocosis
in his extremities. His labs reveal Hb Ans: C
level 8g/dl, MCV 110 fl, Platelet count Remember:
50x10^9/L and TLC 3.4X10^3/L.
What is the diagnosis? Pneumonia + chest Sreprococcal
A) Pernicious anemia pain + pleuratic pain Pneumonia
B) Sickle cell anemia Pneumonia + Normal Atypical
C) Normocytic anemia TLC pneumonia
D) Megaloblastic anemia Pneumonia + Psittocosis
Ans: D Splenomegaly
15) An anemic patient with abdominal Pneumonia + Diarrhea Legionella
pain. An organism responsible is: pneumonia
A) Ancylostoma duodenale Pneumonia + Green Pseudomonas
B) Entamoeba histolytica sputum
C) Cryptococcus Pneumonia + Foul Anerobic
D) Staph Aureus smell sputum
Ans: A Pneumonia + Bilateral Talaremia
16) A patient with lateral spinothalamic Lymphadenopathy
tract lesion would present with: Pneumonia + Klebsiella
A) Pain and temperature lost at same side Chocolate colour
B) Pain and temperature lost at opposite sputum +
sides alcoholic/diabetic
C) Pain and temperature intact on both sides Pneumonia + lung Staph aureus (most
D) Sense of vibration lost on opposite side abscess common)
Ans: B
17) Damage to the Cranial nerve IX (9) 20) The release of growth hormone is
the taste lost would be? stimulated by:
A) Sweet A) Sleep
B) Sour B) Exercise
C) Salty C) Hypoglycemia
D) Bitter D) Rest
Ans: D Ans: C
Remember: Explanation:
SK Origional – Golden 11 220

 Hypoglycemia > Exercise > Sleep C) Ras


21) Lipoproteins are taken by cells in the D) BCR
form of: Ans: A
A) LDL 28) The minor calyces of the kidney
B) HDL receive urine from:
C) Albumin bound A) Renal medulla
D) Free fatty acids B) Renal Papillae
E) Chylomicrons C) Renal pelvis
Ans: E D) Major calyces
22) A patient presents with cough. A CXR Ans: B
was advised and on x-ray there were 29) The brain structure embedded in
apical nodules and opacities along lateral hemisphere:
with air fluid levels. Which organism A) Splenium
causes this? B) Operculum
A) Legionella C) Insula
B) Staph Aureus D) Crus
C) Pseudomonas Ans: C
D) Psittocosis 30) The calcarian sulcus is involved in
Ans: B which sensations?
23) Which of the following anatomical A) Visual
structures is present anteriorly to the B) Olfactory
trachea? C) Motor tone
A) Phrenic nerve D) Coordination
B) Recurrent Laryngeal nerve Ans: A
C) Internal Jagular vein 31) Hypoglycemia in ECG is represented
D) Arch of aorta by:
E) Vagus nerve A) PR Interval shortening
Ans: D B) ST Elevation
24) The maturation of T cells occur in: C) Inverted T wave
A) Bone Marrow D) QT Prolongation
B) Trachea Ans: D
C) Thymus 32) S2 heart sound is produced during:
D) Payers patches A) Mid diastolic snap
Ans: C B) Isovolumetric contraction
25) The high altitude would result in: C) Isovolumetric relaxation
A) Increase 2,3, BPG D) Rapid ventricular filling
B) Decrease Oxygen affinity Ans: C
C) Increase oxygen affinity 33) The part of the CNS responsible for
D) No change shall occur temperature monitoring and control:
Ans: B A) Thalamus
26) The strongest layer of the small B) Mid brain
intestine is? C) Spinal cord
A) Circular muscular layer D) Hypothalamus
B) Mucosa Ans: D
C) Sub mucosa 34) H/K+ Pump in the stomach is
D) Serosa inhibited by:
E) Longitudinal layer A) Cimetidine
Ans: C B) Ranitidine
27) An example of tumor suppressor gene C) Omeprazole
is: D) Metronidazole
A) p53 gene Ans: C
B) C-myc
SK Origional – Golden 11 221

35) What is the side effect of Tricyclic A) Small cell carcinoma


Antidepressants (TCA)? B) HCC
A) Generalized tonic clonic seizures C) RCC
B) Vomiting D) Melanoma
C) Increase in BP Ans: A
D) Decrease Heart Rate 43) A hypertensive patient who was on
Ans: A medication for it presents with altered
36) Nerve with only afferent fibers: mental state and his Potassium level
A) Occulomotor nerve was 6mmol/l, what is the cause of his
B) Facial nerve condition?
C) Trochlear nerve A) Diuretics
D) Optic nerve B) Chronic Renal Failure
Ans: D C) ARF
37) A chronic renal failure patient with D) Hypertension
anemia requires: Ans: C
A) Subcutaneous erythropoietin 44) An 70-year-old obese man after
B) Vitamin K surgery is taking IV Gentamycin in a
C) Vitamin B12 BID dose. What mechanism will be
D) Intrinsic factor affected?
E) Folate A) Excretion
Ans: A B) Distribution
38) Tachypylaxis explains: C) Absorption
A) Gradual decrease in response to drug D) Metabolism
B) Rapid decrease in response to drug Ans: A
C) No response to drug 45) In forceful flexion of the neck, which
D) Enhance response to drug ligament is torn?
Ans: B A) Anterior longitudinal
39) Anaphylaxis to drug is treated by: B) Posterior longitudinal
A) IV Fluids C) Ligamentum Nuchae
B) Nebulization D) Ligamentum Flavum
C) Oxygen administration Ans: C
D) IV Adrenaline 46) Micturition is controlled by:
Ans: D A) Mechanoreceptors in bladder wall
40) A patient presented with severe acute B) Inhibited by a center in pons
asthma, what should be administered C) Stretch of urethral sphincter
to him? D) Condition reflex
A) IV histamine E) Processed in cerebral cortex
B) Oral prednisolone Ans: A
C) Salbutamol nebulization Remember:
D) IV Adrenaline
Micturition center Mid brain
Ans: C
41) Severe acute ulcerative colitis (UC) is Micturition reflex Sacral spinal cord
treated by: Controlled by Mechanoreceptor
A) Sulfasalazine
B) IV Steroids 47) Leucovorin is an antidote for:
C) Methotrexate A) Sulfasalazine
D) NSAIDs B) Potassium
Ans: B C) Methotrexate
Remember: If Mild to moderate UC then  D) Vitamin K
Sulfasalazine. Ans: C
42) Which of the following is para- 48) Alpha adrenergic receptors presence:
neoplastic syndrome? A) Bronchodilation
SK Origional – Golden 11 222

B) GI constrictions D) Slow filling of ventricles


C) Renal Ans: A
D) Eyes 56) The presence of the small salivary
Ans: D glands is in which layer of oral
49) The muscle of mastication involved in mucosa?
yawning and increasing cardiac A) Lamina rata
output: B) Lamina Propria
A) Lateral Pterygoid C) Lamina Lucida
B) Medial Pterygoid D) Submucosa
C) Temporalis Ans: B
D) Masseter 57) The para renal fats is extension of:
Ans: A A) Pretracheal fascia
50) A female patient with immunological B) Preverterbral fascia
diseases and lymphoma and cervical C) Renal Fascia
lymphadenopathy, the condition is: D) Fascia lata
A) Infectious Mononucleosis Ans: C
B) AIDS 58) A patient with blood loss has increase
C) TB heart rate while on standing, this
D) Sarcoidosis particular compensatory response is
Ans: D> B due to?
51) Which of the following arteries lie A) Decrease venous return to heart
behind the sternum? B) Decrease peripheral resistance
A) Carotid artery C) Decrease CO
B) Subclavian artery D) Increase TPR
C) Internal thoracic artery Ans: A
D) Intercostal artery 59) The drug of choice in bacterial
Ans: C (meningococcal) meningitis:
52) When does the secondary oocyte A) Penicillin
completes its maturation? B) Ceftriaxone
A) At fetal life C) Cefotaxime
B) At puberty D) Ciprofloxacin
C) At ovulation Ans: B
D) At fertilization Remember: DOC overall is Ceftriaxone while in
Ans: D neonates it is Cefotaxime, so ponder on scenario
53) The intracellular structure composed and read the age carefully.
of microtubules: 60) Submental and submandibular lymph
A) Centromere nodes drain:
B) Centrioles A) Upper lip
C) Golgi Apparatus B) Lower lip
D) Endoplasmic reticulum C) Tongue
Ans: B D) Nose
54) A penetrating injury to the ischium Ans: B
would result in injury to: 61) The developmental origin of
A) Anal canal pancreatic alpha, beta and delta cells
B) Prostate is from:
C) Rectum A) Endoderm
D) Bladder B) Mesoderm
Ans: C C) Splanchnic mesoderm
55) The S2 splitting sound increases with: D) Ectoderm
A) Late closure of pulmonic valve Ans: A
B) Early closure of pulmonic valve 62) Grasp reflex in babies is present up to:
C) Rapid filling of ventricles A) By the end of 1st postnatal month
SK Origional – Golden 11 223

B) By the end of 3rd postnatal month 69) A hypertension patient presented with
C) By the end of 6th postnatal month vision problems and was diagnosed as
D) Early in 6th postnatal month a Grade 3 retinopathy. Which of the
Ans: C following is the target of hypertension
63) A data is arranged based on presence in retina?
or absence, this is: A) Vein
A) Nominal data B) Arteries
B) Ordinal data C) Retina nerve fibers
C) Interval data D) Arterioles
D) Ratio data E) Venules
Ans: A Ans: D
64) pH level that decreases upon 70) A patient exhibiting FEV1 84% on
10mmHg increase in CO2: spirometer, what is his condition?
A) 0.04 A) Normal lung
B) 0.01 B) Increased lung compliance
C) 0.1 C) Decreased lung compliance
D) 1 D) Emphysema
Ans: D Ans: A
Explanation: Decreases by 0.08 so 1 here is the 71) A patient presents with galactorrhea
closest answer. and amenorrhea, his labs reveal a
65) A patient with labs showing decreased prolactin level of 6500ng/ml. This
calcium level, increased phosphate condition best describes:
level and increased PTH is a case of: A) Micro adenoma
A) Acute renal failure B) Macro adenoma
B) Renal tubular acidosis C) Normal condition
C) CRF D) Pituitary adenoma
D) RCC Ans: B
Ans: C 72) A patient exhibiting
66) A patient presents with tingling hyperprolactinemia signs with
sensation and spastic contraction, he decreased vision loss has a problem
is a case of: in:
A) Vitamin B12 deficiency A) Hypothalamus
B) Tetany B) Thalamic nuclei
C) UMN Lesion C) Pituitary
D) LMN Lesion D) Occulomotor nerve
Ans: B Ans: C
67) A diuretic of choice in pulmonary 73) A damage to the right optic chiasm
edema is: would result in which kind of vision
A) Loop diuretic loss?
B) K+ Sparing diuretic A) Bitemporal Hemianopia
C) Thiazides B) Right Homonymous Hemianopia
D) Spironolactone C) Left Homonymous Hemianopia
Ans: A D) No vision impairment
68) Complication of diabetes mellitus: Ans: A
A) Wet gangrene 74) Ultra violet (UV) radiation is involved
B) Dry gangrene in causing:
C) Caseous necrosis A) Teratogenesis
D) Apoptosis B) Deafness
Ans: B C) Cataract
Remember: DD – FW D) Telangiectasia
Complication of DM Dry gangrene Ans: C
Complication of Diabetic Foot Wet gangrene.
SK Origional – Golden 11 224

75) A diabetic patient with a draining D) Eosinophil


sinus/pus. What will histology show? Ans: D
A) Fibrosis 82) An analgesic that has bronchodilator
B) Granuloma with abscess effect:
C) Serosis A) Paracetamol
D) Increase Vascularity B) Tramadol
Ans: B C) Ketamine
76) The Type-IV hypersensitivity reaction D) Pethidine
involves macrophages and: Ans: C
A) B-cells Remember:Post-op asthmatic analgesic of choice
B) Neutrophils is pethidine.
C) Eosinophil 83) The shock associated with pelvic
D) T-cells fracture is owing to:
Ans: D A) Blood loss
77) A patient presents with cough, non B) Fat embolism
caseating granuloma and hilar C) Anaphylaxis
lymphadenopathy, this condition is of: D) Cardiogenic shock
A) TB Ans: A
B) RCC 84) The facilitated diffusion in intestine
C) CRF suits with which one of the following?
D) Sarcoidosis A) Glucose
Ans: D B) Galactose
78) In congenital rubella termination of C) Sucrose
pregnancy is indicated if one of the D) Fructose
following is positive: Ans: D
A) IgG 85) The ulnar nerve supplies:
B) IgA A) Medial 1/3rd of the palmar hand
C) IgM B) Medial 1/3rd of the dorsal hand
D) IgD C) Medial 1/3rd of the palmar and dorsal
Ans: C hand
Explanation: IgM shows acute infection and if it D) Medial 2/3rd of the palmar hand
is positive during the period of organogenesis it is Ans: C
an indication of termination of pregnancy. Remember:
79) An inorganic substance involved in
causing cancer:
A) Asbestos
B) Iron
C) Silica
D) Zinc oxide
Ans: A
80) The abscess along with boils is mainly
composed of:
A) Macrophages
B) Neutrophils
C) Dead bacteria
D) Eosinophil
Ans: B (Dead Neutrophils)
81) Which of the followings is elevated in
nasal polyps?
A) Macrophages 86) PPD Skin test is which type of
B) Neutrophils hypersensitivity?
C) Monocytes A) Type – I
SK Origional – Golden 11 225

B) Type – II A) Normal variant


C) Type – III B) Leukoplakia
D) Type – IV C) Erythroplakia
E) Cell – Mediated D) Barret Esophagus
Ans: D Ans: A
Explanation: PPD skin test is used for 94) Chlamydia is cause by:
Tuberculosis and it is Type-IV hypersensitivity. A) Bacterial vaginosis
remember it is not type-III as discussed by few in B) Lymph granuloma venerum
FCPS Prep.Batch (2005-10) group. C) Gonorrhea
87) What lies in front of the right kidney? D) Syphilis
A) Spleen Ans: B
B) Liver 95) A person who had a huge blow to the
C) 1st part of the duodenum temporal portion of the head and as a
D) 2nd part of the duodenum result blood got accumulated in the
Ans: D epidural space, the vessel ruptured
88) Most important opsonin for would be:
combating acute infection: A) ASA
A) Fab portion of IgG and C3b B) MMA
B) FC portion of IgG and C3b C) PCA
C) IgG D) ACA
D) C3b Ans: B
Ans: B 96) When to breach patient‟s
89) Important antigen that is involved in confidentiality?
transplantation: A) Whenever needed
A) MHC B) When patient allows
B) HLA C) With his family
C) Rh factor D) Should be kept forever
D) AB blood antigen Ans: B
Ans: B Remember: Click HIV/STD/TB or other
90) Which of the following is a pain dangerous disease if in option.
mediator in inflammation? 97) Secondary cartilaginous joints
A) Bradykinin contain:
B) Prostaglandin A) Fibrocartilage in between
C) Histamine B) Hyaline cartilage
D) C3b C) Elastic cartilage
Ans: A D) No cartilage
91) Which of the following is Ans: A
ketoacidosis? 98) The portion of brain lesion that would
A) Lactic acid result in fluent aphasia:
B) Salicyclic acid A) Wernicke‘s Area
C) Carbonic acid B) Broca‘s Area
D) Acetoacetate C) Premotor cortex
Ans: D D) Somatosensory association area
92) Acetyl CoA used which vitamin as a Ans: A
cofactor? Remember:
A) Niacin
Non-fluent aphasia Broca‘s area (broken
B) Thiamine
words)
C) Biotin
D) Riboflavin Fluent aphasia Wernicke‘s area
Ans: C 99) Most important function of
93) Squamocolumnar junction at distal progesterone?
1/3rd of the esophagus with stomach: A) Breast alveolar duct development
SK Origional – Golden 11 226

B) Decrease uterine contraction in pregnancy C) Aorta


C) Secretory phase of endometrium D) Femoral artery
D) Maintains thickness of endometrium E) Radial artery
E) Deposition of fat Ans: C
Ans: D 108) A diabetic patient with abdominal
Remember: D>C pain, severe vomiting and visible
100) A patient who has undergone ileum peristalsis and audible sound on
resection, should be given: examination, the condition is:
A) Vitamin B12 A) GERD
B) Iron B) Achalasia
C) Vitamin A C) Crohn Disease
D) Vitamin E D) Pyloric stenosis
E) Vitamin C Ans: D
Ans: A 109) The venous return is directly
101) A patient complains of hearing sound proportional to:
better in a noisy environment than a A) Intrathoracic pressure
quite one, what is the cause? B) Intra-abdominal pressure
A) Foreign body C) Jagular venous pressure
B) Osteoma D) TPR
C) Osteosclerosis Ans: B
D) Traumatic bone cyst 110) A patient presents to the clinic with
Ans: C polyuria, polydipsia and urine
102) Cell swelling that occurs in reversible Osmolality of 310 mosm, he is a case
injury is owing to: of multiple myeloma, what is the
A) Fat accumulation cause of polyuria in him?
B) Inward water flow A) Renal tubules do not respond to ADH
C) Calcium entrance to cells B) Psycogenic polydipsia
D) Na-K Pump malfunctioning C) Decreased ADH
Ans: B D) Increased aldosterone
103) Blood loss during C-Section would Ans: A
result in: 111) The difference between
A) Infarction hydrocortisone and betamethasone
B) Atrophy of adrenal cortex lies in:
C) Atrophy of medulla A) Potassium sparing
D) Renal atrophy B) Sodium retention
Ans: B C) ADH Increase
105) When is atrial Repolarization visible D) Aldosterone decrease
on ECG? Ans: B
A) 1st Degree block 112) A 40 year old female with no
B) 2nd Degree block respiratory or cardiac problems, has
C) 3rd Degree block swelling in front of the neck and feels
D) Mobitz block difficulty with breathing while in
Ans: C supine position; this is due to:
106) PR Interval coincides with: A) Asthma
A) Filling of atria B) Colloid goiter
B) Closure of mitral valve C) Follicular carcinoma of goiter
C) Filling of ventricles D) Retrosternal goiter
D) Closure of tricuspid valve Ans: D
Ans: B 113) Somatosensory loss is due to lesion in:
107) Most common site of atherosclerosis: A) Area 3,2,1
A) IVC B) Area 5
B) SVC C) Area 6
SK Origional – Golden 11 227

D) Area 22 D) Biotin
Ans: A Ans: C
114) Arterial blood as compared to venous 121) Infectious mononucleosis is
blood has: diagnosed by:
A) Increase PO2 A) ANA
B) More packed cell volume B) Antimitochondrial antibody
C) Decrease bicarbonate C) Antimicrosomal antibody
D) Increase chloride D) Heterophile sheep antibody
Ans: A Ans: D
115) A patient with low level of thyroid 122) A female presented with menorrhagia
hormones, TSH level is also low but and her platelet count is 30,000 mcL,
increases when TRH is administered which antibody test should be done?
to the patient. The structure most A) ANA
likely damaged: B) Antimitochondrial antibody
A) Thyroid gland C) Antimicrosomal antibody
B) Pituitary D) Heterophile sheep antibody
C) Hypothalamus E) Antiplatelet antibody
D) Adrenal gland Ans: E
Ans: C Explanation: She is a case of ITP.
116) Intraembryonic coelom is derived 123) Starting a conversation with a patient
from: should initiate with:
A) Lateral plate mesoderm A) Asking name, age and address
B) Intermediate mesoderm B) Leading question
C) Endoderm C) Open ended questions
D) Ectoderm D) Sympathetically approach
Ans: A Ans: A
117) The time period of absolute refractory 124) The auscultation area of mitral valve
resting period of heart: is:
A) 200-300 msec A) Left 5th intercostal space at midclavicular
B) 150-200 msec line
C) 10-20 msec B) Right 5th intercostal space at midclavicular
D) 50-60 msec line
Ans: B C) 2nd intercostal space
118) Insulin is secreted by which cells of D) 4th intercostal space at midaxillary line
the pancreas: Ans: A
A) Delta cells 125) Vitamin A prophylaxis is an example
B) Alpha cells of:
C) Beta cells A) Primordial protection
D) Parenchymal cells B) Specific protection
Ans: C C) Health promotion
119) Pellagra is owing to: D) Disability limitation
A) Thiamine deficiency Ans: B
B) Niacin deficiency 126) Low depolarization occurs in:
C) Riboflavin deficiency A) Hyponatremia
D) Biotin deficiency B) Hypokalemia
Ans: B C) Hyperkalemia
120) A 35 week primigravida female, she D) Hypercalcemia
takes iron supplements only, which Ans: A
deficiency might occur in her? 127) Receptors present on finger tips?
A) Vitamin D A) Pacinian corpuscles
B) Vitamin K B) Meissners corpuscle
C) Folic acid C) Krause bulb
SK Origional – Golden 11 228

D) Raffini receptors 134) ECF Volume increases maximum on


Ans: B giving:
128) Which of the following conditions A) Hypotonic saline
involved lymphatics? B) Hypertonic saline
A) Aneurysm C) Hypertonic dextrose
B) Cystic Hygroma D) Ringer lactate
C) Haemangioma Ans: B
D) Teratoma 135) What is the rationale behind giving
Ans: B anti-TB drugs in combination?
129) Staging is done for: A) For better potency
A) Degree of differentiation B) Increase absorption
B) Extent of spread C) Increase half life
C) Benign tumor D) To prevent resistance
D) Recovery Ans: D
Ans: B 136) An organ with sympathetic
Remember: cholinergic muscarinic receptors:
GD: Grading for Differentiation. A) Tear gland
SS: Staging for Spread. B) Sweat gland
130) Which part of the kidney is damaged C) Gut
in severe blood loss? D) Adrenal medulla
A) PCT Ans: B
B) DCT 137) Epigastric pain is transmitted via:
C) Collecting tubules A) Greater splanchnic nerve
D) Loop of Henle B) Lesser splanchnic nerve
Ans: A C) Least splanchnic nerve
131) Decussation of motor fibers occur at: D) Phrenic nerve
A) Pons Ans: A
B) Mid Brain 138) Temporomandibular dislocation has
C) Medulla paralysis of which muscle?
D) Cerebellum A) Medial pterygoid
Ans: C B) Lateral Pterygoid
132) Hormone involved in conversion of C) Buccinator
testosterone to 17 Beta estradiol in D) Masseter
granulosa cells: Ans: B
A) LH 139) Ebstein barr virus is associated with:
B) FSH A) Cervical carcinoma
C) HCG B) Erythroplakia
D) Estradiol C) Lichen Planus
Ans: B(Prefer Aromatase) D) Burkitt lymphoma
133) An RH negative female who gave Ans: D
birth to an RH negative baby. She had 140) What is the earliest manifestation of
a history of hemolytic reactions. Due vitamin A deficiency?
to puerperium blood loss, she was A) Conjuctival dryness
transfused an RH positive blood. B) Bitot spot
Which option suits her best? C) Night blindness
A) Transfusion reaction D) Mouth dryness
B) Hemolytic reaction might ensue in future Ans: C
pregnancies 141) A 6 year old child with history of joint
C) No need to give Anti-D antibody swelling and bleeding during
D) Give anti-D antibody Immedietly circumcision. Which test should be
Ans: D done to confirm the diagnosis?
A) aPTT
SK Origional – Golden 11 229

B) Factor 9 assay 148) A child presents with frontal bossing


C) Factor 8 assay and bowing of legs. The deficiency of
D) PT which of the following substance
Ans: C better explains him?
Explanation:In hemophilia, initial test is aPTT A) Calcium
while confirmatory is Factor 8 assay. Please! Do B) Vitamin D
not confuse. C) Zinc
142) Most common congenital bleeding D) Vitamin A
disorder: E) Vitamin K
A) Factor V laden Ans: B
B) vWD 149) What is true about a new born?
C) Hemophilia A) Lower limbs are stronger than upper
D) ITP limbs
Ans: B B) Can hold neck
143) What should be given to a patient with C) C-Shaped vertebral column
factor 8 deficiency? D) All the four limbs are equal in size
A) Whole blood Ans: C
B) Fresh Frozen Plasma (FFP) 150) The amplification of nmyC gene
C) Normal saline causes:
D) Cryoprecipitate A) Retinoblastoma
Ans: D B) Nephroblastoma
144) Renin is produce by which part of the C) Hodgkin lymphoma
kidney: D) Neuroblastoma
A) Peritubular capillaries Ans: D
B) JG Cells 151) A female with fat deposition at the
C) DCT side of the neck and short stature:
D) Ascending loop of Henley A) 45 XO
Ans: B B) 46 XX/XXY
145) Which of the following proteins C) 47 XY
maintain the biconcave shape of the D) 46 XO
RBC? Ans: A (turner syndrome)
A) Spectrin 152) Blood supply of the proximal 1/3rd of
B) Band 3 the esophagus:
C) Ankyrin A) Inferior thyroid artery
D) C3b B) Branches from descending thoracic aorta
Ans: A C) Left gastric artery
146) The difference of ECF from ICF lies D) Superior thyroid artery
in the fact that: Ans: A
A) Anions are inorganic Remember:
B) Decrease PH
Upper 1/3rd Inferior thyroid artery
C) Increase Volume
D) Hypo tonicity Middle 1/3rd Thoracic descending aorta
Ans: A branches
147) A buffer that is exclusive to kidney: Lower 1/3rd Left gastric artery
A) Ammonia 153) Hepatitis B spreads through:
B) Phosphate A) Coughing
C) Bicarbonate B) Sneezing
D) Hemoglobin C) Sexual contact
Ans: A D) Breast feeding
Remember: Ammonia is present only in kidney Ans: C
and no other bodily fluid as a buffer. 154) A leukemia patient who underwent a
successful BMT, now presents with
SK Origional – Golden 11 230

scaly rash on arm with increase in Hb A) Always involves hormone


and platelets, this condition explains? B) Depends on nutrients
A) Anaphylactic reaction C) Brings the concentration back to the
B) Type-II Hypersensitivity mean
C) Graft Vs Host rejection D) Follows positive feedback
D) Type-III HSR Ans: C
Ans: C 162) Crohn disease differs from Ulcerative
155) Mass in ligamentum arteriosum would colitis in:
compress: A) Clubbing
A) Vagus nerve B) Severity
B) Phrenic nerve C) Perianal lesions
C) Right recurrent laryngeal nerve D) Necrosis
D) Left recurrent laryngeal nerve Ans: C
Ans: D 163) Transfer of O2 from alveoli to
156) Muscles have the following capillaries involves which mechanism?
intermediate filament: A) Active transport
A) Desmin B) Passive diffusion
B) Vementin C) Facilitated diffusion
C) Keratin D) Secondary diffusion
D) Laminin Ans: B
Ans: A 164) Absence of ganglion cells occur in
157) The level of PTH is usually raised in: which of the following conditions?
A) Precocious puberty A) Hirschprung‘s disease
B) Pseudohypoparathyroidism B) Toxoplasmosis
C) Carcinoma C) Ulcerative Colitis
D) CCF D) IBD
E) Adrenal failure E) Crohn‘s disease
Ans: B >C Ans: A
158) A patient with loss of touch sensation
on face while pain and temperature
being intact, which nucleus lesion is ===============
involved?
A) Primary sensory nucleus
B) Spinal nucleus
C) Mesencephalic nucleus
D) Thalamic nuclei
Ans: A
159) Corneal opacity is the adverse effect
of:
A) Ciprofloxacin
B) Chlorpromazine
C) Diazepam
D) Chloroquine
Ans: D
160) Left to right shunt before reversal
occurs in:
A) Eisenmenger Syndrome
B) Transportation of greater arteries
C) Truncus arteriosus
D) Coarctation of aorta
Ans: A MEDICINE & ALLIED
161) Regarding negative feedback: November 26th, 2020 – Afternoon
SK Origional – Golden 11 231

1) What type of necrosis is seen in Folate (Vitamin B9) Vegetables


omentum? Vitamin B12 Animals
A) Fat necrosis 8) Most remarkable and profound effect
B) Coagulative necrosis on the heart by an extracellular
C) Fibrinoid Necrosis presence of:
D) Liquefactive Necrosis A) Sodium
E) Medial necrosis B) Calcium
Ans: A C) Potassium
2) The semilunar valve opening is seen D) Chloride
at the beginning of: Ans: C
A) Reduced ejection 9) Nerve at risk while doing surgery at
B) Isovolumetric contraction the lower pole of thyroid:
C) Isovolumetric relaxation A) Superior laryngeal nerve
D) Rapid ejection B) Inferior laryngeal nerve
Ans: D C) External laryngeal nerve
3) Blood supply of the thalamus is by: D) Recurrent laryngeal nerve
A) ACA Ans: D
B) PCA Remember:
C) ASA
D) MCA Pole Artery Nerve
Ans: B Upper Superior thyroid External laryngeal
4) The anterior boundary of the lumbar artery
triangle is made by: Lower Inferior thyroid Recurrent laryngeal
A) Internal oblique artery
B) External oblique
C) Latissimus dorsi 11) The effect of PaO2 on hemoglobin
D) Iliac crest oxygen dissociation curve:
Ans: B A) Haldene effect
5) A pregnant lady in her third trimester B) Bohr effect
uses only iron supplements, what C) No effect
would she be deficient in? D) Hamburger effect
A) Vitamin B12 Ans: B
B) Calcium 12) Most important cause of infarction in
C) Folic Acid solid organ:
D) Magnesium A) Infarction
Ans: B B) Necrosis
6) Incidence is: C) Atheroma
A) Number of new cases D) Air embolism
B) Number of pre-existing cases Ans: C
C) Number of old and new cases 13) The metabolic fuel for liver in the post
D) Definition of the total population absorptive phase is:
Ans: A A) Ketone bodies
7) A patient merely eats vegetables, he B) Lactate
will be deficient in: C) Fatty acids
A) Folic Acid D) Glucose
B) Vitamin B12 Ans: D
C) Calcium 14) The highest protein contents are
D) Iron present in:
Ans: B A) Chylomicron
Explanation: B) HDL
C) LDL
Vitamins Source D) VLDL
SK Origional – Golden 11 232

Ans: B 21) A 4-year-old boy has undergone


Remember: splenectomy; which of the following
organisms will be responsible for
Rich in Triglycerides Chylomicrons > LDL
infections in him?
Rich in cholesterols LDL A) Staph Aureus
Rich in protein HDL B) Staph Epidermidis
15) A patient with abdominal pain, C) Sterptococcus Pnemoniae
sudden headache, fever and chills. D) S.Viridians
The organism responsible for this is Ans: C
geimsa stain positive: 22) The contents of the anterior
A) Leishmania mediastinum include:
B) E.Coli A) Thyroid gland
C) Plasmodium Falciparum B) Aorta
D) Cryptococcus C) Thymus
Ans: C D) Jagular vein
16) Most common cause of lung abscess: Ans: C
A) Staph Aureus 23) A female on a family visit to a remote
B) E.Coli village developed jaundice and
C) S. Pneumoniae hepatitis. Which of the following
D) Klebsiella organisms is the causative agent?
Ans: A A) Hep. B
17) Granuloma is a microscopic feature of B) Hep. C
which kind of cells? C) Hep. D
A) Kuffer cells D) Hep. E
B) Endothelial cells E) Hep. A
C) Epithelioid cells Ans: D
D) Mesothelium cells 24) Which part of the intestine does
Ans: C Entamoeba Histolytica reside in?
18) In OT procedure alcohol was used for A) Duodenum
cleansing purpose but the organism B) Jejunum
was resistant. Which of the following C) Transverse colon
organisms is it? D) Ascending colon
A) HIV E) Cecum
B) E.Coli Ans: E
C) Staph Aureus 25) The hormone responsible for delayed
D) Cryptococcus gastric emptying:
Ans: A A) Selectin
19) What is the most important form of B) CCK
protection against influenza? C) Gastrin
A) Patient isolation D) Peptidase
B) Vaccination and Immunization Ans: B
C) Wearing mask 26) The glands atrophy in the stomach of
D) Sun exposure an alcoholic and smoker patient would
Ans: B result in:
20) Vibrio cholera causes diarrhea by A) Anemia
which of the following mechanisms? B) Gastric cancer
A) Inhibits cAMP C) Hypomobility of stomach
B) Inhibits IP3 D) Diminished blood supply
C) Increases Cl channel opening Ans: A
D) Decrease HCO3 27) The parameters of HBsAg positive,
Ans: C HBeAg positive and HBV-DNA
positive would mean?
SK Origional – Golden 11 233

A) Acute Hep. B D) Myocarditis


B) Chronic Hep. B Ans: D
C) Chronic Hep. C 35) Decrease of which of the following
D) Acute Hep. C parameters occurs at the apex of
Ans: A lungs?
28) HLA-DR 4 represents: A) Oxygenation
A) Reiter‘s syndrome B) PCO2
B) Rheumatoid arthritis C) V/Q
C) DM Type II D) Ventilation
D) Ankylosing Spondylitis Ans: D
Ans: B 36) The slope pertaining to the action
29) Which of the following are potential changes with:
predominantly the anti-tumor cells? A) Hyponatremia
A) CD-4 B) Hypocalcemia
B) CD-8 C) Hypokalemia
C) NK Cells D) Hypo magnesia
D) IL-I Ans: A
Ans: C 37) Renin is secreted by:
30) Which of the following is the feature A) Endothelial cells of afferent arterioles
of SLE? B) JG Cells
A) Anti-coagulants increase C) PCT
B) Increase in red blood cells D) DCT
C) Complement system level decreases E) Collecting ducts
D) Complement system level increases Ans: B
Ans: C 38) Erythropoietin is secreted by:
31) An autopsy of dead cardiac tissue on A) Peritubular capillary cells
5th day will show which type of cells B) JG Cells
marginalization: C) PCT
A) Neutrophils D) DCT
B) Eosinophil E) Collecting ducts
C) Macrophages Ans: A
D) Basophils 39) Daily protein requirements in a
Ans: C normal adult:
Remember: Within initial 24-48 hours it will be A) 1g/kg
migration of neutrophils. B) 2g/kg
32) Drug that increases BP: C) 3g/kg
A) Phenoxy benzamine D) 4g/kg
B) Alpha agonist Ans: A
C) Dibenzyline 40) Which is the correct relation to
D) Beta blockers Pituitary gland?
Ans: B A) Anteriorly sphenoidal (paranasal sinus) is
33) Neural crest cells are separated from: present
A) Mesoderm B) Abducent nerve is present medially to
B) Yolk sac pituitary gland
C) Neural Tube C) Laterally it has relation of cavernous sinus
D) Neural crest D) Oculomotor nerve is present anteriorly
Ans: C Ans: C
34) Most common cause of death in 41) A damage to the right optic tract
rheumatic heart disease: would result in which kind of vision
A) Endocarditis loss?
B) Pericarditis A) Bitemporal Hemianopia
C) Epicarditis B) Right Homonymous Hemianopia
SK Origional – Golden 11 234

C) Left Homonymous Hemianopia C) Concentrated urine


D) No vision impairment D) Increase blood osmolality
Ans: C Ans: B
42) The exclusive function of the 50) A smoker with non-keratinizing
glucagon is: stratified squamous metaplasia can
A) Gluconeogenesis originate in:
B) Glycogenolysis A) Tongue
C) Lipolysis B) Intestine
D) Protein synthesis C) Kidney
Ans: A D) Bronchi
43) The thyroid scan is for: E) Esophagus
A) Warm nodule Ans: D
B) Cold nodule 51) A lady with cervical cancer of 3.5 cm.
C) Carcinoma ipsilateral and lymph nodes are
D) Cyst enlarged with suspected metaplasis.
Ans: B The staging classification will be:
44) High level of ACTH is secreted in: A) T1N2M1
A) Hepato adenoma B) T2N1M1
B) Renal adenoma C) T2N1M0
C) Adrenal adenoma D) T1NOMO
D) Polycythemia vera Ans: B
Ans: C 52) Drug of choice for cryptococcus:
45) A patient with urine frequency and A) Penicillin G
unable to hold urine: B) Trimethoprim
A) ADH insufficiency C) Sulfamethoxalate
B) Aldosterone insufficiency D) Amphotericin B
C) SIADH Ans: D
D) Adrenal insufficiency Remember: Cryptococcus is Fungus &
Ans: A Amphotericin B is Anti-Fungal.
46) Artery present anterior to the 3rd part 53) A 40 year old man presented to the
of the duodenum: clinic with jaundice and generalized
A) Inferior mesenteric artery itching. His LFTs reveal Direct
B) Superior mesenteric artery Bilirubin 11mg/100 ml, SGPT 75 IU/L
C) Left Gastric artery and ALP 2300 IU/L. The most like
D) Celiac artery cause of this condition is:
Ans: B A) Extra hepatic cholestasis
47) Least dilatable part of the urethra: B) Intrahepatic cholestasis
A) Membranous part C) Drug induce jaundice
B) Prostate part D) Autoimmune Hepatitis
C) Spongy Ans: A
D) All are equally dilatable 54) The median nerve has been damaged
Ans: A above flexor retinaculum. What will
48) The femoral pulse is felt at: happen?
A) Mid part below inguinal canal A) Loss of sensation of lateral palm
B) Femoral canal B) Loss of opposition of the thumb
C) Adductor canal C) Loss of thumb abduction
D) Midpoint of inguinal ligament D) Opposition of the thumb shall remain
Ans: D intact.
49) The carbonic anhydrase inhibitor Ans: B
(Acetazolamide) would result in: 55) A patient presents to clinic with his
A) Increase PH Hb level being 4 g/dl and Coomb‟s
B) Decrease PH Test positive. What is the cause?
SK Origional – Golden 11 235

A) G6PD deficiency C) HCC


B) Autoimmune hemolytic anemia D) Ameloblastoma
C) Megaloblastic anemia Ans: A
D) Pernicious anemia 61) Which of the following receptors
Ans: B senses low frequency vibration?
56) A 9 month old girl vaccinated two A) Pacinian corpuscles
weeks back now develops rash over B) Meissner corpuscle
body and fever. Her CBC report C) Krause bulb
reveals Hb 13g/dl, WBC 6X10^9/ D) Raffini receptors
microliter and platelet count was low. Ans: B
Her peripheral smear showed large Remember:
platelets. The condition is:
High Frequency Pacinian corpuscles
A) TTP
vibrations
B) ITP
C) Spurious thrombocytopenia Low Frequency Meissner corpuscle
D) Bernard soulier syndrome vibrations
Ans: B 62) Most rapidly adapting receptors:
57) A young girl with history of gum bleed A) Raffini
and menorrhagia. Her peripheral B) Krause
blood smear shows large platelets and C) Meissner
platelet count is 85,000/L. What is the D) Hair end organ
diagnosis? Ans: C
A) TTP Remember the Sequence: Pacinian > Meisnner
B) ITP > Hair end organ > Krause.
C) Spurious thrombocytopenia 63) The damage in the nucleus of Arcuate
D) Bernard soulier syndrome would result in:
Ans: D A) Parkinsonism
Remember: Bernard soulier syndrome is a B) Alzheimer disease
functional defect; so patient will bleed regardless C) Horner Syndrome
of the platelet count. While for ITP the platelet D) Kallman Syndrome
count needs to be lower than 20000/L for Ans: D
bleeding to start. 64) A patient with vertigo and sensation of
58) A patient with Anti-jo 1 antibodies nausea and vomiting, the site
positive serology, the best responsible would be:
manifestation ih him would be? A) Juxtapose dorsal vagal and
A) Skin rash vestibulocochlear nucleus
B) Pruritus B) Dorsal nucleus
C) Arthritis C) Ventral Nucleus
D) Bleeding D) Caudate
Ans: A Ans: A
59) A patient exhibits signs of Myasthenia 65) The nerve palsy of 6th and 7th nerves
Gravis, what is the confirmatory test in would involve:
him? A) Medulla
A) Biopsy B) Mid Brain
B) EMG C) Forebrain
C) Edrophonium D) Pons
D) Ach receptors antibodies Ans: D
Ans: B 66) A patient with slow wriggling
60) Which of the following is termed as movements in order to remove fly of
aggressive form of tumor? the nose is called:
A) Melanoma A) Athetosis
B) RCC B) Prospognosia
SK Origional – Golden 11 236

C) Ataxia C) Arachnoid mater and pia mater


D) Astigmatism D) Dura mater and cerebrum
Ans: A Ans: C
67) The portion of brain lesion that would 74) A 70 years old presents with slurred
result in fluent aphasia: speech and difficulty in reading paper
A) Wernicke‘s Area and descending stairs. He has past
B) Broca‘s Area history of falls many times. On
C) Premotor cortex examination there is impairment of
D) Somatosensory association area vertical gaze. What is the most likely
Ans: A diagnosis?
Remember: A) Parkinsonism
B) Alzheimer disease
Non-fluent aphasia Broca‘s area (broken words)
C) Pseudo tumor cerebri
Fluent aphasia Wernicke‘s area D) Progressive supranuclear palsy
68) Hyperextension due to vertebral Ans: D
column injury at: 75) Wrist joint is an example of:
A) Posterior longitudinal damage A) Condyloid
B) Anterior longitudinal damage B) Ellipsoid
C) DCML damage C) Pivot
D) Cauda equina damage D) Synovial
Ans: B Ans: B
69) The stroke due to berry aneurysm 76) Taste sensations from the anterior
involves which artery? part of the tongue are carried by:
A) Anterior communicating artery A) Facial nerve
B) Posterior communicating artery B) Glossopharyngeal nerve
C) MMA C) Hypoglossal nerve
D) ASA D) Lingual nerve
Ans: A Ans: A
70) A farmer presented with migrating 77) The neurons present in tractus
linear lesion on foot: solitareus:
A) Cutanea larva migrans A) 1st order neuron
B) Discoid lupus B) 2nd order neuron
C) Lichen planus C) 3rd order neuron
D) Melanoma D) Spinal neurons
Ans: A Ans: B
71) Which of the following muscles mark 78) The tumor of the posterior cranial
the neck? fossa:
A) Scalene posterior A) Medulloblastoma
B) Serratus Anterior B) Glioblastoma
C) Trapezius C) Astrocytoma
D) Scalene Anterior D) Ependymoma
Ans: D Ans: A
72) The CSF drainage to the internal 79) The side-effect of Morphine:
jugular vein occurs via: A) Diarrhea
A) Veins in pia mater B) Constipation
B) Veins in Dura Mater C) Tachypnea and myadriasis
C) Veins in subarachnoid D) Bradypnea and meiosis
D) Veins in cerebrum Ans: B
Ans: C 80) Breaking a bad news to the patient
73) The arachnoid space lies in between: regarding a fatal disease:
A) Pia mater and dura mater A) In a formal session
B) Arachnoid mater and dura mater B) In a hurried manner
SK Origional – Golden 11 237

C) Through written form D) Ependymoma


D) Shouldn‘t be told E) Craniopharyngioma
Ans: A Ans: B
Remember: If ‗formal session‘ is not in the 88) The inspiratory capacity is denoted
option and instead ‗in a crispy and acceptable way‘ by:
is present, choose the later then. A) ERV + RV
81) A patient with loin pain that does not B) ERV + TV
respond to medications: C) TV + IRV
A) Chronic pyelonephritis D) ERV+TV+RV
B) Acute pyelonephritis Ans: C
C) Chronic UTI 89) A lady drown in water was taken out
D) Appendicitis and resuscitated. Later on she
Ans: B developed ARDS. Likely mechanism
82) In diabetes insipidus the correct involved is:
parameter would be: A) Fluid overload
A) Increase plasma osmolality B) Interstitial edema
B) No change in osmolality of urine C) Collection of fluid in lungs
C) Decreased urine osmolality D) Oxygen causes reversal of symptoms
D) Increase urine osmolality Ans: B
Ans: C 90) Smoker related long scenario and in
83) The endocrine tissues develop from the end there was neutrophil elastase
which of the following? inhibitor:
A) 1st and 2nd pharyngeal arch A) Silica
B) 3rd and 4th pharyngeal pouch B) Nicotine
C) 6th Pharyngeal pouch C) Chloride
D) 3rd pharyngeal arch D) Asbestos
Ans: B Ans: B
84) The haploid DNA is related to: 91) The infarct of the posterior septum
A) Primary oocyte would cause injury to:
B) Secondary oocyte A) AV node
C) Primary spermatocytes B) SA node
D) Spermatids C) Bundle of His
Ans: D D) Purkinje fibers
85) A female baby with ambiguous Ans: A
genitalia and large clitoris, this feature 92) What is lost in carpal tunnel
is related to: syndrome?
A) Down Syndrome A) Loss of sensation of lateral palm
B) Complete mole B) Loss of opposition of the thumb
C) Adrenogenital syndrome C) Loss of thumb abduction
D) Partial mole D) Opposition of the thumb shall remain
Ans: C intact
86) Snow storm appearance is related to Ans: A
which of the following? 93) A lady with myxedema would most
A) Down Syndrome likely present with:
B) Complete mole A) Hyperthyroidism
C) Adrenogenital syndrome B) Grave‘s disease
D) Partial mole C) Carpal tunnel syndrome
Ans: B D) Exophthalmos
87) Most radiosensitive tumor is: Ans: C
A) Medulloblastoma 94) Growth hormone inhibition is done by:
B) Brain stem glioma A) Beta Blockers
C) Astrocytoma B) Somatostatin
SK Origional – Golden 11 238

C) Secretin 102) A farmer with the symptoms of


D) Hypokalemia Cushing syndrome, the cause is:
Ans: B A) Adrenal adenoma
95) What is the basic function of middle B) Renal adenoma
ear cavity bones? C) Hepatic adenoma
A) Protection of ear D) PTH adenoma
B) Amplification of sound waves Ans: A
C) Humidification of air 103) A patient in CCU will experience:
D) Stability A) Anxiety and stress
Ans: B B) Depression
96) The medicine to be taken by a C) Schizophrenia
psoriasis patient: D) Nightmares
A) Hydroquinone Ans: A
B) Retinoids 104) The basophilia of cytoplasm is due to:
C) Terbinafine A) RER
D) Fluconazole B) Ribosomes
Ans: A C) Golgi bodies
97) The normal respiratory drive is D) SER
dependent on: Ans: B
A) PaO2 Remember: Ribosomes > RER
B) PaCO2 105) Which laboratory test would be
C) Nitrogen in air suitable for Vitamin K related factors?
D) Altitude A) aPTT
Ans: B B) PT
Remember: PaCO2 > PaO2 C) BT
98) Venous blood is higher to arterial D) CT
blood in: Ans: B
A) Oxygen 106) A patient has congestive cardiac
B) Hemoglobin disease and secondary
C) Hematocrit hyperaldosteronism. This condition is
D) Platelets due to:
Ans: C A) ACTH
99) In uremic nephropathy there is: B) Renin
A) Hypochromic Normocytic anemia C) Cortisol
B) Normochromic Normocytic anemia D) Insulin
C) Micro chromic Normocytic anemia Ans: B
D) Megaloblastic anemia 107) After two days of an infection which
Ans: B cells will be present in the sputum?
100) Methyldopa drug acts on which A) Neutrophils
system? B) Eosinophil
A) CNS C) Macrophages
B) PNS D) Basophils
C) Autonomic nervous system Ans: A
D) Blood vessels Remember: Within initial 24-72 hours it will be
Ans: A migration of neutrophils then macrophages.
101) Regarding lymphatic function: 108) Irreversible platelet inhibition is done
A) Have some valves by:
B) Superficial with vein A) Acetaminophen
C) Deep with arteries B) Diclofenac sodium
D) Drains the lymph from tissues C) Aspirin
Ans: D D) Morphine
Ans: C
SK Origional – Golden 11 239

109) Lysosome has: 116) A G6PD patient infected with


A) Oxidases Plasmodium ovale malaria and took
B) Hydrolases drug, which one will cause hemolytic
C) Proteases crisis?
D) Lipases A) Primaquine
Ans: B B) Streptomycin
110) A baby with sex chromatin positive C) Artemeter
and enlarged clitoris: D) Penicillin
A) True hermaphrodite Ans: A
B) Turner syndrome 117) Why is Ranitidine chosen over
C) Klinefelter syndrome cimetidine?
D) Testicular feminization A) It is less potent
E) Adrogenital syndrome B) Short acting
Ans: E C) It has anti androgenic action
111) The range of droplets infection: D) No CNS penetration
A) 1-3 feet Ans: D
B) 3-6 feet 118) A patient presented with excessive
C) 6-9 feet salivation and sweating. What is the
D) 9-12 feet Drug of choice in his initial
Ans: A management?
Remember: Airborne infection range is 3-6 feet. A) Atropine
112) Standard deviation plus mean is: B) Adrenaline
A) Confidence interval C) Nor-Adrenaline
B) Confidence error D) Protamine Sulphate
C) Sample Bias Ans: A
D) Variance 119) Posterior one third of the heart
Ans: D septum is supplied by:
113) Highly radiosensitive tumor is: A) LCX
A) Lymphoma B) RCA
B) Glioma C) LCA
C) Seminoma D) Both LCA and LCX
D) Craniopharyngioma Ans: B
Ans: A 120) Which of the following protein is
Remember: Lymphoma > Seminoma > Glioma mainly deficient in Kwashiorkor
> Craniopharyngioma. disease?
114) Only LDH increases with no other A) Globulin
tumor marker. The condition is: B) Keratin
A) Lymphoma C) Albumin
B) Glioma D) Cysteine
C) Seminoma Ans: C
D) Craniopharyngioma 121) Reticular fibers are present in:
Ans: A A) Dermis
115) Most common virus spreading B) Lymphoid organs
through oro-fecal route: C) Nephrons
A) Hep. B D) Neurons
B) Hep. C Ans: B
C) Hep. D Remember: Lymphoid organ > Dermis.
D) Hep E 122) Diabetic foot with ulceration and loss
E) Hep A of sensation. The mechanism involved
Ans: E is:
Remember: Hepatitis A > Hepatitis E A) Vasculopathy
B) Neuropathy
SK Origional – Golden 11 240

C) Neuropathy followed by Vasculopathy Elevation. What test would you call for
D) Vasculopathy followed by neuropathy to confirm the diagnosis?
Ans: C A) Trop-T
123) The chronic exposure to radiation B) Trop-I
causes: C) CK-MB
A) Lichen Planus D) LDH
B) Melanoma Ans: B
C) Endarteritis obliterans 131) What does the baro-receptors
D) Anemia increase?
Ans: C A) MAP
124) Dicrotic notch is related to: B) SV
A) Pulmonic valve closure C) CO
B) Aortic valve closure D) HR
C) Aortic valve opening Ans: A
D) Mitral valve closure 132) Gastric ulcers common location:
Ans: B A) Fundus
125) Tall “T” waves on ECG would B) Anterior wall of stomach
indicate: C) Antrum
A) Hypercalcemia D) Lesser curvature
B) Hypokalemia Ans: D
C) Hperkalemia 133) Malignant change is:
D) Hypoglycemia A) Invasion of the basement membrane
Ans: C B) Necrosis
126) Increase in oxytocin results in: C) Local involvement
A) Cervix dilation D) Increase vascularity
B) Cervix contraction Ans: A
C) No effect on cervix 134) A 55-Year-old patient with generalized
D) Increase blood supply to cervix lymphadenopathy. Platelets and TLC
Ans: A count was raised. Peripheral smear
127) What is less in CSF in comparison to showed mature lymphocytes. What is
plasma? the diagnosis?
A) K A) CML
B) Na B) Hairy cell leukemia
C) Mg C) Burkitt lymphoma
D) Protein D) Hodgkin lymphoma
Ans: D E) CLL
128) Which of the following factors must be Ans: E
matched for transplant patient? 135) The type of anemia that occurs after
A) HLA gastrectomy is termed as:
B) Rh factor A) Sickle cell anemia
C) DNA B) Hemolytic anemia
D) MHC C) Pernicious anemia
Ans: A D) Normocytic anemia
129) Cardiolipin is present in: Ans: C
A) RER 136) A patient has signs of Byssinosis. He
B) Mitochondria works in which industry?
C) SER A) Coal
D) Plasma membrane B) Textile
E) Golgi bodies C) Road
Ans: B D) Plumber
130) A patient with chest pain for the last 4 Ans: B
hours and on ECG there is ST 137) Sympathetic activation causes:
SK Origional – Golden 11 241

A) Bronchoconstriction 144) A renal function important for


B) Broncho dilation conservation of blood volume:
C) Increase secretions A) Release of renin
D) Decrease HR B) Release of erythropoietin
Ans: B C) Decrease BP
138) Which of the following properties best D) Increase HR
define Vitamin-E in executing its Ans: A
function? 145) Which of the following structures is
A) Anticoagulant similar in adult and a child?
B) Antioxidant A) Inner ear
C) Antilipidemic B) Outer ear
D) Co-enzyme C) Nose
E) Co-factor D) Mouth
Ans: B Ans: A
Remember: Glutathione > Transferrin > 146) Malignancy occurs after how many
Vitamin E > Vitamin C > Vitamin A. years of exposure to radiations:
139) Hypoventilation occurs in: A) 5 years
A) COPD B) 8 years
B) Anemia C) 10 years
C) Polycythemia D) 20 years
D) Poisoning Ans: C
Ans: A 147) In chemotherapy the size of the tumor
140) Increase pulse pressure in an athlete decreases due to:
will result in: A) Apoptosis
A) Increase CO B) Necrosis
B) Increase HR C) Lysis
C) Increase SV D) Infarction
D) Increase MAP Ans: A
Ans: C 148) A patient who had ptosis and
141) Pap smear an economical and easy mydriasis with vertical gaze. The issue
test is ordered for: lies in:
A) Bronchial dysplasia A) Optic nerve
B) Renal cell carcinoma B) Trochlear nerve
C) Cervical dysplasia C) Abducens nerve
D) Esophageal metaplasia D) Occulomotor nerve
Ans: C Ans: D
142) A chronic smoker smoking 20 Remember:
cigarettes per day for around 20 years
Ptosis + miosis Horner Syndrome
would develop:
(constricted pupil)
A) Squamous metaplasia
B) Burkitt lymphoma Ptosis + mydriasis Occulomotor nerve
C) Hyperplasia (dilated pupil) palsy
D) Hypertrophy with hyperplasia Ptosis + Normal pupil Myasthenia Gravis
Ans: A
143) Prothrombin time is taken into 149) Thyroid moves with swallowing due
consideration for which of the to:
following factors? A) Prevertebral fascia
A) Factors 2, 7, 9 and 10 B) Pretracheal fascia
B) Factor 8 C) Subcutaneous fat
C) Factor 12 D) Muscles
D) Hageman factor Ans: B
Ans: A 150) The site for thoracocentesis is:
SK Origional – Golden 11 242

A) Upper border of the 9th rib at mid C) IgM


axillary line D) IgE
B) 5th intercostal place Ans: D
C) Lower border of 7th rib at mid clavicular 158) What lies closest to crus cerebri?
line A) Caudate
D) Lower border of 9th rib at midaxillary line B) Globus Pallidus
E) Lower border of 9th rib at mid-clavicular C) Substantia Nigra
line D) Pons
Ans: A Ans: C
151) Virus causes cancer by undergoing 159) During lactation, the process of
alteration in: ovulation is inhibited due to:
A) Nuclear integration A) GnRH
B) Cell membrane proliferation B) ACTH
C) Proto-oncogene B) CRH
D) Lysis of Nucleic Acids D) FSH
Ans: C Ans: A
152) Most common hepatitis that is found 160) Acute respiratory distress syndrome is
in Asia and has least amount of related to:
complications: A) Late birth
A) Hep. C B) Prematurity
B) Hep. E C) Genetical disease
C) Hep. D D) Obstetrics complications
D) Hep.A Ans: B
Ans: D 161) If there is less response to oxytocin, it
153) Standard deviation measure: means there are:
A) Confidence interval A) Less receptors on endometrium available
B) Confidence error for oxytocin.
C) Sample Bias B) Oxytocin is ineffective
D) Variability among individual values C) Less vascularity
Ans: D D) Low half-life
154) All drug receptors are: Ans: A
A) Active 162) Least conduction velocity is present
B) Increase in size in:
C) Protein in nature A) AV Node
D) Inactivated easily B) Purkinje fibers
Ans: C C) Bundle of His
155) High altitude cause: D) Ventricle
A) Decrease PO2 in blood Ans: A
B) Increase PCO2 in blood 163) The nature of agglutinins is:
C) Increase PO2 in blood A) Keratin
D) No change in blood B) Albumin
Ans: A C) Glycoprotein
156) MAP is dependent on: D) Globulins
A) TPR Ans: C
B) CO 164) Extracellular buffer is:
C) SV A) Ammonia
D) HR B) Hb
E) CO and TPR C) Bicarbonate
Ans: E D) Phosphate
157) The antibodies involved in urticarial: Ans: C
A) IgA 165) The combination of Clavulanic acid
B) IgG with ampicillin is due to:
SK Origional – Golden 11 243

A) Increasing half-life Ans: D


B) Decreasing first pass effect
C) Increase concentration in plasma
D) Inhibit beta lactase ===============
Ans: D
166) Patient sweats 2L and drinks two liters
of water. It will result in:
A) Increase ECF volume
B) Increase ICF Osmolality
C) Increase ECF Osmolality
D) Increase ICF volume

ALL GYNAE & OBS


September – November – 2020 Papers
SK Origional – Golden 11 244

GYNAE & OBS 5) Which laboratory test best separate


th those who have disease from those
September 30 , 2020 - Morning who don‟t have disease?
a. Sensitivity
1) True hermaphrodite karotype sex b. Specificity
chromosome pattern is: c. True positive
a. XY d. True negative
b. XXY e. PPV
c. XO Ans: A
d. XYY Explanation:
Ans: B  A highly sensitive test (increase ability to
Explanation: XX > XX/XY > XXY detect true positive) if test negative,
2) Anterior surface of the heart is formed should rule out the disease : SNOUT.
by:  A highly specific test(increase ability to
a. Right ventricle detect true negative) if test positive
b. Right atrium should rule in the disease : SPIN.
c. Left ventricle 6) A person got a stab wound 3cm
d. RV + SVC vertical on lateral right side of linea
Ans: A alba, initially he was fine but then
Explanation: developed hypotension & shock.
 Anterior surface (Sternocostal surface) of Which vessel is most likely damaged?
heart is formed mainly by right ventricle. a. IVC
 Posterior surface (Base) of heart is mainly b. Superior mesenteric Artery
formed by Left atrium. c. Inferior Mesenteric Artery
 Right surface of heart is formed by right d. Abdominal Aorta
atrium. e. Ileal branch of superior mesenteric artery
3) Inherited form of endometrial Ans: A
carcinoma is associated with: Explanation: The surface marking of the inferior
a. BRCA vena cava is a vertical line 2.5 cm – 3 cm to the
b. Breast Cancer right of the midline from the intertubercular plane
c. HNPPC to the sixth costal cartilage.
d. CA-125 7) Most common complication of DVT:
Ans: C a. Emboli
Explanation: DNA mismatch repair causes b. Thrombi
colon, endometrial & ovarian cancer called cancer c. Clot
syndrome. d. Death
4) A 30 years old female with no CVS or Ans: A
respiratory ailment is having swelling Explanation: The most serious complication of
in front of the neck. She feels difficulty DVT happens when a part of the clot breaks off
in breathing while in the supine and travels through the bloodstream to the lungs,
position. What can the type of causing a blockage called PE.
swelling in this patient? 8) A post adrenalectomy patient died 04
a. Multinodular Goiter days after the surgery, what is the
b. Follicular CA of thyroid most likely cause:
c. Asthma a. Deficiency of Norepinephrine
d. Retrosternal Goiter b. Deficiency of Aldosterone
e. Colloid Goiter c. Deficiency of testosterone
Ans: D d. Deficiency of ACTH
Explanation: Retrosternal goiter extends to e. Deficiency of Cortisol
trachea and causes difficulty in breathing & neck Ans: B
veins engorgement. Explanation: Aldosterone is extremely important
for life & it‘s deficiency is the leading cause
SK Origional – Golden 11 245

toward death in post adrenalectomy patient if not  During thryoidectomy the nerve most
treated. commonly damaged is external laryngeal
9) Stratum functionalis of uterus is nerve.
formed by: 13) Carcinoma of the skin of glans penis
a. Stratum compactum & Spongiosum is most likely to spread via lymphatics
b. Stratum basale & Spongiosum to:
c. Stratum compactum & basale a. External iliac nodes
d. Stratum basale & Spongiosum b. Internal iliac nodes
Ans: A c. Superficial inguinal nodes
Explanation: Stratum functionalis of uterus is d. Deep inguinal nodes
composed of stratum compactum & stratum Ans: C
spongiosum while the regenerating layer is stratum Explanation: Skin of glans penis is drained via
basale. superficial inguinal nodes while glans penis itself is
10) OCP discontinued due to the presence drained by deep inguinal nodes.
of: 14) Early diagnosis & prompt treatment
a. Migraine is:
b. Cervical neoplasia a. Counseling
c. Cluster Headache b. Primary prevention
d. Tension headache c. Secondary prevention
Ans: A d. Simple prevention
Explanation: According to MRCOG guidelines e. Tertiary prevention
OCPs are contraindicated in migraine. Ans: C
11) Fine skilled motor movements are Explanation:
controlled by:  Primary prevention: health promotion &
a. Corticospinal tract specific protection.
b. Cerebellum  Secondary prevention: early diagnosis &
c. Rubrospinal tract prompt treatment.
d. Spinothalamic tract  Tertiary prevention: Disability limitation
Ans: A & rehabilitation.
Explanation: 15) Water loss through sweating after
 Fine skilled & discrete movements are exercise is replaced with normal saline
controlled by CST. which of the following will increase?
 Fine movements are controlled by a. Extracellular volume
cerebellum. b. Extracellular osmolarity
12) During thyroidectomy superior c. Intracellular volume
thyroid artery is ligated. The nerve d. Intracellular potassium
likely to be injured: Ans: C
a. External Laryngeal nerve 16) Alpha 5 reductase stimulates which
b. Internal Laryngeal nerve hormone?
c. Recurrent Laryngeal nerve a. Testosterone
d. Superior Laryngeal nerve b. Cortisol
e. Vagus nerve c. Epinephrine
Ans: A d. NE
Explanation: Ans: A
 If superior artery is ligated the nerve most 17) Infection of the neck anterior (infront)
likely to be damaged is external laryngeal of pretracheal fascia will spread to:
nerve. a. Anterior Mediastinum
 If inferior artery is ligated the nerve most b. SuperiorMediastinum
likely to be damaged is recurrent laryngeal c. Posterior Mediastinum
nerve. d. Middle Mediastinum
e. Inferior mediastinum
Ans: A
SK Origional – Golden 11 246

Explanation: d. Uterus
 Infection of neck in front of pretracheal Ans: B
fascia will spread to anterior Explanation: Urogenital folds make the ventral
mediastinum. surface of penis in a boy & the same structure
 Infection of neck behind pretracheal makes labia minora in a girl.
fascia will spread to superior & posterior 22) The maternal mortality ratio (MMR):
mediastinum. a. The number of maternal deaths during a
18) A 02 years old child presented with a given time period per 100,000 live births
midline swelling just below the hyoid during the same time period.
bone. The swelling was noticed at 03 b. The number of maternal deaths during a
months of age and is slowly increasing year.
in size, with no movements on c. The number of maternal deaths in a given
protrusion of tongue, the condition period per population of women who are
most likely is: of reproductive age.
a. Accessory thymic tissue d. The number of deaths of young women.
b. Branchial cyst Ans: A
c. Cranipharngioma Explanation:
d. Ectopic thyroid gland  Maternal mortality ratio is the number of
e. Thyroglossal duct cyst maternal deaths per live births.
Ans: D  Maternal mortality rate is the number of
Explanation: Thyroglossal duct cyst will move maternal deaths in a given period per
with protrusion of the tongue & protrusion of population of women who are of
tongue test is positive while in ectopic thyroid reproductive age.
gland there will be no movement of the swelling 23) Which of the following is always
with protrusion of tongue and hence, protrusion present in anemia?
test is negative. a. Low MCV
19) A patient having serum creatinine b. High TIBC
level of 8 mg/dl and blood pressure is c. Low Hemoglobin
180/100 mmhg. Which cells are most d. High hemoglobin
likely damaged: Ans: C
a. Glomerular cells Explanation: Low hemoglobin is found in all
b. JGs cells types of Anemia while MCV & TIBC maybe
c. Macula Densa cells increased or decreases in various types of Anemia.
d. Afferent arteriole 24) The best option for accurate
Ans: B calculation of gestational age by
20) During summer a fasting man has ultrasound scan will be measurement
concentrated urine due to: of:
a. Decrease water intake a. Crown rump length at 8 weeks of fetal life
b. Increase ADH b. Crown rump length at 6 weeks of fetal life
c. Increase sweating c. Biparietal diameter at 18 weeks of life
d. Decrease ADH d. Bitemporal diameter at 18 weeks of life
Ans: B Ans: A
Explanation: During summer and fasting there‘s Explanation: CRL upto 14 weeks while BPD in
less water intake & hence ADH is released that 2nd trimester.
leads to concentrated & low volume urine. 25) Abnormal overgrowth thickening of
21) In a boy on examination a structure bone is due to:
which fails to develop on ventral a. Chief cells of parathyroid
surface of penis, what‟s the b. Cells of adenohypophysis
counterpart of structure in a girl? c. Pars intermedia
a. Labia Majora d. Leydig cells
b. Labia Minora Ans: B
c. Clitoris
SK Origional – Golden 11 247

Explanation: Cells of adenohypohysis secretes a. Puborectalis muscle


growth hormone & that leads to abnormal growth b. Pubococcygeus muscle
& thickening of bones as in gigantism & c. Rectococcygeus muscle
acromegaly. d. Anococcygeus muscle
26) A 25 years old female patient Ans: B
developed fever, neck stiffness along Explanation: Pelvic diaphragm is constructed by
with maculopapular rash. Most likely iliococcygeus, pubocccygeus & puborectalis
organism involved is: muscles, but here among puboccygeus &
a. N. Meningitidies puborectalis muscles we choose pubococcygeus
b. Streptococcus Pneumoniae muscle.
c. E. coli 31) Which of the following is Autosomal
d. Staph Aureus dominant disorder?
Ans: A a. Familial adenomatous polyposis
Explanation: N. meningitides presents with b. Tay sach disease
meningitis with peculiar behavior of c. SLE
maculopapular rash. d. DMD
27) A patient with liver disease & pedal Ans: A
edema, cause of edema in this patient 32) A patient presented with
is: hypersensitivity reaction. He was
a. Decrease oncotic pressure given diphtheria toxoid 2 weeks ago,
b. Increase onctoic pressure what‟s the type of reaction?
c. Increase hydrostatic pressure a. Type 1 HSR
d. Increase vascular permeability b. Type 2 HSR
Ans: A c. Type 3 HSR
Explanation: In liver disease there‘s deficiency of d. Type 4 HSR
Albumin that leads to low oncotic pressure & Ans: C
edema develops. Explanation: Post vaccination history the HSR
28) A patient having CHF, pleural that may occur is serum reaction type & that‘s
effusion. What will you see to consider type III HSR.
it as transudate? 33) A patient is administered a tuberculin
a. Lymphocytes test. Which of the following types of
b. Neutrophils hypersensitivity reaction is being
c. Macrophage tested and which cells would be
d. Few serosal cells expected to mediate a positive test
Ans: D result?
Explanation: Exudate is found in inflammatory a. Type 1 HSR, Mast Cells
conditions & there will be inflammatory cells b. Type 2 HSR, Cytotoxic T Lymphoctyes
while in transudate the cells will be less in number. c. Type 4 HSR, Mast cells
29) Conduction system of heart is present d. Type 4 HSR, T-1 helper cells
in: e. Type 4 HSR, T-2 Helper cells
a. Epicardium Ans: D
b. Subendocardium Explanation: Tuberculin test is type 4 HSR & the
c. Endocardium cells involved are macrophages & type 1 helper T
d. Myocardium cells.
Ans: B 34) Tunica Vaginalis is remnant of:
Explanation: a. Inguinal canal
 SA node is present in subepicardium. b. Process vaginalis
 AV node is present in Endocardium. c. Testis
 Conduction system is present in d. Ductus deferens
subendocardium. Ans: B
30) Which structure forms pelvic 35) Derivate of neural crest cells:
diaphragm? a. Melanocytes
SK Origional – Golden 11 248

b. Autonomic ganglia Ans: A


c. Lens of eye 41) An eleven month baby brought by her
d. Cardiac chambers mother complains that the baby is
Ans: B semi-conscious and listless. O/E no
Explanation: Both melanocytes & Autonomic response to vocal stimulation, weight
ganglia are derived from neural crest cells, it might 60% of age and a small purple
be a poor recall or prefer autonomic ganglia over contusion is found on the thigh, what
melanoctyes if the options are the same. is the probable diagnosis?
36) A 6 years old girl with Hb 5 gm/dl, a. Marasmus
MCV 58 fl, MCHC 27, pallor and b. Kwashiorkor
lethargic with platelet count of c. Physical Abuse
250,000 and TLC count 6200 is most d. Edema
likely suffering from: Ans: C
a. IDA Explanation: The above scenario is related to
b. Thalassemia Trait physical abuse as the baby isn‘t responding to
c. Sideroblastic Anemia vocal stimulation, it could be kwashiorkor if there
d. Megaloblastic Anemia was positive response toward vocal stimulation.
Ans: B 42) Patient has difficulty in
Explanation: The scenario is related to comprehension & intellectual brain
thalassemia & there are lots of clues like low hb, function. Which area of the brain is
MCV, normal MCHC & decrease platelets counts affected?
that would be raised if IDA. a. Broca‘s area
37) Supporting cells of pars nervosa: b. Wernickes area
a. Stores in nerve endings c. Medial fasiculus
b. Pitucityes d. Corpus collosum
c. Astrocytes Ans: B
d. Glial cells Explanation:
Ans: B  If defective comprehension &
38) Atopic asthma, foreign substance meaningless sentences then it‘s most
reacts with: likely wernickes aphasia.
a. Mast cells  If unable to produce words then it‘s most
b. Eosinophils likely Broca‘s Aphasia.
c. Alveolar Epithelial cells 43) Which one is strongest anesthetic but
d. Lymphoctyes weak analgesic?
Ans: A a. Thiopental
Explanation: Follow the following sequence in b. Halothane
case of atopic & allergic condition: Mast Cells > c. NO
Basophils > Eosinophils. d. Sevoflurane
39) A patient crying to tell you about her Ans: B
sister death, what should be your Explanation:
response?  Halothane is strong anesthetic but weak
a. Take your time analgesic.
b. Don‘t worry everything will be fine with  No is strong analgesic but weak
time anesthetic.
c. Take her out 44) Brain tissues are damaged by stroke,
d. Call the nurse cell will be regenerated by:
Ans: A a. Fibroblast
40) Crude tactile sensation is carried by: b. Glial cells
a. Ventral column c. Astrocytes
b. Dorsal Column d. Schwann Cells
c. Lateral Column Ans: C
d. All Explanation: Astrocytes > Glial Cells
SK Origional – Golden 11 249

45) Ovaries develop from: d. Sciatic nerve


a. Paramesonephric duct Ans: A
b. Mesonephric duct 52) The lab reports of a pregnant lady
c. Gonads showed HBs (Surface) antigen –ve,
d. Gubernaculum Anti HBc (core) reactive, IgM Non -
Ans: C reactive. These results indicate:
Explanation: Gonads give raise to Ovaries, a. Active disease
ovarian follicles & rete ovarii in a female. b. Active Chronic disease
46) Anti-cancer drug that causes c. Past exposure
nephrotoxicity: d. Resolving Phase
a. Cyclosporin e. Immunized
b. Cisplatin Ans: C
c. Penicillin Explanation: IgM indicates acute phase now that
d. Macrolides IgM is negative and anti HbC is positive this
Ans: B indicates past exposure.
47) Main component of lipids: 53) Patient has suffered from 3rd degree
a. Magnesium burn, he is worried about it, which of
b. Carbon the following complications can arise:
c. Phosphorus a. Wound dehiscence
d. Calcium b. Contracture
Ans: B c. Keloid
Explanation: Carbon makes the backbone of the d. Trauamtic Neuroma
lipids structure. e. None
48) CO2 diffuses rapidly than O2 through Ans: B
the alveoli due to: Explanation: Contracture is the likely
a. High pressure of CO2 in the lungs complication to occur after 3rd degree burns while
b. High lipid solubility in black people it‘s keloid.
c. High water solubility 54) Metabolic function of thyroid
d. High diffusion coefficient hormone at physiological state:
Ans: D a. Decrease fatty acid synthesis
49) Middle rectal artery is a branch of: b. Increase fatty acid synthesis
a. Common iliac artery c. Decrease protein synthesis
b. External iliac artery d. Increase protein synthesis
c. Inferior vesical artery Ans: D
d. Aorta 55) Failure rate of IUCD is:
Ans: A a. 2-5 %
Explanation: Middle rectal artery is a branch of b. 3-5 %
internal iliac artery but that wasn‘t in option so we c. 10%
choose common iliac artery here, if internal iliac d. Less than 1%
artery was given prefer that. Ans: D
50) Drainage of ischiorectal fossa, 56) Regarding sarcomere of muscle fiber
structure damage is: it‟s between:
a. Pudendal artery a. Two Z-lines
b. Inferior rectal nerve b. Transverse tubules
c. Perineal nerve c. H-Band
d. Internal Iliac artery d. A-Band
Ans: B Ans: A
51) Injury below and lateral to pubic 57) First & important sign of cholinergic
tubercle will damage: overdose:
a. Obtruator nerve a. Bradycardia
b. Femoral nerve b. Tachycardia
c. Ilioinguinal nerve c. Volume depletion
SK Origional – Golden 11 250

d. Increase Urinary output b. A band


Ans: A c. H zone
58) Philadelphia chromosome is a feature d. Sarcomere
of: Ans: B
a. CML 64) A patient receives radiotherapy for
b. AML cancer, after a few days he develops
c. ALL scaring & ulceration of the skin it‟s
d. CLL due to:
Ans: A a. Endarteritis Obliterans
Explanation: Philadelphia chromosome occurs in b. Desquamation
CML 9:22 translocation. c. Venous thrombosis
59) Which is not related to pain d. Infection
transmission? Ans: B
a. Thalamus Explanation: Within few days desquamation
b. Substantia Gelatinosa develops while endarteritis obliterans takes time to
c. C fibers develop.
d. B fibers 65) Doppler studies during pregnancy:
e. Hypothalamus a. Can detect fetal cardiac activity before 8
Ans: E weeks
60) Half-life of Casium 137 is: b. Pulstility in uterine areries increase in
a. 33 years growth restricted fetus
b. 40 years c. Fetus with raised middle cerebral artery
c. 50 years Doppler has low probability of anemia
d. 100 years d. Can detect renal changes in fetus during
Ans: A last months
Explanation: Exact half-life is 30.17 years but Ans: B
that wasn‘t in option so next closest is 33 years. 66) Diagnostic test for infective
61) Polycystic kidney disease in adult is: mononucleosis:
a. Autosomal dominant a. Heterophil antibody test
b. Autosomal recessive b. Monospot test
c. X linked dominant c. Paul bunnel test
d. X linked recessive d. Anti-Sheep Antibody test
Ans: A Ans: B
Explanation: Polycystic kidney disease in adult is Explanation: Monospot test > Heterophil
Autosomal dominant while in children it‘s antibody test > Paul Bunnel test.
autosomal recessive. 67) True regarding kindey:
62) Submandibular & submental lymph a. Cuboidal epithelium in parietal layer of
nodes are enlarged in which bowman‘s capsule
carcinoma: b. Podocytes in visceral layer of bowman‘s
a. Ca tongue capsule
b. Ca lower lip c. Slit like space between macula and JG
c. Ca upper lip apparatus
d. Ca cheeks d. DCT is longer than PCT
Ans: B e. Glomerulus has one artery
Explanation: Submandibular & submental lymph Ans: B
nodes are enlarged both in carcinoma lower lip & 68) Fastest control of blood pressure is
tongue but most specifically in carcinoma of lower achieved by:
lip. a. CNS Ischemiac response
63) Which portion of muscle doesn‟t show b. Chemoreceptors
change in length during muscle c. Baroreceptors
contraction? d. RAAS
a. I band Ans: C
SK Origional – Golden 11 251

Explanation: Rapid & fasted control of blood c. Estrogen


pressure is brought by baroreceptor while most d. Progesterone
potent is CNS ischemiac response and most Ans: C
prolong is RAAS. 75) Local anaesthetic drug with most
69) In developmental stages scanty hair, toxic (lethal) effects:
coarse triangular shaped hairline a. Lignocaine
appears in: b. Prilocaine
a. Stage 1 c. Bupivacaine
b. Stage 2 d. Xylocaine
c. Stage 3 Ans: C
d. Stage 4 Explanation:
Ans: C  Lignoacaine have local anesthetic effect
70) An old age diabetic lady with CRF is with short half-life & lesser side effect
on dialysis. Her X-ray Lower limb,  Bupivacaine has longer half-life & more
femur head shows loss of trabecular toxic effect.
bone. Which hormone is responsible 76) Initial symptom of bupivacaine
for such X-ray findings: overdose:
a. PTH a. Unconsciousness
b. Cortisol b. Ventricular arrhythmias
c. Calcitonin c. Ringing in the ears
d. Throxine d. Convulsion
Ans: A Ans: C
71) Stoke volume increases with no 77) Right border of the heart is formed by:
changes in total peripheral resistance, a. Right atrium
what will be the effect on heart rate or b. Right ventricle
arterial competence? c. Right atrium + SVC
a. Increase arterial competence with d. SVC
decrease CO Ans: A
b. Increase arterial competence with increase Explanation:
CO  Right boarder of the heart is formed by
c. No change right atrium.
d. Increase Venous Compliance  In PA view of CXR right boarder of heart
Ans: B is formed by right atrium + SVC.
72) Mean Arterial Pressure is: 78) DOC for 5th month pregnant lady
a. CO/TPR with thyrotoxicosis:
b. CO * TPR a. Methimazole
c. SV * TPR b. PTU
d. SV/TPR c. Thyroxine
Ans: B d. T3
73) Continuous stimulus to the muscle Ans: A
causing muscle contraction without Explanation:
relaxation is:  During first trimester the drug of choice
a. Tetany for thyrotoxicosis is PTU
b. Tetanus  Second & third trimester it‘s methimazole
c. Tetanic Contraction 79) Regarding cardiac cycle:
d. Spasm a. S2 at start of T wave
Ans: C b. P wave is followed by T wave
74) Proliferation of the mammary c. 1st heart sound is produced in
ducts(lactiferous ducts) and fat isovolumetric ventricular contraction
deposition in breast is primarily by: d. 1st heart sound is produced in
a. LH isovolumetric ventricular relaxation
b. FSH Ans: C
SK Origional – Golden 11 252

Explanation: 1st heart sound is produced in latissimus dorsi posteriorly, the external oblique
isolvolumetric ventricular contraction due to muscle anteriorly & the iliac crest inferiorly.
closure of AV valves that‘s mitral & tricuspid 85) Uterinecarcinoma (or infection)
valves. travels to Labia majora through which
80) Progress to grade 4 hepatic route:
encephalopathy is caused by: a. Round ligament
a. Alcohol b. Broad ligament
b. Diuretics c. Cardinal ligament
c. Paracetamol d. Suspensory ligament
d. Antibiotics Ans: A
Ans: B 86) Medial border of inguinal triangle is
81) Baby came with intestinal obstruction formed by:
and was diagnosed to have a. Lateral boarder of rectus abdominis
hirschsprung disease. Which segment b. Inguinal ligament
will be involved? c. Inferior epigastric vessels
a. S2, S3, S4 d. Medial boarder of rectus abdominis
b. S4, S5 Ans: A
c. L5, L5 Explanation:
d. L2, L3  Medial boarder is formed by Rectus
Ans: A abdominis
82) 60 year old man with painless  Lateral boarder is formed by inferior
testicular swelling, biopsy shows epigastric vessels
mature stem cells: What is your likely  Inferior boarder is formed by inguinal
diagnosis: ligament
a. Choriostoma 87) The inferior rectal vein drains into:
b. Lymphoma a. Inferior mesenteric vein
c. Mixed cell tumor b. Superior mesenteric vein
d. Teratoma c. Internal pudendal vein
e. Adenoma d. Left renal vein
Ans:. D e. Internal iliac vein
83) Scenario about a patient, tooth Ans: C
extraction was done,doctor prescribed Explanation: The inferior rectal vein drains into
him multivitamins and calcium internal pudendal vein that in turn drains into
supplements. Patient is already taking internal iliac vein.
candesartan. Which of the following 88) Counter part of labia majora in men:
antibiotics will affect its efficacy? a. Scrotum
a. Tetracycline b. Penis
b. Metronidazole c. Testis
c. Amoxcillin d. Appendix testis
d. Penicillin Ans: A
Ans: B 89) Which structure act as a conduit
84) Anterior border of lumbar Triangle is between peritoneal cavity and uterus?
formed by: a. Birth canal
a. Posterior boarder of Latissimus dorsi b. Cervix
b. Iliac crest c. Fallopian tube
c. Posterior boarder of quadratus lumborum d. Ovaries
d. Posterior boarder of external oblique Ans: C
muscle. 90) Antihypertensive drug which causes
e. Serratus anterior postural hypotension, tachycardia &
Ans: D angina:
Explanation: The boarders of petit‘s triangle, also a. Hydralazine
known as inferior lumber triangle is bounded by b. Nifidipine
SK Origional – Golden 11 253

c. Propranolol  SVC +Right atrium > SVC > Right


d. Atenolol Atrium.
Ans: A 96) Dopamine infusion steady state is
Explanation: Hydralazine may increase heart achieved in:
rate, cardiac output and in people with coronary a. 7 minutes
artery disease may cause angina pectoris or MI. b. 9 minutes
91) Following is the derivative of 3rd c. 5 minutes
pharyngeal arch: d. 2 minutes
a. Stylopharyngeus Ans: B
b. Posterior belly of diagastric muscle Explanation: Half-life of dopamine is 2 minutes
c. Anterior belly of diagastric muscle an steady state is achieved in 4-5 half-lives that‘s 8-
d. Platysma 10 minutes so here 9 minutes is the closest.
Ans: A 97) Which antithrombin is present
92) Critical tactile sensation is carried by: naturally in blood?
a. Dorsal white column a. Dimercaprol
b. Lateral white column b. Plasminogen
c. Ventral white column c. Heparin
d. None d. Vitamin C
Ans: A Ans: C
Explanation: Explanation:
 Critical tactile sensation is carried by  Natural anticoagulant/antithrombotic:
DCML that‘s dorsal white column. Heparin.
 Crude tactile sensation is carried by  Natural thrombolytic is:
anterior spinothalamic tract. Plasminogen/Plasmin.
93) Raynaud‟s phenomenon is present in: 98) A Young patient has chronic
a. Arteriolar constriction anemia.The investigations revealed
b. Venoconstriction high serum ALP, decreased hb and
c. Peripheral vascular disease increased retic count. Most likely
d. CNS cause of jaundice is:
Ans: C a. Cholangiocarcinoma
94) Increase PCO2 in venous blood, will b. Cholesterol stone
lead to: c. Haematological malignancy
a. Decrease chloride concentration in RBC‘s d. Hepatitis
b. Increase affinity for Hb e. Pigment stone
c. Decrease HCO3 concentration in blood Ans: E
d. Increase volume of RBC 99) Absolute Contraindication of
Ans: D thrombolytic (antithrombotic)
95) Anteroposterior radiograph of heart therapy:
will take part in it‟s formation: a. Active/ongoing bleeding
a. SVC b. Preveious history of cardiovascular events
b. Right ventricle c. Stroke
c. Left ventricle d. GIT bleeding in past 03 months
d. Aorta Ans: A
Ans: A 100) Volume remaining in the lungs after
tidal volume expiration is:
a. Residual volume
b. Expiratory reserve volume
Explanation: c. FRC
 AP view of heart will show SVC + right d. Vital Capacity
atrium but right atrium wasn‘t in option e. TLC
so SVC. Ans: C
SK Origional – Golden 11 254

101) A day following hysterectomy, a a. Axons


patient complains of severe pain in the b. Oligodendrocytes
right upper lumbar region of the back. c. Protoplasmic astrocytes
Which of the following structures was d. Microglia
most likely damaged during the Ans: C
surgery? 106) Ultrasound frequency used for
a. Inferior mesenteric artery prenatal diagnostic purpose is
b. Internal iliac artery obstetrics:
c. Psoas major muscle a. 1 to 20 mhz
d. Pudendal nerve b. 20 to 40 mhz
e. Ureter c. 40 to 60 mhz
Ans: E d. 1 to 10 mhz
Explanation: Ureter is commonly ligated Ans: A
mistakenly during hysterectomy & patient can 107) Alpha antagonist (Blocker) is:
present with above mentioned findings. a. Doxazosin
102) Pulmonary valve opens when right b. Clonidine
ventricular pressure exceeds? c. Propranolol
a. 25 mmhg d. Atenolol
b. 8 mmhg Ans: A
c. 80 mmhg Explanation: Doxazosin is selective Alpha1
d. 120 mmhg receptor antagonist.
e. 15 mmhg 108) Intravaginal administration of
Ans: B prostaglandin initiates cervical
Explanation: ripening by mechanism:
 Pulmonary valve opens when right a. Activating oxytocin receptors
ventricular pressure exceeds 8 mmhg. b. Hydration of collagen tissue
 Aortic valve opens when left ventricular c. Increase capillary permeability
pressure exceeds 80 mmhg. d. Increase local progesterone concentration
103) Diagnostic criteria for pre malignant e. Inhibiting thromboxane synthesis
condition is: Ans: B
a. Increase N/C ratio 109) Severe diarrhea:
b. Pleomorphism a. Metabolic acidosis with normal anion gap
c. Hyperplasia b. Metabolic acidosis with high anion gap
d. Dysplasia c. Metabolic acidosis with low anion gap
Ans: A d. Metabolic alkalosis
Explanation: Ans: A
 Malignancy: Metastasis > Invasion > Explanation:
Pleomrophism > Increase N/C ratio  Acute/severe diarrhea leads to metabolic
 Pre malignant lesion : Pleomorphism acidosis with normal anion gap
 Pre malignant condition: Increase N/C  Chronic diarrhea leads to metabolic
ratio alkalosis
104) Histological diagnostic criteria for 110) Best time for IV prophylactic
malignant tumour: antibiotic administration during
a. Pleomorphism surgery is:
b. Local invasion a. 12 hours before surgery
c. Mitotic figures b. 6 hours before surgery
d. Increase N/C ratio c. At the end of surgery
Ans: B d. At the time of induction of anaesthesia
Explanation: In histology the diagnostic criteria Ans: D
for malignancy is local invasion. 111) Major supply of bladder:
105) Cells of grey matter with short a. Sympathetic
cytoplasmic process: b. Parasympathetic
SK Origional – Golden 11 255

c. Both b. Anterior & posterior longitudinal


d. None ligaments
Ans: C c. Lagamentum nuchae
112) Counter part of labia minora in male d. Interspinous ligament
is: Ans: B
a. Ventral aspect of Penis 119) Scenario of a child with mental
b. Scrotum retardation, umbilical hernia &
c. Testis protruded tongue:
d. Tunica Vaginalis a. Cretinism
Ans: A b. Down syndrome
Explanation: Urogenital folds give raise to labia c. Klienfilter syndrome
minor in female & Ventral aspect of penis in male. d. Patau syndrome
113) Cause of night blindness is the Ans: B
deficiency of: 120) Female was diagnosed Alpha 1
a. Retinol antitrypsin deficiency. Which part of
b. Retinoic acid respiratory tract would be most
c. Retinal aldehyde affected?
d. Vitamin E a. Alveolar duct
Ans: A b. Terminal bronchioles
114) Glucagon & epinephrine both have c. Alveoli
the same effect in: d. Medium sized bronchi
a. Cause hypertension Ans: A
b. Activate phosphorylase kinase 121) Ventricles are maximally filled during:
c. Glycogenolysis in liver & muscle a. Rapid inflow
d. Gluconeogensis in muscle b. Atrial systole
Ans: B c. Diastasis
115) Sesamoid cartilage is found in: d. Ventricular systole
a. Epiglottis Ans: B
b. Ala of nose Explanation:
c. Larynx  Maximum filling during rapid inflow
d. Ear pinna  Maximum filled during atrial systole
Ans: B 122) Most potent stimulus for adlosterone
Explanation: release is:
 Sesamoid cartilage is found both in ala of a. Hyperkalemia
nose & larynx but prefer ala of nose over b. Hypernatremia
larynx. c. Increase glucose
 Sesamoid bone is patella. d. Rennin
116) Elastic cartilage is found in: Ans: A
a. Epiglottis 123) A structure with ciliated and non-
b. Larynx ciliated epithelium having three layers
c. Ala of nose of muscles & tubular glands is:
d. Thyroid cartilage a. Ureter
Ans: A b. Fallopian tube
117) Spinal cord is supplied by which of the c. Ovary
following? d. Uterus
a. Spinal artery Ans: B
b. Vertebral artery 124) A patient with right hypochondrium
c. Basilar artery pain, liver biopsy showed large
d. Internal thoracic artery epitheloid cells with granulomatous
Ans: A disease in center ,one giant cell with
118) Vertebral bodies limited by: multiple nucleoli in periphery what is
a. Ligamentum flavum it indicating:
SK Origional – Golden 11 256

a. Caseous necrosis drugs,develops headache. LP result


b. Granuloma of liver showed no glucose and proteins,
c. Fat necrosis culture showed gram positive oval
d. Liquefective necrosis bodies with halos around them.Most
Ans: A likely organism is:
125) An Old man who was being treated by a. Cryptococcus neoformans
a GP for arthritis comes to you with b. Coccidiodicoccus
complaints of Generalised weakness, c. Histoplasmosis
pallor, easy bruising.Hb 5g/dl and d. Aspergillus
dec RBC,WBC PLT. Retic count 0.1%. Ans: A
Bone tap hypocellular bone marrow 131) Ovarian follicle embryological
with few cells: counterpart:
a. Aplastic anemia a. Gonads
b. Acute leukemia b. Primary Sex cord
c. Myelofibrosis c. Zygote
d. Megaloblastic anemia d. Secondary follicle
Ans: A Ans: A
126) Sarcoidosis is diagnosed 132) Venous thrombosis worst
microscopically by: complication:
a. Granuloma with asteroid bodies a. Gangrene
b. Non caseating granuloma b. Infection
c. Macrophages & giant cells c. Pulmonary embolism
d. Lymphocytes d. Thrombus
Ans: B Ans: C
Explanation: Histological & microscopic findings 133) Which of the following causes
of sarcoidosis is non caseating granuloma. decrease PO2?
127) Scenario of a patient having acute a. Anemia
tonsillitis. Cytokienes involved in b. Cyanide poisoning
inflammation: c. Hypoxic hypoxia
A. IL1 & TNF d. Polycythemia
B. Bradykinin Ans: C
C. Histamine 134) Bilateral S2, S3 and S4 damaged:
D. Basophils a. No delivery pain
Ans: A b. Fecal incontinence
128) Scenario regarding meningitis in c. Urinary incontinence
newborn & organism was asked: d. Both fecal & urinary incontinence
a. Streptococcus lancefield A Ans: D
b. Streptococcus lancefield B 135) Non-essential amino acids take alpha
c. Streptococcus lacefield C group from:
d. Streptococcus lancefield D a. Pyruate dehydrogenase
Ans: B b. Oxaloacetate
129) Atropine causes: c. Glutamate
a. Sinus bradycardia d. None
b. Mydriasis Ans: C
c. Increase gastric secretion 136) H/O fall resulted in bladder
d. Increase gastric motility incontinence, unable to void urine.
Ans: B Spinal segment involved:
Explanation: Atropine is muscarinic antagonist a. S2, S3,S4
so it will lead into mydriasis, tachycardia and b. S1,S2,S3
decrease gastric secretion & motility. c. L5,S1,S2
130) Heart transplantation done, patient is d. L4,L5b
on immunosuppressive Ans: A
SK Origional – Golden 11 257

137) During 3rdtrimester of pregnancy, Ans: A


vaginal infection usually occurs with 143) Cause of amenorrhea during lactation
following organism: is:
a. E. coli a. Stimulation of gonadotropin due to
b. Chlamydia nipple stimulation
c. Candida albicans b. Due to release of prolactin
d. Trichomonas c. Milk secretion by oxytocin
Ans: C d. It inhibits goandotropin releasing
138) Substantia gelatinosa carries: hormone (GnRH)
a. Pain fibers Ans: B>D
b. Fine touch 144) Post-operative intra-abdominal
c. Pressure bleeding following lower segment C-
d. Proprioception Section. Most likely source of
Ans: A bleeding:
139) Corpus callosum: a. Uterine vein
a. Connected to fornix by septum b. Branches of uterine artery
pellucidum c. Ovarian artery
b. Most fibers interconnect symmetrical d. Internal iliac artery
areas of cerebral cortex Ans: B
c. Related superiorly to falx cerebri 145) The most common organ injured
d. Fibers of genu curve into frontal lobe as during blunt/penetrating trauma to
forcep minor abdomen is:
e. Rostrum connects genu to lamina a. Liver
terminlais b. Spleen
Ans: B c. Pancreas
Explanation: B>A d. Kidney
140) Hypertensive patient with aortic Ans: A( Bailey)
stenosis died,what will be the finding 146) Renin secretion is inhibited by:
on heart autopsy? a. AG2
a. Hypertrophy b. ADH
b. Hyperplasia c. ANP
c. Atrophy d. Beta 1 receptors
d. Dystrophy e. Low blood volume
Ans: A Ans: A
Explanation: Myocardial cells are permanent so The hormone which triggers onset of
only hypertrophy can occur in it without parturition in mother is:
hyperplasia. a. Fetal estrogen
141) Major lymphatics of breast go parallel b. Fetal thyroxine
to: c. Fetal cortisol
a. Axillary vessels to pectoral nodes d. None of the above
b. Along internal thoracic vessels to internal Ans: C
thoracic nodes 147) Saw tooth appearance of glands,
c. Along internal thoracic vessels to vacuolisation and edematous stroma:
posterior axillary nodes a. Secretory phase
d. Along inferior epigastric vessels b. Proliferative phase
Ans: A c. Menstrual phase
142) Columnar epithelium with indistinct d. Ovulation
smooth muscle and crypts: Ans: A
a. Uterine cervix 148) Plasma colloid oncotic pressure is
b. Ovary maintained by:
c. Fallopian tube a. Albumin
d. Uterus b. Gamma globulin
SK Origional – Golden 11 258

c. Fibrinogen Ans: C
d. Antibodies 154) cAMP pathway is stimulated by:
Ans: A a. Receptor
149) Chronic smoker working in a tyre b. Protein
factory presented with painless c. Hormone
haematuria & histopathology shows d. Enzymes
transitional cell bladder carcinoma Ans: A
most likely cause: 155) Man with blood group A negative
a. Aromatic amines can‟t receive blood from:
b. Tobacco smoking a. A+Ve
c. Nitrites b. A-ve
d. Aluminium c. AB-ve
Ans: B d. O-ve
Explanation: Smoking is the overall commonest Ans: C
cause for bladder carcinoma followed by aromatic Explanation: AB is universal acceptor and it can
amines. only donate blood to AB blood groups.
150) A non-insulin dependent diabetic and 156) Bone density is best assessed by:
hypertensive female patient had a. Dexa scan
Cholecystectomy. On the 3rd post- b. MRI
operative day, she complained of c. Ultrasound
sudden onset of dyspnea, chest pain d. X-ray
and haemoptysis. The most likely Ans: A
cause of her symptoms would be: 157) Question regarding liver grauloma:
a. Myocardial infarction a. Coagulative necorsis
b. Pneumonia b. Caseous necrosis
c. Pulmonary embolism c. Liquefective necrosis
d. Septicaemia d. Fat necrosis
e. Spontaneous pneumothorax Ans: B
Ans: C 158) Diaphragm development:
151) Ureter can be damaged at pelvic brim a. With transverses abdominis and costal
while ligating: margins
a. Uterine artery b. With external oblique muscle
b. Internal iliac artery c. With internal oblique muscle
c. Ovarian artery d. From septum transversum
d. Aorta Ans: D
Ans: C 159) Hasselbachtriangle‟s boundaries were
Explanation: The artery at pelvic brim is ovarian mentioned and associated hernia was
artery. asked:
152) Root value of femoral nerve is: a. Inguinal hernia
a. L2, L3 b. Indirect inguinal hernia
b. L2, L3, L4 c. Direct inguinal hernia
c. S1, S2 d. Femoral hernia
d. T2, T6 Ans: C
e. L1, L2 160) Most prominent prepotential:
Ans: B a. SA Node
153) Scenario regarding raised JVP, Low b. AV node
BP and high lactate level. Most likely c. Purkinjee fibers
cause: d. Bundle of HIS
a. Hypovolumic shock Ans: A
b. Cardiac tamponade 161) Taste sensation from anterior 2/3rd of
c. Septic shock tongue:
d. Cardiac shock a. Facial nerve
SK Origional – Golden 11 259

b. Trigeminal nerve b. Mean


c. Vagus nerve c. Mode
d. Abducent nerve d. Standard deviation
Ans: A Ans: A
Explanation: Taste sensation from anterior 2/3rd 168) 6 year old child having H/O
of tongue is carried by chorda tympani nerve prolonged bleeding during
that‘s branch of facial nerve. circumcision. He had tender joint.
162) German measles causes which What should be done to confirm the
abnormality: diagnosis?
a. Congential cataract a. APTT
b. Congential deafness b. PT
c. Cardiac abnormality c. Factor 8 assay
d. Limb defect d. Factor 9 assay
Ans: B Ans: C
Explanation: Explanation: In haemophilia the initial test is
 The commonest abnormality seen with Aptt while the confirmatory and best test is factor
german measles is congenital deafness. 8 assay as classical haemophilia or haemophilia A
 Congenital cataract is seen in first 7 is commonest among all hemophilias.
weeks. 169) Decrease sympathetic outflow leads to
163) Offspring have 50% chances of shock due to:
developing disease and there‟s no a. Loss of vasovagal tone
carrier: b. Decrease vasomotor tone
a. Autosomal dominant c. Acidosis
b. Autosomal recessive d. Increase CVP
c. X linked recessive e. Increase cardiac output
d. X linked dominant Ans: B
Ans: A 170) Scenario of Megaloblastic anaemia
164) Puerperal sepsis not responding to IV and peripheral neuropathy. Likely
fluids is usually responsive to: cause:
a. IV antibiotics a. Vitamin B12
b. IV steroids b. Folic Acid
c. IV vasoactive agents c. Iron
d. IV antiviral agents d. Vitamin B2
Ans: A Ans: A
165) Thyroxine derived from: Explanation: Peripheral neuropathy in
a. Tyrosine megaloblastic anemia is seen with Vitamin B12
b. Thyroglobulin deficiency most likely.
c. Tryptophan 171) Initial mediator of Inflammation:
d. Niacin a. Histamine
Ans: A b. Prostaglandin
166) Lymphatic drainage of area around c. Corticosteroids
Anus: d. Basophils
a. Medial horizontal group of superficial Ans: A
inguinal nodes Explanation:
b. Lateral horizontal group of superficial  Initial mediator of inflammation is
inguinal nodes Histamine.
c. Internal iliac nodes  Most important mediator of inflammation
d. Pudendal nodes is prostaglandin.
Ans: A
167) Mid value in upper and lower limit is
called: ===============
a. Median
SK Origional – Golden 11 260

GYNAE & OBS b. Iron deficiency Anemia


th
September 30 , 2020 – Afternoon c. Sidereoblastic Anemia
d. Thalassemia
Ans: A
1) Rh Blood group antigen present on:
a. Not present in tissues other than blood Explanation: Megaloblastic anemia is associated
with increased MCV & sub-acute combined
b. Autosomal recessive
degeneration of spinal cord that leads to difficulty
c. Present on Hemoglobin
in walking.
d. Present on RBC membrane
6) Post-operative patient taking
Ans: D
gentamicin having problem in which
Explanation: Rh blood group antigen is found
of the following:
on RBC surface > Glycoprotein > Immunogenic
a. Excretion
> Dominant Mendalian Inheritance.
2) In Rh Blood group, Rh agglutinins: b. Distribution
c. Absorption
a. Are always present in Rh negative blood
d. Potency
b. Are predominantly 1 gm
Ans: A
c. Are present on red cell membrane
Explanation: Gentamicin is eliminated through
d. Develop in Rh negative woman pregnant
renal system and in post-operative patient there‘s
with Rh positive fetus
chances of dehydration that‘s why it will lead to
e. Do not cross the placenta in the first
difficulty with excretion of the drug.
trimester of pregnancy
Ans: D 7) Most potent antioxidant:
a. Vitamin C
Explanation: Erythroblastosis fetalis is the
b. Vitamin A
disease of the fetus and newborn infants,
c. Vitamin E
characterized by agglutination and hemolysis of
d. Vitamin B
RBCs due to incompatibility between child‘s and
Ans: C
mother Rh factors.
Explanation: Most potent antioxidant is
3) Patient having diabetic retinopathy &
Glutathione but that wasn‘t in options so next we
IHD developed chest pain due to:
a. Thrombosis have to choose Vitamin E among the given
options followed by Vitamin C & Vitamin A.
b. Embolism
8) End arteries are the branches of:
c. Coronary artery disease
a. Coronary Arteries
d. DVT
b. Central Retinal Artery
Ans: B
c. Mesenteric Artery
Explanation: In DVT & prolong immobilization
d. Aorta
history the patient presents with chest pain & that
Ans: B
most likely indicates embolism of the detached
thrombus. Explanation: End arteries can be found both in
coronary arteries & Central retinal artery while in
4) Correct sequence of medical ethics:
the preferences order we prefer Central retinal
a. Autonomy, Justice & Consent
artery more than coronary artery.
b. Autonomy, Non-maleficence & Consent
9) Circle of willis is formed by:
c. Autonomy, beneficence, non maleficence
a. One anterior cerebral arteries
& Justice
b. Three posterior cerebral arteries
d. Autonomy & Justice
c. Two Posterior Communicating arteries
Ans: C
Explanation: Remember ABNJ (Autonomy, d. Two anterior communicating arteries
Ans: C
Beneficence, Non-Maleficience & Justice)
Explanation: Circle of willis is formed by two
.
anterior cerebral arteries, two posterior cerebral
5) 80 years old developed difficulty in
arteries, two posterior communicating arteries &
walking with increased MCV is most
one anterior communicating artery.
likely suffering from:
10) After pelvic fracture patient went into
a. Megaloblastic Anemia
shock due to:
SK Origional – Golden 11 261

a. Hemorrhage the adaptation time for many carotid and


b. Air Embolism aortic baroreceptors. Conversely, some of
c. Pulmonary Embolism the non-mechanoreceptors and pain
d. Cardiogenic Shock receptors for instance probably never
Ans: A adapt completely.
Explanation: 15) Slowest nerve fiber:
 Hemorrhage due to femoral artery or iliac a. C fiber
artery rupture b. A fiber
 For Air embolism there will be history c. B fiber
procedure like passing CVP then d. A delta fiber
developed SOB Ans: A
 For pulmonary embolism there will be Explanation: Among the mentioned fibers ―C-
history of stasis like patient with long Fibers‖ are the slowest nerve fibers that transmit
bones fracture was operated and on 3rd slow pain.
day developed sudden SOB that‘s more 16) Tachyphylaxis:
like pulmonary embolism a. Rapidly decrease responsiveness to a drug
11) Vitamin A is maximally found in: b. Slow decrease responsiveness to a drug
a. Carrots c. Sudden decrease response to a drug
b. Oranges d. Rapid decrease in action
c. Apples Ans: A
d. Banana Explanation:
Ans: A  Pharacodynamics: Drug action on the
12) Angular chelosis is caused by body
deficiency of:  Pharmacokinetics: Action of body on the
a. Riboflavin drug
b. Panothenic acid  Tachyphylaxis: Rapid decrease in
c. Vitamin A responsiveness to a drug
d. Vitamin C 17) Patient having Hb 8 gm/dl, ferritin
Ans: A 600mg/dl & retic count 10% with hx
13) Increase protein intake requires which of gallstones:
vitamin side by side: a. Hemolytic anemia
a. Riboflavin b. Megaloblastic anemia
b. Thiamine c. Sideroblastic anemia
c. Biotin d. Pernicous anemia
d. Vitamin A Ans: A
Ans: A Explanation: The above scenario clearly points
Explanation: toward hemolytic anemia as the reticulocyte count
 Increase protein intake requires is increased along with increased ferritin and gall
Riboflavin (B2) stones history.
 Increase Fat intake requires biotin (B7) 18) Lung compliance is increased in:
 Increase Carbohydrates require Thiamine a. Bronchial Obstruction
(B1) b. Decrease atmospheric pressure
14) Non adaptive receptors: c. Increase elastic recoil
A. Nociceptors (Pain Receptors) d. Old age
B. Baroreceptors e. Stiff lungs
C. Free nerve endings Ans: D
D. Pacinin Explanation: In old age stiffness of lungs
Ans: A decreases and hence lung compliance increases.
Explanation: 19) Maximum aldosterone secretion is
 The longest measured time for almost stimulated by:
complete adaptation of a a. Increase K+
mechanoreceptor is about 2 days which is b. Decrease K+
SK Origional – Golden 11 262

c. Thirst c. Firing of baroreceptor


d. Decrease Sodium d. Firing of chemoreceptor
Ans: A Ans: C
Explanation: Aldosterone is maximally Explanation: Whenever there‘s low BP or
stimulated by hyperkalemia as aldosterone will hemorrhage the baroreceptor detects it and hence
lead to excretion of Potassium & absorption of there‘s decrease firing of baroreceptor that leads
Sodium. to increasing BP & Sympathetic outflow &
20) Which of the following is beta-2 decrease Parasympathetic outflow.
adrenergic drug? 25) The ability of the kidney to excrete
a. Salbutamol concentrated urine will increase if:
b. Propranolol a. The permeability of the proximal tubules
c. Prazosin to water decreases
d. Metoprolol b. The rate of blood flow through the
Ans: A medulla decreases
Explanation: Among the aforementioned drug c. The rate of flow through the LOH
only Salbutamol is beta-2 adrenergic agonist while increases
rest like Propranolol & metoprolol are beta d. The activity of Na-K pump in the LOH
adrenergic antagonist while prazosin is alpha-1 decreases
receptor blocker. e. The permeability of the collecting duct to
21) The network of arteries inside water decreases
glomerulus: Ans: B
a. Tuft of arteries Explanation: The rate of blood flow decreases in
b. Tuft of Capillaries hypovolemia through medulla & hence
c. Tuft of Veins concentrated urine is produced.
d. Tuft of Venules 26) Flexor reflex is carried by:
Ans: B a. Monosynaptic
Explanation: Glomerulus consists of tuft of b. Noxious Stimulus
capillaries those are branches of afferent renal c. Mechanoreceptor
arterioles. d. Cerebral cortex
22) Highest systolic pressure is present in: Ans: B
a. Renal artery 27) Linear glomerular basement
b. Aorta membrane deposits:
c. Pulmonary Artery a. Good Pastures Syndrome (GPS)
d. Carotid artery b. FSGC
Ans: A c. IGA Nephropathy
Explanation: Highest systolic pressure is present d. SLE
in large arteries distal to heart like renal artery. Ans: A
23) Stroke volume is equal to: Explanation: Typical linear deposits of IgG along
a. CO/HR the glomerular basement membrane are the
b. HR/CO characteristic of GPS.
c. CO * HR 28) High altitude will causes:
d. HR * TPR a. Decrease Respiratory rate
Ans: A b. Pulmonary Vasoconstriction
Explanation: c. Tachycardia
 CO = SV * HR d. Decrease Hemoglobin concentration
 SV = CO/HR Ans: B
24) Patient presented in shock with Explanation: Prefer increase Respiratory rate if
decreased MAP & decreased TPR present; if not then go for pulmonary
which one of the following parameter vasoconstriction as in this case.
will decrease to increase the BP: 29) Splenic Immunologic response seen
a. Veins Compliance in:
b. Resistance a. Red pulp
SK Origional – Golden 11 263

b. White pulp d. Hemophilus


d. Splenic Arteries Ans: A
e. Splenic Venules 35) A young patient presented with
Ans: B chronic cough and thick currant jelly
Explanation: sputum with CXR showing pleural
 According to First Aid page # 98, white effusion is most likely caused by:
pulp is made up of rmarginal zone, a. Klebsiella Pneumoniae
periarteriole lymphatic sheath & follicles b. Streptococcus Pnumoniae
which plays an important role in immune c. Cystic Fibrosis
system by producing T cells, macrophages d. Tuberculosis
and B cells. Ans: A
 Red pulp is only involved in destroying Explanation: The clue toward Klebsiella
old and abnormal RBCs organism is thick currant jelly sputum.
36) Cortical area with maximum
30) 0.5 gram/dl protein is present in representation:
which fluid? a. Lips
a. Lymph b. Nose
b. CSF c. Little Finger
c. Saliva d. Knee
d. Tears Ans: A
Ans: A Explanation:
31) Nerve supply of SCM  Maximum sensory cortical area is Lips
(Sternoclidomastoid muscle):  Maximum motor cortical area is Thumb
a. Facial nerve > Hand.
b. Accessory nerve 37) 70 kg man normal urine output:
c. Vagus nerve a. 1 Liter
d. Trigeminal nerve b. 1.5 Liter
Ans: B c. 500 ml
32) True about motor cortex, only present d. 1.2 Liter
in: Ans: B
a. Frontal Lobe Explanation: Normal urine output in a 70 kg
b. Temporal Lobe man in 24 hours is 1500 – 2200 ml.
c. Parietal Lobe 38) Maxillary artery is the branch of which
d. Occipital Lobe branchial arch:
Ans: A a. 1st Arch
Explanation: Precentral gyrus that‘s motor cortex b. 2nd Arch
is present in frontal lobe while sensory cortex is c. 3rd Arch
present in parietal lobe. d. 4th Arch
33) Name intrauterine growth hormone: Ans: A
a. Thyroid Hormone (T3) Explanation:
b. Thyroxine (T4)  Maxillary artery is the branch of 1st arch
c. Reverse T3 (rT3) (Mandibular Arch)
d. Tyrosine  The ventral end of the second develops
Ans: A into the ascending pharyngeal artery & its
Explanation: T3 is the active thyroid hormone & dorsal end gives origin to Stapedial artery
has beneficial role in fetal mental health  The third arch gives raise to the proximal
development. end of internal carotid artery
34) Most common pathogen causing ward  The fourth arch forms the right
infection: subclavian artery
a. Staph Aureus  The fifth arch regresses
b. Pseudomonas  The sixth arch persist as proximal portion
c. Streptococcus of right pulmonary artery
SK Origional – Golden 11 264

39) Osteoporosis shows which of the b. Tuberculosis


following lab values? c. Staph Aureus
a. Normal PTH d. Hemophilus
b. Increase PTH Ans: A
c. Decrease PTH Explanation: Lesion on skin & that too on face
d. Increase Calcium with granulomatous findings is most likely Tb
Ans: A leprae.
Explanation: 44) Most common local factor for delayed
 Osteoporosis indicates normal calcium, wound healing:
phosphorus & PTH level a. Foreign body
 Osteomalacia shows Low calcium, High b. Trauma
or low Phosphate & normal PTH c. Decrease blood supply
 Hyperparathyrodism shows high calcium, d. Infection
low phosphate & high PTH Ans: D
40) A patient had trauma to femur now Explanation:
presented with bony growth in soft  Most common local factor for delayed
tissue due to: wound healing is infection
a. Metaplasia  Most common systemic factor for delayed
b. Neoplasia wound healing is Diabetes Mellitus
c. Dysplasia 45) Most specific indicator in urine for
d. Abscess diabetic patient is:
Ans: A a. Microalbuminiuria
Explanation: This is the case of “myositis b. Nitrates
ossificans”is essentially metaplasia of the c. Increase Urinary Albumin
intramuscular connective tissue resulting in extra d. Increase Urinary Globulin
osseous bone formation without inflammation. Ans: A
41) Endotoxic shock is initiated by: 46) Newborn with recurrent respiratory
a. Endothelial injury infection which of the following
b. Peripheral vasodilation physiological antibody is missing:
c. Increase vascular permeability a. IgA
d. Cytokine action b. IgG
Ans: D c. IgM
Explanation: Sequence of production of d. IgD
mediators in endotoxic shock: LPS (Endotoxin) Ans: B
followed by TNF, IL-1, IL-6, IL-8 and NO, PAF. Explanation:
42) Patient presented with lumber pain  Remember Newborn (NG) & the
& Pus in urine with no response to deficient antibody is IgG
antibiotics within 2 weeks, what could  In Child the deficient antibody could be
be the possible diagnosis: IgA
a. Acute Pyelonephritis 47) Pulmonary valve opens at which of the
b. Chronic Pyelonephritis following pressure?
c. UTI a. 80 mmhg
d. Renal Carcinoma b. 120 mmhg
Ans: A c. 8 mmhg
Explanation: As the patient still has symptoms & d. 25 mmhg
no improvement with antibiotics, it‘s most likely Ans: C
acute pyelonephritis as the treatment takes 3-4 Explanation:
weeks of IV antibiotics to respond.  Pulmonary valve opens at 8 mmhg
43) Lesion near the orbit of the eye on  Aortic valve opens at 80 mmhg
histopathology shows langhans giant 48) Difference between pulmonary &
cells: systemic circulation:
a. Tb Leprae a. Pulmonary resistance lower compare to
SK Origional – Golden 11 265

systemic d. Renin
b. Systemic resistance lower than pulmonary Ans: A
c. Pulmonary Resistance equal with systemic Explanation: Vasopressin (ADH) absorbs water
d. None & decreases plasma osmolarity.
Ans: A 54) Fever with lymphadenopathy & EBV
49) Confidence interval is: infection:
a. Mean & its standard error a. Burkitt‘s Lymphoma
b. Standard error of mean & T-value b. AML
c. Mean & Mode c. CML
d. Median & Standard Error d. IM
Ans: A Ans: A
Explanation: Explanation: Burkitt lymphoma is caused by
 Confidence interval: Mean & its standard EBV & presents with cervical lymphadenopathy.
Error 55) Which of the following increases
 Confidence limit: Standard error of mean during sleep?
& T-value a. Insulin
50) Trachea starts at which level: b. ADH
a. C6 c. Growth Hormone
b. C7 d. Renin
c. C4 Ans: C
d. C5 Explanation: Cortisol, melatonin & prolactin
Ans: A increases during sleep but as these weren‘t in
Explanation: Trachea starts at C6 level while options, the next among these are GH.
trachea bifurcates at T4-5 > T5. 56) Cytoskeleton connected to ECM by:
51) Septal blood supply: a. Intermediate filament
a. Anterior Interventricular Artery b. Proteoglycan
b. Posterior Interventricular Artery c. Integrin
c. Right Marginal Artery d. Cadehrin
d. Left Marginal Artery Ans: C
Ans: A Explanation:
Explanation:  Cytoskeleton connected to ECM by
 Anterior 2/3rd of septum is supplied by: Integrin
Anterior interventricular Artery  ECM connected to ICM by Intermediate
 Posterior 1/3rd of septum is supplied by filament
posterior interventricular Artery 57) Atlas has no:
 Hence, Anterior interventricular Artery > a. Spinous process
Posterior Interventricular Artery b. Articulation with Skull
52) Epithelium of Main salivary duct: c. Lumen
a. Stratified cuboidal d. Arch
b. Simple Cuboidal Ans: A
c. Simple Columnar Explanation: Atlas is made of a thick anterior
d. Stratified Columnar arch and a think posterior arch with lumen & skull
Ans: D articulation but has no spinous process.
Explanation: Epithelium of Main salivary gland 58) Atlanto-axial joint causes which
duct is lined by Stratified columnar epithelium movement?
while epithelium of sweat gland main duct is lined a. Rotation
by stratified cuboidal epithelium. b. No movement
53) Which of the following decreases c. Yes
osmolarity? d. None
a. Vasopressin (ADH) Ans: A
b. Epinephrine Explanation:
c. Aldoesterone  Atlanto-Axial joint produces rotational
SK Origional – Golden 11 266

movement that‘s when we say ―NO‖ b. Spinal nerve+ Sympathetic ganglion


 Atlanto-Occipital joint produced c. Spinal nerve + Parasympathetic ganglion
Flexion/Extension movement that‘s d. Cranial + Sympathetic ganglion
when we say ―YES‖ Ans: A
59) Hemoglobin synthesis begins at 64) Extreme sympathetic stimulation
which stage? leads to:
a. Early normoblast a. Bronchoconstriction
b. Intermediate normoblast b. Bronchodilation
c. Late normoblast c. Bradycardia
d. Reticulocyte d. Miosis
Ans: A Ans: B
Explanation: Explanation: Sympathetic stimulation leads to
 Hemoglobin synthesis starts at early bronchodilation, tachycardia & mydriasis.
Normoblast, prefer proerythroblast if 65) Parasympathetic effect on the heart:
present in options a. Decrease HR
 In intermediate normoblast hemoglobin b. Increase HR
starts to appear c. Increase TPR
 In late normoblast nucleus disappear d. Increase Cardiac Contractility
 Reticulocyte contains the highest amount Ans: A
of Hemoglobin Explanation: Parasympathetic effect on heart
60) Upper outer quadrant of breast is leads to decrease Heart Rate, decrease cardiac
drained by which of the following contractility & decrease TPR.
lymph nodes? 66) Which of the following causes
a. Anterior pectoral (Axillary) Lymph Nodes thrombocytopenia?
b. Posterior Lymph Nodes a. Cloramphenicol
c. Internal thoracic lymph nodes b. Thiazide
d. Supraclavicular lymph nodes c. Furosamide
Ans: A d. Quinine
Explanation: Upper outer quadrant of breast is Ans: D
drained by Anterior (Pectoral) Axillary lymph Explanation:
nodes & that‘s what drains the maximum of breast  Follow the following sequence for
too. thrombocytopenia:
61) Maximum lymphatic drainage of  Heparin > Quinine > Thiazide
breast is parallel with which of the 67) Proliferation of renal system:
following artery? a. Mesangium
a. Axillary Artery b. Cortex
b. Internal carotid artery c. Medulla
c. Internal thoracic artery d. Afferent Arteriole
d. Subclavian artery Ans: A
Ans: A 68) Type of HSR in grave‟s disease:
Explanation: Maximum of breast is drained via a. Type 1 HSR
Anterior (Pectoral) Axillary lymph nodes & that‘s b. Type 2 HSR
parallel with Axillary artery. c. Type 3 HSR
62) Peptic ulcer pain is transmitted via: d. Type 4 HSR
a. Greater Splanchnic Nerve Ans: B
b. Lesser Splanchnic Nerve Explanation:
c. Least Splanchnic Nerve  Grave‘s disease is associated with Type-2
d. Costal Nerve HSR
Ans: A  Hashimoto thyroiditis is associated with
63) Peripheral nervous system (PNS) Type-4 HSR
comprises of: 69) Type of HSR in myasthenia Gravis:
a. Cranial + Spinal nerve a. Type 1 HSR
SK Origional – Golden 11 267

b. Type 2 HSR Ans: B


c. Type 3 HSR Explanation:
d. Type 4 HSR  Intraepithelial lesion on vulva: Paget
Ans: B disease
Explanation:  Ulcer on Vulva: Condyloma & is caused
 MG is type-2 HSR by HPV
 SLE is type-3 HSR 76) Metastasis is pathognomic for:
70) Hemoptysis & hematuria a. Carcinoma
characteristically seen in: b. Dysplasia
a. IGA nephropathy c. Meteplasia
b. Good Pastuers Syndrome (GPS) d. Desmoplasia
c. SLE Ans: A
d. DM Explanation: Carcinoma is characterized by
Ans: B Metastasis > Invasion > Pleomorphism >
71) First meiotic division starts at: Increase N/C ratio.
a. Begins from fetal life 77) Protrusion of jaw is caused by which
b. Begins in puberty of the following muscle?
c. Begins during ovulation a. Medial Pterygoid
d. Begins during fertilization b. Lateral Pterygoid
Ans: A c. Buccinator
Explanation: First meiotic division starts at fetal d. Platysma
life & got arrested in diplotene stage. Ans: B
72) Chromosome exchange between Explanation: Lateral pterygoid muscle causes
autosomes: protrusion & depression of the jaw.
a. Miosis 78) Immediate effect of Insulin:
b. Mitosis a. Entry of K+ into cells
c. Binary Vision b. Protein Synthesis
d. Prophase c. Entry of glucose into cell
Ans: A d. Lipogenesis
73) Characteristic of irreversible heart cell Ans: A
injury: Explanation:
a. Cell Swelling  Immediate effect of Insulin: Entry of K+
b. Mitochondrial Swelling into cells
c. Contraction band necrosis  Intermediate effect of Insulin: Protein
d. Lysosomal Release Synthesis
Ans: C  Prime function of Insulin: Entry of
74) Change in the following electrolyte glucose into cell
has profound effect on the heart:  Delayed action of Insulin: Lipogenesis
a. Calcium 79) Glucagon secretion is increased by:
b. Sodium a. Increase Glucose
c. Nitrogen b. Somatostatin
d. Potassium c. Exercise
Ans: D d. Fatty Acids
Explanation: Change in Potassium has profound e. Secretin
effect on the heart that‘s hypokalemia makes a Ans: C
flaccid heart & hyperkalemia leads to arrhythmias. 80) Short term response to blood pressure:
75) Condyloma Accuminatum occurs due a. Baroreceptor
to: b. RAAS
a. HSV c. CNS Ischemic Response
b. HPV d. ADH
c. Chlamydia Trachomatis Ans: A
d. Gonococcus Explanation:
SK Origional – Golden 11 268

 First response to change in arterial b. Bronchi


pressure which occurs within seconds is c. Bronchioles
baroreceptor response. d. Bronchopulmonary Segment of lung
 Long term blood pressure control is Ans: D
through RAAS. 87) Which antimetabolite can be used for
 Most potent response is CNS ischemic non-neoplastic purposes?
response. a. Methotrexate
81) Which of the following will be b. Cytarabine
increased in hypothyroid patient? c. 5 Flurouracil
a. T3 d. Mercaptopurine
b. T4 Ans: A
c. T3 Explanation:
d. Cholesterol  Methotrexate is anti-metabolite that has
Ans: D non-neoplastic functions as well such as
82) ESR is decreased by increase of which used in termination of ectopic pregnancy.
of the following?  Azathoprine which is anti-metabolite and
a. Increase Plasma globulin to albumin ratio has non neoplastic & immunosuppressive
b. Increase fibrinogen properties.
c. Increase plasma Albumin 88) Structure found 2 feet away from
d. Temperature ileocecal junction, at anti-mesenteric
Ans: C boarder is most likely:
Explanation: Increase Albumin will decrease a. Omphalocele
ESR while decrease albumin will increase ESR. b. Meckel‘s diverticulum
83) Finger lacerated what will be c. Volvulus
immediate response? d. Gastroschisis
a. Vasoconstriction e. Intussuception
b. Vasodilation Ans: B
c. Increase Blood flow 89) Phagocytosis in blood is done by:
d. Decrease Blood flow a. Neutrophils
Ans: A b. Lymphocytes
84) Which of the following is chemical c. Basophils
carcinogen? d. Macrophages
a. Benzidine Ans: A
b. Ethyl Alcohol 90) Vitamin-K dependent factor:
c. Methanol a. Factor 12
d. Alpha 1 Antitrypsin b. Factor 5
Ans: A c. Calcium
Explanation: Benzidine is aromatic amine & d. Prothrombin
comes under the list of chemical carcinogen. Ans: D
85) Investigation of choice for infective Explanation: Vitamin K dependent factors are 2,
endocarditis: 7, 9, 10.
a. Blood Culture 91) Oxidases are present in:
b. Urine culture a. Peroxisome
c. ASO titer b. Lysosome
d. None c. Mitochondria
Ans: A d. Nucleus
Explanation: Most preferable test in infective Ans: A
endocarditis is blood culture followed by ECHO 92) Normal adult hemoglobin:
to confirm the diagnosis. a. Gower Hb
86) Segmental bronchus aerates which of b. HbA
the following? c. HbF
a. Trachea d. HbS
SK Origional – Golden 11 269

e. HbA2 d. Rupture of Aorta


Ans: B Ans: B
93) RTA case presented with toxemia & 99) Ureter is at risk of damage during
necrosis: hysterectomy:
a. Septic Shock a. Broad Ligament
b. Hypovolumic Shock b. Pelvic Brim
c. Cardiogenic Shock c. Bifurcation of common iliac artery
d. Cardiac Tamponade d. Bifurcation of Aorta
Ans: A Ans: A
94) Fibroblast produces: Explanation: Ureter is at risk of damage during
a. Collagen hysterecotomy at the following site: Cardinal
b. Amorphous ground substance Ligament > Broad Ligament > Pelvic Brim.
c. Elastin 100) Spinal Anesthesia is given in:
d. Hyaline a. Subarachnoid Space
Ans: B b. Epidural Space
Explanation: c. Extra Dural Space
 Fibrocartilage has collagen d. None
 Fibroblast has amorphous ground Ans: A
substance 101) Little girl developed purpura after
95) Vein accompanying posterior vaccination:
interventricular artery: a. TTP
a. Middle cardiac vein b. ITP
b. Anterior cardiac vein c. VWD
c. Posterior cardiac vein d. Hemophilia
d. Great cardiac vein Ans: B
Ans: A 102) Half-life of Cobalt:
Explanation: a. 5 years
 Vein accompanying posterior b. 10 years
interventricular artery is middle cardiac c. 15 years
vein. d. 20 years
 Vein accompanying anterior Ans: A
interventricular artery is great cardiac 103) Type of regional anesthesia associated
vein. with greater serum concentration of
96) Most toxic anesthetic: local anesthetic:
a. Bupvicane a. Brachial Plexus
b. Procaine b. Epidural
c. Lidocaine c. Femoral Block
d. Lignocaine d. Cardiac Block
Ans: A e. Intercostal Block
97) Duration of action of protamine zinc Ans: E
insulin: 104) Wedge shaped fibromuscular
a. 6 hours structure in perineum:
b. 12 hours a. Levator Ani
c. 24 hours b. External Anal sphincter
d. 48 hours c. Perineal Body
Ans: C d. Iliacus
98) Dai handled DNC, patient afterward Ans: C
presented with hematoma most likely 105) Which of the following muscle is cut
due to: in mediolateral episiotomy?
a. Rupture of Ovarian Artery a. Bulbospongiosus
b. Rupture of Uterine Artery b. Levator Ani
c. Rupture of Internal Iliac Artery c. External Anal Sphincter
SK Origional – Golden 11 270

d. Iliacus Ans: A
Ans: A Explanation: Obtruator internus muscle is
Explanation: supplied by anterior division of sacral plexus.
 Structure cut in mediolateral episiotomy is 111) Which of the following forms pelvic
bulbospongiosus inlet boundary?
 Structure at risk during mediolateral a. Pubic Crest
episiotomy is Levator Ani b. Levator Ani muscle
 In Median Episiotomy structure at risk is c. Ischial Tuberosity
external anal sphincter d. Iliacus
106) Ultrasound wave frequency more Ans: A
than: 112) Post-operative pain relief in asthmatic
a. 5 khz patient:
b. 10 khz a. Keterolac
c. 15 khz b. Pethidine
d. 20 khz c. Paracetamol
Ans: D d. SSRI
107) Ultrasound wave frequency per Ans: B
second range: Explanation:
a. 5000 hz  Mild – Moderate pain is relieved with
b. 10,000 hz PCM
c. 15,000 hz  Severe Pain is relieved with Pethidine
d. 20,000 hz  Keterolac is contraindicated in Asthmatic
Ans: D patient
108) Main support of uterus: 113) 20 years old patient presented with
a. Lateral Cervical ligament increase urea, creatinine & increase
b. Uterosacral ligament K+ level:
c. Broad ligament a. CKD
d. Uterine ligament b. RTA
Ans: A c. DKA
Explanation: d. HONK
 Main support of uterus is: Lateral Cervical Ans: A
Ligament. 114) GFR in a healthy person is about:
 Dynamic Support of uterus is: Pelvic a. 100 ml/min
Diaphragm. b. 125 ml/min
 Associated support of Uterus is: c. 200 ml/min
Uterosacral Ligament. d. 250 ml/min
109) Diagnosis of thalassemia: Ans: B
a. Hb Electrophoresis 115) Distance from sacral promontory to
b. CBC inner part of pubic symphsis:
c. Bone marrow Biopsy a. True Conjugate
d. Serum Ferritin b. Obstetric Conjugate
Ans: A c. False Conjugate
Explanation: d. Diagonal Conjugate
 Thalassemia is diagnosed accurately with Ans: B
Hb Electrophoresis. Explanation:
 IDA is diagnosed with serum ferritin.  True Conjugate: Distance from sacral
110) Obtruator internus muscle is supplied Promontory to upper boarder of Pubic
by: Symphysis.
a. Sacral plexus  Obstetric Conjugate: Distance from
b. Femoral plexus Sacral Promontory to inner part of Pubic
c. Lumber Plexus Symphysis.
d. Tibial nerve  Diagonal Conjugate: Distance from Sacral
SK Origional – Golden 11 271

promontory to lower boarder of Pubic b. IV nitroglycerine


Symphysis. c. Methyldopa
 Remember ―TOD‖ from upper to lower. d. BB
116) Which antihypertensive drug is given Ans: B
to Asthmatic patient during surgery? 122) How prostaglandin causes ripening of
a. IV hydralazine cervix:
b. IV Nitroglycerine a. Hydration of collagen
c. IV Nitrate b. Hydration of Hyaline
d. CCB c. Hydration of Lysine
Ans: B d. None
117) Mid diastolic murmurs with irregular Ans: A
R-R interval most likely findings 123) Cervical lymphatic drainage:
would be: a. Internal + External Iliac Nodes
a. P-Pulmonale b. Internal Iliac nodes
b. P-Mitrale c. External Iliac Nodes
c. Pulsus Paradox d. Para Aortic Nodes
d. CHF Ans: A
Ans: B Explanation:
118) Grasp reflex ends at:  Body of uterus/cervix/superior part of
a. 02 months bladder: External iliac nodes.
b. 04 months  Prostate/Corpus Cavernosum/proximal
c. 06 months Vagina/Cervix: Internal iliac nodes.
d. 08 months 124) Which of the following drug has
Ans: C decrease risk of nausea & vomiting?
Explanation: Grasp reflex starts at term & ends a. Ketamine
at 5-6 months. b. Propofol
119) Primary oocyte surrounded by zone c. Thiopentone
pellucida, zona granulosa and fluid d. Halothane
filled cavity is called: Ans: B
a. Primary Follicle Explanation: Pain on injection seen in upto 20%,
b. Secondary Follicle mild anti-emetic properties is seen with propofol.
c. Tertiary Follicle 125) A mother brings her 5 years old
d. Antral Follicle daughter to the ER and relates a 4
Ans: B days history of sour throat & painless
Explanation: but persistent serosangious discharge
 Primary Oocyte surrounded by zona from the child‟s vagina. Pelvic
pellucida & having fluid filled cavity is examination confirms history and the
called secondary follicle. saline/potassium chloride preparation
 Oocyte surrounded by flat epithelial cells are negative. The most likely etiology
is called primary follicle. is:
120) Epithelium that‟s partially ciliated & a. Candida
partially non-ciliated with numerous b. FB
folding & crypts with outer c. Neisseria gonorrhea
longitudinal & inner circular layer: d. Streptococcus
a. Fallopian Tube e. Trichomonas
b. Uterus Ans: D
c. Cervix Explanation: Serosangious discharge is seen in
d. Prostate Foreign body as well but here history of sore
Ans: A throat followed by serosangious discharge is most
121) Which of the following is potent likely due to streptococcus infection.
venodilator? 126) Fallopian tube distended with
a. CCB purulent discharge suspected of PID
SK Origional – Golden 11 272

is most likely caused by: disorder that‘s self-limiting.


a. Chlamydia 132) A patient having acromegaly
b. E. Coli &amenorrhea adenoma of:
c. Streptococcus a. Acidophils
d. Gonorrhea b. Adenohyophysis
Ans: A c. Basophils
Explanation: d. Posterior Pituitary
 The commonest cause of PID is Ans: A
Chlamydia followed by gonorrhea. 133) Decrease PO4, Increase Calcium will
 The commonest cause of Tubo-Ovarian decrease which of the following?
abscess is gonorrhea. a. 1,25 Vitamin D
127) Pregnant lady having cat as pet is at b. 24,25 Vitamin D
risk of: c. 1,24 Vitamin D
a. Toxoplasmosis d. Renin
b. Syphilis Ans: A
c. Gonorrhea Explanation: Increase calcium & decrease PO4
d. Chlamydia will lead to negative feedback and will decrease
Ans: A 1,25 vitamin D.
128) Test done before contacting the 134) A patient having chronic pancreatitis
disease is: will have deficiency of:
a. Primary Prevention a. Biotin
b. Secondary Prevention b. Vitamin D
c. Tertiary Prevention c. Ascorbic Acid
d. Primordial Prevention d. Cobalamine
Ans: A Ans: B
Explanation: 135) Granulosa cell tumour has:
 Primary Prevention: Prevention before a. Call Exener Bodies
disease & by promotion in health and b. Estrogen Bodies
protection c. Lutein Cyst
 Secondary Prevention: early diagnosis & d. None
treatment Ans: A
 Tertiary Prevention: To reduce disabilities 136) Lymphatic drainage of glans of
& impairments clitoris:
129) 02 days before Ovulation LH surge is: a. Superficial inguinal nodes
a. 2 folds b. Deep Inguinal nodes
b. 6 – 10 folds c. Internal Iliac nodes
c. 20 folds d. External Iliac nodes
d. 30 folds Ans: B
Ans: B Explanation: Skin of Glans drains to superficial
130) Vesico-Vaginal fistula type: inguinal nodes while glans itself drains into deep
a. Early division of ureteric bud inguinal nodes.
b. Late division of Ureteric bud 137) Thyroglobulins produced by:
c. Fault in mesonephric duct a. Thyroid follicular cells
d. Fault in paramesonephric duct b. Parathyroid Glands
Ans: A c. Papillary Cells
131) True regarding G6PD: d. Anaplastic Cells
a. Self-limiting Ans: A
b. Unlimiting Explanation: TG is produced by thyroid
c. Autosomal dominant follicular cells & secreted in papillary carcinoma.
d. X-Linked Dominant 138) Tumor marker of carcinoma of ovary:
Ans: A a. CA-125
Explanation: G6PD is X-Linked Recessive b. CA 19-9
SK Origional – Golden 11 273

c. CEA b. Epiblast
d. PSA c. Hypoblast
Ans: A d. Intermediate mesoderm
Explanation: Ans: D
 CA-125 is raised in ovarian carcinoma 146) Uterine changes during pregnancy:
 CA 19-9 is raised in pancreatic & a. Hyperplasia
colorectal carcinoma b. Hypertrophy
 CEA is raised in colorectal & pancreatic c. Hypertrophy + Hyperplasia
carcinoma d. Atrophy
 PSA is raised in Prostate carcinoma Ans: C
139) Antidote for atropine reversal: 147) Middle value with exact numbers
a. Neostagmine above & below it:
b. Physostagmine a. Median
c. Edrophonium b. Mode
d. Epinephrine c. SD
Ans: B d. Mean
140) Pink frothy vaginal discharge with Ans: A
vulval pruritus is caused by: 148) Muscle paralyzed during difficult
a. HSV child birth:
b. HPV a. Iliacus
c. Gonorrhea b. Levator Ani
d. Chlamydia c. Perineal body
e. Tichomonas Vaginalis d. Obtruator Internus
Ans: E Ans: B
141) Whole body Edema due to: 149) Superficial inguinal ring is traversed
a. Hypernatremia by:
b. CCF a. Ilioinguinal nerve
c. Excessive Renin secretion b. Genitofemoral nerve
d. Excessive Aldosterone secretion c. Iliohyopgastric nerve
Ans: B d. Obtruator nerve
142) Chorioamniotis most specific Ans: A
indicator: 150) Deep inguinal ring is formed by defect
a. IL-1 in:
b. IL-6 a. External oblique muscle
c. IL-8 b. Transversalis Fascia
d. TNF c. Internal Oblique muscle
Ans: B d. Psoas
143) A patient with yellowish vaginal Ans: B
discharge & motile trophozite: 151) Diabetic mother presented with
a. Gonorrhea respiratory distress fetus should be
b. Trichomonas Vaginalis given:
c. HPV a. Betamethasone
d. HSV b. Promethasone
Ans: B c. Momethasone
144) Opsonization is done by: d. Dexamethasone
a. FC portion of IgG & C3b Ans: A
b. FC portion of IgM Explanation: Dexamethasone can cause a bit of
c. FC portion of IgM & C5a neurotoxicity that‘s why betamethasone is
d. C5b & IgM preferred.
Ans: A 152) Inferior rectal artery is a branch of:
145) Genitourinary system is derived from: a. Internal pudendal artery
a. Lateral Plate mesoderm b. External Pudendal artery
SK Origional – Golden 11 274

c. Internal Iliac artery bacteroids.


d. Inferior Mesenteric artery  Pyogenic peritonitis is caused by
Ans: A bacteroids.
Explanation:  Only peritonitis caused by E. coli
 Superior rectal artery is branch of Inferior  Peuperal sepsis caused by: Group B strep
mesenteric artery. > Bacteroids.
 Middle rectal artery is branch of internal 158) Remnant of gubernaculum in female
iliac artery. is:
 Inferior rectal artery is branch of internal a. Ovarian Ligament
pudendal artery. b. Lateral Cervical ligament
153) Parturition is initiated by: c. Transverse cervical ligament
a. Oxytocin d. Broad ligament
b. Prostaglandins Ans: A
c. Fetal Cortisol Explanation:
d. Fetal Adrenals  Gubernaculum remnant in female is
Ans: C round ligament & ovarian ligament.
154) Histology of Vagina:  While in male it‘s Scrotal ligament.
a. Stratified columnar epithelium 159) Which of following hormone placenta
b. Simple cuboidal epithelium produces in last trimester?
c. Stratified Squamous Epithelium a. Estrogen
d. Simple Columnar Epithelium b. Progesterone
Ans: C c. Estrogen + Progesterone
155) Vagina has abundant: d. Relaxin
a. Elastic Tissue Ans: C
b. Elastic & Collagen Tissue 160) Which of the following is true
c. Hyaline Tissues hermaphrodite?
d. Hyaline & Elastic a. XXY
Ans: A b. XXX
Explanation: Vagina has abundance of Elastic c. XYY
tissues while it has both elastic and collagen d. XO
tissues. Ans: A
156) 6 years old girl presented to OPD with Explanation: XX > XX/XY > XXY.
excessive facial & chest hair, adult 161) Turner syndrome is associated with:
size external genitalis & deepening of a. Normal Ovaries
voice. She also gives history of two b. Streak Ovaries
siblings death suffering from same c. Postural Hypotension
condition which test would best d. Anemia
confirm the diagnosis: Ans: B
a. 17 Alpha Hydroxylase deficiencies 162) Tumour marker of HCC:
b. 21 Hydroxylase a. AFP
c. 12 Beta Hydroxylase b. B-HCG
d. 22 Hydroxylase c. CA-125
Ans: B d. CA 19-9
157) Post hysterectomy infection caused Ans: A
by: 163) Pregnant lady with delayed reach to
a. Bacteroids hospital presented with dyspnea most
b. Staph Aureus likely due to:
c. E. coli a. Fat Embolism
d. Chlamydia b. Air Embolism
Ans: A c. Amniotic fluid embolism
Explanation: d. Hemorrhage
 Post hysterectomy infection is caused by Ans: C
SK Origional – Golden 11 275

Explanation: Pregnant lady with dyspnea without 170) Appendicular artery is a branch of:
any other factors like trauma is most likely a. Posterior cecal artery
amniotic fluid embolism. b. Iliocolic artery
164) Suckling response causes: c. Middle colic artery
a. Decrease GnRH d. Superior mesenteric artery
b. Decrease FSH Ans: B
c. Decrease LH Explanation:
d. Decrease Oxytocin  According to RJ last Iliocolic artery give
Ans: A raise to appendicular artery
165) Primitive streak mesoderm migrates  Snell however, mentions Posterior cecal
to anterior abdominal wall around artery
cloaca. A rupture of this mesoderm  Follow RJ last
cranially will produce: 171) Virus causes cancer due to alteration
a. Ectopic anal canal of:
b. Imperforate anus a. Protein
c. Bladder extrophy b. DNA
d. None c. RNA
Ans: A d. Ribosome
Explanation: Ans: A
 Cranial opening: Ectopic anal canal Explanation:
 Caudal Opening: Extrophy of bladder  Virus has oncogenes
166) Bulbourethral gland is present in:  Virus cause cancer by proto-oncogenes
a. Superficial perineal pouch  Virus cause cancer by alteration of
b. Deep perineal pouch protein.
c. Intermediate pouch 172) Body/tissue source of cholesterol:
d. Sphincter a. LDL
Ans: B b. HDL
167) A tall man with normal external c. VLDL
genitalia married but had no kids, he d. Chylomicrons
had normal ejaculation period but Ans: A
semen analysis showed azospermia 173) Hypersensitivity reaction is reduced
cause maybe in: in:
a. Epidydymis a. Immunocompromised
b. Sertoli cells b. Immunocompetent
c. Scrotum c. Malignancy
d. Seminiferous Tubules d. Hepatitis
Ans: B Ans: A
Explanation: Sertoli cells found in seminiferous 174) Most common cause of
tubules and play important role in hydrocephalus:
spermatogenesis. a. Cerebral aqueduct stenosis
168) Decrease surfactant leads to: b. Increase CSF production
a. RDS c. Lateral Ventricles obstruction
b. Immature Heart d. 3rd ventricles obstruction
c. Immature intestine Ans: A
d. Immature Brain 175) A patient with abdominal pain,
Ans: A bloating and foul smelling most useful
169) Heart can‟t be tetanized due to: investigation would be:
a. Short refractory period a. Stool D/R
b. Long refractory period b. Culture
c. Gap junctions c. Gram staining
d. Desmosome d. Colonscopy
Ans: B Ans: A
SK Origional – Golden 11 276

Explanation: Stool D/R (Detailed Report) gives


information about all stool contents foul smelling GYNAE &OBSTETRIC
stools are due to fat malabsorption i.e steatorrhea November 25th, 2020 – Morning
& culture and sensitive has nothing with it.
176) AJK earthquake leading to death of his 1) A patient With T.B undergoing
uncle & now in depression:
treatment complains of red-green
a. Social Crisis
color blindness. Which of the
b. Situational Crisis
following drug is responsible for this?
c. Emotional Crisis
A. Ethambutol
d. Developmental Crisis
B. Pyrazinamide
e. None
C. Rifampicin
Ans: B
D. Isoniazid
177) To rule out false positive: E. Streptomycin
a. Specificity
Ans: A
b. Sensitivity
2) Which ligament keeps vertebral
c. True Negative
bodies in place?
d. Positive predictive value
A. Ligamentum falvum
Ans: A
B. Transverse ligament
178) After mastectomy patient presented
C. Interspinous ligament
with limb swelling:
D. Anterior n posterior longitudinal ligament
a. Lymphatic edema Ans: D
b. Vascular Edema
3) Standing from supine position which
c. Neurogenic Edema
the most compensatory mechanism?
d. Arteriolar
A. Decrease venous return
Ans: A
B. Decrease venous compliance
179) Second stage of labour is caused by
C. Increased heart rate
which factor?
D. Decrease heart rate
a. Oxytocin
Ans: C
b. Abdominal Muscle contraction 4) The structure which prevents muscle
c. PGs
from tearing under a constant
d. Cortisol
pressure?
Ans: B
A. Ruffinis endings
Explanation:
B. Golgi tendon organ
 If ask about substance: Oxytocin C. Merkle cells
 If asked about factor: Abdominal Muscle D. Muscle spindle
contraction. E. Pacinians corpuscles
180) Melanoma invade orbital cavity Ans: B
through: 5) Lady had vaginal infection with foul
a. Sclera smelling clue cells on gram stain.
b. Optic Nerve Most likely organismis ?
c. Emissary Vein A. Chlamydia
d. Retina B. Gonococcus
e. Lymphatic C. Gardenella Vaginalis
Ans: C D. Giardiasis
Explanation: Uveal malignant melanoma is the Ans: C
most Common primary intraocular tumour of 6) Toxoplasmosis is?
adulthood & orbital extension probably occurs A. Parasite
through the emissary vein. B. Helminth
C. Virus
D. Bacteria
=============== Ans: A
SK Origional – Golden 11 277

7) The DOC for tapeworm infestation? Ans: B


A. Niclosamide 14) Following is true about exudate:
B. Metronidazole A. Few inflammatory cells
C. Albendazole B. Specific gravity of 1.012
D. Prazequintal C. Protein > 3.5 gram
E. Mebendazole Ans: C
Ans: D 15) Melatonin is derivative of:
8) Raw and uncooked meat can cause A. Epinephrine
infestation by: B. Nor epinephrine
A. Tapeworm C. Prolactin
B. Liver Fluke D. Thyroxin
C. Schistosoma Hematobium E. Serotonin
D. Echinococcus Ans: E
Ans: A 16) Apex of the heart is supplied by:
9) Micturation controlled by: A. Anterior interventricular artery(LAD)
A. Mechanoreceptors in bladder wall B. Diagonal
B. Inhibited by a centre in pons. C. RCA marginal artery
C. Condition reflex. D. Posterior descending artery
D. Processed in cerebral cortex. Ans: A
E. Stretch of urethral sphincter 17) Breast upper outer quadrant lymph
Ans: A drainage accompanies which vessel?
10) Local anesthesia cross placenta A. Lateral thoracic
through: B. Anterior axillary
A. Active transport C. Internal thoracic
B. Bulk flow Ans: B
C. Facilitated diffusion 18) Clara cells are present in:
D. Simple diffusion A. Terminal bronchioles
Ans: D B. Respiratory bronchioles
11) A 7 year old child with cough and C. Larynx
mucus, increase eosinophills x-ray D. Trachea
shows hyperinflation? E. Bronchi
A. Cystic fibrosis Ans: A
B. Bromchial asthma 19) MMR definition:
C. Hypersensitivity pneumonitis A. No of maternal deaths/1000 population
D. Gastric aspiration B. No of maternal deaths /100000
E. Aspergillosis C. No of maternal deaths /100
Ans: B Ans: B
12) Lymphatic drainage of external nose: 20) Regarding surfactants which are
A. Submental appropriate:
B. Submandibular A. Slow turnover
C. Sublingual B. Secreted by type 1 pneumocyte
D. Preauricular C. Secreted by type 2 pneumocyte
E. Post auricular D. Decrease lung compliance
Ans: B Ans: C
13) Lymphatic drainage of anus: 21) Below hymen boil group of lymph
A. Vertical group of superficial inguinal nodes involved:
lymph nodes A. Medial group of superficial inguinal
B. Medial group of superficial inguinal lymph node
lymph nodes B. Vertical group of superficial inguinal
C. Deep Inguinal Lymph node lymph node
D. Horizontal Group of Superficial Inguinal C. Horizontal group of superficial inguinal
Lymph nodes lymph node
SK Origional – Golden 11 278

Ans: A A. Facial vein


22) Distance of pelvic part of sacrum to B. Superior ophthalmic vein
posterior inner part of pubic C. Inferior ophthalmic vein
symphysis is called as: Ans: B
A. Obstetric conjugate 29) Child who had episode of influenza
B. Diagonal conjugate pneumonia presented with cough.
C. True conjugate Thick yellow mucoid sputum along
Ans: A with cough most likely organism:
23) Which protein defect is responsible for A. Klebsella
Marfans syndrome? B. S-pnemomina
A. Elastin C. S-aureus
B. Fibrillin D. Pseudomonas
C. Collagen Ans: C
D. Actin 30) Ureter us lined by:
E. Myosin A. Simple squamous epithelium
Ans: B B. Stratified squamous keratinized
24) Child with multiple fractures, blue epithelium
sclera, defects in which gene? C. Transitional epithelium
A. Fibrillin D. Stratified squamous non keratinized
B. Collagen epithelium
C. Elastin E. Tall columnar epithelium
D. Actin Ans: C
E. Myosin 31) Fetal blood leaving placenta have
Ans: B PaO2:
25) Lady working in a kitchen wound not A. 20 mmhg
healing wound is clean but gaping B. 30 mmhg
patient is febrile glucose is 130 mg/dl C. 40 mmhg
and Hb is 5 cause is: D. 50 mmhg
A. Diabetes Ans: B
B. Infection 32) Epithelium of Hymen is lined by:
C. Anemia A. Simple squamous epithelium
D. Foreign body B. Simple columnar
Ans: C C. Transitional epithelium
26) 31 year old man with AIDS presets D. Stratified squamous
with multiple purple colour skin E. Tall columnar epithelium
nodules (1mm to1cm) on hands and Ans: D
feets, which virus is implicated in 33) Conexant is a part of:
these skin lesions? A. Centriole
A. HIV B. Gap Junctions
B. HPV C. Sodium Channels
C. CMV D. Mitochondrion
D. HHV 8 E. Sarcoplasmic Reticulum
E. HHV 6 Ans: B
Ans: D 34) Lymphatic drainage of labia majora is:
27) First response to vessel injury: A. Internal iliac
A. Activation of intrinsic pathway. B. Internal n external iliac
B. Activation of extrinsic pathway C. Superficial Inguinal
C. Vasoconstriction D. Inferior mesenteric
D. Platelet aggregation E. Para aortic
Ans: C Ans: C
28) Spread of infection from face to 35) Iron deficiency anemia best diagnosed
cavernous sinus: by:
SK Origional – Golden 11 279

A. Serum ferritin A. Hyponatremia


B. Serum iron+inc TIBC B. Hypernatremia
C. Serum iron C. Hypokalemia
D. Hemoglobin D. Hyperkalemia
Ans: A Ans: C
36) Niacin deficiency causes: 43) A pt with HB 6, platelet count 450,000,
A. PNS defects with hyper cellular bone marrow,
B. CNS defects diagnosis is:
C. Epithelial defects A. Drugs using for anemia
D. Limb defects B. Leukemia
Ans: B C. Iron def. anemia
37) In addition to folic acid deficiency, D. G6pd def.
what other nutritional problem is E. Acute blood loss
believed to help cause neural tube Ans: C
defects? 44) Toxic shock syndrome:
A. Vitamin A deficiency A. Discrete popular rash
B. Vitamin A toxicity B. Its misnomer
C. Vitamin B12 deficiency C. Painful papules
D. Vitamin D deficiency D. Viral infection
E. Vitamin D toxicity Ans: A
Ans: C 45) Which space is assessed after piercing
38) Gas having maximum diffusion the inter laminar ligament during LP?
capacity: A. Subdural Space.
A. CO B. Subarachnoid Space.
B. CO2 C. Pia
C. Oxygen D. Epidural Space
D. NO E. Intracranial space.
Ans: B (Inter laminar ligament =Ligamentum flavum)
39) The ecchymosis of flank seen in Ans: D
patients with acute pancreatitis is 46) Subarachnoid space level:
named after: A. L1 – L2
A. Trousseau B. L4 – L5
B. Cullen C. S2 – S3
C. Grey turner D. S4 – S4
D. Good Pasteur Ans: C
Ans: C 47) A thorn strikes a young laborer in
40) Maximum bioavailability of drug is via anterior aspect of thigh, after few days
which route? he developed Abscess at that area,
A. Sublingual which of the following organisms is
B. IM most likely responsible for that:
C. Per rectal A. Streptococcus pneumonia
D. Per oral B. Staphylococcus aureus
(IV was not in options) C. Pseudomonas aeriginosa
Ans: A D. Proteus
41) Disease caused by mosquito bite: E. E.coli
A. Leishmaniasis Ans: B
B. Yellow fever 48) Posterior arcuate fibers:
C. Q fever. A. Cuneo cerebellar
D. African sleeping sickness. B. Spinocerebellar
Ans: B C. Tectocerebellar
42) Nerve stimulation is more difficult to D. Magnocerebellum
create action potential in: Ans: A
SK Origional – Golden 11 280

49) The diagnostic lab test for the 56) Nissil bodies are abundant in:
infection by streptococcus beta A. SER
haemolyticus is: B. RER
A. ASO titre C. Golgi apparatus
B. Blood culture D. Mitochondria
C. ESR Ans: B
D. Leukocytes culture 57) Which of the following types of drugs
E. Urine for bile pigment will have maximum oral
(Best initial ➡ASO titer) bioavailability?
(Diagnostic ➡Culture) A. Drugs with high first-pass metabolism.
Ans: B B. Highly hydrophilic drugs.
50) Most common congenital heart C. Largely hydrophobic, yet soluble in
disease: aqueous solutions
A. ASD D. Chemically unstable drugs.
B. VSD E. Drugs that are P-glycoprotein substrates
C. TOF Ans: C
D. TGA 58) Patient has blunt truma to abdomen
E. PDA presented in shock state most likely
Ans: B organ damage:
51) Slowly adaptive receptors: A. Liver
A. Merkle‘s disk B. Spleen
B. Carotid baroreceptors C. Kidney
C. Rufini's corpuscles D. Pancreases
D. Pacinion corpusles E. Intestine
E. Fine nerve endings Ans: B
Ans: C 59) Conducting part of lung:
52) Anesthetic drug in asthmatic patient: A. Bronchi
A. Propofol B. Alveoli
B. Ketorolac C. Alveolar duct
C. Halothane D. Bronchioles
D. Ketamine Ans: A
Ans: D 60) Which of is strong Analgesic but weak
53) Manubriosternal joint: anesthetic?
A. Primary cartilaginous joint A. Ketamine
B. Secondary cartilaginous joint B. Morphine
C. Tertiary cartilaginous joint C. Nitric oxide
Ans: B D. Toradol
54) Doppler ultrasound: Ans: C
A. Findings are modified by fetal 61) Choline is component of:
presentation A. Sphingomylein
B. Pulstality is increased in growth restricted B. Phsophotidicholine
fetus C. Cephalic
C. Pulstality is decreased in growth restricted D. Glycerol
fetus Ans: A
Ans: B 62) Growth hormone uses which as main
55) Female patient wants to lose weight source for energy production?
what is best strategy for her? A. Acth
A. Self-punishment B. Glucose
B. Self-reward C. Fat
C. Self-monitoring D. Proteins
D. Social withdrawl Ans: C
Ans: C
SK Origional – Golden 11 281

63) Derivatives of latent chemotactic from C. Vestibular neuritis


arichodonic acid pathway: Ans: B
A. LTB4 70) PO2: 50, Hb: 9, which type of hypoxia?
B. C5a A. Anemic hypoxia
C. C4b B. Hypoxic hypoxia
D. C6a C. Histotoxic hypoxia
Ans: A D. None
64) Suprapubic transverse incision, what Ans: B
is at risk of damage? 71) An increase in the concentration of
A. Superior epigastric artery plasma potassium causes increase in:
B. Inferior epigastric artery A. Release of renin
C. Cremasteric artery B. Secretion of aldosterone
D. None C. Secretion of ADH
Ans: B D. Release of natriuretic hormone
65) In voluntary contraction, descending E. Production of angiotensin II
pathways cause excitation of: Ans: B
A. Alpha motor neurons discharge 72) Which cells cause lymphocyte
B. Both alpha and gamma motor neuron production and cell turnover?
discharge A. Principle cells of thyroid
C. Presynaptic nerves inhibited B. Zona fasciculata
Ans: B C. Zona glomerulosa
66) Deep inguinal ring is found in: D. Zona reticularis
A. Transversalis fascia Ans: A
B. External oblique 73) Superficial inguinal ring is a defect in:
C. Internal oblique A. External oblique
D. Transverse abdominis B. Tranversalis Fascia
Ans: A C. Internal obligue
67) Increased in CVP is caused by: D. Transversus abdominus
A. Increase in compliance of veins Ans: A
B. Decrease in compliance of veins 74) Women lower abd pain on 18 day of
C. Hypovolemic shock LMP presented with secondary
D. Increased peripheral vascular resistance infertility, diagnostic D n C will show
Ans: D endometrium in which stage?
68) A 67 year female presents with balance A. Ovulatory
problem Exam: nystagmus on left B. Secretory
lateral gaze, a loss of the left corneal C. Proliferative
reflex and reduced hearing in the left D. Premenstrual
ear. What is the most likely dx? Ans: B
A. Meniere‘s disease 75) Chest infection after Bone marrow
B. Acoustic neuroma transplant with chest infiltration
C. Cerebral abscess organism:
D. Pituitary tumor A. CMV
E. Gentamicin B. H.pylori
Ans: B C. Toxoplasmosis
69) Patient presented with nausea, D. Staphylococus aureus
vomiting, nystagmus, tinnitus and E. Pseudomonas
inability to walk unless he Ans: A
concentrates well on a target object. 76) Mother worry about her young
His cerebeller function is intact, what daughter who has loss of pubic and
is the diagnosis? axillary hair with secondary sexual
A. Benign positional vertigo character cause is decrease in:
B. Meniere‘s disease A. Diheandrosterone
SK Origional – Golden 11 282

B. Estrogen 84) A 67 year old man presents with


C. Progesterone urinary incontinence and difficulty
D. Androstendione walking. Which dermatome represents
Ans: B the perianal region?
77) EBV is associated with Burkett A. S1
lymphoma because? B. S2
A. It causes defect in DNA repair C. L5,S1
B. African with Burkett lymphoma has EBV D. S2,S3,S4
genome 3 E. S4,S5
C. EBV has toxins Ans: D
D. Mitochondrial damage 85) Drug used in treatment of hirsutism:
Ans: B A. Antiandrogens
78) Highest systolic pressure is in: B. Cryptoterone acetate
A. Aorta C. Diacrone
B. The left ventricle D. Estrogen
C. Renal artery Ans: B
D. Pulmonary artery 86) A woman has hirsutism,
Ans: C hyperglycemia, and obesity, muscle
79) Aortic pressure tracing highest in: wasting and increased circulating
A. Rapid ejection phase levels of adrenocorticotropic hormone.
B. Slow ejection phase The most likely cause of her
C. Isovolumetric contraction symptoms is:
D. Isovolumetric relaxation A. Primary adrenocortical deficiency
Ans: B B. Pheochromocytoma
80) Earliest step in formation of C. Cushing Disease
thrombus: D. Treatment with exogenous
A. Formation of fibrin glucocorticoids
B. Adherence of platelets E. Hypophysectomy
C. Trapping of erythrocytes Ans: C
D. Activation of hageman factor 87) An eight year old girl brought by
Ans: B mother with breast development and
81) High output cardiac failure is found appearance of pubic/ axillary
in: hair...cause?
A. Anemia A. Gonadal tumours
B. Beri beri B. CNS tumour
C. Hypothyroidism C. Congenital adrenal hyperplasia
D. Low blood pressure D. Idiopathic
E. Vitamin D deficiency Ans: C
Ans: B 88) A young female presented with fever
82) Which ion is raised in malignant 101 degree, gum bleeding, petechiae
hyperthermia? and epistaxis. Best test to diagnose is:
A. Na A. CBC
B. K B. Bone marrow biopsy
C. Ca C. Coagulation profile
D. Mg D. CT/BT
Ans: C E. Electrophoresis
83) Extra systole will cause: Ans: B
A. Increase in contractility 89) Lymphatic‟s flow is decreased by:
B. Dec. in contractility A. Inc.capillary oncotic pressure
C. Increase in contractile strength B. Inc.capillary hydrostatic pressure
D. Dec in contractile strength C. Inc.capillary permeability
Ans: B D. Inc.interstitial oncotic pressure
SK Origional – Golden 11 283

Ans: A Ans: B
90) The female child is at risk for clear cell 96) Characteristic of carcinoma:
carcinoma of the vagina of she had A. Keratin Pearls
utero exposure to the following drug: B. S – 100
A. Diethylstilbestrol(DES) C. Desmin
B. Nicotine D. Vemtin
C. Clomiphene Ans: A
D. Isotretinoin 97) Spermatogenesis take how many days:
E. Methotrexate A. 30
Ans: A B. 60
91) A 38 week pregnant woman visited her C. 90
family and a remote area after which D. 120
she develop jaundice suddenly. Which Ans: B
type of hepatitis is most likely 98) Person had swelling in upper arm
prevalent and the pregnant women? after injury. Swelling remained for 3
A. Hep. A months but resolved in 1 year. What
B. Hep. B was the cause of swelling?
C. Hep. C A. Hypertrophy
D. Hep. D B. Hyperplasia
E. Hep. E C. Metastatic calcification
Ans: E D. Dysplasia
92) Due to stress of surgery what will E. Metaplasia
increase and cause vasoconstriction? Ans: E
A. ACTH 99) Which cause of intestinal obstruction
B. Cortisol gives worst prognosis?
C. Catecholamines A. Adhesion
D. Dopamine B. Diverticulitis
Ans: C C. Mesenteric ischemia
93) A patient was brought to the hospital D. Hernia
with a history of RTA eight hours E. Intrinsic cause
back. A few drops of blood are noted Ans: C
at the external urethral meatus. He 100) What pulmonary disease, most
has not passed urine? commonly associated with smoking,
A. Urethral injury results in enlarged, overinflated lungs
B. Rupture bladder owing to the destruction of the
C. Ureteral injury alveolar walls with diminished elastic
D. Anuria due to hypovolemia recoil?
Ans: A A. Asthma
94) In pulmonary embolism respiratory B. Lung carcinoma
distress is due to: C. Emphysema
A. V/Q mismatch D. Pneumoconiosis
B. V/Q increase Ans: C
C. V/Q decrease 101) Patient brought in shock in
Ans: A emergency department. Successful
95) The number of new cases of breast fluid replacement will be indicated by?
cancer per 100000 women in a year is A. Decrease in tachycardia
expressed as: B. Increase in BP
A. Rate C. Increase in urine output
B. Incidence Ans: C
C. Prevalence 102) Downward displacement of enlarged
D. Proportion spleen is prevented by?
E. Ratio A. Linorenal ligament
SK Origional – Golden 11 284

B. Phrenicocolic ligament B. Childhood


C. Sigmoid colon C. Adolescent
D. Upper pole of right kidney D. Old age
Ans: B Ans: C
103) A person working in coal mine. His 110) A 50 year old women had
work is stone polishing and sand hysterectomy for uterine fibroids. She
blasting. He might can develop which develops right lumbar pain within
of the following disease in the future? 48hour of surgery. Which of the
A. Pneumoconiosis following structure is to be ligated?
B. Silicosis A. Cardinal ligament
C. Anthracnose‘s B. Meso ovarium
Ans: B C. Round ligament
104) A patient has s hip fracture in RTA D. Ureter
after which when he stands on his left E. Uterine artery
leg his right hip sinks down lesion is Ans: D
as? 111) Intestinal flora that causes lobar
A. Left gluteus medius pneumonia in alcholics and diabetes
B. Right gluteus medius having dark red "currant jelly"
C. Left gluteus maximus sputum:
D. Right gluteus maximus A. E. coli
Ans: A B. Pseudomonas
105) Osteoblastic activity is decreased by: C. Klebsiella
A. GH D. S. aureus
B. Cortisol Ans: C
C. Estrogen 112) Total body water can be calculated by:
D. PTH A. Evans blue
Ans: B B. Mannitol
106) Fibroblast made: C. Antipyrine
A. Collagen fibers only D. Albumin
B. Amorphous groud substance Ans: C
C. Reticular only 113) Pain in epigastrium due to gastric
D. Elastic only ulcer is carried by:
Ans: B A. Greater splanchnic nerves
107) A patient working in mine presented B. Lesser splanchnic
with cough fever weight loss; C. Least splanchnic
tuberculin test positive with hilar D. Phrenic nerve
lymphadenopathy diagnosis? Ans: A
A. Tuberculosis 114) Early diagnosis and prompt treatment
B. Silicosis is:
C. Asbestosis A. Counseling
D. Coal worker B. Primary prevention
Ans: A C. Secondary prevention
108) Gold standard investigation for D. Simply prevention
placenta previa: E. Tertiary prevention
A. Pelvic usg Ans: C
B. Pelvic MRI 115) Albumin does not pass basement
C. Transvaginal USG membrane due to:
D. Per vaginal examination A. Negative charge
Ans: C B. Large size
109) Maximum growth occurs in which C. Positive charge
phase? D. Medium size
A. Intrauterine Ans: A
SK Origional – Golden 11 285

116) During development of genitourinary 123) Systemaic lupus erythematosus type


system mesonephric duct give rise to: of hypersensitivity reaction:
A. Rete testes A. Type 1
B. Ductus deferens B. Type 2
C. Prostate C. Type 3
D. Interstitum D. Type 4
Ans: B Ans: C
117) Mechanism of action of oral 124) Mother used which medicine for Acne
contraceptive pills is: which caused cleft lip in baby and
A. Inhibiting ovulation cardiac anamoly?
B. Inducing endometrial atrophy A. Vitamin A
C. Increasing cervical mucus hostility B. Lithium
D. Inducing endometritis C. Valproic acid
E. Inhibiting prolactin D. Thalidomide
Ans: A Ans: A
118) Child with frontal bossing bow legs 125) Major storage of iron in body is:
and lock knee it is seen in: A. Spleen
A. Vitamin D deficiency B. Liver
B. Vitamin B deficiency C. Pancreas
C. Vitamin C deficiency D. Bone marrow
D. Vitamin A deficiency Ans: B
Ans: A 126) Premotor area is supplied by:
119) Confidence interval represents: A. Anterior and middle cerebral artery
A. Median and its standard deviation B. Posterior cerebral
B. Mode n its standard error C. PICA
C. Mean n its standard error D. Middle n posterior cerebral art
D. Mode n its standard error E. Anterior n posterior cerebral art
E. Standard deviation Ans: A
Ans: C 127) Defect in anterior pituitary is
120) Reaction after diphtheria vaccine: indicated by:
A. Anaphylaxis A. No menses
B. Type 1 B. Uterine atrophy
C. Type2 C. Congenital adrenal hyperplasia
D. Type3 D. Hisutism
E. Type4 Ans: A
Ans: D 128) Bromocriptine causes:
121) During muscle contraction which part A. Decrease prolactin release from pituitary
remain the same length: B. Enhance effect of dopamine on pituitary
A. I band C. Increase prolactin release
B. A band D. Increase oxytocin release
C. H band Ans: A
D. M line 129) Most Common cause of mutation of
E. Z lines proto-oncogene is:
Ans: B A. Virus
122) Acute intravascular hemolysis most B. Ionizing radiation
likely finding: C. Bacteria
A. Dec. haptoglobin D. Parasite
B. Reticulocytosis Ans: A
C. Hemoglobinemia 130) 5 year child with rigors chills and
D. Hemoglobinuria headache organism detected on
Ans: A geimsa:
A. Falciparum
SK Origional – Golden 11 286

B. Ovale 136) Lacrimal glands are supplied by which


C. Vivax of following?
D. Malarea A. Trigeminal nerve
Ans: A B. Facial Nerve
131) Some neoplasms appear to develop C. Glossopharyngeal
from viral oncogenesis. Which of the D. Vagus
following neoplasms is most likely to Ans: B
arise in this manner? 137) Patient presented in shock after pelvic
A. Retinoblastoma fracture in RTA:
B. Small cell carcinoma A. Excessive blood loss
C. T-cell leukemia B. Fat embolism
D. Prostatic adenocarcinoma C. Bladder injury
E. Hepatic angiosarcoma D. Bowel injury
Ans: C Ans: A
132) Pulmonary TB patient has cavitation 138) Pulmonary edema develops due to
with calcification. Type of change: which of the following?
A. Dystrophic calcification A. Increase hydrostatic pressure
B. Metastatic calcification B. Decrease hydrostatic pressure
C. Benign calcification C. Increase oncotic pressure
D. Malignant calcification d. Decrease oncotic pressure
Ans: A Ans: A
133) Vitamins synthesized by intestinal 139) Astronaut in space for very long time.
bacteria and absorbed in significant He could have which chronic health
quantities include: problem:
A. Vitamin B6 A. Polycythemia
B. Vitamin K B. Decrease lung compliance
C. Thiamine C. Decrease bone mass
D. Riboflavin D. Decreased surfactant
E. Folic acid Ans: C
Ans: B 140) Week pregnant women have irregular
134) The BP recording of 2 males, a uterine contractions which of
hypertensive with reading of 228/160 following drugs will be beneficial for
and a normotensive with 120/80 were this lady?
noted respectively. If 10mmhg is A. Progesterone
added to the mean BP of both, how B. Estrogen
will the response of Hypertensive C. Oxytocin
patient vary from that of normal? D. Prolactin
A. No change in sympathetic activity of both E. LH
B. Increase sympathetic activity in Ans: C
hypertensive 141) Benedict test is for:
C. Decrease sympathetic activity A. Glucose
D. Increase parasympathetic activity B. Diabetes
E. Decrease parasympathetic C. Blood flow
Ans: B D. Reducing the substance of urine
135) 41% hematocrit means: Ans: D
A. 41% of formed elements are RBC 142) Parotid duct is also known as:
B. 41% of blood is Hb A. Duct of santorini
C. 41% of blood volume is RBCs,WBCs,and B. Duct of wirsung
platelets. C. Stenson's duct
D. None of the above D. Wharton's duct
Ans: A Ans: C
143) Microtia is due to:
SK Origional – Golden 11 287

A. 1stbranchial Arch B. Two equal sized cells


B. 2ndbranchial Arch C. Addition of DNA
C. 1stand 2nd brachial arch D. One secondary oocyte and polar body
D. 3rdand 4th branchial Arch Ans: D
Ans: C 151) One bar body present in nuclei/cell
144) In down syndrome robersonian of:
translocation transmit in how much A. Turner syndrome
percentage of next generation: B. Down syndrome
A. 15% C. Normal female
B. 50% D. Normal Male.
C. 33% Ans: C
D. 80% 152) Which of the following is rapidly
Ans: A adapting receptors?
145) Which ligament has extra and intra A. Pacinian
peritoneal folds: B. Meisseners
A. Broad ligament C. Baroceptors
B. Round ligament of uterus D. Unencapsulated receptors
C. Suspensory ligament Ans: A
D. Coopers ligament 153) Most important criteria for the
Ans: B diagnosis of malignancy is:
146) Chylomicrons synthesized in: A. Invasion
A. G.I tract B. Metastasis
B. Kidney C. Pleomorphism
C. Liver D. Dysplasia
D. Small intestine E. Metaplasia
Ans: D Ans: B
147) Side effect of morphine: 154) Which transport depends on cell
A. Bradypnea meiosis membrane thickness?
B. Tachpenia meiosis A. Diffusion
C. Bradypnea mydriasis B. Osmosis
D. Tachypnea mydriasis C. Phagocytosis
Ans: A D. Primary active transport
148) The most common organism amongst E. Secondary active transport
the following that causes acute Ans: A
meningitis in an AIDS patient: 155) A chronic smoker showing ulcer on
A. Streptococcus pneumonia lower lip margin which is indurated
B. Streptococcus agalactiae and pale most likely it is:
C. Cryptococcus neoformans A. Squamous cell carcinoma
D. Listeria monocytogenes B. Basal cell carcinoma
Ans: C C. Pleomorphic adenoma
149) Correct statement regarding Ans: A
oogenesis: 156) Post menopause women present with
A. First meiotic division starts during feral breast carcinoma is treated by:
life A. Aromatase inhibitor
B. Secondary meiotic division completes just B. Tamoxifen
before fertilization C. Metformin
C. Graffian follicle contains primary oocyte D. Tolbutamide
D. Adult ovary has columnar epithelium Ans: A
Ans: A 157) Patient with ccl4 poisoning died due
150) The primary oocyte after first to:
maturation division produces: A. Free radical formation
A. Two haploid cells B. Acute renal failure
SK Origional – Golden 11 288

C. Acute hepatic failure Ans: A


Ans: A 165) Endometrium with tortious gland:
158) Tall man with normal seminal fluid A. Early Secretary phase
secretion but azospermia defect is in: B. Late secretary phase
A. Sertoli cells C. Glandular phase
B. Leydig cells D. Follicular phase
C. Seminiferous tubules Ans: B
Ans: A 166) Remnant of gubernaculum in females:
159) Branch of internal iliac artery remains A. Spermatic cord
within the true pelvis: B. Testis
A. Superior gluteal C. Round ligament of ovary
B. Middle rectal D. Prostate
C. Inferior gluteal Ans: C
D. Internal pudendal 167) The primary use of the standard error
E. Iliolumbar of the mean is in calculating the:
Ans: B A. Confidence interval
160) What is passing through groove of B. Error rate
mediastinum on left side? C. Standard deviation
A. Aorta D. Variance
B. Esophagus Ans: A
C. IVC 168) Most common infectious cause of
D. Azygous vein blindness:
Ans: B A. Cataract
161) Middle rectal artery is branch of: B. Trachoma
A. SMA C. Vitamin A deficiency
B. IMA D. Glaucoma
C. Internal iliac artery E. Nectalopia
D. External iliac Ans: B
Ans: C 169) A man with AIDS no symptoms cd4
162) White infract occur in which of 800 presented with supportive otitis
following site? media causative agent:
A. Heart A. Mycobacterium
B. Lungs B. Strep pneumonia
C. Intestine C. Psudo aergnosa
D. Testicular torsion D. Staph aureus
E. Atrial occlusion after angioplasty Ans: B
Ans: A 170) Feature of dysplasia:
163) Anti-sera agglutination with blood A. Loss of polarity and architecture
group B and D and plasma causes B. Pleomorphism
agglutination with A Blood group is: C. Metastasis
A. A+ D. Invasion
B. B+ Ans: A
C. O+
D. B-
E. A- ===============
Ans: B
164) Para renal fascia is an extension of:
A. Renal fascia GYNAE & OBSTETRICS
B. Gerota fascia November 25th, 2020 – Afternoon
C. Retroperitoneal fat
D. Psoas
1) A 40 years old man by keeping in
E. Quadratus lamborum
SK Origional – Golden 11 289

mind that he is diagnosed case of 6) Which structure is absent in C1 (Atlas)


Ischemic Heart Disease which vertebrae?
antihypertensive you choose which a. Spinous Process
also have vasodilatory property? b. Articular Surface
a. Isosorbide Dinitrate c. Anterior Arch
b. Metaprolol d. Posterior Arch
c. Nifidipine Ans: A
d. Verapamil Explanation: Atlas C1 is Atypical cervical
Ans: B vertebrae which does not possess a Body and
Explanation:Beta blockers are first choice in Spinous process.It do have articular surfaces for
Hypertensive patient with ANGINA. In case of Atlanto-occipital and Atlanto-axial joints.Other
Hypertensive with Chronic renal failure Diuretics Atypical cervical vertebrae are C2 (Unique feature
are first line therapy. Odontoid process) and C7 (unique feature long
2) After 5 year of transplant surgery there Spinous process).
is increased risk of? 7) Right ovarian vein drain into:
a. Liver malignancy a. Right Renal Vein
b. Skin malignancy b. Inferior Vena Cava
c. Lymphoproliferative c. Celiac Trunk
d. Bronchogenic Carcinoma d. Internal Iliac Vein
An: C(Overall most common malignancy Ans: B
after Transplant is Skin malignancy but Explanation: Right ovarian/testicular vein is
usually after 10 years.So here answer is direct tributary of inferior vena cava.In case of left
C>B ovarian/testicular vein choose left renal
3) Inferior epigastric artery lateral to vein>inferior vena cava.
which hernia? 8) Plasma cells from:
a. Indirect Inguinal Hernia a. Helper T cells
b. Direct Inguinal Hernia b. T lymphocytes
c. Umbilical Hernia c. B lymphocytes
d. Femoral Hernia d. NKCs
e. Epigastric Hernia e. Monocytes
Ans: B Ans: C
Explanation: Direct hernia is medial to Inferior Explanation: Mature naive B cells  plasma cells
epigastric arteryso artery is lateral to direct hernia which differentiate into IgA, IgE or IgG?
and vice versa for indirect hernia 9) Decidua cells part of:
4) The most common cause of Impair or a. Breast
delayed wound healing is: b. Liver
a. Infection c. Uterus
b. Foreign body in the wound d. Cervix
c. Mobility e. Placenta
d. Old Age Ans: C
e. Male Gender Explanation: Decidua is the modified mucosal
Ans: A lining of the Uterus that forms in preparation for
5) Concomitant Hyperplasia and pregnancy.It forms functional layer and sheds
Hypertrophy can be seen in? during mensuration.
a. Breast development in puberty 10) Development of alveoli and lobules of
b. Uterine changes in pregnancy breast?
c. Endometrial Hyperplasia a. Prolactin
d. Exercising Muscles b. Estrogen
Ans: B c. Progesterone
Explanation: Hyperplasia of uterine blood d. Oxytocin
vessels and Hypertrophy of Uterine smooth Ans: C
muscles. Explanation: Development of Alveoli and
SK Origional – Golden 11 290

lobules hormone responsible is progesterone d. 24m


while for fat deposition and alveoli development e. 36 m
hormone responsible is estrogen. Ans: A
11) Criteria for diagnosing malignant Explanation:According to snells anatomy
tumor: anterior fontanelle closes upto 18 months.12
a. Pleomorphism months is too early here I choose 24 because it‘s
b. Invasion of adjacent tissues definitive closure period.
c. Dysplasia Anterior Fontanelle: Also known as Bregma
d. Metastasis Diamond in shape present between
Ans: D Frontal,Coronal and Parietal sutures.
12) Bicornuate uterus abnormality of: Posterior Fontanelle: Also known as Lamda
a. Mesonephric duct Triangular in shape present between Sagittal and
b. Paramesonephric duct Lambdoid suture.
c. Metanephros Range of closure: 2 – 3months after birth
d. Urogenital Septum 16) Cyclic GMP which hormone included?
e. Ureteric Bud a. TSH
Ans: B b. ADH
Explanation: Bicornuate uterus occur when there c. ANP
is partial fusion of the paramesonehric ducts. d. Glucagon
Derivatives of Paramesonehric duct (Mullerian Ans: C
Duct): Fallopian Tubes,Uterus Cervix and 2/3 or Explanation:Hormones which uses cyclic GMP
Upper part of Vagina. Ovaries are not included. as second messenger are Atrial Natriuretic Peptide
13) Superior pelvis has short anterior (ANP) and Nitric Oxide.Remember both.
posterior diameter and long transverse 17) A patient is having Mitral Valve
diameter. Type of pelvis? disease now present with Fever,
a. Gynecoid Malaise and Petechia. On
b. Platypelloid examination he was also having
c. Contracted Splinter hemorrhages and
d. Android splenomegaly. What is the causative
e. Anthropoid agent of such Illness?
Ans: B a. Staphylococcus Aureus
Explanation:Gynecoid have Round Inlet while b. Streptococcus Viridian
Short antero-Posterior and Long Transverse make c. Streptococcus Pneumonia
Inlet shape FLAT so its Platypelloid. d. Streptococcus Epidermidis
Contacted Pelvis: Inlet or Obstetric Conjugate is Ans: B
<10cm Mid spinous <8cm Outlet <8cm 18) A semiconscious patient aspirate
Android Pelvis: Prominent Ischial spine is while lying on his back the segment of
particular Feature. the lung most likely affected would be:
Anthropoid: Most Common Variant of Gynecoid a. Anterior segment of right upper lobe
Pelvis. b. Apical segment of right lower lobe
14) In female neck of urinary bladder: c. Posterior segment of left upper lobe
a. Lies below the level of urogenital d. Base of Both Lungs
diaphragm Ans: B
b. Within urogenital diaphragm 19) Lateral part of breast drain by:
c. Lies superior surface of urogenital a. Anterior axillary
diaphragm b. Posterior axillary
d. Deep Perineal pouch c. Inferior Phrenic
Ans: C (It‘s above urogenital diaphragm). d. Internal Thoracic
15) Anterior fontanelle closes at: e. Central Lymph nodes
a. 24 months Ans: A
b. 6m 20) Lymphatic drainage of fundus of
c. 12m stomach:
SK Origional – Golden 11 291

a. Celiac Lymph nodes a. Cell membrane


b. Superior Mesenteric Lymph nodes b. Inside cell membrane
c. Inferior Mesenteric Lymph nodes c. Cytoplasm
d. Illioinguinal Lymph nodes d. Nuclear membrane
e. Splenic Lymph nodes e. Inside Nucleus
Ans: A Ans: E( Ganong)
21) Embryonal counterpart of mesonehric 26) Wedge shaped fibromuscular
duct is: structure in perineum:
a. Rete Testis a. Levator Ani
b. Seminal Colliculus b. External Anal sphincter
c. Ureter c. Perineal Body
d. Ejaculatory Duct d. Iliacus
Ans: D Ans: C
Explanation: Counterpart of mesonephric duct 27) Most common cause of
are seminal vesicle,ductus deferens and ejaculatory hydrocephalus:
duct. a. Cerebral aqueduct stenosis
22) A Young man had a road side b. Increase CSF production
accident in which he suffered from c. Lateral Ventricles obstruction
Spinal cord Injury. Some soft filling d. 3rdventricles obstruction
located in the center of the disc of Ans: A
spinal cord known as nucleus 28) Heart can‟t be tetanized due to:
pulposus. It is derived from which a. Short refractory period
embryological structure? b. Long refractory period
a. Mesonephric Duct c. Gap junctions
b. Neural Tube d. Desmosome
c. Notochord Ans: B
d. Mullerian Duct 29) Diabetic mother presented with
e. Neural Crest respiratory distress fetus should be
Ans: C given:
23) In a group of Diseased people a test is a. Betamethasone
performed which tells about true b. Promethasone
negative from affected people is c. Momethasone
known as: d. Dexamethasone
a. Positive predictive value Ans: A
b. Negative prediction value 30) A patient presented in Labor room
c. Probability with Pre-term labour. Amniotic fluid
d. Sensitivity sample is taken to asses Sphengo-
e. Specificity Myelin ratio. What this ratio
Ans: E indicates?
Explanation: a. Lung Maturity
True positive =Sensitivity b. Cardia Defect
True negative=Specificity c. Chorioamnitis
24) Difference in maternal and fetal d. Liver Maturity
Circulation which fetal has less? Ans: A
a. Less H ion Explanation: Neonatal respiratory distress
b. O2 syndrome occurs in pre mature infants because of
c. CO2 lack of surfactant. Generally, Lecithin-
d. Respiratory vascular resistance sphingomyelin ratio greater than 2:1 in amniotic
e. Affinity for oxygen fluid reflects mature levels of surfactant.
Ans: B Reference:BRS Physiology.
25) Receptors of thyroid hormone present 31) Muscle paralyzed during difficult
on: child birth:
SK Origional – Golden 11 292

a. Iliacus c. B fiber
b. Levator Ani d. A delta fiber
c. Perineal body Ans: A
d. Obtruator Internus Explanation:―C – Fibers‖ are the slowest nerve
Ans: B fibers because they are Un-myelinated that
32) A female present with pink frothy transmits slow pain.
vaginal discharge with vulval pruritus 37) An old lady presented in medical
caused by: emergency in unconscious state. Her
a. HSV Bp 180/90 Temp 39 Pulse 90.Labs are
b. Trichomonas Vaginalis Glucose 350mg/dl Na 130, K 3.5, Urea
c. Gonorrhea is 10. Diagnosis?
d. Syphilis a. Chronic Renal Failure
Ans: B b. RTA
Explanation: In trichomonas vaginitis patient c. Diabetic Keto acidosis
have frothy,yellow-green, foul smelling discharge d. Honk
(strawberry cervix).On lab finding motile pear Ans: A
shaped trichomonads can be seen. Explanation: Since patient is Hypertensive,Urea
Treatment: Metronidazole is increased,Unconscious due to Urosepsis, All are
33) Primitive streak mesoderm migrates indicating about chronic renal failure.Since
to anterior abdominal wall around potassium level is also normal range in CKD
cloaca. A rupture of this mesoderm potassium level is less likely to fluctuate.
cranially will produce: 38) Of 20 women with neonates getting
a. Ectopic anal canal Tetanus 19 had not taken Tetanus
b. Imperforate anus Toxoid Vaccine and of 40 whose
c. Bladder extrophy babies did not have Tetanus 30 has
d. None taken two tetanus toxoid shots during
Ans: A Pregnancy. The odd ratio will be?
34) A patient is suffering from chronic a. 10
renal failure and on dialysis. Increase b. 11
in which parameter of blood is c. 7
alarming? d. 8
a. Calcium e. 9
b. Creatinine Ans: E
c. Urea 39) One gram of haemoglobin binds with
d. Sodium how much oxygen?
e. Potassium a. 3.0ml
Ans: E b. 1.34ml
35) Highest systolic pressure is present in: c. 2.45ml
a. Renal artery d. 5ml
b. Aorta Ans: B
c. Pulmonary Artery Explanation: Healthy person have 15gm of Hb
d. Carotid artery which binds with 20.1 of oxygen i.e 1.34×15=20.1
Ans: A so each 1 gm of Hb binds with 1.34ml of oxygen.
Explanation: Glomerular capillaries are unique 40) Tamoxifen is given to a patient who is
because they have Arterioles on both having Ca Breast. Keeping in mind
sides so pressure is high around Tamoxifen have Estrogenic and Anti
50mmHg.Since there is no dissipation of estrogenic effect what would be
arterial pressure that‘s why renal artery possible side effect:
have highest systolic pressure. a. Endometrial Carcinoma
36) Slowest nerve fiber: b. Endometrial hyperplasia
a. C fiber c. Endometriosis
b. A fiber d. Stroke
SK Origional – Golden 11 293

Ans: A Explanation:Cholesterol is made up of Sterol a


Explanation:Tamoxifen is selective Estrogen type of Lipid and embedded in Lipid Bilayer thus
Receptor modulator. allowing Ion movement across it.
Anti-Estrogenic action: Reduce Breast Tumor 45) A laboratory boy was analysing CSF.
size and Causes hot flushes. What he finds in it‟s composition?
Estrogenic Action: Endometrial a. Increase Protein
Proliferation,Reduces Cholesterol and Anti b. Increase pH
resorptive effect on bones. c. Decrease Magnesium
41) An alcoholic female presented with d. Decrease Glucose
history of 4 kg weight loose in last few Ans: D
months,Temp 38.5 Pleuritic chest Explanation:
pain,Cough with Hemoptysis.On All about CSF
chest x ray b/l Perihilar finding was  Average adult has 90-150ml of CSF,
present.Her alveolar wash shows a few although 400-500ml produced daily.
epitheliod cells.CBC shows:  Normal CSF is Isotonic (Na is equal to
Neutrophils-78, Lymhocytes-8, serum i.e. 138mEq/L
Eosinophils-3. What is Probable  pH 7.33
Diagnosis?  High Concentration of Chloride and
a. Sarcoidosis Magnesium
b. Hamartoma  Lower concentration of Potassium
c. TB calcium and Bicarb as well as glucose.
d. Pneumonia  Secreted by Choroid Plexus and
Ans: C Absorbed by Arachnoid Granulations.
Explanation:Peri hilar lymphadenopathy and Reference: Kaplan Neuroanatomy.
Epitheliod cells are Hallmark of TB. 46) An 11 years old boy presented with
42) A 70 years old Man non-smoker bleeding after minor trauma. Her
presented with painless Hematuria siblings have same issue. His Pt. time
and a Diagnosis of Ca badder is made. is normal but Aptt and bleeding time
What is the Risk factor for Ca bladder? is prolonged. What is the cause?
a. Aniline dye a. Platelet Deficiency
b. Benzidine b. Intrinsic pathway
c. Vinyl chloride c. Extrinsic pathway
d. Silica d. Liver failure
e. Hydrocarbons Ans: B
Ans: A Explanation:
43) A man having Obstructive lung This is the case of Von willebrand disease which is
disease which gradually decreases intrinsic pathway coagulation defect i.e. dec. level
gaseous exchange between Alveoli? of vW factor so no platelet plug is formed despite
a. Pneumonia of normal Platelet count.
b. Pulmonary fibrosis 47) Which one is correct match?
c. Pulmonary edema a. Ca Lung with Alcohol
d. Emphysema b. Ca liver with Asbestos
e. Asbestosis c. Ca Skin with alcohol
Ans: D d. Ca liver with alcohol
44) Lipid bilayer of cell membrane have Ans: D
cholesterol in its structure.Function of 48) Cytokeratin stains for purpose to
this cholesterol in membrane is: diagnose:
a. Intercellular attachment a. Sarcoma
b. Permits Ion movement b. Endothelial Injury
c. Solubility c. Carcinoma
d. Phagocytosis d. Hypertrophy
Ans: B e. Metaplasia
SK Origional – Golden 11 294

Ans: C 54) Most important factor for kidney


49) A Lady with Galactorrhea, transplant is:
Amenorrhea, Hypogonadism a. WBC
Diagnosed as Hyperprolactinoma b. HLA
what is it‟s treatment? c. RBC
a. Bromocriptine d. MHC
b. Levothyroxine e. Platelets
c. Insulin Ans: B
d. Vit. D Explanation: MHC is encoded by human
Ans: A leukocyte antigen i.e. HLA thus regulate immune
Explanation: Bromocriptine is Dopamine system.
(D2)Agonist which shrinks macroadenomas. 55) Water Measurement is done by:
50) Oxidation of iodine occurs in: a. Antipyrine
a. Para follicular b. Creatinine
b. Follicular cells c. PAH
c. Principle cells d. Inulin
d. Peripheral tissue e. Insulin
e. Liver Ans: A
Ans: B 56) Young patient with burning
51) Propyl thiouracil act by deactivation micturition and cloudy urine, No
of: fever, No pelvic tenderness. Lactose
a. Triiodothyronine fermenting rods on MacConkey agar.
b. Iodide The Causative organism is:
c. T4 a. Proteus
d. Thyroxin b. Mycobacteria
e. Tyrosine c. Staph Aureus
Ans: B d. E. coli
Explanation: PTU acts by inhibiting oxidation of e. Shigella
iodide by inhibiting thyroid per oxidase enzyme. Ans: D
52) O2 and CO2 diffuse through cell 57) Mark the correct sequence of
membrane because: leukocyte extravasation:
a. Their size is small a. Margination Rolling Attachment
b. They are lipid soluble Transmigration
c. They are water soluble b. Rolling Margination Attachment
d. Do not cross cell membrane Transmigration
e. Carrier mediated movement across cell c. Transmigration Margination Rolling
membrane Attachment
Ans: B d. Attachment Margination Rolling
Explanation: Transmigration
Lipid soluble substance: CO2,O2, Steroid Ans: A
Hormones. Explanation: In acute Inflammatory response
Water soluble: Na,Cl,Glucose and H2O. Leukocyte Extravasation has 4 Steps:
They cannot dissolve in membrane so may cross  Margination
through water filled channels or pores or maybe  Rolling
transported by Carriers.  Attachment
53) Patient blood agglutinates with anti-  Transmigration and Migration
sera A and D which blood Group? (Chemoattraction).
a. A+ Reference: First Aid page 215.
b. AB - 58) Which mechanism causes decrease
c. O- tension in muscle fibres?
d. A- a. Withdrawal Reflex
Ans: A b. Stretch Reflex
SK Origional – Golden 11 295

c. Golgi tendon refelx Ans: B


d. Knee Jerk 64) Which parameter decrease in
Ans: C pregnancy?
Explanation: a. Blood
The inverse Stretch Reflex i.e Golgi Tendon reflex b. Respiratory Rate
uses Golgi tendon organ (GTOs) which are c. Tidal volume
oriented in series with extrafusal fibres and d. CO2
respond to increase in Force or tension generated Ans: D
in that muscle.Hence monitors Muscle Tension. 65) Plasmodium falciparum is:
59) Posterior relation of First part of a. Unicellular Amoeba
duodenum is: b. Cyst
a. Aorta c. Schizoid
b. IVC d. Ciliate
c. Left Gastric Artery e. Flagellum
d. Gastrodeodenal artery Ans: A
e. Splenic Artery 66) Dormant stage of malaria:
Ans: D a. Egg
60) A 38 years old man got Pelvic fracture b. Cyst
in RTA. He loses 2L of Blood in c. Sporozyte
accident. After 5 days he died and his d. Hypnozoite
brain CT scan shows multiple areas of Ans: D
infarction. What is the cause of his 67) From behind, Lateral to medial
death? arrangement of these three in males
a. Air Embolism are:
b. Fat Embolism a. Ureters, Seminiferous ducts and Vas
c. Sepsis defrens
d. Neurogenic shock b. Ampulla, Seminiferous ducts and Ureters
e. Violence c. Ureters, Seminiferous ducts and Ampulla
Ans: B d. Kidney,Ureters, Ampulla
61) Burkitt lymphoma translocation: e. Seminiferous Ducts,Ampulla,Urethra
a. 9:22 Ans: C
b. 8:21 68) A Security guard got gun-shot injury
c. 21:11 on lower back.After physiotherapy and
d. 8:14 rehabilitation he had lost contralateral
Ans: D Pain/temp below the level of lesion
62) A six years old girl presented with iron and ipsilateral loss of Vibration.What
deficiency anaemia. Her spleen is is diagnosis?
palpable and frontal bossing is also a. Damaged 1stdosral root
present. What is the cause of her b. Damaged ventral root
illness? c. Brown sequard syndrome
a. Malaria d. Developing tabes dorsalis
b. Iron deficiency Anemia e. Syringomyelia
c. Thalasemia major Ans: C
d. Megaloblastic Anemia Explanation:
Ans: C Classical scenario of Brown sequard syndrome in
63) A 2 years old boy died his autopsy which Hemisection of spinal cord occurs followed
reports shows multiple cysts in by trauma/injury in which ipsilateral loss of
kidney. What is the pattern of disease? vibration and position sense below level of
a. Autosomal Dominant lesion,Contralateral loss of pain below 1-2
b. Autosomal recessive segments of lesion,Ipsilateral Paresis, LMN -
c. X linked Recessive Flaccid paralysis.
d. X linked Dominant Syringomyelia is cavitation of spinal cord in which
SK Origional – Golden 11 296

bilateral loss of pain and temperature occurs Ans: C


below lesion. 74) A person stands up from supine
Tabes dorsalis is related to Late stage Syphilis. position what will happen in order to
69) Common feature of Glucagon and compensate?
Epinephrine? a. Increase pulse pressure
a. Proteolysis b. Vasodilation
b. Vasoconstriction c. Decrease TPR
c. Decrease Glucose d. Decrease stroke volume
d. Phosphorylase kinase activity e. Decrease Venous return
Ans: D Ans: A
70) A patient with Sciatic nerve Explanation:
compression, root value: Orthostatic hypotension — also called postural
a. S1 hypotension — is a form of low blood pressure
b. S2 that happens when you stand up from sitting or
c. S3 lying down.The most important parameter raised
d. S4 in order to compensate this is heart rate which
Ans: A wasn‘t in option so here answer is ‗Increase Pulse
71) Liver maintain metabolism of body Pressure ‗which is due to Increase Stroke Volume.
between food intakes by: Other compensatory responses are
a. Gluconeogenesis Heart = Increase heart rate,increase
b. Fatty acid metabolism contractility,increase cardiac output
c. Proteolysis Arterioles=Increase TPR
d. Glycogenolysis Veins:Increase Venous return
Ans: D 75) A 22 years old female presented in
72) Derivative of 6thbranchial arch: Gynae opd with grey color vaginal
a. Laryngeal cartilage discharge which was non painful. On
b. Facial Bone histopathology clue cells were
c. Thyroid positive. What is the cause of disease?
d. Cricoid a. Trichomonas vaginalis
e. Stylopharyngneus b. Bacterial vaginosis
Ans: D c. Chlamydia
Explanation: d. Herpes type 1
Branchial Arch 6:Cartilage Derivatives e. HPV
 Thyroid cartilage Ans: B
 Cricoid Explanation:
 Arytenoids Bacterial Vaginosis is caused by
 Corniculate Gardnerellavaginalis whose characteristic finding
 Cuneiform is clue cells.
Branchial arch 6: Muscle derivatives 76) Uterine blood supply:
 All intrinsic muscles of larynx (except a. Uterine and ovarian artery
cricothyroid) b. Uterine and superior vesicle artery
Branchial arch 6: Nerve derivatives c. Uterine and inferior vesicle artery
 CN X (recurrent laryngeal branch- d. Renal and Ovarian artery
speaking). e. Ureteric and Uterine Artery
73) What is codominance? Ans: A
a. A dominant gene will overpower all other 77) Descent of ovary out of abdomen is
genes to be the only one prevented by?
b. One is recessive and other gene is a. Inguinal Ligament
dominant b. Rectus Muscle
c. Where two different traits are both c. Narrow Inguinal canal
expressed alongside each other d. Uterus
d. None e. Bladder
SK Origional – Golden 11 297

Ans: D c. Tyrosine
78) A Patient is having severe diarrhoea d. Spectrin
from 3 days what will happen: e. Myosin
a. Metabolic alkalosis Ans: B
b. Metabolic acidosis 83) Covid-19 current name is:
c. Respiratory acidosis and Alkalosis a. Influenza type 2
d. No acid base balance disturbance b. Sars 2
e. Metabolic Alkalosis and Metabolic c. Sars 1
acidosis. d. Paramyxovirus
Ans: B e. HHV-8
Explanation: Ans: B
Acute Diarrhea=Normal anion Gap Metabolic 84) A patient presents with hypoglycemia
Acidosis. and Hepatomegaly. A glycogen
Chronic Diarrhea =Metabolic Alkalosis storage disease known as von gierke
79) 1 degree decrease in body disease is diagnosed. Which enzyme
temperature. What %age of CMRO2 is deficient in this patient?
decreases? a. Alpha 1,4 Glucosidase
a. 3% b. Glucose 6 Phosphatase
b. 8% c. Myophosphorylase
c. 15% d. Alpha 1,6 Glucosidase
d. 28% Ans: B
e. 32% 85) The Glucose molecule present In
Ans: B Glycogen side chain is metabolized by
Explanation: which enzyme?
Normal CMRO2 i.e. Cerebral metabolic rate of a. ATP
oxygen is 3.0-3.8 ml O2/ 100gm b. ADP
brain/min.Hypothermia decreases cerebral c. Glucose 1 phosphorylase
metabolic rate (CMR) by 6% to 7%. d. Kinin
80) In post streptococcal e. Reductase
glomerulonephritis finding is: Ans: C
a. Linear depositions 86) Cobalamin transported in blood by:
b. Mesangial proliferation a. Albumin
c. Sub epithelial deposits b. Transcobalamin 1
d. Alveolar damage c. Transcobalamin 2
e. Sub Endothelial Deposits d. Heptoglobin
Ans: C e. Transferrin
81) A 4 years old female child is Ans: C
diagnosed case of Cystic fibrosis.She 87) 45 years female was on prolonged bed
also developed lung abscess due to rest and now she is having shortness
Pseudomonas auruginosa.What of breath and peripheral edema. Her
complication is most likely to chest X-ray shows many areas of
develop? infarction.
a. Bronchiectasis a. Massive pulmonary embolism
b. Atelectasis b. Recurrent pulmonary embolism
c. Pneumothorax c. Left Heart Failure
d. Rib Fracture d. Pulmonary Edema
e. Pulmonary Edema e. Stroke
Ans: A Ans: B
82) Duchene muscular destroy is due to Explanation: Here we choose Recurrent
deficiency of: Pulmonary Embolism because in x ray areas of
a. Amino acid infarct shows previous history of mild pulmonary
b. Dystrophin infarction and patient is also having peripheral
SK Origional – Golden 11 298

edema a major risk factor for DVT which leads to 93) Vesicles on vulva are due to:
embolism and also look for signs of Right Heart a. HPV 18
Failure(Cor pulmonale) secondary to Chronic b. Herpes 1
Pulmonary Disease i.e. Recurrent c. Herpes 2
Thromboembolism in such stems. d. Varicella
88) Cacl2 was given to a Rat in a e. Chlamydia
laboratory which leads to Ans: C
Hypercalcemia. Now what will 94) Pap smear positive by:
increases? a. HPV 18
a. 1,25 Dihydroxycholecalciferol b. HPV 16
b. 24,25 Dihydroxycholecalciferol c. Chlamydia
c. Cortisol d. Trichomonas
d. Epinephrine Ans: B
e. Cholecalciferol 95) Preganglionic sympathetic fibers:
Ans: B a. White rami
89) In hypernatremia what will happen? b. Gray Rami
a. Cell Lysis c. Dorsal Root
b. Degranulation of lysomal enzymes d. Dorsal primary ramus
c. Cell shrinkage e. Ventral primary ramus
d. Cell swelling Ans: A
e. Decrease osmolarity of ICF Explanation: Sympathetic polyneuronal signals
Ans: C starts from Hypothalamus and Pre ganglionic
Explanation:Hypertonic solution sympathetic reach in LATERAL or
(Hypernatremia) water shifts from ICF to ECF as INTROMEDIAL HORN of spinal cord.Then
a result of this shift dec. in ICF which causes cell from here these pre ganglionic sympathetic fibres
shrinkage. leaves as WHITE RAMI( Name white rami
90) A patient is suffering from Sepsis and because these fibers are Myelinated). So
Hypotensive. We give him I.V fluids Intromedial/Lateral horn>White Rami
but didn‟t resolve. What happen to 96) Cortisol and Fatty acid metabolism.
this patient? Hormone involved is:
a. Hypovolemic Shock a. Inulin
b. Anaphylaxis b. Prolactin
c. Haemorrhagic Shock c. Insulin
d. Refractory to fluid therapy d. T4
e. Sepsis e. GnrH
Ans: D Ans: C
Because of Third spacing 97) Thiamin deficiency causes:
91) Primary gravida normal delivery a. Rickets
which structure damage? b. Osteoporosis
a. Ischial spine c. Color blindness
b. Sacrotuberous Ligament d. Beri Beri
c. Perineal Body Ans: D
d. Ureter 98) A baby with Edema
e. Levator ANI hepatosplenomegaly and
Ans: E hyperpigmentation. Diagnosis:
92) Potent anabolic hormone is: a. Kwashiorkor
a. Cortisol b. Celiac Disease
b. Prolactin c. Marasmus
c. Testosterone d. Rickets
d. Insulin e. Scurvy
e. Glucagon Ans: A
Ans: C 99) 1g of fat give:
SK Origional – Golden 11 299

a. 4 kcal here I prefer C Because Parasympathetic


b. 8kcal has pronounced effect on Enteric
c. 12kcal Nervous system.
d. 25kcal 105) Dorsal column medial lemniscal
e. 9kcal pathway is made up from?
Ans: E a. Nucleus Gracilis+ Nucleus Cuneatus
100) By increasing the specific gravity of b. Nucleus Ambiguous +Nucleus Cuneatus
urine which kidney function is Being c. Inferior olivary nucleus
assessed? d. Nucleus Gracilis + Nucleus Ambiguous
a. Filtration Ans: A
b. Reabsorption 106) A Tumor with mature cells in male is:
c. Concentration a. Teratoma
d. Secretion b. Seminoma
e. Dehydration c. Theca cell Tumor
Ans: C d. Lymhoma
101) Brunner glands are present in: e. Yolk sac tumor
a. Cecum Ans: A
b. Jejunum 107) What is Correct about Ultrasound
c. Sigmoid colon waves?
d. Pylorus a. Can pass easily through bones than
e. Duodenum muscles
Ans: E b. Iron shield is protective
102) A Soldier came from high altitude c. Can only do in last trimester
presents with Peripheral Cyanosis and d. Have frequency more than 20,000 hz
breathlessness. Cause: Ans: D
a. Polycythemia Rubra Vera 108) Chorioamnitis most specific indicator:
b. ITP a. IL-1
c. Megaloblastic Anemia b. IL-6
d. Secondary Polycythemia c. IL-8
e. Leukemia d. TNF
Ans: D Ans: B
103) A newborn baby have blood group A+ 109) A little boy was stung by bee. He had
and her mother have Blood group B- generalized body swelling and
which results in Haemolytic disease of anaphylactic shock. Which one is
new born.Which antibody is culprit for mediator for this reaction?
this condition of newborn? a. Histamine
a. IgA b. Bradykinin
b. IgM c. Interleukin 1
c. IgM and IgG d. Creatinine
d. IgG e. Neutrophils
e. IgD Ans: A
Ans: D 110) A patient is having Post abortion
Explanation: Because IgG is the only antibody sepsis.What is causative organism?
which can cross placenta. a. Group B Streptococcus
104) Parasympathetic Activation causes: b. Staphylococcus
a. Decrease Heart Rate c. E.coli
b. Increase Secretions from various glands d. Syphilis
of body e. Bacteriods
c. Relaxation of GIT sphincters Ans: A> C > E
d. Bronchoconstriction 111) Cystercosis mode of transmission is:
e. Pupillary Dilatation a. Ingestion of Eggs of Taenia coli
Ans: Although A,B,C and D all are correct b. Ingestion of Cyst of Taenia coli
SK Origional – Golden 11 300

c. Ticks columnar epithelium with cilia:


d. Mosquito bite a. Cervix
e. Ingestion of Uncooked meat. b. Fallopian Tube
Ans: A c. Vagina
112) Most difficult way of controlling d. Salivary Glands
infection is: e. Duodenum
a. Air borne Ans: B
b. Water borne 120) Tubular structure with thick three
c. Blood transfusion layers of muscle; walls are Elastic and
d. Contaminated Food have adventitia around it. Which
Ans: A structure is this?
113) Prevention in taking Pap smear of a. Vein
ladies above age 30 is: b. Muscular artery
a. Primary Prevention c. Arteriole
b. Secondary Prevention d. Capillary
c. Tertiary Prevention Ans: B
d. Disease Analysis 121) Which muscle is classified as white
Ans: B muscle?
114) About female urethra: a. Soleus
a. Longer than in Males b. Gastrocenemous
b. Prone to infection c. C: Cardiac Muscle
c. Have External and Internal sphincter d. Trapezius Muscle
d. Lies within Pelvis e. Bicep Femoris
Ans: B Ans: B
115) Pituitary adenoma anterior extension 122) First minute of Apnea level of CO2
will compress: rises:
a. Optic nerve a. 8
b. Optic Chiasma b. 10
c. Cavernous Sinus c. 11
d. Hippocampus d. 20
e. Ciliary Ganglion e. 6
Ans: B Ans: E
116) Staging of tumour: 123) How total peripheral resistance is
a. Extend and severity of spread of Tumor increased?
b. Cell Differentiation a. By sympathetic stimulation via Apha 1
c. Degree of Metaplasia receptors.
d. Nuclear Cytoplasmic Ratio b. By sympathetic stimulation of B2
Ans: A receptors
117) Why rupivacaine preferred c. Parasympathetic Activation
bupivacaine? d. Increasing Stroke Volume
a. Less Toxic Ans: A
b. Shorter duration of Action 124) Pus is drained from ischiorectal fossa.
c. Long Duration effect nerve injured during this procedure is:
d. Causes Sedation a. Pudendal Nerve
Ans: A b. Illiohypogastric
118) Venous return directly proportional to: c. Inferior Rectal Nerve
a. Mean systemic filling pressure d. Genitofemoral
b. Decrease Stroke volume e. Femoral
c. Skeletal muscle relaxation Ans: C
d. Venous Stasis 125) Intestine loops failed to return in
Ans: A abdomen termed as:
119) Tubular structurehas smooth muscles, a. Gastroschisis
SK Origional – Golden 11 301

b. Omphalocele b. Epinephrine
c. Umbilical Hernia c. Insulin
d. Bladder Extrophy d. Cortisol
Ans: A e. Thyroxin
126) A 46-year-old man immunodeficiency Ans: C
virus (HIV)-negative man presented 131) Largest cell in blood:
with an exudative effusion. Dense a. Monocyte
black pigment within macrophages b. Megakaryocytes
was identified in the both lung Hilus. c. Neutrophils
Diagnosis? d. Basophils
a. Pulmonary Tuberculosis e. Eosinophils
b. Anthracotic pigment Ans: A
c. Silicosis Explanation:
d. Asbestosis Largest cell in Blood=Monocytes
e. Bronchogenic Carcinoma Largest cell in Bone marrow= Megakaryocytes
Ans: B 132) Hyaline cartilage Present between:
Explanation: In pneumoconiosis of coal workers a. Symphysis pubis
carbon-laden macrophages can be found in b. Intervertebral Joint
exudative and transudative pleural effusion as well c. Manubrium-sternal joint
as on peri hilar area. d. 1stchondrosternal joint
127) In trauma what is the metabolic Ans: D
response of skeletal muscle: 133) 24, 25 dihydroxycholecalciferol
a. No change increase in:
b. Increase Proteolysis a. Chronic Renal failure
c. Decrease proteolysis b. Hyper parathyroidisam
d. Ketogenesis c. Hyperthyroidism
e. Lipolysis d. Prolactinoma
Ans: B e. Paget‘s Disease
Explanation: Ans: A
Surgical stress and trauma result in a reduction in 134) Goblet cells are present up-to:
protein synthesis and moderate protein a. Alveoli
degradation. Severe trauma, burns and sepsis b. Primary Bronchiole
result in increased protein degradation. c. Terminal Bronchiole
128) Basic function of kidney: d. Alveolar Duct
a. Excretion of wastes e. Secondary Bronchus
b. Maintenance of body Ph Ans: C
c. Hormone production Explanation:
d. Maintain BP Terminal bronchioles have Simple cuboidal
e. Osmoregulation epithelium, comprising of Clara cells and
Ans: A ABSENT Goblet cells.Before (up-to) terminal
129) Patient has malnutrition and bronchiole Respiratory tract have Goblet cells.
dependent edema which test should If we choose option D that means terminal
be done? Bronchiole have Goblet cells so better option is C.
a. Coagulation Profile 135) Carotid supply to foetus from aorta is:
b. Serum Electrolytes a. Ductus Venosum
c. Serum Folic acid level b. Ductus Arteriosus
d. Serum Ferritin c. Pulmonary Artery
e. Serum Albumin d. Jugular Vein
Ans: E Ans: B
130) Which one of these use fats for 136) A 6 year old girl presented in medical
energy? emergency in Unconscious state.She
a. Nor Epinephrine had a history of diarrhoea 3 days
SK Origional – Golden 11 302

ago.Her breath was fruity and serum e. Taenia solium


glucose was 456mg/dl. What is Ans: C
Diagnosis? 142) A 48 years old hypertensive patient
a. HONK presented with severe Headache and
b. Diabetes Mellitus vomiting. On lumbar puncture CSF
c. Diabetic Keto-acidosis was blood stained. What is the cause?
d. Renal Failure a. Hydrocephalus
e. Parasitic Infection b. Cerebral Edema
Ans: C c. Dengue
137) Which one is benign tumor? d. Sub arachnoid hemorrhage
a. Wilms tumour e. Cerebral Abscess
b. Warthin tumor Ans: D
c. Hepato-cellular 143) An old patient presented to the
d. Bronchogenic emergency room for a fatigue,
e. Pleomorphic dyspnea and chest pain which started
Ans: B yesterday morning. He has been
Explanation: experiencing cough with sputum from
Warthin's tumor, also known as papillary many years.WBC 10,000,Hematocrits
cystadenoma lymphomatosum, is a benign cystic 25.4%,Haemoglobin 8.4 g/dL,MCV85
tumor of the salivary glands. fL,Platelets 154,000 and Glucose 124
138) During surgery of thyroid gland which mg/dL.The clinical scenario is most
nerve is at risk while ligating superior consistent with which type of anemia?
thyroid artery? a. Folate deficiency
a. Recurrent Laryngeal Nerve b. Megaloblastic anemia
b. External Laryngeal Nerve c. Anemia of chronic disease
c. Accessory Nerve d. Thalassemia
d. Facial Nerve e. Iron deficiency anemia
e. Mandibular branch of trigeminal nerve. Ans: C
Ansr: B Explanation: Normocytic- Normochromic
Explanation: Anemia
Relation of superior thyroid artery is with external 144) A 60-year-old man w/ fatigue,
laryngeal nerve while with inferior thyroid artery is weakness, and exercise intolerance.
recurrent laryngeal nerve. Labs: Hemoglobin 9.1 mg/dL,
139) Columnar epithelium with muscular Ferritin 9 ng/mL, Mean corpuscular
clefts and smooth muscles: volume (MCV) 110 fL. Diagnosis:
a. Lungs a. Iron Deficiency Anemia
b. Uterus b. Megaloblastic Anemia
c. Cervix c. Sideroblastic Anemia
d. Fallopian tubes d. Auto immune haemolytic Anemia
e. Ovaries Ans: B
Ans: C 145) A 57 year old male with a history of
140) In Pakistan thalassemia major: alcoholism presented to the
a. 5% emergency room with abdominal pain,
b. 10% jaundice and hyperbilirubinemia.His
c. 20% labs show AST (SGOT) 88 U/L,ALT
d. 25% (SGPT) 121 U/L, Bilirubin Total 6.5
Ans: A mg/dL.What is the cause of leakage
141) Brain eating parasite is: of these liver enzymes in blood?
a. Dengue a. Liver Metabolism
b. Plasmodium falciparum b. Metastasis
c. Naegleria fowleri c. Due to Hepatocyte cell membrane
d. Ascariasis Damage
SK Origional – Golden 11 303

d. Fibrosis e. Hep. D
e. Due to Hepatocyte Degeneration Ans: D
Ans: C 151) About standard deviation true is:
146) A 13 years old boy presented with a. Statistic that measures the dispersion of a
Rose spots on Chest with data set relative to its mean on either
fever,Headache Nausea and sides.
vomiting.On examination his b. Calculated as the square root of mean.
abdomen was Doughy.What c. Prevalence
confirmatory Test is needed to d. Not a main measure of variability
diagnose this disease? Ans: A
a. VDRL 152) The most important factor for
b. Sputum AFB coagulation to begin is:
c. Widal Test a. ITP
d. Stool Culture b. Rough endothelial surface
e. Bone marrow Biopsy c. Acute inflammation
Ans: C d. Increase viscosity
Explanation: Rose spots are diagnostic for Ans: B
enteric fevercausative agent is salmonella typhi. 153) Angiotensin II causes:
147) A person got a lesion on basal ganglia. a. Increase Thirst
What will happen? b. Decrease urine osmolarity
a. Intension Tremors c. Bronchoconstriction
b. Involuntary Movement d. Volume loss
c. Peduncular knee jerk Ans: A
d. Nystagmus 154) During poisoning best route of
e. Lack of Coordination antidote is:
Ans: B a. IM
148) Nerve supply of levator ANI muscle b. Oral
is: c. IV
a. Inferior Hypogastric d. Sub lingual
b. Femoral Nerve Ans: C
c. Obturator Nerve 155) A kid is anaemic and history of a
d. Superior Gluteal Nerve worm coming out of mouth. Which
e. Pudendal Nerve parasite is involved?
Ans: E a. Loa Loa
149) Hormone responsible for development b. Entamoeba Histolytica
and proliferation of Duct, Alveoli and c. Ascariasis lumbercoides
secretion of breast lobules? d. Malaria
a. Prolactin Ans: C
b. Estrogen 156) An abscess is formed on big toe.
c. Progesterone Where would be its Lymph drainage?
d. Oxytocin a. Vertical Group of inguinal lymph nodes
Ans: A b. Horizontal group of inguinal lymph
Don‘t choose Progesterone here because it has no nodes
role in Breast Secretion in contrast it inhibits c. Popliteal lymph nodes
secretion. d. Deep inguinal lymph nodes
150) A Pregnant Lady visited to Remote Ans: A
area and developed jaundice 157) A 32 years old G2+P1+0 presented
afterwards. What is cause? with Gestational Amenorrhea of 33
a. Hep. A weeks. She complains of severe
b. Hep. C headache and vomiting.Her Bp is
c. Hep. B 180/90mmHg.She also has history of
d. Hep. E Fits during last pregnancy.Which
SK Origional – Golden 11 304

antihypertensive drug you will d. Upper border of 9thintercostal space at


immediately give to this patient? mid clavicular line.
a. MgSo4 e. Diaphragmatic recess.
b. Hydralazine Ans: B
c. Methyl Dopa 162) A patient got RTA after which he had
d. Captopril winging of scapula. Which Nerve is
e. Diuretic most likely involved?
Ans: B a. Axillary
158) In a medical research some doctors b. Facial Nerve
comparing two group of people and c. Spinal part of Accessary Nerve
checking the relation of a disease with d. Long Thoracic Nerve
it‟s Risk factors.Which type of study? Ans: D
a. Cross sectional data 163) A young Man present with complain
b. Case Control of one sided facial flushing and heat
c. Cohort up with no sweating. Where is defect?
d. Cross sectional Analysis a. Sympathetic loss
e. Probability b. Facial nerve
Ans: D c. Autonomic outflow Loss
Explanation: d. Cranial outflow
Case control: Disease and non-disease. e. Parasympathetic loss
Cohort: Group with risk factor and group without Ans: A
risk factor. Explanation:
Cross sectional:Risk with disease and non-disease. Don‘t confuse the stem with Frey‘s syndrome in
159) A young lady presented in OPD with which gustatory sweating is present here no
primary amenorrhea. On examination sweating only facial flushing is present?
breast is present but uterus is absent 164) A Young man presented with
and blind vagina. What is karyotype? recurrent episodes of Syncope. These
a. XYY syncope episodes are related to reduce
b. XO cerebral perfusion when the arm
c. XY ipsilateral to the subclavian stenosis is
d. XXY exercised. subclavian steal
Ans: C phenomenon (SSP) occurs when there
Testicular feminization syndrome. is stenosis or occlusion of the
160) A young boy presented with enlarged subclavian artery. In SSP which artery
genitals, early pubic hair is stealing blood and shunting this to
appearance,excessive body hair and arms?
rapid growth. What is Cause?
a. Acromegaly a. External carotid
b. Prolactinoma b. Internal carotid
c. Turner Syndrome c. Thyrocervical trunk
d. Adrenogenital syndrome d. Vertebral artery
e. Klinefelter Syndrome e. Cerebral artery
Ans: D Ans: D
161) A patient is having pleural effusion 165) The potency of anaestheic directly
from where pleural drainage should be related to:
done? a. Water Solubility
a. Upper border of 9thintercostal space at b. Lipid Solubility
mid axillary line. c. Drug-Drug Interaction
b. Lower border of 9thintercostal space at d. Age of Patient
mid axillary line. Ans: B
c. Lower border of 9thintercostal space at 166) Particular feature of cardiac muscle as
mid clavicular line. compare to skeletal muscle is:
SK Origional – Golden 11 305

a. No Gap Junctions e. Autonomy


b. Striated Ans: D
c. Never get tetanized 172) Stenson‟s duct anatomical location is:
d. Contains T tubule a. Inside buccal membrane
e. Actin and Myosin forms sarcomere b. Lateral to Ramus of Mandible
Ans: C c. Between superficial and deep parotid
167) If Heart pumps blood and then gland
stopped so that circulatory system d. Auriclar border
becomes static after this pressure e. Runs below masseter
measured in rest of the body is? Ans: A
a. Stroke volume Explanation:
b. Total peripheral resistance Parotid duct or stenson‘s duct is 5cm long and
c. Mean systemic filling pressure arises from anterior part of parotid gland,runs
d. Haemodynamic over masseter a finger‘s breadth below the
e. Pulse pressure zygomatic arch to pierce the buccinators and open
Ans: C opposite the second upper molar tooth.
168) A group of students were studying —Between superficial and deep parotid gland
difference between PO2 and PCO2 in Facial Nerve is present not Parotid Duct.
different arteries of the body. They Submandibular Gland=Wharton‘s Duct –Opens
find major differences between: in Floor of Mouth
a. Aorta and Pulmonary Artery Sublingual=Duct of rivinous- Multiple small
b. Skeletal muscle arteriole and venule Ducts.
c. Aorta and Left ventricle 173) Bone density depends on:
d. Right and Left Ventricle a. Collagen to Osteon matrix ratio
e. Aorta and Superior Vena cava b. Decrease density of haversion canal
Ans: B c. Increase parathyroid hormone
169) Parasymthetic loss affects which d. Vitamin D and calcium levels
major component of body? Ans: A
a. Skeletal 174) Left supra renal gland is supplied by:
b. GIT a. Parasympathetic
c. Light Reflex b. Inferior phrenic
d. Vascular smooth muscles c. Drain into IVC
Ans: B d. Internal thoracic
170) The main component of Ans: B
doctor/patient relationship is: 175) About Astrocytes action potential:
a. Ambiguity a. Action Potential is Generated in
b. Mutual trust Hypernatremia
c. Informed Consent b. Action Potential is Generated when
d. Justice intracellular Na is increased
Ans: B c. Action Potential is Generated with
171) A 18 years depressive female was Increase Extracellular Potassium
taking Psychotherapy from a Doctor d. No Action Potential
and she was improving. One day her Ans: C
Aunt call her doctor and asked him Explanation: Astrocytes are extracellular K
about her Niece issues. Doctor buffers.
refused to disclose any information 176) An 11 years old girl got a fracture on
related to her. This action of doctor Humerus. Where does Secondary
lies in which core ethical principles? ossification center is located in this
a. Beneficence age?
b. Non-Maleficence a. Medial Epicondyle
c. Justice b. Trochlea
d. Confidentiality c. Lateral Epicondyle
SK Origional – Golden 11 306

d. Capitulum a. Sympathetic trunk


e. Distal end of Femur b. Internal Jugular Vein
Ans: C c. Ansa Cervicalis
177) After anesthesia heart rate increase d. Internal Carotid artery
and hypertension develops: e. Accessory Nerve
a. Ketamime Ans: A
b. Propofol 183) Where does rbb present in heart?
c. Procainamide a. Left Atrium
d. Rupivacaine b. Sulcus Terminalis
e. Halothane c. Crista terminalis
Ans: A d. Septomarginal
178) New inflammatory marker for Ans: D
ischemic heart disease which is easy 184) A girl presented with persistent cough
to do is? with thick mucus,wheezing and
a. C-Reactive protein Repeated lung infections.She is also
b. IL-1 having petechia all over body and
c. TNF-a have bleeding tendency.She is
d. IL-8 diagnosed with cystic fibrosis.Which
Ans: A Vit is deficient in this case?
179) Similarity between skeletal muscle a. Vit. A
and cardiac muscle: b. Vit. k
a. Gap Junctions c. Vit. D
b. Do not Contain T tubule d. Vit. C
c. Multi nucleated Ans: B
d. Involuntary Explanation:
e. Contains T tubule Usually in cystic fibrosis Vit. D is deficient
Ans: E because of gi disturbance related to pancreatic
180) The iron requirement in pregnant lady insufficiency but in this case patient is having
is: petechia which indicates her disease progressively
a. 600 involved liver as well.So here Vit. K is correct.
b. 1200 185) The most common side effect of
c. 1000 hyperlipidemic drugs is:
d. 1500 a. GIT Disturbance
Ans: C b. Hypotension
(800 wasn‘t in options) c. Tachycardia
Iron Requirement in Pregnancy=800mg/dl d. Urinary Retention
Folic Acid =1000mg/dl Ans: A
Calcium =1200mg/dl 186) A patient visits a doctor due to
181) An 80 years old male cardiac patient headache.His B.P. was
have sob and progressively developing 170/90mmHg.What are the cells
orthopnea and presented with pleural which are involved in increasing blood
effusion. Tap was done. Which factor pressure?
tells that it is transudative or a. Principal cells
exudative? b. Juxtaglomerular Cells
a. High protein c. Serous cells
b. Low cell count d. Follicular Cells
c. Cloudy appearance Ans: B
d. Massive amount 187) Side effects of atracurium:
e. Hemorrhagic a. Hypotension
Ans: B b. Hypertension
182) Posterior and parallel structure to c. Skin Cyanosis
carotid sheath is: d. Respiratory arrest
SK Origional – Golden 11 307

Ans: A d. CA-125
Explanation:Most adverse reactions were Ans: A
suggestive of histamine release which leads to 192) A female patient presented with
Hypotension and anaphylactic shock. Incisional hernia. Which layer comes
188) A patient is alcoholic and his liver biopsy out first after skin and sub cutenous
shows mallory bodies.What is it‘s fats?
composition: a. Cremesteric Fascia
a. Intermediate filament b. Transversalis Fascia
b. Desmin c. Internal onlique
c. Hyaline d. Rectus Abdominis
d. Colllagen fibers e. Conjoint tendon
Ans: A Ans: B
189) Diaphragmatic surface of the heart is 193) Primary oocyte surrounded by zone
formed by: pellucida, zona granulosa and fluid
a. Left ventricle filled cavity is called:
b. Right atrium a. Primary Follicle
c. Left ventricle b. Secondary Follicle
d. RV + SVC c. Tertiary Follicle
Ans: A d. Antral Follicle
190) An 11 years old boy with no CVS or Ans: B
respiratory ailment is having swelling
in front of the neck.He feels difficulty
in breathing while in the lying ===============
position. Diagnosis:
a. Multinodular Goiter
b. Follicular CA of thyroid
c. Asthma
d. Retrosternal Goiter
e. Colloid Goiter
Ans: D
191) A female is worried about a lump in
her breast. She also has family history
of Ca breast. What is main risk factor
in her?
a. BRCA-1
b. Smoking
c. HNPPC
SK Origional – Golden 11 308

ALL RADIOLOGY
September – November – 2020 Papers
SK Origional – Golden 11 309

RADIOLOGY Ans: A
th
September 29 , 2020 – Morning 7. Vertical Diplopia, proptosis, fracture
of orbit:
A. Floor
1. Grading of carcinoma shows:
A. Dysplasia B. Roof
C. Lateral
B. Permeability to basement membrane
D. Medial
C. Mitosis inside cell
Ans: B
D. Invasion
8. Transfusion related acute lung injury
E. Degree of differention
(TRALI) within?
Ans: E
A. Almost Immediately
Grading….Degree of differention
B. Within 4hrs
Staging…Distance to spread
2. Blood supply to capsule of kidney? C. Within 24hrs
D. Within 48hrs
A. Actuate
Ans: B
B. Renal
9. After trauma patient has rustling
C. Interlobar
sound and proptosis of eye:
D. Interlobular
A. Carotid cavernous fistula
Ans: D
B. Floor damage
Explanation:
C. Roof damage
 Renal Column: Interlobar
D. Retro optic Hemorrhage
 Renal Capsule: Inerlobular
Ans. C
3. Definitive clinically for malignancy:
10. New born baby was alright for 6hrs
A. Staging
after that she developed blue
B. Grading
discoloration of lips and hands.
C. Pleomorphism
Diagnosis?
Ans: A
A. Tetralogy of falot
4. In the last 7 month of pregnancy
B. VSD
estrogen and progesterone are
C. Transposition of great vessel with no
produced by:
shunt
A. Placenta
D. PDA
B. Corpus luteum
Ans: C
C. Hypothalamus
Explanation: Not compatible with life unless a
D. Anterior pituitary
shunt is present to allow mixing of blood. Most
E. Posterior pituitary
infants die within the 1st few months of life.
Ans: A
Ref. 1st AID
Ref. Guyton Physiology
11. 10-year-old boy developed shortness of
5. Patient oral surgery done and
breathing on exertion? Diagnosis:
mandibular nerve damage its place of
A. TOF
origin:
B. VSD
A. Under Incisor teeth
C. ASD
B. Under Premolar teeth
Ans: A
C. Under canine teeth a little below
Explanation: Most common cause of early
Ans: C
childhood cyanosis.
6. Anterior to posterior, optic chiasma
Ref. 1st AID
compressed by berry aneurysm artery
12. Intrahepatic protein anabolism and
involved:
extra hepatic catabolism:
A. ACA
A. Cortisol
B. PCA
B. Glucagon
C. Basilar Artery
C. Epinephrine
D. Ophthalmic
Ans: A
E. Internal carotid
SK Origional – Golden 11 310

13. During Meal liver maintain C. Lingual


gluconeogenesis by? D. Trigeminal Nerve
A. Glycolysis Ans: A
B. Glycogenolysis SVA - Chorda Tympani
C. Gluconeogenesis GSA-Lingual Nerve
Ans: B 21. Posterior triangle divided by:
14. Medial to inguinal triangle is: A. Ant. Belly of digastric
A. Adductor longus B. Sup. Belly of omohyoid
B. Rectus abdominis C. Inf. belly of omohyoid
C. Lateral Transverse muscle D. Post. Belly of digastric
Ans: B E. Ant. Border of sternocleidomastoid
15. After surgery patient developed Ans: C
swelling incisional hernia confirmed. Ref. clinical anatomy
It most likely comes from: 22. In fetus which part of bone is ossified?
A. Fascia transverse A. Medial malleolus
B. Internal oblique B. Medial epicondyle of femur
C. External oblique C. Lower end of femur
D. Transverses abdominis D. Lower end of tibia
Ans: A Ans: C
Ref. Clinical Anatomy Ref. B.D Chourasia
16. A patient presented to her doctor with 23. Erythropoiesis during 4th intrauterine
a small axillary mass the doctor months done in:
examined her and found a mass in A. Liver
upper outer quadrant of breast the B. Spleen
lymphatic drainage of that part is: C. Yolk Sac
A. Lateral Axillary LN D. Bone marrow
B. Central Axillary LN Ans: A
C. Anterior Axillary LN Young Liver Synthesis Blood
D. Posterior Axillary LN Young - Yolk Sac
Ans: C Liver - Liver
17. Drug excretion and metabolism is Synthesis - Spleen
effected on poor: Blood - Bone marrow
A. Liver function Ref. 1st AID
B. Kidney Function 24. Hypernatremia causes:
C. GIT function A. Muscle weakness
Ans: A B. Cell swelling
18. Ligamentum teres: C. Cell shrinkage
A. Remnant of left umbilical vein D. Decrease amplitude of action potential
B. Free margin of coronary ligament Ans: C
C. Medial to Ligamentum venosum 25. Common adverse effect of protamine:
Ans: A A. Hypertension
Ref. 1st AID B. Hypotension
19. External arcuate fibers form? C. Bleeding
A. Posterior spinocerbeller D. Anaphylaxis
B. Anterior spinocerbeller Ans: B
C. Cunocerebller In normal dose - Hypotension
D. Ponto cerebellar In toxic - Bleeding
Ans: C 26. Maximum cross sectional area of:
20. Loss of taste sensation of ant. 2/3 of A. Artery
tongue due to: B. Vein
A. Chorda tympani C. Venules
B. Facial D. Capillaries
SK Origional – Golden 11 311

Ans: D D. Synchondrosis
Ref. Ganong Physiology E. Fibrous
27. Notching of rib occurs in? Ans: B
A. Pre ductal coarctation of aorta 34. Vitamin A deficiency:
B. Post ductal coarctation of aorta A. Abnormal epithelial growth
C. Co-arctation of arc of aorta B. GIT disturbance
D. PDA Ans: A
E. VSD 35. Blood supply of urinary bladder
Ans: B through which ligament?
Ref. Clinical anatomy A. Puboparastatic
28. Fluid which increasers ECF then B. Ventrolateral
decrease ICF and overall increases C. Dorsolateral ligament
blood volume? D. Inguinal
A. 3 percent NaCl E. Suprapubic
B. 0.45 NaCl Ans: C
C. 5% dextrose 36. Specific gravity measurement of urine
D. Ringer Lactate is associated with:
Ans: A A. Volume
29. In primary dehydration ECF becomes: B. Frequency
A. Hypotonic C. Sepsis
B. Isotonic D. Concentration
C. Hypertonic E. Filtration
D. None Ans: D
Ans: C 37. Patient has 5-liter urine output which
30. First 2cm of duodenum which artery investigation?
supply? A. Water deprivation test
A. Right gastric Artery B. Electrolyte imbalance
B. Superior pancraticoduodenal Artery Ans: A
C. Inferior pancraticoduodenal Artery 38. Which of the following Artery
D. Celiac Artery damaged at pelvic brim?
E. Hepatic Artery A. Superior gluteal
Ans: B B. Internal Iliac
Explanation: Superior pancraticoduodenal artery C. Uterine
is a br. Of gastrodudenal artery. D. Inferior Gluteal
Ref. Clinical Anatomy E. Ovarian
31. Insoluble form of iron stored is: Ans: E
A. Hemosiderin. 39. Superficial inguinal LN:
B. Ferritin A. Lower body of uterus
C. Transferrin B. Cervix
D. Hemoglobin C. Vagina above hymen
Ans: A D. Ureter
32. Diaphragmatic parietal pleura is E. Vagina below hymen
supplied by: Ans: E
A. Intercostal Nerve Explanation: The vaginal orifice in a virgin
B. Phrenic and Intercostal nerves possesses a thin mucosal fold called the Hymen.
C. Phrenic nerve The hymen usually consists of tags.
Ans: C Upper1/3rd drain into ext. and int. iliac node
33. Manubriosternal joint is: Middle1/3rd drain into the internal iliac node
A Primary Cartilaginous joint Lower1/3rd into the superficial inguinal node
B. Sec. cartilaginous joint Ref. Clinical Anatomy
C. Primary cartilaginous joint between 40. Nerve damaged in lateral pelvic wall:
epiphysis and Diaphysis A. Obturator nerve
SK Origional – Golden 11 312

B. Pedundal nerve doctor advised her to get into left


C. Ilioinguinal nerve lateral position, and this position will
Ans: A increase blood flow to brain,Doctor
Explanation: The lateral pelvic wall has a large advised this position to avoid
hole, The Obturator Foramen, Which is closed by compression of which abdominal
the Obturator membrane. , except structure?
for a small opening that permits the Obturator A. Spleen
nerve to leave the pelvis and enter the thigh. B. Liver
Ref. Clinical Anatomy C. SVC
41. Structure passing through Foramen D. Lungs
rotundum? E. IVC
A. Mandibular nerve Ans: E
B. Accessary nerve 47. Patient having right sided weakness
C. Ophthalmic nerve and loss of vibration sense and tongue
D. Maxillary nerve deviated to left lesion was?
E. Occulomotor nerve A. Middle of medulla
Ans: D B. Lateral of medulla
42. Vesicle below clavicle and above C. Superior to medulla
sternum? D. Middle of midbrain
A. C2 – C3 Ans: A
B. C3 – C4 48. Root value of femoral and Obturator
C. C1 – C2 nerve?
D. C1 – C3 A. L2 – L3
Ans: B B. S1 – S2
43. Most important cause of male C. T2 – T6
infertility: D. LI – L2
A. Kline filter syndrome E. L2 – L3 – L4
B. Cryptorchidism Ans: E
C. Down syndrome Ref. 1st ID
D. Malunion of ejaculatory duct 49. Pattern of infective endocarditis:
E. Edwards A. Damage valve - bacteremia- perforation-
Ans: B thrombus
Ref. Robbins pathology B. Damage valve - thrombus - bacteremia-
44. Tall infertile man with normal external perforation.
genitalia married but had no kids. He C. Damage valve- bacteremia-thrombus
had normal ejaculatory period but formation-perforation
semen analysis shows azoospermia D. Damage valve- perforation- thrombus
cause may be in: formation-perforation-bacteremia
A. Epididymis Ans: B (DTBP)
B. Scrotum 50. Damage to later thigh lateral popliteal
C. Sertoli cells nerve and eversion lost:
D. Seminiferous tubules A. L2
Ans: C B. L3
Ref. 1st AID C. Sciatic
45. Turner syndrome Karyotyping: D. L4
A. 45x0 Ans: D
B. 46X0 51. Muscle which keeps the scapula
C. 47X0 opposite to chest wall?
D. 44X0 A. Rhomboid major
Ans: A B. Serratous Anterior
46. A Pregnant lady with 3rd trimester C. Levator scapulae
experienced an episode of fainting, D. Deltoid
SK Origional – Golden 11 313

Ans: B C. Preganglionic somatic


52. Metabolic acidosis: Ans: B
A. Decrease HCO3 60. Treatment of cystic acne in pregnant
B. Decrease H+ female that lead to fetal abnormalities?
C. Increase PH A. Vit.A
D. Decrease CO2 B. Thilamide
Ans: A C. Metronidazole
53. Causative organism of pneumonia in D. Vit.D
alcoholic patients? Ans: A
A. Mycoplasma Ref. 1st AID
B. Pneumocystis carinii 61. Ligamentum arteriosum recurring
C. Klebsella nerve?
D. Pseudomonas A. Left recurrent laryngeal nerve
Ans: C B. Right recurrent laryngeal nerve
54. Patient with normal ADH and glucose C. Right vagus nerve
level but with 5 lit urine., Urine D. Phrenic nerve
specific gravity low: Ans: A
A. Central D.I Ref. 1st AID
B. Nephrogenic D.I 62. Facial muscle nerve supply:
C. DM 1 A. Facial
D. DM 2 B. Mandibular
Ans: B C. Facial and mandibular
Ref. Ganong Physiology D. Buccal nerve
55. Patient in emergency room presented Ans: C
with high grade fever in unconscious 63. Platelet formation:
condition. Most likely Diagnose is? A. ADP and thromboxane A2
A. Septic shock B. Fibrinogen and thromboxane 2
B. Hypovolemic shock Ans: B
C. Cardiogenic shock 64. Upper eye lid fat deposits in:
D. Anaphylactic shock A. Xanthelasma
Ans: A B. DM
56. Radial artery lies between which 2 C. Hypothyroidism
tendons? D. Hyperlipidemia
Ans: Brachioradialis and Flexor carpi radials Ans: A
57. Pregnant woman came to emergency 65. Laryngotracheal groove develop in
after RTA and died within which week?
6hours.Cause? A. 4th Week
A. Septic embolism B. 5th Week
B. Fat embolism C. 6th Week
C. Air embolism D. 3rd Week
D. Amniotic embolism Ans: A
Ans: D 66. Pectoral major flap blood supply:
Explanation: Fat Embolism occur after 12hrs A. Inf. thyroid
58. Acidic duodenum cause: B. Highest intercostal
A. Decrease H2CO3 C. Thoracoacromal
B. Decrease gastric empting Ans: C
C. Increase gastric emptying 67. Direct branch of subclavian artery
Ans: B taking part in shoulder joint
59. Cardiac muscles have sympathetic anastomosis:
and parasympathetic these fibers are? A. Suprascapular artery
A. Preganglionic autonomic B. Thyrocervical trunk
B. Post ganglionic C. Transverse cervical artery
SK Origional – Golden 11 314

D. Thoracoacromal artery A. 9th ICS lower border mid axillary line


E. Dorsal scapular artery B. 9th ICS upper border mid axillary line
Ans: E C. Lower border of 4th IC mid clavicular line
Ref. B. D Chourasia D. Lower border of 5th IC mid clavicular line
68. An uncovered Person is sitting at Ans: A
room temperature. Mechanism of 76. Petrous part of temporal damage
room temperature heat lost will be: nerve:
A. Conduction A. Forms base of middle cranial fossa
B. Sweating B. Medial to Apex
C. Convection C. Anterior
D. Radiation D. Posterior
E. Evaporation E. Known as normal basalis
Ans: D Ans: A
69. Appendix lymphatic drainage: 77. Structure Anterior to IVC:
A. SMA A. Rt. Renal vein
B. IMA B. Lft. renal vein
C. Celiac artery C. Rt. Renal artery
D. Sup. Pancraticoduodenal artery D. Lt. renal artery
Ans: A E. Rt. testicular artery
70. Categorical data? Ans: E
A. Statistical analysis Ant. To IVC….Rt testicular artery
B. Chi sq. test. Ant. To Aorta…Left renal vein
C. Frequency of event 78. Proptosis diplopia vertical lesion:
Ans: B A. Roof of orbit
71. Mean media mode same in: B. Floor of orbit
A. Normal distribution curve C. Lateral
B. Confidence Interval D. Medial
C. Standard deviation Ans: A
D. Frequency polygon 79. In portal hypertension dilatation of
Ans: A which vein occur?
72. Dual nerve supply to which muscle? A. Inferior epigastric
A. Biceps femoris B. Left colic
B. Quadratus femoris C. Left ovarian
C. Adductor Magnus Ans: B
Ans: C 80. Patient with high grade fever yellow
B.D Chourasia sputum no chest symptoms?
73. Growth hormone needs which A. Klebsella
hormone for its function? B. Mycoplasma
A. Insulin C. Pseudomonas
B. Epinephrine D. TB
C. Glucagon Ans: A
D. Insulin like growth factor 81. Lady accidently cut radial artery
Ans: D which mechanism detects pressure?
74. Common site of ureteric stone A.A alpha
obstruction? B.A beta
A. At level of ureter cross common iliac C.A delta
artery D.C fibers
B. Pelvic brim Ans: B
C. Superior to ischial spine Explanation: A Alpha detects proprioception,
D. Nearest to kidney somatic motor
Ans: B>>A A Beta Detects touch and pressure
75. Pleurocentasis: C fibers detects pain and temperature
SK Origional – Golden 11 315

Ref. Ganong Physiology C. 60mints


82. Hypertrophic cardiomyopathy D. 20mints
scenario: Ans: B
A. Decrease diastole filling Half-life multiplies by 4 or 5.
B. Increase diastole filling For example 10*4=40
C. Normal filling 89. Soldier coming from siachin having
D. Increase systole filling breathlessness and cyanosis cause is?
E. Increase diastole filling A. Heart Failure
Ans: A B. Polycythemia Vera
83. A girl with primary amenorrhea. On C. Methemoglobolinimia
examination breasts not developed D. Secondary polycythemia
and uterus absent on USG. Karyotype Ans: D
was asked: 90. Irreversible sign of cardiac cell Injury?
A. 46XY A. Cell Swelling
B. 46XX B. Contraction bands in cytoplasm
C. 47XXY C. Hydropic change
D. 46X0 D. Clumping of nuclear chromatin
Ans: A Ans: B
84. Which structure lies ant to pancreas? 91. Barr body diagnostic for:
A. CBD A. Down syndrome
B. SMA B. Turner syndrome
C. Pylorus c. Kline filter syndrome
D. Lesser Sac Ans: B
E. 1st part of Duodenum 92. Patient has test come positive for
Ans: B infectious mononucleosis which organ
85. Ovarian artery is branch of: will be susceptible to injury?
A. Femoral A. Liver
B. Celiac B. Kidney
C. Abdominal aorta C. Spleen
D. SMA D. Lung
Ans: C Ans: C
B.D Chourasia 93. Apex of heart is supplied by:
86. Virus causes cell damage by which A. Diagonal
mechanism? B. Marginal
A. Oncogenes C. LAD
B. Proto-oncogene D. Anterior interventricular
C. Nuclear damage Ans: C
D. Altering the formation of proteins 94. Phrenic nerve root value:
Ans: D A. C3 to C5
87. Newly wed lady having pain in RIF B. C2 to C4
with fever and vomiting and raised C. C1 to C3
WBCs. Her LMP was 3weeks back D. C4 to C6
which structure affected? Ans: A
A. Ovary Ref. Clinical Anatomy
B. Uterus 95. Stab injury at 6th costal cartilage right
C. Appendix to sternum will pierce:
D. Kidney A. LA
Ans: C B. RA
88. A drug having half-life 10min when C. SVC
will it achieve steady state? D. IVC
A. 30mins Ans: B
B. 40mints
SK Origional – Golden 11 316

96. Dilatation of which of the following B. Hepatitis B


structure causes dysphagia by C. Hepatitis D
compressing on the esophagus? D. Hepatitis C
A. LA Ans: A
B. Aortic arch Explanation: Most common in travellers and
C. Left bronchus daycare centers.
D. Trachea Ref. 1st AID
Ans: A 105. Inferior alveolar nerve damaged
Ref. 1st AID during left check accident where
97. Regarding Breast: injury?
A. 15 to 20 lactiferous ducts Ans: Lower jaw fracture
B. Extends from 2 to 6 intercostal spaces 106. Cleft lip and palate:
C. Tuboalveolar gland Ans: Medial nasal and maxillary
D. Does not arise from place other than 107. Patient having right sided weakness
thorax. and loss of vibration sense and tongue
Ans: A deviated to left side. Lesion was:
Ref. Clinical Anatomy A. Anterior spinal artery
98. Renal pyramid is supplied by: B. Internal carotid artery
A. Interlobar artery C. Basilar Artery
B. Interlobular artery D. PICA
C. Inferior phrenic artery Ans: A
Ans: A Explanation:Medial medullary syndrome
Ref. Clinical Anatomy Contralateral paralysis---upper and lower limbs
99. Neurovascular bundles: Dec. Contralateral proprioception
A. Upper border of rib Epsilateral hypoglossal dysfunction (Tongue
B. Lower border of rib deviate to epsilaterally).
C. Lower end of intercostal space Ref. 1st AID
Ans: B 108. A Patient has fracture of surgical neck
Ref. B.D Chourasia of Humerous. Now he can‟t raise his
100. CN4 palsy...what is affected? arm above his shoulder and also has
Ans: Nystegmous of abducting eye sensory loss on lateral surface of arm.
101. Posterior arcuate fibers form Which of the following muscle
Ans: Ceunocerebellar tract damaged?
102. In adults, Injury to neck of femur, A. Teres minor
Which artery will be damaged? B. Teres major
A. Medial Circumflex C. Deltoid
B. Lateral circumflex D. Pectoral major
C. Retinacular artery Ans: C
D. Posterior circumflex Artery Explanation: Deltoid muscle Abducts and
Ans: C laterally rotate arm. So, fracture of humerus of
Retinacular >>Media cxl>>>Lateral cx artery humerus at surgical neck, damage Axillary nerve
103. X-ray of middle aged man smoker that paralyses the deltoid muscle.
working in Construction factory shows Ref. Clinical Anatomy
lower lobe plaques. He is most likely 109. In diabetic ketoacidosis…Acidosis is
suffering from: caused by:
A. Asbestosis A. Increased anion gap
B. Berryliosis B. Metabolic acidosis
C. Silicosis C. No insulin
D. Anthrocosis D. Ketone bodies
Ans: A Ans: A
104. Feco-oral route: 110. Which cell line seen in type 1
A. Hepatitis A hypersensitivity?
SK Origional – Golden 11 317

A. Neutrophils‘ C. Posterior part of LT lower lobe


B. Eosinophil‘s Ans: A
C. Mast cells Ref. 1st AID
D. Monocytes 119. Terminal portion of CBD in relation
E. Lymphocytes to pancreas:
Ans: C A. Lies posteriorly
Ref. Robbins pathology B. Lies anteriorly
111. Sudden lumber region pain. What is C. Imbedded in pancreas
diagnostic approach? D. Lies laterally
A. USG KUB E. Medial to the head of pancreas
B. X-ray KUB Ans: C
Ans: A 120. Vertebrae C1, C2:
112. Prostatic carcinoma spread to skull Ans: Atlantoaxial joint
through? 121. Breast upper outer quadrant lymph
A. Lymphatic drainage accompany which vessel?
B. Blood vessels A. Lateral thoracic
C. Vertebral venous plexus B. Long thoracic
Ans: C C. Anterior axillary
Ref. Clinical Anatomy D. Internal thoracic
113. Patient with backache after lifting Ans: C
heavy radiating to back thigh and 122. Clarke‟s column nerve root value?
lateral foot weekend ankle jerk and A. T1 –L2
flexor hallucis longus, pain on raising B. T6-T9
leg till 30, which structure C. S1 S2
compressed? D. T2 –T5
A. Sciatic nerve E. L3 –L5
B. S1 nerve root Ans: A
C. L4 123. Hypercalcemia and renal failure.
D. L3 Cause?
Ans: B A. PTH adenoma
114. Shoulder joint dislocation…. ligament B. Sarcodosis
injury? C. Hypervitaminosis D
A. Subscapular D. Metastatic
B. Acromioclavicular Ans: A
C. Coracoclavicular 124. Wadding gait nerve damage:
Ans: C A.Sup. Gluteal
115. Pheochromocytoma: B.Inf. Gluteal
Ans: VMA C. Obturator nerve
116. Portal vein to liver run in which D. Femoral nerve
ligament? Ans: A
A. Ligamentum teres 125. Collagen type in healthy skin:
B. Hepatoduodenal A. Type 2and 3
Ans: B B. Type 3 and 5
117. AIDS patient what will be early C. Type 1and 3
change in peripheral blood smear? D. Type 1 and 2
A. Decrease T.cells Ans: C
B. Follicular hyperplasia 126. Type 3 hypersensitivity reactions:
Ans: A Ans: SLE
118. Supine position aspirate content goes: 127. A patient with massive spleen. Spleen
A. Sup. Bronchopulmonary segment of Rt was palpable below 10th ICS:
lower lobe A. Mylofibrosis
B. Posterior segment of RT upper lobe B. Thalassemia
SK Origional – Golden 11 318

Ans: A Explanation: Because the long thoracic nerve


128. Most common Site of fertilization in passes down the lateral thoracic wall on the
humans? external surface of the Serratous anterior muscle,
A. Ovary just deep to skin and subcutaneous fascia, It is
B. Uterus vulnerable to damage. Loss of function of this
C. Cervix muscle causes the inferior angle of scapula to
D. Ampulla of fallopian tube elevate from the thoracic wall, resulting in
E. Peritoneal cavity characteristic ―WINGING‖ of scapula.
Ans: D Ref. Gray‟s Anatomy
129. Sarcomere between: 3) Which of following is major diagnostic
A. A band tumor marker for colon carcinoma?
B. I Bands A) CEA
C. Z lines B) AFP
D. H zones C) HCG
Ans: C D) CA125
130. A patient with alcoholic hepatitis with Ans: A
Mallory bodies. Interwoven structure? Explanation
A. Intermediate filaments AFP…for Hepatocellular carcinoma
B. Beta Amyloid HCG… for Seminoma
C. Alpha syncline CA125…For Ovarian Carcinoma
Ans: A Ref. 1st AID
131. Direct inguinal hernia lies: 4) Which of the following muscle causes
A. Medial to inferior epigastric vessels abduction of arm above 90?
B. Lateral to inferior epigastric vessels A) Trapezious & Serratus Anterior
Ans: A B) Deltoid
C) Supraspinatus
D) Infraspinatus
=============== E) SCM
Ans: A
Explanation: Supraspinatus causes initian of
RADIOLOGY abduction to 15 degree.
Deltoid…90 degree
September 29th, 2020 – Afternoon Ref. B.D Chourasia
1) A post-operative patient presented in 5) What is anterior relation of Hilum of
OPD with oozing greenish discharge Right Kidney?
from Wound. Which of following is A) Liver
causative agent? B) Spleen
A) Bacteriods C) Large intestine
B) E.coli D) 2nd part of Duodenum
C) Lesteria Ans: D
D) Pseudomonas Explanation: A large part of upper part of the
E) Klebseila anterior surface is against the Liver and is
Ans: D separated from it by a layer of peritoneum.
2) After mastectomy for breast Medially, , The descending( 2nd ) part of the
carcinoma female developed winging of duodenum is retroperitoneal and contacts the
scapula, which is due to damage to? Kidney.
A) Axillary nerve damage Ref. Gray‟s Anatomy
B) Long thoracic nerve damage 6) Regarding Hodgkin lymphoma True
C) Radial nerve statement is:
D) Long head of A) Absent RS cells
Ans: B B) Contiguous spread
SK Origional – Golden 11 319

C) Common in old age in biopsy specimen. The biopsies


D) Extra nodal involvement is common were perform patients who had
Ans: B palpable mass lesions on digital
7) Female pelvis is tilted in relation of examination. Of the following
spine? microscopic finding is not an
A) Upward indicator of malignancy?
B) Downward A) Pleomorphism
C) Forward B) Atypia
D) Backward C) Encapsulated
E) Medially D) Increased N/C ratio.
Ans: C E) Necrosis
8) A patient is suffering from 2nd degree Ans: C
heart Block? Which of following is 12) 65-year-old man is suffering from
true? pneumonia and unable to recognize
A) The ventricular ECG complexes are distorted. faces and died. Which of following
B)Ventricular rate is lower than atrial rate Bug is responsible?
C) There is high incidence of ventricular A) Staphylococcus aureus
tachycardia B) Hemophelius
D) Stroke volume is decreased. C) Streptococcus P
E) Cardiac output increased. D) Klebseila
Ans: B E) Pseudomonas
Explanation: In 2nd degree heart block, not all Ans: C.
atrial impulses are conducted to the ventricles. For Explanation: Causes of pneumonia with age
example, a ventricular beat may follow every 1st day to 6m...Group B streptococcus(
second or every third atrial beat (2:1 or 3:1). Aglactassai)
Ref. Ganong Physiology 7m to teen age…Hemophilus influenza
9) Opening of Esophagus in diaphragm Old age…Streptococcus pneumonae
at which level? Ref. 1st AID
A) T8 13) Loss of water by evaporation and
B) T10 insensible loss from body:
C) T12 A) Controlled by hypothalamus
D) C6 B) Thermal gradient
E) L1 C) Remains constant
Ans: B D) Depends on core body temperature
Explanation: Formula for remembering of Ans: D
opening of Diaphragm. 14) First pass metabolism of drug can be
VEA. avoided if given via:
V…Inferior vena cava at T8 A) Per rectal
E…Esophageal at T10 B) Per oral
A… Aortic at T12 C) IV
Ref. Gray‟s Anatomy D) IM
10) Which of the following muscle of E) Sublingual
mastication helps in deep inspiration Ans: C
during yawning and increases CO? 15) Which of the following Structure is
A) Medial pterygoid between celiac plexus and SMA?
B) Lateral pterygoid A) Stomach
C) Masseter B) Spleen
D) Supraspinatus C) Gall bladder
E) Buccinators D) Liver
Ans: B E) Pancreas
11) A study is performed to analyze Ans: E
characteristic of malignant neoplasm
SK Origional – Golden 11 320

16) Structures passing through posterior Ans: D


hiatus(Aortic Opening) of diaphragm? Ref: B.D Chourasia
A) Aorta & Thoracic duct 22) Biochemical change in wilsons
B) Esophagus disease?
C) Trachea A) Ceruloplasmin
D) Inferior thyroid artery B) Copper in liver brain cornea kidneys
E) IVC C) Urinary Copper
Ans: A
D) Ceruloplasmin
Explanation: ATA…Aorta, Thoracic Duct,
Ans: A
Azygous vein.
23) Slow writhing movements are called
17) Fracture of acetabulum
athetosis due to defect in which of
posteroalterally which bones
following?
fractured?
A) Globus pallidus
A) Ischium & Ilium
B) Caudate
B) Pubis
C) Subthalamus
C) Sacrum
D) Thalamus
D) Pubic symphysis
E) Cerebellum
Ans: A
Ans: A
Ref, Clinical Anatomy
Ref. ?
18) Left gastric artery is the branch of
24) Potts fracture (Ankle joint fracture)
which of following?
occurs due to which of following
A) SMA
excessive movement?
B) IMA
A) Inversion
C) Celiac trunk
B) Eversion
D) Pancreatic magna
C) Dorsiflexion
E) Right Gastric
D) Abduction
Ans: C
E) Lateral rotation
Explanation: It is smallest Br. Of celiac trunk
Ans: B
Ref. Gray‟s Anatomy
Ref. B.D Chourasia
19) Left Gastric vein drains into:
25) Right gastroepiploic artery is branch
A) Portal vein
of?
B) Splenic vein
A) Gastroduodenal
C) Superior mesenteric
B) Splenic
D) Inferior mesenteric
C) Gastric
E) Aorta
D) Right hepatic
Ans: A
Ans: A
Ref. B.D Chourasia
26) Which structure lies anterior to
20) Beri-Beri is caused by which of
inferior vena cava?
following vitamin deficiency?
A) Portal Vein
A) B1
B) Right gonadal vessel
B) B2
C) Renal artery
C) B6
D) Inferior phrenic artery
D) B12
E) Sacral artery
E) B9
Ans: B
Ans: A
27) Which of following is intra-articular
Ref. 1st AID
tendon?
21) Testicular tumor drains into which of
A) Anconeus
following group of lymph nodes?
B) Popliteus
A) Iliac lymph node
C) Sartorius
B) Popliteal lymph nodes
D) Semitendinosus
C) Inguinal lymph nodes
E) Semimembranosus
D) Para aortic/Lumbar
SK Origional – Golden 11 321

Ans: B B) Latissmus dorsi


28) What is Clarke's column nerve root C) Serratus Anterior
value? D) SCM
A) T1-L2 E) Diaphragm
B) S1-S2 Ans: C
C) T2-T5 Ref. Gray‟s Anatomy
D) T6-T9 35) Major lymphatic drainage of Breast
E) L3-L5 follow which vessels?
Ans: A A) Axillary
29) Sarcomere present between: B) Internal Thoracic
A) Two Z lines C) Supraclavicular
B) Two M lines D) Inferior phrenic
C) Two H zones Ans: A
D) A & I band 36) Medial palpebral ligament (Medial
Ans: A canthal ligament) attachment?
Ref. BRS physiology A) Anterior & Posterior lacrimal ridges
30) Intercostal-brachial artery originates B) Frontonasal
from which of following? C) Anterior lacrimal ridge
A) Arch of aorta D) Posterior lacrimal ridge
B) Anterior of descending aorta Ans: A
C) Posterior lateral portion of descending 37) Basal ganglia component include:
aorta A) putamen + Cerebrum
D) Ascending Aorta B) Hypothalamus + cerebellum
Ans: C C) Substantia nigra + Globus pallidus + sub
31) Septum pellucidum is bounded thalamic nucleus
superiorly by? D) Globus pallidus + cerebrum
A) Body of carpus callosum Ans: C
B) Rostrum of carpus callosum Ref.1st AID
C) Fornix 38) Regarding Atherosclerotic plague
D) Genu which of following is true?
Ans: A Ans: Necrotic lipid core with fibromuscular
Ref. Neurosnell Anatomy cap
32) Movement at radio-ulnar joint by 39) ADH is increased in dehydration Max
which of following muscles? water absorption occurs in presence of
A) Pronator & supinator ADH is:
B) Flexor carpi ulnaris A) Collecting Ducts
C) Extensor carpi ulnaris B) DCT
D) Biceps brachii C) PCT
Ans: A D) LOH
33) Xiphoid process is present at which E) DCT + CD
vertebral level? Ans: C
A) T7 Ref. Ganong Physiology
B) T8 40) Costo-diaphragmatic recess is present
C) T9 between?
D) T10 A) Costal and diaphragmatic pleura
E) T11 B) Costal & visceral pleura
Ans: C C) Costal & Diaphragm
34) During Mastectomy Long thoracic D) Diaphragm & Ribs
nerve is damaged resulting in Ans: A
weakness of which of following 41) Circumflex artery supplies which of
muscle? following?
A) Trapezious A) Right Atrium
SK Origional – Golden 11 322

B) Right Ventricle B) Viral meningitis


C) Left ventricle and anterolateral papillary C) Bacterial meningitis
muscle D) Fungal meningitis
D) Anterior surface of heart E) Brain tumor
Ans: C Ans: A
Ref. 1st AID 48) A patient is suffering from fever &
42) Macrophages in lungs are termed as: chest pain. He is producing Greenish
A) Alveolar dust cells sputum:
B) Langerhans cells A) Mycobacterium tuberculosis
C) Histocytes B) Pseudomonas
D) Kupffer cells C) Klebseila
E) Microglia D) Streptococcus pneumonia
Ans: A E) Ligenella
Ref: Wheatear‟s Histology Ans: B
43) A patient is known case of Asthma. 49) Regarding Bladder which of following
He is suffering from shortness of is true?
Breath. Increased Airway constriction A) Anterior surface covered by peritoneum
is due to: B) Posterior-inferior surface is covered are
A) Parasympathetic peritoneum
B) Sympathetic C) Anterior-inferior surface is devoid of
C) Sympathetic adrenergic peritoneum
D) Parasympathetic & sympathetic D) Only supply by sympathetic
Ans: A E) Only supply by parasympathetic
44) Which of following is feature of Ans: C
Benign Tumor? 50) Which of the following organism
A) Invasion causes Lobar pneumonia?
B) Pleomorphism A) Klebseila
C) Irregular margin B) Ligenella
D) Encapsulated C) Staphylococcus aureus
Ans: D D) Pseudomonas
45) A patient is diagnosed case of COPD E) Streptococcus
(Chronic bronchitis) diagnostic Ans: E
feature histologically is: 51) Which of following shares a largest
A) Hypertrophy and hyperplasia of mucus contribution to root value of Phrenic
secreting glands Nerve?
B) Atrophy of mucus secreting gland A) C2
C) Hypertrophy of serous glands B) C3
D) Atrophy of serous glands C) C4
Ans: A D) C5
46) Omphalocele is associated with which E) C1
of following cardiac anomaly? Ans: C
A) Transportation of vessel 52) Left gastric artery is branch of?
B) Coartation of Aorta A) Celiac trunk
C) VSD B) Gastroduodenal
D) ASD C) Common Hepatic
E) TOF D) Pancreatic Magna
Ans: E E) Pancreaticoduodenal
47) A child with disturbed behavior Ans: A(Repeated)
suffering from Fever and Convergent 53) What is most common location of
squint. Which of following is most ureter damage in oophorectomy?
likely diagnosis? A) Broad ligament
A) Tuberculosis meningitis B) Cardinal ligament
SK Origional – Golden 11 323

C) Pelvic brim Ref. B.D Chourasia


D) Suspensory ligament 60) Herring bruer reflex prevent which of
Ans: C following?
54) Which of following structure is A) Collapse of lung
anterior to pancreases? B) Deflation
A) Right renal artery C) Overinflation
B) Right renal vein D) None
C) Left renal artery Ans: C
D) SMA & SMV Ref. Guyton Physiology
E) IMA 61) Dorsum of foot is supply by which of
Ans: D following nerve?
Ref. Gray‟s Anatomy A) Sural
55) Right gastric artery branch of which of B) Saphenous
following? C) Tibial
A) Proper hepatic artery D) Superficial peroneal
B) Splenic artery Ans: D
C) Gastroduodenal 62) Delayed gastric emptying is which of
D) SMA following?
E) IMA A) Vagotomy
Ans: A B) Secretin
Ref: Gray‟s Anatomy C) CCK
56) Numbness of ear lobule while doing D) Motilin
internal jugular lymph node biopsy? Ans: C
Which of following nerve is damaged? Ref.1st AID
A) Posterior auricular 63) A patient is presented in Emergency
B) Accessory following RTA. Lateral cord of
C) Vagus brachial plexus is damaged. Which of
D) Greater Auricle following muscle will be affected?
Ans: D A) Pec. Major
57) Infection from face can travel to: B) Latissimus
A) Cavernous sinus C) Serratus
B) Sagittal sinus D) Deltoid
C) Sigmoid sinus E) Diaphragm
D) Bone of skull Ans: A
Ans: A 64) Which of following is supply of
Infection can travel through superior ophthalmic adrenal gland?
vein to cavernous sinus. A) Least thoracic
58) Deltoid ligament is damaged due to B) Lesser thoracic
which of following excessive C) Internal thoracic
movement? D) Greater thoracic splanchnic
A) Eversion Ans: D
B) Inversion 65) Antenatal checkup for down syndrome
C) Dorsiflexion at 11th week of Gestation?
D) Plantarflexion A) CVS
Ans: A B) USG
59) Patient with difficulty in breathing are C) MRI
comfortable in? D) Amniocentesis
A) Sitting up Ans: A
B) Standing 66) Mullerian duct develops into?
C) Supine A) Uterus & Vagina
D) Prone B) Penis & Testis
Ans: A C) Bladder
SK Origional – Golden 11 324

D) Prostate 74) Regarding Bile duct which of


Ans: A following is true?
67) Which of the following is consequence A) Posterior portal vein
of polycythemia Vera? B) Right to hepatic artery
A) Myelodysplastic syndrome C) Anterior to pancreas
B) Iron deficiency D) Posterior to duodenum
C) Megaloblastic Ans: B
D) Sickle cell 75) A patient is suffering from Severe
Ans: A Diarrhea K+ infusion given. K+ go
68) Golgi Tendon Organ detects tension into cell through which channel?
in which of following? A) Na-k pump
A) Agonists B) K-ATPase
B) Antagonist C) H-pump
C) Synergistic D) Ca- pump
D) Agonist & Antagonist Ans: A
Ans: A 76) Conduction in nerve fibers is slow
69) Clara cells are abundant in which of down due to?
following? A) Myelinated
A) Terminal Bronchiole B) Non-myelinated
B) Trachea C) Short fiber
C) Bronchi D) Membrane resistance
D) Larynx E) Increase diameter
Ans: A Ans: B
70) Gold standard test for detection of low 77) Nissels granules are present in?
lying plecenta? A) SER
A) Per vaginal Examination B) RER
B) Per vaginal USG C) Mitochondria
C) Doppler USG D) Golgi apparatus
D) Pelvic MRI E) Lysosomes
Ans: B Ans: B
71) Microtia is due to defect in: 78) Primary oocyte remain dormant until
A) 1stand 2ndBrachial groove puberty at which phase?
B) 3rdand 4thbrachial groove A) Diplotene stage of prophase of meiosis 1
C) 2ndand 3rdbrachial groove B) Metaphase
D) 1stand 4thbrachial groove C) Anaphase
Ans: A D) Telophase
72) What is amount of interstial fluid Ans: A
present in 70 kg man? 79) In omentum which type of necrosis
A) 5L occurs?
B) 6L A) Liquefactive
C) 8L B) Fat
D) 11.5 L C) Coagulative
E) 16 L D) None
Ans: D Ans: B
Ref. Guyton physiology 80) Brocas area present in:
73) Which helps in regulation of A) Superior frontal gyrus
extracellular potassium? B) Inferior frontal gyrus
A) Insulin C) Superior temporal gyrus
B) Glucagon D) Inferior temporal gyrus
C) Cortisol Ans: B
D) Epinephrine 81) 70yr old man with urinary retention
Ans: A comes to emergency and has urgency
SK Origional – Golden 11 325

frequency since 1 month PSA is 87) 4thcranial nerve palsy causes difficulty
significantly raisedmoderately in?
enlarged prostate palpable on PR A) Downward and adduction
examination lobe involved is: B) Downward and abduction
A) Lateral lobe C) Upward
B) Middle lobe D) Downward
C) Posterior lobe Ans: B
D) Anterior lobe 88) A patient is suffering from some
Ans: C bleeding disorder. His PT is normal
Explanation: Prostatic carcinoma involve but APTT is raised. Defect is in which
posterior lobe. of following pathway?
BPH Involve meddle lobe A) Extrinsic
82) A patient is known case of CLD with B) Intrinsic
caput medusae formation which of C) Extrinsic + Intrinsic
following veins will involve? D) None
A) Para umbilical veins Ans: B
B) Gastric vein Explanation: Features are suggestive of
C) Esophageal veins Hemophilia that is a defect in intrinsic pathway.
D) Splenic vein Ref. 1st AID
Ans: A 89) Terminal ileum resection?
83) Muscle having same nerve supply as A) Inc. water content in feaces
mylohyoid: B) Dec fat content
A) Ant belly of digastric C) Inc. bile salt absorption
B) Posterior belly of digastric D) Dec. water content in faced
C) Latissimus Ans: A
D) SCM Ref. BRS physiology
Ans: A 90) During Hysterectomy doctor should
84) Which of following structure is present know the relation of ureter along?
between stomach and pancreas? A) At pelvic brim
A) Lesser sac B) Base of broad ligament
B) Greater sac C) Internal iliac artery
C) Small intestine D) Infundibulapelvic ligament
D) Spleen Ans: B
Ans: A Ref. Clinical Anatomy
85) A 45-year male presented in 91) Ulceration of posterior wall of
emergency following RTA with Duodenum involves which of
Fracture of neck of femur operated following artery?
next day died: A) Gastroduodenal
A) Fat embolism B) Splenic
B) Air embolism C) Gastric
C) Thromboembolism D) Pancreaticoduodenal
D) Amniotic embolism Ans: A
Ans: C Ref. Gray‟s Anatomy
Explanation: Fat embolism develops after 48hrs. 92) Conductive Aphasia which of
86) Pyloric ulcer rupture abscess initial following artery is involved?
goes to: A) ACA
A) Lesser sac B) PCA
B) Right sub hepatic C) MCA
C) Left sub hepatic D) ACA + PCA
D) Greater sac Ans: C
Ans: A 93) Ca pancreas patient is admitted in
ward his brother is not letting you tell
SK Origional – Golden 11 326

the patient that the patient has cancer 99) Action potential depolarization in
because according to his brother it ventricular muscle is caused by:
would kill the patient even faster at A) Na influx
the evening the patient's family is B) Ca influx
going to come to hospital to discuss C) Increased k conductance
and we'll have a conference together D) Decrease k conductance
what will you do? Ans: A
A) Give family time to discuss among 100) The most likely complication in the
themselves mouth due to chewing peanut and pan
B) Tell his brother that it will be dishonesty would be?
with patient family and medical staff A) Sub mucus fibrosis
C) Will ask him to mind his own business B) Leukoplakia
Ans: B C) Erythroplakia
94) A patient is known case of CKD died D) Lichen planus
and on autopsy found hypertrophy of Ans: A
which of following glands? 101) Which of the following systemic factor
A) Parathyroid is responsible for poor wound healing?
B) Thyroid A) Decrease Blood supply
C) Pituitary B) Decreased Vit. C
D) Adrenal C) Diabetes
Ans: A D) HTN
95) Regarding confidence interval? Ans: C
A) Mean and standard error of mean 103) Which of following type of hernia is
B) Only mean associated with hesselbach triangle?
C) Only standard error A) Femoral hernia
D) Standard deviation B) Direct inguinal
Ans: A C) Indirect inguinal
96) Blood supply to which organ remains D) Umbilical
unchanged during moderate exercise? E) Strangulated
A) Heart Ans: B
B) Skin 102) 26yr old man had gunshot wound to
C) Brain the spine after which he lost
D) Splanchnic sensations to lower limbs loss of
E) Skeletal muscle power but increased tone and up
Ans: C going plantar with central clonus
Ref: BRS physiology lesion at?
97) Thirst is stimulated by which of A) Corticospinal
following? B) Lateral spinothalamic
A) ADH C) Both corticospinal and lateral
B) Angiotensin – 2 spinothalamic
C) Renin D) Cerebellum
D) Aldosterone E) Corticobulbar
E) Angiotensin-1 Ans: C
Ans: B 105) Which of the following zone is most
98) Pulse pressure is increased in which of commonly involved in BPH?
following conditions? A) Peripheral zone
A) Aortic stenosis B) Middle zone
B) Tachycardia C) Anterior zone
C) Thyrotoxicosis D) Central/periurethral zone
D) Pericardial effusion Ans: D
Ans: C 106) Most common risk factor for penile
carcinoma?
SK Origional – Golden 11 327

A) Phimosis D) Renal artery stenosis


B) Alcohol Ans: A
C) Smoking 113) A man drinks 2 L of water to replenish
D) STDs the fluids lost by sweating during a
Ans: A period of exercise. Compared with the
107) Growth hormone needs which situation prior to the period of
hormone for regulation? sweating:
A) Insulin A) His intracellular fluid will be hypertonic
B) Cortisol B) His extracellular fluid will be hypertonic
C) Thyroxine C) His intracellular fluid volume will be
D) Glucagon greater
Ans: A D) His extracellular fluid volume will be
108) Blood supply of SA node mainly from? greater
A) RCA E) His intracellular and extracellular fluid
B) LAD volumes will be unchanged
C) Marginal Ans: C
D) PDA 114) Water proofing of skin is done by:
Ans: A A) Keratin
Explanation: SA node and AV node are usually B) Elastin
supplied by RCA. C) Hair follicles
Ref. 1st AID Ans: A
109 Which of the following vein run 115) Patient has hemoptysis &
parallel to anterior interventricular glomerulonephritis. The most
artery? probable diagnosis is:
A) Great cardiac A) Wegner‘s Granulomatosis
B) Middle cardiac B) Good Pasture Syndrome
C) Small cardiac C) Diabetic glomerulopathy
D) Left Circumflex D) SLE nephritis
Ans: A Ans: B
Explanation: Great cardiac vein run parallel to Ref: Robbins Pathology
anterior interventricular artery, while middle 116) A patient is suffering from cough
cardiac vein run parallel to posterior hemoptysis and weight loss. There is
interventricular artery. history of smoking. Most likely
Ref: B.D Chourasia diagnosis?
110) Blood supply of rectum is from: A) Pneumonia
A) Inferior mesenteric B) Bronchogenic carcinoma
B) Superior mesenteric C) Asthma
C) Celiac trunk D) COPD
D) Gastroduodenal E) ABPA
Ans: A Ans: B
111) Sinus artery is branch of which of 117) Karyotype of Kline filter syndrome is?
following? A) 46XXY
A) RCA B) 47XXY
B) LAD C) 45XO
C) LCX D) 44XO
D) Marginal Ans: B
Ans: A 118) True hermaphroditism?
112) What is most likely cause of A) 45xxy
paroxysmal hypertension? B) 45xyy
A) Pheochromocytoma C) 46xyy
B) Cushion D) 46xxy
C) Conns syndrome E) 47xxy
SK Origional – Golden 11 328

Ans: D Ans: B
119) Which of following is single motor 126) Part of heart form most of sternal
supply muscle? surface?
A) Diaphragm A) Right atrium
B) Mylohyoid B) Right ventricle
Ans: A C) Left Atrium
120) A female with non caseating D) Left ventricle
granuloma and hilar Ans: B
lymphadenopathy (sarcoidosis). The 127) Which of the following structure
type of cells involved: arches over right lung?
A) Basophils A) Azygous
B) T- lymphocytes B) Vagus
C) Eosinophils C) Right phrenic
D) Plasma cells D) Left phrenic
E) Neutrophils Ans: A
Ans: B 128) A child with decrease chest sounds on
121) In pituitary adenoma, the bitemporal left side? And decreased chest
hemianopia occurs due to lesion of: movements on left side, improve on
A) Central part of optic chiasma holding up and intestine is present on
B) Optic nerve left side of chest. Cause is:
C) Optic tract A) Defective pleura peritoneal membrane
D) Peripheral part of optic chiasma B) Hiatus hernia
Ans: A C) Diaphragm eventration
122) Which sutures were prematurely Ans: A
closed in frontal and occipital Explanation: most common cause of
bossing? diaphragmatic hernia is defective pleura peritoneal
A) Sagittal membrane defect.
B) Coronal Ref. Langmans Embryology
C) Lambdoid 129) Which of the following is dynamic
D) Coronal + lambdoid support of uterus?
Ans: A A) Pelvic diaphragm
123) Right vocal cord weakness which of B) Broad ligament
following nerve is involved? C) Urogenital diaphragm
A) Right recurrent laryngeal D) Transverse cervical ligament
B) Left recurrent laryngeal Ans: A
C) Left vagus nerve 130) Prostate lymph drainage:
Ans: A A) Internal Iliac
124) What is age of closute of Anterior B) External iliac
fontanelle? C) Internal and external
A) 24 month D) Inguinal
B) 12 month to18months E) Para aortic
C) 9 month Ans: A
D) 36 month Ref: Clinical Anatomy
Ans: A 131) Loss of water by evaporation and
Explanation: After 18 month range is 18- insensible loss from body:
24months(KLM+BD) A) Controlled by hypothalamus
125) Blood supply of middle of B) Thermal gradient
oesophagus: C) Remains constant
A) Superior thyroid D) Depends on core body temperature
B) Descending thoracic aorta Ans: D(Repeated)
C) Inferior thyroid
D) Left gastric
SK Origional – Golden 11 329

132) Person naked in room, Temp 21°C, D) Histoplasma capsultum


humidity present, heat loss will occur E) Malassezia furfur
by: Ans: C
A) Insensible perspirations Ref: 1st AID
B) Sweating 139) Mother gave which immunoglobin to
C) Radiation hydrops baby?
D) Urination A) IgG
Ans: C B) IgA
133) In carpal tunnel syndrome there is C) IgM
compression of following nerve? D) IgD
A) Median Nerve E) IgE
B) Ulnar nerve Ans: A
C) Radial nerve 140) Patient during surgery blood
E) Axillary nerve transfusion is given. She develops
Ans: A high grade fever, hypotension and
134) Diabetic patient has increase shock in post-operative period due to:
urination and frequency due to? A) Graft versus host reaction
A) Glomeral protein defect B) Febrile non hemolytic reaction
B) Decrease water absorption C) Bacterial contaminated blood
C) Decrease glucose absorption D) Anaphylactic shock
D) Increase glucose absorption Ans: C
Ans: C 141) In anaphylactic shock what changes
135) Capillary oncotic pressure is: occur in patient?
A) 10 mmHg A) Hyperpyrexia
B) 15 mmHg B) Arteriolar and venous vasodilation
C) 26 mmHg C) Vasoconstriction
D) 50 mmHg Ans: B
Ans: C 142) A patient came in emergency 2cm
136) Osmotic pressure is due to which of water jvp 2L cardiac output, lactate 30
following? in blood BP 70/30 , what type of
A) Albumin shock is this:
B) Insulin A) Anaphylactic shock
C) Na B) Cardiac tamponade
D) Cl C) Hypovolemic shock
E) K D) Septic shock
Ans: A Ans: C
137) In burn patient edema is due to: 143) A child with erythroblastosis fetalis
A) Dec albumin having blood group A+ve need blood
B) Decrease water transfusion which blood to give?
C) Decrease cl A) A negative
D) Decrease Na B) A positive
Ans: A C) O negative
138) 38 years old HIV positive patient is D) B negative
complaining of headache. On E) B positive
examination neck stiffness is positive Ans: A
fungal meningitis is suspected to be 144) Which of following is strong abductor
the cause. Which of the following is of femor?
thought to be the number one cause in A) Sartorius
HIV/AIDS patient? B) Gluteus Maximus
A) Aspergillus fumigates C) Gluteus Minimus
B) Blastomycosis dermatitis‘s D) Gluteus Medius
C) Cryptococcus neoformans Ans: B
SK Origional – Golden 11 330

145) Proximal artery stenosis is: C) Ct/mri pituitary


A) Proximal 2cm of Renal Artery D) Perimetry
B) Distal 2cm of RenalArtery Ans: A
C) Distal 1cm 153) A patient with Hb 8, TLC 3.5, PLT
D) Distal 0.5cm 90×10^9. Spleen enlarged 3cm. What
Ans: A investigation you will do to confirm
146) Which of the following is true cause of Hyperspleenism?
regarding Median nerve in upper arm? A) Bone marrow biopsy
A) Medial to brachial artery in cubital fossa B) CT spleen
B) Lateral to brachial artery in cubital fossa C) UsG abdomen
Ans: A D) CBC
147) Tumor marker for Hepatic Ans: A
carcinoma? 154) 26 yr old girl 28 week of gestation
A) AFP developed vomiting, bilirubin
B) CEA 20mg/dl AST272 and ALT 220 two
C) HCG weeks back when she went to meet
D) CA-125 her parents in a remote village. She
E) CA-19.9 has no history of any blood
Ans: A transfusion or past surgery. What is
148) What is Common relation of ureter? her diagnosis?
A) Anterior to gonadal vessels and posterior A) Hep. A
to iliac artery B) Hep. B
B) Posterior to gonadal vessel and anterior C) Hep. C
to iliac artery D) Hep. D
Ans: B E) Hep. E
149) In hypertensive patient what is cause Ans: E
of edema? 155) Most potent chemotactic agent:
A) Increase Hydrostatic pressure A) LTB4
B) Decreased Hydrostatic pressure B) C5a
C) Increased oncotic pressure C) C4b
D) Lymphatic vessel blockage D) C3b
Ans: A E) C4a
150) What is most common genetic cause Ans: B
of mental retardation? Explanation: Most potent… C5b
A) Down syndrome In Arichdonic acid pathway…LT4
B) Fragile X syndrome 156) Diapedesis refer to:
C) Turner syndrome A) Migration
D) Kline filter syndrome B) Margination
Ans: A C) Chemotaxis
151) Warfarin affects which of following Ans: A
pathway of coagulation? 157) Macrophages are derived from?
A) Extrinsic A) Monocytes
B) Intrinsic B) Neutrophils
C) Common pathway C) Eosinophils
D) Extrinsic + Intrinsic D) Lymphocyte
Ans: A Ans: A
Ref: 1st AID Ref: Wheatear‟s Histology
152) In a young female pt with prolactin 158) Ejaculation is done due to:
level 28 (normal 26) but no A) Parasympathetic
agalactorrhea, what will you do? B) Sympathetic
A) Rule out physiological cause C) Pudendal
B) Pituitary profile D) Sympathetic + parasympathetic
SK Origional – Golden 11 331

Ans: C A) Sensory supply from mid cervical


Ref: 1st AID dermatome
159) Metabolic response to trauma, skeletal B) Sensory supply from T10 dermatome
muscles respond by: C) Sensory supply from splanchnic
A) Decreased protein loss D) from vagus nerve
B) Increased proteolysis Ans: A
C) Decreased proteolysis 166) Acute laryngotracheobronchitis
D) No change (Croup) is caused by which of
Ans: B following?
160) Carbohydrates malabsorption is A) Parainfluenza virus type 1
because of absence of: B) Respiratory Syncytial virus
A) Intestinal lactase C) Hemophelius
B) Intestinal maltase D) Legionella
C) Salivary amylase Ans: A
Ans: A 167) What lies posterior to uncinated
161) A severely will pt. with bleeding process of pancreas?
tendency now. Deranged clotting A) Aorta
profile and raised FDPs: B) SMA
A) ITP C) IVC
B) DIC D) IMA
C) Factor 8 deficiency E) Celiac
D) Thrombocytosis Ans: A
Ans: B Explanation:
162) A sea diver after diving develops Ant. To uncinate process of pancreas is SMA
shortness of breath and joint pain. Post. To uncinate process of pancreas is Aorta
Cause is: Ref: B.D Chourasia
A) Lactic Acidosis 168) The chance of bleeding in a patient
B) Nitrogen bubble with epidural will be greatest in:
C) Excessive fatigue A) Patient on warfarin
Ans: B B) Heparin
163) Ankylosing spondylitis is associated C) LMWH
with which of following HLA subtype? D) Aspirin
A) HLA-b27 Ans: C
B) HLA-b8 169) Most potent Na retaining hormone is:
C) HLA-b17 A) Aldosterone
D) HLA-DQ8 B) Angiotensin 2
Ans: A C) ADH
Explanation:Following associated with HLA- D) Cortisol
B27. Ans: A
PAIR, Psoriasis, Ankylosing spondylitis, Irritable Ref: Guyton Physiology
bowel syndrome, RA 170) Percentage of Thalassemia trait in
Ref: 1st AID Pakistan?
164) Marker raised 60% in colon cancer n A) 5
80% in pancreatic cancer: B) 15
A) CEA C) 17
B) CA19-9 D) 12
C) CA125 Ans: A
D) CA15-3 171) All are neural crest cell derivative
E) PSA except:
Ans: B A) Vomer bone
165) Gall bladder pain over neck due to: B) Choroid and sclera of eye
C) Tunica media of great vessels
SK Origional – Golden 11 332

D) Olfactory epithelium Sensitive test..ANA


E) Vestibule & cochlea Specific test…Antismth
Ans: C
172) Ovulation occurs on which day of
menstrual cycle? ===============
A) 10th
B) 14th
C) 18th RADIOLOGY
D) 19th November 24th, 2020 – Morning
Ans: B
173) In which of the following maximum
1) A cardiac surgeon during open heart
chemotaxis occur?
surgery found bleeding vessel
A) Migration accompanying LAD in anterior
B) Acute inflammation
interventricular groove. Most likely
C) Diapedesis
the vessel is?
D) Margination
a. Anterior cardiac vein
Ans: B
b. Oblique vein
174) Anti tuberculosis drug causes Red
c. Great cardiac vein
green color blindness?
d. Small cardiac vien
A) Ethambutol
e. Marginal vein
B) INH Ans: C
C) Rifampicin
2) An 32 years old obese male comes
D) Streptomycin
complaining of rapid heart rate, 9kg
Ans: A
weight loss in 2 months and heat
175) AV septum is ruptured which of
intolerance. His pulse is 110/min and
following valve is affected most?
serum T4 levels are elevated. Upon
A) Bicuspid
questioning, he admits to taking
B) Tricuspid
thyroxin tablets for losing weight. The
C) Aortic parameter which shows an increase is?
D) Pulmonary
a. Serum cortisol
Ans: B
b. Serum free T4
176) Which of the following is not
c. Serum TSH
characteristic of hypopituitarism?
d. Serum T3
A) Cachexia
Ans: B
B) Infertility
3) Cephalohematoma most common site
C) Pallor
is:
D) Low BMR a. Loose areolar tissue
Ans: A
b. Connective tissue
177) Spermatogenesis complete in how
c. Under the skin
much period:
d. Under periostium
A) 74 days
Ans: D
B) 60 days
4) A patient is suffering from dysphagia
C) 87 days
dysarthria analgesia thermo
D) 49 days
anesthesia on ipsilateral side of face
Ans: A and ipsilateral Horner
178) Most sensitive test for screening of
syndrome.Occlusion of which of the
SLE?
following artery would produce these
A) Anti smith
symptoms and signs?
B) Anti phospholipid
a. Posterior inferior cerebellar artery
C) Anti DS
b. Anterior inferior cerebellar artery
D) ANA
c. Posterior spinal artery
Ans: D
SK Origional – Golden 11 333

d. Anterior spinal artery c. Pancreas


e. Pons d. Spleen
Ans: A Ans: C
5) A scenario of a child having recurrent 12) During hysterectomy, ureter can be
bacterial infections and increased damaged at which site?
neutrophils in blood while absence of a. Pelvic brim
neutrophils at the infection sites. b. Behind the broad ligament
Neutrophils can‟t attach to c. Crossing common iliac arteries
endothelium to deficiency of? d. Entering into bladder
a. LFA-1 (Integrin) Ans: B
b. Icam 1 13) Which of the following ligaments
c. CD-21 keeps the vertebral bodies in place?
d. P-selections a. Ligamentum flavum
Ans: A b. Transverse ligament
6) Parasympathetic stimulation causes: c. Ant and post longitudinal ligament
a. Bronchial constriction d. Interspinous ligament
b. Increase heart rate Ans: C
c. Decrease secretions 14) SA node is supplied usually by:
d. Increase sphincter tone a. Left anterior descending artery
Ans: A b. Right coronary artery
7) An old lady with cushing syndrome c. Circumflex artery
what will be characteristics? d. posterior interventricular
a. Hyperpigmentation Ans: B
b. Abdominal stria 15) Anomic aphasia occurs due to lesion
c. Pot belly. in which of the following areas?
d. Amenorrhea a. Frontal
Ans: B b. Temporal
8) Blood supply of apex heart? c. Angular gyrus
a. Marginal d. Brocas
b. RCA Ans: C
c. LAD 16) A patient is presented in OPD with
d. Anteriror circumflex bitemporal hemianopia aneurysm is
e. PDA most likely in which of following?
Ans: C a. Anterior communicating artery
9) Weakness of right side of body and b. Posterior cerebral artery
diplopia on looking left? c. Middle cerebral artery
a. Left internal capsule d. Posterior communicating artery
b. Right internal capsule Ans: A
c. Midbrain 17) What is site of tracheostomy?
d. Right cerebral cortex a. C2
e. Left cerebral cortex b. C3
Ans: C c. C4
10) Blood supply to middle of esophagus? d. C5
a. Inferior thyroid artery Ans: A
b. Left gastric artery 18) A patient presents with vertical
c. Descending thoracic aorta diplopia, head tiliting, Blurred vision
Ans: C and also difficulty in coming
11) Which of the following structure is downstairs. The likely nerve damaged
derived from dorsal as well as ventral is:
bud? a. Oculomotor nerve
a. Liver b. Trochlear nerve
b. Gall bladder c. Trigeminal nerve
SK Origional – Golden 11 334

d. Abducent nerve b. Suspensory ligament of the ovary


e. Optic nerve c. Cardinal (transverse cervical) ligament
Ans: B d. Suspensory ligament of the clitoris
19) During game boy injured which area e. Round ligament of the uterus
of orbit that now there is vertical Ans: E
diplopia and enophthalmos? 26) A patient died of chronic renal failure
a. Apex which of the following will likely be
b. Base hypertrophied on autopsy?
c. Roof a. adrenal gland.
d. Floor b. adrenal medulla.
Ans: D c. adrenal glnd.
20) Burkitt lymphoma is caused by EBV d. parathyroid gland
due to: Ans: D
a. Interfere DNA repair 27) Potassium depletion may cause:
b. Produce toxin against DNA a. Fall in plasma PCO2
c. Produce mitochondrial toxins b. Fall in plasma pH
d. Antibody to viral capsid c. Rise in hydrogen ion in plasma
e. Common in African d. Rise in plasma bicarbonate
Ans: E Ans: D
21) Mechanism of action of cabergoline: 28) A child born with genetically male but
a. D1 agonist external appearance of genital are like
b. D2 agonist female what‟s is diagnosis?
c. D3 agonist a. Testicular feminization syndrome
d. D2 antagonist b. Down syndrome
Ans: B c. Klinefilter
22) Which of following causes d. Turner syndrome
endometrial carcinoma in post- Ans: A
menopausal women? 29) Breast lies above
a. Progesterone a. Pectoralis major
b. Prolactin b. Pectoralis fascia
c. Estrogen c. Pectoralis minor
d. TSH d. Ribs
Ans: C Ans: B
23) Bicornuate uterus abnormality of: 30) The extracellular matrix and the
a. Mesonephric duct cytoskeleton communicate across the
b. Paramesonephric duct cell membrane through which of the
c. Both following?
d. None a. Proteoglycans
Ans: B b. Integrins
24) Droplet infection can spread up to: c. Cadherins
a. 1-2 feet d. Intermediate filaments
b. 3-6 feet e. Microtubules
c. 8-9 feet Ans: B
d. 15 feet 31) A 50 year old man known to have
Ans: A hepatoma develops dependent edema
25) A 68-year-old woman with uterine with dilated abdominal walls veins.
carcinoma undergoes surgical The most likely structure involved in
resection. This cancer can spread this patient is:
directly to the labia majora in a. Hepatic artery
lymphatics that follow which of the b. Hepatic vein
following structures? c. Inferior vena cava
a. Pubic arcuate ligament d. Portal Vein
SK Origional – Golden 11 335

e. Right side of the heart d. Vital protein degradation


Ans: C e. Hepatic gluconeogenesis
32) Cimetidine with sucralfate interaction Ans: B
causes? 39) A 50 year old housewife is suffering
a. Cimetidine decreases sucralfate from gastric ulcer and arthritis. A safe
metabolism n causes its toxicity drug to give her relief from joint pain
b. Sucralfate Decrease the absorption would be:
cimetidine a. Aspirin
c. Cimetidine interfere with b. Celecoxib
pharmacodynamics of sucralfate c. Ibuprofen
Ans: B d. Indomethacin
33) In mastectomy which nerve is e. Ketoprofen
damaged? Ans: B
a. Spinal accessory nerve 40) A patient is diagnosed as case of
b. Long thoracic nerve Aplastic anemia. He is suffering from
c. Axillary nerve osteomyelitis the most likely
d. Radial ner6 organism?
Ans: B a. Staphylococcus aureus
34) A thyroid mass usually moves with b. Streptococcus
swallowing because the thyroid gland c. Salmonella
is enclosed by which of the following d. E.coli
fascia? Ans: A( C when Sickle cell disease)
a. Carotid sheath 41) Gastric Ulcer pain carried by:
b. Investing layer of the deep cervical fascia a. Vagus nerve
c. Pretracheal fascia b. Greater splanchnic
d. Prevertebral fascia c. Intercostal nerves
Ans: C d. Lesser splanchnic
35) Biopsy taken from posterior triangle of Ans: B
neck damage to which of following 42) Stony dull percussion note shows?
nerve? a. Fluid in pleural cavity
a. Spinal part of accessory nerve b. Pneumonia
b. Long thoracic nerve c. Atelectasis
c. Axillary d. Spontaneous rupture of bulla
d. Radial nerve Ans: A
Ans: A 43) Which structure lies posterior to arch
36) Most potent stimulus for Gallbladder of aorta?
contraction? a. Left phrenic nerve
a. CCK b. Rt. phrenic nerve
b. Fatty meal in duodenum c. SVC
c. Gastrin d. Brachiocephalic vein
d. Secretin e. Left recurrent laryngeal nerve
Ans: A Ans: E
37) Ossification of sternum completed by: 44) A patient was quickly diagnosed with
a. 15yr perforation of the posterior wall of the
b. 1 yr first part of the duodenum and erosion
c. 21 yr of artery behind it .The artery is most
Ans: C likely the:
38) Glucose level between meal is a. Common hepatic
maintained by: b. Gastro duodenal
a. Glycolysis c. Left gastric
b. Hepatic glycogenolysis d. Proper hepatic
c. Adipose tissue release FFA e. Superior mesenteric
SK Origional – Golden 11 336

Ans: B b. Autoimmune
45) After surgical neck of humerus c. TB
fracture there is loss of abduction and d. Streptococcus
proximal upper and lateral side Ans: A
sensation lesion in? 52) All of the following cause edema
a. Axillary nerve except?
b. Musculocutaneous nerve a. Obstruction of lymphatic flow
c. Brachial artery b. Dec oncotic pressure
d. Ulnar nerve c. Inc. hydrostatic pressure in capillaries
e. Radial nerve d. Dec. venous pressure
Ans: A e. Inc. lymphatic pressure
46) Injury at the anterior inferior iliac Ans: D
spine.Pelvic avulsion: 53) Maxillary nerve enters through which
a. Rectus femoris foramen?
b. Sartoius a. Foramen Ovale
c. Gluteous maximus b. Foramen rotundum
d. Adductor magnus c. Foramen spinosum
Ans: A d. Jugular foramen
47) Women developed painless gradual Ans: B
protrusion of one eye with normal X- 54) A middle aged woman presented in
Ray cause is? the OPD with complaint of pain in the
a. Orbital vessel aneurysm epigastric region, posterior thoracic
b. Optic nerve glioma wall and right shoulder after a fatty
c. Thyroid ophthalmology meal, the pain is most likely
d. Meningioma originating from:
Ans: B a. Appendix
48) True regarding scalenus anterior b. Gall bladder
muscle? c. Kidneys and ureters
a. Arises from C3-C6 vertebral bodies. d. Right deltoid muscle
b. Inserted at the anterior border of first rib. e. Right trapezius muscle
c. Phrenic nerve passes anterior to it. Ans: B
d. Subclavian artery/vein passess anterior to 55) Structure crossing superficial to the
it. flexor retinaculum:
Ans: C a. Median nerve.
49) Categorical data analysis by which of b. Ulnar nerve
following? c. Flexor digitorum superficialis.
a. Chi square d. Flexor pollicis longus.
b. T test e. None of the above.
c. Bar chart Ans: B
d. Cohort 56) Mono spot test is diagnostic for which
Ans: A of following disease?
50) The action potential of a neuron? a. HIV
a. Is initiated by efflux of Na+ b. Typhoid
b. Is terminated by efflux of K+ c. Infectious mononucleosis
c. Declines in amplitude as it moves along d. Measles
the axon e. GBS
d. Results in a transient reversal of the Ans: C
concentration gradient of Na+ 57) Pericardiocentesis is best performed
Ans: B by passing a needle through:
51) Waterhouse frederiksen most common a. The fourth intercostal space.
cause: b. The 5thintercostal space left to sternum
a. Neisseria meningitides
SK Origional – Golden 11 337

c. The second intercostal space at the a. Mitotic figure


midclavicular line. b. Dysmoplasia
d. The second intercostal space at the right c. Dysplasia
sternal angle. d. Invasion
Ans: B Ans: D
58) Sesamoid cartilage is present in: 65) Difference between right and left
a. Acetubulum kidney?
b. Epiglottis a. Size
c. External auditory canal b. Weight
d. Knee c. Color
e. Alae of the nose d. Relation at hilum
Ans: E e. None
59) Aspiration of foreign body in supine Ans: D
position, the object will be lodged in 66) Highly susceptible to ischemic injury
which lung lobe? in ATN:
a. Posterior segment of right upper lobe a. PCT
b. Superior bronchopulmomary segment of b. Thin descending and DCT
right lower lobe c. DCT
c. Right middle lobe d. Collecting ducts and DCT
d. Left inferior lobe Ans: A
Ans: B 67) The diagnosis of amyloidosis is best
60) Voluntary inhibition of micturition confirmed by:
reflex: a. Bence jone's proteinuria
a. Pudendal nerve b. Bone marrow
b. Detrusor muscle contraction c. Clinical observation
c. Trigon d. Congo red stain
Ans: A Ans: D
61) Newborn baby intubated. Breathe 68) A 35 year old female presented with
sounds absent on Rt side of chest. secondary infertility. Fallopian tubes
Cause of endobronchial intubation? were blocked. Endometrium showed a
a. Dystocia collection of epitheloid cells,
b. Cervical vertebra fracture histiocytes and giant cells:
c. Short neck a. Chlamydia infection,
d. Long neck b. Gonococcal infection
Ans: C c. Herpes simplex infection
62) A patient has accident BP less than 60 d. Trichomonas vaginalis infection
which effect is most pronounced? e. Tuberculous infection
a. CNS ischemic response Ans: E
b. Chemoreceptor 69) HLA- DR4 is associated with:
c. Baroreceptor a. Rheumatoid arthritis
d. RAAS b. SLE
Ans: A c. Scleroderma
63) In a 60 yr old female with breast d. None of the above
cancer and osteoporosis HX of Ans: A
ischemic heart disease which should 70) Structurally which lobe of prostate is
be given? the largest?
a. Raloxifen a. Median
b. Bisphosphonates b. Central
c. Vit. d c. Lateral
d. Calcium Ans: A (Anatomically lateral)
Ans: A 71) Through Hasselbach triangle which
64) Microscopic feature of malignancy is: hernia comes out?
SK Origional – Golden 11 338

a. Direct inguinal hernia 78) Appendices epiploic are presented in:


b. Indirect inguinal hernia a. Rectum
c. Femoral hernia b. Duodenum
d. Abdominal hernia c. Sigmoid colon
Ans: A d. Jejunum
72) A patient was diagnosed as a case of Ans: C
incisional hernia. The 1st structure in 79) Labia majora lymphatic drainage:
this lump is most likely to push from a. Medial group of superficial inguinal
within out ward is: b. Vertical group of superficial inguinal
a. Internal oblique muscle c. Deep inguinal
b. Transversalis muscle d. Para aortic
c. Transversalis fascia Ans: A
d. External oblige muscle 80) Pancreatic insufficiency leads to
Ans: C deficiency of which of following
73) In liver cell injury, enzyme decreased vitamin?
is: a. Vitamin C
a. ALP b. Vitamin B3
b. 5-nucleotidase c. Vitamin B1
c. AST d. Vitamin D
d. Pseudocholinestrase Ans: D
e. ALT 81) Inversion of foot by which muscle?
Ans: D a. Peroneus longus.
74) A tall man with normal external b. Peroneus tertius.
genitalia married but had no kids he c. Tibialis posterior
had normal ejaculation period but Ans: C
semen analysis showed azoospermia 82) Prostate lymphatic drainage:
cause may be: a. Internal iliac
a. Epididymis b. External iliac
b. Sertoli cells c. Para aortic
c. Scrotum d. Inguinal
d. Tubules in males. e. None
Ans: B Ans: A
75) Portal hypertension vein dilated: 83) Nerve supply of sternocleidomastoid
a. Left gastric vein muscle is:
b. Inferior mesenteric a. Trigeminal nerve and C2,3
c. Inferior venal cava b. Accessory nerve and C2,3
d. Superior mesentric vein c. Facial nerve and T5,7
Ans: A Ans: B
76) Assessment of muscle injury by: 84) The deep branch of radial nerve
a. Myoglobin supply all except:
b. CKMB a. Deep extensor of forearm
c. CKBB b. Long and medial head of triceps
d. CKMM c. Elbow joint
e. LDH d. Interossi muscle of hand
Ans: D e. Supinator muscle
77) Type of necrosis seen in liver: Ans: D
a. Caseous 85) A young man presents with complain
b. Coagulative of low grade fever, generalized
c. Gas weakness, backache and swelling in
d. Liquefactive front of thigh. On examination he has
e. None swelling above and below the inguinal
Ans: B
SK Origional – Golden 11 339

ligament which disappears on lying c. Inferior cerebellar peduncle


down diagnosis? d. Lower pons and pyramidal tracts
a. Aneurysm of femoral Artery Ans: A
b. Femoral hernia 93) Confidentiality should not be
c. Inguinal hernia breached in situation:
d. Psoas abcess a. An HIV
e. Sphena varix b. An adult male sexually abusing daughter
Ans: D c. A girl told you she has hid 100 tablets to
86) Nerve supply of triceps muscle is: commit suicide
a. C6,7,8 d. A women told you she is depressed and
b. C3,4,5 occasionally she has suicidal thoughts
c. C2,3,4 Ans: C
d. C1,2 94) A patient of acute tonsilitis mediators
Ans: A of acute inflammation:
87) Patient presented in ER with open a. TNF and IL – 1
fracture of tibia. BP 80/50mmHg, b. Bradikinin
decrease GFR due to: c. Histamine
a. Decrease arterial blood flow d. Nitric oxide
b. Decrease arterial blood pressure Ans: A
c. Decrease TPR 95) BPH is due to enlargement of which
d. Increase arterial blood flow lobe?
Ans: A a. Median lobe
88) Fracture neck of fibula artery involved: b. Medial lobe
a. Posterior tibial artery c. Posterior lobe
b. Anterior tibial artery d. Lateral lobe
Ans: B Ans: A
89) In a pregnant lady, to diagnose open 96) Hyperaldosteronism associated with
neural tube defect early which one of all except:
the following is used? a. Hypernatremia
a. Chorionic villus sampling b. Hypokalemia
b. Amniocentesis c. Hypertension
c. Ultrasound d. Metabolic acidosis
Ans: C Ans: D
90) Measles in pregnant lady doctor 97) Old man with C/O anosmia and
worried about: hypogonadism,, lesion on CT Scan is
a. TOF in:
b. Peripheral neuropathy a. Amygdala
c. Cataract b. Arcuate nucleus of hypothalamus
d. Anencephaly c. Dorsomedial nucleus of thalamus
e. Cleft plate Ans: B
Ans: C 98) A player was hit by a ball on the
91) Corona radiata in secondary oocyte posterior and medial aspect of elbow
which layer forms it? joint with fracture of medial
a. Granulosa epicondyle and damage ulcer
b. Primary oocyte nerve.This would result in sensory loss
c. Theca external over?
d. Theca internal a. Lateral 2/3 over dorsal surface
Ans: A b. Lateral 2/3 over palmer surface
92) Superolateral relation of floor of 4th c. Medial 1/3 over dorsal
ventricle: d. Medial 1/3 over palmer and dorsal
a. Superior cerebellar peduncle surface
b. Middle cerebellar peduncle e. Medial 1/3 over medial surface
SK Origional – Golden 11 340

Ans: D a. Mesoderm
99) Blood supply of head of femur in b. Endoderm
adults: c. Ectoderm
a. Obturator artery d. Neural crest
b. Retinacular artery Ans: A
c. Nutrient artery 107) All of these supply the first 2cm of
d. Medial Circumflex duodenum except:
Ans: B a. Supraduodenal artery
100) Vein accompanying posterior b. Common hepatic artery
interventricular artery: c. Gastro duodenal artery
a. Middle cardiac vein d. Superior pancreaticoduodenal artery
b. Anterior cardiac vein e. Right gastroepiploic artery
c. Posterior cardiac vein Ans: D
Ans: A 108) Lymphatic drainage of the fundus of
101) In avascular necrosis of femur head stomach is?
which artery is involved? a. Celiac node
a. Medial circumflex artery b. Superior gastric node
b. Obturator artery c. Hepatic nodes
c. Deep circumflex artery d. Inf. mesenteric nodes
d. None Ans: A
Ans: A 109) Which structure is not present in
102) A patient having ST elevation in lead 2 lesser omentum?
and complete heart block vessel a. Portal vein
involved? b. Bile duct
a. RCA c. Left gastric artery
b. Right marginal artery d. Hepatic artery
c. LAD Ans: C
Ans: A 110) Hydrocarbon causes which
103) Traid of carcinoma?
thrombocytopenia,eczema,and a. Bronchogenic carcinoma
recurrent infections is seen in: b. Bladder carcinoma
a. Wiskott-Aldrich syndrome c. Hepatocellular carcinoma
b. Williams syndrome d. Renal cell carcinoma
c. SCID Ans: B
d. IgA deficiency 111) Wrist joint is an example of:
e. HUS a. Condylar type
Ans: A b. Sadle type
104) Most common valve involved in c. Ellipsoid type
endocarditis in IV drug users: Ans: C
a. Mitral 112) In which of the following area, the
b. Tricuspid vomiting center is located:
c. Aortic a. Thalamus
Ans: B b. Hypothalamus
105) Regarding the pubic symphysis, what c. Medulla oblongata
type of joint is? d. Pons
a. Fibrous Ans: C
b. Condyloid 113) Child with frontal bossing bow legs
c. Synovial and chest deformity it is seen in:
d. Primary Cartilaginous a. Malnutrition
e. Secondary Cartilaginous b. Vit. D deficiency
Ans: E c. Myrasmas
106) Adrenal cortex is derived from? Ans: B
SK Origional – Golden 11 341

114) Patient having positive trendelenburg d. Depression of mandible


sign muscles involved: Ans: A
a. Gluteus maximus 121) A person has difficulty in language
b. Sartorius muscle comprehension and intelligence. The
c. Gluteus medius and minimus leison in which in which area?
Ans: C a. Calcarine cortex
115) Clostridium botulinum specific b. Premotor cortex
symptoms in food poisoning: c. Broca area
a. Diarrhea d. Prefrontal cortex
b. Flacid paralysis of respiratory muscles e. Wernicke area
c. Projectilevomiting Ans: E
d. Convulsions 122) Blood leaving placenta have oxygen
Ans: B saturation of:
116) While attempting to suture the distal a. 10%
end of a coronary bypass onto the b. 30%
anterior interventricular artery, the c. 80%
surgeon accidentally passed the d. 26 %
needle through the adjacent vein. e. 100%
Which vein was damaged? Ans: C
a. Anterior cardiac vein 123) An epileptic lady who currently on
b. Coronary sinus phenytoin has become pregnant .She
c. Great cardiac vein is concerned about her medication in
d. Middle cardiac vein Pregnancy. Which of the following is
e. Small cardiac vein correct?
Ans: C a. Add lithium
117) The most commonest hospital b. Switch phenytoin
acquired organism for UTI: c. Increase dose of phenytoin
a. E. Coli d. Add valproic acid
b. P-auriginosa e. Replace with barbiturates
c. S-aureus Ans: E
d. Citrobacter species 124) A 14thyear old boy eats fast food
e. Klebsella species only,peripheral blood picture of
Ans: A macrocyte,diagnosis?
118) Terminal part of cBD in relation to a. Folic acid deficiency
pancreas is? b. Pernicious anaemia
a. Lie posteriorly c. Iron deficieny anaemia
b. Lie anteriorly d. Megaloblastic anaemia
c. Embedded into pancreas Ans: A
d. Lies laterally 125) Which structure causes esophageal
Ans: C compression?
119) In histology slide serous acini with a. Left atrium
stratified epithelium in ducts: b. Left bronchus
a. Parotids c. Left ventricle
b. Lacrimal glands d. Right atrium
c. Sublingual glands Ans: A
d. Sub mandibular glands 126) A Patient of TB is on Att for 2 months.
Ans: A Tb antibodies:
120) Posterior fibers of temporalis a. Circulating in plasma
contraction causes: b. Bound to nucleus
a. Retraction of mandible c. Cell bound
b. Elevation of mandible Ans: C
c. Chewing 127) S3 heart sound occurs in which phase?
SK Origional – Golden 11 342

a. Isovolumetric relaxation a. Epinephrine


b. ISO volumetric contraction b. Norepinephrine
c. Rapid ventricular filling c. ACTH
Ans: C d. Cortisol
128) Basophilia of the cell is due to: Ans: A
a. SER 136) Durning transport of oxygen and CO2.
b. Ribosomes Amount of CO2 dissolved in plasma at
c. Mitochondria 45mmhg is:
d. Golgi apparatus a. 0.3%
Ans: B b. 2.7%
129) A 4 year boy present with periorbital c. 23%
edema. Mechanism of edema is? d. 70%
a. Increase albumin Ans: B
b. Hypoalbonemia 137) In 15 year old boy medullary
c. Decrease hydrostatic pressure carcinoma of thyroid is considered
d. Lymphatic obstruction familial because of:
Ans: B a. Age
130) Man in marathon in July excessive b. Solitary
sweating collapses cause? c. Metastasis
a. Dehydration d. Increase secretion of hormone
b. Heat stroke Ans: A
c. Heat exhaustion 138) Nonbacterial thrombotic endocarditis
Ans: C is associated with?
131) Presence of pancreatic tissue in a. Congenital heart disease
gastric mucosa is termed as: b. Ischaemic heart disease
a. Haematoma c. Rheumatic endocarditis
b. Metaplasia d. Systemic lupus erythematosis
c. Neoplasia e. Terminal neoplastic disease
d. Choristoma Ans: E
Ans: D 139) After cholecystectomy drain placed in:
132) Most common cause of hereditary a. Left subphrenic compartment
mental retardation? b. Right paracolic gutter
a. Down syndrome c. Right subhepatic compartment
b. Fragile X syndrome d. Right subphrenic compartment
c. Klinefilter e. Upper right infracolic compartment
d. Turner Ans: C
Ans: A 140) Scapula is attached to thoracic wall by
133) Which part of the male genital tract muscle:
has partially motile sperm? a. Quadratus lumborum
a. Ductus deference b. Lattismus Dorsi
b. Seminal tubercle c. Rhomboids Major
c. Epididymis d. Serratus Anterior
d. Rete testes Ans: D
Ans: C 141) A baby dies after birth with edema
134) Karyotype of klinefelter syndrome: jaundice and anemia on post partym
a. 47XXY you find which of da following
b. 47XYY changes:
c. 46XO a. Bile pigments in basal ganglia
Ans: A b. Heart problem
135) Which of the following is released in c. CNS problem
Stress due to surgery which causes d. Liver problem
vasoconstriction its due to release of? e. GIT problem
SK Origional – Golden 11 343

Ans: A a. Ectopic anal opening


142) In acute MI Changes on ECG: b. Imperforate anus
a. Flat T waves c. Bladder extrophy
b. Displaced ST Ans: A
c. Tall T 150) Epithelial cells neoplasia is associated
d. U waves with:
Ans: B a. Cytokeratin
143) Regarding pulmonary varix: b. Myc gene
a. Opens into left atrium c. Rb 53
b. Opens into right atrium d. Keratin pearls
c. Opens into left ventricle Ans: D
d. Opens into right ventricle 151) Tumor marker for pancreatic cancer:
Ans: A a. CA 19.9
144) An Anemic lady with raticulocyte 10% b. CEA
Jaundice. Peripheral smear shows c. CA 125
small spherocytes. Test for diagnosis: Ans: A
a. Direct coomb's test 152) Majority of CO2 transport in blood
b. Indirect coomb's test occurs by:
c. RBC survival study a. Dissolved form
Ans: C b. Carbamino compound
145) Liquefactive necrosis is seen in: c. Sodium bicarbonate compound
a. Acute appendicitis d. Potassium carbonate compound
b. Acute pancreatitis Ans: C
c. Ischemia of brain 153) Virus cause cancer by:
d. MI a. Proto oncogene
e. TB b. Oncogene
Ans: C Ans: A
146) Prostate cancer metastasisreaches the 154) Chronic pancreatitis steatorrhea
skull via: occurs due to:
a. Paravertebral plexus a. Trypsin
b. Lymphatic‘s b. Pepsin
c. Blood c. Amylase
Ans: A d. Lipase
147) Female with amenorrhea of 5 weeks Ans: D
presented with rite sided iliac pain, 155) As compared to the arterial blood, the
TLC raised and was febrile: venous blood has more:
a. Acute appendicitis a. Dissolved oxygen
b. Ectopic pregnancy b. Hemoglobin
c. Acute cholecystitis c. PH
d. Acute pancreatitis d. Packed cell volume (PCV
Ans: A e. Red cell count
148) Intrinsic factor is secreted by which Ans: D
area? 156) Free redicals are removed by:
a. Fundus of stomach a. Superoxide
b. Body of stomach b. Vit. D
c. Antrum c. Transferrin
d. Lesser curvature d. Lymphocytes
Ans: A e. Vit. K
149) Primitive streak mesoderm migrates Ans: C
to ant abd wall n around cloaca. A 157) Ferrous form of iron;
rupture of this mesoderm cranially will a. Ferritin
produce: b. Transferrin
SK Origional – Golden 11 344

c. Hemosiderin
d. Hemoglobin
Ans: D RADIOLOGY
158) Asthmatic patient with RR 30/min November 24th, 2020 - Afternoon
dyspenic will have:
a. Decrease airway entry 1) Regarding pulmonary varix which of
b. Increase PaCO2 following is true?
c. Decrease PaCO2 a. Opens into left atrium
d. Increase lung compliance b. Opens into right atrium
e. Decrease airway trapping c. Opens into right ventricle
Ans: C d. Opens into Left ventricle
159) A young female presented with fever Ans: A
for last 10 days and continuous chest 2) Sesamoid cartilage found in:
pain unrelated to respiration the likely a. Epiglottis
cause is: b. Nasal septum
a. Mycardium c. Ala of nose
b. Pericardium d. Ear pinna
c. Pleurisy Ans: C
d. Chostrochonditis 3) Which one of the following
e. Sub endocardium neurological condition is most
Ans:. A commonly associated with
160) Which of the following occurs in Rheumatoid Arthritis?
severe diarrhea? a. Carpal tunnel syndrome
a. Metabolic alkalosis b. Meningitis
b. Hyperkalemia c. Peripheral neuropathy
c. Increase plasma osmolariy d. Gillian barre syndrome
d. Hyponatremia Ans: A
Ans: D 4) Vein accompanying posterior
161) A child with retinoblastoma interventricular artery:
radiotherapy was given a second a. Middle cardiac vein
tumor arises in lower limb what could b. Anterior cardiac vein
be the tumor: c. Posterior cardiac vein
a. Lymphoma d. Lateral cardiac vein
b. Osteosarcoma Ans: A
Ans: B 5) Inferior epigastric artery is a branch
162) Retinoblastoma give rise to which of:
tumor? a. External iliac
a. Ewing sarcoma b. Internal iliac
b. Osteosarcoma c. Sup epigastric
c. Giant cell tumor d. Internal pudendal artery
Ans: B Ans: A
6) Largest lobe of prostate is:
a. Medial lobe
=============== b. Median lobe
c. Lateral lobe
d. Posterior lobe
Ans: C
7) Testicular tumor drains into which
lymph nodes?
a. Inguinal lymph nodes
b. Popliteal lymph nodes
SK Origional – Golden 11 345

c. Para-Aortic lymph nodes 15) A young athlete injuries his ankle and
d. Infaclavicular lymph nodes develops ecchymosis He can stand on
Ans: C his toes but painful:
8) Posterior wall of middle ear cavity a. Achilles tendon damage
gives attachment to: b. Plantaris tendon damage
a. Tensor tympani c. Medial mellolus fracture
b. Stepedius muscle d. Lateral mellolus fracture
c. Eustachian tube Ans: B
d. Tympanic membrane 16) Anterior duodenal perforation
Ans: B contents will move into?
9) Recurrent laryngeal nerve is a. Lesser sac
associated with: b. Right iliac fossa
a. Superior thyroid vein c. Hepatorenal pouch
b. Middle thyroid vein d. Right posterior sub phrenic
c. Inferior thyroid vein e. Right anterior sub phrenic
d. None Ans: D
Ans: C 17) Three segments apical, anterior and
10) Internal jugular vein anterior relation posterior in which lobe of lung?
to which structure after exiting from a. Right upper
skull? b. Right middle
a. Carotid sheath c. Right inferior
b. Internal carotid artery d. Left superior
c. SCM Ans: A
d. Accessory nerve 18) After esophagectomy which artery is
Ans: B used?
11) After mastectomy for breast Ca a a. Left gastric artery
female developed winging of scapula b. Left gastroepiploic
due to: c. Right gastroepiploic
a. Axillary nerve damage d. Splenic
b. Long thoracic nerve damage Ans: C
c. Metastatic calcification 19) During endoscopy 32 yr old women
Ans: B ulcer found on post wall of stomach it
12) Relation of left renal vein is: is likely to cause bleeding by causing
a. Passes infront of Aorta damage to?
b. Behind Aorta a. Left gastric artery
c. Left ovarian b. Gastro duodenal artery
d. Right ovarian c. Splenic artery
Ans: A Ans: C
13) In which of the following AFP is 20) The most common asbestos related
raised? tumor in human being is:
a. Seminoma a. Bronchogenic carcinoma
b. Teratoma b. Pleural mesothelioma
c. Embryonal carcinoma c. Bladder carcinoma
d. Sertoli cell tumor d. Breast carcinoma
Ans: C Ans: A
14) In femoral sheath which is most 21) Staph aureus infection causing
lateral structure? valvular damage. On autopsy, spleen
a. Femoral artery pale and 1.5 something lesion. What
b. Femoral nerve happened?
c. Femoral vein a. Coagulative necrosis
d. Lymphatic‘s b. Abscess formation
Ans: A c. Caseous necrosis
SK Origional – Golden 11 346

d. Liquifactive necrosis Ans: B


Ans: A 28) EEG of anesthetized patient shows:
22) 16 year old girl. Abdominal pain for 6 a. Straight line
months intermittent. Primary b. Normal frequency waves
amenorrhea. On p/A exam, no c. High frequency waves
finding. Hymenal ring normal. d. Low frequency waves
Abnormality: Ans: D
a. Transverse vaginal septum 29) Monocyte macrophages derived from:
b. Cervical atresia a. Bone marrow
c. Bicornuate uterus b. Spleen
d. Didelphus uterus c. Thymus
e. Imperforate hymen d. Lymphoid tissue
Ans: B Ans: A
23) Most likely paralyzed muscle taking 30) Repeated infections in a newborn are
origin from femur causing instability deficiency of:
of knee joint: a. IgG
a. Rectus femoris b. IgA
b. Sartorius c. IgM
c. Semimembranosus d. IgD
d. Semitendinosus e. IGE
e. Vastus lateralis Ans: A
Ans: E 31) In polycythemia vera which of the
24) During bronchoscopy after entering following will be increased?
right bronchus, first structure to be a. Increases blood concentration
seen: b. Increase RBC mass
a. Anterior branch of upper lobe c. Increase RBC concentration
b. Upper lobe d. Myelofibrosis
c. Middle lobe Ans: B
d. Lower lobe 32) Whole wheat contains which vitamin?
Ans: B a. Thiamine
25) Patient having traveler‟s diarrhea not b. Riboflavin
responding too antidiarrheal drugs, c. Biotin
which drug should be given? d. Pantothenic acid
a. Metronidazole Ans: A
b. Erythromycin 33) Person has problem with reading
c. Norfloxacin writing and speaking. Lesion in:
d. Chloramphenicol a. Left frontal and temporal
Ans: C b. Right frontal and temporal
26) Following is true regarding blood c. Right parietal
group antigens: d. Left parietal
a. Are present on Hb molecule. Ans: A
b. Are secreted in saliva 34) Most of lymphatic drainage of breast
c. Are called agglutinins. parallel to?
d. Are enzymes. a. Axillary artery and axillary nodes
e. Inherited as autosomal recessive. b. Internal thoracic arteries and parasternal
Ans: B nodes
27) Lichen planus, transformation to c. Thoracoaxromial artery and apical
malignancy probability? (subclavicular) nodes
a. 1 percent Ans: A
b. 10-15 percent 35) A12 year old boy/girl repeated
c. 15-20 percent infections. Deficiency of:
d. 25-30 percent a. IgG
SK Origional – Golden 11 347

b. IgA Ans: A
c. IgE 43) Sacral hiatus:
d. IgM a. Fused S4 and S5
e. IgD b. Unfused S4 and S5
Ans: B c. Between S1 and S2
36) Tumor at porta hepatis. What is d. Between S2 and S3
compressed? Ans: B
a. Common bile duct 44) What is weight of prostate gland?
b. Portal vein a. 28 gram
c. Hepatic artery b. 34 gram
d. Hepatic vein c. 18 gram
Ans: B d. 50 gram
37) During needle thoracotomy, what Ans: C
structure damaged first? 45) Free margin of leseer omentum
a. IC muscle contains:
b. IC nerve a. Portal vein,hepatic artery,common bile
c. IC vein duct
d. IC artery b. Hepatic vein and CBD
e. Internal thoracic artery c. Hepatic artery and hepatic vein
Ans: B d. Only CBD
38) Lucid interval after injury, then again Ans: A
loss of consciousness: 46) Most aggressive of all:
a. Epidural hematoma a. Melanoma
b. SAH b. Squamous cell carcinoma
c. Subdural hemorrhage c. Basal cell carcinoma
d. Intracerebral hemorrhage d. Nevo
Ans: A Ans: A
39) Lower lumbar vertebra slipped over 47) Abdominal bloating, foul smelling
the sacram so it came anterior to it stools, what should be investigation?
problem? a. Stool culture and sensitivity
a. Spondylosis b. Stool detailed report
b. Spondylolisthesis c. CBC
c. Pott disease d. Blood culture
d. Retroisthesis Ans: B
Ans: B 48) Pancreatic secretion compromised.
40) Apex of the heart is supplied by: What will happen in relation to
a. Anterior interventricular artery (LAD) absorption and digestion?
b. Diagonal a. Failure to digest triglycerides
c. RCA marginal artery b. Failure to form micelles in the intestine
d. Posterior descending artery c. Lactose intolerance
Ans: A d. Secretory diarrhea
41) Midbrain attached to cerebellum by: Ans: A
a. Superior cerbellar peduncle 49) Bile salt absorption compromised by
b. Inferior cerbear peduncle damage to:
c. Middle cerebral peduncle a. Jejunum
Ans: A b. Colon
42) Severe diarrhea. hylonatremia, c. Ileum
hypokalemia, hypocholremia. Resp d. Stomach
rate 22 Bicarb is normal. Abnormality: Ans: C
a. Metabolic acidosis 50) Contents from cecum to ileum are
b. Metabolic alkalosis prevented by:
c. Respiratory alkalosis a. Ileocecal valve
SK Origional – Golden 11 348

b. Ileocecal sphincter Ans: A


c. Ileocecal pump 58) Hypertelorism, oblique palpebral
d. Pyloric sphincter fissure:
Ans: A a. Down
51) Primordial germ cells derived from: b. Turner
a. Ectoderm c. Klienfelter
b. Endoderm d. Fragile X syndrome
c. Mesoderm Ans: A
d. Neural crest cells 59) Insulin independent uptake by:
Ans: A a. Exercising skeletal muscles
52) Theophylline, isopranaline side effect: b. Resting skeletal muscles
a. Tachycardia c. Myocardium
b. Bradycardia d. Hypothalamus
c. Hypertension Ans: A
d. Hyperglycemia 60) Right coronary artery after giving off
Ans: A marginal branch supplies:
53) Scurvy also causes: a. AV node
a. Pericardial effusion b. SA node
b. Chyothorax c. Left atrium
c. Hemopericardium d. Left ventricle
d. Pleural effusion (transudative) Ans: A
Ans: C 61) Left circumflex blocked. What
54) Specific appropriate diagnosis of affected?
primary gout by: a. Left atrium and left ventricle
a. Urate crystals in the joints b. Right atrium
b. Plasma urate c. Right ventricle
c. Urine urate d. Right atrium and right ventricle
d. Urate creatine ratio Ans: A
Ans: A 62) Regarding coronary blood supply
55) Ligamentum arteriosum connects: which statement is true?
a. Left pulmonary artery to aorta a. Occurs in systole
b. Pulmonary vein to aorta b. Independent of heart rate
c. Ductus arteriosus to aorta c. Controlled by sympathetic nervous
Ans: A system
56) Child vaccination. Hb. WBC was d. Remains same during Exercise
normal. Petechie developed. Now e. 5% of cardiac output
bleeding. Platelets decreased. Large Ans: E
platelets seen: 63) Skeletal muscles seen on light
a. ITP microscopy. What is likely to appear?
b. Hemophilia a. Mononuclear
c. TTP b. Have branching fibers
d. HUS c. Light and dark stripes
e. VWD d. No light and dark stripes
Ans: A Ans: C
57) A lady having immune problems for 6 64) Skin of the anus drains to:
months. Now developed lymphoma a. Medial horizontal group of superficial
with lymph node in neck. What inguinal lymph nodes
happened? b. Lateral horizontal group of superficial
a. AIDS inguinal lymph nodes
b. SCID c. Vertical group of lymph nodes
c. Syphilis d. Para aortic lymph nodes
d. CMV Ans: A
SK Origional – Golden 11 349

65) Most common glial cells in brain b. Submental


white matter? c. Jugulodigastric
a. Oligodendrocytes d. Supraclavicular
b. Fibrous Astrocytes Ans: A
c. Ependymal cells 73) Sensory supply on the lateral side of
d. Schwan cells scalp and also temporal and parietal
Ans: A bones?
66) Peripheral nervous system consists of: a. Auricotemporal nerve
a. Spinal nerves only b. Greater occipital nerve
b. Spinal nerves and sympathetic chains c. Suortrochlear nerve
c. Spinal nerves and ganglia d. Supraorbital nerve
d. Spinal nerves and cranial nerves Ans: A
Ans: D 74) Diabetic person with sensory
67) Microglial cells: abnormalities of foot developed ulcer
a. Are known as spider cells of foot:
b. Found in grey matter of CNS a. Microangipathy
c. Found in white matter of CNS b. Angiopathy and neuropathy
d. Macrophages of CNS c. Thrombosis
Ans: D d. Atherosclerosis
68) Reversible change in size, shape: Ans: B
a. Dysplasia 75) Epinephrine decreases:
b. Metaplasia a. Heart rate
c. Anaplasia b. Contractility of heart
d. Neoplasia c. Basal metabolic rate
Ans: A d. Cutaneous vascular dilation
69) Lymph drainage from skin behind T10 e. Serum glucose concentration
vertebra to: Ans: D
a. Axillary lymph nodes 76) Colonic carcinoma marker:
b. Posterior mediastinal nodes a. Carcino embryonic antigen
c. Superficial inguinal nodes b. CA 125
d. Deep inguinal lymph nodes c. CA 19-9
Ans: A d. CA 15-8
70) Infection between investing layer of e. AFP
deep cervical facia and pretracheal Ans: A
fascia goes to: 77) Extracellular matrix bound to cell
a. Superior mediastinum through cell membrane via?
b. Posterior mediastinum a. Cadherin
c. Middle mediastinum b. Intermediate filament
d. Anterior mediastinum c. Integrin
Ans: D Ans: C
71) To delineate swelling in neck that 78) Heart muscle works as syncytium
moves with swallowing: because of:
a. X-ray neck a. Sarcomere
b. Fnac b. SER
c. Thyroid scan with radioisotopes c. Gap junction
d. Ultrasound of swelling d. Tight junction
e. T4 estimation e. Intercalated disk
Ans: D Ans: C
72) Ulcer on left of anterior 2/3rd of 79) A post-operative patient presented in
tongue. Proved to be neoplasia. What OPD with oozing greenish discharge
is Drainage? from Wound. Which of following is
a. Submandibular causative agent?
SK Origional – Golden 11 350

a. Bacteriods b. Internal capsule


b. E.coli c. Left temporal
c. Lesteria d. Right temporal
d. Pseudomonas Ans: B
e. Klebseila 87) Mid shaft humerus fractured. Hand is
Ans: D flexed. Sensations dimished in lateral
80) A 60 year male e fracture on x-ray half of dorsum of hand:
osteoporosis is diagnosed cause of a. Radial
osteoporosis is: b. Median
a. Nutritional deficiency c. Ulnar
b. Endocrinology disturbance/hormonal Ans: A
insufficiency 88) The maximum effect (emax) achieved
c. Disuse atrophy by a drug is a measure of its:
d. Diminished blood supply a. Bioavailability
Ans: B b. Efficacy
81) Erythropoietin is stimulated by: c. Intrinsic activity
a. Hypoxia d. Potency
b. Anemia e. Safety
c. Erythropoietin Ans: B
d. Low blood volume 89) Women with obesity and myxedema.
Ans: A Has less sensations on lateral side of
82) Common bile duct relation to hand (till half of ring finger) thinner
pancreatic head: eminence affected:
a. Posterior a. Carpal tunnel syndrome
b. Lateral b. Cuboidal tunnel syndrome
c. Medial c. UMNL
d. Anterior d. Multiple sclerosis
Ans: A Ans: A
83) Splenic artery course throughout: 90) A 2 year old child presented with
a. Along superior border of neck and body midline swelling just below the hyoid
of pancreas posteriorly bone. The swelling was noticed 3
b. Anteriorly to neck and body of pancreas months of age and is slowly increasing
c. Anterior to head of pancreas in size, which means no protrusion of
d. Inferior to head of pancreas tongue the condition most likely is?
Ans: A a. Accessory thymic tissue
84) Diminished knee reflex: b. Bronchial cyst
a. L3 – L4 c. Cranio pharyngioma
b. L4 – L5 d. Ectopic thyroid gland
c. L5 – S1 e. Thyroglossal duct cyst
d. C5 – T1 Ans: D
Ans: A 91) During hysterectomy, ureter is
85) Fibres that connect lateral geniculate damaged while ligating which artery
nucleus of visual pathway to occioital close to ischial spine?
cortex: a. Internal pudendal artery
a. Geniculocalcarine tract b. Uterine artery
b. Corticospinal c. Obturator artery
c. Spinocerebellar d. Femoral artery
Ans: A Ans: B
86) Homonymous hemianopia. Right side 92) If contents go into processes
hemiparesis, Babinski sign positive. vaginalis, and through the spermatic
Damage to: cord, what hernia?
a. Left frontal lobe a. Direct inguinal hernia
SK Origional – Golden 11 351

b. Umbilical hernia b. Fetal adrenal


c. Femoral hernia c. Placenta
d. Indirect inguinal hernia d. Ovary
Ans: D Ans: C
93) During surgery, direct inguinal hernia 100) Cut finger by knife. What will mediate
is identified by: inflammatory reaction?
a. Medial to inferior epigastric artery a. Histamine
b. Lateral to inferior epigastric artery b. Bradykinin
c. Posterior to inferior epigastric artery c. Serotonin
d. Anterior to inferior epigastric artery d. No
Ans: A Ans: A
94) Regarding newborn: 101) Aptt. normal PT is deranged. What is
a. C shaped vertebral column affected?
b. Thorax in cross section is oval a. Intrinsic pathway
c. Upper limbs not fully developed b. Extrinsic pathway
d. Small thymus c. Both
e. External ear remains same d. None
Ans: A Ans: B
95) Scenario of anemia. Hb 6.8. MCV 59 102) Patient has pain in calf when walking.
MCH 18. MCHC 27. PCV 20. Platelets Relieved by resting. Artery affected:
256. WBC 6.5 Presented to opd with a. Anterior tibial artery
lethargy and pallor: b. Posterior tibial artery
a. Iron deficiency anemia c. Common tibial artery
b. Anemia of chronic disease d. Dorsal pedis artery
c. Thalassemia trait e. Popliteal artery
d. Sideroblastic anemia Ans: E
Ans: A 103) Example of drug-receptor interaction:
96) Most sensitive test for amoebic a. Bradycardia by atenolol
abscess: b. Diuresis by mannitol
a. CT scan c. Protamine sulphate antagonism to
b. Stool test heparin
c. Serology (hemagglutinin) Ans: A
d. Urine examination 104) First response to vessel injury:
Ans: C a. Activation of intrinsic pathway.
97) Thyroglobulin released by: b. Activation of extrinsic pathway
a. Follicular cells c. Vasoconstriction
b. Medullary cells d. Platelet aggregation
c. Papillary cells Ans: C
d. Para follicular cells 105) Renin function:
Ans: A a. Vascular contraction by angiotensin – II
98) When needle is passed into tunica b. Increase in volume by ADH
vaginalis which structure is not c. Decrease in volume by aldosterone
pierced? Ans: A
a. Dartos fascia 107) The most common organism amongst
b. Cremasteric fasica the following that causes acute
c. Skin meningitis in an AIDS patient with
d. Tunica albuginia Halo around nucleus:
Ans: D a. Streptococcus pneumonia
99) Progesterone and estrogen is b. Streptococcus agalactiae
produced in last 7 months of c. Cryptococcus neoformans
pregnancy by: d. Listeria monocytogenes
a. Corpus leuteum Ans: C
SK Origional – Golden 11 352

108) Patient in abducted hand is asked to 114) A stab just above the xiphisternal joint
move hand slowly and smoothly at sixth costal cartilage just to right
downward but he dropped hand will damage which structure:
suddenly by his side. Which muscle is a. Right ventricle
most likely torn? b. Right atrium
a. Deltoid c. IVC
b. Supraspinatus d. Liver
c. Subscapularis Ans: B
d. Teres major 115) The lumen of uterine cavity is
e. Pectoral obliterated by fusion of:
Ans: A a. Decidua basalis n spongy layer
106) Normal amount of iron in adult b. Decidua basalis n decidua capsularis
female: c. Decidua capsularis n decidua parietalis
a. 1 gram d. Decidua parietalis n decidua basalise)
b. 2 gram Ans: C
c. 3 gram 116) A player has suffered fracture of right
d. 4 gram first rib. Which of the following
Ans: B arteries will be damaged?
109) Asthma affects: a. Subclavian
a. Medium size bronchiole b. Brachiocephalic
b. Medium size bronchus c. Axillary
c. Respiratory bronchiole Ans: A
d. Tertiary bronchioles 117) Rib notching on x-ray is feature of:
Ans: A a. Preductal coarctation
110) Absence of cartilage in respiratory and b. Postductal coractation
terminal ducts differentiates it from: c. PDA
a. Conducting bronchiole d. VSD
b. Terminal bronchiole Ans: B
c. Alveoli 118) Extent of invasion depends upon:
d. Trachea a. Staging
Ans: D b. Grading
111) Patient had RTA injury at the level of c. Pleomorphism
T10. o/e he was pale and hypotensive. d. Dysplasia
Organ likely damaged is: Ans: A
a. Stomach 119) Vessel of the base of bladder lie in
b. Spleen which ligament?
c. Liver a. Puboprostatic
d. Kidney b. Ventrolateral
Ans: B c. Dorsolateral
112) Structure passing posterior to d. Median umbilical
diaphragm: e. Illiofemoral
a. Aorta, azygous, thoracic duct Ans: C
b. Esophagus vagus 120) Regarding muscle connective tissue
c. Vena cava which of following is true?
d. Hemiazygous esophagus vagus a. Desmin
e. Splanchnic vessels b. Vimetin
Ans: A c. Keratin
113) Thoracic aorta passes posterior to: d. None
a. Median arcuate ligament Ans: B
b. Lateral arcuate ligament 121) A patient head injury to tibial part of
c. Medial arcuate ligament sciatic nerve but he is still able to do
Ans: A flexion on the knee because of:
SK Origional – Golden 11 353

a. Short head of biceps femoris


b. Long head of biceps femoris 129) Injection given 2cm below & lateral to
c. Gastrocnemius pubic tubercle will damage:
Ans: A a. Obturator nerve
122) Rheumatic fever involves which of b. Femoral nerve
following valves? c. Sciatic nerve
a. Mitral and aortic valve d. Tibial nerve
b. Mitral and tricuspid valve Ans: A
c. Pulmonary only 130) Kaplan - Meier curve indicates:
d. Tricuspid only a. Incidence of disease
Ans: A b. Demography
123) Factor for co enzyme A: c. Severity of disease
a. Biotin d. Drug side effects
b. Pantothenic acid e. Survival
c. Tracheostomy done at Ans: E
d. C2 131) A lady has bicornuate uterus which
e. C3 system is involved?
Ans: B a. Limb
124) Labyrinth artery is a branch of: b. Renal system
a. Internal carotid c. Cardiovascular system
b. Anterior cerebral d. CNS
c. PICA Ans: B
d. AICA 132) Nerve supply of levator ANI:
Ans: D a. S4
125) A patient has AIDS but does not wish b. Pudendal nerve
to inform his wife and asks you not to c. Perineal nerve
tell her, you should? d. Obturator nerve
a. Persuade him to tell his wife, and if he Ans: B
doesn't tell her your self 133) Malignant tertian malaria caused by:
b. Advise contraception a. P. falciparum
c. Discharge the patient b. P. vivax
Ans: A c. P. ovale
126) Most likely about mammary glands: d. P. malariae
a. Extend vertically from 2 to 6 ribs Ans: A
b. 15 to 20 lactiferous ducts 134) A patient has enlarged parotid gland
c. Sebaceous glands in areola with pain in this region. Which nerve
d. In pregnancy contains lactiferous ducts is carrying pain fibers from the parotid
and secretory alveoli gland?
Ans: B a. Auriculotemporal nerve
127) Decrease proprioception in lower leg b. Glossopharyngeal nerve
association: c. Facial nerve
a. Nucleus gracilis d. Temporal nerve
b. Fasciculus gracilis e. Greater Auricular nerve
c. Nucleus cutaneous Ans: A
Ans: B 135) Dysplasia is recognized by:
128) Structure passing anterior to uncinate a. Dyskeratosis
process of pancreas: b. Loss of polarity and architecture
a. Aorta c. Pleomorphism
b. Superior mesenteric vessels d. Mitotic figures
c. Splenic vein e. Invasion
d. Inferior vana cava Ans: B
Ans: B 136) Grading of carcinoma shows:
SK Origional – Golden 11 354

a. Dysplasia Ans: B
b. Permeability to basement membrane 143) Nasopharyngeal carcinoma is most
c. Mitosis inside cell commonly associated with:
d. Invasion a. HPV
e. Degree of differentiation b. Hep.B
Ans: E c. Aflatoxin
137) During thyroidectomy, inferior thyroid d. EBV
artery is ligated. The nerve likely to be Ans: D
damaged is: 144) Trypsinogen is activated to trypsin by:
a. External laryngeal nerve a. Dec. PH
b. Recurrent laryngeal nerve b. Inc. PH
c. Internal laryngeal nerve c. Enterokinase
Ans: B d. None
138) Initiation of action potential in Ans: C
skeletal muscle by stimulating its 145) After few weeks of delivery regression
motor nerve: of enlarged uterus & breast occurs due
a. Require the release of acetylcholine to:
b. Require temporal facilitation a. Cell membrane
c. Require spatial facilitation b. Endoplasmic reticulum
d. Require the release of NE c. Golgiappratus
Ans: A d. Lysosomes
139) If a foot is permanently Doris flexed e. Mitochondria
and averted which nerve is most likely Ans: D
damaged? 146) Temperature increases above set
a. Tibial nerve point, what will decrease?
b. Common peroneal nerve a. Shivering
c. Femoral nerve b. Sweating
d. Superficial peroneal c. Vasoconstriction
Ans: A d. None of above
140) What is the landmark for pudendal Ans: A
nerve block? 147) Pillar of medical ethics:
a. Sacral promontory a. Equality justice
b. Ischial spine b. Treatmentfollows up
c. Ischial tuberosity c. Equality,justice,confidelity
d. Pubic tubercle d. Autonomy, Justice, beneficence, non-
Ans: B malfeince
141) 0.5 gram of albumin/100 ml is present Ans: D
in which fluid? 148) Lymphoma in AIDS patient is caused
a. Lymph by which virus?
b. Bile a. EBV
c. Pancreatic juice b. CMV
d. Gastric juice c. HSV
Ans: A d. Influence
142) After total gastrectomy pt who Ans: A
receives no further treatment will 149) A lesion in the arcuate fasciculus
usually develop? would cause what?
a. Symptoms to dec. abs of iron a. Sensorineural deafness
b. Symp due to dec. abs of Vit. B12 b. A global aphasia
c. Exaggerated fluctuations in plasma gluc c. Anarthria
levels after meals d. Conduction aphasia
d. Exaggerated fluctuations in blood volume e. Sound localization problems
e. All of the above Ans: D
SK Origional – Golden 11 355

150) In atopic asthma antigens binds to: 157) Regarding lymphocytes:


a. Mast cell a. Increase in bacterial infection
b. IgE b. Not affected by hormones
c. Einosinophils c. Are monocytes
d. Basophils d. Inc. in viral infection
e. Alveolar epithelium Ans: D
Ans: A 158) Child with unilateral hydronephrosis.
151) After RTA right leg injured. Swollen Site of obstruction:
and warm and crepitus. What type of a. Post ureteral stricture
shock? b. PUJ
a. Hypovolemic c. VUJ
b. Cardiogenic Ans: B
c. Neurogenic 159) During exercise blood flow to which
d. Toxemic shock organ increases?
Ans: D a. Skeletal muscle
152) In hanging death occurs due to b. Kidney
fracture of: c. Heart
a. Base of skull d. Brain
b. Odontoid process of C2 Ans: A
c. Body of atlas 160) Blood supply of gall bladder is:
d. Pedicle of C2 vertebra a. Superior pancreatic duodenal artery
e. C6 vertebra b. Sup mesenteric artery
Ans: B c. Cystic artery
153) Cardiac markers after 4 hours of MI: d. Right hepatic art
a. CKMB e. Left hepatic artery
b. LDH Ans: C
c. Troponin 161) A Urologist doing transurethral
d. Myoglobin surgery, 2 arteries at 5 and 7o' clock
Ans: C position started bleeding and his way
154) A 56yo male presents with persistent blocked. Name of those arteries?
watery diarrhea. What is the most a. Floch's artery
likely dx? b. Badenoch arteries
a. Treponema pallidum c. Branches of superior vesicle artery
b. Nesseria meningitides d. Capsular artery
c. Vibro cholera Ans: B
d. Staph aureus 162) A 6 weeks old infant presents with
e. Pseudomonas aeruginosa jaundice on exam diaper is yellow
Ans: C stained:
155) Innate immunity achieved by: a. Gilbert syndrome
a. Monocytes b. Biliary atresia
b. Macrophages c. Crigller najar
c. Hepatocytes d. Physiologic
d. NK cells Ans: B
Ans: D 163) Post sympathetic and autonomous
156) The substance used to measure total ganglia secrete:
body water is: a. Epinephrine
a. Thiosulfate b. Nor epinephrine
b. Inulin c. Acetylcholine
c. Antipyrine d. Catecholamine
d. Albumin Ans: C
Ans: C
SK Origional – Golden 11 356

164) 2nd Heart Sound occur at the 171) PO2= 50 and Hb=9, which type of
beginning of which phase of cardiac hypoxia?
cycle? a. Anemic hypoxia
a. Atrial systole b. Hypoxic hypoxia
b. Rapid ventricular filling c. Stagnant
c. Reduced ventricular filling d. Histotoxic
d. Isovolmetric contraction Ans: B
e. Isovolumetric ventricular relaxation 172) Bergers disease is associated with:
Ans: E a. Asbestosis
165) GVE of vagus arise from: b. Silicosis
a. Dorsal nucleus c. Anthracosis
b. Nucleus ambigus d. Smoking
c. Nucleus soliterus e. Berryless
d. Trigeminal nucleus Ans: D
Ans: A 173) Regarding fat embolism syndrome:
166) Gall bladder pain over neck due to: a. Mostly due to fracture of long bone
a. Sensory supply from mid cervical b. 1-3 days after initiating event
dermatome c. Most patients survive after treatment
b. Sensory supply from T10 dermatome Ans: A
c. Sensory supply from splanchnic 174) A child present with occular
Ans: A anomalies, corneal defect, lense
167) Right bronchial artery is single in it is prolapsed. Diagnosis?
a branch of? a. Potter syndrome
a. Arch of aorta b. Trisomy 21
b. Thoracic descending aorta c. Trisomy 13
c. Right 2ndintercostal artery d. Trisomy 18
d. Right 3rdintercostal artery Ans: C
Ans: D 175) Arrangement of chromosomes on
168) Patient presented with jaundice equatorial plate:
anemia raised ALP cause is: a. Prophase
a. Hepatitis b. Metaphase
b. Pigmented stone c. Prometaphase
c. Ca pancreas d. Telophase
d. Cholangitis Ans: B
Ans: B 176) Exopthalamos in thyroid eye disease
169) Which hormone always acts through is due to:
cAMP second messenger system? a. Fat accumulation
a. TSH b. Blood
b. Dopamine c. GAGs
c. Glucocirticoid d. Proteins
d. ADH Ans: C
Ans: A 177) Patient IVC blocked just above the
170) In the first week of typhoid fever, emergence of azygous vein, where
which is the most probable diagnostic blood will divert:
test? a. Left gastric vein
a. Widal test b. Hepatic vein
b. Blood culture c. Left ascending lumbar vein
c. Typhidot d. Umbilical vein
d. Stool culture e. Splenic vein
e. Urine culture Ans: A
Ans: B 178) Diagnostic investigation for
hemophilia investigation needed:
SK Origional – Golden 11 357

a. aPTT
b. Factor 8 assay
c. BT
d. CT
Ans: B
179) Muscles that elevate the palate:
a. 4thand 6tharch
b. 3rdand 6tharch
c. 1stand 2ndarch
d. 2ndand 3rdarch
Ans: A
180) The most commonest hospital
acquired organism for UTI:
a. E. Coli
b. P-auriginosa
c. S-aureus
d. Citrobacter species
e. Klebsella species
Ans: A
181) Posterior fibers of temporalis
contraction causes:
a. Retraction of mandible
b. Elevation of mandible
c. Chewing
d. Depression of mandible
Ans: A

===============
SK Origional – Golden 11 358

ALL ANESTHESIA

September – November – 2020 Papers


SK Origional – Golden 11 359
ANESTHESIOLOGY d. 20 Gauge
th
September 29 , 2020 – Morning e. 22 Gauge
Ans: D
7) Pain of Mumps is transmitted through
1) Muscle relaxant that can be given to
which nerve?
asthmatic patient:
a. Cisatracurium a. Auriculotemporal Nerve
b. Greater Auricular Nerve
b. Atracurium
c. Superior Laryngeal nerve
c. Succinylcholine
d. Facial nerve
d. Suxamethonium
Ans: B( Grey‟s Anatomy)
Ans: A
8) Hb is 10g, O2 saturation 90%, PaO2
Explanation: According to applied therapeutics
60mmhg, calculate oxygen content:
there‘s no release of histamine by cisatracurium
a. 7
that‘s why it‘s considered safe in Asthmatic
Patients b. 10
c. 13
2) What is the first line treatment of
d. 21
gastroparesis in a long term diabetic
e. 50
patient:
Ans: C
a. Aspirin
Explanation:
b. Odensteron
c. Metoclopramide  To make it simple use the following
d. Metoprolol formula:
Ans: C  Oxygen Content = Hb * 1.34
3) Which of the following is used for  Oxygen Content = 10 * 1.34
spinal Anesthesia in a pregnant lady?  Oxygen Content = 13.4
a. Bupivacaine 9) Which of the following is a prod rug?
b. Rupvicaine a. Enalapril
c. Lidocaine b. Clonidine
d. Adrenaline c. Salmeterol
Ans: B d. Acetazolamide
Explanation: Rupvicaine is used for spinal Ans: A
Anesthesia in a pregnant lady as its less cardio Explanation: ACEI drugs are mostly prodrugs
toxic. 10) Hypertension is managed
4) Hydrocephalus most common cause: intraoperative by which of the
A. Aqueduct Stenosis following drug?
B. 4th Ventricular Obstruction a. IV Hydralazine
C. Lateral Ventricles Obstruction b. IV Nitroglycerine
D. None c. IV Steroids
Ans: A d. ACEI
5) Most common drug causing grade 4 Ans: A
Encephalopathy: Explanation: Hypertension is managed quickly
a. Paracetamol with IV hydralazine while if asthmatic patient with
b. Aspirin IHD then we pick IV nitroglycerine.
c. Diuretics 11) Drug interfering with glycemic control:
d. ACEI a. Hydrochlorothiazide
Ans: A b. Spironolactone
6) If radial artery is chosen to monitor c. Aldosterone
intra-arterial pressure in adult, the d. Renin
appropriate size Cannula in order to Ans: A
avoid artifacts is: 12) Management of severe Ulcerative
a. 14 Gauge colitis:
b. 16 Gauge a. IV steroids
c. 18 Gauge b. Sulfasalazine
SK Origional – Golden 11 360
c. Beta Blockers d. Right Ventricle
d. Aluminium Hydroxoide Ans: A
Ans: A 20) Highest A-V Oxygen difference occurs
Explanation: IV steroids for severe cases of at rest in which of the following:
Ulcerative Colitis while for mild to moderate cases a. Kidney
Oral amino-salicylates (sulfasalazine‘s) are used. b. Liver
13) MOA of tramadol: c. Heart
a. Strong Meu agonist d. Skeletal Muscle
b. Weak Meu agonist e. Skin
c. Kappa Agonist Ans: C
d. Meu Antagonist 21) Goblet cells & cartilage extends in
Ans: B respiratory tree upto:
Explanation: Tramadol is weak meu agonist while a. Bronchi
morphine is strong meu agonist. b. Terminal Bronchioles
14) MOA of cabergoline: c. Respiratory Bronchioles
a. D1 Agonist d. Alveolar Sac
b. D2 Agonist Ans: A
c. D1 Antagonist Explanation: Cartilage & Goblet Cells extends
d. D2 Antagonist upto bronchi (Ref:FA Page # 608)
Ans: B 22) Which of the following fluid can be
15) MOA of Odensterone: given to increase ECF & decrease
a. 5HT3 Blocker ICF?
b. Beta 2 Receptors Blockers a. 3% N/S
c. D2 Antagonist b. 0.9% N/S
d. D1 Antagonist c. 0.45% N/S
Ans: A d. 5% D/W
16) Short acting anti diabetic drug: Ans: A
a. Tolbutamine Explanation: Among the mentioned options 3%
b. Glargine N/S is hypertonic solution that will increase ECF
c. Bovine volume & then water will start moving from ICF
d. Ultralente to ECF thus the end result will be Increase ECF &
Ans: A decrease ICF
17) Nerve traversing superficial inguinal 23) Barret esophagus can cause which of
ring: the following carcinoma:
a. Ilioinguinal Nerve a. Adenocarcinoma
b. Pudendal Nerve b. Squamous cell Carcinoma
c. Obtruator Nerve c. Melanoma
d. Sciatic Nerve d. Small cell carcinoma
Ans: A Ans: A
18) Best site for lumber puncture: 24) Middle ear ossicles function:
a. Above L4 a. Conduction of sound
b. Below L4 b. Amplification of Sound
c. Below L3 c. Increase Sound pressure Level
c. Above L3 d. Increase distance of sound waves
Ans: B Ans: A
Explanation: Site for Lumber Puncture is L4-5, 25) To measure accurate BP, cuff size
here best site for LP is Below L4 (width) should be how much times of
19) Stab Injury at 6th intercostals space on arm circumference:
the right side of the sternum will pierce a. 0.5 – 1.5 Times the diameter
which of the following structure? b. 1.0 -1.5 Times the diameter
a. Right Atrium c. 1.5 – 2.0 times the diameter
b. IVC d. 2.0 – 3.0 times the diameter
c. SVC e. 6.0 – 10.5 times the diameter
SK Origional – Golden 11 361
Ans: A d. L4
26) Drug for SAH (Subarachnoid e. L5
hemorrhage): Ans: D
a. Nifedepine Explanation: L4 dermatome supplies the medial
b. Nimodipine side while L5 the lateral side
c. Captopril 31) A 35 year old male laborer presented
d. Acetazolamide with acute sudden pain starting from
Ans: B the lower lumber are and radiating
27) A child aspirated a peanut while he along the posterior aspect of the left
was lying in supine position, the lower limb, his symptoms started after
peanut will most probably lodge in: lifting heavy container 03 days back,
a. Apical segment of right lower lobe on examination straight leg raising is
b. Posterior segment of right lower lobe restricted to only 30 degree on the left
c. Anterior segment of right lower lobe side decreased sensation along lateral
d. Posterior segment of right upper lobe boarder of left foot weakness of left
Ans: A flexor hallucis longus and diminished
Explanation: ankle jerk this patient has compression
 Aspiration of FB in supine position will go of which of the following:
into superior/Apical bronchopulmonary a. S1 root
segment of right upper lobe b. S3 root
 Aspiration of FB in standing position will c. S4 root
go into posterobasal segment of right d. S5 root
lower lobe e. Sciatic nerve
 Aspiration of FB in right sided position Ans: A
will lodge in posterior bronchopulmonary 32) H2 blockers given pre operatively to:
segment of right upper lobe a. To decrease Acid secretion
28) In determining rate of flow through an b. To decrease volume secretion
orifices depends on which of the c. To decrease both acid & volume secretion
following properties of the gas: d. To decrease motility
a. Conductivity Ans: C
b. Critical temperature 33) Main advantage for using esmolol in
c. Density anesthetic practice is due to:
d. Radioactivity a. No effect on intracranial pressure
e. Viscosity b. No rebound hypertension
Ans: C c. Reduction in plasma renin activity
29) Concomitant use of monoamine d. Selective beta blocker activity
oxidase inhibitor and the following e. Short duration of action
opioid has been most common Ans: E
association with fetal excitation 34) MOA of Local Anesthetic:
reaction: a. Blocks Na+ Channel
a. Alfentanyl b. Blocks K+ Channel
b. Fentanyl c. Block Ca++ Channel
c. Morphine d. None
d. Pethidine Ans: A
e. Sufentanil 35) Cimetidine time of action if given
Ans: D intravenously:
30) A young lady complains of lower limb a. 10 – 45 Minutes
pain radiating to the back of the thigh b. 45 – 60 Minutes
below knee joint upto medial melleolus c. 60 – 90 Minutes
which root value of nerve involved: d. More than 90 Minutes
a. L1 e. Within 5 minutes
b. L2 Ans: A
c. L3 Explanation:
SK Origional – Golden 11 362
 Time of Action: 10 – 45 Minutes a. Treated with hydrocortisone
 Peak of action: 45 – 90 Minutes b. Given as 2% solution to avoid adverse
36) Lymphatic drainage of external nose & effect
anterior part of nasal cavity: c. Causes convulsions before cardiac toxicity
a. Submandibular Lymph Nodes for several hours
b. Deep cervical lymph nodes d. Perioral parasthesia
c. Retropharyngeal Lymph nodes Ans: D
d. Occipital Lymph Nodes Explanation:
Ans: A  Initial : Perioral Parasthesia
Explanation:  Later on: Convulsion
 External nose & Anterior nasal cavity 42) Patient of thyrotoxicosis presented
drained by submandibular lymph nodes with decreased visual acuity and visual
 Posterior Nasal cavity drained by field defects, likely cause would be:
retropharyngeal lymph nodes & JD nodes. a. Exopthalamos
37) Fibers after decussating in medulla are b. Occulomotor nerve damage
called: c. Optic Nerve compression
a. Pyramid d. Occulomotor nerve palsy
b. Leminscus Ans: C
c. Callosum Explanation: Eye discomfort, grittiness, tear
d. Splenium production, photophobia, diplopia, decreased
Ans: A visual acuity and afferent papillary defect may be
38) Pain, temperature & crude touch from seen with optic nerve compression in graves
face is carried by: ophthalmopathy.
a. DCML 43) Internal jugular vein joins subclavian
b. Anterolateral Spinothalamic tract vein to form:
c. Trigeminothalamic tract a. Brachiocephalic vein
d. Lateral Tract b. SVC
Ans: C c. IVC
39) Mandibular nerve passes through d. Common Jugular Vein
which foramen: Ans: A
a. Foramen Spinosum 44) Nerve supply of anterior abdominal
b. Foramen Ovale wall:
c. Foramen Rotundum a. T7 – L1
d. Foramen Magnum b. T6 – T9
Ans: B c. T4 – T5
Explanation: d. T5 – T6
 Mandibular nerve passes through Ans: A
Foramen Ovale Explanation: Lower six intercostal nerves & first
 Maxillary Nerve passes through Foramen lumber nerve supply anterior abdominal wall.
Rotundum 45) Nerve supply of rectum & anal canal:
 Middle Meningeal Artery passes through a. S2 – S4
foramen spinosum b. S1 _ S2
40) Eye Injury via ball resulting in c. L3 _ L4
numbness of cheeks which nerve is d. L5
damaged: Ans: A
a. Infraorbital nerve 46) Charles Law states that:
b. Supraorbital nerve a. The pressure of gas is inversely
c. Supratrochlear nerve proportional to volume
d. Ophthalmic nerve b. The temperature is inversely proportional
Ans: A to volume
41) Initial, most common side effect of c. The temperature of as gas is related to
Lidocaine: absolute humidity
SK Origional – Golden 11 363
d. The volume of a gas is directly Explanation: For Mild heart failure ACEI is the
proportional to temperature, pressure is drug of choice while for acute heart failure with
constant dypnea then we give furosemide.
e. The volume of a gas is directly 52) Patient in room with optimum
proportional to pressure zero environmental condition, how heat
Ans: D loss will occur:
47) A 30 years old female with no CVS or a. Conduction & Radiation
respiratory ailment is having swelling b. Convection
in front of the neck. She feels difficulty c. Sweating
in breathing while in the supine d. Breathing
position. What can the type of swelling e. Voiding
in this patient? Ans: A
a. Multinodular Goiter 53) Ankle jerk diminished nerve root
b. Follicular CA of thyroid involved:
c. Asthma a. S1 Root
d. Retrosternal Goiter b. S2 Root
e. Colloid Goiter c. S3 Root
Ans: D d. S4 Root
Explanation: Retrosternal goiter extends to e. L4 Root
trachea and causes difficulty in breathing & neck Ans: A
veins engorgement 54) In burn patient succinylcholine isn‟t
48) Initial step in phagocytosis: given due to which of the following
a. Activation of C3b effect:
b. Membrane evagination a. Hyponatremia
c. Margination b. Hypernatremia
d. Adhesion c. Hyperkalemia
Ans: A d. Hpokalemia
Explanation: Neutrophils arrival & function starts Ans: C
with Margination, rolling, adhesion, transmigration 55) Digoxin toxicity is due to:
& chemotaxis and phagocytosis, while the first step a. Hyponatremia
in phagocytosis is activation of C3b or IgG to act b. Hypernatremia
as opsonin c. Hyperkalemia
49) Drug used in Angina but doesn‟t cause d. Hypokalemia
vasodilation: Ans: D
a. Nifedipine Explanation: Both Digoxin & potassium compete
b. GTN for same site & when potassium is low then it will
c. Verapamil lead to digoxin toxicity.
d. Metoprolol 56) What will increase in serum due to
Ans: D vitamin D/function of vitamin D?
50) First observable sign of irreversible cell a. Increase serum PO4 level
injury in cardiac cell: b. Decrease serum PO4 Level
a. Clumping of nuclear chromatin c. Decrease Serum Calcium Level
b. Hydrphic change d. Increase Serum Magnesium Level
c. Cell swelling Ans: A
d. Contraction bands in cytoplasm Explanation:
Ans: D  Vitamin D increases serum PO4 &
51) First drug for mild heart failure: Calcium level.
a. Thiazide diuretics 57) Treatment of cystic acne in pregnant
b. Acetazolamide lady that lead to fetal abnormality:
c. ACEI (Captopril) a. Vitamin A
d. Renin b. Thaladomide
Ans: C c. Metronidazole
d. Vancomycin
SK Origional – Golden 11 364
Ans: A d. Maxillary nerve
Explanation: Vitamin A derivate that‘s e. Lingual nerve
isotretenoin is used for acne‘s treatment & that Ans: C
leads to fetal abnormalities that‘s why Explanation: Taste sensation from anterior 2/3rd
contraindicated in pregnancy. is carried by chorda tympani nerve while general
58) Maximum urine concentration is due sensation is carried by lingual nerve.
to: 64) In an ICU patient with CVP line in
a. Aldosterone subclavian vein for the past 10 days is
b. ANP on parentral nutrition. He developed
c. ADH fever, blood culture showed clusters of
d. AG2 gram positive cocci the infection is
Ans: C most likely caused by:
59) Sterilization method that kills the a. Staph Aureus
bacteria but not the spores increases b. Streptococcus
the chances of infection of which of c. Psudomonas
the following organism: d. Enterococci Fecalis
a. Clostridium Tetani Ans: A
b. Psudomonas Explanation: The infection is most commonly
c. Staph Aureus caused by MRSA, as staph aureus is commonly
d. Streptococcus found in hospitalized patient & found in clusters.
Ans: A 65) Characteristics of metabolic alkalosis
Explanation: C.Tetani mainly causes infection in chronic loop diuretic use:
through spores. a. Hyperkalemia
60) Which of the following causes post b. Hypokalemia
renal transplantation opportunistic c. Hpernatremia
infection? d. Hponatremia
a. HPV Ans: B
b. EBV 66) Which of the following has maximum
c. HIV cross sectional area?
d. HSV a. Capillaries
Ans: B b. Artery
Explanation: Prefer CMV if present in options c. Vein
but if not then the second virus is EBV. d. Arterioles
61) A female accidently cut her radial Ans: A
artery which mechanism will reduce 67) Hemolysis produced in acute immune
pressure in vessel by stopping the response:
bleeding? a. IgG
a. C Fibers b. IgM
b. Beta Fibers c. IgE
c. Alpha Fibers d. IgD
d. Delta Fibers Ans: B
Ans: B Explanation:
62) Lateral Sensation of foot lost, nerve  IgM is produced in acute inflammatory
involved: response.
a. S1 Root  IgG is produced in late stages of infection.
b. S2 Root  IgE is produced in parasitic & allergic
c. S3 Root conditions.
d. S4 Root 68) Anterior hypothalamus regulates
Ans: A temperature via:
63) Taste to anterior 2/3rd is carried via: a. Cutaneous vasodilation & Sweating
a. Glosopharryngeal nerve b. Cutaenous Vasoconstriction
b. External Laryngeal nerve c. Shivering
c. Chorda Tympani d. Thermogenesis
SK Origional – Golden 11 365
Ans: A c. Erythromycin
Explanation: Anterior Hypothalamus is like an d. All of the above
AC that cools the body. Ans: A
69) In pregnancy: 76) Protamine side effect:
a. Increase Tidal volume a. Bleeding
b. Increase TLC b. Hypotension
c. Decrease FRC c. Hypertension
d. VC remains unchanged d. Vomiting
Ans: D Ans: B
Explanation: In pregnancy VC remains same Explanation: Protamine sulphate in normal
while TLC decreases in late stages of pregnancy. standard dose leads to hypotension as a side effect
70) 15% blood loss should be replaced while in excess amount it can lead to bleeding
with: 77) The most likely effect of over dose of
a. 0.9% N/S tricyclic antidepressants is:
b. Ringer Lactate a. Constricted pupil
c. Blood b. Generalized Tonic/Clonic fits
d. Vasopressors c. High blood pressure
Ans: B d. Loose motion
71) Tympanic membrane retraction due e. Urinary incontinence
to: Ans: B
a. Eustachian Tube Blockage 78) In TB multiple drugs are prescribed to:
b. Adhesive otitis media a. Decrease resistance
c. Otitis Externa b. Better outcome
d. Ossicular bones rupture c. Organism more than one
Ans: A d. To increase toxicity
72) Calcium in skeletal muscle ER is Ans: A
bound with: 79) Scenario of epistaxis, hematuria,
a. Desmin saddle nose & granulomatous
b. Myosin inflammation:
c. Calmodulin a. Churg strauss syndrome
d. Calcequestrin b. Wegner granulomatosis
Ans: D c. SLE
73) Artery present at the base of renal d. HIV
pyramid: Ans: B
a. Arcuate artery 80) In less than 15% blood loss which fluid
b. Inerlobular artery should be given:
c. Inerrlobar artery a. 0.9% N/S
d. Segmental artery b. Ringer Lactate
Ans: A c. Blood
74) Corpus callosum contains which type d. FFP
of fibers: Ans: B
a. Association fibers 81) In the CVP trace the right atrial filling
b. Commissural fibers with a closed tricuspid valve is
c. Projection fibers signified by:
d. Internal arcuate fibers a. A wave
Ans: B b. C wave
75) Patient of chronic renal failure c. V wave
presented to OPD with pain and d. X descent
discharge from ear, physician advised e. Y descent
her doctor to prescribe him drug in Ans: C
lower dose: Explanation:
a. Penicillin
b. Gentamicin
SK Origional – Golden 11 366
 V wave is passive filling of the right atrium increase protein & decrease glucose
when tricuspid valve is closed while A level what will be the drug of choice:
wave is for atrial contraction pressure. a. Cefotaxime
82) Case of rheumatic fever with mitral b. Penicillin
regurgitation murmur will be heard at: c. Ceftrixone
a. Pansystolic murmur at apex d. Ampicillin
b. Diastolic murmur Ans: C
c. Systolic click Explanation:
d. Decrescendo  Ceftrixone is the drug of choice in adults
Ans: A with meningitis while in cases of neonates
83) Diuretic that alters diabetic control: cefotaxime is advised.
a. Hydrochlorothiazide  If rash is seen and N.Meningitides is
b. Amiloride suspected then can go with penicillin.
c. Renin
d. Angiotensin 89) If ventilation of alveoli is stopped
Ans: A because of bronchiolar blockage what
84) Platelets release: will be PO2 in blood:
a. TxA2 + ADP a. Equal to mixed venous blood
b. PG b. Less than mixed venous blood
c. Kinin c. Equal with arterial blood
d. Steroids d. Equal with atmospheric Oxygen
Ans: A Ans: A
85) You will be most worried of the 90) Insoluble form of iron in overdose
following effect of suxamethonium state:
after a second dose administration: a. Ferritin
a. Bradycardia b. Hemosidern
b. Malignant hyperthermia c. Transferrin
c. Nausea d. Ferric form
d. Fasciculation Ans: B
Ans: B 91) Which muscle abduct the vocal cord?
86) Liver makes energy by: a. Posterior cricoarytenoid
a. Glycogenolysis b. Thryoarytenoid
b. Gluconeogensis c. Vocalis
c. Protein anabolism d. Thyropharyngeus
d. Fat anabolism Ans: A
Ans: A Explanation: Posterior cricoarytenoid is the only
87) A 48 years old lady at last stage of Ca muscle that abducts vocal cords.
fibroid and she tells you not to tell her 92) Phonation is caused by:
family, her son asks about her mother a. Tightening of vocal cords
condition & disease what you will do? b. Loosening of vocal cords
a. Tell something c. Vibration of vocal cords
b. Tell Nothing d. Nasal turbulence
c. Inform the son Ans: C
d. Inform the whole family 93) Type III hypersensitivity occurs in:
Ans: B a. SLE
Explanation: According to medical ethics you b. Graves
can‘t tell anything to anyone until the patient c. IDA
doesn‘t allow doing so, only in few exceptional d. TB
cases the information can be shared Ans: A
88) A 28 years old patient was diagnosed 94) Pulse oximeters:
as meningitis for having high grade a. Are based on boyle‘s law
fever and CSF examination showed b. Can derive carbon dioxide levels
SK Origional – Golden 11 367
c. Can differentiate deoxyhemoglobin from  Deficiency of Niacin (B3) leads to 3D
oxyhemoglobin that‘s dementia, diarrhea & dermatitis
d. Can‘t differentiate methhemoglobin from  Niacin as NAD and NADP act as a
oxyhemoglobin coenzyme.
e. Use a single wave length of red light 101) When interpreting a capnogram at?
Ans: D a. Rise in baseline demonstrates hypocarbia
95) Pleurocentasis is done at: b. Rise in baseline demonstrates rebreathing
a. Lower boarder of 9th intercostals space at c. Rise in height demonstrates expiratory
midaxillary line obstruction
b. Upper boarder of 9th intercostals space at d. Slow fall in height demonstrates expiratory
midaxillary line obstruction
c. Lower boarder of 9th intercostals space at e. Slow rise in height demonstrates
midclavicular line inspiratory obstruction
d. Upper boarder of 9th intercostal space at Ans: B
midclavicular line 102) Clinically important for tumor:
Ans: A a. Staging
96) Maximum effect of local anesthetic b. Grading
depends on: c. Invasion
a. Lipid solubility d. Polymorphism
b. Water solubility Ans: A
c. PKa 103) Highest potassium loss will be
d. Diffusion capacity because of resection of:
Ans: A a. Colon
97) TRALI occurs within: b. Ileum
a. 6 hours c. Stomach
b. Immediately d. Jejunum
c. 24 hours Ans: A
d. Weeks 104) Organ that regenerates on its own:
Ans: A a. Liver
Explanation: TRALI is defined as an acute lung b. Skeletal muscle
injury that is temporally related to a blood c. Cardiac muscle
transfusion and specifically occurs within first six d. Neuron
hours after transfusion. Ans: A
98) In type 1 HSR which cells are 105) Methhemoglinemia side effect of
increased mainly: which local anesthetic?
a. Eosinophils a. Bupvicaine
b. Neutrophils b. Rupvicaine
c. Lymphoctyes c. Prilocaine
d. Mast cells d. Lignocaine
Ans: D Ans: C
99) MOA of clonidine: 106) What should be given to increase
a. Alpha 1 Antagonist vitamin in diet?
b. Alpha 2 Antagonist a. Nuts
c. Alpha 1 Agonist b. Yogurt
c. Alpha 2 Agonist c. Egg
Ans: D d. Green vegetables
100) Regarding Niacin: e. Margarine
a. Deficiency causes CNS symptoms Ans: D
b. Act as a coenzyme 107) Soldier coming back from Siachin
c. Deficiency causes graves disease having breathlessness & cyanosis due
d. Use to treat ulcers to:
Ans: A a. Secondary Polycythemia
Explanation: b. Primary Polycythemia
SK Origional – Golden 11 368
c. Pulmonary Edema condition then the fluid will come out of cell into
d. Cardiac Failure the solution & final result will be cellular shrinkage.
Ans: A 115) A drug having half-life of 10 hours,
108) Scenario related to increase urinary when will it reach steady state?
volume & normal blood glucose level: a. 10 hours
a. Diabetic Mellitus b. 20 hours
b. Nephrogenic Diabetic Insipidus c. 40 hours
c. SIADH d. 60 hours
d. ARF Ans: C
Ans: B Explanation: A drug reaches its steady state in 4-5
109) Which one decreases gastric half-lives that‘s in this case in 40 – 50 hours.
emptying? 116) Cleft lip is caused by:
a. Aluminum hydroxide a. Failure of maxillary process to fuse with
b. Magnesium hydroxide lateral nasal process
c. Alumnium chloride b. Failure of mandibular process
d. Zinc oxide c. Failure of palatine shelves
Ans: A d. Failure of nasal process
110) Endocarditis sequence of events: Ans: A
a. Bacteremia, perforation, thrombus 117) Pregnant lady is avoided to lay on right
b. Throbmus, bacteremia, perforation lateral side due to compression of:
c. Damage valves, Thrombus formation, a. Aorta
bacteremia, perforation b. Azygous Vein
d. Perforation, bacteremia, damaged valves c. IVC
Ans: C d. Ovaries
Explanation:Remember ―DTBP‖. Ans: C
111) Post mediastinal tumour can 118) Change of hemoglobin in a pregnant
compress: lady from 12 to 10.8 gram/dl during
a. Thoracic Aorta 38th week of gestational age is probably
b. Thymus due to:
c. Jugular vein a. IDA
d. Azygous Vein b. Thalassemia
Ans: A c. Sideroblastic Anemia
112) What will decrease immediately after d. Physiological change
hypoglycemia? Ans: D
a. Glucagon 119) Child having history of consumption of
b. Cortisol fast foods presented with anemia &
c. Insulin raised MCV > 104 the most probable
d. Growth Hormone cause is:
Ans: C a. Folic Acid deficiency
113) Drug used for diabetic gastroparesis: b. Vitamin B12 deficiency
a. Prokinetics c. IDA
b. H2 blockers d. Sideroblastic Anemia
c. Antacids Ans: A
d. PPI Explanation: Folic acid is found naturally in green
Ans: A leafy vegetables & due to most of fast foods
114) Hypernatremia will result in: consumption can lead to folic acid deficiency.
a. Cell swelling 120) Vitamin A deficiency can cause:
b. Cell Shrinkage a. Diarrhea
c. Cell lysis b. Dermatitis
d. Cell swelling & Shrinkage c. Epithelial damage
Ans: B d. Cardiac enlargement
Explanation: When cell like RBC is put in Ans: C
hypertonic solution that‘s hypernatremia like
SK Origional – Golden 11 369
121) Out of the following characteristics the Ans: B
most similar between pulmonary & Explanation: Vasopressin > Theophyline >
skeletal muscle capillaries is: Propronolol.
a. Capillary hydrostatic pressure 129) Transitional epithelium characteristic:
b. Capillary oncotic pressure a. Dome Cells
c. Capillary permeability b. Flat Cells
d. Interstitial hydrostatic pressure c. No change in cellular layer
e. Interstitial protein concentration d. Keratinized covering
Ans: E Ans: A
122) Cushing reflex triad: 130) Thyrotoxicosis & tachycardia drug to
a. Hypertension, Bradycardia, Increase ICP be given:
b. Hypertension, Tachycardia, Increase ICP a. Propranolol
c. Hypertension, Bradycardia, Increase ICP b. Ropinirole
d. Hypotension, Bradycardia, Decrease ICP c. ACEI
Ans: C d. Dopamine
123) Sarcomere: Ans: A
a. Between two Z lines 131) A child aspirated peanut (Position
b. Between Two I lines wasn‟t mentioned in 2nd stem) it will
c. Between Actin & Myosin lodge in:
d. Between two A lines a. Right upper lobe
Ans: A b. Right lower lobe
124) Rapid ventricular filling: c. Right middle lobe
a. 1st heart sound d. Left upper lobe
b. 2nd heart sound Ans: B
c. 3rd heart sound 132) Categorical data:
d. 4th heart sound a. Chi square test
Ans: C b. T test
125) Supraoptic nucleus lesion with loss of c. Anova
ADH will lead to: d. Paired T test
a. More loss of water in urine Ans: A
b. Less loss of water in urine 133) The end product of carbohydrate
c. Concentrated urine metabolism:
d. No effect on urine a. Fructose
Ans: A b. Glucose
126) Test which rule out the patient not c. Galactose
having disease that are true negative: d. Starch
a. Sensitivity e. Pyruvate
b. Specificity Ans: E
c. Positive predictive value 134) Malignancy of tumour is best
d. Negative predictive value demonstrated by:
Ans: B a. Invasion
127) Normal distribution curve will have: b. Metastasis
a. Same mode, median & mean c. Metaplasia
b. Different mode & mean d. Increase N/C ratio
c. No relation with mean e. Pleomorphism
d. None Ans: B
Ans: A Explanation: Malignancy is demonstrated by:
128) Which of the following drug will Metastasis > Invasion> Pleomorphism > Increase
worsen angina? N/C ratio.
a. Propranolol 135) Grading of tumour:
b. Vasopressin a. Degree of differentiation
c. Theophylline b. Extent of tumour
d. ACEI c. Metastasis
SK Origional – Golden 11 370
d. Anaplasia Explanation: After hemorrhage, baroreceptors
Ans: A will activate sympathetic & inhibit parasympathetic
Explanation: system thus decreasing venous compliance &
 Grading of tumour: Degree of increasing venous tone & CO.
differentiation 142) An old patient with history of IHD
 Staging of tumour: Extent of tumour now presented with arrhythmia, ECG
136) Ball hit child on temporal region, now shows wide QRS complex with heart
he is complaining of headache & rate of 140 – 200 beats/min best
blurring of vision what‟s damaged: treatment is:
a. Posterior branch of middle meningeal a. Lignocaine
artery b. Amiodarone
b. Anterior branch of middle meningeal c. Verapamil
artery d. Adenosine
c. Posterior branch of middle meningeal Ans: A
artery Explanation: In ventricular tachycardia drug of
d. Anterior cerebral artery choice is Amiodarone, but if IHD & VT then
Ans: B lignocaine.
137) Patient having petechiae with increase 143) Parasympathetic effect:
bleeding time what‟s deficient: a. Contracts GIT sphincters
a. Vitamin K b. Pupillary constriction
b. Vitamin C c. Pupillary dilation
c. Vitamin D d. Decrease Salivation
d. Vitamin A Ans: B
Ans: A 144) A young female with amenorrhea, wide
138) Highest chances of bleeding after web neck & broad chest, karotype:
some procedure: a. 45 XO
a. LMWH prophylaxis b. 45 XX
b. Heparin after 1 hour of the procedure c. 42 XY
c. Warfarin with normal INR d. 42 XX
d. Antiplatelet Ans: A
Ans: A Explanation: The scenario is related to turner
139) Stroke volume of a patient increased syndrome & the karotyping is 45 XO.
without change in heart rate and 145) Collagen type present in Skin:
peripheral resistance, it will lead to: a. Type 1 & 3
a. Decrease MAP b. Type 2 & 4
b. Decrease pulse pressure c. Type 1 & 2
c. Increase MAP & Pulse Pressure d. Type 4
d. No change in pulse pressure & MAP Ans: A
Ans: C 146) Dehydration increases all except:
140) MOA of PTU: a. ADH
a. Decrease thyroxin synthesis b. AG2
b. Increase thyroxin synthesis c. Baroreceptor firing
c. Increase rT3 d. Aldosterone
d. Decrease TSH Ans: C
Ans: A Explanation: Baroreceptor firing decreases during
141) Baroreceptor after hemorrhage will dehydration & hemorrhage.
reduce: 147) Emergency management of
a. Arterial Compliance hyperkalemia:
b. Venous compliance a. IV calcium Gluconate
c. Venous tone b. IV Chloride
d. Cardiac output c. IV Sodium
Ans: B d. IV lactate
Ans: A
SK Origional – Golden 11 371
148) Nuclear changes seen in cell injury 155) Which of the following is associated
include: with complete gastric resection?
a. Karolysis a. IDA
b. Exocytosis b. Vitamin B12 deficiency
c. Pinocytosis c. Folic acid deficiency
d. Autophagic vacuoles d. Sideroblastic anemia
e. Residual bodies Ans: B
Ans: A 156) Free radicals include:
149) Why is glycopyrrolate preferred over a. Glutathione peroxidase
atropine? b. H2O2
a. Glyocpyrolate has faster onset of action c. Ferric form of Iron
than atropine d. Superoxide dismutase
b. Glycopyrolate metabolizes more quickly e. Cysteine
than atropine Ans: B
c. Atropine has faster action 157) A 20 year old full term pregnant female
d. None is brought to a tertiary care hospital
Ans: A from distant village. She has been in
150) Facial muscles are supplied by: labor for the last 20 hours, while
a. Facial nerve preparation for CS were made she
b. Mandibular nerve became breathless went into shock and
c. Maxillary nerve died. The most likely cause of her
d. Trigeminal nerve death is:
Ans: A a. Air embolism
Explanation: Facial muscles are supplied by facial b. Amniotic fluid embolism
nerve while masticatory muscles are supplied by c. Fat embolism
mandibular nerve, a branch of trigeminal nerve. d. FB embolism
151) Person chronic smoker, working in e. Thromboembolism
construction factory presented with Ans: B
bilateral diaphragmatic lung lesion: 158) A soft, low pitched third heart sound
a. Silicosis auscultated in many normal
b. Asbestosis individuals due to vibrations set up by:
c. Siderosis a. Closure of Aortic Valve
d. Bysinosis b. Closure of AV valve
Ans: B c. Flow of blood into aorta
152) Diaphragmatic parietal pleura are d. Opening of semilunar valves
supplied by: e. Rapid inflow of blood into ventricles
a. Phrenic nerve Ans: E
b. Vagus nerve 159) A young surgeon operated upon a
c. Intercostals nerve middle aged lady for enlarged thyroid
d. Lumber nerve and ligated the superior and inferior
Ans: A thyroid arteries as close to the poles as
153) Kidney changes in hypovolemic shock: possible, after two weeks the lady
a. Acute tubular necrosis presented with persisting hoarseness,
b. Increase concentrated urine the most likely cause could be:
c. Increase dilute urine a. External laryngeal nerve damage
d. Increase Renin b. Laryngeal edema
Ans: A c. Recurrent laryngeal nerve damage
154) Renal capillary filtration: d. Sore throat
a. Increase with arterial constriction e. Vagus nerve damage
b. Decease with increase hydrostatic pressure Ans: C
c. Decrease with dehydration 160) Patient with suspicion of drug intake
d. Increase with dehydration presented to ER with deep respiratory
Ans: C
SK Origional – Golden 11 372
rate of 6/mins the most likely drug November 24th, 2020 - Morning
responsible is:
a. Aspirin 1) Cardiac output measure by dye
b. Morphine dilution method and dye should be
c. Nalbuphine measure in:
d. Pentazocine a. Right atrium
e. Phenobarbitone b. Aorta
Ans: B c. Saphenous vein
161) The most toxic local anesthetic agent d. Pulmonary artery.
is: Ans: B
a. Bupivacine 2) Mechanism of action of cabergoline?
b. Cocaine a. D1 agonist
c. Dibucaine b. D2 agonist
d. Lidocaine c. D3 agonist
e. Procaine d. D4 agonist
Ans: A Ans: B
162) When measuring the CVP the 3) What is mechanism of action of
reference point which best correlates to labetalol?
the right atrium is? a. Alpha blocker
a. Anterior axillary line 4th intercostals space b. Beta blocker
b. Manubriosternal junction c. Alpha and beta blocker
c. Midaxillary line 3th intercostals space d. B2 agonist
d. Midaxillary line 4th intercostals space e. Alpha 2 agonist
e. Midaxillary line 5th intercostals space Ans: C
Ans: D 4) Medial geniculate body, a thalamic
163) NSAID which inhibits prostaglandin nucleus?
synthesis by inhibiting COX-2 rather a. Lies on the mid brain
than COX-1 is: b. Is attached to the main mass of thalamus
a. Aspirin c. Receive fibers from oclomotor nerve
b. Ibuprofen d. Relays fibers to frontal lobe
c. Indomethacin e. Relays fibers to visual cortex
d. Meloxicam Ans: A
e. Piroxicam 5) While performing bronchoscopy first
Ans: D structure visualized beyond carina is?
164) The hormone which produces protein a. Upper lobe bronchus
catabolism in extra hepatic tissues and b. Superior segmental bronchus
protein synthesis in liver is: c. Apical bronchus
a. Cortisol d. Middle lobe bronchus
b. Epinephrine e. Inferior lobe bronchus
c. NE Ans: A
d. Growth hormone 6) Extreme left shift of oxygen
e. Thyroxin dissociation curve?
Ans: A a. CO poisoning
b. Increase temperature
c. Fetal HB
=============== d. Acidosis
Ans: A
7) Autopsy finding of the patient having
chronic renal failure?
a. Hypertrophied parathyroid gland
b. Hypertrophied adrenals
ANESTHESIA c. Hypertrophied pituitary
SK Origional – Golden 11 373
d. Hypertrophied thyroids d. Mannitol
Ans: A Ans: C
8) Contraindicated in renal failure? 16) Penicillin works on bacteria through?
a. Gentamycin a. Destruction of cell wall
b. Chloramphenicol b. Inhibition of cell wall synthesis
c. Vancomycin c. Protein Synthesis.
d. Penicillin d. Trans peptidase inhibitor
Ans: A Ans: B
9) Drugs which are excreted by active 17) Mycobacterium does not produce
renal tubular secretion include? endo/exotoxins then what features
a. Benzyl penicillin makes it so virulent?
b. Streptomycin a. Tuberculin
c. Ethy alcohol b. Tubuloproteins
Ans: A c. Cell wall glycoproteins
10) Young pt. developed ARF after PPH d. Resides in macrophages
which part of kidney is most likely Ans: C
damaged? 18) Glycopyrolate is preferred over
a. PCT atropine?
b. Loop of Henle a. Cause less tachycardia
c. Collecting tubules b. Cannot cross BBB
d. DCT c. Don't affect fetus
Ans: A d. Short acting
11) Due to prolonged use of diuretics Ans: B
characteristic change in blood seen is? 19) Most common side effect of H1
a. Dec. K blockers is?
b. Dec. Na a. Gastritis
c. Dec. Cl b. Sedation
d. Dec. Mg c. Bloating
Ans: B d. Cough
12) Most common thiazide side effect? Ans: B
a. Hypocalcemia 20) Which of the following is H2 blocker?
b. Hyperuricemia a. Cetirizine
c. Hyperkalemia b. Fexofenadine
d. Hypoglycemia c. Diphenhydrinate
Ans: B d. Cimetidine
13) What is site of action of Loop Ans: D
diuretics? 21) PPI Inhibits which of following pump?
a. PCT a. Na/K
b. DCT b. H/k
c. Thick ascending limb c. Ca/H
d. Collecting Duct d. Ca/K/Cl
Ans: C Ans: B
14) Which drug is used in pulmonary 22) Which one is potent bronchodilator,
edema? can be used in asthmatic patients?
a. Hydralazine a. Ketamine
b. Labetalol b. Propofol
c. Furosemide c. Thiopental
d. Nifidipine d. Midazolam
Ans: C Ans: A
15) Hyperchloremic metabolic acidosis? 23) Which of the following narcotic when
a. Loop diuretic given epidural causes delayed
b. Hydrochlorthiazide respiratory depression?
c. Acetazolamide a. Morphine
SK Origional – Golden 11 374
b. Fentanyl 31) Which of the following is released in
c. Pethidine Stress due to surgery which causes
d. Xylocaine vasoconstriction its due to release of?
e. Ketamine a. Epinephrine
Ans: A b. Norepinephrine
24) Ketamine used in burn dressing? c. ACTH
a. It causes analgesia d. Cortisol
b. Causes hypotension Ans: A
c. Can be given IV 32) Safe system depends on:
d. It doesn‘t causes addiction a. PO2
Ans: A b. Pressure
25) What is drug of choice for TIVA? c. Flow
a. Propofol d. Reserve
b. Midazolam Ans: A
c. Triazolam 33) Adrenaline should not be used with
d. Prilocaine lignocaine in which of following areas
Ans: B that causes. Ischemic injury?
26) Lady with cushing syndrome most imp a. Upper limb
symptom? b. Lower limb
a. Abdominl stria c. Penis
b. Hyperglycemia d. Abdomen
c. Hypertension e. Thigh
d. Weight gain Ans: C
Ans: A 34) 41% hematocrit means:
27) Pt.had accident, BP less than 60.Which a. 41% of formed elements are RBC
effect is most pronounced? b. 41% of blood is Hb
a. Baroreceptors c. 41% of blood volume is
b. CNS Ischemic response RBCs,WBCs,platelets
c. Chemoreceptor d. None of the above
d. RAAS Ans: A
Ans: B 35) Cushing triad:
28) Best intercostal block for rib fracture a. Increase ICP bradycardia & hypertension
is: b. Increase ICP tachycardia & hypotension
a. Bupivacaine c. Decrease ICP tachycardia & hypotension
b. Lignocaine d. Increase ICP tachycardia & hypertension
c. Prilocaine Ans: A
d. Propofol 36) Critical temperature of oxygen:
Ans: A a. 80C
29) After something hapened pt heart is b. 37C
failing bp is 80/40 he is sweating and c. -91C
salivating profusely treatment d. -118C
indicated is? Ans: D
a. Atropine 37) Micturition reflex is integrated in:
b. Edrophonium a. Brainstem
c. Norepinephrine b. Hippocampus
d. Physostigmin c. Cerebral cortex
Ans: A d. Sacral segments of spinal
30) Muscle relaxant that can be given to Ans: D
asthmatic patient? 38) Dorsal nucleus of vagus nerve is
a. Suxamethonium present:
b. Tubocurare a. Medulla
c. Cisatracurium b. Midbrain
Ans: C c. Cerebellum
SK Origional – Golden 11 375
d. Cortex 46) True regarding scalenus anterior
Ans: A muscle:
39) Man got an accident and had complete a. Arises from C3-C6 vertebral bodies.
transection at T1 level he will have: b. Inserted at the anterior border of first rib.
a. Permanent loss of tendon reflexes below c. Phrenic nerve passes anterior to it
the level of lesion. d. Subclavian artery/vein passes anterior to
b. Temporary loss of stretch reflex below the it.
lesion Ans: C
Ans: B 47) Vertebral bodies limited by:
40) Muscle that increases transverse a. Ligamentum Flavum
diameter of rib cage: b. Anterior and posterior longitudinal
a. External intercostal ligament
b. Diaphragm c. Ligamentum nachue
c. Internal intercostal d. None
d. External oblige Ans: B
Ans: A 48) Needle inserted at paramedian
41) Gunshot injury to the cervical vertebra approach vertebral canal pierces?
patient now experiencing respiratory a. Posterior spinal ligament
distress which muscles likely affected? b. Anterior spinal ligament
a. Diaphragm c. Supraspinous ligament
b. Scalenus anterior d. Interspinal ligament
c. Pectoralis major e. Ligamentum flavum
d. Sternocleidomastoid Ans: E
Ans: A 49) Ideal site for lumbar puncture is:
42) 15% Percent blood loss replaced by: a. Above L4
a. Whole blood b. Below L5
b. Three times normal saline c. Above L3
c. Ringer lactate d. Above L2
d. 5 percent dextrose Ans: A( Prefer below L4)
Ans: C 50) Thyroidectomy was done in left lower
43) Cricothyroid muscle is supplied by: lobe artery was ligated which supply
a. Internal laryngeal nerve this area. Which nerve has more
b. External laryngeal nerve chances to be damaged during
c. Recurrent laryngeal nerve operation?
d. Hypoglossal nerve a. Recurrent laryngeal nerve
Ans: B b. External branch of superior laryngeal
44) Resident is not allowed to do right side nerve
adrenalectomy by his seniors this was c. Internal branch of superior laryngeal nerve
due to presence of which structure? Ans: A
a. Aorta 51) Sternocleidomastoid muscle action?
b. IVC a. Turn head to opposite side against
c. Liver resistance
d. Ureter b. Flexion of head
e. Duodenum c. Extension of head
Ans: B Ans: A
45) Which structure loops around arch of 52) During epidural LP all of following
aorta? structure is pierced except?
a. Left recurrent laryngeal nerve a. Posterior longutidual ligament
b. Azygous vein b. Ligamentous flevum
c. Left vagus nerve c. Interspinous ligament
d. Subclavian d. Supraspinous ligament
Ans: A Ans: A
53) Dose of atropine in bradycardia is:
SK Origional – Golden 11 376
a. 1 mg 61) Action of Norepinephrine is
b. 2 mg terminated after being released from
c. 0.5 mg presynaptic terminal?
d. 1.5 mg a. Reuptake
Ans: C b. Metabolism by COMT
54) Square root of variance: c. Metabolism by MAO
a. Mean d. Diffusion into post synaptic space
b. Co-efficient of variance Ans: A
c. Standard deviation 62) Which of the following enzyme
d. Mode decreases in acute liver disease?
Ans: C a. Alp
55) Best beta blocker in bronchogenic b. ALT
asthma patients? c. Aspartate transaminase
a. Bisoprolol d. Pseudo cholinesterase
b. Propranolol e. Neuclosidase
c. Metoprolol Ans: D
d. Carvidolol 63) Most common pattern of genetic
Ans: A disease is?
56) Pain from lower inferior segment of a. Autosomal Dominant
diaphragm: b. Autosomal Recessive
a. Phrenic nerve c. X-Linked Recessive
b. Subcostal nerves d. Multifactorial
c. Intercostal nerve Ans: D
d. Vagus nerve 64) Burkett lymphoma oncogenes is by
Ans: B EBV:
57) Rectus sheath at the level of umbilicus a. Proto-oncogene c-myc amplification
anteriorly formed by? b. Proto-oncogene c-myc translocation
a. By layer of internal oblique muscle c. Ras proto-oncogene
aponeurosis with external oblique d. BCL2 proto-oncogene
b. Fasia transversalis Ans: B
c. Only external oblique aponeurosis 65) Young male anti HCV positive.
d. Only transverse abdominis muscle Asymptomatic and LFTs are normal.
Ans: A What is the next best investigation?
58) Upper origin of external oblique a. Repeat anti HCV
muscle interdigitate with: b. Repeat LFT
a. Lattismus dorsi c. Biopsy
b. Pectoralis major d. PCR RNA
c. Serrates anterior Ans: D
Ans: C 66) Succinylcholine is used during
59) Mechanism of action of nerve gas in anesthesia as skeletal muscle relaxant.
military war is: Which is the enzyme responsible for its
a. Acetylcholine esterase inhibitor metabolism?
b. Acetylcholine inhibitor a. Acetyl cholinesterase.
c. Pseudo choline inhibitor b. Pseudocholine esterase
d. None c. Phosphorylase
Ans: A d. Succinyl cholinesterase
60) Which of following drug is competitive e. Plasma cholinesterase
antagonist of acetylcholine receptors? Ans: B
a. Atropine 67) Pregnancy which of the following
b. Cyclizine occurs?
c. Dopamine a. Increase peripheral resistance
d. Epinephrine b. Decrease Tidal volume
Ans: A c. Decrease FRC
SK Origional – Golden 11 377
d. Decrease Respiratory rate Ans: A
Ans: C 75) Floor of 4th ventricle's superolateral
68) Chronic liver disease which of the relation:
histological finding suggest a. Superior cerebellar peduncle
chronicity? b. Middle cerebellar peduncle
a. Fibrosis c. Inferior cerebellar peduncle
b. Councilman bodies(apoptosis) d. Lower porns and pyramidal tract
c. Neutrophils Ans: A
d. Macrophages 76) Difference between right and left
Ans: A kidney:
69) Local anesthesia cross placenta a. Size
through: b. Weight
a. Active transport c. Colour
b. Bulk flow d. Realtion at hila
c. Facilitated diffusion Ans: D
d. Simple diffusion 77) Femoral and obturator nerve originate
Ans: D from which root of sacral plexus?
70) Equalization of gases on both side of a. S1 – S2
membrane: b. L4 – L5
a. Active transport c. L2 – L4
b. Simple diffusion Ans: C
c. Pinocytosis 78) Mechanism of action of Propyl Thio
d. Facilitated diffusion Uracil (PTU), antithyroid drug:
Ans: B a. Inhibits iodide pump
71) Charles law applicable to volume of b. Blocks the synthesis of thyroid hormone
gas: c. Blocks the synthesis of thyroglobulin
a. At constant temperature d. Inhibits hormone release
b. At constant pressure e. Controls hypothalamic temperature set
c. At constant volume point
d. At constant density Ans: B
Ans: B 79) 19-year-old boy sitting in a well-
72) Initial sign of lidocaine overdose is: ventilated room of 26 'C. profusely
a. Prolong Seizure for several hours sweating. His pulse 120beats/ min, BP
b. Perioral paresthesia 150/90 .What is the diagnosis?
c. Hypotension a. Heat stroke
d. Headache b. Mental stress
Ans: B c. Exercise
73) Inspiratory ramp signals are produced d. Sympatholytic drugs
from dorsal respiratory neurons. Their Ans: B
rate increases in response to impulses 80) Which part of the male genital tract
from? has partially motile sperm?
a. Stretch receptors in lung a. Ductus deference
b. Pneumotaxic centre in pons b. Seminal tubercle
c. Apneustic centre c. Epididymis
d. Mid brain d. Rete testes
Ans: B Ans: C
74) Medial geniculate body, a thalamic 81) Most important function of
nucleus? progesterone:
a. Lies on the mid brain a. Alveolar duct development
b. Is attached to the main mass of thalamus b. Decrease uterine contraction
c. Receive fibers from oclomotor nerve c. Secretory phase endometrium
d. Relays fibers to frontal lobe d. Maintain thickness of endometrium(or
e. Relays fibers to visual cortex proliferation of endometrium)
SK Origional – Golden 11 378
e. Fat deposition c. Basilar artery with external carotid
Ans: D d. Anterior cerbellar with Posterior
82) No of divisions of lower airways? communicating artery
a. 17 Ans: B
b. 21 89) Overdose of TCA antidepressant
c. 23 physician is concerned about:
d. 26 a. Bowel sound
Ans: C b. Bilirubin
83) Rh negative mother delivered a baby at c. ALT
34 weeks with anemia pallor. Baby d. Cardiac rhythm
died 6 hours after transfusion. Autopsy Ans: D
findings? 90) Droplet infection can spread upto?
a. Basal ganglia staining a. 1–2
b. Cardiac malformations b. 3–6
c. Hydrops fetalis c. 8–9
d. None of the above d. 15 feet
Ans: A Ans: A
84) Which of the following is less in fetus 91) Basophilia of the cell is due to:
than mother? a. SER
a. PO2 b. Ribosomes
b. PCO2 c. Mitochondria
c. HB d. Golgi
d. Neutrophils Ans: B
Ans:. A 92) Which of following covers myosin and
85) Which of the following is released in blocks the interaction between actin
stress due to surgery which causes and myosin?
vasoconstriction its due to release of? a. Tropomyosin
a. Epinephrine b. Troponin T
b. Norepinephrine c. Troponin I
c. ACTH d. Calmodulin
d. Cortisol Ans: A
Ans: A 93) When patellar tendon hit, which of the
86) Most immediate precursor of following occurs?
epinephrine is: a. Quadrataus femoris muscle contracts
a. Tyrosine b. Muscle spindle afferent sends transmission
b. Phenylalanine c. Golgi tendon sends transmission
c. Dopamine Ans: B>A
d. Nor – Epinephrine 94) The diagnosis of amyloidosis is best
Ans: D confirmed by:
87) Cardiac surgeon during open heart a. Bence jone's proteinuria
surgery founds bleeding vessel b. Bone marrow
accompanying LAD in anterior c. Clinical observation
interventricular groove. Most likely d. Congo red test
involved vein is: Ans: D
a. Great cardiac vein 95) Voluntary inhibition of micturition
b. Anterior cardiac vein reflex:
c. Small cardiac vein a. Pudendal nerve
d. Marginal Vein b. Detrusor muscle contraction
e. Oblique vein c. Trigone
Ans: A Ans: A
88) Circle of Willis is formed by? 96) Drug causes fluid retention:
a. Vertebral artery and middle cerebral a. Oxytocin
b. Two internal carotid and basilar artery b. Ergometrin
SK Origional – Golden 11 379
c. Misoprostol Ans: B
d. Mifepristone 104) Action of captopril:
Ans: A a. Hypercalcemia
97) Anesthetist wants to give his patient b. Decrease production of angiotensin – II
most potent muscle relaxant who is c. Increase degradation of bradykinin
about to undergo cesearation so that Ans: B
she may relax completely. He would 105) A patient in an ICU is on a CVP line.
give: His blood culture shows growth of
a. Bupivacaine grain positive cocci which are catalase
b. Ropivacaine positive and coagulase negative. The
c. Procaine most likely etiological agent is?
d. Lidocaine a. Staplylococcus aureus
Ans: A b. Staplylococcus epidermidis
98) Progressive grade 4 hepatic c. Streptococcus pyogenes
encephalopathy caused by: d. Enterococcus faecalis
a. Paracetamol Ans: B
b. Alcohol 106) Patient has diarrhea which IV should
c. Uremia be given to increase ECF and decrease
d. Diuretics ICF?
Ans: D a. 0.9% Nacl
99) A pt. with chemotherapy induced b. 0.45%NS
vomiting which antiemetic should be c. 3% Nacl
given? d. R/L
a. Methochlopramide e. Pure water
b. Domeperidone Ans: C
c. Promethazine 107) A person come with shortness of
d. Ondansetron breath he has been diagnosis with
Ans: D pulmonary edema last week what
100) 3rdheart sound is due to: could be reason?
a. Mitral valve closure a. Decrease blood flow
b. Tricuspid valve closure b. Decrease lymphatic flow
c. Rapid flow in ventricles c. Increase intravascular pressure
d. Semilunar valve opening d. Increase oncotic pressure
Ans: C Ans: C
101) Corona radiata is formed by? 108) Lymph flow decreases due to increase
a. Granulosa cells in:
b. Theca interna a. Increase capillary oncotic pressure
c. Theca externa b. Increase venous compliance
d. Zona pelluceda c. Increase interstitial oncotic pressure
Ans: A d. Exercise
102) The subclavian vein joins the internal e. Venous return
jugular vein to form the Ans: A
brachiocephalic vein at which of the 109) Wrist joint is example of:
following structures? a. Condyloid
a. The outer border of first rib b. Hinge joint
b. Behind the sternal end of clavicle c. Pivot
c. Inner border of Rib d. Ellipsoid
Ans: B e. Synovial
103) In metabolic alkalosis what happens? Ans: D
a. Hyperkalemia 110) Main cause of edema in nephrotic
b. Hypokalemia syndrome:
c. Hyponatremia a. Proteinuria
d. None b. Hypoalbuminemia
SK Origional – Golden 11 380
c. Hyperalbubinemia c. Anemia of chronic disease
Ans: B d. Folic acid deficiency
111) Diabetic type 2 pt. with increase e. Pernicious anemia
urinary frequency, cause of urination? Ans: D
a. Dec. reabsorption of glucose 117) Which hormone responsible for
b. Dec. reabsorption of water delayed gastric emptying?
c. Fixed specific gravity of urine a. CCK
d. Dec sensitivity of ADH on aquaporin b. Gastrin
Ans: A c. Secretin
112) Between meals liver maintain sugar d. Somatostatin
levels by: Ans: A
a. Gluconeogenesis 118) Aspirin inhibits platelets aggregation
b. Glycogenolysis by decrease production of:
c. Lipolysis a. Thromboxin A2
d. Proteolysis b. Leukotriene‘s
Ans: B c. Prostaglandins
113) A young man developed hemolytic d. Endothelial cells
transfusion reaction after transfusion Ans: A
of incompatible blood. Free 119) Liver shows centrally distorted area
hemoglobin liberated due to hemolysis surrounded by epitheloid cells,
would start excretion in the urine after lymphocytes and giant cells?
a plasma protein becomes saturated a. Coagulative necrosis
with free hemoglobin. This plasma b. Caseous necrosis
protein is? c. Acute hepatitis
a. Albumin Ans: B
b. Alpha globulin 120) Tramadol acts through:
c. Beta globulin a. Strong mu receptor
d. Haptoglobin b. Weak mu receptor
e. Hemopexin c. Alpha receptor
Ans: D d. Gamma receptor
114) Postoperative after 2 hours developed Ans: B
hypotension tachycardia and 121) Factory worker e exposure to
abdominal distension, cause: hydrocarbon has increased risk of:
a. Sepsis a. Ca bladder
b. Anaphylactic b. Bronchogenic
c. Hypovolemic c. Hepatocellular
d. Neurogenic d. Prostatic carcinoma
Ans: C Ans: A
115) A scenario of a child having recurrent 122) Labetalol acts on:
bacterial infections and increased a. Alpha 1
neutrophils in blood while absence of b. Beta 1
neutrophils at the infection sites. c. Alpha + Beta receptor
Neutrophils can‟t attach to d. Gamma
endothelium to deficiency of: Ans: C
a. LFA-1 (integrin) 123) Long term use of HRT with positive
b. Icam 1 family history of breast CA. Now
c. Vcam increased risk of developing?
Ans: A a. Breast CA
116) A 14 yrs boy eats only fast food. His b. Endometrial CA
peripheral blood picture shows low hb, c. Lung carcinoma
raised MCV. What is diagnosis? d. Cervical carcinoma
a. IDA Ans: A
b. Megaloblastic
SK Origional – Golden 11 381
124) In which of the following area, the Ans: A
vomiting center is located? 132) A thyroid mass usually moves with
a. Thalamus swallowing because the thyroid gland
b. Hypothalamus is enclosed by which of the following
c. Medulla oblongata fascia:
d. Pons a. Carotid sheath
Ans: C b. Investing layer of the deep cervical fascia
125) Directly control autonomic reflex c. Pretracheal fascia
system of body: d. Paravertebral fascia
a. Thalamus Ans: C
b. Hypothalamus 133) Filling of right atrium is shown in JVP:
c. Cerebrum a. A Wave
d. Cerebellum b. X Wave
Ans: B c. V Wave
126) Muscles of back are supplied by: d. Y Decent
a. Dorsal rami of spinal nerves Ans: C
b. Cervical plexus 134) Aspiration of foreign body in supine
c. Ventral rami of spinal nerve position, the object will be lodged in
d. None which lung lobe?
Ans: A a. Posterior segment of right upper lobe
127) Pain relief in Aspirin ingestion is by b. Superior bronchopulmomary segment of
decrease production of: right lower lobe
a. Thromboxins c. Right middle lobe
b. Leukotriene‘s d. Left inferior lobe
c. Prostaglandins Ans: B
d. Endothelial cells 135) Umbilical vein has % of oxygen:
Ans: C a. 20%
128) Primary scavenger cells are: b. 30%
a. Neutrophils c. 40%
b. Macrophages d. 60%
c. Plasma cells e. 80%
d. Lymphocytes Ans: E
e. Langerhans‘s cells 136) Virus cause cancer by alteration in:
Ans: B a. Oncogene
129) Mitral valve is best heard at: b. Proto-oncogene
a. 2ndR intercostal c. DNA
b. 2ndL intercostal d. RNA
c. 3rdL intercostal Ans: B
d. 5thL intercostal 137) Sesamoid cartilage is present in:
Ans: D a. Acetubulum
130) Fever in inflammation caused by: b. Epiglottis
a. IL 1 and TNF – a c. External auditory canal
b. Prostaglandins d. Knee
c. TNF – B e. Alae of the nose
d. Vasodilation Ans: E
Ans: A 138) Lesion at right precentral gyrus:
131) Cranial Nerve 3rd and 4th at which a. Exaggerated knee jerk on left
level? b. Right hemiplegia
a. Midbrain c. Loss of voluntary control on left
b. Pons d. Normal plantars on right
c. Medulla e. Right side sensory los
d. Cerebellum Ans: C
e. Cortex
SK Origional – Golden 11 382
139) For epidural anesthesia rupivacain is d. Xerophthalmia
preferred over bupivacaine due to: Ans: A
a. Less toxic 146) Root value of phrenic nerve is:
b. Rupivacaine has short duration of action a. C234
c. Cardiac toxicity reverse b. C345
d. Short acting c. C567
e. No effect on fetus d. C678
Ans: A Ans: B
140) Hypotension after spinal anesthesia is 147) Fat embolism:
due to: a. Is a common autopsy finding after
a. Hypovolemic fracture of long bones
b. Loss of vasomotor tone b. Is less common in females
c. Gain of vasomotor tone c. Is not usually present as respiratory
d. Hypoglycemia insufficiency
Ans: B d. Produce clinical symptoms 5-10 days after
141) Confidentiality should not be breached injury
in situation: Ans: A
a. An HIV 148) Left axis deviation on ECG in:
b. An adult male sexually abusing daughter a. Aortic stenosis
c. A girl told you she has hid 100 tablets to b. Aortic regurgitation
commit suicide c. Mitral stenosis
d. A women told you she is depressed and d. Rbbb
occasionally she has suicidal thoughts e. Lbbb
e. None Ans: E
Ans: E 149) Most potent antioxidant:
142) Pancreatic insufficiency leads to a. Glutathione
deficiency of which of following b. Vit. E
vitamin? c. Vit. A
a. Vitamin C d. Vit. C
b. Vitamin B3 e. Transferrin
c. Vitamin B1 Ans: A
d. Vitamin D 150) Type of anemia in acute blood loss:
Ans: D a. Microcytic
143) Deficiency of brush border enzyme b. Normocytic
that causes carbohydrates c. Macrocytic
malabsorption? d. Anemia of chronic disease
a. Int amylase Ans: B
b. Lactase 151) Lateral curvature loss of spine;
c. Enterokinase a. Kyphosis
d. Peptidase b. Scoliosis
Ans: B c. Lordosis
144) Vitamin K directly influences the Ans: B
reaction: 152) Man in marathon in July excessive
a. Conversion of fibrinogen to fibrin sweating collapses cause:
b. Prothrombin to thrombin a. Dehydration
c. Activation of factor 4 b. Heat stroke
d. Formation of fibrinogen c. Heat exhaustion
Ans: B Ans: C
145) Combine deficiency of riboflavin & 153) Soldier at Siachin developed dyspnea
niacin causes? likely cause would be:
a. Pellagra a. Pulmonary hypertension
b. Beriberi b. Cardiogenic shock
c. Scurvy c. High ouput cardiac failure
SK Origional – Golden 11 383
d. Pulmonary edema c. Sodium bicarbonate compound
Ans: D d. Potassium carbonate compound
154) Non-bacterial thrombotic endocarditis Ans: C
is associated with: 161) Clostridium botulinum specific
a. Congenital heart disease symptoms in food poisoning:
b. Ischemic heart disease a. Diarrhea
c. Rheumatic endocarditis b. Flacid paralysis of respiratory muscles
d. Systemic lupus erythematosis c. Projectilevomiting
e. Terminal neoplastic disease d. Convulsions
Ans: E Ans: B
155) Patient with renal artery stenosis may 162) Measles in pregnant lady. Doctor
present with very high blood pressure worried about:
due to increased renin secretion. a. TOF
Which of the following structure in b. Peripheral neuropathy
kidney responsible for sensing c. Cataract
inadequate perfusion and secreting d. Anencephaly
renin? e. Cleft plate
a. Afferent Arterioles Ans: C
b. Collecting duct 163) Assessment of muscle injury by:
c. Juxtaglomerular cells a. Myoglobin
d. Efferent Arterioles b. CKMB
e. Arcuate artery c. CKBB
Ans: C d. CKMM
156) Role of lymphatic‟s: e. LDH
a. Dissemination of infection Ans: D
b. Removal of extravascular fluid 164) Inhibitory neurotransmitter is:
c. To present antigen to lymphocytes a. Ach
Ans: B b. GABA
157) Sympathetic supply of vessel is by: c. Aspartate
a. Beta 1 adrenergic d. Glutamate
b. Beta 2 adrenergic Ans: B
c. Cholinergic 165) Absent p wave on ECG occur in:
d. Muscarinic a. MI
e. Nicotinic b. Nodal rhythm
Ans: B c. 1stdegree heart block
158) Patient of TB is on Att for 2 months. d. 2nddegree heart block
Tb antibodies are present: Ans: B
a. Circulating in plasma 166) Action potential in neuron is
b. Bound to nucleus terminated by:
c. Cell bound a. K influx
Ans: C b. K efflux
159) Structures which prevent c. Na influx
communication of unwanted materials d. Ca influx
between cells? Ans: B
a. Tight junction‘s 167) Metabolite of vitamin D most
b. Zona adherence commonly assessed in laboratory, to
c. Desmosomes rule its deficiency:
d. Gap junctions a. Cholecalciferol
Ans: A b. 25 hydroxy Vit. D
160) Majority of CO2 transport in blood c. 1,25 Dihydroxycholecalciferol
occurs by: Ans: B
a. Dissolved form 168) In acute MI changes on ECG:
b. Carbamino compound a. Flat T waves
SK Origional – Golden 11 384
b. Displaced ST
c. Tall T
d. U waves
Ans: B
169) Which of following feature seen in
lower motor neuron?
a. Flaccid paralysis
b. Spasticity
c. Hyper active stretch reflex
d. Up going Toe
Ans: A
170) Grading of carcinoma shows:
a. Dysplasia
b. Permeability to basement membrane
c. Mitosis inside cell
d. Invasion
e. Degree of differentiation
Ans: E
171) Microscopic feature of malignancy is:
a. Mitotic figure
b. Dysmoplasia
c. Pleomorphism
d. Invasion
Ans: D

===============
SK Origional – Golden 11 385

(500 – SK Golden Points)

Most Important 500 SK – Golden Points


of FCPS Part – 1 Exam that will be
helpuful to you in exam.
SK Origional – Golden 11 386

MOST REPEATED POINTS & BCQs

1. In Bitemporal hemianopia blockage will be seen in?


A. Middle (central) part of optic chiasma
B. Upper part of optic chiasma
C. Optic Tract
D. Optic Nerve
Ans: A
If middle part option is not given then prefer Upper. Also note that Bitemporal hemianopia is caused by optic
chiasma lesion and homonymous hemianopia is caused by contralateral involvement of optic tract if middle
part options are not given then prefer Upper. Also note that Bitemporal hemianopia is caused by optic
chiasma lesion and homonymous hemianopia is caused by contralateral involvement of optic tract.
2. Adult polycystic kidney disease is?
Ans: Autosomal dominant while infantile type is autosomal recessive
3. Sensitive test for SLE:
Ans: ANA it is sensitive for almost all types of autoimmune disorders. Also note that ANA is sensitive but
not specific. Anti Sm Abs are most specific.
4. After exercise a person lost fluids by Sweating. He drinks 2liters of water. What will happen?
a. Increased ECF volume
b. Increased ICF volume
c. Hyper-osmolality of ECF
d. Hyper-osmolality of ICF
e. No change
Ans: B
# Sequence of Events:
Increase ICF volume > Decrease ICF osmolarity > Increase ECF volume > Decrease ECF osmolarity
5. Which one of the following is true about RBC antigen?
Ans: Present on RBCs surface > Show mandellian inheritance > Glycoproteins > Immogenic
6. Viruses cause cancer by?
Ans: Alteration in protein synthesis
SOME CONFUSING POINTS REGARDING VIRAL ONCOGENESIS:
Viruses cause cancer because they have  Oncogenes
Viruses cause cancer by altering
• Protoncogenes
• Alteration in protein synthesis
• Altering the host DNA ,some viruses integrate in to the host cell DNA which results in
overexpression of viral proteins called virions, So when asked in Exam.
Prefer # Protoncogenes > Alteration_In Protein_Synthesis > Altering DNA
7. A resident has done 2 successful adrenalectomies. The head of surgery doesn't let him do
adrenalectomy on this patient because he implies it might be difficult for him to perform
surgery on right adrenal. What relation of right adrenal makes it difficult?
a. Posterior relation to lower ribs
b. Right renal artery
c. IVC
d. Left hepatic vein/artery
SK Origional – Golden 11 387

e. Aorta
Ans: C
8. Left Undescended testes, risk of:
a. Infertility
b. Avascular necrosis
c. Neoplasia
Ans: C
9. Dilatation of aortic arch will compress?
a. Left Bronchus
b. Trachea
c. Esophagus
d. Right Bronchus
Ans: A
10. At which level trachea starts?
a. C6
b. T1
c. T2
d. T4
Ans: A
11. Pt suffering from appendicitis, having pain in right iliac fossa that is referred to umbilicus.
Which spinal nerve root is involved in this referred pain?
a. T10
b. T11
c. L1
d. L2
Ans: A
12. Primary active transport:
a. Pump
b. Carriers
c. Channel proteins
Ans: A
13. Renal threshold for glucose:
a. 180
b. 200
c. 250
d. 375
Ans: B (Ref: Guyton)
14. Patient with discoid rash, arthritis and hemolytic anemia. What‟s most specific investigation?
a. ANA
b. Anti-ds DNA Ab
C. Anti Sm Ab
Ans: C
15. What is used for traveller's diarrhea?
a. Metoclopramide
b. Diphenoxylate
SK Origional – Golden 11 388

c. Ofloxacin
Ans: B
16. Drug of choice for traveler's diarrhea:
A. Bismuth subsalicylate  1st best choice
B. Diphenoxylate/Loperamide  2nd best choice
C. Antibiotics (ciprofloxacin>>norfloxacin)  Third best If diarrhea is not responding to antidiarrheal.
16. Metastasis occurs due to:
A. Loss of E cadherin
B. Migration of tumor cells
Ans: A
17. After chemotherapy, cancer cells die, nucleus and cytoplasm show fragmentation. What is the
mechanism?
Ans: Apoptosis
18. Patient is given oral anti-coagulant. How to monitor?
a. PT
b. APTT
Ans: A
INR > PT. Better option is PT/INR if both options separately given then prefer INR only PT is given in
options and no INR option available then go for PT.
19. Chronic Intravascular hemolysis?
Ans: Hemoglobinuria > Hemosiderinuria > Decrease Heptoglobin Levels
20. Obstructive pattern disease diagnosed by:
Ans: FEV1/FVC Less than 70
21. A lady with 6 weeks of German measles:
a. Cataract
b. Deafness
c. PDA
Ans: B
Nerve deafness is most commonly associated abnormality with Rubella. Ref: Davidson
22. Air embolism, quantity of air required to produce symptoms:
a. 100ml
b. 50 ml
Ans: A (Ref: Robbins)
23. Severe hypomagnesaemia causes:
a. Hypocalcaemia
b. Hypercalcemia
Ans: A
Severe Hypomagnesaemia leads to Hypocalcaemia
Mild Hypomagnesaemia Hypercalcemia
Both Mg and Ca compete for excretion through Kidney, but if Mg is too low it causes decreased production of
PTH which in turn causes hypocalcaemia.
24. 7th left pulmonary segment. What is it called?
a. Lingual lobe
b. Right medial basal
c. Lateral basal
SK Origional – Golden 11 389

d. Apical
Ans: B (Ref. RJ Last)
25. A child who is apathic, peripheral edema, low albumin?
a. Kwashiorkor
b. Marasmus
Ans: A
26. Cimetidine is used preoperatively because?
a. Inhibits H2 receptors competitively
b. Decrease gastric acid
c. Does not affect gastric motility
d. Does not interact with induction agent‘s
Ans: B
27. Diagnostic criteria for sarcoma?
a. Increased vascularity
b. Pleomorphism
Ans: A
28. Soldier coming down from height, now with discoloration of fingers, dyspnea?
Ans: Secondary polycythemia
29. Dual nerve supply but not reciprocal?
a. Sweat gland
b. Thyroid gland
c. Ciliary body
d. Pupils
Ans: A (Ref. BRS)
Salivary glands have dual supply from sympathetic and parasympathetic both. But both of them increase
salivation.
Reciprocal supply means parasympathetic and sympathetic working opposite to each other ideally should be
like if parasympathetic is increasing salivation, sympathetic should decrease. But what happens in salivary
glands is that both innervations increase salivation so salivary glands have dual supply, but not reciprocal
supply.
30. Typhoid first week test?
a. Blood culture
b. Widal
Ans: A (BASU)
1st week Blood Culture
2nd week Blood plus Widal
3rd week Stool Culture
4th week Urine Culture
31. Patient has pericardial friction rub, with deranged Renal Function Tests, what is the cause of
pericarditis?
a. Serous
b. Fibrinous
c. Exudative
Ans: B (Uremic Pericarditis)
32. Pus contains?
SK Origional – Golden 11 390

a. Dead Bacteria
b. Dead Neutrophils
Ans: B
33. Thorn prick on the leg abscess due to:
Ans: Staph Aureus
34. Increase LES tone and speeds up gastric emptying?
a. Metoclopramide
b. Ondansetron
Ans: A
35. Heparin primary action achieved by binding to?
a. Anti-thrombin 3
b. Xa
c. Thrombin
Ans: A
36. Short arms /legs Lordosis With normal mental health. Disease mode of transmission
(achondroplasia)?
a. Autosomal dominant
b. Autosomal recessive
c. X-linked
Ans: A
37. Adult Polycystic kidney disease mode of transmission?
a. Autosomal dominant
b. Autosomal recessive
Ans: A
38. Philadelphia chromosomes:
a. CML
b. AML
Ans: A
39. Isthmus of thyroid?
a. 2, 3, 4 rings
b. 3, 4 rings
c. 1, 2 rings
Ans: A
40. Which cause malignancy in GIT?
a. H.Pylori
b. CMV
Ans: A
41. Most abundant in serum:
a. IgG
b. IgM
Ans: A
42. Microscopic findings were epithelium and rete ridges:
a. Verrocus carcinoma
b. Squamous cell carcinoma
Ans: A (it‟s a type of SCC, if not given in options then SCC can be opted)
SK Origional – Golden 11 391

43. Flexor withdrawal reflex:


a. Multisynaptic
b. Monosynaptic
c. Disynaptic
Ans: A
44. A postmenopausal woman is on Hormone Replacement Therapy for PM related
Osteoporosis. What complications she is at risk of?
a. Anxiety
b. Thromboembolism
Ans: B
45. 55 year old lady taking HRT and her sister died of breast cancer. She is at risk of:
a. Breast cancer
b. Cervical cancer
Ans: A (Family history is a stronger risk factor than any other)
46. Broncho pulmonary segment supplied by:
Ans: Tertiary bronchus
47. Asthmatic patient, Post-op Analgesic needed:
a. Acetaminophen
b. Ketorolac
Ans: B (Ref. Goodman Pharma)
48. Woman underwent hysterectomy. She has history of Asthma. Which drug can be given for
postoperative analgesia?
a. Morphine
b. Ketorolac
c. Diclofenac
d. Paracetamol
e. Pethidine
Answer: B (Ref. Goodman Pharma)
49. Method to detect Circulating Tumor cells in blood used widely:
Ans: Peripheral blood smear and method to detect cancer is tumor marker.
50. 59 years old female has strong family history of IHD and Breast Cancer. Wants to know what
to take to protect bones from fracture. She knows someone who had a fracture after Trivial
Trauma?
a. Bisphosphonates
b. Tibolone
c. Raloxifen SERM
d. Complete HRT
e. Ca and Vit. D
Ans: C (Ref. Goodman Gilman Pharma)
51. In Primary syphilis best specimen for diagnosis is:
a. Blood
b. Genital sore
c. Lymph node
d. Bone biopsy
SK Origional – Golden 11 392

e. Semen
Ans: B
52. Acute toxicity of oral anticoagulant is immediately reversed by:
a. Protamine sulphate
b. FFP
c. Vitamin K
Ans: B (Specific antidote is Vitamin K. But it takes time to act)
53. Most radiosensitive tumor:
a. Ovarian CA
b. Lymph node
c. Germ cell
d. BCC
e. SCC
Ans: B
Remember the mnemonic LSGS = Larkana Sui-Gas Company (Lymphoma > Seminoma> Glioma>
Craniopharyngioma).
54. Burn patient presented with tachycardia hypotension high grate fever, bleeding diathesis
oozing from wounds and hematuria. Diagnosis?
a. Infection
b. Septic shock
c. Hypovolemic shock
d. DIC
Ans: D
55. 1st mechanism after injury:
a. Vasoconstriction
b. Platelet recruitment
c. Fibrin deposition
Ans: A
56. Fisherman diet consists of fish and rice he suffered from megaloblastic anemia causative
agent is?
a. Diphylobothrium Latum
b. Ancylostoma duodenale
c. Ascaris Lumbericoides
d. Taenia Solium
Ans: A (Causes B12 deficiency)
57. First defense against tumor are:
a. Natural killer cells
b. Th1 cell
c. Th2 cells
d. B cells
Ans: A (Ref: Robbins)
58. Patient comes with sinking heart and flat T waves what could be the cause?
a. Hypokalemia
b. Hypernatremia
c. Hyperkalemia
SK Origional – Golden 11 393

Ans: A (Hyperkalemia causes Tall T waves, Hypo causes Flat T waves and U waves)
59. Type of hypersensitivity in Myasthenia gravis?
a. Type 1
b. Type 2
c. Type 3
d. Type 4
Ans: B
60. A child having recurrent viral infections sinusitis otitis media and having 2 month history of
diarrhea, he also has a family history of such complains what test should be diagnostic?
a. Fat absorption test
b. Sweat chloride test
c. Jujenal biopsy
Ans: B (Typical cystic fibrosis)
61. Natural antithrombotic in blood?
a. Heparin
b. Warfarin
c. Plasminogen
Ans: A
Explanation: Anti-coagulant and anti-thrombotic are no different things. They have same meaning.
anticoagulation or antithrombotic means the given substance works before the formation of clot or thrombus
in simple words; it stops platelets aggregation and stops/prevents formation of clot. Now what is plasminogen
then?
It's naturally occurring fibrinolytic/thrombolytic:
Understand the difference. Plasminogen is Fibrinolytic. Which means it breaks down the already formed clot.
It doesn't prevent clot formation rather it breaks down the formed thrombus or clot.
Comprehensively:
Anti-Coagulant/Anti-Thrombotic  Heparin which Prevents clots formation
Fibrinolytic/Thrombolytic  Plasminogen which Breaks down the already formed clot
Prefer plasmin (aka fibrinolysin) over plasminogen if present in option.
62. Liver Enzymes and Regarding Liver:
ALT>AST Nonalcoholic Hepatic Necrosis, Drug Induced, Viral Hepatitis
AST>ALT Alcoholic Hepatitis
ALP, GGT Extra Hepatic Cholestasis/Enzymes of Biliary Tract
PT Best prognostic indicator for Hepatic Failure if not given then Factor V Estimation
SGPT/ALT Best to monitor Hepatitis B
Differentiate between Acute and Chronic Hepatic Failure Serum Albumin
Enzyme that can be released from cardiac myocytes as well AST
Reference: Davidson
Most common cause of fatty liver in our society  Starvation > Fatty food/obesity > Hepatitis B & C
> Alcohol
Most common cause of cirrhosis in Asian population  Alcohol
Most common cause of cirrhosis in Pakistan  Viral Hepatitis> Alcohol
Most common hepatitis In our population  Hepatitis A
Most commonly transfused via transfusion  Hepatitis C
Most dangerous when transfused CMV > Hepatitis B
SK Origional – Golden 11 394

63. Regarding blood grouping:


First of all there are 2 important things in blood groups. To have the knowledge of Antigens and Antibodies in
respective blood groups. Antigens, as we all know are kind of foreign bodies against which our immune system
fights by forming Antibodies.
Blood Group A:
Has# A Antigen on RBCs surface and #Anti B Antibodies circulating in plasma can receive blood from blood
group A and O. Transfusion reaction will occur if receives blood from B or AB.
Blood Group B:
Has# B Antigen on RBCs surface and #Anti A Antibodies circulating in plasma can receive blood
from blood group B and O Transfusion reaction will occur if receives blood from A or AB.
Blood Group AB:
Has both# A and #B Anti gene on RBCs surface but no antibodies can receive blood from blood group A, B
and O.
That is why known as Universal Recipient of RBCs and Universal donor of plasma because of No antibodies.
Blood Group O:
No antigens, but has #Anti A and #Anti B antibodies
Transfusion reaction will occur if receives any NON-O blood
Now about Rh+ve and Rh-ve:
Positive blood groups have Rh (D) Antigens on RBCs surface, and no antibodies
against Rh negative blood groups that is why Rh +ve blood groups can receive blood from their respective
type of both Rh positive and Rh negative blood groups and no reaction will occur
Negative blood groups do not have Rh Antigen but it has antibodies against all the Rh +ve blood groups.
That's why severe transfusion reaction will occur if receives any positive blood group it can only receive
negative blood group.
64. A 3 months old baby presented critically ill condition. His labs shows pH 7.2 pco2:32
HCO3:14.Na:142 and K: 2.5 most likely diagnosis?
a. Addison disease
b. DKA
c. Lactic Acidosis
d. Renal Tubular Acidosis
Ans: D
Explanation:
First of all age is supporting congenital cause. Can be Fanconi's Secondly, most common presentation of Renal
Tubular Acidosis is #Hypokalemia except Type 4
Why not Addison?
Because in Addison's there is Aldosterone deficiency which leads to retention of potassium and decreased
reabsorption of sodium.
Why not DKA?
Again DKA is also acidosis. It causes hyperkalemia also no evidence like gestational diabetes or any specific
feature of DM1 is given.
Why not Lactic Acidosis?
Same again acidosis hyperkalemia
65. Hypoxic pulmonary vasoconstriction accentuate by?
a. Increase PO2
b. Decrease PO2
SK Origional – Golden 11 395

c. Increase PCO2
d. Decrease PCO2
Ans: C
66. Pulmonary vasoconstriction occurs due to?
a. Raised PCO2
b. Reduced systemic PO2
c. Reduced pH
d. None of above
Ans: B
Explanation:
As we all know hypoxia causes vasodilation in all the vasculature other than pulmonary where it causes
Vasoconstriction.
In the 1st question they are asking about "Hypoxic Vasoconstriction Accentuated By?‖ which means patient
already has vasoconstriction due to hypoxia but now what is ―aggravating it‖? So, the answer is Increased
PaCO2 because it further worsens vasoconstriction in pulmonary vessels.
In the 2nd scenario question is simply about pulmonary vasoconstriction which is a direct effect of hypoxia so
for that prefer B here but better option is‖ Reduced Alveolar Oxygen‖. If not in options then reduced
systemic PO2 is fine.
67. A 59 years old female known Hypertensive on medication brought in semi collapsed state Her
K=6.1 most likely cause in patient is?
a. Acute Renal Failure
b. Chronic Renal Failure
c. Cushing Syndrome
d. Diuretic intake
e. Hemolysis
Ans: A
Explanation:
ARF can be pre-renal, renal and post renal Initially oliguria sets in which leads to decreased excretion of
potassium causing Hyperkalemia Hypernatremia and Hypervolemia It can occur in the matter of 7 to 10 days
and if uremia sets in because of oliguria it will lead to uremic encephalopathy and unconsciousness so patient
will present in emergency in case of sudden collapse or semi-comatose condition.
Potassium sparing diuretics will cause Hyperkalemia which is specifically not mentioned other diuretics cause
excretion of potassium and also diuretics alone cannot alone do this until given with ARBs.
ESR is the rate at which red blood cells settle out when anti-coagulated whole blood is allowed to stand. It is
affected by the concentrations of immunoglobulin and acute phase proteins like fibrinogen, C-reactive protein,
alpha-1 antitrypsin, Heptoglobin, ferritin, albumin etc.
68. Regarding ESR:
Factors that increase ESR are:
Raised HCT
Raised Fibrinogen
Decreased Albumin
Macrocytosis
And any inflammatory process because of acute phase reactants Like Multiple myeloma, RA, SLE etc.
Factors that decrease ESR are:
Decreased HCT
SK Origional – Golden 11 396

Increased albumin
Sickle Cell Anemia
How albumin affects ESR?
Presence of albumin in plasma increases negative charge of RBCs and decreases its sedimentation
69. Regarding proteins:
Ferritin  Normal stored form of iron
Hemosiderin  Endogenous pigment, abnormally produced by the breakdown of RBCs
Heptoglobin  A protein that binds to free hemoglobin
Hemopexin  Protein that binds to free Heme
Transferrin  Iron binding proteins
Ceruloplasmin  Copper carrying proteins
Hepcidin  Caries iron into circulation
Ferroportin  Transports iron from inside to outside of the cell.
70. Regarding action potential and resting membrane potential:
Hyperpolarization and #Repolarization is d/t Efflux of Potassium
The opening of channels that let positive ions flow out of the cell or negative ions flow in causes
hyperpolarization For example, Opening of channels that let K+ out of the cell causes hyperpolarization
which is followed by repolarization.
Depolarization is d/t influx of Na+ means more positivity inside the cell:
The opening of channels that let positive ions flow into the cell can cause depolarization For example, opening
of channels that let Na+ into the cell.
SK Origional – Golden 11 397

For MCQs purpose just remembers that when question is about #RMP answer is going to be #Potassium
When asked about #AP answer is going to be #Sodium.
The difference in total charge between the inside and outside of the cell is called the membrane potential. A
neuron at rest is negatively charged, the inside of a cell is approximately 70mv more negative than the outside.
This voltage is called the resting membrane potential. It is caused by differences in the concentrations of ions
inside and outside the cell.
The difference in the number of positively charged potassium ions inside and outside the cell dominates the
resting membrane potential.
The actions of the sodium potassium pump (Na+/K+) help to maintain the resting potential.
71. Patient comes with tender right hypochondrium; stool with occult blood on colonoscopy
ascending colon was involved showing ulcers while other parts of colon were spared on
histology/microscopy what will be seen?
a. Necrotizing Vasculitis
b. Crypt Abscess
c. Entamoeba Histolytica
d. Creeping Fat
Ans: C
Can‘t be B. As it is a feature of UC which involves the whole length of colon not only the ascending part
Can‘t be D As it is a feature of CD which most commonly involves rectum and there is no RHC pain
associated with that
Reference # Harsh Mohan # Robbins
72. Urine osmolarity for diagnosis of:
# Central DI:
After fluid deprivation: < 300
After desmopressin: >800
Nephrogenic DI:
After fluid deprivation: < 300
After desmopressin: < 300
Partial DI:
After fluid deprivation: 300-800
After desmopressin: < 800
73. In response to compensation of hemorrhage what will decrease?
a. Heart Rate
b. Cardiac Contractility
c. Venous Capacitance
d. Blood Pressure
Ans: A
74. In response to hemorrhage what will decrease?
A. Heart Rate
B. Cardiac Contractility
C. Venous Capacitance
D. Blood Pressure
Ans: C
Explanation:
Look at both scenarios and the difference.
In hemorrhage immediate response of body is vasoconstriction which in other words can be called as
increased TPR or decreased venous capacitance and this happens to compensate and minimize the blood loss
this happens in response to hemorrhage.
In the first scenario question is about in response to#compensation of hemorrhage which means after
compensation what will happen?
We know compensation and successful resuscitation is assessed by increased urine output, Normalization of
HR and BP so in that case we are going to optioned decreased HR.
SK Origional – Golden 11 398

Conclusion:
In response to Hemorrhage  Venous capacitance decreases
In response to compensation of Hemorrhage  HR decreases
74. Regarding hypoxia:
There are 4 main types of hypoxia
#Hypoxic Hypoxia decreased O2 tension causes include hypoventilation, high altitude
#Anemic Hypoxia decreased O2 carrying capacity causes include anemia CO poisoning In CO poisoning CO
binds with HB and occupies the O2 binding sites that's why causes anemic hypoxia.
#Stagnant/Circulatory Hypoxia d/t decreased tissue perfusion because of shock, HF or ischemia/infarction
#Histotoxic /Cellular Hypoxia cells can't utilize O2. Cytochrome oxidase system is paralyzed decreased SaO2
but normal PaO2 occurs in cyanide poisoning mainly.
75(a) Regarding dead space:
Actually dead space is ventilation without perfusion which increases in the following conditions like PE,
Hemorrhage, Hypotension, Surgical manipulation of pulmonary artery tree anything that decreases perfusion
to well ventilated alveoli Emphysema d/t loss of alveolar septa and vasculature and asthma anesthetized
patient and upright position.
Any area which is not well profuse or has decrease blood for V/Q maintenance may increase dead space
V/Q ratio directly proportional to dead space.
Atelectasis and shunt decreases dead space
75.(b) In chronic respiratory acidosis what is normal?
a. pH
b. Bicarbonate
Ans: A
In chronic respiratory acidosis renal compensation (increased HCO3 reabsorption) has occurred. Thus arterial
pH is increased towards normal (i.e. compensation).
Reference # BRS
76. In the presence of ADH which segment filtrate will be isotonic to plasma?
Ans: Cortical collecting duct
In the absence of ADH cortical and medullary collecting tubules and ducts are impermeable to water.
ADH increases the water permeability of these nephron segments and allows the filtrate to reach osmotic
equilibrium with the interstitial fluid surrounding the nephron. Interstitial fluid in the cortex of the kidney is
isotonic to plasma so the filtrate can become isotonic to plasma in the cortical collecting tubule On the other
hand interstitial fluid is hypertonic to plasma in the medullary collecting tubule so the filtrate becomes
hypertonic to plasma in this region of the nephron.
77. In a patient with hyperthyroidism TSH is low. TRH is given that will increase TSH where is
the problem?
a. Pituitary
b. Hypothalamus
c. Thyroid
Ans: C
#Explanation:
Primary hyperthyroidism increase T3/T4, decreases TSH
For secondary hyperthyroidism increase T3/T4 increase TSH but TRH is low problem is in pituitary
Here it is primary hyperthyroidism increase T3 /T4 have negative feedback on TRH, so TRH will be low and
TSH so if you will give TRH, it will increase creation of TSH so problem lies in Thyroid.
78. Sarcoidiosis is microscopically diagnosed by:
a. Granuloma with Asteroid
b. Non-Casseating Granuloma
Ans: B
#Explanation:
Microscopically Sarcoidiosis is diagnosed by non-Casseating granuloma. Presence of asteroid bodies is not
important and non-specific for the diagnosis as they may be presenting in granulomas of other origins too.
Reference# Robbins
SK Origional – Golden 11 399

79. Defecation is mainly carried out by:


a. Mass movement
b. Sacral Parasympathetic
Ans: B
#Explaination
The defecation reflex starts when the muscles in the colon contract to move stool towards the rectum which
is called ―mass movement‖. The myenteric defecation reflex is responsible for increasing peristalsis and
propelling stool toward the rectum. This eventually signals the internal anal sphincter to relax and reduce
sphincter constriction when enough stool moves to the rectum the amount of stool causes the tissues in the
rectum to stretch or distend inside these tissues are special #stretch receptors that signal the brain when they
are stretched.
The reflex triggers the two main sphincter surround the anal canal first is the internal anal sphincter which is
involuntary second is the external anal sphincter which is relatively voluntary.
The defecation reflex occur when the internal anal sphincter relaxes and the external anal sphincter contracts.
There is another reflex recto-anal inhibitory reflex is an involuntary internal anal sphincter relaxation in
response to rectal distention this type of defecation reflex is the parasympathetic defecation reflex. While the
motions of moving stool are similar a person can voluntarily control the parasympathetic defecation reflex
but they can‘t control the myenteric one.
So basically the concept here is that"carriedout"means"to complete/accomplish or finish something" which is
done by parasympathetic here and most important reflex is Recto-anal.
80 (a). A plastic factory worker exposure to hydrocarbons weight loss and fever cause?
a. Bladder carcinoma
b. Bronchogenic carcinoma
Ans: A
Hydrocarbons are more of a risk factor for Bladder CA than Bronchogenic CA
80. (b) A plastic factory worker chronic smoker and exposure to hydrocarbons weight loss and fever
Diagnosed as case of Bronchogenic CA cause?
a. Smoking
b. Hydrocarbons
Ans: A
Smoking major risk factor for CA of bronchus as compared to hydrocarbons.
80. (c) A plastic factory worker and chronic smoker exposure to hydrocarbons weight loss and fever
Cause?
a. Bladder carcinoma
b. Bronchogenic carcinoma
Ans: B
Smoking is more carcinogenic than hydrocarbon
81. Regarding gastrectomy
Partial Gastrectomy  IDA > Pernicious
Complete Gastrectomy  Pernicious > Megaloblastic > IDA
Gastrectomy before 6months  IDA
After 6months  Pernicious
82. Heat exhausation and heat stroke:
Heat Exhaustion  Sweating, Dizziness, cold periphery, weak pulse with tachycardia
Heat Stroke  Severe headache, no sweating, increased body temperature, tachycardia with strong pulse
83. Plasma osmolarity 300 and urine osmolarity 1200:
a. SIADH
b. Dehydration
Ans: B
Explanation:
This is case of #Dehydration/Water deprivation. This patient‘s plasma and urine osmolarity are consistent
with water deprivation. The plasma osmolarity is on the high side of normal stimulating the posterior pituitary
to secrete ADH which acts on the collecting ducts to increase water reabsorption produce hyper osmotic
urine.
SIADH would also produce hyperosmotic urine, but the plasma osmolarity would be lower than normal
means less than 270 because of the excessive water retention.
Central and Nephrogenic diabetes insipidus and excessive water intake would result in hypo-osmotic urine.
SK Origional – Golden 11 400

84. Site of Action:


ADH  Collecting Ducts > Distal Convoluted Tubule > Cortical Collecting Tubule
Aldosterone  Cortical Collecting Tubule > Distal Convoluted Tubule > Collecting Ducts
85. Which of the following decreases blood supply to brain?
a. I/V Epinephrine
b. Inhalation of 17% CO2
c. Seizures
d. Hyperbaric O2
Ans: D
#Explanation:
Blood supply of brain is CO2 dependent as it causes vasodilation so when therapeutic O2 is given it causes
vasoconstriction of cerebral vessels lead into decreased blood supply to brain.
86. Most common site of carcinoid tumor is:
a. Small Bowel
b. Appendix
Ans: A
Although most common site is Bronchus but when you have to choose one out of these 2 then it is Small
Bowel.
Most common tumor of #Appendix is carcinoid but appendix is not the most common site.
Ref: Big Robbins, Medscape
87. Inspiratory ramp signals are produced from dorsal respiratory neurons their rate increases in
response to impulses from?
a. Stretch receptors in Lungs
b. Pneumotaxic Centre in Pons
c. Apneustic Centre
d. Mid Brain
Ans: B
#Explanation:
Inspiratory ―Ramp‖ Signal the nervous signal that is transmitted to the inspiratory muscles mainly the
diaphragm is not an instantaneous burst of action potentials. Instead it begins weakly and increases steadily in
a ramp manner for about 2 seconds in normal respiration then it ceases abruptly for approximately the next 3
seconds which turns off the excitation of the diaphragm and allows elastic recoil of the lungs and the chest
wall to cause expiration next the inspiratory signal begins again.
For another cycle this cycle repeats again and again with expiration occurring in between thus the inspiratory
signal is a ramp signal the obvious advantage of the ramp is that it causes a steady increase in the volume of
the lungs during inspiration rather than inspiratory gasps.
A Pneumotaxic Center Limits the Duration of inspiration and increases the Respiratory Rate
it is located dorsally in the nucleus Para brachialis of the upper pons transmits signals to the inspiratory area.
The primary effect of this center is to control the ―switch-off‖ point of the inspiratory ramp thus controlling
the duration of the filing phase of the lung cycle When the Pneumotaxic signal is strong inspiration might last
for as little as 0.5 seconds thus filling the lungs only slightly when the Pneumotaxic signal is weak.
Inspiration might continue for 5 or more seconds thus filling the lungs with a great excess of air. The
function of the Pneumotaxic center is primarily to limit inspiration. This has a secondary effect of increasing
the rate of breathing because limitation of inspiration also shortens expiration and the entire period of each
respiration. A strong Pneumotaxic signal can increase the rate of breathing to 30 to 40 breaths per minute
where as a weak Pneumotaxic signal may reduce the rate to only 3 to 5 breaths per minute?
Reference #Guyton
88. Anatomy:
• Nerve injured in Ischio-rectal drainage Inferior Rectal Nerve
• Nerve supply to Adrenal Gland Greater Splanchnic Nerve
• Which Lobe/Segment of Liver is directly in contact with IVC Caudate
• Para-aortic Lymph Nodes at L2 to L4 Level
• Prostate Cancer Drains to which nodes Internal iliac Lymph Nodes
• Dependant part of abdomen while supine is right sub hepatic (Morrison Pouch) and while standing is
Pouch of Douglas in female and para-colic gutters in males
• What lies posterior and superior to epi-ploic foramen (PosteriorIVC SuperiorCaudate Lobe)
• Which ligament transmit tumors to Labia Majora Round ligament
• Which part of liver is directly in contact with diaphragm Bare Area
SK Origional – Golden 11 401

89. Most common nerve damage in Appendectomy?


a. Iliohypogastric Nerve
b. Ilioinguinal Nerve
Ans: A
90. Most commonly damaged structure in Appendectomy?
a. Iliohypogastric Nerve
b. Deep circumflex iliac Artery
c. Superficial External iliac Vessels
d. Ilioinguinal Nerve
Ans: A
91. A 33-year-old man with a perforated gastric ulcer complains of excruciating pain in his
stomach. It is observed that the pain comes from peritoneal irritation by gastric contents in
the lesser sac. Which of the following nerves contain sensory nerve fibers that convey this
sharp stabbing pain?
a. Vagus nerves
b. Greater splanchnic nerves
c. Lower intercostal nerves
d. White rami communicants
e. Gray rami communicants
Ans: C
92. A young boy is brought to the hospital after a bicycle accident and possible pelvic fracture.
While awaiting a computed tomography (CT) scan of his pelvis, a physician proceeds with a
focal neurologic examination. In testing the child‟s reflexes, which of the following nerves
would carry afferent impulses of the cremasteric reflex?
a. Subcostal Nerve
b. Lateral Femoral Cutaneous Nerve
c. Genito-femoral Nerve
d. Iliohypogastric Nerve
e. Femoral Nerve
Ans: C
93. A 67-year-old woman with a long history of liver cirrhosis was seen in the emergency
department. In this patient with portal hypertension, which of the following veins is most
likely to be dilated?
a. Right Colic Vein
b. Inferior Epigastric Vein
c. Inferior phrenic Vein
d. Suprarenal Vein
e. Ovarian vein
Ans: A
94. A 57 year-old patient has a tumor in the body of the pancreas that obstructs the inferior
mesenteric vein just before joining the splenic vein. Which of the following veins is most
likely to be enlarged?
a. Middle Colic Vein
b. Left gastroepiploic Vein
c. Inferior pancreatico-duodenal Vein
d. Ileocolic Vein
e. Left Colic Vein (Left colic vein is tributary of inferior mesenteric vein)
Ans: E
95. While examining radiographs and angiograms of a 52-year-old patient, a physician is trying
to distinguish the jejunum from the ileum. He has observed that the jejunum has?
a. Fewer Plicae Circulares
b. Fewer Mesenteric Arterial Arcades
c. Less Digestion and Absorption of Nutrients
d. Shorter Vasa Recta
e. More Fat in its Mesentery
Ans: B
The jejunum has fewer mesenteric arterial arcades but longer vasa recta than the ileum. The plicae circulares
(circular folds) are tall and closely packed in the jejunum and are low and sparse in the ileum, and the lower
part of the ileum has no plicae circulares. More digestion and absorption of nutrients occurs in the jejunum
than in the ileum, and less fat is found in the mesentery of the jejunum.
96. An acute infection involving the dartos muscle layer of the scrotum most likely leads to an
enlargement of which of the following lymph nodes?
a. Preaortic Nodes
b. Lumbar Nodes
c. External iliac Nodes
SK Origional – Golden 11 402

d. Superficial inguinal Nodes


e. Common iliac Nodes
Ans: D
97. Sigmoid Colon:
a. Is drained by systemic veins
b. Is a retroperitoneal organ
c. Receives parasympathetic fibers from the vagus nerve
d. Receives its blood from the superior mesenteric artery
e. Has teniae coli and epi-ploic appendages
Ans: E
98. The surgeon will ligate all arteries that send branches to the stomach. Which of the following
arteries may be spared?
a. Splenic artery
b. Gastro-duodenal artery
c. Inferior pancreatico-duodenal artery
d. Left gastroepiploic artery
e. Proper hepatic artery
Ans: C
99. A 38-year-old woman with peptic ulcer disease of the stomach experiences severe abdominal
pain. Which of the following nervous structures is most likely involved?
a. Greater Splanchnic Nerve
b. Ventral Roots of the Spinal Nerve
c. Lower intercostal nerve
d. Vagus Nerve
e. Gray Ramus Communicants
Ans: A
100. Postal ductal coarctation of the aorta flow to lower limb is maintained by increasing blood
flow through?
a. Inferior phrenic and pericardiophrenic vessels
b. Intercostal and Superior Epigastric
c. Subcostal and umbilical
Ans: B
101. Most commonly seen renal vessel anomaly is:
a. Supernumerary Renal Artery
b. Supernumerary Renal Vein
Ans: A
102. BPH involves which prostatic lobe?
a. Posterior Lobe
b. Median Lobe
c. Entire Prostate Gland
Ans: B
103. CA Prostate lobe commonly involves is:
a. Median Lobe
b. Posterior Lobe
Ans: B
104. 50 years old man with CA prostate showed sclerosis and collapse of T10 & T11 vertebrae on
x- ray. This spread is due to?
a. Internal Vertebral Plexus of Veins
b. Lymphatic
Ans: A
105. The endocrine portion of pancreas is derived from:
a. Mesoderm
b. Endoderm
Ans: B
106. Pain caused by passage of a stone down the lower end of the left ureter may be referred?
a. Hypogastric Region
b. Penis or Clitoral
c. Lumber Region
Ans: B
107. Short gastric arteries are branches of:
a. Splenic Artery
b. Left Gastric Artery
Ans: A
108. Left gastroepiploic artery is branch of:
A. Celiac Trunk
SK Origional – Golden 11 403

B. Gastro-duodenal Artery
C. Splenic Artery
Ans: C
109. Truncal Vagotomy:
a. Decrease gastric secretion
b. Decrease gastric emptying
Ans: A
110. Thoracic vagotomy:
a. Decrease gastric secretion
b. Decrease gastric Motility
Ans: B
111. Insulin dependent uptake in:
a. Exercising muscle
b. Myocardium
c. Hypothalamus
Ans: B
112. CVS Physiology:
• Alpha 1- Adrenergic receptors are found on the arterioles of the skin, splanchnic, and renal
circulations
• Beta 2- Adrenergic receptors are found on arterioles of skeletal muscle
• Arterioles are the site of highest resistance in the cardiovascular system
• Capillaries have the largest total cross-sectional and surface area
• Veins contain the highest proportion of the blood in the cardiovascular system called unstressed
volume have α1-adrenergic receptors
• Right atrial pressure is measured by JVP
• Left atrial pressure is estimated by the pulmonary wedge pressure
• QT interval represents the entire period of depolarization and repolarization of the ventricles
• ST segment represents the period when the ventricles are depolarized
• Plateau phase is caused by a transient increase in inward Ca2+ current, and by an increase in K+
conductance
• Fatty acids are the primary energy source for stroke work
• ANP right atrium
• BNP right ventricle
• Skin does not have auto-regulation
• The most important local vasodilator for the cerebral circulation is C02
113. Regarding pressure of chambers of heart:
• Maximum Aortic Pressure  Slow Ejection Phase
• Minimum aortic pressure  Iso-volumetric Contraction Phase
• Maximum intraventricular pressure  Rapid Ejection Phase
• Minimum intraventricular pressure  Rapid Filling Phase
114. Which of the following normally has the most prominent prepotential?
a. Sinoatrial Node
b. Atrial Muscle Cells
c. Bundle of His
d. Purkinje Fibers
e. Ventricular Muscle Cells
Ans: A
115. Cardiac output of the right side of the heart is what percentage of the cardiac output of the
left side of the heart?
a. 25%
b. 50%
c. 75%
d. 100%
SK Origional – Golden 11 404

e. 125%
Ans: D
116. The work performed by the left ventricle is substantially greater than that performed by the
right ventricle, because in the left ventricle:
a. The contraction is slower
b. The wall is thicker
c. The stroke volume is greater
d. The preload is greater
e. The afterload is greater
Ans: E
117. When a person moves from a supine position to a standing position, which of the following
compensatory changes occurs?
a. Decreased heart rate
b. Increased contractility
c. Decreased total peripheral resistance (TPR)
d. Decreased cardiac output
e. Increased PR intervals
Ans: B
118. At which site is systolic blood pressure the highest?
a. Aorta
b. Central vein
c. Pulmonary Artery
d. Right Atrium
e. Renal Artery
Ans: A
119. Which of the following would you expect to decrease in a normal individual who stands
quietly in the same position for l hour?
a. Haematocrit
b. Diameter of the Thigh
c. Plasma Renin Activity
d. Plasma Vasopressin Concentration
e. Central Venous Pressure
Ans: E
120. Which of the following organs has the greatest blood flow per l00 gram of tissue?
a. Brain
b. Heart muscle
c. Skin
d. Liver
e. Kidneys
Ans: E
121. Which of the following organs has the most permeable capillaries?
a. Brain
b. Posterior pituitary gland
c. Liver
d. Small intestine
e. Kidneys
Ans: C
122. Which of the following is not increased during isotonic exercise?
a. Respiratory rate
b. Stroke volume
c. Heart rate
d. Total peripheral resistance
e. Systolic blood pressure
SK Origional – Golden 11 405

Ans: D
123. Which of the following agents or changes has a negative inotropic effect on the heart?
a. Increased heart rate
b. Sympathetic stimulation
c. Norepinephrine
d. Acetylcholine (ACh)
e. Cardiac glycosides
Ans: D
124. The low-resistance pathways between myocardial cells that allow for the spread of action
potentials are the:
A. Gap junctions
B. T tubules
C. Sarcoplasmic reticulum (SR)
D. Intercalated disks
E. Mitochondria
Ans: A
125. Which agent is released or secreted after a haemorrhage and causes an increase in renal Na+
reabsorption?
A. Aldosterone
B. Angiotensin I
C. Angiotensin II
D. Antidiuretic hormone (ADH)
E. Atrial natriuretic peptide
Ans: A
126. Regarding pulse pressure:
a. It is decreased in shock
b. It is difference between systolic & diastolic pressure
c. It is diastolic pressure + 1/3 of systolic pressure
d. It is normal in cardiac failure
e. It is 1/3 of systolic pressure
Ans: B
127. High output cardiac failure is associated with:
a. Vit. C
b. Riboflavin
c. Vit. K
d. Thiamine
e. Vit. B12
Ans: A
128. Cimetidine may significantly prolong the Prothrombin time and cause bleeding in patient
who has been treated with warfarin because of?
a. Decreasing the hepatic clearance of warfarin
b. Displacing warfarin from plasma protein
c. Increasing the oral bioavailability of warfarin
d. Inhibiting renal tubular secretion of warfarin
e. Inhibiting the conversion of Prothrombin to thrombin
Ans: A
129. T wave occurs during:
a. Depolarization of Atria
b. Hyperpolarization of Ventricle
c. Repolarization of Ventricle
d. Depolarization of Ventricle
e. Atrial Repolarization
Ans: C
SK Origional – Golden 11 406

130. An unconscious patient is pale, cold and clammy. His pulse is rapid and thready. His mean
arterial blood pressure is 50mm of Hg. The mechanism which provides maximum
compensation at this stage of shock is?
a. Baroreceptor reflex
b. Brain bridge reflex
c. C.N.S. ischaemic response
d. Chemo receptor reflex
e. Volume reflex
Ans: C
131. Pulse pressure is increased in:
a. HTN
b. Haemorrhage
c. Heart failure
d. Aortic stenosis
e. Tachycardia
Ans: A
132. End systolic volume in a normal healthy person is:
a. 30 ml
b. 50 ml
c. 60 ml
d. 70 ml
e. 80 ml
Ans: B
133. In a young athlete at rest, the heart has:
a. Increase heart rate
b. Increase stroke volume
c. Increase cardiac output
d. Increased vascular resistance
e. Increased coronary blood flow
Ans: B
134. Least HB in newborn at which age?
a. 1st month
b. 6th month
c. 3rd month
d. 1st year
Ans: C
135. In a patient with raised intracranial pressure, the blood flow to the brain tends to be
maintained by:
a. Cushing‘s reflex
b. Auto regulation
c. Baroreceptors
d. Chemoreceptors
e. Low pressure stretch receptors
Ans: A
136. The following factors are associated with elevated blood levels of high density lipoprotein
(HDL) except?
a. Diabetes Mellitus
b. Oestrogen Therapy
c. Family History of Longevity
d. Low levels of Very Low Density Lipoprotein (VLDL)
e. Regular Strenuous Exercise
Ans: A
137. Following are the findings in normal ECG except:
SK Origional – Golden 11 407

a. Maximum duration of normal P-R interval 0.1 sec


b. P wave is due to atrial excitation
c. QRS complex is due to ventricular excitation
d. R wave may be inverted in lead I and upright in lead III
e. T wave occurs during ventricular repolarization
Ans: A
138. In which of following condition PO2 is always decreased?
a. VSD
b. Coarctation of Aorta
c. Left to Right Shunt
d. Right to Left Shunt
e. Pulmonary Hypertension
Ans: D
139. Malaria:
 Shortest pre-erythrocytic phase seen in Falciparum
 Longest proerythrocytic intrahepatic Malariae
 Species that cause relapse Vivax>Ovale
 Malignant tertian malaria Falciparum
 Benign tertian malaria Ovale/Vivax
 Quartan malaria Malariae
 Quotidian malaria  Falciparum /Knowlesi
 Anemia in malaria Normocytic Normochromic
 Dormant phase of malaria Hypnozoite
 Malaria spread to human Sporozoites
 Sporozoites divide in liver is Merozoites
 Hypnozoite present in ovale and Vivax
 New species of malaria Knowlesi
140. Head & Neck:
Lesser Petrosal Nerve  Parotid Gland
3rd part of Subclavian Artery lies in  Posterior Triangle
Danger Area of Scalp  Loose Areolar Tissue
Excessive spasm of Lateral Pterygoid leads to Dislocation of  TMJ
Frontal Sinus is not present at birth and appear at  2nd year of life
Skull has  22 Bones
Nerve Supply of Thymus  Phrenic Nerve
Main Supply of Tonsil  Facial Artery
Widest Epidural Space  L2
Length of Spinal Cord  18 Inch
Primary Somatosensory Cortex Area  3, 2, 1.
Pre Motor Cortex Area  6
Frontal Eye Field Area  8
Primary Motor Area  4
Primary Auditory Cortex Superficial Temporal Area  41, 42
Wernicke‘s Area  22
Brocas Area  44
141. Nasolacrimal duct drains into:
SK Origional – Golden 11 408

a. Superior meatus
b. Inferior Meatus
c. Middle Meatus
Ans: B
142. Protrusion of tongue muscle involves:
a. Palatoglossus
b. Genioglossus
c. Styloglossus
Ans: B
143 Which tongue papillae has taste receptors?
a. Fungiform
b. Folliate
c. Circumvallate
d. Filliform
Ans: C
144. Which papillae of tongue impart red colour to tongue?
a. Fungiform
b. Filliform
Ans: A
145. An old man with crocodile tears syndrome which of the following nerve has been injured?
a. Facial nerve at stylomastoid foramen
b. Lacrimal nerve
c. Facial nerve proximal to the geniculate ganglion
Ans: C
146. Dryness of nose and the palate lesion in which ganglion?
a. Pterygopalatine Ganglion
b. Otic Ganglion
c. Ciliary Ganglion
Ans: A
147. Numbness above the vocal card?
a. External Laryngeal Nerve
b. Internal Laryngeal Nerve
c. Recurrent Laryngeal Nerve
Ans: B (Below  Recurrent Laryngeal Nerve)
148. Patient loss cutaneous sensation over tip of nose:
a. Maxillary Nerve
b. Ophthalmic Nerve
c. Facial Nerve
Ans: B
149. Muscle of protrusion of jaw:
a. Medial pterygoid
b. Lateral pterygoid
c. Masseter
Ans: B
150. Superior laryngeal artery is associated with:
a. External laryngeal nerve
SK Origional – Golden 11 409

b. Internal laryngeal nerve


c. Hypoglossal nerve
Ans: B
151. Inferior thyroid artery is associated with:
a. Recurrent Laryngeal Nerve
b. Internal Laryngeal Nerve
Ans: A
152. External laryngeal nerve associated with:
a. Superior Thyroid Artery
b. Middle Thyroid Artery
c. Inferior Thyroid Artery
Ans: A
153. Numbness of nasophyrnx after adenoid removal nerve involves is:
a. Vagus Nerve
b. Glossopharyngeal Nerve
Ans: B
154. Which muscle act as sphincter at pharynx?
a. Middle constrictor
b. Cricophyrngeous
Ans: B
155. All muscles of tongue are supplied by hypoglossal except?
a. Styloglossus
b. Palatoglossus
c. Hyoglossus
Ans: B
156. Pure serous gland:
a. Parotid
b. Submandibular
Ans: A
157. Most common artery involve in posterior nasal bleeding is:
a. Anterior ethmoidal artery
b. Posterior ethmoidal artery
c. Lateral septal artery
Ans: B
158. Immunoglobulins:
 Most Abundant in blood stream IgG>IgA>IGM >IgD>IgE
 Least abundant IgE
 Maximum production per day IgA which is most abundant in body secretions
 IgG cross placenta
 IGM pentamer can‘t cross placenta
 IgA both monomer and diamer
159. The neuromuscular bundle present between:
a. External and Internal Intercostal
b. Internal Intercostal and Innermost Intercostal
c. Innermost intercostal and cardiothoracic fascia
d. External Intercostal and Skin
SK Origional – Golden 11 410

Ans: B
160. Base of the heart is formed by:
a. Left Atrium
b. Right Atrium
c. Right Ventricle
d. Left Ventricle
Ans: A
161. 22 years old man Sickle cell anaemia, active complaints pallor lethargy headache. Labs are
HB is 4.6 retic is 3%. Parvovirus infection is suspected, what is your diagnosis?
a. Thrombotic crisis
b. Sequestration crisis
c. Haemolytic crisis
d. Aplastic crisis
Ans: D
#Explanation:
• Sequestration crisis a sickling of RBCs within organs like spleen or lungs.
• Acute chest syndrome very painful.
• Aplastic crisis a Infection with parvovirus. Sudden fall in HB.
• Haemolytic crisis  Rare Fall in Hb due to increase haemolysis.
• Thrombotic or painful crisis  Painful or vasoocclusive  Precipitated by infection, dehydration
and deoxygenation.
• Avascular necrosis of head of femur, hand foot syndrome in children.
162. Regarging bladder carcinoa:
Risk factors in order of frequency:
• Squamous Cell Carcinoma  Schistosomiasis > Calmette Guerin Treatment> Smoking
• Transitional Carcinoma  Smoking > Exposure to Aniline Dye in Printing Industry >Rubber
Manufacture. > Cyclophosphamide.
163. Regarging Ligament:
Anterior Cruciate Ligament Prevents
a. Anterior dislocation of Tibia on Femur
b. Posterior dislocation of Femur on Tibia
Posterior Cruciate Ligament Prevents
a. Anterior dislocation of Femur on Tibia
b. Posterior dislocation of Tibia on Femur
164. Coagulopathy related to liver disease which clotting factor is characteristically increased?
a. Factor 8
b. Factor 2
c. Factor IX
d. Factor 7
e. Factor 12
Ans: A
#Explanation:
In liver failure all clotting factors are low except for factor 8 which is paradoxically supernormal because it is
synthesized by endothelial cells throughout the body and for clearance of activated factor 8 good hepatic
function is required leading to increased circulating level of Factor 8.
165. ATT drugs side effects:
• Streptomycin Ototoxity
• Isoniazid Neuropathy
SK Origional – Golden 11 411

• Pyrazinamide Gout, Severe Hepatotoxicity


• Rifampicin Orange/Red Colour Urine
• Ethambutol Optic Neuritis
166. Pathological conditions:
• Premalignant change in mouth Chronic Ulcer
• Premalignant Condition Lichen Planus
• Premalignant Lesion Leucoplakia (Most Common)
• Premalignant Lesion Erythroplakia (Most Lethal)
• Bettlenut Chewing  Submucosal Fibrosis
167. Pan-hypopituitarism is not associated with  Cachexia.
Reference: Ganong physiology
168. Room temp 21 °Humidity 80% heat is lost from uncovered body by  Radiation and
Conduction.
Reference: Ganong Physiology
169. Diabetic Autonomic neuropathy manifests in GIT as GastroparesisRx is Metoclopramide
Reference: Katzung Pharma
170. The pressure diff between the heart and aorta is least in the Left Ventricle during Systole
Reference: Ganong Physiology
171. Regarding GFR:
• Best way to measure GFR Inulin Clearance
• Best way to estimate GFR Creatinine Clearance
• Best way to measure renal Plasma/Blood Flow PAH
• Best way to measure GFR clinically Creatinine Clearance
• Highest renal clearance PAH
172. Regarding Ulcers:
• Martorell  Uncontrolled HTN
• Marjolin SCC/External Burn
• Meleny Microaerophilic Strept/Staph
• Cushing Raised ICP
• Curling Inhalational Burn (Gastric)
• Trophic/Neurotrophic ulcer Untreated Lower Limb venous Ulcers
• Malum Perforans Pedis Trophic Ulcer on Sole of Foot
173. Regarding thyroid:
• Primary Hypothyroidism (Thyroid dysfunction) T3/T4 & TSH
• Secondary Hypothyroidism T3/T4 & TSH
• If TSH is low now give TRH If TSH increases means hypothalamic disorder because Pituitary is
functional (Tertiary hypo) If TSH does not increase It means Pituitary disorder (Secondary hypo).
174. Regarding markers:
Desmin Muscle
Keratin Epithelial
Vimentin Mesenchymal
Neurobibrillin Neuron
GFAP Oligodendrocytes
S100 Melanoma
Chromogranin or 5HIAA Carcinoid
Neuron Specific Enolase Small Cell Cancer
Blue cells Ewing Medulloblastoma and Kulchitsky Cells of Small Cell Lung Cancer
175. Pharmacology:
• Therapeutic Dose Potency
SK Origional – Golden 11 412

• Therapeutic Index Safety


• Therapeutic Effectiveness Efficacy
• Therapeutic Window Therapeutic Dose Range
176. Anatomy:
• Tarsal Bones 7 in Number
• CalcaneumLargest Tarsal Bone
• Cuneiform Forms Transverse Arch of Foot
• Talus Has no Muscular Attachment
• Nerve Affected in Tarsal Tunnel Syndrome Tibial Nerve
• Locking of Knee Quadriceps Femoris
• Unlocking of Knee Popliteus Muscle
• All Short Muscles of Hand involved C8T1
• Lateral Part of Thenar Eminence spared Ulnar Nerve
• Ring finger has got sensory supply from all three nerves Ulnar/Radial/Median Nerves
• Thumb has got motor supply from All Three Nerves Ulnar+Radial+Median
• Chief gripping muscle of hand Flexor Digitorum Profundas
• Breast surgery Damage to Long Thoracic Nerve Winging of Scapula
• Teres major muscle Lower Scapular Nerve Medially Rotates and Adducts the Shoulder
• Teres minor Axillary Nerve Laterally Rotates the Arm
• Musculocutanous Nerve C567
• Pisiform give attachment to both Flexor and Extensor Retinaculum
• Basilic Vein CVP Line
• Labourer Nerve Median Nerve
• Musician Nerve Ulnar Nerve
• Ulnar Artery Larger than Radial Artery
• Saddle joint Base of Thumb
• Ulnar Artery and nerve pass above flexor retinaculum on medial side of hook of hamate.
177. Erythropoiesis occur in all except?
a. Kidney
b. Spleen
c. Long bones
d. Flat bones
Ans: A
178. A 15 years old female presents with recurrent Sino pulmonary infection. What test is most
likely to confirm primary immunodeficiency?
a. IgG Level
b. IgM Level
c. IgA Level
Ans: A
#Explanation:
• Most common primary immuno deficiency is IgA deficiency but it is most commonly asymptomatic
• Most common clinically significant immune deficiency is CVID (common variable immune
deficiency)
• It is characterised by 3 features Hypogammaglobulinemia of two or more subtypes IgG, IgA or IgM
• Most important IgG in Recurrent Sino-pulmonary infection & impaired functional antibody response
179. Which muscle is flexor of forearm and supplied by radial nerve?
a. Brachialis
b. Flexor Carpi Radialis
c. Brachioradialis
d. Flexor Digitorum Profundas
SK Origional – Golden 11 413

e. Tricep Brachii
Ans: C
180. A patient climbed a peak with a heavy bag on his shoulder after that he develop loss of
adduction of arm with loss of flexion at elbow and loss of extension at wrist. Most likely
damage occurs to?
a. Radial Nerve
b. Musculocutanous Nerve
c. Axillary Nerve
d. Upper Part of Brachial Plexus
Ans: D
181. Loss of extension of little and ring finger, along with Hypothenar atrophy is seen in injury
of?
a. Radial Nerve
b. Ulnar Nerve
c. Median Nerve
d. Posterior introsseus Nerve
Ans: B
182. Patient presented with weakness of all short muscle of hand and #weak #radial pulse
diagnosis?
a. Ulnar Nerve Damage
b. C8 -T1 Lesion
c. Cervical Rib
Ans: C
183. Axillary sheath is continuation of:
a. Pre-Vertebral Fascia
b. Deep Cervical Fascia
Ans: A
184. Patient with flattened Thenar eminence, sensory loss at lateral three and half fingers nerve
involve:
a. Carpel tunnel syndrome
b. Median nerve trapped at middle of forearm
c. Anterior introsseus nerve
Ans: A
185. Muscles of hand are supplied by:
a. Ulnar Nerve
b. Median Nerve
c. Radial Nerve
d. Ventral Branches of T1
e. Ventral Rami of C8
Ans: D
186. Axillary vein is formed by:
a. Union of Basilic Vein and Venae Comitantes
b. Union of Cephalic Vein and venae Comitantes
c. Union of Basilic Vein and Cephalic Vein
Ans: A
187. Carpometacarpal joint is:
SK Origional – Golden 11 414

a. Hinge Joint
b. Ball and Socket
c. Saddle Joint
Ans: C
188. About axillary artery which is true?
a. Divided into 2 parts
b. Pectoralis minor crosses it posteriorly
c. It extends from lateral border of first rib to upper border of Teres Minor
d. It extends from lateral border of first rib to lower border of Teres Minor
e. It extends from lateral border of first rib to lower border of Teres Major
Ans: E
189. Humeral surgical neck fractures which artery damages?
a. Profunda Brachii Artery
b. Anterior Circumflex Artery
c. Brachial Artery
d. Posterior Circumflex Artery
e. Axillary Artery
Ans: D
190. In rotator cuff muscles which muscle is more frequently damages?
a. Supraspinatus
b. Infraspinatus
c. Teres Minor
d. Subscapularis
Ans: A
191. At the site of taking radial pulse, radial artery relations are:
a. Brachioradialis on medial side and flexor carpi radialis on lateral side
b. Brachioradialis on lateral side and flexor carpi radialis on medial side
c. Brachioradialis on medial side and flexor carpi ulnaris on lateral side
d. Flexor carpi ulnaris on medial side and flexor carpi radialis on lateral side
Ans: B
192. Opponens Policis is supplied by:
a. Median Nerve
b. Ulnar Nerve
c. Radial Nerve
Ans: A
193. Pectoralis major muscle action:
a. Abducts the arm
b. Adducts and rotates laterally
c. Adducts and rotates medially
d. Adducts, rotates medially and flex the arm
Ans: D
194. Patient dropped from tree. He holds a branch of tree to save himself but he injured his
nerve?
a. Radial Nerve
b. Upper Trunk
c. Lower Trunk
SK Origional – Golden 11 415

d. Posterior Cord
Ans: C (BAG on shoulder or over extension of shoulder Upper Trunk Injury)
195. Rhomboids are supplied by:
a. Axillary Nerve
b. Supra-scapular Nerve
c. Lower Sub-scapular Nerve
d. Dorsal Scapular Nerve
Ans: D
196. Nerve lesions upper limb:
• Fracture of Surgical Neck of Humerus or Dislocated Humerus Axillary Nerve
• Musculocutanous Nerve Lesion  Rare
• Saturday Night Palsy mid-shaft fracture of Humerus, Subluxation of Radius, fracture of Lateral
Epicondyle Radial nerve
• Carpal Tunnel, supracondylar fracture, Pronator Teres Syndrome Median Nerve
• Fracture of Medial Epicondyle, Fracture of Hook of Hamate, Fracture of Clavicle Ulnar nerve
• Surgical Neck of Humerus Axillary Nerve Injury
• Anatomical Neck of Humerus Radial Nerve Injury
197. Carpal bones:
• Common Fracture Scaphoid
• Common Dislocated Lunate
• Hook of Hamate Fracture Ulnar Nerve Palsy
• Dislocation of Lunate Median Nerve Palsy
• Fracture of Scaphoid Radial artery
• Fracture dislocation of Scaphoid Carpal tunnel syndrome
• Capitate Largest Carpal Bone
• Pisiform Smallest Bone
• Capitate Ossifies First in 1st year
• Pisiform Ossifies last 10 year
198. Unable to grip a glass of water win his right hand after fell asleep in an arm chair the most
likely nerve affected?
a. Radial Nerve
b. Ulnar Nerve
c. Median Nerve
d. Posterior Cord of Brachial Plexus
Ans: A
199. Lady producing thick saliva:
A. Alpha1Beta 1
B. Alpha1
C. Beta1 and alpha2
D. Beta2
Ans: A
200. Inferior angle of scapula:
a. T2
b. T7
Ans: B
201. The surface marking of the oblique fissure of lung would be:
a. From T2 spine around thorax to 5th costochondral junction
b. From T2 spine around thorax to 6th costochondral junction
c. From T3 spine around thorax to 6th costochondral junction
d. From T4 spine around thorax to 6th costochondral junction
Ans: C
SK Origional – Golden 11 416

202. Patient is having myocardial infarction. Posterior 1/3rd on inter ventricular septum is
involved which artery is affected?
a. Right coronary Artery
b. Left coronary Artery
c. Anterior Descending Artery
Ans: A
203. About trachea wrong is:
a. Starts at C6
b. Bifurcates at T4
c. Is About 15 cm
d. Starts at superior border of cricoid
e. Has 16 to 20 incomplete cartilage
Ans: D
204. ST elevation in lead II III aVF artery involved is:
a. Left Anterior Descending
b. Right Coronary Artery
c. Left Circumflex Artery
d. Posterior Descending Artery
Ans: B
205. Thorax:
• Contraction of Diaphragm Causes Increase Thoracic Volume
• Major Inspiratory Muscle Diaphragm
• Diaphragmatic hernia occurs through Oesophageal Opening & Costovertebral Opening
• Most common site of Morgagni hernia  Right Anterior
• Congenital Diaphragmatic Hernia Incomplete Pleuroperitoneal Membrane
• Patient can‘t survive without ventilation if lesion Above C5
• Patient can survive without ventilation if lesion Below C5
• Medial Arcuate Ligament Arches over psoas muscle as it passes through it
• A Stab at 6th Costal Cartilage Just Right will Damage Right Atrium
• Best Site of Aspiration of Pleural Fluid 4th intercostal Space Midaxillary Line
• Intercostal Space  1 Large Posterior and 2 Small Anterior Intercostal Arteries
206. Tuberculosis:
• Initial Chest x-ray
• Definitive Diagnosis Culture
• Microscopic Diagnostic AFB
• Microscopic Caseous Necrosis
• Histologic Epithelioid Cells with Casseating Granuloma
• Type of Hypersensitivity Delayed Type 4
• MarginsUndermined
207. Insulin and Drugs:
• Short Term Control of Blood Sugar Regular Insulin
• Long Term Control of Blood Sugar Metformin (S/E Lactic Acidosis+ Weight Loss+ Metallic
Taste)
• Starting new drug in obese patient Metformin
• Starting new drug in elder patient Tolbutamide
• Excessive Exercise Causes Hyperkalemia
• Insulin causes Hypokalemia
208. Deep vein thrombosis:
• Most common source of Emboli Femoral Vein
SK Origional – Golden 11 417

• Most common site of DVT Popliteal Vein


• Most common cause  Immobilization
• Sensitive for DVT D dimers
• Specific for DVTFDP
209. Congenital cardiac problems:
• Most Common Congenital Disorder  VSD
• Most Common Congenital Cyanotic Disorder Tetralogy of Fallot
• Most Common Congenital Cyanotic Anomaly at Birth Transposition of Great Vessel
• Female living at Hill or if Baby is Premature Patent Ductus Arteriosus
• Hyaline disease/disease of Prematurity Decreased Type 2 Pneumocytes (Decreased surfactant
increased tendency of lungs to collapse Betamethasone is the Treatment
210. Lymphatics Drainage:
• Breast Upper Lateral Quadrant Anterior/Pectoral Group Nodes
• Breast Medial Quadrant Medial Group Nodes
• Stomach Celiac Nodes
• Uterus Ovary and Testis Para-aortic Lymph Nodes
• Cervix Internal and External Iliac Nodes
• Prostate Internal Iliac Nodes
• Lateral Surface of Foot + Posterior Compartment of Leg Popliteal Group of Nodes
• Great Toe Horizontal group of Superficial Inguinal Nodes
• Above Pectinate Line Internal Iliac Nodes
• Below Pectinate Line Superficial Inguinal Nodes
• Glans Penis Deep Inguinal Nodes
• Urinary Bladder Internal and External iliac Nodes
211. Collagen and Wound Healing:
• Type 1 collagen Fibrocartilage, Bone, Tendons and Fascia
• Type 2 collagen Elastic and Hyaline Cartilage + Vitreous Body
• Type 3 collagen Blood Vessels + uterus
• Type 4 collagen Basement membrane + Lens
• Early wound healing + Granulation Tissue Type 3 collagen
• Late Wound Healing + Wound Strength Type 1 collagen
• Hyaline Cartilage Larynx + Articular Surfaces of Synovial Joints
• Elastic cartilage Ear Pinna + Nose
• Diet Deficient in Fruits and Vegetables Decreased Tensile Strength
• Vitamin C Hydroxylation of Glycine and Proline
• Factor for Delayed Wound healing Infection
• Systemic factor for delayed Wound healing  Anemia> Malnutrition
• Old age decreased wound healing Hormonal and Endocrinal Changes
• Old man fell and fractured his Bone and delayed wound healing Decreased Blood Supply
212. Blood:
• Factor V Mutation (Leiden) leads t0 Thrombosis
• Factor V deficiency leads to Bleeding
• Factor 12 deficiency leads Thrombosis
• Most common acquired thrombotic disorder is Anti Phospholipid Antibody
Syndrome
• Most common inherited coagulopathy vWB Disease
• Most common inherited disorder is Factor V Mutation (Leiden)
• Natural anticoagulant and ant-thrombotic Heparin
• Natural Thrombolytic  Plasmin>Plasminogen
213. Coagulation Pathway:
• Extrinsic and intrinsic pathway converge on Stuart factor (Factor X)
• Factor VIII is produced by Endothelium
SK Origional – Golden 11 418

• Clotting factor not completely synthesized in liver is Factor VIII


• Action of extrinsic pathway by Thromboplastin
• In vitro coagulation is initiated by Factor XII
• Vitamin K dependent factor with shortest half-life is --- Factor VII
• Vitamin K dependent factor with longest half-life is Factor II
214. Lab findings in haematology:
• aPTT raised only Hemophilia
• aPTT and BT raised vWB Disease
• aPTT and PT raised Clotting factors deficiency or Liver disease
• If all 3 raised DIC
• If only BT raised ITP, HUS or TTP
215. Related to Pregnancy:
• Most common cause of recurrent spontaneous abortions Anti-phospholipid
antibody syndrome
• Most common cause of abortion in first trimester is Chromosomal abnormalities
• Chromosomal causes are overall most common cause Abortion in 2nd trimester is
mostly related to maternal causes like fibroids or Cervical Abnormalities
• Down Syndrome Most common chromosomal abnormality in Pregnancy
216. Heparin vs warfrarin:
• Heparin Acts on Anti Thrombin III
• Heparin inhibits Factor X
• Anti-dote of Heparin is Protamine Sulphate
• Heparin monitoring is done by aPTT
• Heparin is given Intravenous (I/V)
WHILE:
• Warfarin acts on Vitamin K
• Inhibiting the synthesis of Vitamin K dependent clotting factors (II, VII, IX, X)
• Warfarin anti-dote is Vitamin K
• Immediate effect is reversed by FFP‘s
• Warfarin monitoring is done by INR>PT
• Warfarin in given Orally
217. Heart Blood Supply:
• Anterior wall of Heart supplied by Left Anterior Descending
• Anterior surface of Heart supplied by Right Coronary Artery
• Conduction system of heart supplied by Right Coronary Artery (except
RBB supplied by LCA)
• Anterior inter ventricular groove contains  Great Cardiac Vein and
Anterior inter-ventricular artery (LAD)
• Posterior inter-ventricular groove contains Middle Cardiac Vein +
Posterior inter-ventricular Artery (RCA)
• Epicardium supplied by Coronary Artery
• Pericardium supplied by Pericardiophrenic Arteries + Phrenic Nerve
218. Arterial Supply and ECG Leads:
• Inferior wall MI (Artery is RCA) Leads II, III, aVF
• Anterior wall MI (Artery is LAD) Leads V1-V6
• Lateral wall MI (Artery is LCX ) Leads 1, aVL , V5 and V6 )
219. Heart Sounds:
• First Heart Sound Isovolumetric contraction Phase (Closure of Mitral and Tricuspid Valve) +
Felt at Mitral Area
• Second Heart Sound Isovolumetric relaxation Phase (closure of Aortic and Pulmonary Valve ) +
Felt at Left Upper Sternal Border
• Third Heart Sound Rapid Ventricular Filling + Best at Apex + Normal in children , Pregnancy
SK Origional – Golden 11 419

and Athletes
• Fourth Heart SoundHeard in Atrial Systole, Non- compliant Left Ventricle + Best at Apex
+ Due to Increased Atrial Pressure.
220. Regarding heat loss:
• Heat Loss Depends Upon Core Body Temperature (if not present choose Temperature of
External Environment)
• Set Point Increased than Hypothalamus Shivering Happens and Sweating Decreases (Vice
Versa if Set Point Decreased)
• If a person is Lying Naked Heat Loss By Radiation and Conduction
• Marathon Runner Having excessive Sweating Heat Exhaustion>Dehydration
• If a Person has no Sweating and then Collapsed Heat Stroke
• Farmer in a sunny day collapsed Hypernatremia due to Excessive Sweating.
221. Regarding myasthenia gravis:
• Myasthenia Gravis Antibodies against Pre synaptic Voltage Gated Channels
• Diagnostic Test Ach Receptor Antibodies
• Most accurate/Confirmatory/Gold Standard EMG
• Initial drug for Myasthenia Gravis Neostigmine
• For Maintained Therapy Pyridostigmine
• Physostigmine Crosses BBB Immediately and Increases Acetylcholine
• In Eaton Lambert Syndrome Antibodies are directed against Calcium Channels.
222. Regarding skin malignancies:
• Most common Skin Malignancy Basal Cell Carcinoma (Nodular Variant )
• Most common site of Basal Cell CA Upper Lip
• Most common after Basal Cell CA Squamous Cell
• Most common site of SCC Lower Lip
• Most Aggressive Melanoma
• Locally Malignant Ameloblastoma
• Pre-malignant lesion on cheeks which must be Excised Actinic Keratosis
• Condition having Highest Malignant Potential Basal Cell CA
• Blue Cell Tumor in Children+ releasing Catecholamine + Gene Amplification
Neuroblastoma.
223. Regarding Exercise:
• During Exercise Blood Flow Increased to Exercising Skeletal Muscle
• During Exercise Blood Flow Decreased to Splanchnic Vessels> Kidney
• During Exercise Blood Flow to Exercising Muscles is maintained by Local
Metabolites
• During Exercise Blood Flow to Non-Exercising Muscle is maintained by Sympathetic
Adrenergic
• During Exercise there is an increase in Ventricular Contractility
• During Exercise Subjective Feelings of Getting Tired is due to Increased Heart Rate
• After Exercise Feeling of Getting Tired is due to Increased Lactic Acid
224. Regarding body fluids:
• Normal Individual Body Water  In 70 kg is 42 Liters
• ICF 2/3rd is 28 Liters
• ECF 1/3rd is 14 Liters
• ECF is divided into Two Parts Plasma which is 1/4th of
ECF 3.5 Liters
• Interstitial Fluid which is 3/4th of ECF 10.5 Liters
225. Regarding ECG:
• Most Prominent Feature of Hypokalemia U wave + Inverted T wave
• In Hyperkalemia Tall T wave
• Normal ECG is unable to record Electrical Activity of SA Node
SK Origional – Golden 11 420

• ECG Changes can be seen in the following conditions Except Sleep


• Saw Tooth Appearance on ECG Atrial Flutter
• P wave Absent + Irregular RR inter al in Atrial Fibrillation
• QRS Complex Shows Ventricle Depolarization
• ST Segment Shows  Complete Ventricular Depolarization
• Isoelectric Line PR Segment
• Isoelectric Period ST Segment
• TP Segment Relaxation of Ventricles
• Hypoglycemia Increases QT Interval
• Most Specific Finding of Pericarditis PR Depression
• Time Taken by the Impulse to Travel from Endocardium to Epicardium QRS Complex
• Absent P wave Pacemaker is in AV Node
• PR inter al on ECG coincides with – A wave of JVP
• Tall QRS Complex Hypertrophy of Ventricles
• Low Voltage QRS complex Obesity, Old MI, COPD/Emphysema, Pericardial Effusion,
Constrictive Pericarditis
• Cannon Wave is seen in Complete Heart Block (3rd Degree)
• P-Mitrale occurs in Mitral Stenosis
226. Derivatives:
• Extra embryonic mesoderm derived from Epiblast
• Intra embryonic coelom derived from Lateral Plate Mesoderm
• Adrenal cortex derived from Mesoderm
• Adrenal medulla derived from Neural Crest Cell
• Dura matter derived from Mesoderm
• Pia and arachnoid matter derived from Neural Crest Cell
• Schwann cell derived from Neural Crest Cell
• Oligodendrocytes derived from Neural Tube
• Renal agenesis result from Failure of ureteric bud to reach metanephric mesoderm (Langman)
• Two kidneys with two ureter and pelvis result from Early division of ureteric bud (Langman)
• Hepatocyte Derived from Endoderm
• Kupffer Cell Derived from Mesoderm
• Transitional epithelium bladder derived from Endoderm
• Extra embryonic coelom derived from Hypoblast
227. Regarding ulcers:
• Curling Ulcer by Inhalation burn
• Cushing Ulcer by Brain injury (Raised ICP)
• Marjolin Ulcer by External Burn(Squamous cell carcinoma of Skin)
• Martorell Ulcer by Hypertension
• Most Common location of duodenal ulcer 1st part of Duodenum
• Perforation of posterior wall of duodenum bleed by Gastro duodenal artery
• Most common location of gastric ulcer near Incisura angularis on Lesser curvature
• Perforation of lesser curvature bleed by Left Gastric Artery
• Perforation of posterior wall of stomach bleed by Splenic Artery
228. Regarding micturition & defecation:
• Micturition center (stimulatory ) Pons
• Initiation of Micturition reflex  Stretch receptors in Bladder Wall
• Micturition center (Inhibitory ) Midbrain
• Processing of micturition Cerebral cortex
• Micturition reflex / integration Sacral segments of spinal cord
• Voluntarily Inhibition Activation of Pudendal Nerve
• Pain and filling sensations of bladder are carried by Sympathetic(Guyton)
• Defecation is initiated by Mass Movement
SK Origional – Golden 11 421

• Defecation is Carried out by Sacral Parasympathetic


• Defecation Reflex Recto Anal
• Defecation Reflex in Baby Gastrocolic
• Bulbar Urethra Rupture(Below Urogenital Diaphragm) urine into Superficial Perinial Pouch
• Membranous Urethra Rupture(At Urogenital Diaphragm) urine into Deep Perinial Pouch
• Prostatic Urethra Rupture(Above Urogenital Diaphragm) urine into Retropubic Space
• Penile Urethra Rupture Urine into Scrotum> Anterior Abdominal Wall
• Injury above Sacral Segment Cause Spastic Bladder, Automatic Bladder, Urge Incontinence
• Injury at Sacral Segment Cause Atonic Bladder, Autonomous Bladder, Overflow Incontinence
Note: Neurogenic is common term for both spastic and atonic bladder.
229. Regarding HIV/AIDS:
• Initial Test ELISA
• Confirmatory Test Western Blot
• In children Confirmatory PCR
• Hall mark of HIV Proliferation of Virus in T Cell
• All mark of AIDS Progressive Immunodeficiency(Decrease CD4 Count)
• Most common opportunistic infection in HIV is TB
• Most common opportunistic infection in- AIDS Pneumocystis Jirovecii
• Follow up is done by CD4 Count
• Progression from asymptomatic to Symptomatic stage can be assessed by PCR
230. Regarding GIT:
• Pancreatic Secretion Increased by CCK(First Aid Edition 2019)
• HCO3 secretion Increased by Secretin
• CCK increase Calcium by IP3 mechanism
• Gastric Motility Increased by Gastrin
• Gastric Motility Decreased by Somatostatin
• Increase Salivary Flow Cranial Nerve ( Facial>Glossopharyngeal)
• Increase Small intestine Motility and Decrease Gastric motility CCK
• Fundus removed Decrease Gastric compliance>> Decrease receptive relaxation
• Antrum removed Decrease Gastrin
• Pylorus removed Solids pass easily
• Intrinsic factor released by Fundus of Stomach
• Colostomy Result in Secretory Diarrhea
• Jejunostomy Result in Osmotic Diarrhea
• ileostomy Result in Osmotic Diarrhea
• Jejunostomy + ileostomy Result in Secretory+ Osmotic Diarrhea
• Iron and Calcium absorption Duodenum
• Max water + electrolytes absorption Jejunum
• Active (Aldosterone Dependent) water absorptionColon
• Passive (Aldosterone independent) water absorption Jejunum
• Absorption of LONG chain fatty acids Jejunum
• Vitamin B12 and Bile Salts Absorption Terminal ileum
• For B12 Absorption Needed Intrinsic Factor
• For B12 Transport Needed – Transcobalamin 2
• Absorption of SHORT chain FA -COLON
• Max Fluid loss Colon
• Part of gut removed that cause Fluid loss ileum
• Ilium resection Increase water content of feces>Decrease Bile salt absorption
Explanation: Though Max absorption occur in Jejunum but Fluid loss will always from Colon and
part of Gut remove which cause fluid loss is ileum as when Jejunum is removed no Fluid loss will
occur because ileum will Start absorbing Fluid but when ileum removed then Colon will be over
loaded so ileum Removal will cause fluid loss through Colon (Reference: Bailey and Love)
SK Origional – Golden 11 422

• Carcinoid syndrome Most common site is small intestine (ileum)>lungs, Trachea & Bronchi
• Most common tumor of appendix Carcinoid Syndrome
• Most common site of Carcinoid syndrome overall GIT>Respiratory system
• Sympathetic is always Adrenergic
• Sympathetic is Cholinergic only in Sweat glands
• Sympathetic is for Fight and Flight (Pupillary dilation+Bronchodilation +decreased GIT Motility)
• Parasympathetic effect on GIT Relaxation of Fundus, Contraction of body of stomach +
Relaxation of sphincter+Increasead GIT motility
• Posterior Duodenal Wall and Pancreatic Rupture Fluid Leaks into Lesser Sac
• Anterior Duodenal Wall Rupture Fluid Leaks into Right Posterior Sub phrenic Space
• Right Paracolic Gutter Greater Sac and Right iliac Fossa
231. Regarding thyroid:
• Maximum concentration of thyroid hormone T4
• Thyroid hormone Enter in Circulation T4
• Thyroid hormone cause fetal Brain development T3
• Active form of thyroid hormone T3 (Free form or Unbound is Active)
• Thyroid gland is enclosed by Pretracheal fascia
• Metabolism of TSH in Liver Demethylation
• Metabolism of Thyroid hormones In Liver De-iodination
• Dyspnea on lying down Retrosternal Goiter
• Superior Thyroid artery Related to External Laryngeal Nerve
• Inferior Thyroid artery Related to Recurrent Laryngeal Nerve
• Superior Laryngeal Artery Related to Internal Laryngeal Nerve
• Most Common Nerve injured during Thyroidectomy External Laryngeal Nerve
• Most Common Nerve injured during Tracheostomy Recurrent Laryngeal Nerve
• Most Common Cause of Bleed During Thyroidectomy Inferior Thyroid Vein
• Most Common Cause of Heavy Bleed During Thyroidectomy Anterior Jugular vein>Isthmus
• During Tonsillectomy Bleed due to Tonsilar artery, Palatine vein and Ascending Pharyngeal Artery
232. Regarding buffer system:
• Major blood/Extracellular/Interstitium/Plasma Buffer  Bicarbonate
• Major RBCs bufferHemoglobin
• Major Intracellular Buffer Proteins
• Major Renal/Renal Tubular Buffer Phosphate
• Major Urinary Buffer(Quantitatively) Ammonium
• Major Bone Buffer Calcium Carbonate
233. Regarding malignancy:
Microscopic feature of Malignant tumor:
• Metastasis >Invasion of adjacent tissues >Pleomorphism >Increased N/C ratio
• Histologically Invasion Diagnostic For Malignancy
• Grading Nuclear differentiation /Degree of differentiation /Mitosis
• Staging Extent of Spread /Lymph Nodes Involvement
• Pre-malignant lesion Pleomorphism Diagnostic
• Pre-malignant condition Increased N/C ratio Diagnostic
• Most Common Nevus In Children Junctional
• Most Common Nevus In Adult is Intradermal
• Highest Malignant Potential Dysplastic Nevus
• Male age Less than 40 Seminoma
• Male age more than 50 + increased LDH Lymphoma
• Tumor containing mature cells Teratoma
• AFP raised +Schiller Duval Bodies Yolk Sac Tumor
• Call Exner Bodies Granulosa Cell Tumor
• Giant cell tumor (Soap bubble appearance ) Epiphysis
SK Origional – Golden 11 423

• Osteosarcoma (Codman triangle sunburst appearance + osteochondroma(most common benign)


Metaphysis
• Ewing sarcoma (onion skin ) Diaphysis
• Osteoblastoma occurs in Vertebrae
• Osteoid osteoma Cortex of long bones, has radiolucent osteoid core
• Radio sensitive Phase G2-M Phase(M>G2)
• Radio Resistant Phase S Phase
• Chemo Sensitive S Phase
• Angiosarcoma in Plastic Factory Worker Vinyl chloride
• Angiosarcoma in Farmer Arsenic
• Plastic Factory Worker Liver Angiosarcoma
• Liver CA Alcohol >Aflatoxin >Smoking
• Acute Effect of Radiotherapy Desquamation
• Chronic Effect of Radiotherapy Endarteritis Obliterans
• Late Effect of Radiotherapy Lymphoproliferative Disorder
• Common method to detect Tumor Tumor Marker
• Common method to detect Tumor cells Peripheral Smear
Regarding sensitivity to radiotherapy:
• Lymph Node Tumor>Seminoma>Glioma>Craniopharyngioma
• Least Radio Sensitive Blood Cell Platelets
• Most Radiosensitive Organ Skin
• Least Radiosensitive Organ Bone
• Most Radio Sensitive Mucosa Intestinal
• Radiation induced Brain malignancy Meningioma
• Overall radiation induced CA Leukemia
• Chemotherapy caused cell death Apoptosis
• Chemotherapy induced vomiting is treated by Ondansetron
234. Regarding energy:
• Carbohydrates/ Protein gives 4.1 kcal
• Fats gives 9.1 kcal
• Alcohol gives 7 kcal
• Carbs Required 50-60%
• Fats Required 25-35%
• Proteins Required 10-20%
• Major source of energy Adipose Tissue
• Max glycogen Skeletal Muscles
• Maximum glycogen concentration/ per 100 grams Liver
• Highest energy compound ATP
• Highest energy molecule ATP
• Highest energy content  Starch
• Highest quantity of unsaturated fatty acid Sun Flower
• Highest quality of unsaturated fatty acid Soya Bean
• In Fasting upto 48 hours Glucose from 48 hours till 72 hours Fatty Acids/Triglycerides
After 72 hours Proteins/Amino Acids.
235. Regarding Hepatitis:
• Oro-fecal Route Hepatitis A>Hepatitis E
• Most lethal Hep Hepatitis D
• Most common in pregnancy Hepatitis A
• Most lethal/Remote Area in pregnancy Hepatitis E
• Most lethal/common virus after blood transfusion/transplantation CMV>Hep B>Hep C
• Most common sexual spread Hepatitis B
• HCC Hepatitis B>Hepatitis C
SK Origional – Golden 11 424

• Cirrhosis Hepatitis C>Hepatitis B


• Needle prick injury risk Hepatitis B (30%)>C (3%)>HIV (0.3%)
• Hepatitis A and E Naked Viruses
236. Regarding pelvis:
• Finger couldn't reach sacral promontory Android Pelvis
• If Finger reaches Sacral promontory Contracted Pelvis
• Pelvis in males Android (heart shaped )
• Pelvis is females Gynecoid (rounded shape)
• Most favorable pelvis for delivery Gynecoid Pelvis
• Reference point for head during labour Ischial Spine
• Bony landmark for Pudendal nerve block Ischial Spine
237. Regarding cranial nerve palsy:
• In CN3 Palsy Pupils dilation occurs + Ptosis
• In Hornor syndrome Damage to cervical Sympathetic chain + Pupil constriction + Ptosis and
Anhydrosis
• In CN 4 palsy Vertical Diplopia (Superior Oblique)
• In CN 6 palsy Horizontal Diplopia (Lateral Rectus)
• In CN3 palsy Both Vertical and Horizontal Diplopia
238. Regarding lesions:
• Dopamine loss in substantia nigra and striatum Parkinson Disease
• GABA loss in substantia nigra and globus Pallidus Huntington Disease
• GABA loss in Caudate Nucleus Chorea
• GABA loss in Globus Pallidus Athetosis
• GABA loss in Subthalamic Nuclei Hemiballismus
239. General Pathology:
• Tissue Macrophages
• Blood Neutrophils
• Surface Skin
• Major scavengers Macrophages
• Phagocytosis done by Neutrophils
• Opsonisation C3b
• Phagocytosis by C5a
• Most potent Chemo tactic Factor LTB4>C5a
• Pain Bradykinin>Prostaglandins E2
• IL-1 and TNF Fever Mediator
• Prostaglandins F2, E2 and D2 causes Vasodilation
• Late mediator of Inflammation PG & Leukotriene
• Initial mediator of inflammation Histamine
Reference: First Aid Edition 2019
240. Hematology:
• Hemoglobin synthesis starts in Proerythroblast or Early Normoblast
• Hemoglobin First appears in Intermediate Normoblast
• RBC Nuclei disappear in Late Normoblast >Reticulocytes
• Max Production Late Normoblast
• Maximum concentration Reticulocytes
• Iron Transport form Transferrin
• Iron Excess storage form Hemosiderin
• Iron Normal Storage Form Ferritin
• Iron Fe2+ in reduced form binds to Hemoglobin
• Heme binds with Hemopexin
• Hemoglobin binds with Heptoglobin
• Iron is absorbed from Duodenum
SK Origional – Golden 11 425

• Diagnostic for IDA Serum Ferritin


• Diagnostic For B12 Deficiency B12 Assay
• Diagnostic For Folate Deficiency RBC Folate Level
• Diagnostic For Pernicious Anemia Anti Intrinsic Factor Antibodies
• Diagnostic feature of Aplastic anemia Fatty marrow>Pancytopenia
• Acute Intravascular Hemolysis Decrease Heptoglobin>Reticulocytosis
• Chronic Intravascular Hemolysis Hemosiderinuria>Hemoglobinuria
241. Deep vein thrombosis and embolsim:
• Most common source of emboli Femoral Vein
• Most common site of DVT Popliteal Vein
• Most common cause Immobilization
• D dimers Sensitive for DVT
• FDP Specific for DVT
242. Neural tube defects:
• AFP raised in Anencephaly
• AFP Decreased in Down syndrome
• Specific For Neural Tube Defects Acetyl cholinesterase
• Sensitive For Neural Tube Defects AFP
• NTD in early Pregnancy Diagnose USG
• NTD in Late Pregnancy Diagnose Amniocentesis
• Neural tube defects-- most common is Meningo-myelocele
• Neural tube defects occur due to Folic Acid Deficiency and Vitamin A Toxicity
243. End Arteries:
• Functional end Arteries Heart
• Anatomical true End ArteriesRetinal Artery
• End arteries are present in Central Retinal Artery>Spleen>Heart
244. Regarding neonates & children:
• C-shaped Vertebral column
• Has Circular abdominal cavity
• Liver has 5% of body weight
• Sample taken from Radial Artery>Posterior Tibial Artery>Dorsalis Pedis Artery
• Blood Volume is 85 ml/kg
• Hematocrit Level is 55 %
• Lowest Testosterone level in Pre-school children
• Water percentage in neonate 70-75 %
• Heart Rate of infant is 110-150 beats per minute
• Hemoglobin lowest at 3rd Month of age
245. Regarding Pregnancy Changes and Hormones:
Remains Same:
• Respiratory Rate
• Vital Capacity
Increases:
• Tidal volume
• Minute ventilation
• Inspiratory Capacity
Decreases:
• Total Lung Capacity
• Residual Volume
Other Changes:
• Increase GFR – Decrease BUN and
• Increase Cardiac output
• Dilutional Anemia
SK Origional – Golden 11 426

• Hyper coagualability
• Increase Lipolysis
• HCG maintains Pregnancy upto 8-10 Weeks
• After 10 weeks by Estriol and Progesterone of Placenta
• HCG In blood Upto10 days
• HCG In Urine After 10 days
• Labour Initiated by Fetal Cortisol>Fetal ACTH(Pituitary)
246. Regarding prostate:
• Median Structurally Largest Lobe
• Lateral Anatomically Largest Lobe
• Peripheral Largest Zone
• CA Prostate Peripheral Zone/Posterior Lobe+ Metastasize to Vertebral Column and Brain by
Anterior Intervertebral Venous Plexus
• BPH Median Lobe/Transitional Zone
247. Regarding cardiovascular system:
• Right ventricle pressure during Systole 25 mmHg
• R ventricle pressure during Diastole 0-8 mmHg
• Right ventricle pressure at which Pulmonary Valve open 8mmhg
• Left ventricle pressure at which Aortic Valve open 80mmhg
• Maximum Aortic Pressure Reduced Ejection Phase
• Minimum Aortic Pressure Isovolumetric Contraction Phase
• Maximum Ventricular Pressure Rapid Ejection Phase
• Minimum Ventricular Pressure Rapid Filling Phase
• Maximum Ventricle Filled Atrial Systole
• Maximum Ventricle Filling Rapid Filling Phase
• Maximum Pulse Pressure Dorsalis Pedis>Femoral Artery>Popliteal Artery > Aorta
• Highest Systolic Pressure Renal Artery
• Max O2 Pulmonary Capillaries
• Low O2 SVC > Pulmonary Artery>Umbilical artery
• Highest O2 Saturation Umbilical Vein
• Cardiac output unchanged in Sleep
• Pace Maker activity of SA Node due to Sodium Channels
• Action Potential of Pace Maker due to Calcium Channels
• Action Potential of Cardiac Muscle due to Sodium Channels
• Becks Triad (Cardiac tamponade) Muffled Heart sound, Hypotension and Raised JVP
• Rustling Sound /Pericardial RubPericarditis
• Pain unrelated to respirationMyocarditis
• SA node located inUpper one third Sulcus Terminalis
• SA node located in Sub Epicardium
• AV node located in Endocardium
• Conducting System Sub Endocardium
Heart Rate and Location of Pacemaker:
• SA Node--60-80
• AV Node--40-60
• Purkinje fibers --20-40
• SA Node Slowest pre-potential
• SA Node works as syncytium as it is able to generate impulses at a faster rate
• Purkinje Fibers have highest speed of conduction due to Wide Diameter>Large number of Gap
Junctions>Large number of Sodium Channels>Less number of myofibril>Short Refractory Period
Speed of Conduction:
Purkinje fibers>Bundle of His>Atria>Ventricles>SA Node>AV Node
Speed of Rate:
SK Origional – Golden 11 427

SA Node>AV Node>Purkinji Fibers


Heart Sounds and Cardiac Markers
• First Heart Sound Isovolumetric Contraction Phase
• Second Heart Sound Isovolumetric Relaxation Phase
• Third Heart Sound Rapid Ventricular Filling Phase
• Fourth heart sound Atrial Systole
• Best Initial ECG
• 1-2 hour Myoglobin
• Within 4 hour CK MB
• After 4 hour Troponins
• Sensitive Trop T
• Specific Trop I
• 4h Post MI Arrhythmia
• 4-24h Post MI Arrhythmia
• 1-3 Days Post MI Fibrinous Pericarditis
• 4-7 Days Post MI Cardiac Tamponade
• Month Post MI Aneurysm
• Preload determine by End Diastolic Volume (Depends on Venous return)
• After load determine by Mean Atrial Pressure
• TPR determine by Diastolic Blood Pressure
248. Regarding ECG Leads and JVP Pattern:
• Hypokalemia U wave + inverted T wave
• Hyperkalemia  Tall T wave
• Normal ECG is unable to record  Electrical activity of SA Node
• ECG changes can be seen in the following conditions Except Sleep
• Saw Tooth appearance on ECG  Atrial Flutter
• P wave absent + Irregular RR Interval in Atrial Fibrillation
• QRS complex shows Ventricle Depolarization
• ST Segment shows Complete Ventricular Depolarization / Plateau pPase
• Isoelectric Line PR Segment
• Isoelectric Period ST Segment
• TP segment Relaxation of Ventricles
• Hypoglycemia increase QT Interval
• Most specific finding of Pericarditis PR Depression
• Time taken by the impulse to travel from Endocardium to Epicardium QRS Complex
• Time taken by the impulse to travel from Epicardium to Endocardium QT Interval
• Absent P wave Pacemaker in AV Node(also right ventricle-moderator band)
• PR Segment on ECG coincides with A wave of JVP
• Large A wave Tricuspid Stenosis
• Cannon A wave 3rd Degree Heart Block & AV Dissociation
• Tall QRS complex Hypertrophy of Ventricles
• Low voltage QRS complex Obesity, Old MI, COPD & Constrictive Pericarditis
249. Regarding endocrinology:
• Immediate action of Insulin  Entry of Potassium into Cells
• Intermediate Action of Insulin  Protein Synthesis
• Late Action of InsulinLipogenesis
• Action of insulin at Cellular level is Entry of glucose into cells
• Insulin Independent Glucose uptake Exercising Skeletal Muscle>Brain(Guyton)
• Anti Ketotic Insulin
• Ketogenic  Glucagon
• Hypoglycemia increases  Glucagon>Gastrin>Secretin (Reference: Ganong)
• GH is increased In Hypoglycemia > Exercise > NREM (Reference: Guyton+Ganong)
SK Origional – Golden 11 428

• Potent stimulant of Aldosterone Hyperkalemia & Hyponatremia


• Potent stimulant of Renin Sympathetic>Hyponatremia>Hypokalemia
• Potent Stimulant For ADH Nausea>Increase Plasma Osmolarity
• Renin is Inhibited by Increased Angiotensin 2
• Thirst center is stimulated by Angiotensin 2
• Renin Increase by Hypokalemia and Decrease by Hyperkalemia (Reference: Ganong)
• Angiotensin 2 cause -- Vasoconstriction > Thirst Stimulation(Reference: Ganong)
• Cortisol main function Gluconeogenesis
• Glucagon main function Glycogenolysis
• Cortisol Increase Neutrophil and Decrease Lymphocytes
• ACTH effectively control Cortisol(Hydrocortisone)>Androgens (Reference: Guyton)
• Excessive Exercise causes Hyperkalemia
• Early Pregnancy Endometrium Sensitive to Progesterone
• Late Pregnancy Endometrium Sensitive to Oxytocin
• In Pregnancy Lactation inhibited by Estrogen+ Progesterone>Estrogen>Progesterone
• During Lactation Amenorrhea due to Decrease GnRH
• Milk Production-- Prolactin
• Milk Ejaculation Oxytocin
• In Pre-menopausal Breast Atrophy Decrease Estrogen +Progesterone
• In Post-menopausal Breast Atrophy Decrease Estrogen
• Most Common Cause of Breast Atrophy Decreased Estrogen
• Most Potent Anabolic Testosterone
• Potency DHT>Testosterone
• Potency Estradiol>Estrone >Estriol
• Increase Estriol Indicates Fetal Well Being
• Osteoporosis Thin and wide trabeculae + long term use of Steroid
• Osteoclasts in Howhship lacunae
• Osteoblast Bone making cells + Produce ALP+ Laid Bone Matrix
• Osteoclast Bone Resorption
• Osteocytes Maintain Bone Matrix & Integrity
• Demineralization of bone done byPTH
• Bone RemineralizationVitamin D
• Bone Remodeling Vitamin C
• PTH directly Regulates Vitamin D levels
• PTH Indirectly Regulates Calcium Absorption from intestine By Vitamin D
• Osteon Has Concentric lamellae
• Patient has cast applied now has Decreased muscle mass Disuse Atrophy or Decreased number of
Actin and Myosin
• Estrogen Function Breast Duct Development + Endometrial Proliferation
• Progesterone Function Maintain Endometrial Thickness(Proliferation) > Secretory phase >
Alveoli and Lobule development
250. Regarding respiratory:
Central Chemoreceptor responds to:
• CSF pH or Interstitial pH (Increase H ions)
• Increase CO2 In Arterial Blood
Peripheral Chemoreceptor (Carotid and Aortic body) responds to:
• Decrease O2
• Arterial pH (H ions)
Increase A-a Gradient seen in:
• Fibrosis
• V/Q Defect
• R-L Shunt
SK Origional – Golden 11 429

• A-a Gradient Normal Value 0 -10mmhg


• A/a Ratio Normal 0.8 (>0.75)
In Airway Obstruction:
• V/Q is Less than Norma called R-L Shunt (Reference: Guyton)
• Composition of Systemic arterial blood approaches Mixed Venous Blood
• V/Q is Infinite Called Dead Space
• Composition of Alveolar air approaches Inspired air
At High Altitude:
• Survival due to Increase Hb Concentration
• Pulmonary Vasoconstriction (due to Hypoxia)
• HB-O2 Curve Shift to Right
• Increase 2,3 DPG Concentration
Exercise Increase:
• O2 Consumption
• CO2 Production
Small Cell Lung CA associated with:
• ACTH(Cushing disease)
• SIADH
• Eaton Lambert Syndrome
• Poor Prognosis
Squamous Cell CA associated with:
• Smoking
• Hypercalcemia(PTHrP)
• Chloride Shift occur by Band 3
• Venous Blood have more HCO3 & PCV(PCV >HCO3)
• RBC Venous Blood Have more Chloride Ions
• Erythropoietin Produced By Hypoxia
• Erythropoietin Inhibited By Theophylline
• Right Shift of O2 Curve Bohr Effect
• Left Shift of O2 Curve Haldane Effect
• O2 100 Percent in  L-R Shunt
• Pulmonary Blood flow and Ventilation Highest at Base
• Pulmonary Blood flow and Ventilation Lowest at Apex
• V/Q and Arterial PO2 Highest at Apex
• V/Q and Arterial PO2 Lowest at Base
• Pulmonary Vasoconstriction Caused by Hypoxia
• Pulmonary Vasoconstriction Accentuated by Increase CO2
• Asthma Most Imp Diagnosed By FEV1/FVC
• In Fibrosis FEV1/FVC Remain  Normal
• In Asthma and COPD FEV1/FVC Decreased
• Lung CA Smoking >Radon >Asbestos
• Asbestos Lung CA >Mesothelioma
(Ref: First Aid Edition 2019)
• Primary TB Ghon Complex + Lower Lobe
• Secondary TB Cavitation + Upper Lobe
• Activated in Lung Angiotensin 2
• Inactivated in Lung Bradykinin( By ACE) & Serotonin
• Sarcoidiosis Characteristic Erythema Nodosum (Reference: Davidson)
• Sarcoidiosis Diagnosed Histologically by Non Casseating Granuloma (Reference: Robins)
• Hydrocortisone different from Dexamethasone Because they Retain Na
251. Regarding microbiology:
• Ascetic Tap E-coli
SK Origional – Golden 11 430

• Peritonitis E-coli
• Pyogenic Peritonitis Bacterioides
• Puerperal Sepsis Group B streptococcus > E coli > Bacterioides
• Nasopharyngeal CA EBV
• Oropharyngeal CA HPV
• Lung CA CMV
• Respiratory Symptoms Ascaris
• Fisherman with anemia +Ecchymosis Vitamin C deficiency
• Fisherman with anemia, Vitamin B12 DeficiencyDiphylobothrium
• Microcytic Anemia Ancylostoma
• Conjunctival Swelling Loa Loa
• Global Blindness  Cataract > Chlamydia > Glaucoma
• Rectal Prolapse Trichuris Trichura
• Muscle(Myalgia) Trichinella Spiralis
• Portal HTN Schistosoma Mansoni + Japonicum(Lateral Spine)
• Pulmonary HTN Schistosoma Haematobium(Terminal Spine)
• Cholangiocarcinoma Clonorchis Sinesis
• Hydatid Cyst Echinococus
• Neurocysticercosis (Brain Cyst) Taenia Solium
• Sexually Transmitted Diseases by Chlamydia > Gonorrhea > Syphilis
• Tubo-ovarian mass Gonorrhea
• Honeymoon Cystitis E coli
252. Regarding Cancer Markers:
• Cytokeratin Carcinoma (Epithelial)
• Vimentin Sarcoma (Mesenchymal)
• Desmin Muscle
253. Regarding Malaria:
• Plasmodium malariae can lead to Nephrotic syndrome (Membranous GN)
• Plasmodium Vivax and ovale has Hypnozoite stage
• Primaquine is DOC to kill Hypnozoite
• Shortest pre-erythrocytic phase is seen in Plasmodium Falciparum
• Longest proerythrocytic phase is in Plasmodium Malariae
• Species that cause relapse are  Vivax>Ovale
• Most common non-falciparum malaria is  Plasmodium Vivax
• DOC for non-falciparum malaria is Chloroquine
• Malignant tertian malaria is caused by  Plasmodium Falciparum
• Benign tertian malaria (48 hours) is caused by  Ovale/Vivax
• Quartan malaria (72 hours ) caused by  Plasmodium Malariae
• Quotidian malaria caused by  Plasmodium Knowlesi > Falciparum
• Anemia in malaria is  Normocytic Normochromic Anemia
• Dormant phase of malaria Hypnozoite
• Malaria enter into human body as  Sporozoites
• Sporozoites divide in liver as  Merozoites
• New species of malaria is Knowlesi
254. Regarding renal pathology physiology:
• Dilute urine (decrease osmolarity) Early DCT (Macula Densa )>Thick Limb of LOH
• Dilute urine (decrease osmolarity) Thick Limb of LOH>DCT (Note: Difference between Early
DCT and Only DCT)
• In dehydration (ADH) concentrated urine(Increase Osmolarity) Collecting Ducts (Vasa Recta)
• Erythropoietin secreted by Peritubular capillaries > Meningeal Cells (Reference: Ganong Mcq)
• Kidney Podocytes  At visceral layer of bowman capsule
• Crescents  Formed by parietal cells lining bowman capsule
SK Origional – Golden 11 431

• GFR measure Clinically / Best Estimated  Creatinine clearance


• GFR best Way to Measure  Inulin
• Best way to Measure RPF  PAH
• Best test for renal failure  Creatinine clearance
• Clearance  PAH >K>Inulin>Urea>Sodium>Amino Acids and Glucose
• Concentrating urine in summer or during fasting ADH
• Loop diuretics act on  Thick Ascending Limb
• Thiazide diuretics act on  Early DCT
• Osmotic diuretics act on  PCT
• Renal columns contains Interlobar Artery
• Capsule contain Interlobular Artery
• Glomerulus contain Interlobular Artery
• Hilum contain Segmental Artery
• Medullary rays contain  Collecting ducts
• Prone to Ischemia PCT
• Maximum water & sodium absorption  PCT
• Maximum water & sodium absorption with any hormone  PCT
• Maximum potassium absorption  PCT
• Potassium loss due to dietary irregularities  Distal tubules
• ADH Increase Urea transport to DCT
• ADH  Inhibited by alcohol
• ADH  Regulates plasma volume / urine osmolarity
• ADH  V1 receptors cause vasoconstriction
• ADH  V2 receptors act on kidney
• Highest Tubular transport maximum Glucose>PAH>Lactate
• Threshold for Glucose 200(Guyton)
• Sodium Absorption  Aldosterone
• Sodium excretion  ANP
• Net SODIUM Absorption  Aldosterone & ANP
• Renin  Produced by JG cells
• Renin  Long term B.P regulation
• Baroreceptor respond maximally to Increasing BP (Reference: Ganong)
• Most Rapid Response to Decrease BP Baroreceptor
• Most Potent Response to Decrease BP CNS Ischemic
• CNS Ischemic Activate at 60 or Below 60 mmHg (Reference: Guyton)
• Maintain During Shock Baroreceptor
• Maintain as a Whole RAAS
• Long Term BP Regulation RAAS
• Over all most important RAAS
• Sub Endothelial deposits  SLE, Diffuse proliferated GN ,Membrane proliferated GN
• Sub Epithelial deposits  PSGN
• Intramembranous Deposits  Diffuse Proliferated GN , Membrane Proliferated GN Type 2
• Focal segmental Glomerulonephritis  Massive Proteinuria
• Mild Hypomagnesaemia Stimulate Parathyroid  Hypercalcemia
• Severe Hypomagnesaemia suppress Parathyroid  Hypocalcaemia
• Hypermagnesemia Cause Hypocalcaemia
• Hypokalemia is associated with Metabolic Alkalosis
• Hyperkalemia associated with Metabolic Acidosis
• Hypokalemia Decrease Nerve Excitability In RMP
• Hyperkalemia Increase Nerve Excitability In RMP
• Hyponatremia Decrease Height of Action Potential
SK Origional – Golden 11 432

• Hypernatremia Increase Height of Action Potential


• Hypocalcaemia Increase Excitability
• Hypercalcemia Decrease Excitability
255. Regarding Biochemistry:
Vitamins:
• B1 Deficiency  Dry Beri Beri ,Wet Beri Beri, Wernicke Korsakoff Syndrome measured by
Transketolase activity
• B2 Deficiency Corneal Vascularization
• B3 Deficiency Pellagra( Diarrhea,, Dementia , Dermatitis )
• B5 Required Co factor for Co enzyme A
• B5 Deficiency Adrenal Insufficiency
• B7(Biotin) Binds Avidin in Eggs and carrier of One carbon
• B7 Role in Liver Metabolism
• B9(Folic acid) One Carbon Transfer
• B9 Deficiency Neural Tube Defects
• B12 Deficiency  Megaloblastic Anemia
• Vit A deficiency Early sign Night Blindness (Reference: Lippincott)
• Vit A Toxicity Jaundice>Scaly Dermatitis, Neural Tube Defects
• Optic Neuritis B12>B6
• Peripheral Neuritis  B1 >B6 >B12
• Carbohydrate Metabolism Vitamin B1
• Protein Metabolism Vitamin B2
• Lipid MetabolismVitamin B7
(Reference: First Aid Edition 2019)
Amino Acids
• Ketogenic Leucine and Lysine
• Positive charge HAL( Histidine ,, Arginine & Lysine)
• Negative Charge Aspartate and Glutamate
• DNA has Histidine
• Amino Acid deficiency causes cell injury  Glycine
• Amino acid cause injury to cell  Choline
• Amino Acid causing renal stones Lysine(COLA)
• Amino acid in abnormal metabolismTryptophan
(Reference: First Aid Edition 2019)
Cell Cycle:
• Interphase Chromosomes replicates
• Interphase  DNA replicates
• Interphase Barr Bodies are studied
• Interphase is divided into
1. G1 Phase
2. S Phase (SYNTHESIS Phase)
3. G2 Phase
4. Mitosis (also called M phase & cytokinesis is a part of it)
• G1 Primary Growth , Proteins , Organelles , mRNA Synthesis )
• G1 Also Called growth phase
• G1 Longest phase ( 8 to 10 Hours)
• G1 Check point To check if DNA is damaged
• S DNA Replicates
• S Cytotoxic & cancer drugs act here to destroy DNA
• S 5 to 6 hours
• G2 Secondary growth ( between S phase & Mitosis)
• G2 Short ( 3 to 4 hours)
SK Origional – Golden 11 433

• G2 Check point To check if DNA has replicated properly


• MITOSIS/M Phase
• M Shortest (2 hours)
• M To see spindle assembly and alignment
• MNuclear content divide
• M genetic material is Chromosome (Genetic material is chromatid when NOT in M phase)
• M Divided into Prophase, prometaphase, Metaphase, Anaphase , Telophase, Cytokinesis
• PROPHASE Spindle fibers appear Chromosome Condensation
• PROPHASE Centriole start moving to the opposite end & chromosomes first appear
• PROMETAPHASE Spindle fibers attach to chromosome & chromosome movement
• PROMETAPHASE Nuclear membrane dissolve marking the beginning of prometaphase
• METAPHASE Chromosome Alignment at Equatorial Plate
• METAPHASE Chromosome Thickest
• METAPHASE Chromosome begin to divide
• METAPHASE Karyotyping
• ANAPHASE Division of chromatids & sister chromatids move to opposite side
• ANAPHASE NON Disjunction
• TELOPHASE Spindle fibers disappear & Decondensation of Chromosomes
• TELOPHASE Complete division
• TELOPHASE Nuclear membrane formation
• CYTOKINESIS Cytoplasm divide
• Barr Bodies Heterochromatin
• Barr Bodies have X chromosome
• Barr Bodies seen under light microscope
• Barr Bodies ABSENT in Turner
• Barr Bodies Scanty In Turner
• Barr Bodies Diagnostic for Turner
• One Barr body in Klinefelter
• Number of Barr bodies in OX No Barr Bodies
• Number of Barr bodies in XX 1 Barr Body
• Number of Barr bodies in XXX 2 Barr bodies
• Best Test for chromosomal abnormalities is Karyotyping
• Cell to Cell Cadherin
• ECM TO ICM Intermediate Filament
• ECM to Cytoskeleton Integrin
• Leukocyte Adhesion to Endothelium ICAM(CD18 Subunit)
256. Regarding organelles:
• Detoxification of drugs SER
• Detoxification of Alcohol in Toxic Dose SER
• Detoxification of Alcohol in Normal Dose Peroxisomes( Oxidase and H2O2 )
• Lysosomes Contain Hydrolytic enzyme
• SER Originate from Peroxisome
• Lysosome Originate from Golgi Body
• Continue with Nuclear membrane RER
• Nissle Substance in RER
• Leydig Cell Has SER
• Mitochondria power house + self-replicating + Short chains Fatty acids Metabolism
• Double membrane bounded organelles Nucleus and Mitochondria
• Golgi bodies Packaging of cells
• Production of proteins Ribosomes > RER
• Centrioles make Basal body (basal body makes cilia and flagella)
257. Regarding pharmacology:
SK Origional – Golden 11 434

• DOC for Taenia Solium Praziquental > Niclosamide > Albendazole


• DOC for Clostridium Difficile Metronidazole > Vancomycin (Reference: Levinson)
• DOC for mild Clostridium Difficile Metronidazole (Reference: Levinson)
• DOC for Severe and resistant Difficile Vancomycin (Reference: Levinson)
• DOC for Traveller Diarrhea Norfloxacin>Metronidazole
• 1st Line in Atrial Fibrillation Beta Blockers (Reference: Davidson)
• For Rate control in Atrial Fibrillation Beta Blockers & Digoxin (Reference: Davidson)
• For Rhythm Control in Atrial Fibrillation with no IHD – Flecainide (Reference: Davidson)
• For Rhythm Control in Atrial Fibrillation with IHD – Amiodarone (Reference: Davidson)
• DOC For Cardiogenic shock Dopamine & Dobutamine
• DOC for Hypotension Unrecordable BP  Dopamine
• DOC for Anaphylactic shock Adrenaline(Epinephrine)
• DOC for Pseudomonas Ceftazidime
• DOC for UTI by Pseudomonas  Ciprofloxacin
• DOC for OCD Clomipramine
• DOC for SAH Nimodipine
• DOC for acute Pancreatitis Morphine>Pethidine
• DOC for Post-Surgery Analgesia in Asthma Patient Ketorolac Acid
• DOC for Post-Surgery Analgesia Ketorolac>Pethidine
• Drug in morning Sickness Pyridoxine
• Drug in motion Sickness Meclizine
• Drug in Mountain Sickness Acetazolamide
• DOC for Meningitis Adults Ceftriaxone > Cefotaxime
• DOC for Meningitis in Baby Penicillin G
• DOC in endometriosis Medroxy progesterone > Leuprolide > Danazol
(Reference: Ten Teacher)
• DOC for Pregnancy induced HTN Labetalol >Methyldopa
• DOC for Pregnancy Hypertensive Emergency Hydralazine > Labetalol
• DOC for Eclampsia MgSO4
• DOC in Uterine Atony  Oxytocin>Ergometrine (Reference: Ten Teacher)
• Anti-Thyroid in 1st Trimester PTU
• Anti-Thyroid in 2nd & 3rd Trimester Mathimazole
• Crosses Placenta and affect FetusMathimazole > PTU
• Don‘t Cross Placenta Thyroxin
• Elderly Diabetic Tolbutamide
• Obese Diabetic Metformin
• Non Obese Diabetic Sulfonylurea
• Organophosphate poisoning Antidote Pralidoxime
• Organophosphate poisoning Symptoms Reversal Atropine
• Drug in Liver Decompensation used Lorazepam & Oxazepam
• Drug in liver Disease Pre operatively Fentanyl
• Drug Contraindicated in Liver Disease Pentazocin > Paracetamol
• Hepatic Encephalopathy Progress by Diuretic >Paracetamol
• NM Blocker in Asthma Cisatracurium
• NM Blocker in Liver disease Atracurium
• Pancuronium Eliminated by Kidney (80 percent)
• Rocuronium Eliminated by Liver (75-90 percent) & Kidney
• Diazepam Act through Interneuron
• Morphine Release Histamine
• Terbutaline cause Fetal Hypoglycemia and Maternal Hyperglycemia
• Analgesic effect of TCA 1-2 weeks (Reference: Davidson)
SK Origional – Golden 11 435

• Antidepressant effect of TCA 3-4 weeks


• Safe Analgesic dose NO 25 ppm
• Safe Analgesic dose NO In 8 hours 25 ppm
• Safe Analgesic dose NO In 24 hours 100 ppm
• Morphine Decrease Apnea Threshold
• Local Anesthesia cross Placenta by Simple Diffusion
• Scrulfate doesn‘t Let Cimetidine to absorb
• Cimetidine Decrease Scrulfate Metabolism
• More Local Anesthetic in Blood in Intercostal Block
• Delayed Respiratory Depression Fentanyl
• Therapeutic Index Determine Drug Safety
• Potency Determines Dose
• Benzodiazepine Cause Hypotension in Hypervolemia>Old age
• Highly Selective COX 2 Celecoxib
• Highly Potent COX 2 Meloxicam
• Irreversible COX 1 & COX 2 inhibitor Aspirin
• Reversible COX 1 & COX 2 inhibitor NSAID
• Low dose Aspirin inhibit Thromboxane A2
• Cardio toxic Bupivacaine
• Priapism by Trazodon
• Gingival Hyperplasia by Phenytoin
• Pulmonary Fibrosis by Methotrexate
• Cardiomegaly by Adriamycin
• Kernicterus by Sulphonamide
• Reversible Oligospermia by Sulfasalazine
• Indirect Hyperbilirubinemia by Methyldopa
• Orange urine by Rifampicin
• Gout by Pyrazinamide
• Methmoglobenemia by Procaine
• Red Man Syndrome by Vancomycin
• Grey Man Syndrome by Amiodarone
• Gray Baby Syndrome by Chloramphenicol
• Aluminum hydroxide(No receptor in Body) cause Constipation
• Magnesium Hydroxide cause Diarrhea
• Low estrogen OCP cause Hepatic Adenoma
• High Estrogen OCP Prolong/Long term use cause Endometrial CA
• Hormone Replacement Therapy causes Breast CA>DVT (Reference: Robbins)
• Digoxin Toxicity increase by Hypokalemia & Alkalosis
• Digoxin Toxicity Cause Hyperkalemia
• Thiazide cause Hypokalemia> Hyperglycemia > Hyperlipidemia> Hyperuricemia >Hypercalcemia
• Thrombocytopenia byHeparin >Quinidine>Thiazide>Chloramphenicol
• Diazepam Side effect Loss of Beat to beat Variability > Neonatal Hypotonia
• Thiopental Action terminated by redistribution in tissue and fat (Reference: First Aid Edition
2019)
KETAMINE:
• Profound Analgesia
• Increase Heart Rate and Blood Pressure (Sympathomimetic)
• Anesthetic in Asthma
• Cause Bronchodilation
• Raised ICP (avoided in Head Injury)
• Used in Burn Patient and Haemodynamically Unstable
HALOTHANE:
SK Origional – Golden 11 436

• Increase Cerebral Blood Flow


• Cause Malignant Hyperthermia
• Cause Skeletal Muscle Contractions
• Dissociates on Light
Bupivacaine:
• First Sign of toxicityRinging in Ear
• Serious Side effect Arrhythmia
• Safe Dose 150mg
• Rupivacain preferred over it because of Less CNS Toxic
258. Highly repeated mcqs from ganong physiology:
• Pan hypopituitarism is not associated with Cachexia
• Room temp 21 °Humidity 80% heat is lost from uncovered body by Radiation and Conduction
• Diabetic Autonomic neuropathy manifests in GIT as Gastroparesis Rx is Metoclopramide
• Mole for mole, which of the following has the greatest effect on Na excretion ALDOSTERONE
• Mole for mole, which of the following has the greatest effect on plasma osmolality
VASOPRESSIN (ADH)
• Aldosterone main site of action Cortical Collecting Ducts
• The pressure differential between the heart and aorta is least in the Left Ventricle during Systole
• Injection tPA is more beneficial in MI during 2nd hour of occlusion of Coronary Artery.
259. Patient after post thyroidectomy developed hoarseness of voice. Examination reveals
loosening of vocal cords. Which of the following is most likely damaged?
a. Cricothyroid
b. Post cricoarytenoid
c. Cricoarytenoid
Ans: A
Explanation:
Hoarseness of voice is not only the feature of RLN. It can also occur in injury to ELN. So do not get
confused between both.
Reference: Snell's Anatomy
260. Trachea bifurcates at the level of?
a. T4
b. T5
c. T4-T5
Ans: C
Explanation:
Trachea bifurcates at the lower border of T4 - Snell's Anatomy
Trachea bifurcates at the upper border of T5 - RJ Last
261. Stability of knee joint mainly by?
a. Vastus Intermedius
b. Vastus Lateralis
c. Vastus Medialis
Ans: C (Reference: Snell and RJ Last)
262. Patient brought to ER in state of shock (Unrecordable Blood Pressure) with heart failure,
which should be given?
a. Norepinephrine
b. Dopamine
c. Dobutamine
Ans: B
Explanation:
This is simply a case of cardiogenic shock! Why NE isn't preferred, details in references.. NE can be given in
Dopamine refractory shock. But not before Dopamine.
Reference: Katzung, goodman gilman pharma.
263. Sesamoid cartilage is found in?
SK Origional – Golden 11 437

a. Larynx
b. Ala of Nose
Ans: B (Reference: KLM and Dhingra)
264. Most common cause of autoimmune hemolytic anemia:
a. Idiopathic
b. SLE
c. Drugs
Ans: A (Reference: Oxford handbook of clinical medicine)
265. Hematocrit is defined as:
a. RBCs+WBC s+Platelets
b. Only RBCs
Ans: B (Reference: Ganong)
266. The most likely cause of hypercalcemia in malignancy:
a. Parathormone related proteins
b. Parathormone
c. Calcitonin
d. Bone metastasis
Ans: D
267. A very frequently asked topic in exam:
Sickle Cell Anemia:
It is an autosomal recessive disorder. Occurs as a result of point mutation.
Pathophysiology:
Hemoglobin S (HbS) is the result of a single base-pair change, thymine for adenine, at the 6th codon
of the β-globin gene. This change encodes valine instead of glutamine in the 6th residue in the β-globin
molecule. Sickle cell anemia (HbSS), homozygous HbSS, occurs when both β-globin alleles have the sickle
cell mutation (βs).
CLINICAL MANIFESTATIONS AND COMPLICATIONS:
As early as 6 months of age, infants with sickle cell anemia develop abnormal immune function
because of splenic dysfunction. By 5 years of age, most children with sickle cell anemia have complete
functional asplenia Regardless of age; all patients with sickle cell anemia are at increased risk of infection and
death from bacterial infection, particularly encapsulated organisms such as Streptococcus pneumoniae,
Haemophilus influenza type b, and Neisseria meningitides.
APLASTIC CRISIS:
Human parvovirus B19 infection poses a unique threat for patients with sickle cell disease because
this infection results in temporary red cell aplasia , limiting the production of reticulocytes and causing
profound anemia. Any child with sickle cell disease, fever, and reticul-cytopenia should be presumed to have
parvovirus B19 infection until proven otherwise.
SPLENIC SEQUESTRATION:
Acute splenic sequestration is a life-threatening complication occurring primarily in infants and
young children with sickle cell anemia. Sequestration can occur as early as 5 week of age but most often
occurs in children between ages 6 months and 2 year. Patients with the SC and Sβ+thalassemia types of sickle
cell disease can have acute splenic sequestration events throughout adolescence and adulthood. Splenic
sequestration is associated with rapid spleen enlargement causing left sided abdominal pain and Hb decline of
at least 2 g/dL from the patient's baseline. Sequestration may lead to signs of hypervolemia as a result of the
trapping of blood in the spleen and profound anemia, with total Hb falling below3 g/dL. A decrease in WBC
and platelet count may also be present. Sequestration may be triggered by fever, bacteremia, or viral
infections. Treatment includes early intervention and maintenance of hemodynamic stability using isotonic
fluid or blood transfusions. Careful blood transfusions with RBCs are recommended to treat both the
sequestration and the resultant anemia.
SICKLE CELL PAIN/VASOOCCLUSIVE CRISIS:
Dactylitis, referred to as hand-foot syndrome, is often the first manifestation of pain in infants and
young children with sickle cell anemia, occurring in 50% of children by their 2nd year of life. Dactylitis often
manifests with symmetric or unilateral swelling of the hands and/or feet. The Cardinal clinical feature of
SK Origional – Golden 11 438

sickle cell disease is Acute Vasoocclusive Pain Acute sickle cell pain is characterized as unremitting
discomfort that can occur in any part of the body but most often occurs in the chest, abdomen, or
extremities. These painful episodes are often abrupt and cause disruption of daily life activities and significant
stress for children and their caregivers. The pathogenesis may be initiated when blood flow is disrupted in the
microvasculature by sickled red blood cells and other cellular elements, resulting in tissue ischemia. Acute
sickle cell pain may be precipitated by physical stress, infection, dehydration, hypoxia, local or systemic
acidosis, exposure to cold, and swimming for prolonged periods.
AVASCULAR NECROSIS AND PRIAPISM:
Avascular necrosis (AVN ) occurs at a higher rate among children with sickle cell disease than in the
general population and is a source of both acute and chronic pain. Most often the femoral head is affected.
Priapism, defined as an unwanted painful erection of the penis, affects males of all genotypes but most
frequently affects males with sickle cell anemia. The mean age of first episode is 15 yr, although priapism has
been reported in children as young as 3 years some Neurological complications like Ischemic Stroke can also
occur.
268. Lungs activate which of the following?
a. Angiotensin 1
b. Angiotensin 2
Ans: B
Explanation:
Conversion and activation are different angiotensin 1 is not an active form and can't be activated. When it's
structurally altered by ACE, it's converted into Angiotensin 2 in lungs which is an active form Angiotensin 1
is never activated. It's converted, and then that converted form is activated.
Reference: Ganong
269.
• Most common cause of impaired wound healing - Infections
• Most common pathogen in wound infections - S. Aureus
270. Hormone acting is relation between puberty and weight gain:
a. Leptin
b. Estradiol
Ans: A (Reference: Ganong Physiology)
271. CVP is increased by:
a. Decrease Heart Rate
b. Increase Total Peripheral Resistance
Ans: A (Reference: Guyton)
Explanation:
Central venous pressure is decreased during negative pressure breathing and shock. It is increased by
positive pressure breathing, straining, expansion of the blood volume, and heart failure. It's directly
proportional to peripheral venous pressure but inversely proportional to TPR.
272. Patient was diagnosed to have a pale infarct. In which condition pale infarct occurs?
a. Hypertensive stroke
b. Embolic/ischemic stroke
c. Rupture of berry aneurysm
d. Atherosclerosis
e. Rupture of Av fistula
Ans: B (Reference: Goljan Pathology)
Explanation:
Pale (ischemic) types of infarctions:
Increased density of tissue (heart, kidney, spleen) prevents RBCs released from damaged vessels from
diffusing through the necrotic tissue; therefore the tissue has a pale appearance.
Hemorrhagic (red) types of infarctions:
Loose-textured tissue (lungs, small bowel, testicle) allows RBCs released from damaged vessels to diffuse
through the necrotic tissue; therefore the tissue has a hemorrhagic appearance.
Atherosclerotic (thrombotic) stroke
SK Origional – Golden 11 439

1. Most common overall type of stroke


2. Ischemic type of stroke caused by a platelet thrombosis that develops over a disrupted
atherosclerotic plaque and causes pale infarction.
273. Best for diagnosis of folate deficiency?
a. Serum Folate levels
b. RBCs Folate levels
Ans: B (Reference: Oxford Clinical Medicine)
274. Growth hormone secretion is increased by:
a. Sleep
b. Hyperglycemia
c. Exercise
Ans: C
Sequence to follow is: Hypoglycemia>Strenuous Exercise>NREM Sleep>REM Sleep
275. Septic shock is mainly caused by:
a. Gram Positive Bacteria
b. Gram Negative Bacteria
Ans: A (Reference: Robbins, Goljan)
276. Diagnosis of TB is confirmed by:
A. PCR
B. Sputum for AFB
Ans: B
Q. Microscopic diagnosis of TB is made by?
Ans: Caseous necrosis/Casseating granulomas
Q. Histological diagnosis of TB is made by?
Ans: Epithelioid cells
277. In presence of vasopressin greatest fraction of filtered water is reabsorbed in which part of
nephron?
a. PCT
b. DCT
c. CT
d. Loop of Henle
Ans: A
Q. In absence of ADH maximum absorption of water takes place in.
a. PCT
b. DCT
c. CT
d. Loop of Henle
Ans: A
Explanation:
Maximum fraction of water is reabsorbed from PCT that is 60-70% of total water reaching kidneys. 15% of
water is reabsorbed from descending loop of Henle and ascending part is impermeable to water 20% of
filtered water is reabsorbed from DCT.
Vasopressin (ADH) has no role on PCT. Its action is mainly on DCT and CD. So, by the time water reaches
DCT its maximum fraction is already reabsorbed from PCT and descending loop of Henle where ADH
(vasopressin) has no role.
Hence, whether it's the presence or absence of ADH, maximum fluid is reabsorbed from PCT
Reference: Ganong physiology
278. Primigravida came to you and her HB is 12 mg/dl. She wants to know about her iron
requirement what would u prefer?
a. Apple
b. Fresh green leafy vegetables
c. Elemental Iron
Ans: C (Reference: Beckman and Ling's Gynecology and Obstetrics)
SK Origional – Golden 11 440

Explanation:
Supplemental iron use in pregnancy is intended to prevent iron deficiency in the mother not to
prevent either iron deficiency in the fetus or to maintain maternal Hb concentration. To meet maternal iron
needs in a woman who is not anemic, 60 mg of elemental iron is recommended daily.
Because iron from dietary sources may not be sufficient, the National Academy of Sciences
recommends an iron supplement of 27 mg (present in most prenatal vitamins). In the form of ferrous sulfate,
60 mg of iron is a dosage of 300 mg. Patients who are anemic should receive 60 to 120 mg of iron.
279. IDA in pregnancy is best diagnosed by:
a. TIBC
b. Serum Ferritin
Ans: B
Explanation:
Both of them are required to make diagnosis. But serum ferritin is confirmatory.
280. Iron requirement in pregnancy:
a. 800mg
b. 1200mg
Ans: A
281. Calcium requirement in pregnancy:
a. 800mg
b. 1200mg
Ans: B
282.
• S1 Low Pitched, Low Frequency, Duration of about 0.15 seconds
• S2 High Pitched, High Frequency, Duration of about 0.12 second
283. Foot drop is caused by injury to:
a. Deep Peroneal (fibular) Nerve
b. Common Peroneal (fibular) Nerve
Ans: B (References: Snell Anatomy, Atlas, RJ Last)
284. During pregnancy, lactation is inhibited by which of the following?
a. Progesterone
b. Estrogen
c. Prolactin
d. Oxytocin
Ans: B
Explanation:
During pregnancy both estrogen and progesterone levels are increased and help in development of
breast. No lactation occurs during pregnancy due to inhibitory effects of both estrogen and progesterone.
285.
Opioid of choice in terminally ill cancer patients  Morphine
Opioid of choice in Acute Pancreatitis  Morphine
286. Appendicular artery is the branch of:
a. Posterior cecal artery
b. Ileocolic artery
Ans: B (References: RJ Last, Atlas Anatomy, BRS Anatomy)
287. Person had swelling in upper arm after injury. Swelling remained for 3 months but resolved
in 1 year. What was the cause of swelling?
a. Hypertrophy
b. Hyperplasia
c. Metastatic calcification
d. Dysplasia
e. Metaplasia
Ans: E (Reference: Robbins)
Explanation:
SK Origional – Golden 11 441

Myositis ossificans is benign process characterized by heterotopic ossification usually within large muscles.
288. What is the most common feature of autoimmune disorders?
A. Hematological
B. Arthritis
Ans: A
Explanation:
What are hematological changes? These are most commonly.
Anemia
Thrombocytopenia
Leukopenia
Autoimmune diseases are immune reactions to self-antigens (autoimmunity) are the underlying cause of
numerous human diseases Autoimmune reactions can present as hypersensitivity type 2, 3 and 4
These reactions include antigens plus antibodies and cell mediated hypersensitivity (T-cell mediated). All of
them involve blood and its components so affect all the cell lines causing hematological changes.
289. Least Hb in infant at which age?
a. 1st month
b. 6th month
c. 3rd month
d. 1st year
Ans: C
290. Gardener spraying insecticides become unconscious with increased salivation. Which drug
will antagonize symptoms?
a. Pralidoxime
b. Atropine
Ans: B
Q. Gardener spraying insecticides become unconscious with increased salivation. Which drug
acts as Specific Antidote?
a. Pralidoxime
b. Atropine
Ans: A
Explanation:
Organophosphates are insecticides that exert their toxicity through inhibition of acetyl cholinesterase, with
subsequent accumulation of excess acetylcholine producing nicotinic (mydriasis, fasciculation‘s, muscle
weakness, hypertension) and muscarinic (diarrhea, urination, miosis, bradycardia, bronchorrhea, emesis,
lacrimation, salivation) effects.
Pralidoxime can reactivate inhibited AChE. However, it is unable to penetrate into the CNS and therefore is
not useful in treating the CNS effects of organophosphates.. So we need to give it within minutes to stop
AChE from permanent inhibition. This is the specific antidote if we give it within a specified period of time.
Atropine on the other hand antagonizes the symptoms (mentioned above) caused by OP poisoning.
Precisely, Specific Antidote – Pralidoxime
Antagonizes symptoms – Atropine
291. A 28-year-old primiparous woman, with no previous history of infection with herpes zoster, is
18 weeks pregnant. She had significant contact with a young girl with widespread chicken
pox 12 days ago. Serum stored from an antenatal booking blood sample was sent for serology
and came back negative for VZV IgG. She has no rash at present. What is the SINGLE most
appropriate management?
A. Oral acyclovir
B. Intravenous acyclovir and varicella-zoster immunoglobulin (VZIG)
C. Intravenous acyclovir
D. Nothing required, start acyclovir once rash appeared
E. Intravenous varicella-zoster immunoglobulin (VZIG)
Ans: D
292. Blood is warmed at 37°c before transfusion for the following reason?
SK Origional – Golden 11 442

a. To Prevent Infection/Reaction
b. Left to Right Shift
Ans: B
Explanation:
Recall O2-Hb curve. What causes it shift to right and what shifts it to left. Shift to left means O2 affinity is
increased and that will be hard to deliver O2 to tissues. And one of the causes is decrease in temperature
Shift to right means O2 affinity is decreased and now it is easier to deliver O2 to tissues. And increased
temperature shifts to curve to right, warming of blood to prevent reaction are less likely. That's basically
washed RBCs that are used to prevent reaction.
293. Fine voluntary motor movement controlled by:
a. Corticospinal
b. Cerebellum
Ans: B
294. Fine skilled motor movements controlled by:
a. Corticospinal Tract
b. Cerebellum
Ans: B
295. Fine voluntary discrete moments are controlled by:
a. Corticorubrospinal Tract
b. Corticospinal Tract
Ans: B
296. Most common feature of carpal tunnel syndrome:
A. Wasting of Thenar eminence
B. Loss of sensation on Lateral aspect of Hand
Ans: B (Reference: RJ Last)
Explanation:
Do not confuse yourself with "spared sensation on Thenar eminence" because lateral side of hand has palm
and fingers too, not only Thenar eminence.
Sensation is spared on Thenar eminence only but lost on lateral 3 and a half finger. Wasting of Thenar
eminence may occur in long term but still not the most common feature.
297. Loss of sensation in Thumb, index finger and wasting of Thenar eminence. Most common
cause:
A. Myxedema
B. Pregnancy
C. Obesity
Ans: B (Ref: Goljan Pathology)
298. Anatomy:
• Biceps Jerk C5-C6 But Main is C6
• Triceps Jerk C6-C7 But Main is C7
• Knee Jerk L3-L4 But Main is L3
• Ankle Jerk S1(Reference: RJ Last)
299. In hypovolemic shock the decreased glomerular filtration is due to:
a. Dec arterial blood flow
b. Dec arterial volume
c. Dec glomerular pressure
d. Dec arterial pressure
Ans: A (Ref: Ganong + GuytonExplanation)
In hypovolemic shock there is already reduced blood volume hence the blood flow to kidneys decrease,
because of compensatory constriction as well which leads to decrease in GFR.
300. A dislodged thrombus from femoral will first go to?
a. Pulmonary artery
b. IVC
Ans: B
SK Origional – Golden 11 443

Q. A dislodged thrombus from femoral will lodge into?


a. Pulmonary artery
b. IVC
Ans: A
Explanation:
A dislodged thrombus from lower extremity will be primarily dislodged most commonly from
popliteal vein as that's the most common site for DVT. So when it is dislodged it goes to femoral vein which
becomes the most common source of PE. It then goes to iliac veins and then to IVC. IVC is larger in
diameter that's why that's not possible that the embolus gets trapped (lodged) here. And after traveling
through heart it finally reaches pulmonary artery, and here it gets trapped (lodged) because of smaller
diameter. So, comprehensively, It first GOES to IVC
It LODGES in Pulmonary artery.
301. While performing bronchoscopy first structure visualized beyond carina is?
A. Upper lobe bronchus
B. Superior segmental bronchus
C. Apical bronchus
D. Middle lobe bronchus
E. Inferior lobe bronchus
Ans: A
302. Patient is found to have bone pain and pallor. His blood picture shows numerous cells with
an off center nucleus. There is an increased number of immunoglobulin in serum that are
monoclonal in origin. What's the likely percentage of immunoglobulin in this patient?
a. IgG 15%, IgA 2.1%, IgM 0.8%
b. IgG 17%, IgA 0.6%, IgM 0.3%
c. IgG 23%, IgA 0.5%, IgM 0.2%
d. IgG 60%, IgA 20%, IgM 15%
Ans: D (Ref: Robbins and Harshmohan)
Explanation: This is classical case of multiple myeloma, although IgM is rarely observed.
303. Rubella aka german measles:
• Most common congenital anomaly Deafness
• Most common before week 7 Cataract
• Cardiac anomaly 1st trimester but not so common
• Commonest Cardiac anomaly associated with Rubella PDA
References: Snell's Ocular Anatomy and Davidson
304. Female with arterial disease pulsatile mass in abdomen on examination originating from?
a. T12-L2
b. L1-L3
Ans: B
Explanation:
This is the scenario of Abdominal Aortic Aneurysm! So now know these facts. Renal artery origin level L2
Clinical box from Snell is saying that abdominal aneurysms more often originate below the level of Renal.
Artery which makes it L3 at least? In 95% cases it is below renal arteries below L2
so L3 is the most frequent site for AAA Remember abdominal aorta is between below T12 till L4.
305. A 2 year old child presented in outpatient department with fever pallor and bowel
disturbance. Lab investigation revealed Hb 5.6 gm/dl, MCV 107 fl, MCH 340 pg., MCHC
36.0 gm/dl. What investigation will confirm the diagnosis?
a. Intrinsic Factor Antibodies
b. Lactate Dehydrogenase
c. Reticulocyte Count
d. Serum Vitamin B12
e. Transcobalamin
Ans: D (Reference: Oxford Pediatrics)
SK Origional – Golden 11 444

Explanation:
Autoimmune disorders are rare in infants and children. They most commonly present in middle age. Also
presentation seems to be acute with altered bowel habits. And deficiency may have resulted from low intake
of vitamin B12.
306. During pharyngeal phase of swallowing, what occurs?
a. Vocal cord approximate
b. Larynx moves upward and forward
c. Closure of epiglottis
d. Abduction of vocal cord
Ans: A (Reference: Guyton)
Explanation:
During pharyngeal phase of swallowing vocal cord approximation and upward and anterior movement of
larynx occur simultaneously. But Approximation of Cords is most important.
307. GFR is best measured by:
a. Creatinine clearance
b. Inulin clearance
Ans: B
308. GFR is best estimated by:
a. Creatinine Clearance
b. Inulin Clearance
Ans: A
309. Best to monitor dialysis therapy:
a. Urea
b. Creatinine
Ans: B
Creatinine Clearance (GFR) Levels in CRF:
Normal  >90
Mild  60-89
Moderate  30-59
Severe  15-29
ESRD  < 15
310. Most common cause of congenital hypothyroidism is?
a. Thyroid dysgenesis
b. Maternal iodine deficiency
Ans: A (Reference: Davidson, Oxford handbook of pediatrics, Harshmohan, MRCPCH)
Explanation:
Maternal iodine deficiency was previously the commonest cause before the interventions were taken to
reduce the deficiency with the use of iodine salt etc. Maternal iodine deficiency is the common cause in
underdeveloped countries but not in developed.
311. Submental and submandibular LNs drain which of the following?
a. CA Tongue
b. CA Lower Lip
Ans: B (References. Snell's Anatomy)
Explanation:
Submandibular Lymph Nodes:
These lie superficial to the submandibular salivary gland just below the lower margin of the jaw. They receive
lymph from the front of the scalp; the nose; the cheek; the upper lip and the lower lip (except the central
part); the frontal, maxillary, and ethmoid sinuses; the upper and lower teeth (except the lower incisors) the
anterior two thirds of the tongue (except the tip and remember tongue has posterior 1/3rd too that this group
as well as submental don't receive) the floor of the mouth and vestibule; and the gums.
Submental Lymph Nodes:
SK Origional – Golden 11 445

These lie in the submental triangle just below the chin. They drain lymph from the tip of the tongue, the floor
of the anterior part of the mouth, the incisor teeth, the center part of the lower lip, and the skin over the chin.
Posterior 1/3rd of the tongue is drained in Deep Cervical LNs.
312. What is the most important response to shock?
A. Baroreceptor reflex
B. CNS ischemic response
Ans: A
313. Maximum feedback gain is via?
A. Baroreceptors
B. CNS ischemic
C. RAAS
Ans: C
314. Long term BP control:
A. Baroreceptors
B. RAAS
Ans: B
315. A shopkeeper suddenly collapsed with thready pulse. O/E his BP was 80/50. Cold clammy
skin and gives history of sudden crushing chest pain before collapse Diagnosis?
a. Massive Pulmonary Embolism
b. Cardiogenic Shock.
Ans: A (Reference: Davidson)
Explanation: The majority emboli result from propagation of lower limb deep vein thrombosis
Clinical presentation varies, depending on number, size and distribution of emboli and on underlying
cardiorespiratory reserve. A recognized risk factor is present in 80–90% cases. (And here we have a
recognized risk factor him being a Shopkeeper).
316. Which drug does not cause Gynecomastia?
a. Griseofulvin
b. Androgen
Ans: B (Ref: Davidson)
Explanation:
Gynecomastia is the presence of glandular breast tissue in males. Normal breast development in women is
estrogen-dependent, while androgens oppose this effect. Gynecomastia results from an imbalance between
androgen and estrogen activity, which may reflect androgen deficiency or estrogen excess.
317. Down syndrome robertsonian translocation transmits in how much percentage to next
generation?
A. 15%
B. 50%
C. 33%
D. 80%
Ans: A
318. A couple with first child with Down syndrome. Both are young, father normal, mother has
robertsonian translocation. What is risk of Down syndrome in next baby?
A. 33%
B. 66%
C. 100%
D. 5%
Ans: D
Explanation: Down's syndrome occurs as a result of two things.
• Nondisjunction 95%
• Robertsonian Translocation5%
The recurrence risk for female Robertsonian translocation carriers is 10-15%, and that for male translocation
carriers is 1-2%.
319. What is most important for transplant?
SK Origional – Golden 11 446

a. HLA matching
b. ABO compatibility
Ans: B
Explanation:
In case of allogeneic bone marrow transplant HLA typing is most important and we take bone marrow of
ABO incompatible but HLA matched related siblings so HLA is right answer
and same principle is followed for solid organ transplant. HLA matching is more important than ABO group
matching.
Transplant best investigation:
HLA Matching that is first and single best and most important test for solid organ transplant (In that patient
case donor blood group is O+ and recipient is A+ and HLA matching is 5/5 i.e. HLA-A,B,C and DQ DP
matched) same rule applies for bone marrow transplant. (Do remember hematopoietic stems cells don‘t have
ABO antigens on them so no need to match ABO).
320. Patient comes with tender right hypochondrium, stool with occult blood. on colonoscopy
ascending colon was involved, showing ulcers while other parts of colon were spared. on
histology/ microscopy what will be seen?
a. Necrotizing Vasculitis
b. Crypt Abscess
c. Entamoeba Histolytica
d. Creeping Fat
Ans: C
321. Regarding nevus:
a. Benign neoplasm of melanocytes
b. Congenital nevus is present at birth; often associated with hair
c. Acquired nevus arises later in life.
1. Begins as nests of melanocytes at the dermal-epidermal junction (Junctional nevus); most common
mole in children.
2. Grows by extension into the dermis (compound nevus)
3. Junctional component is eventually lost resulting in an intradermal nevus, which is the most common
mole in adults.
d. Characterized by a flat macule or raised papule with symmetry, sharp borders, evenly distributed
color, and small diameter (< 6 mm)
e. Dysplasia may arise (dysplastic nevus), which is a precursor to melanoma.
Early lesions:
Are composed of round-to-oval cells that grow in ―nests‖ along the dermo epidermal junction. Nuclei are
uniform and round, and contain inconspicuous nucleoli with little or no mitotic activity. Such early-stage
lesions are called Junctional nevi eventually, most Junctional nevi grow into the underlying dermis as nests or
cords of cells (compound nevi), and in older lesions the epidermal nests may be lost entirely, creating an
intradermal nevi.
So from this we conclude that:
• Most common nevus in children Junctional
• Most common nevus in adults  Intradermal
• Highest malignant potential  Dysplastic
References: Pathoma and Robbins.
322. Most common site of metastatic calcification:
a. Lungs
b. Kidneys
Ans: A
323. A 78 year old women presents with unilateral headache & blurry vision. She also complains
of pain on chewing. Her ESR is 70 mm/hour. She was subsequently started on high dose
oral steroids. What is the single most appropriate additional treatment?
a. Bisphosphonates
b. Hormone replacement therapy
SK Origional – Golden 11 447

c. ACE inhibitors
d. Beta blockers
e. Timolol
Ans: A
324. A 40 years old alcoholic presented with an Epigastric swelling and fever. 10 days ago he had
pain epigastrium with increased s/amylase which is still elevated Diagnosis?
a. Acute pancreatitis
b. Pseudocyst pancreas
Ans: B
325. A child is born to hepatitis B +ve mother. When must the HBV vaccine first dose be given?
a. Within 1 week
b. Within 2 weeks
c. Within 48 hours
d. Within 24 hours
Ans: D
Explanation: Immunoglobulin and Hepatitis B vaccine should be given within 12 hours ideally.
326. A child in the newborn nursery is A+ and mother is O+. The child has anemia and high
reticulocytes count. What test will confirm diagnosis?
a. Indirect coombs test
b. Direct coombs test
c. Ferritin levels
d. Genetic testing
Ans: B
327. A child is brought to your clinic with protruding tongue, delayed milestones, constipation
and soft belly with a protruding umbilicus Diagnosis?
a. Congenital Adrenal Hyperplasia
b. Congenital Hypothyroidism
c. Disaccharides Deficiency
d. Down's Syndrome
Ans: B
328. How is lactulose excreted from body:
a. Urinary excretion after gut absorption
b. Liver
c. Fecal route without absorption in gut
d. Renal
Ans: C
329. Which vessels are involved in heat exchange?
a. Arteries
b. Arterioles
c. Venules
d. Capillaries
Ans: B
330. 8 years old female presented with high grade fever and severe RIF pain. She was operated
with suspicion of acute appendicitis. When abdomen was opened her right fallopian tube
was inflamed. What is the best management option?
a. Conservative therapy with antibiotics alone
b. Copious irrigation
c. Unilateral adnexectomy
d. Both antibiotics and surgery
Ans: D
331. Investigation of choice in Wilson's disease?
a. Genetic Testing
b. Liver Biopsy
SK Origional – Golden 11 448

Ans: B
332. Putting cell in urea what will happen to the cell?
a. Remains the same
b. Increase ECF volume
c. Decrease ECF volume
d. Increase ICF osmolality
Ans: A (Ref: Guyton)
Explanation:
The terms hyperosmotic and hypo-osmotic refer to solutions that have a higher or lower osmolarity,
respectively, compared with the normal extracellular fluid.
Highly permeating substances, such as urea, can cause transient shifts in fluid volume between the
intracellular and extracellular fluids, but given enough time, the concentrations of these substances eventually
become equal in the two compartments and have little effect on intracellular volume under steady-state
conditions.
Hence no matter solution is hypertonic or hypotonic urea shift between both solution will ultimately bring
equal osmolarity in both and hence no change in the end.
333. A 50 year old man known to have hepatoma develops dependent edema with dilated
abdominal walls veins. The most likely structure involved in this patient is?
a. Hepatic artery
b. Hepatic vein
c. Inferior vena cava
d. Portal Vein
Ans: C (Reference: Robbins)
Explanation:
The inferior vena cava syndrome can be caused by neoplasms that compress or invade the inferior vena cava
or by a thrombus from the hepatic, renal, or lower extremity veins that propagate upward.
Certain neoplasms particularly hepatocellular carcinoma and renal cell carcinoma show a striking tendency to
grow within veins, and these may ultimately occlude the IVC
IVC obstruction induces marked lower extremity edema, distention of the superficial collateral veins of the
lower abdomen, and—with renal vein involvement—massive proteinuria.
334. Female after major abdominal surgery admitted in hospital with respiratory problems,
pulmonary infarct was dx where did the emboli lodged?
a. Paradoxical
b. Emboli in small artery
c. Emboli in medium artery
d. Emboli in large artery
e. None
Ans: B (Ref: Robbins)
Explanation:
Embolic obstruction of medium sized arteries with subsequent vascular rupture can result in pulmonary
hemorrhage but usually does not cause pulmonary infarction. This is because the lung has a dual blood
supply, and the intact bronchial circulation continues to perfuse the affected area
Embolic obstruction of small end-arteriolar pulmonary branches usually does result in hemorrhage or
infarction.
335. Amoeba causes lesions in which part of gut?
a. Terminal ileum
b. Cecum
c. Ascending colon
d. Transverse colon
e. Sigmoid and rectum
Ans: B (Ref: Robbins, Davidson and Goljan)
336. Loss of water by evaporation and insensible loss from body:
a. Controlled by hypothalamus
SK Origional – Golden 11 449

b. Thermal gradient
c. Remains constant
d. Depends on core body temperature
Ans: D (Reference: Guyton)
Explanation:
Insensible water loss water loss of which a person is unaware that is, loss by evaporation from skin excluding
sweat and respiratory passage lining.
Evaporation when water evaporates from the body surface, 0.58 Calorie (kilocalorie) of heat is lost for each
gram of water that evaporates. Even when a person is not sweating, water still evaporates insensibly from the
skin and lungs at a rate of about 600 to 700 ml/day. This causes continual heat loss at a rate of 16 to 19
Calories per hour/
This insensible evaporation through the skin and lungs cannot be controlled for purposes of temperature
regulation because it results from continual diffusion of water molecules through the skin and respiratory
surfaces.
Evaporation Is a Necessary Cooling Mechanism at Very High Air Temperatures. As long as skin temperature
is greater than the temperature of the surroundings, heat can be lost by radiation and conduction.
337. Majority of Serotonin is secreted by:
a. Platelets
b. Agraffian cells of intestine
c. Enterochromaffin cells
d. RBCs
Ans: C (Ref: Vander‘s and Ganong)
Explanation:
Serotonin is found in both neural and non-neural cells with the majority located outside of the CNS
In fact approximately 90% of the body‘s total serotonin is found in the digestive system
8% is in blood platelets and immune cells, only 1% to 2% is found in the brain.
338. Which of following decreases the neuronal excitability in RMP?
a. Hyperkalemia
b. Hypokalemia
c. Hypocalemia
d. Hyponatremia
Ans: B
339. Which of following increases the neuronal excitability in RMP?
a. Hyperkalemia
b. Hypokalemia
c. Hypocalemia
d. Hyponatremia
Ans: A (Ref: Ganong)
Explanation:
Decreasing the external Na+ concentration reduces the size of the action potential but has little effect on the
resting membrane potential. The lack of much effect on the resting membrane potential would be predicted,
since the permeability of the membrane to Na+ at rest is relatively low.
In contrast, since the resting membrane potential is close to the equilibrium potential for K+, changes in the
external concentration of this ion can have major effects on the resting membrane potential.
If the extracellular level of K+ is increased (hyperkalemia), the resting potential moves closer to the threshold
for eliciting an action potential, thus the neuron becomes more excitable
If the extracellular level of K+ is decreased (hypokalemia), the membrane potential is reduced and the neuron
is hyperpolarized.
340. Buffer important in kidney but not in plasma:
a. Ammonia
b. Phosphate
c. HCO3
d. Protein
SK Origional – Golden 11 450

Ans: B (Ref: Guyton)


Explanation:
The phosphate buffer system is composed of HPO4= and H2PO4–. Both become concentrated in the
tubular fluid because of their relatively poor reabsorption hence can provide better buffering in renal tubule
Therefore, phosphate is not an important extracellular fluid buffer; it is much more effective as a buffer in the
tubular fluid.
Another factor that makes phosphate important as a tubular buffer is the fact that the pK of this system is
about 6.8. Under normal conditions, the urine is slightly acidic, and the urine pH is near the pK of the
phosphate buffer system. Therefore, in the tubules, the phosphate buffer system normally functions near its
most effective range of pH.
341. Humans most easily tolerate a lack of which of the following nutrients?
a. Protein
b. Lipids
c. Carbohydrates
d. Calcium
e. Iodine
Ans: C
342. Adrenal medullary substance comprising of how much of adrenal gland:
a. 80%
b. 50%
c. 28%
d. 90%
e. 10%
Ans: C (Reference: Guyton)
Adrenal medulla constitutes 28% of the mass of adrenal gland and in Guyton its upto 20%.
In adult mammals, the adrenal cortex is divided into three zones.
Zona glomerulosa makes up 15% of the mass of the adrenal gland.
Zona fasciculata makes up 50% of the mass of the adrenal gland.
Zona reticularis makes up 7% of the mass of the adrenal gland.
343. Erythropoiesis is stimulated by:
a. Hypoxia
b. Anemia
c. Erythropoietin
d. Low blood volume
Ans: C
344. Erythropoietin is stimulated by:
a. Hypoxia
b. Anemia
c. Erythropoietin
d. Low blood volume
Ans: A (Reference: Guyton)
Explanation: Feel the difference:
Hypoxia stimulates  Erythropoietin
Erythropoietin stimulates  Erythropoiesis
The principal stimulus for red blood cell production in low oxygen states is a circulating hormone called
erythropoietin a glycoprotein with a molecular weight of about 34,000.
In the absence of erythropoietin, hypoxia has little or no effect in stimulating red blood cell production. But
when the erythropoietin system is functional, hypoxia causes a marked increase in erythropoietin production,
and the erythropoietin in turn enhances red blood cell production until the hypoxia is relieved.
345. Shortest acting benzodiazepine:
a. Alprazolam
b. Temazepam
c. Triazolam
SK Origional – Golden 11 451

d. Midazolam
Ans: D (Ref: Katzung, Goodman & Gilman)
Explanation:
Midazolam is most rapid.
Sedative doses of midazolam (0.01-0.05 mg/kg intravenously) reach peak effect in ~2 minutes and provide
sedation for ~30 minutes.
Midazolam has the shortest context-sensitive half-time, which makes it the only one of the three
benzodiazepine drugs suitable for continuous infusion.
Elimination half-life of midazolam is 1.7-2.6 hours and that of Triazolam is 2 to 3 hours.
346. In case of reimplantation of an amputated digit, which of the following structures is fixed
first?
a. Nerve
b. Artery
c. Vein
d. Bone
Ans: D (Ref: Sabiston Text book of Surgery)
Explanation:
In case of replantation of digits 1st Bone shortening is done which allows skin to be debrided back to where
it is free of contusion and direct tension-free closure can be achieved.
Than the order of repair is usually bone tendons, muscle units, arteries, nerves, and finally veins
Establishment of arterial flow before venous flow clears lactic acid from the replanted part. The functional
veins can now also be detected by spurting bleeding. However, blood loss must be closely monitored. With
only a few minor variations, the sequence of replantation has been standardized.
347. Negative charge of basement membrane is due to:
a. Elastin
b. Laminin
c. Cadherin
d. Heparan Sulphate
e. Selectin
Ans: D (Ref: Krauses, Grays and BRS)
Explanation:
Heparan sulfate is a negatively charged (polyanionic) molecule of the glomerular basal lamina. The sialo-
protein (podocalyxin) coats the Podocytes foot processes and together with Heparan sulfate gives the
filtration barrier a net negative charge. The negatively charged glomerular basement membrane prevents or
restricts the filtration of molecules such as albumin and other highly negatively charged molecules.
348. A pregnant hyperthyroid lady, during her 5th month of pregnancy, which antithyroid drug
shoud be given?
a. Propylthiouracil
b. Mathimazole
c. Radioiodine
d. None
Ans: B (Ref: Davidson, Harrison & Sabiston)
Explanation:
Mathimazole is preferred over PTU except for during the first trimester of pregnancy and in life-threatening
thyroid storm for the treatment of hyperthyroidism because it can reverse hyperthyroidism more quickly, can
be dosed once daily, and has fewer concerns for hepatotoxicity.
So 1st Trimester  PTU
Thyroid Storm  PTU from 2nd Trimester  Mathimazole
349. Common direction of Shoulder joint dislocation?
a. Superior
b. Inferior
c. Anterior
d. Medial
SK Origional – Golden 11 452

e. Lateral
Ans: C (Reference: Grays, RJ Last, KLM)
350. Common direction of humeral head dislocation:
a. Superior
b. Inferior
c. Anterior
d. Medial
e. Lateral
Ans: B (Ref: Grays, RJ Last, KLM)
Explanation:
Because of its freedom of movement and instability, the glenohumeral joint is most common dislocated joint
in body.
Because the presence of the coraco-acromial arch and support of the rotator cuff are effective in preventing
upward dislocation, most dislocations of the humeral head occur in the downward (inferior) direction.
However, they are described clinically as anterior or posterior (rarely) dislocations, indicating whether the
humeral head has descended anterior or posterior to the infraglenoid tubercle and long head of the triceps.
The head ends up lying anterior or posterior to the glenoid cavity.
351. Most common predisposing factor for squamous cell carcinoma of skin:
A. Marjolin ulcer
B. Actinic keratosis
C. Bowen's disease
D. None
Ans: C (Ref: Davidson & Goljan)
Explanation:
Both Actinic keratosis and Bowen's disease leads to squamous cell carcinoma of skin.
But in almost all books conversion rate of Bowen's disease is greater than actinic keratosis.
352. Drug of choice for MRSA:
a. Fusidic Acid
b. Metronidazole
c. Aztreonam
d. Penicillin
e. Co-amoxiclav
Ans: A (Ref: Davidson)
Explanation: Following drugs are used for MRSA
Clindamycin, vancomycin, rifampicin, linezolid, daptomycin, Tetracycline, Tige-cycline, Co-trimoxazole so if
these are not present as in above question fusidic acid is right option.
Fusidic Acid, active against Gram-positive bacteria, is available in intravenous, oral or topical formulations. It
is lipid-soluble and distributes well to tissues. Fusidic acid is used in combination for MRSA with clindamycin
or rifampicin.
353. Marker raised 60% in colon cancer n 80% in pancreatic cancer?
a. CEA
b. CA19-9
c. CA125
d. CA15-3
e. PSA
Ans: B (Ref: Goljan)
Explanation:
CEA More specific for colorectal carcinoma and also involved in pancreatic, lung, stomach and heart.
CA19-9 more specific for pancreatic tumor and also involved in colorectal carcinoma.
As here asked about 80% pancreatic and 60% colorectal hence CA19-9 seems more preferable.
354. Serious adverse effects of valproate include:
a. Chronic pancreatitis
b. Fulminant hepatitis
SK Origional – Golden 11 453

c. Acute renal failure


d. Coagulation disorder
e. Myasthenia syndrome
Ans: B (Ref: Goodman & Gilman)
Explanation:
The most common side effects are transient GI symptoms, including anorexia, nausea, and vomiting in 16%
of patients. Effects on the CNS include sedation, ataxia, and tremor; these symptoms occur infrequently
Rash, alopecia, and stimulation of appetite have been observed occasionally and weight gain has been seen
with chronic Valporic acid treatment in some patients.
Valporic acid has several effects on hepatic function. Elevation of hepatic transaminases in plasma is
observed in up to 40% of patients and often occurs asymptomatically during the first several months of
therapy.
A rare complication is a Fulminant hepatitis that is frequently fatal
355. Purkinje fibers are fastest in transmitting impulse because:
a. They have wider diameter
b. They have large no of sodium channels
c. Intercalated junction
d. Gap junction
Ans: A (Reference: Guyton)
Explanation:
1) Large diameter or 2) high permeability of gap junctions at intercalated discs so as in these bcqs it‘s just
simply gap junctions, scanty gap junction or just intercalated disc or junctions given this is wrong.
Purkinji fibers are very large fibers, even larger than the normal ventricular muscle fibers, and they transmit
action potentials at a velocity of 1.5 to 4.0 m/sec, a velocity about 6 times that in the usual ventricular muscle
and 150 times that in some of the A-V nodal fibers. This allows almost instantaneous transmission of the
cardiac impulse throughout the entire remainder of the ventricular muscle.
356. The most probable cause of edema in cirrhosis of liver is?
a. Hypoprotenemia
b. Increase in Systolic Blood Pressure
c. Inflammation of Serosal Layer
d. Portal Hypertension
e. Sodium Retention
Ans: D (Reference: Goljan & Davidson)
Explanation:
Actually both hypoprotenemia and portal hypertension are major causes of ascites in cirrhosis but if you have
to choose one choose portal hypertension coz this is main contributing factor along with splanchnic
vasodilation.
Splanchnic vasodilatation is thought to be the main factor leading to ascites in cirrhosis. This is mediated by
vasodilators (mainly nitric oxide) that are released when portal hypertension causes shunting of blood into the
systemic circulation. Systemic arterial pressure falls due to pronounced splanchnic vasodilatation as cirrhosis
advances. This leads to activation of the renin–angiotensin system with secondary aldosteronism, increased
sympathetic nervous activity, increased atrial natriuretic hormone secretion and altered activity of the
kallikrein–kinin system. These systems tend to normalize arterial pressure but produce salt and water
retention. In this setting the combination of splanchnic arterial vasodilatation and portal hypertension alters
intestinal capillary permeability, promoting accumulation of fluid within the peritoneum.
357. Quotidian or malignant tertian malaria is caused by?
a. P Malaria
b. P Falciparum
c. P ovale
d. P vivax
Ans: B (Reference: Goljan)
358. Simple Tertian malaria is caused by:
a. P Malaria
SK Origional – Golden 11 454

b. P Falciparum
c. P ovale
d. P vivax
Ans: D
359. Quartan malaria is caused by:
a. P. Malaria
b. P. Falciparum
c. P. Ovale
d. P. Vivax
Ans: A
Explanation:
Quotidian fever daily spikes with no pattern
Tertian fever every 48 hours
Quartan fever every 72 hours
Quotidian also called Malignant Tertian
360. Histopathology report showing pleomorphism, rete ridges, loss of polarity and increased
nuclear to cytoplasmic ratio what is the diagnosis?
a. Squamous cell CA
b. Verrocus CA
c. Sarcoma
d. None
Ans: B (Reference: Abeloff's clinical oncology)
Explanation:
Verrocus carcinoma of the vulva represents a variant of squamous cell carcinoma. These lesions originally
were described as occurring in the oral cavity, but also have been described involving the vagina, cervix, and
vulva histologically, it is composed of large papillary fronds covered by dense keratin. Its deep margin creates
a pushing border of well oriented rete ridges
361. Which of the following is anti-cancer?
a. Chocolate
b. Apple
c. Cheese
d. Nitrated meat
e. Rum
Ans: B (Ref: Text book of Oncology)
Explanation:
Chocolate is associate with aggravation of symptoms of carcinoid tumors
Fruits rich in dietary fiber are protective in certain cancers like colorectal carcinoma.
362. Patient presented with inability to write, read and speech, most likely lesion is in:
a. Medulla
b. Frontal lobe
c. Temporal lobe
d. Parietal lobe
e. Both temporal and parietal
Ans: E (Ref: Snell Neurology and Text book of Neurology)
Explanation:
It is because of lesion of Wernicke‘s area and the sensory speech area of Wernicke is localized in the left
dominant hemisphere, mainly in the Superior Temporal Gyrus, with extensions around the posterior end of
the lateral sulcus into the parietal region.
363. In a study carotid chemoreceptor denervated in animals so which one is least effected?
a. PO2 Arterial
b. PCO2 Arterial
c. PO2 Venous
d. H+ Ion
SK Origional – Golden 11 455

e. HCO3 Level
Ans: B (Reference: Ganong)
Explanation:
After denervation of the carotid chemoreceptors, the response to a drop in Po2 is abolished. As it is not
mentioned arterial or venous. So we consider abolished to overall response of both.
The response to changes in arterial blood H+ concentration in the pH 7.3–7.5 range is also abolished.
The response to changes in arterial PCO2, on the other hand, is affected only slightly it is reduced no more
than 30–35%.
Why not E? Because HCO3 has direct link with body pH↓ in HCO3 can raise H+ ions in body and hence
decrease pH also HCO3 can ultimately converts into H+ and CO2.
364. In a patient with a tumor in superior mediastinum compressing the superior vena cava, all of
the following veins would serve as alternate pathways for the blood to return to the right
atrium, except:
a. Azygous Vein
b. Hemiazygos Vein
c. Internal Thoracic Vein
d. Vertebral venous Plexus
e. Lumbar Veins
Ans: C (Ref: Grays, KLM and Snell)
Explanation:
In SVC or IVC obstruction there are collateral channels of veins which connect both SVC and IVC and serve
as alternative pathways for draining venous blood in Right atrium.
365. Anterior pituitary tumor will compress:
a. Optic Nerve
b. Occulomotor Nerve
c. Abducent Nerve
d. Trochlear Nerve
e. Trigeminal Nerve
Ans: A (Ref: Text book of Neurology)
Explanation:
The first cranial nerve that is likely to be damaged from a growing pituitary tumor is the optic nerve. The
decussating axons at the optic chiasma will be compressed, followed by the non-decussating fibers.
If the tumor becomes very large, the cranial nerves located on the medial wall of the cavernous sinus (namely
the occulomotor, trochlear, and ophthalmic division of the trigeminal and the abducent nerves) will likely be
compressed and cause deficits in the structures that these nerves innervate.
366. Child with loose skin with easy bruising and defective pro-collagen 3.what can be the most
striking symptom / complication?
a. Hyper extensible skin
b. Hypermobile joints
c. Problem in articular cartilage
d. Rupture of Large Vessels
Ans: D (Ref: Robbins)
Explanation:
It is Scenarios of Ehlers Danlos Syndromes which involve defects in multiple types of collagen hyper
extensible skin and Hypermobile joints are almost present in each type of this syndrome
But in vascular type of this syndrome there is defect of Type 3 collagen so vascular type characteristic feature
is rupture of large vessels.
367. Hypoglycemia increases:
a. CCK
b. Somatostatin
c. Gastrin
d. VIP
e. Secretin
SK Origional – Golden 11 456

Ans: C (Ref: Ganong)


Explanation:
Glucagon is chief hormone of starvation or hypoglycemia state whereas insulin is hormone of feed or
hyperglycemia state.
As in this question Hypoglycemia is mentioned So Body requires Glucagon among above options Gastrin is
its stimulator.
368. Major stress hormone is:
a. ACTH
b. Epinephrine
c. Norepinephrine
d. ADH
Ans: A (Ref: Ganong and Guyton)
Explanation:
By term major stress hormone means during stress body tries to restore normal state by several factors and
among these CORTISOL is most important and ACTH is most imp factor in releasing cortisol.
369. DIC is related to all except:
a. Cancer
b. Amniotic fluid embolism
c. Sepsis
d. Oral Contraceptives
e. Bacteremia
Ans: D (Ref: Robbins & Goljan)
370. Milk is notoriously deficient in:
a. Vitamin C
b. Iron
c. Pantothenic acid
d. Vitamin A
e. Riboflavin
Ans: A (Ref: Chatterjae & Ganong)
371. Type of hypersensitivity in hashimoto's thyroiditis:
a. Type 1
b. Type 2
c. Type 3
d. Type 4
Ans: D (Ref: Goljan)
Explanation:
Cytotoxic T cells destroy parenchyma (Type 4) blocking antibodies against TSH receptor (Type 2) decrease
hormone synthesis (Type 2) Helper T cells release cytokines attracting macrophages that damage tissue (Type
2) Anti-microsomal and anti-thyroglobulin antibodies destroy parenchyma (Type 2)
So both Type 4 and Type 2 are present but mainly Type 4.
372. Low frequency hearing loss in a child?
a. Conductive Deafness
b. Neural Deafness
c. Sensory Deafness
Ans: A (Reference: Dhingra)
Explanation: Features of conductive hearing loss
• Negative Rinne test i.e. BC >>AC
• Weber lateralized to poorer ear
• Low frequency affected more
• Loss is not more than 60 dB
373. 38 year lady has endometrial carcinoma that has involved upto half of the thickness of
myometrium and ovaries. The cancer is in which of the following stages?
a. Stage IA
SK Origional – Golden 11 457

b. Stage IB
c. Stage IIC
d. Stage II
e. Stage III
Ans: E (Ref: Duttass Text book of Gynecology)
Explanation:
Stage 1b Invasion equal to more than half of myometrium but here it‘s also asked about ovaries in the stem
so tumor spread to ovaries occur in stage 3A up to stage 2 tumor doesn't extend beyond the uterus.
374. Basic mechanism involved in aqueous humor production:
a. Active Sodium Secretion
b. Passive Chloride Absorption
c. Glucose Diffusion
d. Ultrafiltration
e. Water Secretion by Simple Diffusion
Ans: A (Ref: Guyton)
Explanation:
Formation rate of 2 to 3 microliters each minute secreted by the Ciliary processes, which are linear folds
projecting from the Ciliary body, Aqueous humor is formed almost entirely as an active secretion by the
epithelium of the Ciliary processes, Secretion begins with active transport of sodium ions into the spaces
between the epithelial cells. The sodium ions pull chloride and bicarbonate ions along with them to maintain
electrical neutrality.
Then all these ions together cause osmosis of water from the blood capillaries lying below into the same
epithelial intercellular spaces.
375. Regarding bone tumors:
• Osteoblastoma Spine (vertebra)
• Osteoid osteoma Long Bones
• Osteochondroma Metaphysis
• Osteosarcoma Metaphysis
• Ewing sarcoma Diaphysis
• Giant cell in teenage Metaphysis
• Giant cell in adults Metaphysis + Epiphysis
Ref: Robbins
376. Premalignant condition is:
a. Myelodysplastic Syndrome
b. Leukoplakia
c. Cervical Erosions
d. Psoriasis
Ans: A (Ref: Goljan)
Explanation:
Leukoplakia is lesion not condition
Myelodysplastic syndromes(MDSs) group of acquired clonal disorders that affect stem cells age 50-80 years
old Frequently progresses to AML (30%ofcases).
377. True hermaphroditism:
a. XX/XY
b. XXY
c. XXX
d. XYY
Ans: A (Ref: Robbins & Goljan)
Explanation: 46xx karyotype is most common with almost 50% if it is not present 2nd best is XX/XY.
378. Aortic dissection is associated with all of the following except:
a. Cystic medial necrosis
b. Hypertension
c. Atherosclerosis
SK Origional – Golden 11 458

d. Old age
Ans: C (Ref: Robbins & Bailey)
Explanation:
Dissection is unusual in the presence of substantial atherosclerosis or other cause of medial scarring such as
syphilis, presumably because the medial fibrosis inhibits propagation of the dissecting hematoma.
379. Most abundant astrocytes in white matter of CNS are:
a. Microglia
b. Fibrous astrocytes
c. Oligodendrocytes
d. Protoplasmic astrocytes
Ans: B (Ref: Snell Neurology and Text book of Neurology, Junqueira Basic Histo & USMLE)
380. Most abundant cells in Gray matter of CNS are:
a. Microglia
b. Fibrous astrocytes
c. Oligodendrocytes
d. Protoplasmic astrocytes
Ans: D
381. Most abundant cells in white matter of CNS are:
a. Microglia
b. Fibrous astrocytes
c. Oligodendrocytes
d. Protoplasmic astrocytes
Ans: C
382. Most abundant glial cells:
a. Astrocytes
b. Oligodendrocytes
c. Ependymal cells
d. Schwann cells
Ans: A
Explanation:
• Overall most common in gray matter are protoplasmic astrocytes
• Overall most common in white matter are Oligodendrocytes
• Astrocytes which are present in white matter are fibrous astrocytes
• Most abundant glial cells are astrocytes
383. Sign of apoptosis:
a. Karyolysis
b. Karyorrhexis
c. Pyknosis
d. Caseation
Ans: B (Ref: Robbins)
384. Apoptosis include the following except:
a. Cell Swelling
b. Chromatin Condensation
c. Increase in Nuclear Size
d. Karyolysis
e. Rupture of Cell Membrane
Ans: A
Explanation: Clearly mentioned in Robbins nuclear breakdown or fragmentation (karyorrhexis) presents in
apoptosis also there is cellular shrinkage in apoptosis not swelling according to Robbins which makes cell
swelling obvious choice.
385. Which one of the following is most likely feature of reversible injury?
a. Cell Swelling
b. Karyorrhexis
SK Origional – Golden 11 459

c. Decreased glycogen
d. Myelin Figure
Ans: A (Ref: Robbins)
Explanation:
 Cellular swelling is the first cellular change in all form of injury and it must be present in reversible injury
 Whereas myelin figures are more pronounced feature of irreversible cell injury than reversible clearly
mentioned in Robbins that they might occur in reversible injury so not necessarily occur which makes
swelling more important.
386. All are features of irreversible cell injury except:
a. Karyolysis
b. Karyorrhexis
c. Autolysis
d. Shrinkage of Mitochondria
e. Appearance of Myelin Figure
Ans: D (Reference: Robbins)
Explanation:
In irreversible cell injury there is mitochondrial swelling not shrinkage.
Also myelin figures are more pronounced feature of irreversible cell injury than reversible clearly
mentioned in Robbins that they might occur in reversible injury.
387. Mass of hair shaft is mainly due to:
a. Cortex
b. Matrix of nail
c. Basle plus Spinosum
d. Sebaceous gland
Ans: A (Ref: Gray‘s Anatomy)
Explanation:
 A fully developed hair shaft consists of three concentric zones which are, from outwards in, the cuticle,
cortex and medulla.
 The cortex forms the greater part of the hair shaft and consists of numerous closely packed, elongated
squamous which may contain nuclear remnants and melanosomes.
388. Infection of the first web space will be first drained by?
a. Epitrochlear Lymph Node
b. Pectoral Group of Lymph Node
c. Supraclavicular Lymph Node
d. Infraclavicular Lymph Node
Ans: D (Ref: Snell & Gray‘s anatomy)
Explanation:.
The lymph from the lateral side of the hand ascends in vessels that accompany the cephalic vein; they drain
into the Infraclavicular nodes and some drain into the lateral axillary nodes/.
389. All are vaginal sphincters except:
a. Pubo-vaginalis
b. External Urethral Sphincter
c. Bulbospongiosus
d. Internal Urethral Sphincter
Ans: D (Reference: KLM)
Explanation:
Four muscles compress the vagina and act as sphincters: Pubo-vaginalis, External urethral sphincter,
urethrovaginal sphincter, and Bulbospongiosus.
390. A young female presented in opd with loss of light reflex but intact accomodation reflex. The
lesion is at:
a. Occulomotor Nerve
b. Edinger Westphal Nucleus
c. Pretectal Area
SK Origional – Golden 11 460

d. Frontal Eye Field


e. Optic Nerve
Ans: C (Ref: Snell Neurology)
Explanation:
The pathway of Accommodation Reflex:
RetinaOptic NerveOptic ChiasmaOptic TractLGBOptic Radiations to Visual CortexOccipital
CortexConnected to Frontal CortexDescend through Internal Capsule to Edinger Westphal Nucleus.
The pathway of Light Reflex:
RetinaOptic NerveOptic ChiasmaOptic TractPretectal NucleusEdinger Westphal Nucleus--
>Occulomotor Nerve
Since accommodation reflex is normal so it excludes lesions of Edinger Westphal, Optic Nerve and
Occulomotor Nerve as all these are common in both pathways except Pretectal Area.
391. Bilateral ptosis at the end of the day, confirmatory test?
a. Tensilon Test
b. Antibodies against ACh receptor
c. EMG
d. Edrophonium
e. CT scan
Ans: C (Ref: Harrison and MTB Internal Medicine)
Explanation:
The best initial test is anti-acetylcholine receptors antibodies and its most commonly used test is well.
But they are present only upto 90% of patients and patients with negative test doesn't exclude the disease
hence it is not most accurate the most accurate test.
392. Lipoma on thigh, local anesthetic xylocaine is given which will be affected first?
a. B Fibers
b. C Fibers
c. A Delta
d. A Alpha
Ans: B (Ref: Katzung, Goodman & Gilman)
Explanation:
Treatment with local anesthetics causes the sensation of pain to disappear first, followed by loss of the
sensations of temperature, touch, deep pressure, and finally motor function  C fibers are smallest among all
and also carrying pain sensation hence they will block 1st and will be preferred over B fibers as they don't
carry pain  A delta has lesser sensitivity than these two.
393. Neurotransmitter for mood:
a. Dopamine
b. Serotonin
c. Acetylcholine
d. Epinephrine
Ans: B (Ref. Guyton & Ganong NOTE: If nor-epinephrine present in options prefer it over serotonin)
394. A female had bilateral silicon breast implant placements, now she has arrived with
inflammation, which cells will be demonstrated?
a. Lymphocytes
b. Giant Cells
c. Plasma cells
d. Neutrophils
e. Eosinophil
Ans: B
Explanation:
Foreign body produces chronic inflammation and on histology giant cells are seen.
So with or without inflammation answer will be giant cells. Follow authentic txt bo0ks references rather than
some assumptions created by others.
395. Internal capsule lies medial to:
SK Origional – Golden 11 461

a. Putamen
b. Caudate Nucleus
c. Globus Pallidus
d. Amygdaloid Nucleus
Ans: C (Ref: Atlas)
Explanation: Actually internal capsule lies medial to both Globus Pallidus and putamen. But its immediately
medial to globus Pallidus so we have to prefer it.
396. In inferior wall MI which artery should be blocked?
a. Left Marginal Artery
b. Diagonal Artery
c. Right Coronary Artery
d. Right Marginal Artery
Ans: C (Ref. Robbins & Goljan)
397. In lateral wall MI which artery should be blocked?
a. Right Marginal Artery
b. Diagonal Artery
c. Right Coronary Artery
d. Left Circumflex Artery
Ans: D (Ref. Robbins & Goljan)
398. In anterior wall MI which artery should be blocked?
a. Left Marginal Artery
b. Left Anterior Descending
c. Right Coronary Artery
d. Right Marginal Artery
Ans: B (Ref. Robbins & Goljan)
399. A patient with pyloric stenosis has chronic vomiting. The most likely biochemical
abnormalities in this would be?
a. High serum bicarbonate
b. Hypokalemia
c. Hyponatremia
d. Hypovolemia
e. Increased Kaliuresis
Ans: B (Ref: Bailey & Love)
Explanation:
The vomiting of HCL results in hypochloraemic alkalosis but, initially, sodium and potassium levels may be
relatively normal. However, as dehydration progresses, Later the urine has a low chloride and high
bicarbonate content, reflecting the primary metabolic abnormality. This bicarbonate is excreted along with
sodium and so, with time, the patient becomes progressively hyponatraemic and more profoundly dehydrated
Because of the dehydration, a phase of sodium retention follows and potassium and hydrogen are excreted in
preference. This results in the urine becoming paradoxically acidic and hypokalemia.
So initially ↓H+↓CL↑, HCO3 Later... ↑HCO3, ↑Na in urine so it causes ↓HCO3, ↓Na in serum.
Results  Hyponatremia Then  Na retention occurs and Na back to normal in the end ↑K in urine along
with H+ results ↓K in serum hypokalemia.
400. One of the most important chemical mediators of inflammation is:
a. Histamine
b. Prostaglandin
c. Serotonin
d. Neutrophils
e. None
Ans: A (Ref: Goljan & Robbins)
401. A boy had extraction of deciduous teeth and now profusely bleeding which classic
hemophilia factor is missing?
a. Factor 9
SK Origional – Golden 11 462

b. Factor 8 major components


c. Factor 8 minor component
d. Factor 7
Ans: C (Ref: Guyton)
Explanation:
Factor VIII has two active components, a large component with a molecular weight in the millions and a
smaller component with a molecular weight of about 230,000. The smaller component is most important in
the intrinsic pathway for clotting, and it is deficiency of this part of Factor VIII that causes classic hemophilia.
402. Which of the following is correct about stored blood?
a. Increased 2, 3 DPG
b. Decrease O2 affinity
c. Platelet deficient
d. Increased hemolysis
e. Deficiency of factor 3 5 only
Ans: D (Ref: Ganong)
Explanation: In stored blood following changes occur ↓2, 3 DPG  ↑O2 affinity  Curve shifts towards
left  ↓P50  ↓O2 delivery to tissues.
403. Patient in abducted hand is asked to move hand slowly and smoothly downward but he
dropped hand suddenly by his side. Which muscle is most likely torn?
a. Deltoid
b. Supraspinatus
c. Subscapularis
d. Teres major
e. Pectoral
Ans: B (Ref: Harrison)
404. When a heavy object in hand is lowered, the extension at elbow is brought about by?
a. Active shortening of extensors
b. Passive shortening of extensors
c. Active lengthening of flexors
d. Active shortening of flexors
Ans: C (Ref: BD Chaurasia)
Explanation:
Prime movers (agonists) bring about the desired movement. When a prime mover helps opposite
action by active controlled lengthening against gravity, it is known as action of paradox. For example, putting
a glass back on the table is assisted by gravity but controlled by a gradual active lengthening of biceps
(paradoxical or eccentric action).
405. Nucleoli in active protein synthesis:
a. Prominent nucleolus
b. Appear dense
c. Heterochromatin
d. Disappear
e. None
Ans: A (Ref: Junqueira & Krauses Text books of Histology)
406. Posterior communicating artery connects:
a. ICA with MCA above occulomotor
b. ICA with MCA below occulomotor
c. ICA with PCA above oculomotor
d. ICA with PCA below occulomotor
Ans: C (Ref: Snell Neurology & RJ Last)
407. Posterior communicating artery a branch of?
a. Internal carotid
b. External carotid
c. Middle cerebral
SK Origional – Golden 11 463

d. Posterior superior cerebellar


Ans: A (Ref: Snell Neurology & RJ Last)
Explanation:
The posterior communicating artery is a small vessel that originates from the internal carotid artery close to
its terminal bifurcation.
The posterior communicating artery runs posteriorly above the occulomotor nerve to join the posterior
cerebral artery, thus forming part of the circle of Willis.
408. Which drug does not cause Gynecomastia?
a. Digoxin
b. Griseofulvin
c. Cimetidine
d. Androgen
e. Spironolactone
Ans: D (Ref: Churchill)
409. Highest diffusing capacity across respiratory membrane and in body fluids is?
a. Helium
b. Carbon monoxide
c. Carbon dioxide
d. Nitrogen
e. Oxygen
Ans: C (Ref: Guyton)
410. Gas used to measure diffusing capacity:
a. Helium
b. Carbon monoxide
c. Carbon dioxide
d. Nitrogen
e. Oxygen
Ans: B (Ref: Guyton)
Explanation: In the average young man, the diffusing capacity under resting conditions.
O2  21 ml/min/mmHg
CO  17ml/min/mmHg
CO2  400-450ml/min/mmHg
411. In anterior duodenal perforation the contents will go to:
a. Right iliac fossa
b. Left iliac fossa
c. Anterior sub hepatic space
d. Posterior sub hepatic space
e. Small bursa
Ans: A (Ref: Snell)
412. In anterior duodenal perforation the contents will go to?
a. Right Paracolic gutter
b. Right iliac fossa
c. Anterior sub hepatic space
d. Posterior sub hepatic space
e. Small bursa
Ans: A (Reference: Snell)
Right Paracolic gutter > Right iliac fossa
As in perforation 1st contents go to Right Paracolic gutter than to Right iliac fossa
413. Anion gap is difference between:
a. Measured anion and measured cation
b. Measured cation and unmeasured anion
c. Unmeasured cation and unmeasured anion
SK Origional – Golden 11 464

Ans: C Reference: Guyton


414. Intestinal obstruction caused by carcinoma colon most likely involves?
a. Cecum
b. Ascending colon
c. Transverse colon
d. Sigmoid colon
Ans: D (Ref: Bailey & love)
415. A quack has given drug to patient after which he developed jaundice. Which enzyme will be
raised?
A. Alanine aminotransferase (ALT)
B. Aspartate aminotransferase (AST)
Ans: A (Reference: Harrison & Davidson)
416. In a patient with acute liver failure the best prognostic indicator is?
a. Serum albumin
b. Serum AFP
c. Serum bilirubin
d. Factor V estimation
Ans: D (Reference: Harrison & Davidson)
417. Drugs which produce postural hypotension include?
a. Propranolol
b. Phenoxybenzamine
c. Halothane
d. Clonidine
Ans: B (Reference: Katzung)
418. In femoral sheath which is the most lateral structure?
a. Femoral Artery
b. Femoral Vein
c. Femoral Nerve
d. Lymphatic
Ans: A (Ref: RJ Last & Snell)
419. In femoral sheath which is the most lateral structure?
a. Femoral Artery
b. Femoral Vein
c. Femoral branch of Genitofemoral Nerve
d. Lymphatic
Ans: C (Reference: RJ Last & Snell)
420. In femoral sheath which is the most lateral structure?
a. Artery
b. Vein
c. Nerve
d. Lymphatic
Ans: A (Reference: RJ Last & Snell)
Explanation:
1. Femoral nerve is lateral to the femoral sheath and itself not present in the sheath so its not the lateral most.
Instead in the sheath femoral artery is present most laterally.
2. Whereas femoral branch of Genitofemoral nerve pierces the sheath and present most laterally so if this
present choose it.
421. A young boy has lost about 1.5 litter blood in road side accident. He has decreased GFR and urinary
output. The part of renal tubules most involved in bringing back the GFR and urinary output to the normal
is?
a. Descending loop of Henle
b. Distal convoluted tubules
c. Proximal convoluted tubules
SK Origional – Golden 11 465

d. Thick ascending loop of Henle


e. Think ascending loop of Henle
Answer: B Reference: Guyton
Explanation:
1. Tubuloglomerular feedback in autoregulation of GFR
2. Myogenic autoregulation of renal blood flow & GFR
The Tubuloglomerular feedback mechanism has two components that act together to control GFR: (1) an
afferent arteriolar feedback mechanism and (2) an efferent arteriolar feedback mechanism. These feedback
mechanisms depend on special anatomical arrangements of the juxtaglomerular complex.
The juxtaglomerular complex consists of macula densa cells in the initial portion of the distal tubule and
juxtaglomerular cells in the walls of the afferent and efferent arterioles. The macula densa is a specialized
group of epithelial cells in the distal tubules that comes in close contact with the afferent and efferent
arterioles
422. Regarding Lumber Puncture Site
In infants prefer b/w L3-L4
In adults prefer b/w L4-L5
Structure pierced from outside
Skinsuperficial fasciaSupraspinous LigamentInterspinous LigamentLigamentum FlavumAreolar
Tissue (containing the internal vertebral venous plexus in the epidural space)Dura Materand Arachnoid
Mater
Post Lumbar Puncture Headache
This headache starts after the procedure and lasts 24 to 48 hours. The cause is a leak of cerebrospinal fluid
through the Dural puncture, and it usually follows the use of a wide-bore needle
How to reduce the incidence of Headache
Using small gauge styletted needles and avoiding multiple Dural holes
Reference: Snell
423. In renal transplantation single most significant test?
a. ABO Grouping
b. HLA
Answer: B Reference: Bailey, Robbins

424. Increase in arterial hydrogen ion concentration will stimulate respiratory center through?
a. Center chemoreceptor
b. Peripheral chemoreceptor
c. In aortic and carotid bodies
Answer: B Reference: Guyton & Ganong
425. Increase in CSF (interstitial fluid) hydrogen ion concentration will stimulate respiratory center through?
a. Center chemoreceptor
b. Peripheral chemoreceptor
c. In aortic and carotid bodies
Answer: A Reference: Guyton & Ganong
Explanation: H+ ions don't cross BBB or cross at very low rate hence its increase will not directly affect
central chemoreceptors but increase in arterial co2 which indirectly increase H+ concentration in the brain
interstitial fluid will stimulate respiratory center through central chemoreceptors.
Whereas arterial h+ can directly stimulate carotid body peripheral receptors to some extent
426. At the end of ventricular contraction, what happens?
a. Aortic Valve Opens
b. AV Valve Close
c. Pulmonary Valve Close
d. AV Valve open
e. Aortic Valve Closes
Answer: E Ref. Guyton
Note: If aortic valve closes not present then pulmonary valve closes 2nd best option
SK Origional – Golden 11 466

427. Factor responsible for patient having two hearts?


a. Allograft
b. Autograft
c. Xenograft
d. Isograft
e. Heterotopic Graft
Answer: E Reference: Bailey & Love
Explanation:
Its heterotopic heart transplantation
Where donor heart is placed adjacent to recipient heart to augment the recipient heart ... so two hearts
working simultaneously
428. Most common cause of cell injury in our country?
a. Alcohol
b. Hypoxia
c. Ischemia
d. Radiation
e. Malnutrition
Answer: E Reference: Robbins
429. Most common cause of cell injury worldwide?
a. Alcohol
b. Hypoxia
c. Ischemia
d. Radiation
e. Malnutrition
Answer: B Reference: Robbins
Explanation:
Nutritional imbalances continue to be major causes of cell injury. Protein-calorie deficiencies cause an
appalling number of deaths, chiefly among underprivileged populations
Whereas Hypoxia is a deficiency of oxygen, which causes cell injury by reducing aerobic oxidative respiration
Hypoxia is an extremely important and common cause of cell injury and cell death
430. During inspiration what decreases?
a. Heart rate
b. Venous return
c. Arterial pulse
d. Pulse pressure
e. Negative intra pleural pressure
Answer: C
431. Antacid decreasing gastric motility Aluminum Hydroxide
Note:
• Aluminum decreases motility causes constipation
• While Mg increases motility causes diarrhea
432. Regarding CVS:
• A woman with 10 days history of fever continuous chest pain radiating to back on auscultation
rustling sound is heard diagnosis is Pericarditis
• Patient has fever from one week on auscultation lungs fields clear there is a rustle murmur on the left
of sternum diagnosis Pericarditis
• Patient presented with pain which radiates to back not related with respiration O/E Muffled heart
sounds are present on the left sternal border diagnosis Pericarditis
• Myocarditis scenario organism involved Coxsackie virus
• 10 days history of fever chest pain not affected with respiration cause Myocardium
• Cushing reflex which one of the following is seen Decrease Heart rate
• Cushing Triad Raised ICP, HTN, Bradycardia
• Beck Triad Hypotension, Distended Neck Veins, Distant Heart Sounds
SK Origional – Golden 11 467

• MI 9th day suddenly die in washroom Cardiac Tamponade


433.
• CHF can be best diagnosed by BNP
• In CHF Captopril works as Increases Cardiac Output
• Mannitol given with caution inCHF
434. Regarding CSF:
• CSF Specific Gravity is 1.005
• CSF Density is 1.0005
• CSF PH is 7.33
• CSF is isotonic with Serum
• CSF has Equal Sodium as compared to Plasma
• CSF has High Magnesium and Chloride as compared to Plasma
• CSF has More Creatinine as compared to Plasma
• CSF is produced by Ependymal cells
• CSF Provides nutrition to CNS
• Arachnoid granulations are seen by Naked Eye
• Indicator for CSF Leak Beta 2 Transferrin
• CSF Has Cushion like effect
• Maximum determinant of CSF composition is Ependymal Cells
• CSF Pressure 10-20 cm H2O or 60-150mm H2O or 6-15mmhg
• CSF production 20ml/h
• CSF production 450 -550 ml /day
• CSF in Ventricular System is 150ml
• Lateral ventricle to 3rd via Intraventricular Foramina of Monro
• 3rd to 4th via Cerebral Aqueduct (blockage can cause hydrocephalus)
• 4th to Subarachnoid space via Foramen Magendie and Foramen Luschka
• CSF made by choroid plexus (Ependymal cells ) in 4th and Lateral Ventricles
• CSF absorbed by Arachnoid Granulations and then drains into Dural Venous Sinus
435. Regarding Lamina:
• Lamina 1-6 Dorsal(Posterior) Horn
• Lamina 8-9 Ventral(Anterior) Horn
• Lamina 7 &10 Intermediate Horn
• Lamina 1 High Threshold mechanoreceptor, Noxious stimulus & A delta pain
• Lamina 2 C fiber Pain & Substantia Gelatinosa
• Lamina 3-4 Low Threshold mechanoreceptor
• Lamina 6Deepest Layer, Joint skin Signal
• Lamina 7 Largest Area, Dorsal Nucleus of Clarke Lamina 10 Central Canal
436. Regarding Medualla Oblangata:
Upper Medulla:
• Posterior Portion Vestibulo-cocchlear Nucleus
• Spinal Trigeminal
• Anterior Portion Inferior Olivary Nucleus
Middle Medulla
• Nucleus Ambiguous
• Hypoglossal Nucleus
• Dorsal Motor Nucleus Vagus
• Medial Longitudinal Fasciculus
Lower Medulla
• Pyramidal Decussation
437. Regarding Tricyclic Antidepressants:
• TCA at Therapeutic dose leads to Hypotension
• At Toxic dose causes Tonic Clonic Seizures
SK Origional – Golden 11 468

• TCA with Tyramine containing food leads toHTN


438. Regarding Hippocampus:
• Long Term Memory Decrease protein synthesis and Structural changes
• Short Term Memory Temporal/Hippocampus/Limbic Area/Mid Brain and Synaptic changes
439. Regarding Embolism and Hemorrhages
• A thrombus in artery will lead to infarction
• Hemorrhage mostly occurs in Central retinal vein
• If person is having previously history of diabetes or MI then occlusion will be due to Embolism
440. Regarding Lobes Obstruction
• Most common site and in Supine position Apical segment of Right Lower Lobe
• While standing or sitting position Posterior segment of Right Lower Lobe
• While lying on right side Posterior segment of Right Upper Lobe
(Ref: First Aid 2019 Edition Page Number 649)
441. Regarding OCPs
• Prolonged use of OCP`s will increase risk of Thrombo-embolism
• Prolong use of all OCP`s increase risk of Thrombo-embolism
• Estrogen containing OCP`s leads to Endometrial CA
• Estrogen containing OCP`s without Progesterone also leads to Endometrial CA
(Ref: Applied Therapeutics)
442. Regarding thyroid drugs:
• In Thyrotoxicosis: Mathimazole/Carbimazole
• In Thyroid Storm: Propylthiouracil
• In First Trimester: PTU
• In 2nd and 3rd Trimester: Mathimazole
• Hyperthyroidism with Arrthymias: Beta Blocker
443. Regarding Pre Malignant Lesions and Conditions:
• Most Common premalignant lesion is Leukoplakia
• Most Lethal premalignant lesion is Erythroplakia
• Most Common premalignant condition is Submucosal Fibrosis
• Most Lethal premalignant condition is Lichen Planus
444. Regarding Esophgus Blood Supply:
• Upper Esophagus is supplied by Inferior Thyroid Artery
• Middle Esophagus is supplied by Descending Thoracic Aorta
• Lower Esophagus is supplied by Left Gastric Artery
445. Regarding Neurology:
• CNS Oligodendrocytes
• PNS Schwann cells
• Repairing Cells Astrocytes
• Grey Mater  Protoplasmic astrocytes (overall abundant)
• White Mater Fibrous astrocytes (abundant astrocytes )
• Adult Spinal Cord Lower border of L1 or Upper Border of L2
• Neonates Spinal Cord Upper Border of L
• Endoneurium Individual Nerve Fiber
• Perineurium  Bundle of nerve fibers
• Epineurium Entire Nerve
• Fasciculus Gracilis Lower Limb Sensations
• Fasciculus Cuneatus Upper Limb Sensations
• Loss of light Reflex but intact accommodation reflex Pretectal Nucleus
• Loss of Accommodation reflex Cerebral cortex
• Loss of Accommodation Cerebral cortex
• Loss of Accommodation + 3rd CN Involvement-- Midbrain
• Corneal Reflex lost Pons
SK Origional – Golden 11 469

• Heating + Sympathetic effect Posterior Hypothalamus


• Cooling + Parasympathetic Anterior Hypothalamus
• Overall Temperature Regulation Anterior Hypothalamus (Pre-optic Nucleus)
• FOLIA Cerebellum Fold
• HYPERACUSIS Geniculate Ganglion (Medial Wall of Middle Ear)
• Brocas Aphasia Motor aphasia + Non-Fluent + Area 44 and 45 + Inferior Frontal Gyrus
• Wernicke‘s Aphasia Sensory Aphasia + Fluent + Area 22+ Superior Temporal Gyrus
• Global Aphasia  Both Wernicke and Brocas Aphasia + Arcuate Fasciculus
• Anomic Aphasia Mild Fluent Aphasia + Failure of Word retrieval + Angular Gyrus
• Lesion in DCML (posterior white column) Sensory Ataxia>Asterognosia
• Loss of Proprioception lesion in Posterior Column (DCML)
• Loss of Proprioception mechanism is Lateral Inhibition (Reference: Guyton)
• Reduced motivation and depression FRONTAL LOBE
• Characteristic of cerebellum lesion Dysdiadochokinesia>Dysarthria
• Resting Tremor Substantia Nigra
• Intentional tremors Cerebellum
• Internal Jugular Vein after Coming out of Jugular foramen relation ICA
• Internal Jugular Vein within jugular foramen relation Accessory Nerve
• Striatum Caudate + Putamen
• Corpus Striatum Caudate + Lentiform
• Lentiform Putamen + Globus Pallidus
• Olfactory Cortex location Posterior inferior Temporal Lobe + Uncas
• Olfactory Area location Anterior Perforating Substance
• Fastest fibers A Alpha
• Fast pain fibers A Delta
• Slow pain fibers C Fibers
• Preganglionic fibers Beta Fibers
• Postganglionic fibers C Fibers
• Pain from finger-tip by A Delta
• Proprioception from finger-tip B Fiber
• Type A>B>C affected by Pressure
• Type B>A>C affected by Hypoxia
• Type C >B>A affected by Anesthesia
• Itching Slow C fibers
• Itching Track Anterior spino-thalamic Track
• Sleep Centre Pre-optic nucleus
• Circadian Rhythm  Suprachiasmatic Nucleus
• Chorea ( jerky+ quick movements) Caudate nucleus
• Athetosis ( slow + writhing ) Globus Pallidus
• Hemiballismus Sub-thalamic Nucleus Lesion
• Fusiform Gyrus (temporal lobe ) Face Recognition (unable to recognize face called
Propognosia)
• Cingulate Gyrus Emotions
• Sexual Centre Nucleus Acumbens
• Large Receptive Field Pain and Temperature
• Structure close to crus cerebri Substantia Nigra
• Medial lemniscuses formed by Decussation of Internal Arcuate Fibers
• In UMN lesion fibers decussate at the level of Pyramids
• Cerebellum connected to Midbrain by Superior Cerebellar Peduncle
• Bleeding from mastoid Antrum Sigmoid sinus
SK Origional – Golden 11 470

• Climbing Purkinje fibers originate from Inferior Olivary nucleus


• Subdural hematoma Superior Cerebral Veins (Crescent Shaped Hematoma)
• Extradural Hematoma Middle Meningeal Artery branch of maxillary artery + Lucid Interval
unconsciousness + Biconvex Shaped Hematoma
• Subarachnoid hemorrhage Rupture of Saccular Aneurysm + worst headache of life +
increase risk for Hydrocephalus
• Spinal Nerves Mixed Nerves (both sensory and motor fibers) + Formed in Intervertebral
Foramina + Exit Intervertebral Foramina
• Spinal Ganglia Pseudo-unipolar Neurons
• Skeletal Muscle Multipolar
• Olfactory Bipolar
• Dorsal column Fine Touch + Pressure + Vibrations + Proprioception
• Meissonier Corpuscles Light Touch and Low Frequency VibrationUpto 40 Hz & At Finger-tips
• Paccinian Corpuscle High Frequency Vibrations 40-400Hz and Deep Touch
• Rapidly Adapting  Paccinian > Meissonier
• Ruffini Nerve Endings Deep Static Pressure
• Merckles Nerve Endings Position + Deep Static Touch + Secrete Serotonin
• Fine movements of hand Controlled by Cerebellum and Carried by Corticospinal Tracts
• Centre for direct autonomic reflexes Hypothalamus
• Middle Cerebral Artery (upper limbs + Aphasia ) Supplies – Insula and Opercula
• Anterior Cerebral Artery Lower Limbs + Aphasia Supplies Secondary Somasthetic Area
• Tabes Dorsalis Atonic Bladder
• Proximal muscle Flexion Rubro-spinal Tract
• Proximal muscle Extension Vestibulo-spinal Tract
• Inhibitory output in CNS Purkinje cells
• Ptosis + Miosis  Horner Syndrome
• Ptosis + Mydriasis  3rd Nerve palsy
• Ptosis + Normal Pupil Myasthenia Gravis
• Site of fusion of binocular vision  Visual Cortex
• Day Vision + Color Vision Cones
• Night Vision + Increased sensitivity to low light Rods
• Anterior layer of retina contains Retinal Pigment Epithelium
446. Regarding Ureter:
• During Hysterectomy Ureter Damage at Cardinal Ligament (uterine vessel)>Behind Broad
Ligament>At Pelvic Brim (ovarian artery)
• Ureter Damage at Pelvic Brim While crossing Common iliac Vessel
• While removing Ovary damage to Internal iliac Artery
• Anterior to Ureter Gonadal Vessel
• Posterior to Ureter iliac Vessel
• Common Site of Lodging of ureteric stones overall/Adults Vesico Ureteric Junction
• Common Site of Lodging of ureteric stones in ChildPelvico Ureteric Junction
• Ureter Narrows at Vesico Ureteric Junction>Where it Enters Bladder
447. Regarding Uterus Prolapse:
• 1st Degree Decent of Cervix within Vagina
• 2nd Degree Decent of Cervix to Introitus
• 3rd Degree Decent of Cervix Outside Introitus
• 4th Degree (procidentia) Whole Uterus outside Introitus
• In 1st and 2nd Degree Uterosacral Ligament Damage
• 3rd Degree Uterosacral > Cardinal Ligament Damage
• 4th Degree Cardinal Ligament Damage
SK Origional – Golden 11 471

448. Regarding Transplant:


• Important Test For Transplant HLA typing
• Best Blood Cell for HLA Sampling WBC
• Best Site for HLA Sampling Buccal Mucosa
• Best Tissue for HLA Sampling Bone marrow
• Hyper acute Rejection Pre Formed Antibodies (Type 2 Hypersensitivity)
• Acute Rejection CD8 Cells (Type 4 Hypersensitivity)
• Chronic Rejection CD4 Cells (Type 2+4 Hypersensitivity)
449. Regarding Antibodies and Rubella:
• Neonatal Recurrent Infection Deficiency of IgG
• Child Recurrent Infection Deficiency IgA
• Neonatal Infection Diagnosed by IgM
• Mother Affected with Rubella and Baby got Symptoms Diagnosed by IgM
• Mother Affected with Rubella and Baby Symptoms less to See immunoglobulin IgG2
• Rubella in pregnancy complication overall Deafness>Cataract
• Rubella in pregnancy complication within 7 weeks Cataract
• Rubella in pregnancy complication After 7 weeks Deafness
• After Transplant most Common malignancy  Skin
• After Transplant Common malignancy in 1 or 5 Year Lymphoproliferative
• After Transplant Common malignancy after 10-15 Years Skin
• Most Common Manifestation of autoimmune disease Hematological
• SLE involve most commonly Joints (90%) And Skin (85%)
• SLE Involve most common organ then Skin
450. Regarding General Pathology:
• Potent Antioxidants Glutathione> Vitamin E>Vitamin C>Vitamin A
• Radicals OH>H2O2>Superoxide
• Uninucleate Most Cells
• Binucleate Liver Cells
• Multinucleate Skeletal Muscle
• Cannot Regenerate Lens>Skeletal Muscle >Neuron>Cardiac
• Cannot Reproduce RBC
451. Regarding Neuroanatomy:
• Vertebral Bodies Limited by Anterior and Posterior Longitudinal Ligament
• Vertebral Bodies kept in position by Anterior and Posterior Longitudinal Ligament
• Vertebrae joined to Adjacent Lamina Ligament Flavum
• Hyperextension of neck Ligament Injured Anterior Longitudinal Ligament
• Hyper flexion of neck Ligament Injured Ligament Nuche
452. Regarding Cell Injury:
Irreversible Cell Injury Causes
• Disruption of cell membrane
• Influx of Ca
• Leakage of Intracellular Enzymes
• Nuclear Degeneration
• Bursting of Lysosome
• Pyknosis (denaturation and condensation of nucleus)
• Karyorrhexis (nuclear fragmentation)
• Karyolysis (Dissolution of Nucleus)
• Severe Lactic Acidosis
Reversible Causes:
• Cell Swelling
• Bleb Formation
SK Origional – Golden 11 472

• Swelling of Endoplasmic Reticulum


• Increase Glycolysis
• Fatty Liver Disease
• Mild Lactic Acidosis
453. Regarding Bronchogenic CA:
• Smokers Squamous cell CA
• Non-smokers Adeno CA
• In females Adeno CA
• In smoker females Adeno CA
• Overall Adeno CA
• Scar tumor associated with Adeno CA
• Uranium associated with SCLC
• Asbestos associated with Adeno CA
454. Regarding Pneumonia:
• Rusty Sputum Streptococcal Pneumonia
• Purulent sputum Staph Aureus Pneumonia
• Green Sputum Pseudomonal Pneumonia
• Red Currant Jelly Sputum Klebsiella
• Leading cause of community acquired pneumonia Streptococcus pneumoniae
• Pneumonia in HIV patient Pneumocystis Jirovecii
• Leading cause of Hospital Acquired Pneumonia Staph Aureus
• Leading cause of Pneumonia in Alcoholic & Diabetics Klebsiella
• Leading cause of Pneumonia in Neonates Group B Streptococcus
• Walking Pneumonia is Mycoplasma Pneumonia
• Most common cause of atypical pneumonia is Mycoplasma Pneumonia
• Pneumonia with coombs test +ve is Mycoplasma pneumonia
• Pneumonia with Hyponatremia is Legionella pneumonia
• Pneumonia with neurological involvement is Legionella pneumonia
• Pneumonia is not responding to Penicillin is Mycoplasma pneumonia
• Travelers pneumonia is Legionella pneumonia
• Pneumonia with muco-purulent sputum is Bacillus Anthrax
455. Regarding CVS Pathology:
• Soft S1 in Severe Mitral valve stenosis
• Variable in Atrial fibrillation and Third degree HB
• Loud S1 in Mild mitral valve stenosis
• Splitting is common of S2 Heart Sound
• Usually it's normal
• Fixed in ASD/VSD
• Fixed means A2 component will come first than P2 in both Inspiration and expiration
• Reversed in Aortic Stenosis
S2 have 2 components:
A2 and P2
A2: Soft in Aortic stenosis and Aortic Regurgitation, Loud in Systemic HTN
P2: Soft in Pulmonary Stenosis and Pulmonary Regurgitation, Loud in Pulmonary HTN
456. Regarding Heart Block:
First Degree Heart Block:
When every impulse from atria reaches ventricles but with some delay, prolonged PR interval
Treatment: if asymptomatic: No Rx
If symptomatic: inj. Atropine or Temporary Pacemaker
Second Degree Heart Block:
When some impulse from atria fails to reach ventricles
SK Origional – Golden 11 473

it has two types: mobitz Type 1 and mobitz Type 2


MOBITZ Type 1:
Progressive lengthening of PR interval and then one P wave is not followed by QRS complex
Rx: same like 1st degree HB
MOBITZ Type 2:
Constant PR interval but prolonged and then one P wave is not followed by QRS complex
Rx: PPM (symptomatic or asymptomatic)
Third Degree Heart Block:
No impulse from atria reaches ventricles so atria and ventricles will contract independently
Rx: PPM (symptomatic or asymptomatic)
457. Regarding Progesterone and Estrogen:
• In breast - Alveolar & Lobular development
• In uterus - endometrial proliferation (have role in secretory phase)
• In pregnancy maintain thickness of endometrium
• Breast fat/stroma deposition - Estrogen (also develops ducts)
458. Structures Present in Substance of Parotid Gland:
1. Facial Nerve and its Terminal Branches
2. External Carotid Artery and its two terminal branches
a. Superficial Temporal Artery
b. Maxillary Artery
3. Retromandibular vein
459. Which nerve brings preganglionic fibers to Ciliary ganglion?
a. Short Ciliary
b. Long Ciliary
c. Nasociliary
d. Occulomotor
Ans: D
460. Patient with decreased neutrophil, increased lymphocytes and increased eosinophil,
increased ACTH?
a. Cushing syndromes
b. Cushing disease
c. Ectopic ACTH
d. Leukemia
e. Addison‘s disease
Ans: E
461. Pulmonary congestion due to mitral stenosis. Autopsy will show in alveoli?
a. Fat necrosis
b. Fibrinoid necrosis
c. Coagulative necrosis
d. Liquefaction necrosis
Ans: B
462. Regarding Antihypertensive:
• Best hypertensive for diabetic  ACEIs or ARBs 2nd line CCBs
• Best hypertensive for asthmatic  CCBs Verapamil
• Best hypertensive for pregnant female during pregnancy Labetalol if not in options than prefer
Methyldopa
• Best hypertensive for pregnant female having hypertensive crisis Labetalol or Hydralazine
• Best painkiller for asthmatic postoperatively Ketorolac
• HTN in Ischemic patients  Thiazides or ACEIs
• HTN Post MI Beta blockers or ACEIs
• Isolated systolic HTN in elderlies CCBs
463. Most sensitive investigation for Liver abscess?
SK Origional – Golden 11 474

a. CT
b. Serology
Ans: A
464. Diagnostic or sensitive test for amoebic Liver abscess is?
a. CT SCAN
b. Serology
Ans: B
465. Most widely used technique for detection of tumors?
a. PBS
b. Tumor Marker
Ans: B
466. Most widely use technique for detection of circulating tumor cells?
a. Tumor marker
b. PBS
Ans: B
467. P53 most beneficial role?
a. Prevent overcrowding
b. Remove mutated cells
Ans: A
468. Permeation is:
a. Tumor invasion to lymphatic
b. Tumor invasion to blood vessels
Ans: A
469. Regarding Cartilage:
Hyaline Cartilage
1. Articular Cartilage
2. Costal Cartilage
3. Cartilage of Nose, Trachea, Larynx
4. Epiphyseal Cartilage (ossified after age of 21 years)
Elastic Cartilage: 3 E
1. Ext Ear
2. Ext auditory canal
3. Epiglottis
Fibro-cartilage
1. Inter vertebral disc
2. Intra articular cartilage
3. Labranal of Glenoid cavity
4. Disc of Pubis Symphysis
470. Sacral S2 S4 spinal cord level?
a. T11 T12
b. L1 L2
c. L3 L4
d. L5 L6
Ans: B
471. More specific for SLE?
a. ANA
b. Anti-ds DNA
c. Anti-smith Ab
Ans: C
472. More specific for Hashimoto thyroiditis?
a. Anti-microsomal
b. Anti-thyroglobulin
SK Origional – Golden 11 475

c. Anti-thyroid peroxidase
Ans: B
473. More specific for Wegener disease?
a. C-ANCA
b. P-ANCA
Ans: A
474. More specific for Sjogren Syndrome?
a. Anti-Ro
b. Anti-La
Ans: B
475. More specific for pernicious anemia?
a. Anti-parietal cell antibody
b. Anti-intrinsic factor antibody
Ans: B
476. The most specific sign of Pericarditis on ECG?
a. Widespread ST Elevation
b. PR Depression
Ans: B (Reference: Pass Medicine Q Bank for MRCP-1)
477. Drug of choice for SVT, Narrow complex Tachycardia, without aberrancy, rhythm control
is?
Ans: Adenosine 6mg IV increased to 12mg IV if not improved
478. Drug of choice for rhythm control of Ventricular tachycardia, broad complex tachycardia is?
Ans: Amiodarone 50mg IV bolus over 10mins, unless there is pre-existing heat failure (reduced LVEF) in
which case digoxin will be preferred?
Reference: Oxford Medicine
479. Which of following does not carry input of taste?
a. Hippocampus
b. Amygdala
c. Ventro medial nucleus
d. Parareticular pontine
e. Ventral thalamic nucleus
Ans: C
480. The collective name for peripheral nerves that exit the brain and brainstem?
a. Spinal
b. Cranial
Ans: B
481. Damage to medullary cutaneous & gracillus causes?
a. Asterognosia
b. Anesthesia
Ans: A
482. Which nerve does not travel through orbital foramen?
a. 2nd CN
b. 3rd CN
c. 4th CN
d. 5th CN
Ans: A
483. Cross eye is caused by lesion in cranial nerve:
a. 3rd CN
b. 4th CN
c. 6th CN
Ans: C
484. Nerve cell bodies for the primary fibers of the facial nerve 7 are located in the?
SK Origional – Golden 11 476

a. Ciliary ganglion
b. Geniculate ganglion
c. Sub mandibular ganglion
d. Pterygopalatine ganglia
Ans: B
485. A patient comes to you with right sided sixth CN palsy and left sided weakness which part of
brain involved?
a. PONS
b. Internal Capsule
Ans: A
486. All are part of PNS except?
a. Radial Nerve
b. V CN
c. 1st CN
d. VII CN
e. III CN
Ans: A
487. Pterygopalatine ganglion anatomically related to:
a. Maxillary Nerve
b. Facial Nerve
Ans: A
488. GVE of Vagus nerve nucleus is?
a. Dorsal Nucleus
b. Ambiguous
Ans: A
489. Difficulty in hair combing?
a. Long Thoracic Nerve
b. Accessory Nerve
Ans: A
490. True regarding mandibular foramina?
a. Provide attachment to Ligament
b. Provide axis for mouth Opening
c. Lingua is posterior to it
d. 12 nerve passes through it
Ans: A
491. Number of Cervical vertebra present is:
a. 7
b. 8
c. 9
Ans: A
492. Optic canal found in which part of sphenoid bone?
a. Body
b. Lesser wing
c. Greater wing
d. Pterygoid process
Ans: B
493. Which of following is true?
a. Presence of teeth is necessary for proper jaw growth
b. Development of teeth determines alveolar growth
Ans: B
494. Mandible is not supplied by branches of?
a. Maxillary artery
SK Origional – Golden 11 477

b. Ascending Pharyngeal Artery


Ans: B
495. Mylohyoid ridge gives attachment to?
a. Sphenomandibular ligament
b. Stylo mandibular ligament
c. Pterygomandibular raphe
Ans: C
496. Foramen Laceram transmits?
a. Internal Carotid Artery
b. Greater Petrosal Nerve
Answer: B
497. Medial geniculate bodies lie on?
a. Midbrain
b. Thalamus
Ans: A
498. Example of Apoptosis:
Physiological
• Menstruation
• Embryogenesis
• Organogenesis
• Selective death of T lymphocytes
Pathological
• Hepatitis
499. Examples of Type IV Hypersenstivity:
• TB
• Contact Dermatitis
• Multiple Sclerosis
• Graft Vs. Host Disease
• Guillain Barre Syndrome
• Mantoux Test
• Scabies
• Poison IVY/ OAK
500. Anatomy
• Most Common Bone injured in Body Clavicle
• Most Common Long Bone Fracture in Body Clavicle
• Most Common Long Bone Fracture in Lower Limb Tibia
• Most Common Carpal Bone Fracture Scaphoid
• Most Common Dislocated Carpal Bone Lunate
• Fracture of Hook of Hamate Ulnar Nerve Damage
• Anatomical Snuff box Pain Radial Artery
• Fracture of Surgical Neck of Humerus Axillary Nerve Damage
• Fracture of medial epicondyle Ulnar Nerve Damage
• Fracture of Shaft of Humerus Radial Nerve Damage
• Wrist Drop Radial Nerve Damage
• Anterior Dislocation of Shoulder joint Axillary Nerve Damage & Post Circumflex Artery Damage
• Quadrangular Space injury Axillary Nerve Damage & Post Circumflex Artery Damage
• Small Muscle of Hand affected lesion in C8-T1>Ulnar + Median>T1>Ulnar
• Ulnar nerve damage at Elbow Loss of Sensations in medial 1/3rd palmar and dorsal aspect +
Hypothenar wasting
• Ulnar nerve damage at Wrist Claw Hand
• Median nerve damage at Elbow Hand of Benedict
SK Origional – Golden 11 478

• Median nerve damage at Wrist Carpal tunnel Syndrome


• In median nerve damage Opposition and abduction is not possible
• Abduction of Arm upto 15 degree Supraspinatus
• Abduction of Arm upto 90 degree Deltoid
• Abduction of Arm Above 90 degree Trapezius
• Serratous Anterior Arm Above 100 degree Long Thoracic Nerve
• Scratching of Back Lattismus Dorsi
• Chief Supinator of Hand Bicep Brachii
• Total Claw Hand Lumbricals Paralyzed
• Tendon Courses through Shoulder Joint Long Head Bicep
• Head of Humerus Supplied by Arcuate >Anterior Circumflex Artery
• Neck of Humerus Supplied by Posterior Circumflex artery
• Most commonly dislocated bone in the lower limb is Patella
• Most common neuropathy in the lower limb is Compression of Common Peroneal Nerve
Against Neck of Fibula
• Longest muscle in the body is Sartorius
• Thickest nerve in the body is Sciatic nerve
• Largest bone of the body is Femur
• Largest & most complicated Joint in the body is Knee joint
• Largest Sesamoid bone in body is Patella
• Strongest ligament in the body is iliofemoral ligament
• Strongest tendon in the body is Tendo-calcaneus
• Largest synovial cavity in the body is synovial cavity of the Knee joint
• Most commonly nerve used in the body for grafting is Sural Nerve
• Most commonly used vein in body for grafting is Great Saphenous Vein
• Most commonly used muscle in the body for grafting is Plantaris & Palmaris Longus
• Locking muscle for knee is Quadriceps Femoris
• Unlocking muscle for knee is Popliteus
• ACL injured Anterior Dislocation of Tibia
• PCL injured Posterior Dislocation of Tibia
• ACL injured Posterior Dislocation of Femur
• PCL injured Anterior Dislocation of Femur
• Ankle Sprain(Inversion) Ligament Injured Lateral Ligament>Anterior Talofibular
• Excessive Eversion ligament Injured Tibial Collateral
• Saphenous nerve accompany Great Saphenous Vein
• Sural nerve accompany Small Saphenous Vein
• Deep peroneal nerve accompany Anterior Tibial Artery
• Head of Femur Supplied by in Adults Retinacular >Medial Circumflex artery
• Head of Femur Supplied by in Child Obturator Artery
• Neck of Femur Supplied by Medial &Lateral Circumflex artery
• Foot Drop Common peroneal nerve Injured
• Left or right dominant supply of heart is determined by Posterior Intraventricular Artery
• Great Cardiac vein accompany Left Anterior Descending Artery
• Middle Cardiac vein accompany Posterior Intraventricular Artery
• Small Cardiac vein accompany Marginal Artery
• Ant. Cardiac vein drains directly into Right Atrium
• Left Lobe 5cm Right Lobe 2.5cm
• Pain of angina from heart is carried by Sympathetic nerves
• Pain of pericarditis is carried by Phrenic nerve
• Fibrous Pericardium and Parietal layer of Serous Pericardium is supplied by Phrenic nerve
SK Origional – Golden 11 479

• Mediastinal Pleura Supplied by Phrenic Nerve


• Visceral layer of serous pericardium is called Epicardium
• Right border of Heart formed by Right Atrium
• Right border of Heart formed on X-ray by SVC
• Base of heart is formed by Left atrium
• Left border of Heart formed by Left Ventricle(Apex Beat)
• Anterior (sternocostal) Surface formed by Right Ventricle
• Inferior (diaphragmatic) Surface formed by Left Ventricle
• Epicardium supplied by Coronary Artery
• Pericardium supplied by Pericardiophrenic artery
• Anterior 2/3rd IV Septum supplied by LAD
• Posterior 1/3rd IV Septum supplied by PDA
• Left Circumflex supply Left and Right Ventricle
• Xiphisternum Vertebrae level T9
• IVC begin at L5
• True Ribs 1-7
• False Ribs 8-10
• Floating Ribs 11-12
• Upper Esophagus  Inferior Thyroid artery
• Middle Esophagus Descending thoracic aorta
• Lower Esophagus Left Gastric
• Azygous vein anterior to Right Root of Lung
• Aorta posterior to Left Root Lung
• Phrenic nerve Anterior to Root of lung
• Vagus Nerve Posterior to Root of Lung
• IVC Blocked Above Azygous vein dilation in Azygous vein>Left Gastric Vein
• IVC Blocked Upto Azygous vein dilation in Right Ascending Lumber Vein & Right Subcostal
Vein
• IVC Blocked Below Azygous vein dilation in Ascending Lumber vein
• Muscle of Quite Inspiration Diaphragm and External Intercostal
• Muscle of Forced Inspiration SCM & Serratous Anterior
• Quite Expiration Passive and by Elastic Recoil of Lung
• Muscle of Forced Expiration Internal Intercostal
• Accessory Muscle of Expiration External Oblique
• T8 Caval Opening Inferior Vena Cava and Right Phrenic Nerve , Extent of IVC = T8 to L5
• T10 Umbilicus, Esophageal Hiatus (Esophagus , Esophageal vessels , and Vagus nerves)
• T12 Aortic Opening (Aorta , Azygous , and Thoracic Duct)
• T3 to T6 Oblique Fissure of Lung
• T4-5 Bifurcation of Trachea ,,Dermatome Nipple ,, Extent of Trachea = C6 to T5
• C3Hyoid in erect position
• C2-C3 Tracheostomy Level Adults
• C3-C4 Tracheostomy Level Children
• C6 Cricoid Level ( Esophagus and Trachea Starts )
• C2-3 Supraclavicular
• C3-4 Infraclavicular
• Foramen Ovale Accessory Meningeal artery Passes
• Foramen Spinosum Middle Meningeal artery Passes
• Superior Orbital Fissure V1 (Ophthalmic nerve) Passes
• Foramen Rotandum V2(Maxillary Nerve) Passes
• Foramen Ovale V3(Mandibular Nerve) Passes
SK Origional – Golden 11 480

• Jugular Foramen CN 9,10,11(Accessory part) & Sigmoid Sinus


• Hypoglossal Canal CN 12
• Foramen Magnum Brainstem & Spinal Part of CN11
• Esophagus Passed through Left Crus of Diaphragm
• Medial Arcuate ligament formed by Psoas Muscle
• Median Arcuate ligament formed by Right & Left Cruras
• T12 Celiac Trunk
• L2 Renal Artery
• L1 Superior mesenteric artery
• L3 Inferior Mesenteric Artery
• L4 Bifurcation of Descending Aorta
• L5 Start of IVC
• Trans pyloric Plane Pylorus of Stomach , Fundus of Gallbladder, Hilum of Kidney, First part
of Duodenum , Origin of SMA , Tip of 9th Costal Cartilage , Lower end of Spinal Cord
• Right Kidney anteriorly Related to Liver
• Right Kidney Hilum Anteriorly Related to 2nd Part Duodenum
• Posterior to Right Kidney 12th Rib & Diaphragm
• Anterior to Left Kidney Stomach & Pancreas
• Posterior to Left Kidney 10th -11th Ribs & Diaphragm
• 2nd Part of duodenum Attached posteriorly to Transverse Mesocolon
• Right Accessory Hepatic Artery Branch of SMA
• Superior Epigastric Artery Branch of Internal Thoracic Artery
• Inferior Epigastric Artery Branch of External iliac Artery
• Superficial Epigastric Artery Branch of Femoral Artery
• Superior Rectal Branch of Inferior Mesenteric artery
• Middle Rectal Branch of Internal iliac artery
• Inferior Rectal Artery Branch of Internal Pudendal Artery
• Bulbourethral Gland Deep Pouch
• Greater Vestibular Gland Superficial pouch
• Difficulty in Standing from Sitting Gluteus Maximus Damage
• Shuffling Gate Gluteus Medius + Minimus
• Right Pelvis Sink Left Gluteus Medius + Minimus Damage
• Injection Given in Superior Lateral Compartment to avoid Damage to Sciatic Nerve
• Mediolateral Episiotomy Damage to Bulbospongiosus > Superficial Transverse Perinial Muscle
• Mediolateral Episiotomy Structure at Risk Levator Ani
• Median Episiotomy Damage to External Anal Sphincter
• During Episiotomy Perinial Body Damage then Muscle Injured Levator Ani
• Major Support of Uterus Cardinal (Transverse Cervical) Ligament
• Dynamic Support Pelvic Diaphragm.

===============

At the End All effort are dedicated to my those students who were unsuccessfull multiple time in FCPS Part
– 1 exam but they did not loose hope.

Regards
Dr. Salahuddin Kamal

You might also like